You are on page 1of 523

ASSESSMENT

& PRACTICE
PACKAGE
THE COMPLETE
ASSESSMENT
& PRACTICE
PACKAGE

Test Drive for


jee ADVANCED
2020
5 Years’ Subjectwise Questions
of JEE Advanced (PCM)

15 PRACTICE SETS
THE COMPLETE
ASSESSMENT
& PRACTICE
PACKAGE

Test Drive for


jee ADVANCED
2020
5 Years’ Subjectwise Questions
of JEE Advanced (PCM)

15 PRACTICE SETS

ARIHANT PRAKASHAN (Series), MEERUT


Test Drive for
jee ADVANCED 2020

Arihant Prakashan (Series), Meerut


All Rights Reserved

© PUBLISHERS
No part of this publication may be re-produced, stored in a retrieval system or distributed
in any form or by any means, electronic, mechanical, photocopying, recording, scanning,
web or otherwise without the written permission of the publisher. Arihant has obtained all
the information in this book from the sources believed to be reliable and true. However,
Arihant or its editors or authors or illustrators don’t take any responsibility for the absolute
accuracy of any information published and the damages or loss suffered there upon.
All disputes subject to Meerut (UP) jurisdiction only.

ADMINISTRATIVE & PRODUCTION OFFICES


Regd. Office
‘Ramchhaya’ 4577/15, Agarwal Road, Darya Ganj, New Delhi -110002
Tele: 011- 47630600, 43518550; Fax: 011- 23280316
Head Office
Kalindi, TP Nagar, Meerut (UP) - 250002
Tele: 0121-2401479, 2512970, 4004199; Fax: 0121-2401648

SALES & SUPPORT OFFICES


Agra, Ahmedabad, Bengaluru, Bareilly, Chennai, Delhi, Guwahati,
Hyderabad, Jaipur, Jhansi, Kolkata, Lucknow, Meerut, Nagpur & Pune

ISBN : 978-93-13199-35-9

Published by Arihant Publications (India) Ltd.


For further information about the books published by Arihant
log on to www.arihantbooks.com or email to info@arihantbooks.com

/arihantpub /@arihantpub Arihant Publications /arihantpub


Test Drive for
jee ADVANCED 2020

PREFACE
JEE Advanced is a gateway examination to India's most reputed Technical Institutes,
Indian Institutes of Technology, IITs. Approximately, the top 2.25 lacs students passed in
JEE Main will be able to attempt JEE Advanced. JEE Advanced is one of the most
difficult examination in the world, so it is quite clear that it requires a lot of practice &
skills to get through it. The practice & complete knowledge of exam level & pattern is a
must to get success in JEE Advanced. Keeping the above point of view, we have come
up with 15 Practice Sets for JEE Advanced 2020, which will perfectly serve the given
cited purpose in a perfect manner.

This book will help you to getting higher score in the JEE Advanced exam which
consists of 15 Practice Sets, Last 5 Years' JEE Advanced Questions in chapterwise
manner and JEE Advanced Solved Paper 2019. Each practice set containing questions
with their authentic solution and also having the conceptual approach.

SALIENT FEATURES
There are some special features associated with this book, which make it stand apart from
others. Some of them are
! Last 5 years' chapterwise question of JEE Advanced for all the three Sections:
Physics, Chemistry and Mathematics.
! 15 Practice Sets strictly based on the latest examination pattern.
! Idea connected to a particular solution enable you to know about the concepts
behind the respective question.
! Test Edge given to a solution will help you to get the additional info come from that
particular question.
! At the end, there is JEE Advanced Solved Paper 2019.

We are highly thankful to ARIHANT PRAKASHAN, MEERUT for giving us such an


excellent opportunity to write this book. The role of Arihant DTP Unit and Proofreading
team is praiseworthy in the making of this book.
Huge efforts have been made from our side to keep this book error free, but inspite of
that if any error or whatsoever is skipped in the book then that is purely incidental,
apology for the same, please write to us about that so that it can be corrected in the
further edition of the book. Suggestions for further improvement of the book will also
be welcomed.
In the end, we would like to wish BEST OF LUCK to our readers!
Authors
Test Drive for
jee ADVANCED 2020

ABOUT THE BOOK


The whole New Test Drive for JEE Advanced aims to provide systematic practice to JEE aspirants. For
achieving its aim book is divided into 2 Sections, Section I (The Time Line) and Section II (Prep Catalysis) and
Given here are the some glimpses of the book to give you the idea of its organisation.

2014-18

THE TIME LINE


The chapterwise question of JEE Advanced given here
will help you to get acquainted with the pattern and
difficulty level of JEE Advanced. Solutions to all
questions are given for your reference.

PREP CATALYSIS
The last two months practice is the catalysis to
preparation of the entire term. To give you this
catalysis i.e. the practice this section contains
15 Practice Sets exactly similar to JEE Advanced
Pattern. All these Practice Sets can be attempted
Online to make you ready for first online format of
JEE Advanced.
Test Drive for
jee ADVANCED 2020

OTHER SPECIAL FEATURES


ANSWER KEY
Answer Key has been provided with each
practice sets, to help you to score yourself.
you can cross check the answers selected
by you with the answer given here and
mark your answer as correct or incorrect.

SCORE SHEET
The Score Sheet given with each
practice sets in this book will help you to
know the real level of your performance.

ANALYTICAL EXPLANATIONS
With the detailed explanations given for all the
practice sets (1-10) you can check the point of
mistake or the complete solutions in case you do
not get the correct answer.
But we advise you to see explanations only after
you put your best efforts in solving the
questions.
Test Drive for
jee ADVANCED 2020

CONTENTS
THE TIME LINE (Last 5 Years' JEE Advanced Questions )
Physics 3-33
Chemistry 34-55
Mathematics 56-74

PREP CATALYSIS (Practice Sets)


Practice Set 1 77-95
Practice Set 2 96-111
Practice Set 3 112-126
Practice Set 4 127-144
Practice Set 5 145-162
Practice Set 6 163-178
Practice Set 7 179-195
Practice Set 8 196-213
Practice Set 9 214-229
Practice Set 10 230-247
Practice Set 11 248-263
Practice Set 12 264-279
Practice Set 13 280-296
Practice Set 14 297-312
Practice Set 15 313-327

Analytical Explanations 331-472

JEE Advanced Solved Paper 2019 1-47


The Time Line~JEE Advanced Questions (2014 –2018) 1

JEE ADVANCED
Joint Entrance Examination

SECTION I
THE TIME LINE
(Last 5 Years' JEEAdvanced
Years JEE Advanced Questions)
Questions)
TheThe
chapterwise question
chapterwise of JEE
question Advanced
of JEE given
Advanced here
given willwill
here help you to
help
youget acquainted
to get with
acquainted thethe
with pattern andand
pattern difficulty level
difficulty of of
level
JEE Advanced. Solutions
JEE Advance. to alltoquestions
Solutions are given
all questions for your
are given reference.
for your
reference.

THE TIME LINE


TIME LINE
JEE Advanced 2014-18

PHYSICS
1. General Physics 4. A length-scale (l) depends on the permittivity
( ε ) of a dielectric material, Boltzmann’s
1. The energy of a system as a function of time t is
constant ( kB ), the absolute temperature (T ), the
given as E( t ) = A exp ( −αt ),
2
number per unit volume ( n ) of certain charged
where α = 0.2 s −1. The measurement of A has an particles, and the charge ( q ) carried by each of
error of 1.25%. If the error in the measurement the particles. Which of the following expression
of time is 1.50%, the percentage error in the (s) for l is (are) dimensionally correct?
value of E( t ) at t = 5 s is (Integer Type, 2015) (More than One Correct Option, 2016)

2. Planck’s constant h, speed of light c and  nq 


2
 εk T 
(a) l =   (b) l =  B 2 
gravitational constant G are used to form a unit  εkBT   nq 
of length L and a unit of mass M. Then, the  q2   q2 
correct options is/are (c) l =  2 / 3 
 (d) l =  1/ 3 

(More than One Correct Option, 2015)  εn kBT   εn kBT 

(a) M ∝ c (b) M ∝ G 5. In an experiment to determine the acceleration


(c) L ∝ h (d) L ∝ G due to gravity g, the formula used for the time
3. Consider a Vernier caliper in which each 1 cm 7( R − r )
period of a periodic motion is T = 2π .
on the main scale is divided into 8 equal 5g
divisions and a screw gauge with 100 divisions The values of R and r are measured to be ( 60 ± 1)
on its circular scale. In the Vernier calipers, 5 mm and (10 ± 1) mm, respectively. In five
divisions of the Vernier scale coincide with 4 successive measurements, the time period is
divisions on the main scale and in the screw found to be
gauge, one complete rotation of the circular 0.52 s, 0.56 , 0.57 s, 0.54 s and 0.59 s. The least
scale moves it by two divisions on the linear count of the watch used for the measurement of
scale. Then (More than One Correct Option, 2015) time period is 0.01 s. Which of the following
(a) if the pitch of the screw gauge is twice the least statement(s) is (are) true?
count of the vernier caliper, the least count of the (More than One Correct Option, 2016)
screw gauge is 0.01 mm (a) The error in the measurement of r is 10%
(b) if the pitch of the screw gauge is twice the least
(b) The error in the measurement of T is 3.57%
count of the Vernier caliper, the least count of the
screw gauge is 0.05 mm (c) The error in the measurement of T is 2%
(c) if the least count of the linear scale of the screw (d) The error in the measurement of g is 11%
gauge is twice the least count of the Vernier
6. There are two vernier calipers both of which
calipers, the least count of the screw gauge is
0.01 mm have 1 cm divided into 10 equal divisions on the
(d) if the least count of the linear scale of the screw main scale. The vernier scale of one of the
gauge is twice the least count of the vernier calipers (C1 ) has 10 equal divisions that
caliper, the least count of the screw gauge is correspond to 9 main scale divisions. The
0.005 mm. vernier scale of the other caliper (C2 ) has 10

THE TIME LINE


4 The Time Line~JEE Advanced Questions (2014 –2018)

equal divisions that correspond to 11 main scale series expansion and truncating the expansion at the first
divisions. The readings of the two calipers are power of the error. For example, consider the relation
shown in the figure. The measured values (in z = x /y. If the errors in x , y and z are ∆x, ∆y and ∆z
cm) by calipers C1 and C2 respectively, are respectively, then
−1
(Single Correct Option, 2016) x ± ∆x x  ∆x   ∆y 
z ± ∆z = = 1 ±  1 ± 
y ± ∆y y  x 
2 3 4
y 
C1 −1
 ∆y 
The series expansion for 1 ±  , to first power in
0 5 10  y 
2 3 4 ∆y /y, is 1 m ( ∆y /y ). The relative errors in independent
variables are always added. So, the error in z will be
C2
 ∆x ∆y 
∆z = z  + 
0 5 10  x y 
(a) 2.87 and 2.87 (b) 2.87 and 2.83
The above derivation makes the assumption that
(c) 2.85 and 2.82 (d) 2.87 and 2.86
∆x / x << 1, ∆ y /y << 1. Therefore, the higher powers of
7. A person measures the depth of a well by these quantities are neglected. (Paragraph Type, 2018)
measuring the time interval between (1 − a )
dropping a stone and receiving the sound of 10. Consider the ratio r = to be determined
(1 + a )
impact with the bottom of the well. The error in
his measurement of time is by measuring a dimensionless quantity a. If the
δT = 0. 01 s and he measures the depth of the error in the measurement of a is
well to be L = 20 m. Take the acceleration due ∆ a ( ∆ a / a << 1), then what is the error ∆r in
determining r ?
to gravity g = 10 ms −2 and the velocity of sound
∆a 2 ∆a 2 ∆a 2a∆a
is 300 ms −1. Then the fractional error in the (a)
(1 + a) 2
(b)
(1 + a) 2
(c)
(1 − a) 2
(d)
(1 − a 2 )
δL
measurement, , is closest to
L (Single Correct Option, 2017) 11. In an experiment, the initial number of
(a) 1% (b) 5% (c) 3% (d) 0.2% radioactive nuclei is 3000. It is found that
1000 ± 40 nuclei decayed in the first 1.0 s. For
|x|<< 1, ln (1 + x ) = x upto first power in x. The
Paragraph (Q. Nos. 8-9) error ∆λ, in the determination of the decay
In electromagnetic theory, the electric and magnetic constant λ in s −1, is
phenomena are related to each other. Therefore, the (a) 0.04 (b) 0.03 (c) 0.02 (d) 0.01
dimensions of electric and magnetic quantities must also
be related to each other. In the questions below, [E] and
[B] stand for dimensions of electric and magnetic fields 2. Kinematics
respectively, while [ε 0 ] and [µ 0 ] stand for dimensions of
the permittivity and permeability of free space, 12. Airplanes A and B are flying with constant
respectively. [L ] and [T ] are dimensions of length and velocity in the same vertical plane at angles 30°
time, respectively. All the quantities are given in SI units. and 60° with respect to the horizontal
(Paragraph Type, 2018) respectively as shown in figure. The speed of A
is 100 3 ms −1. At time t = 0 s, an observer in A
8. The relation between [E] and [B] is finds B at a distance of 500 m. This observer
(a) [E] = [B] [L] [T] (b) [E] = [B] [L]−1[T] sees B moving with a constant velocity
(c) [E] = [B][L][T]−1 (d) [E] = [B][L]−1[T]−1 perpendicular to the line of motion of A. If at
9. The relation between [ε 0 ] and [µ 0 ] is t = t0 , A just escapes being hit by B, t0 in
seconds is (Integer Type, 2014)
(a) [µ 0 ] = [ε0 ][L]2 [T]−2 (b) [µ 0 ] = [ε0 ][L]−2 [T]2
(c) [µ 0 ] = [ε0 ]−1 [L]2 [T]−2 (d) [µ 0 ] = [ε0 ]−1[L]−2 [T]2 A

Paragraph (Q. Nos. 10-11)


B
If the measurement errors in all the independent
quantities are known, then it is possible to determine the
30° 60°
error in any dependent quantity. This is done by the use of

THE TIME LINE


The Time Line~JEE Advanced Questions (2014 –2018) 5

13. A rocket is moving in a gravity free space


with a constant acceleration of 2 ms −2 along
3. Laws of Motion
+x direction (see figure). The length of a 17. A block of mass m1 = 1 kg another mass m2 = 2 kg
chamber inside the rocket is 4 m. A ball is are placed together (see figure) on an inclined
thrown from the left end of the chamber in plane with angle of inclination θ. Various values
+x direction with a speed of 0.3 ms −1 relative of θ are given in List I. The coefficient of friction
to the rocket. At the same time, another ball between the block m1 and the plane is always
is thrown in −x direction with a speed of 0.2 zero. The coefficient of static and dynamic friction
between the block m2 and the plane are equal
ms −1 from its right end relative to the rocket.
to µ = 0.3. In Column II expressions for the
The time in seconds when the two balls hit
friction on the block m2 are given. Match the
each other is (Integer Type, 2014)
correct expression of the friction in Column II
with the angles given in Column I, and choose
a=2 m/s2 the correct option. The acceleration due to
0.3 ms–1 0.2 ms–1 x gravity is denoted by g. (Matching Type, 2014)

4m µ1

14. Three vectors P, Q and R are shown in the m


1
m
figure. Let S be any point on the vector R. 2
The distance between the points P and S is b θ
[R]. The general relation among vectors P,Q θ µ2
and S is (Single Correct Option, 2017) [Useful information tan ( 5.5° ) ≈ 0.1;
Y tan (11.5° ) ≈ 0.2; tan (16.5° ) ≈ 0.3]
bR
P Column I Column II
R =Q – P P. θ = 5° 1. m2 g sin θ
S
P S Q Q. θ = 10° 2. (m1 + m2 ) g sin θ
Q
R. θ = 15° 3. µ m2 g cos θ
S. θ = 20° 4. µ (m1 + m2 ) g cos θ
X
O
Codes
(a) S = (1 − b 2 )P + b Q (b) S = (b − 1)P + b Q P Q R S P Q R S
(c) S = (1 − b)P + b Q (d) S = (1 − b)P + b 2Q (a) 1, 1, 1, 3 (b) 2, 2, 2, 3
15. A ball is projected from the ground at an angle (c) 2, 2, 2, 4 (d) 2, 2, 3, 3
of 45° with the horizontal surface. It reaches a 18. In the figure, a ladder of mass m is shown
maximum height of 120 m and returns to the leaning against a wall. It is in static
ground. Upon hitting the ground for the first equilibrium making an angle θ with the
time, it loses half of its kinetic energy. horizontal floor. The coefficient of friction
Immediately after the bounce, the velocity of between the wall and the ladder is µ 1 and that
the ball makes an angle of 30° with the between the floor and the ladder is µ 2.
horizontal surface. The maximum height it
reaches after the bounce, in metres, is .......... . The normal reaction of the wall on the ladder is
(Integer Type, 2018) N 1 and that of the floor is N 2. If the ladder is
about to slip, then
16. A solid horizontal surface is covered with a thin (More than One Correct Option, 2014)
layer of oil. A rectangular block of mass mg
(a) µ 1 = 0, µ 2 ≠ 0 and N 2 tan θ =
m = 0.4 kg is at rest on this surface. An impulse 2
of 1.0 N s is applied to the block at time t = 0, so (b) µ 1 ≠ 0, µ 2 = 0 and N1 tan θ =
mg
that it starts moving along the X-axis with a 2
velocity v ( t ) = v0 e− t/ τ , where v0 is a constant (c) µ 1 ≠ 0, µ 2 ≠ 0 and N 2 =
mg
and τ = 4 s. The displacement of the block, in 1 + µ 1µ 2
metres, at t = τ is .............. . (Take, e−1 = 0.37). (d) µ 1 = 0, µ 2 ≠ 0 and N1 tan θ =
mg
(Integer Type, 2018) 2

THE TIME LINE


6 The Time Line~JEE Advanced Questions (2014 –2018)

19. A uniform wooden stick of mass 1.6 kg of length 23. Consider an elliptically shaped rail PQ in
l rests in an inclined manner on a smooth, the vertical plane with OP = 3 m and
vertical wall of height h (< l ) such that a small OQ = 4 m. A block of mass 1 kg is pulled
portion of the stick extends beyond the wall. along the rail from P to Q with a force of
The reaction force of the wall on the stick is 18 N, which is always parallel to line PQ
perpendicular to the stick. The stick makes (see figure). Assuming no frictional losses,
an angle of 30° with the wall and the bottom the kinetic energy of the block when it
of the stick is on a rough floor. The reaction of reaches Q is ( n × 10) J. The value of n is
the wall on the stick is equal in magnitude to (take acceleration due to gravity = 10 ms −2 )
the reaction of the floor on the stick. The ratio (Integer Type, 2014)
h / l and the frictional force f at the bottom of Q
the stick are ( g = 10 ms −2 )
(Single Correct Option, 2016)
4m
h 3 16 3 h 3 16 3
(a) = ,f = N (b) = ,f = N
l 16 3 l 16 3
h 3 3 8 3 h 3 3 16 3 90°
(c) = ,f = N (d) = ,f = N
l 16 3 l 16 3 O 3m P

24. A tennis ball is dropped on a horizontal smooth


4. Work, Power and Energy surface. It bounces back to its original position
after hitting the surface.
20. A particle of mass 0.2 kg is moving in one
dimension under a force that delivers a constant The force on the ball during the collision is
power 0.5 W to the particle. If the initial speed proportional to the length of compression of the
(in ms −1) of the particle is zero, the speed (in ms −1) ball. Which one of the following sketches
after 5 s is (Integer Type, 2013) describes the variation of its kinetic energy K
with time t most appropriately? The figures are
only illustrative and not to the scale.
Paragraph (Q. Nos. 21-22) (Single Correct Option, 2014)
A small block of mass 1 kg is released from rest at
the top of a rough track. The track is a circular
arc of radius 40 m. The block slides along the
track without toppling and a frictional force acts (a) K (b) K
on it in the direction opposite to the instantaneous
t t
velocity. The work done in overcoming the friction
up to the point Q, as shown in the figure, is 150 J.
(Take the acceleration due to gravity, g = 10 ms−2) (c) K
(Paragraph Type, 2013) (d) K
y
t t
R P
30°
25. A wire, which passes through
A
R the hole in a small bead, is bent
Q in the form of quarter of a circle.
The wire is fixed vertically on
O x ground as shown in the figure. 90°
B
The bead is released from near
21. The speed of the block when it reaches the
the top of the wire and it slides
point Q is
along the wire without friction. As the bead
(a) 5 ms −1 (b) 10 ms−1
moves from A to B, the force it applies on the
(c) 10 3 ms−1 (d) 20 ms−1
wire is (Single Correct Option, 2014)]
22. The magnitude of the normal reaction that acts (a) always radially outwards
on the block at the point Q is (b) always radially inwards
(a) 7.5 N (b) 8.6 N (c) radially outwards initially and radially inwards later
(c) 11.5 N (d) 22.5 N (d) radially inwards initially and radially outwards later

THE TIME LINE


The Time Line~JEE Advanced Questions (2014 –2018) 7

26. A particle of unit mass is moving along the where k is a positive constant of appropriate
x-axis under the influence of a force and its dimensions. This particle is moving in a circular
total energy is conserved. Four possible forms of orbit of radius R about the point O. If v is the
the potential energy of the particle are given in speed of the particle and L is the magnitude of
Column I (a and U 0 are constants). Match the its angular momentum about O, which of the
potential energies in Column I to the corresponding following statements is (are) true?
statements in Column II (Matching Type, 2015) (More than One Correct Option, 2018)
k k
(a) v = R (b) v = R
Column I Column II 2m m
2
U  x 
1. The force acting on the
2 mk 2
P. U1( x) = 0 1 −    particle is zero at x = a (c) L = mk R 2 (d) L = R
2   a  2

U0  x 
2
2. The force acting on the 29. A particle of mass m is initially at rest at the
Q. U 2 ( x) =   particle is zero at x = 0 origin. It is subjected to a force and starts
2  a
moving along the X-axis. Its kinetic energy K
U0  x 
2   x 2  3. The force acting on the
R. U 3 ( x) =   exp −    changes with time as dK / dt = γt, where γ is a
2  a   particle is zero at
 a  x= −a
positive constant of appropriate dimensions.
Which of the following statements is (are) true?
U0  x 1  x 3  4. The particle (More than One Correct Option, 2018)
S. U 4 ( x) =  −    experiences an
2  a 3  a   (a) The force applied on the particle is constant
attractive force towards
x = 0 in the region (b) The speed of the particle is proportional to time
| x| < a (c) The distance of the particle from the origin
5. The particle with total increases linearly with time
U (d) The force is conservative
energy 0 can
4
30. In the Column I below, four different paths of a
oscillate about the
point x = − a. particle are given as functions of time. In these
functions, α and β are positive constants of
Codes appropriate dimensions and α ≠ β. In each case,
P Q R S the force acting on the particle is either zero or
(a) 1,2,3,4 2,3 2,1,3,4 1,3,5 conservative. In Column II, five physical
(b) 3,2,4,1 1,2 1,2,3,4 2,1,3 quantities of the particle are mentioned: p is the
(c) 2,3,1,4 4,1 3,2,1,4 5,1,2 linear momentum, L is the angular momentum
(d) 4,3,2,1 5,2 5,4,2,1 4,2,3
about the origin, K is the kinetic energy, U is
27. A flat plane is moving normal to its plane the potential energy and E is the total energy.
through a gas under the action of a constant Match each path in Column I with those
force F. The gas is kept at a very low pressure. quantities in Column II, which are conserved
The speed of the plate v is much less than the for that path. (Matching Type, 2018)
average speed u of the gas molecules. Which of
the following options is/are true? Column I Column II
(More than One Correct Option, 2017)
P. r(t ) = α t $i + β t $j 1. p
(a) At a later time the external force F balances the
resistive force Q. r(t ) = α cos ω t $i + β sin ω t $j 2. L
(b) The plate will continue to move with constant
R. r(t ) = α (cos ω t $i + sin ωt $j ) 3. K
non-zero acceleration, at all time
(c) The resistive force experienced by the plate is β
S. r(t ) = at $i + t 2 $j 4. U
proportional to v 2
(d) The pressure differnce between the leading and
trailing faces of the plate is proportional to uv 5. E

Codes
5. Center of Mass (a) P → 1, 2, 3, 4, 5; Q → 2, 5; R → 2, 3, 4, 5; S → 5
(b) P → 1, 2, 3, 4, 5; Q → 3, 5; R → 2, 3, 4, 5; S → 2, 5
28. The potential energy of mass m at a distance r (c) P → 2, 3, 4; Q → 5; R → 1, 2, 4; S → 2, 5
from a fixed point O is given by V (r ) = kr 2 / 2, (d) P → 1, 2, 3, 5; Q → 2, 5; R → 2, 3, 4, 5; S → 2, 5

THE TIME LINE


8 The Time Line~JEE Advanced Questions (2014 –2018)

31. Consider a body of mass 1.0 kg at rest at the the perimeter of the disc (see figure). One
origin at time t = 0 . A force F = (αt $i + β $j) is second after applying the forces, the angular
applied on the body, where α = 1.0 Ns−1 and speed of the disc in rad s −1 is (Integer Type, 2014)
β = 1.0 N. The torque acting on the body about
35. A ring of mass M and radius
the origin at time t = 1.0s is τ. Which of the ω
R is rotating with angular
following statements is (are) true?
(More than One Correct Option, 2018)
speed ω about a fixed vertical
axis passing through its
1
(a) | τ| = N - m centre O with two
3 O
point masses each of mass
(b) The torque τ is in the direction of the unit vector + k$
M / 8 at rest at O. These
(c) The velocity of the body at t = 1s is
1 masses can move radially
v = ($i + 2 $j) ms −1 outwards along two massless
2
(d) The magnitude of displacement of the body at rods fixed on the ring as shown in the figure. At
1 some instant,
t = 1s is m
6 the angular speed of the system is ( 8 / 9) ω and
3
32. Two vectors A and B are defined as A = a$i and one of the masses is at a distance of R from O.
B = a (cos ωt$i + sin ωt$j), where a is a constant and 5
ω = π /6 rad s −1. If|A + B|= 3|A − B| at time At this instant, the distance of the other mass
from O is (More than One Correct Option, 2015)
t = τ for the first time, the value of τ, in seconds,
2 1
is ............. . (Integer Type, 2018) (a) R (b) R
3 3
33. A horizontal circular platform of radius 0.5 m 3
(c) R
4
(d) R
and mass 0.45 kg is free to rotate about its axis. 5 5
Two massless spring toy-guns, each carrying a
36. Two identical uniform discs roll without
steel ball of mass 0.05 kg are attached to the
slipping on two different surfaces AB and CD
platform at a distance 0.25 m from the centre
(see figure) starting at A and C with linear
on its either sides along its diameter (see
speeds v1 and v2, respectively, and always
figure). Each gun simultaneously fires the balls
remain in contact with the surfaces.
horizontally and perpendicular to the diameter
in opposite directions. After leaving the A v1= 3 m/s
platform, the balls have horizontal speed of
30 m
9 ms −1 with respect to the ground. The
B
rotational speed of the platform in rads −1 after
the balls leave the platform is (Integer Type, 2014) C v2

27 m
D

If they reach B and D with the same linear


speed and v1 = 3 m/s, then v2 in m/s is
(g = 10 m/s 2) (Integer Type, 2015)

37. The densities of two solids spheres A and B of


34. A uniform circular disc of the same radii R vary with radial distance r as
F 5
mass 1.5 kg and radius 0.5 m X r r
ρA (r ) = k   and ρB (r ) = k  respectively,
is initially at rest on a  R  R
horizontal frictionless where k is a constant. The moments of inertia of
surface. Three forces of equal O
F the individual spheres about axes passing
magnitude F = 0.5 N are Y Z through their centres are I A and I B ,
applied simultaneously I n
along the three sides of an
F respectively. If B = , the value of n is
I A 10 (Integer Type, 2015)
equilateral triangle XYZ with its vertices on

THE TIME LINE


The Time Line~JEE Advanced Questions (2014 –2018) 9

38. The position vector r of particle of mass m is reference and r is the position vector of the
given by the following equation r ( t ) = α t 3 $i + β t 2 $j particle with respect to the centre of the disc
10 −3 Now, consider a smooth
where, α = ms , β = 5 ms −2 and m = 0.1 kg. slot along a diameter of a
ω
3
At t = 1s, which of the following statement(s) is disc of radius R rotating
counter-clockwise with a R
(are) true about the particle?
constant angular speed ω
(More than One Correct Option, 2016)
about its vertical axis m
(a) The velocity v is given by v = (10$i + 10$j)ms−1 through its centre. R/2
(b) The angular momentum L with respect to the origin We assign a coordinate
is given by L = (5 / 3) k$ N - ms system with the origin at
(c) The force F is given by F = ($i + 2 $j) N the centre of the disc, the X-axis along the slot,
the Y-axis perpendicular to the slot and the z-axis
(d) The torque τ with respect to the origin is given by $ ). A small block of
20 $ along th rotation axis (ω = ωk
τ = − kN -m
3 mass m is gently placed in the slot at r = (R /2) $i at
t = 0 and is constrained to move only along the
39. Two thin circular discs of mass m and 4m, slot. (Paragraph Type, 2016)
having radii of a and 2a, respectively, are
rigidly fixed by a massless, rigid rod of length 40. The distance r of the block at time t is
R R
l = 24 a through their centers. This assembly (a) cos 2ωt (b) cosωt
2 2
is laid on a firm and flat surface and set rolling
R R
without slipping on the surface so that the (c) (e ωt + e − ωt ) (d) (e 2 ωt + e −2 ωt )
4 4
angular speed about the axis of the rod is ω. The
angular momentum of the entire assembly 41. The net reaction of the disc on the block; is
about the point O is L (see the figure). Which of (a) mω2R sinωt $j − mgk$
the following statement(s) is (are) true? 1
(More than One Correct Option, 2016) (b) mω2R (e ωt − e − ωt )$j + mgk$
2
4m 1
(c) mω2R (e 2 ω t − e − ω t )$j + mgk$
2
z m
l ω (d) −mω2R cosωt $j − mgk$

l 2a
a
o Paragraph (Q. Nos. 42-43)
One twirls a circular ring (of mass M and radius
(a) The magnitude of the z-component of L is 55 ma 2 ω
R) near the tip of one’s finger as shown in Figure
(b) The magnitude of angular momentum of centre of 1. In the process the finger never loses contact
mass of the assembly about the point O is 81 ma 2 ω with the inner rim of the ring. The finger traces
(c) The centre of mass of the assembly rotates about out the surface of a cone, shown by the dotted
the Z-axis with an angular speed of ω /5 line. The radius of the path traced out by the
(d) The magnitude of angular momentum of the point where the ring and the finger is in contact is
assembly about its centre of mass is 17 ma 2 ω / 2 r. The finger rotates with an angular velocity ω 0.
The rotating ring rolls without slipping on the
Paragraph (Q. Nos. 40-41) outside of a smaller circle described by the point
where the ring and the finger is in contact (Figure
A frame of the reference that is accelerated with 2). The coefficient of friction between the ring
respect to an inertial frame of reference is called a and the finger is µ and the acceleration due to
non-inertial frame of reference. A coordinate gravity is g (Paragraph Type, 2017)
system fixed on a circular disc rotating about a
fixed axis with a constant angular velocity ω is an
example of a non-inertial frame of reference. R
The relationship between the force Frot
experienced by a particle of mass m moving on the r
R
rotating disc and the force Fin experienced by the
particle in an inertial r frame
r ofr reference is, Figure 2
Frot = Fin + 2m ( vrot × ω ) + m (ω × r ) × ω, where, vrot is Figure 1
.
the velocity of the particle in the rotating frame of

THE TIME LINE


10 The Time Line~JEE Advanced Questions (2014 –2018)

42. The total kinetic energy of the ring is (c) If the force is applied at point P tangentially, then τ
1 decreases continuously as the wheel climbs
(a) Mω20 (R − r) 2
(b) Mω20 (R − r )2
2 (d) If the force is applied normal to the circumference
3 at point X, then τ is constant
(c) Mω20R 2 (d) Mω20 (R − r )2
2 46. A rigid uniform bar AB of length L is slipping
43. The minimum value of ω 0 below which the ring from its vertical position on a frictionless floor
will drop down is (as shown in the figure). At some instant of
g 3g time, the angle made by the bar with the
(a) (b)
2µ (R − r ) 2 µ (R − r ) vertical is θ. Which of the following statements
g 2g about its motion is/are correct?
(c) (d) (More than One Correct Option, 2017)
µ (R − r ) µ (R − r )

44. Consider regular polygons with number of sides


n = 3, 4, 5, …… as shown in the figure. The θ
centre of mass of all the polygons is at height h L
from the ground. They roll on a horizontal
surface about the leading vertex without slipping B
and sliding as depicted. The maximum increase O
in height of the locus of the centre of mass for (a) Instantaneous torque about the point in contact with
each each polygon is ∆. Then, ∆ depends on n the floor is proportional to sinθ
and h as (Single Correct Option, 2017) (b) The trajectory of the point A is parabola
(c) The mid-point of the bar will fall vertically downward
(d) When the bar makes an angle θ with the vertical, the
displacement of its mid-point from the initial position
h h h is proportional to (1 − cosθ)

π 2π 47. A block of mass M has a circular cut with a


(a) ∆ = h sin2   (b) ∆ = h sin  
n  n  frictionless surface as shown. The block rests on
  the horizontal frictionless surfaced of a fixed
  table. Initially the right edge of the block is at
π
(c) ∆ = h tan2  
1
(d) ∆ = h  − 1 x = 0, in a coordinate system fixed to the table.
 2n   cos π  
    A point mass m is released from rest at the
 n 
topmost point of the path as shown and it slides
45. A wheel of radius R and mass M is placed at down. When the mass loses contact with the
the bottom of a fixed step of height R as shown block, its position is x and the velocity is v. At
in the figure. A constant force is continuously that instant, which of the following option is/are
applied on the surface of the wheel so that it correct? (More Than One Correct Option, 2017)

just climbs the step without slipping. Consider R


the torque τ about an axis normal to the plane m
of the paper passing through the point Q. y
R
Which of the following options is/are correct?
(More than One Correct Option, 2017)
x M
S

x=0
Q
P 2gR
(a) The velocity of the point mass m is v =
R m
1+
M
(b) The x component of displacement of the centre of
mR
mass of the block M is −
M +m
(a) If the force is applied normal to the circumference mR
at point P, then τ is zero (c) The position of the point mass is x = − 2
M +m
(b) If the force is applied tangentially at point S, then m
τ ≠ 0 but the wheel never climbs the step (d) The velocity of the block M is V = − 2gR
M

THE TIME LINE


The Time Line~JEE Advanced Questions (2014 –2018) 11

p  r =
48. A ring and a disc are initially at rest, side by 3R 

 4  = 63
side, at the top of an inclined plane which (a) p (r = 0) = 0 (b)
p  r =
makes an angle 60° with the horizontal. They 2R  80

start to roll without slipping at the same  3 
p  r =
3R 
p  r = 
instant of time along the shortest path. If the R

time difference between their reaching the  5  = 16  2  = 20
(c) (d)
p  r =
ground is (2 − 3 ) / 10 s, then the height of the 2R  21  R  27
 p r = 
 5   3
top of the inclined plane, in metres, is .......... .
(Take, g = 10ms −2) (Integer Type, 2018) 53. A rocket is launched normal to the surface of
the Earth, away from the Sun, along the line
6. Gravitation joining the Sun and the Earth. The Sun is
3 × 105 times heavier than the Earth and is at a
49. A planet of radius R = 1 / 10 × (radius of earth) distance 2 .5 × 104 times larger than the radius
has the same mass density as earth. Scientists
R of Earth. The escape velocity from Earth’s
dig a well of depth on it and lower a wire of
5 gravitational field is ve = 11.2 km s −1. The
the same length and of linear mass density minimum initial velocity ( vs ) required for the
10−3 kgm −1 into it. If the wire is not touching rocket to be able to leave the Sun-Earth system
anywhere, the force applied at the top of the is closest to (Ignore the rotation and revolution
wire by a person holding it in place is (take the of the Earth and the presence of any other
radius of earth = 6 × 106 m and the acceleration planet) (Single Correct Option, 2017)

due to gravity of earth is 10 ms −2 ) (a) vs = 72 km s−1 (b) vs = 22 km s−1


(c) vs = 42 km s−1 (d) vs = 62 km s−1
(Single Correct Option, 2014)
(a) 96 N (b) 108 N (c) 120 N (d) 150 N 54. A planet of mass M, has two natural satellites
with masses m1 and m2. The radii of their
50. A bullet is fired vertically upwards with velocity circular orbits are R1 and R2, respectively.
v from the surface of a spherical planet. When it Ignore the gravitational force between the
reaches its maximum height, its acceleration satellites. Define v1 , L1 , K 1 and T1 to be
due to the planet’s gravity is 1 / 4 th of its value respectively, the orbital speed, angular
at the surface of the planet. If the escape momentum, kinetic energy and time period of
velocity from the planet is vsec = v N , then the revolution of satellite 1; and v2 , L2 , K 2 and T2 to
value of N is (ignore energy loss due to be the corresponding quantities of satellite 2.
atmosphere) (Integer Type, 2015)
Given, m1 / m2 = 2 and R1 / R2 = 1 / 4, match the
51. A large spherical mass M is fixed at one ratios in Column I to the numbers in Column
position and two identical masses m are kept on II. (Matching Type, 2018)
a line passing through the centre of M (see
figure). The point masses are connected by a Column I Column II
rigid massless rod of length l and this assembly P. v1 1. 1
is free to move along the line connecting them. v2 8
M m m
Q. L1 2. 1
r l L2
All three masses interact only through their
mutual gravitational interaction. When the R. K1 3. 2
point mass nearer to M is at a distance r = 3l K2
from M the tension in the rod is zero for
S. T1 4. 8
 M 
m = k  . The value of k is T2
 288 (Integer Type, 2015)
Codes
52. A spherical body of radius R consists of a fluid
(a) P → 4; Q → 2; R → 1; S → 3
of constant density and is in equilibrium under
(b) P → 3; Q → 2; R → 4; S → 1
its own gravity. If p (r ) is the pressure at
r(r < R ), then the correct options is/are (c) P → 2; Q → 3; R → 1; S → 4
(More than One Correct Option, 2015) (d) P → 2; Q → 3; R → 4; S → 1

THE TIME LINE


12 The Time Line~JEE Advanced Questions (2014 –2018)

57. If the piston is pushed at a speed of 5 ms −1, the


7. Mechanical Properties air comes out of the nozzle with a speed of
of Solids and Fluids (a) 0.1 ms −1 (b) 1 ms −1 (c) 2 ms −1 (d) 8 ms −1

55. A glass capillary tube is the shape 58. If the density of air is ρa and that of the liquid
of truncated cone with an apex ρl , then for a given piston speed the rate
angle α so that its two ends have h (volume per unit time) at which the liquid is
cross-sections of different radii. sprayed will be proportional to
When dipped in water vertically, ρa ρl
(a) (b) ρa ρl (c) (d) ρl
water rises in it to a height h, ρl ρa
where the radius of its
cross-section is b. If the surface tension of water 59. A person in a lift is holding a water jar, which
is S, its density is ρ, and its contact angle with has a small hole at the lower end of its side.
glass is θ, the value of h will be ( g is the When the lift is at rest, the water jet coming
acceleration due to gravity) out of the hole hits the floor of the lift at a
(Single Correct Option, 2014) distance d of 1.2 m from the person.
2S 2S In the following, state of the lift’s motion is given
(a) cos (θ − α) b) cos (θ + α)
bρg bρg in Column I and the distance where the water
2S 2S jet hits the floor of the lift is given in Column II.
(c) cos (θ − α / 2) (d) cos (θ + α / 2)
bρg bρg Match the statements from Column I with those
in Column II and select the correct answer using
56. During Searle’s experiment, zero of the vernier the code given below the lists.
scale lies between 3.20 × 10−2 m and 3.25 × 10−2
m of the main scale. The 20th division of the Column I Column II
vernier scale exactly coincides with one of the P. Lift is accelerating vertlcally up. 1. d = 1.2 m
main scale divisions. Q. Lift is accelerating with an 2. d > 1.2 m
When an additional load of 2 kg is applied to acceleration loss than the
gravitational acceleration.
the wire, the zero of the vernier scale still lies
between 3.20 × 10−2 m and 3.25 × 10−2 m of the R. Lift is moving vertically up with 3. d < 1.2 m
constant speed.
main scale but now the 45th division of vernier
scale coincides with one of the main scale S. Lift is falling freely. 4. No water leaks
out of the jar
divisions. The length of the thin metallic wire is
2 m and its cross-sectional area is 8 × 10−7 m 2. Codes (More than One Correct Option, 2015)
The least count of the vernier scale is 1.0 × 10−5 P Q R S P Q R S
m. The maximum percentage error in the (a) 2, 3, 2, 4 (b) 2, 3, 1, 4
Young’s modulus of the wire is (c) 1, 1, 1, 4 (d) 2, 3, 1, 1
(Integer Type, 2014)
60. Two spheres P and Q for equal
radii have densities ρ1 and ρ2, L1
Paragraph (Q. Nos. 57-58) respectively. The spheres are P
A spray gun is shown in the figure where a piston connected by a massless string
pushes air out of nozzle. A thin tube of uniform and placed in liquids L1 and L2 L2
cross-section is connected to the nozzle. The other of densities σ1 and σ 2 and Q
end of the tube is in a small liquid container. As viscosities η1 and η2,
the piston pushes air through the nozzle, the respectively. They float in
liquid from the container rises into the nozzle and equilibrium with the sphere P in L1 and sphere
is sprayed out. For the spray gun shown, the radii Q in L2 and the string being taut (see figure).
of the piston and the nozzle are 20 mm and 1 mm
respectively. The upper end of the container is If sphere P alone in L2 has terminal velocity vP
open to the atmosphere. (Paragraph Type, 2014) and Q alone in L1 has terminal velocity vQ , then
(More than One Correct Option, 2015)
|v | η | vP | η2
(a) P = 1 (b) =
| vQ | η2 | vQ | η1
(c) vP ⋅ vQ > 0 (d) vP ⋅ vQ < 0

THE TIME LINE


The Time Line~JEE Advanced Questions (2014 –2018) 13

61. In plotting stress versus strain curves for two 65. A uniform capillary tube of inner radius r is
materials P and Q, a student by mistake puts dipped vertically into a beaker filled with
strain on the y-axis and stress on the x-axis as water. The water rises to a height h in the
shown in the figure. Then, the correct statements capillary tube above the water surface in the
is/are (More than One Correct Option, 2015) beaker. The surface tension of water is σ. The
angle of contact between water and the wall of
the capillary tube is θ. Ignore the mass of water
in the meniscus. Which of the following
Strain

P statements is (are) true ?


Q (More than One Correct Option, 2018)
(a) For a given material of the capillary tube, h
decreases with increase in r
Stress (b) For a given material of the capillary tube, h is
(a) P has more tensile strength than Q independent of σ
(b) P is more ductile than Q (c) If this experiment is performed in a lift going up with
(c) P is more brittle than Q a constant acceleration, then h decreases
(d) The Young’s modulus of P is more than that of Q (d) h is proportional to contact angle θ

62. Consider an expanding sphere of instantaneous 66. Consider a thin square plate floating on a
radius R whose total mass remains constant. viscous liquid in a large tank. The height h of
The expansion is such that the instantaneous the liquid in the tank is much less than the
density ρ remains uniform throughout the width of the tank. The floating plate is pulled
volume. The rate of fractional change in density horizontally with a constant velocity u 0. Which
 1 d ρ of the following statements is (are) true?
  is constant. The velocity v of any point (More than One Correct Option, 2018)
 p dt 
(a) The resistive force of liquid on the plate is inversely
of the surface of the expanding sphere is
proportional to h
proportional to (Single Correct Option, 2017)
(b) The resistive force of liquid on the plate is
1 independent of the area of the plate
(a) R (b)
R (c) The tangential (shear) stress on the floor of the tank
2
increases with u0
(c) R 3 (d) R 3
(d) The tangential (shear) stress on the plate varies
63. A drop of liquid of radius R = 10−2 m having linearly with the viscosity η of the liquid
01
.
surface tension S = Nm −1 divides itself into
4π 8. Heat, and Thermodynamics
K identical drops. In this process the total
change in the surface energy ∆U = 10−3 J. If 67. A thermodynamic system is taken from an
K = 10α , then the value of α is (Integer Type, 2017) initial state i with internal energy U i = 100 J to
the final state f along two different paths iaf
64. A steel wire of diameter 0.5 mm and Young’s and ibf, as schematically shown in the figure.
modulus 2 × 1011 N m −2 carries a load of mass The work done by the system along the paths
m. The length of the wire with the load is 1.0 m. af, ib and bf are W af = 200 J, W ib = 50 J and
A Vernier scale with 10 divisions is attached to W bf = 100 J respectively. The heat supplied to
the end of this wire. Next to the steel wire is a the system along the path iaf, ib and bf are
reference wire to which a main scale, of least Qiaf , Qib and Qbf respectively. If the internal
count 1.0 mm, is attached. The 10 divisions of energy of the system in the state b is U b = 200 J
the Vernier scale correspond to 9 divisions of and Qiaf = 500 J, the ratio Qbf / Qib is
(Integer Type, 2014)
the main scale. Initially, the zero of Vernier
scale coincides with the zero of main scale. If p
the load on the steel wire is increased by 1.2 kg, a f
the Vernier scale division which coincides with
a main scale division is ......... . (Take,
g = 10 ms −2 and π = 3.2). (Integer Type, 2018)
i b
V

THE TIME LINE


14 The Time Line~JEE Advanced Questions (2014 –2018)

68. Parallel rays of light of intensity I = 912 Wm −2 (d) The ratio of the rms speed of helium atoms to that of
1
are incident on a spherical black body kept in hydrogen molecules is
2
surroundings of temperature 300 K. Take (Single Correct Option, 2015)
Stefan constant σ = 5.7 × 10−8 Wm −2K −4 and
assume that the energy exchange with the 72. An ideal monoatomic gas is confined in a
surroundings is only through radiation. The horizontal cylinder by a spring loaded piston (as
final steady state temperature of the black body shown in the figure). Initially the gas is at
is close to (Single Correct Option, 2014) temperature T1, pressure p1 and volume V1 and
the spring is in its relaxed state. The gas is then
(a) 330 K (b) 660 K (c) 990 K (d) 1550 K
heated very slowly to temperature T2, pressure
Paragraph (Q. Nos. 69-70) p2 and volume V 2. During this process the piston
moves out by a distance x.
In the figure a container is shown to
have a movable (without friction)
piston on top. The container and the
piston are all made of perfectly
insulating material allowing no heat
transfer between outside and inside Ignoring the friction between the piston and the
the container. The container is
cylinder, the correct statements is/are
divided into two compartments by a
(More than One Correct Option, 2015)
rigid partition made of a thermally
conducting material that allows slow transfer of (a) IfV2 = 2V1 and T2 = 3T1, then the energy stored in the
heat. 1
spring is p1V1
4
The lower compartment of the container is filled
(b) IfV2 = 2V1 and T2 = 3T1, then the change in internal
with 2 moles of an ideal monoatomic gas at 700 K
energy is 3 p1V1
and the upper compartment is filled with 2 moles
of an ideal diatomic gas at 400 K. The heat (c) IfV2 = 3V1 and T2 = 4T1, then the work done by the
capacities per mole of an ideal monoatomic gas are 7
gas is p1V1
3 5 3
CV = R, C p = R, and those for an ideal diatomic
2 2 (d) IfV2 = 3V1 and T2 = 4T1, then the heat supplied to the
5 7 17
gas are CV = R, C p = R. gas is p1V1
2 2 (Paragraph Type, 2014) 6

69. Consider the partition to be rigidly fixed so that 73. Two spherical stars A and B emit blackbody
it does not move. When equilibrium is achieved, radiation. The radius of A is 400 times that of B
the final temperature of the gases will be and A emits 104 times the power emitted from
(a) 550 K (b) 525 K (c) 513 K (d) 490 K λ 
B. The ratio  A  of their wavelengths λ A and
λB
70. Now consider the partition to be free to move
without friction so that the pressure of gases in λ B at which the peaks occur in their respective
both compartments is the same. Then total radiation curves is (Integer Type, 2015)
work done by the gases till the time they 74. A gas is enclosed in a cylinder with a movable
achieve equilibrium will be frictionless piston. Its initial thermodynamic
(a) 250R (b) 200R (c) 100R (d) −100R state at pressure Pi = 105 Pa and volume
71. A container of fixed volume has a mixture of one V1 = 10−3 m3 changes to a final state at
mole of hydrogen and one mole of helium in pf = (1/ 32) × 105 Pa and V f = 8 × 10−3 m3 in an
equilibrium at temperature T . Assuming the adiabatic quasi-static process, such that
gases are ideal, the correct statements is/are p3V 5 = constant . Consider another thermodynamic
(a) The average energy per mole of the gas mixture is process that brings the system from the same
2RT initial state to the same final state in two steps
(b) The ratio of speed of sound in the gas mixture to : an isobaric expansion at pi followed by an
6 isochoric (isovolumetric) process at volume V f .
that in helium gas is
5 The amount of heat supplied to the system in
(c) The ratio of the rms speed of helium atoms to that of the two-step process is approximately
1 (Single Correct Option, 2016)
hydrogen molecules is
2 (a) 112 J (b) 294 J (c) 588 J (d) 813 J

THE TIME LINE


The Time Line~JEE Advanced Questions (2014 –2018) 15

75. A water cooler of storage capacity 120 litres can table. Consider only the path from state 1 to state 2.
cool water at a constant rate of P watts. In a W denotes the corresponding work done on the
closed circulation system (as shown system. The equations and plots in the table have
schematically in the figure), the water from the standards notations and used in thermodynamic
processes. Here, γ is the ratio of heat capacities at
cooler is used to cool an external device that
constant pressure and constant volume. The number
generates constantly 3 kW of heat (thermal of moles in the gas is n. (Matching Type, 2017)
load). The temperature of water fed into the
device cannot exceed 30°C and the entire stored Column 1 Column 2 Column 3
120 litres of water is initially cooled to 10°C.
(I) 1 (i) Isothermal (P) p
The entire system is thermally insulated. The W1→ 2 = 1 2
γ −1
minimum value of P (in watts) for which the
( p2 V2 − p1V1 )
device can be operated for 3 hours is
(Single Correct Option, 2016)
V
Cooler Device
Hot p
(II) W1→ 2 (ii) Isochoric (Q)
= − pV2 + pV1 1

2
Cold V
(Specific heat of water is 4.2 kJ kg−1K −1 and the (III) W1→ 2 = 0 (iii) Isobaric (R) p 1
density of water is 1000 kg m−3 ) 2
(a) 1600 (b) 2067 (c) 2533 (d) 3933

76. A metal is heated in a furnace where a sensor V


is kept above the metal surface to read the
(IV) W1→ 2 = − nRT (iv) Adiabatic (S) p
power radiated ( P ) by the metal. The sensor has a 1 2
scale that displays log2 ( P / P0 ), where P0 is a V 
ln  2 
constant. When the metal surface is at a  V1 
temperature of 487°C, the sensor shows a value
1. Assume that the emissivity of the metallic V
surface remains constant. What is the value
displayed by the sensor when the temperature 78. Which of the following options is the only
of the metal surface is raised to 2767°C? correct representation of a process in which
(Integer Type, 2016) ∆U = ∆Q − p∆V ?
(a) (II) (iii) (S) (b) (II) (iii) (P)
77. The ends Q and R of two thin wires, PQ and (c) (III) (iii) (P) (d) (II) (iv) (R)
RS, are soldered (joined) together. Initially,
each of the wire has a length of 1 m 10°C. Now, 79. Which one of the following options is the correct
the end P is maintained at 10°C, while the end combination?
S is heated and maintained at 400°C. The system (a) (II) (iv) (P) (b) (III) (ii) (S)
is thermally insulated from its surroundings. If (c) (II) (iv) (R) (d) (IV) (ii) (S)
the thermal conductivity of wire PQ is twice
80. One mole of a
that of the wire RS and the coefficient of linear p
monoatomic ideal gas
thermal expansion of PQ is 1.2 × 10−5 K −1, the II
undergoes four 3p0 t
change in length of the wire PQ is hermodynamic processes
(Single Correct Option, 2016) I IV III
as shown schematically i
(a) 0.78 mm (b) 0.90 mm (c) 1.56 mm (d) 2.34 mm n the pV-diagram below. p0
Among these four
V
Direction (Q.Nos. 78-79) Matching the processes, one is V0 3V0 i
information given in the three columns of the sobaric, one is isochoric,
following table. one is isothermal and one is adiabatic. Match
An ideal gas is undergoing a cyclic thermodynamic the processes mentioned in Column I with the
process in different ways as shown in the corresponding statements in Column II.
corresponding p-V diagrams in column 3 of the (Matching Type, 2018)

THE TIME LINE


16 The Time Line~JEE Advanced Questions (2014 –2018)

Column I Column II cylinders are K 1 and K 2, respectively. If the


P. In process I 1. Work done by the gas is zero temperature at the junction of the cylinders in
the steady state is 200K, then K 1 / K 2 = .......... .
Q. In process II 2. Temperature of the gas (Integer Type, 2018)
remains unchanged
R. In process III 3. No heat is exchanged Insulating material
between the gas and its
surroundings T1 K1 K2 T2

S. In process IV 4. Work done by the gas is L


6 p0 V0
L
Codes
(a) P → 4; Q → 3; R → 1; S → 2 9. Oscillations
(b) P → 1; Q → 3; R → 2; S → 4
85. Two independent harmonic oscillators of equal
(c) P → 3; Q → 4; R → 1; S → 2
masses are oscillating about the origin with
(d) P → 3; Q → 4; R → 2; S → 1
angular frequencies ω1 and ω 2 and have total
81. One mole of a monoatomic ideal gas undergoes energies E1 and E2, respectively. The variations
an adiabatic expansion in which its volume of their momenta p with positions x are shown
a a
becomes eight times its initial value. If the in the figures. If = n 2 and = n, then the
initial temperature of the gas is 100 K and the b R
universal gas constant R = 8.0 j mol −1 K −1 , the correct equations is/are
decrease in its internal energy in joule, is (More than One Correct Option, 2015)
............ . (Integer Type, 2018) P P
Energy = E1 Energy = E2
82. One mole of a monoatomic ideal gas undergoes
b
a cyclic process as shown in the figure (where, x x
V is the volume and T is the temperature). a R
Which of the statements below is (are) true ?
(More than One Correct Option, 2018)
ω2
T (a) E1ω1 = E 2ω2 (b) = n2
ω1
II E E
(c) ω1ω2 = n 2 (d) 1 = 2
I III ω1 ω2

IV 86. A block with mass M is connected by a massless


V spring with stiffness constant k to a rigid wall and
moves without friction on a horizontal surface.
(a) Process I is an isochoric process
The block oscillates with small amplitude A about
(b) In process II, gas absorbs heat
an equilibrium position x0. Consider two cases : (i)
(c) In process IV, gas releases heat
when the block is at x0 and (ii) when the block is
(d) Processes I and III are not isobaric at x = x0 + A. In both the cases, a particle with
83. Sunlight of intensity 1.3 kWm −2 is incident mass m (< M) is softly placed on the block after
normally on a thin convex lens of focal length which they stick to each other. Which of the
20 cm. Ignore the energy loss of light due to the following statement(s) is (are) true about the
lens and assume that the lens aperture size is motion after the mass m is placed on the mass M ?
much smaller than its focal length. The average (More than One Correct Option, 2016)
intensity of light, in kW m −2 , at a distance 22 (a) The amplitude of oscillation in the first case
cm from the lens on the other side is ............ . M
changes by a factor of , whereas in the
(Integer Type, 2018) m +M
84. Two conducting cylinders of equal length but second case it remains unchanged
different radii are connected in series between (b) The final time period of oscillation in both the cases is
two heat baths kept at temperatures T1 = 300K same
and T2 = 100K, as shown in the figure. The (c) The total energy decreases in both the cases
radius of the bigger cylinder is twice that of the (d) The instantaneous speed at x 0 of the combined
smaller one and the thermal conductivities of masses decreases in both the cases
the materials of the smaller and the larger

THE TIME LINE


The Time Line~JEE Advanced Questions (2014 –2018) 17

87. A spring block system is resting on a 90. Four harmonic waves of equal frequencies and
frictionless floor as shown in the figure. The π 2π
equal intensities I 0 have phase angles 0, ,
spring constant is 2.0 Nm −1 and the mass of the 3 3
block is 2.0 kg . Ignore the mass of the spring. and π. When they are superposed, the intensity
Initially, the spring is in an unstretched of the resulting wave is nI 0. The value of n is
condition. Another block of mass 1.0 kg moving (Integer Type, 2015)
with a speed of 2.0 ms −1 collides elastically with
the first block. The collision is such that the 91. Two loudspeakers M and N are located 20 m
2.0 kg block does not hit the wall. The distance, apart and emit sound at frequencies 118 Hz and
in metres, between the two blocks when the 121 Hz, respectively. A car in initially at a point
spring returns to its unstretched position for P, 1800 m away from the midpoint Q of the line
the first time after the collision is ............ . MN and moves towards Q constantly at 60 km/h
(Integer Type, 2018) along the perpendicular bisector of MN.
It crosses Q and eventually reaches a point R,
1800 m away from Q. Let v( t ) represent the beat
frequency measured by a person sitting in the
2 ms–1 2 kg
car at time t. Let ν P , νQ and ν R be the beat
1kg
frequencies measured at locations P, Q and R
respectively.
10. Wave Motion The speed of sound in air is 330 ms −1. Which of
88. One end of a taut string of length 3 m along the the following statement(s) is (are) true
X-axis is fixed at x = 0. The speed of the waves regarding the sound heard by the person?
in the string is 100 ms −1. The other end of the (More than One Correct Option, 2016)

string is vibrating in the y-direction so that (a) The plot below represents schematically the
stationary waves are set up in the string. The variation of beat frequency with time
possible waveform(s) of these stationary wave ν(t)
is (are) (More than One Correct Option, 2014)
P
πx 50 πt
(a) y (t ) = A sin cos
6 3
νQ Q
πx 100 πt
(b) y (t ) = A sin cos
3 3
5 πx 250 πt R t
(c) y (t ) = A sin cos
2 3
5 πx (b) The rate of change in beat frequency is maximum
(d) y (t ) = A sin cos 250 πt when the car passes through Q
2
(c) νP + νR = 2 νQ
89. A student is performing an experiment using a (d) The plot below represents schematically the
resonance column and a tuning fork of variations of beat frequency with time
frequency 244 s −1. He is told that the air in the ν(t)
tube has been replaced by another gas (assume
that the column remains filled with the gas). If P
the minimum height at which resonance occurs
Q
is (0.350 ± 0.005) m, the gas in the tube is νQ
(Useful information :
167RT = 640 J1/ 2 mole−1/ 2 ; R
t
140RT = 590 J1/ 2 mole−1/ 2. The molar masses
M in grams are given in the options. Take the 92. A block M hangs vertically at the bottom end of
value of 10/ M for each gas as given there.) a uniform rope of constant mass per unit
length. The top end of the rope is attached to a
(More than One Correct Option, 2014)
fixed rigid support at O. A transverse wave
(a) Neon (M = 20, 10 / 20 = 7 / 10) pulse (Pulse 1) of wavelength λ 0 is produced at
(b) Nitrogen (M = 28, 10 / 28 = 3 / 5) point O on the rope. The pulse takes time TOA to
(c) Oxygen (M = 32, 10 / 32 = 9 / 16) reach point A. If the wave pulse of wavelength
(d) Argon (M = 36, 10 / 36 = 17 / 32)

THE TIME LINE


18 The Time Line~JEE Advanced Questions (2014 –2018)

λ 0 is produced at point A (Pulse 2) without (a) The speed of sound determined from this
disturbing the position of M it takes time TAO to experiment is 332 ms −1
reach point O. Which of the following options (b) The end correction in this experiment is 0.9 cm
is/are correct? (More than One Correct Option, 2017) (c) The wavelength of the sound wave is 66.4 cm
(d) The resonance at 50.7 cm corresponds to the
O Pulse 1 fundamental harmonic

96. A parallel plate capacitor has a


dielectric slab of dielectric constant Q1 E1
Pulse 2 K between its plates that covers 1/3
A M of the area of its plates, as shown in E2

(a) The timeTAO = TOA


the figure. The total capacitance of Q2
the capacitor is C while that of the
(b) The wavelength of Pulse 1 becomes longer when it
reaches point A portion with dielectric in between is C1. When
(c) The velocity of any pulse along the rope is the capacitor is charged, the plate area covered
independent of its frequency and wavelength by the dielectric gets charge Q1 and the rest of
(d) The velocities of the two pulses (Pulse 1 and the area gets charge Q2. The electric field in the
Pulse 2) are the same at the mid-point of rope dielectric is E1 and that in the other portion is
E2. Choose the correct option/options, ignoring
93. A stationary source emits sound of frequency edge effects.
f0 = 492 Hz. The sound is reflected by a large (More than One Correct Option, 2014)
car approaching the source with a speed of
2 ms −1. The reflected signal is received by the (a)
E1
=1 (b)
E1
=
1
source and superposed with the original. What E2 E2 K
will be the beat frequency of the resulting signal Q 3 C 2+K
(c) 1 = (d) =
in Hz? (Given that the speed of sound in air is Q2 K C1 K
330 ms −1 and the car reflects the sound at the
frequency it has received). (Integer Type, 2017) 97. Let E1(r ), E2(r ) and E3 (r ) be the respective
electric fields at a distance r from a point
charge Q, an infinitely long wire with constant
11. Electrostatics linear charge density λ, and an infinite plane
94. Two men are walking along a horizontal with uniform surface charge density σ. If
straight line in the same direction. The main in E1(r0 ) = E2(r0 ) = E3 (r0 ) at a given distance r0,
front walks at a speed 1.0ms −1 and the man then (More than One Correct Option, 2014)

behind walks at a speed 2.0 ms −1. A third man is λ


(a) Q = 4 σπr02 (b) r0 =
2 πσ
standing at a height 12 m above the same
horizontal line such that all three men are in a (c) E1 (r0 / 2) = 2 E 2 (r0 / 2) (d) E 2 (r0 / 2) = 4E 3 (r0 / 2)
vertical plane. The two walking men are 98. Charges Q , 2 Q and 4Q are uniformly distributed
blowing identical whistles which emit a sound in three dielectric solid spheres 1, 2 and 3 of
of frequency 1430 Hz. The speed of sound in air radii R / 2, R and 2R respectively, as shown in
330 ms −1. At the instant, when the moving men figure. If magnitudes of the electric fields at point
are 10 m apart, the stationary man is P at a distance R from the centre of spheres 1, 2
equidistant from them. The frequency of beats and 3 are E1 , E2 and E3 respectively, then
in Hz, heard by the stationary man at this (More than One Correct Option, 2014)
instant, is ............. . (Integer Type, 2018)
P P P
95. In an experiment to measure the speed of sound R R R
by a resonating air column, a tuning fork of Q 2Q 4Q
frequency 500 Hz is used. The length of the air R/2
column is varied by changing the level of water 2R
in the resonance tube. Two successive
resonances are heard at air columns of length Sphere-1 Sphere-2 Sphere-3
50.7 cm and 83.9 cm. Which of the following (a) E1 > E 2 > E 3 (b) E 3 > E1 > E 2
statements is (are) true? (c) E 2 > E1 > E 3 (d) E 3 > E 2 > E1
(More than One Correct Option, 2018)

THE TIME LINE


The Time Line~JEE Advanced Questions (2014 –2018) 19

99. Four charges Q1 , Q2 , Q3 and Q4 of same centred at P with distance OP = a = R1 − R2 (see


magnitude are fixed along the x-axis at figure) is made. If the electric field inside the
x = − 2a , − a , + a and +2a respectively. A cavity at position r is E ( r ), then the correct
positive charge q is placed on the positive y-axis statements is/are
at a distance b > 0. Four options of the signs of (More than One Correct Option, 2015)
these charges are given in Column I. The
R2
direction of the forces on the charge q is given
P
in Column II. Match Column I with Column II a
R1
and select the correct answer using the code
given below the lists. (Matching Type, 2014) O

+q (0,b)

(a) E is uniform, its magnitude is independent of R 2 but


its direction depends on r
(b) E is uniform, its magnitude depends on R 2 and its
Q1 Q2 Q3 Q4 direction depends on r
(–2a,0) (–a,0) (+a,0) (+2a,0) (c) E is uniform, its magnitude is independent of ‘a’
but its direction depends on a
Column I Column II (d) E is uniform and both its magnitude and direction
depend on a
P. Q1, Q2 , Q3 , Q4 all positive 1. +x
Q. Q1, Q2 positive; Q3 , Q4 negative 2. −x
102. A parallel plate capacitor having plates of area
S and plate separation d, has capacitance C1 in
R. Q1, Q4 positive; Q2 , Q3 negative 3. +y air. When two dielectrics of d/2
S. Q1, Q3 positive; Q2 , Q4 negative 4. −y different relative
Codes permittivities (ε1 = 2 and

S/ 2
P Q R S P Q R S ε 2 = 4) are introduced ε2
(a) 3, 1, 4, 2 (b) 4, 2, 3, 1 between the two plates as

S/ 2
(c) 3, 1, 2, 4 (d) 4, 2, 1, 3
shown in the figure, the
capacitance becomes C2. ε1
100. An infinitely long uniform line charge C2
The ratio is d
distribution of charge per unit length λ lies C1
3 (More than One Correct Option, 2015)
parallel to the Y -axis in the yz-plane at z = a
2 6 5 7 7
(a) (b) (c) (d)
(see figure). If the magnitude of the flux of the 5 3 5 3
electric field through the rectangular surface
ABCD lying in the xy-plane with its centre at 103. Three identical
λL capacitors C1 , C2 V0
the origin is (ε 0 = permittivity of free space), and C3 have a
nε 0 S2
capacitance of
then the value of n is = 6) (Integer Type, 2015) C3
1.0 µF each and C1
z they are S1 C2
uncharged
initially. They are
L √3 a connected in a
D C 2 circuit as shown in the figure and C1 is then filled
a completely with a dielectric material of relative
y
permittivity ε r . The cell electromotive force (emf)
A
B V 0 = 8V. First the switch S1 is closed while the
x switch S 2 is kept open. When the capacitor C3 is
fully charged, S1 is opened and S 2 is closed
101. Consider a uniform spherical charge distribution simultaneously. When all the capacitors reach
of radius R1 centred at the origin O. In this equilibrium, the charge on C3 is found to be 5µC.
distribution, a spherical cavity of radius R2, The value of ε r = .............. (Integer Type, 2018)

THE TIME LINE


20 The Time Line~JEE Advanced Questions (2014 –2018)

104. A particle of mass 10−3 kg and charge 1.0 C is (a) P → 5; Q → 3,4; R → 1; S → 2


initially at rest. At time t = 0, the particle comes (b) P → 5; Q → 3; R → 1, 4; S → 2
under the influence of an electric field (c) P → 5; Q → 3; R → 1, 2; S → 4
E ( t ) = E0 sin ωt $i, where E0 = 1.0 NC−1 and (d) P → 4; Q → 2, 3; R → 1; S → 5
ω = 103 rad s −1. Consider the effect of only the
electrical force on the particle. Then, the
maximum speed in m s −1 , attained by the particle
12. Current Electricity
at subsequent times is .......... . (Integer Type, 2018) 107. Two ideal batteries of emf V1
and V 2 and three resistances
105. An infinitely long thin λ Z R1 , R2 and R3 are connected as V1 R1
non-conducting wire is parallel to P shown in the figure. The R2
the Z-axis and carries a uniform
R current in resistance R2 would
line charge density λ. It pierces a
be zero if (More than One
120°
thin non-conducting spherical O
Correct Option, 2014) V2
shell of radius R in such a way
(a) V1 = V2 and R1 = R 2 = R 3 R3
that the arc PQ subtends an
angle 120° at the centre O of the Q (b) V1 = V2 and R1 = 2R 2 = R 3
spherical shell, as shown in the (c) V1 = 2V2 and 2R1 = 2R 2 = R 3
figure. The permittivity of free (d) 2V1 = V2 and 2R1 = R 2 = R 3
space is ε 0. Which of the following statements is
(are) true? (More than One Correct Option, 2018) 108. During an experiment with a metre bridge, the
galvanometer shows a null point when the
(a) The electric flux through the shell is 3 Rλ / ε0
jockey is pressed at 40.0 cm using a standard
(b) The z-component of the electric field is zero at all
resistance of 90 Ω, as shown in the figure. The
the points on the surface of the shell
least count of the scale used in the metre bridge
(c) The electric flux through the shell is 2 Rλ / ε0
is 1 mm. The unknown resistance is
(d) The electric field is normal to the surface of the shell (Single Correct Option, 2014)
at all points

106. The electric field E is measured at a point R 90 Ω


P ( 0, 0, d ) generated due to various charge
distributions and the dependence of E on d is
found to be different for different charge
distributions. Column I contains different 40.0cm
relations between E and d. Column II describes (a) 60 ± 0.15 Ω (b) 135 ± 0.56 Ω
different electric charge distributions, along (c) 60 ± 0.25 Ω (d) 135 ± 0.23 Ω
with their locations. Match the functions in
Column I with the related charge distributions 109. In an aluminium (Al) bar of square cross
in Column II. (Matching Type, 2018) section, a square hole is drilled and is filled
with iron (Fe) as shown in the figure. The
Column I Column II electrical resistivities of Al and Fe are
P. E is independent 1. A point charge Q at the origin 2.7 × 10−8 Ωm and 1.0 × 10−7 Ωm, respectively.
of d The electrical resistance between the two faces
Q. E ∝ 1 2. A small dipole with point charges Q P and Q of the composite bar is
d at (0, 0, l ) and − Q at (0, 0, − 1). (More than One Correct Option, 2015)
(Take, 2l << d )
R. E ∝ 1 3. An infinite line charge coincident
d2 with the X-axis, with uniform linear
charge density λ.
S. E ∝ 1 4. Two infinite wires carrying a uniform
d3 linear charge density parallel to the Al
X- axis. The one along ( y = 0, z = l )
m

has a charge density + λ and the


m

Fe
50

one along ( y = 0, z = − l ) has a 2 mm


charge density – λ. (Take, 2l << d ).
7 mm
5. Infinite plane charge coincident with
the xy-plane with uniform surface 2475 1875 1875 2475
(a) µ Ω (b) µ Ω (c) µΩ (d) µΩ
charge density. 64 64 49 132

THE TIME LINE


The Time Line~JEE Advanced Questions (2014 –2018) 21

110. In the following circuit, the current through the (a) The voltmeter display −5 V as soon as the key is
resistor R( = 2 Ω ) is I amperes. The value of I is pressed and displays + 5 V after a long time
(Integer Type, 2015) (b) The voltmeter will display 0 V at time t = ln 2
R(=2Ω) 1Ω seconds
(c) The current in the ammeter becomes 1/e of the
2Ω 8Ω initial value after 1 second
6Ω 2Ω (d) The current in the ammeter becomes zero after a long
4Ω time
6.5V 10Ω
Paragraph (Q. Nos. 113-114)
12Ω 4Ω Consider an evacuated cylindrical chamber of
height h having rigid conducting plates at the
ends and an insulating curved surface as shown in
111. An infinite line charge of uniform electric the figure. A number of spherical balls made of a
light weight and soft material and coated with a
charge density λ lies along the axis of an
conducting material are placed on the bottom
electrically conducting infinite cylindrical shell
plate. The balls have a radius r << h. Now, a high
of radius R. At time t = 0, the space inside the voltage source (HV) connected across the
cylinder is filled with a material of permittivity conducting plates such that the bottom plate is at
ε and electrical conductivity σ. The electrical +V 0 and the top plate at −V 0. Due to their
conduction in the material follows Ohm's law. conducting surface, the balls will get charge, will
Which one of the following graphs best become equipotential with the plate and are
describes the subsequent variation of the repelled by it.
magnitude of current density j (t) at any The balls will eventually collide with the top
point in the material? (Single Correct Option, 2016) plate, where the coefficient of restitution can be
taken to be zero due to te soft nature of the material
j(t) j(t) of the balls. The electric field in the chamber can
be considered to be that of a parallel plate capacitor.
Assume that there are no collisions between the
(a) (b) balls and the interaction between them is
negligible. (Ignore gravity) (Paragraph Type, 2016)
A
t t
(0, 0) (0, 0)


j(t) j(t)
HV
+

(a) (d)

t t 113. Which one of the following statements is


(0, 0) (0, 0) correct?
(a) The balls will execute simple harmonic motion
112. In the circuit shown below, the key is pressed at between the two plates
time t = 0. Which of the following statement(s) is (b) The balls will bounce back to the bottom plate
(are) true? (More than One Correct Option, 2016) carrying the same charge they went up with
(c) The balls will stick to the top plate and remain there
µF
25

40 (d) The balls will bounce back to the bottom plate


kΩ

− carrying the opposite charge they went up with


V
+ 114. The average current in the steady state
µF
50

registered by the ammeter in the circuit will be


kΩ

20

A
(a) proportional toV02
+ – (b) proportional to the potentialV0
Key (c) zero
5V
(d) proportions toV01/ 2

THE TIME LINE


22 The Time Line~JEE Advanced Questions (2014 –2018)

115. An incandescent bulb has a thin filament of 117. In Process 2, total energy dissipated across the
tungsten that is heated to high temperature by resistance ED is
1 1 2
(b) ED = 3  CV02 
passing an electric current. The hot filament 1
(a) ED =  CV0 
emits black-body radiation. The filament is 32  2 
observed to break up at random locations after 1
(c) ED = 3 CV02 (d) ED = CV02
a sufficiently long time of operation due to 2
non-uniform evaporation of tungsten from the
filament. If the bulb is powered at constant 118. In the figure below, the switches S1 and S 2 are
voltage, which of the following statement(s) is closed simultaneously at t = 0 and a current
(are) true? (More than One Correct Option, 2016)
starts to flow in the circuit. Both the batteries
have the same magnitude of the electromotive
(a) The temperature distribution over the filament is
force (emf) and the polarities are as indicated in
uniform
the figure. Ignore mutual inductance between
(b) The resistance over small sections of the filament
the inductors. The current I in the middle wire
decreases with time
reaches its maximum magnitude I max at time
(c) The filament emits more light at higher band of
requencies before it breaks up
t = τ. Which of the following statements is (are)
(d) The filament consumes less electrical power
true ? (More than One Correct Option, 2018)

towards the end of the life of the bulb R L R 2L

Paragraph (Q. Nos. 116-117) V I V


Consider a simple RC circuit as shown in Figure 1.
Process 1 In the circuit the switch S is closed at
t = 0 and the capacitor is fully charged to voltage S1 S2
V 0 (i.e. charging continues for time T >> RC). In V V
(a) Imax = (b) Imax =
the process some dissipation (ED ) occurs across 2R 4R
the resistance R. The amount of energy finally L 2L
(c) τ = ln 2 (d) τ = ln 2
stored in the fully charged capacitor is E c. R R
Process 2 In a different process the voltage is
V0 119. In the xy-plane, the
Y
first set to and maintained for a charging time region y > 0 has a
3
2V 0 uniform magnetic
T >> RC. Then, the voltage is raised to without field B1k$ and the B1
3
discharging the capacitor and again maintained region y < 0 has V0=π ms –1

for a time T >> RC. The process is repeated one another uniform
magnetic field B2k.$ A X
more time by raising the voltage to V 0 and the B2
capacitor is charged to the same final voltage V 0 positively charged
as in Process 1. These two processes are depicted particle is projected from the origin along the
in Figure 2. (Paragraph Type, 2017) positive Y-axis with speed v0 = π ms −1 at t = 0,
V as shown in figure. Neglect gravity in this
Process 1 problem. Let t = T be the time when the particle
V0
S crosses the X-axis from below for the first time.
2V0/3
R Process 2 If B2 = 4B1, the average speed of the particle, in
V + C
V0/3 T >> RC ms −1, along the X-axis in the time interval T

t is............ . (Integer Type, 2018)
T 2T
Figure 1 Figure 2 13. Magnetics
116. In Process 1, the energy stored in the capacitor Paragraph (Q. Nos. 120-121)
EC and heat dissipated across resistance ED are
A point charge Q is moving in a circular orbit of
related by radius R in the x-y plane with an angular velocity
(a) EC = ED ln2 (b) EC = ED ω. This can be considered as equivalent to a loop
1 Qω
(c) EC = 2ED (d) EC = ED carrying a steady current ⋅ A uniform magnetic
2 2π
field along the positive z-axis is now switched on,

THE TIME LINE


The Time Line~JEE Advanced Questions (2014 –2018) 23

which increases at a constant rate from 0 to B in


one second. Assume that the radius of the orbit Q S
remains constant. The applications of the d d
Wire 1 Wire 2
magnetic field induces an emf in the orbit.
The induced emf is defined as the work done by a
an induced electric field in moving a unit positive
charge around a closed loop. It is known that, for
P R
an orbiting charge, the magnetic dipole moment is
proportional to the angular momentum with a
proportionality constant γ. (Paragraph Type, 2013) 124. When d ≈ a but wires are not touching the loop,
it is found that the net magnetic field on the
120. The change in the magnetic dipole moment
axis of the loop is zero at a height h above the
associated with the orbit, at the end of the time
loop. In that case
interval of the magnetic field change, is
(a) current in wire 1 and wire 2 is the direction PQ and
BQR 2 BQR 2
(a) γBQR 2 (b) −γ (c) γ (d) γBQR 2 RS, respectively and h ≈ a
2 2
(b) current in wire 1 and wire 2 is the direction PQ and
121. The magnitude of the induced electric field in SR, respectively and h ≈ a
the orbit at any instant of time during the time (c) current in wire 1 and wire 2 is the direction PQ and
interval of the magnetic field change is SR, respectively and h ≈ 1.2a
BR − BR (d) current in wire 1 and wire 2 is the direction PQ and
(a) (b) (c) BR (d) 2BR
4 2 RS, respectively and h ≈ 1.2a

122. Two parallel wires in the plane of the paper are 125. Consider d >> a, and the loop is rotated about
distance X 0 apart. A point charge is moving its diameter parallel to the wires by 30° from
with speed u between the wires in the same the position shown in the figure. If the
plane at a distance X1 from one of the wires. currents in the wires are in the opposite
When the wires carry current of magnitude I in directions, the torque on the loop at its
the same direction, the radius of curvature of new position will be (assume that the net
the path of the point charge is R1. In contrast, field due to the wires is constant over the loop)
if the currents I in the two wires have µ 0I 2a 2 µ 0I 2a 2
(a) (b)
directions opposite to each other, the radius d 2d
X
of curvature of the path is R2. If 0 = 3, and (c)
3 µ 0I 2a 2
(d)
3 µ 0I 2a 2
X1 d 2d
R1
value of is
R2 (Integer Type, 2014) 126. The figures below depict two situations in
which two infinitely long static line charges of
123. A galvanometer gives full scale deflection with constant positive line charge density λ are kept
0.006 A current. By connecting it to a 4990 Ω parallel to each other.
resistance, it can be converted into a voltmeter
2n
of range 0-30 V. If connected to a Ω
249 λ λ λ λ
resistance, it becomes an ammeter of range
0-1.5 A. The value of n is (Integer Type, 2014)
x x
+q –q

Paragraph (Q. Nos. 124-125)


The figure shows a circular loop of radius a with
In their resulting electric field, point charges q
two long parallel wires (numbered 1 and 2) all in
the plane of the paper. The distance of each wire and −q are kept in equilibrium between them.
from the centre of the loop is d. The loop and the The point charges are confined to move in the x
wires are carrying the same current I. The direction only. If they are given a small
current in the loop is in the counter-clockwise displacement about their equilibrium
direction if seen from above. (Paragraph Type, 2014) positions, then the correct statements is/are
(More than One Correct Option, 2015)

THE TIME LINE


24 The Time Line~JEE Advanced Questions (2014 –2018)

(a) both charges execute simple harmonic motion differences between K and M in strips 1
(b) both charges will continue moving in the direction of and 2, respectively. Then, for a given current
their displacement I flowing through them in a given magnetic
(c) charge + q executes simple harmonic motion while field strength B, the correct statements is/are
charge − q continues moving in the direction of its (a) If w1 = w 2 and d1 = 2d2 , thenV2 = 2V1
displacement (b) If w1 = w 2 and d1 = 2d2 , thenV2 = V1
(d) charge − q executes simple harmonic motion while (c) If w1 = 2w 2 and d1 = d2 , thenV2 = 2V1
charge + q continues moving in the direction of its (d) If w1 = 2w 2 and d1 = d2 , thenV2 = V1
displacement
129. Consider two different metallic strips (1 and 2)
127. A conductor (shown in the figure) carrying of same dimensions (length l, width w and
constant current I is kept in the x-y plane in a thickness d) with carrier densities n1 and n 2,
uniform magnetic field B. If F is the magnitude respectively. Strip 1 is placed in magnetic field
of the total magnetic force acting on the B1 and strip 2 is placed in magnetic field B2,
conductor, then the correct statements is/are both along positive y-directions. Then V1 and V 2
(More than One Correct Option, 2015)
are the potential differences developed between
y K and M in strips 1 and 2, respectively.
R R
I π/6 π/4 Assuming that the current I is the same for
x
L R R L both the strips, the correct options is/are
$ F ∝ (L + R) (a) If B1 = B 2 and n1 = 2n2 , thenV2 = 2V1
(a) if B is along z,
$ F =0 (b) If B1 = B 2 and n1 = 2n2 , thenV2 = V1
(b) if B is along x,
(c) If B1 = 2B 2 and n1 = n2 , thenV2 = 05
. V1
(c) if B is along y, F ∝ (L + R)
$
(d) If B1 = 2B 2 and n1 = n2 , thenV2 = V1
$ F =0
(d) if B is along z,
130. A conducting loop in the shape of a right angled
Paragraph (Q. Nos. 128-129) isosceles triangle of height 10 cm is kept such
that the 90° vertex is very close to an infinitely
In a thin rectangular metallic strip a constant long conducting wire (see the figure). The wire
current I flows along the positive x-direction, as
shown in the figure. The length, width and is electrically insulated from the loop. The
thickness of the strip are l, w and d, hypotenuse of the triangle is parallel to the wire.
respectively. A uniform magnetic field B is applied The current in the triangular loop is in counter-
on the strip along the positive y-direction. Due to clockwise direction and increased at a constant
this, the charge carriers experience a net rate of 10 As −1. Which of the following
deflection along the z-direction.
This results in accumulation of charge carriers on
statement(s) is (are) true?
(More than One Correct Option, 2016)
the surface PQRS and appearance of equal and
opposite charges on the face opposite to PQRS. A
potential difference along the z-direction is thus 90°
10 cm
developed. Charge accumulation continues until
the magnetic force is balanced by the electric
force. The current is assumed to be uniformly
(a) There is a repulsive force between the wire and the
distributed on the cross section of the strip and
carried by electrons. (Paragraph Type, 2015)
loop
(b) If the loop is rotated at a constant angular speed
l y about the wire, an additional emf of (µ 0 / π) volt is
I w K induced in the wire
I x
µ
(c) The magnitude of induced emf in the wire is  0  volt
S R
d M z  π
P Q
(d) The induced current in the wire is in opposite
direction to the current along the hypotenuse
128. Consider two different metallic strips (1 and 2)
of the same material. Their lengths are the 131. Consider two identical galvanometers and two
same, widths are w1 and w2 and thicknesses are identical resistors with resistance R. If the
d1 and d2, respectively. internal resistance of the galvanometers
Two points K and M are symmetrically Rc < R / 2, which of the following statement(s)
located on the opposite faces parallel to the x-y about anyone of the galvanometers is (are)
plane (see figure). V1 and V 2 are the potential true? (More than One Correct Option, 2016)

THE TIME LINE


The Time Line~JEE Advanced Questions (2014 –2018) 25

(a) The maximum voltage range is obtained when all The magnitude of the magnetic field at the
the components are connected in series center of the loop is (Single Correct Option, 2017)
(b) The maximum voltage range is obtained when the
two resistors and one galvanometer are connected
I
in series, and the second galvanometer is
connected in parallel to the first galvanometer
4a
(c) The maximum current range is obtained when all the
components are connected in parallel
(d) The maximum current range is obtained when the
two galvanometers are connected in series, and the
µ 0I µ 0I
combination is connected in parallel with both the (a) 6[ 3 − 1] (b) 6[ 3 + 1]
resistors 4 πa 4 πa
µ I µ I
(c) 0 3 [ 3 − 1] (d) 0 3 [2 − 3 ]
Direction (Q.Nos. 132 to 134) Matching the 4 πa 4 πa
information given in the three columns of the 136. A uniform magnetic field B exists in the region
following table. 3R
between x = 0 and x = (region 2 in the
A charged particle (electron or proton) is 2
introduced at the origin (x = 0, y = 0, z = 0) with a figure) pointing normally into the plane of the
given initial velocity v. A uniform electric field E paper. A particle with charge +Q and
and a uniform magnetic field B exist everywhere.
momentum p directed along X-axis enters
The velocity v, electric field E and magnetic field B
are given in columns 1, 2 and 3, respectively. The region 2 from region 1 at point P1 ( y = − R ).
quantities E 0 , B0 are positive in magnitude. Which of the following option(s) is/are correct?
(Matching Type, 2017)
(More than One Correct Option, 2017)
y
Column 1 Column 2 Column 3 Region 1 Region 2 Region 3
(I) Electron with v = 2 E0 $x (i) E = E0 $z (P) B = − B0 $x
B0 B

(II) Election with v = E0 $y (ii) E = − E0 $y (Q) B = B0 $x


B0
O x
P2
(III) Proton with v = 0 (iii) E = − E0 $x (R) B = B0 $y +Q P1
(IV) Proton with v = 2 E0 $x (iv) E = E0 $x (S) B = B0 $z (y=–R)
B0
(3R/2)
132. In which case would the particle move in a straight (a) When the particle re-enters region 1 through the
line along the negative direction of Y -axis? longest possible path in region 2, the magnitude of
(a) (IV) (ii) (S) (b) (II) (iii) (Q) the change in its linear momentum between point P1
p
(c) (III), (ii) (R) (d) (III) (ii) (P) and the farthest point fromY -axis is
2
133. In which case will the particle move in a (b) For B =
8 p
, the particle will enter region 3
straight line with constant velocity? 13 QR
(a) (II) (iii) (S) (b) (III) (iii) (P) through the point P2 on X-axis
2 p
(c) (IV) (i) (S) (d) (III) (ii) (R) (c) For B > , the particle will re-enter region 1
3 QR
134. In which case will the particle describe a helical (d) For a fixed B, particles of same charge Q and same
path with axis along the positive z-direction? velocity v, the distance between the point P1 and the
(a) (II) (ii) (R) (b) (III) (iii) (P) point of re-entry into region 1 is inversely
(c) (IV) (i) (S) (d) (IV) (ii) (R) proportional to the mass of the particle

135. A symmetric star shaped conducting wire loop 137. Two infinitely long straight wires lie in the
is carrying a steady state current I as shown xy-plane along the lines x = ± R. The wire
in the figure. The distance between the located at x = + R carries a constant current
diametrically opposite vertices of the star is 4a. I1and the wire located at x = − R carries a
constant current I 2. A circular loop of radius

THE TIME LINE


26 The Time Line~JEE Advanced Questions (2014 –2018)

R is suspended with its centre at ( 0, 0, 3R ) and (a) Magnitude of the maximum charge on the capacitor

in a plane parallel to the xy-plane. This loop before t = is 1 × 10−3 C

carries a constant current I in the clockwise
direction as seen from above the loop. The (b) The current in the left part of the circuit just before

current in the wire is taken to be positive, if it t = is clockwise

is in the + $j-direction. Which of the following
statements regarding the magnetic field B is (c) Immediately after A is connected to D, the current
in R is 10 A
(are) true? (More than One Correct Option, 2018)
(d) Q = 2 × 10−3 C
(a) If I1 = I 2 , then B cannot be equal to zero at the origin
(0,0,0) 140. Two inductors L1 (inductance 1mH, internal
(b) If I1 > 0 and I 2 < 0, then B can be equal to zero at the resistance 3 Ω) and L2 (inductance 2 mH,
origin (0,0,0) internal resistance 4 Ω), and a resistor R
(c) If I1 < 0 and I 2 > 0, then B can be equal to zero at the (resistance 12 Ω) are all connected in parallel
origin (0,0,0) across a 5V battery. The circuit is switched on
(d) If I1 = I 2 , then the z-component of the magnetic field at time t = 0. The ratio of the maximum to the
µ I
at the centre of the loop is  − 0  minimum current ( I max / I min ) drawn from the
 2R 
battery is (Integer Type, 2016)

138. A moving coil galvanometer has 50 turns and 141. A rigid wire loop of square shape having side of
each turn has an area 2 × 10−4 m 2 . The magnetic length L and resistance R is moving along the
field produced by the magnet inside the x-axis with a constant velocity v0 in the plane of
galvanometer is 0.02 T. The torsional constant the paper . At t = 0, the right edge of the loop
of the suspension wire is 10 −4 N - m rad −1 . When
enters a region of length 3L where there is a
a current flows through the galvanometer, a
uniform magnetic field B0 into the plane of the
full scale deflection occurs, if the coil rotates by
0.2 rad. The resistance of the coil of the paper, as shown in the figure. For sufficiently
galvanometer is 50 Ω. This galvanometer is to large v0, the loop eventually crosses the region.
be converted into an ammeter capable of Let x be the location of the right edge of the
measuring current in the range 0 − 1.0 A. For loop. Let v ( x ), I ( x ) and F ( x ) represent the
this purpose, a shunt resistance is to be added velocity of the loop, current in the loop, and
in parallel to the galvanometer. The value of force on the loop, respectively, as a function of
this shunt resistance in ohms, is ............. . x. Counter- clockwise current is taken as
(Integer Type, 2018) positive. (More than One Correct Option, 2016)

14. Electromagnetic Induction


and Alternating Current
R
139. At time t = 0, B D L
terminal A in the
circuit shown in the A v0
figure is connected 50V
C=20µF x
to B by a key and an
0 L 2L 3L 4L
alternating current
I ( t ) = I 0 cos (ωt ), with R = 10 Ω Which of the following schematic plot(s) is (are)
I 0 = 1 A and ω = 500 rad s −1 starts flowing in it correct? (Ignore gravity)
with the initial direction shown in the figure. F(x) I(x)

At t = , the key is switched from B to D. Now
6ω (a) (b)
onwards only A and D are connected. A total x
0 L 2L 3L 4L 3L 4L
charge Q flows from the battery to charge the x
capacitor fully. If C = 20 µF, R = 10 Ω and the 0 L 2L
battery is ideal with emf of 50 V, identify the
correct statement(s).
(More than One Correct Option, 2014)

THE TIME LINE


The Time Line~JEE Advanced Questions (2014 –2018) 27
v(x) I(x) switch is closed and current begins to flow.
v0 Which of the following options is/are correct?
(More than One Correct Option, 2017)
S
(c) (d) R
x x
0 L 2L 3L 4L 0 L 2L 3L 4L + V L1 L2

142. In the circuit shown, L = 1 µH, C = 1 µF and


R = 1 kΩ . They are connected in series with an (a) After a long time, the current through L1 will be
AC source V = V 0 sinωt as shown. Which of the V  L2 
 
following options is/are correct? R  L1 + L2 
(More than One Correct Option, 2017) (b) After a long time, the current through L2 will be
L=1µH C=1µF R=1kΩ V  L1 
 
R  L1 + L2 
V0 sin ωt (c) The ratio of the currents through L1 and L2 is fixed at
all times (t > 0)
(a) At ω ∼ 0 the current flowing through the circuit V
(d) At t = 0, the current through the resistance R is
becomes nearly zero R
(b) The frequency at which the current will be in phase
with the voltage is independent of R
145. The instantaneous voltages at three terminals
marked X , Y and Z are given by V X = V 0 sinωt,
(c) The current will be in phase with the voltage if ω = 104
 2π   4π 
rads−1 VY = V 0 sinωt +  and V Z = V 0 sinωt + .
(d) Atω >> 106 rads −1, the circuit behaves like a capacitor  3  3
An ideal voltmeter is configured to read rms
143. A circular insulated copper wire value of the potential difference between its
loop is twisted to form two loops of B
terminals. It is connected between points X and
area A and 2A as shown in the area A Y and then between Y and Z. The reading(s) of
figure. At the point of crossing the the voltmeter will be
wires remain electrically insulated (More than One Correct Option, 2017)
from each other. The entire loop
1
lies in the plane area 2A YZ = V0
(a) V rms
2
(of the paper). A uniform magnetic
3
field B points into the plane of the XY = V0
(b) V rms
paper. At t = 0, the loop starts w 2
rotating about the common (c) independent of the choice of the two terminals
diameter as axis with a constant angular XY = V0
(d) V rms
velocity ω in the magnetic field. Which of
the following options is/are correct?
(More than One Correct Option, 2017)
15. Ray and Wave Optics
(a) The emf induced in the loop is proportional to the 146. A light source, which emits two wavelengths
sum of the areas of the two loops λ 1 = 400 nm and λ 2 = 600 nm, is used in a
(b) The rate of change of the flux is maximum when the Young’s double-slit experiment. If recorded
plane of the loops is perpendicular to plane of the fringe widths for λ 1 and λ 2 are β1 and β 2 and
paper
the number of fringes for them within a
(c) The net emf induced due to both the loops is
proportional to cos ωt
distance y on one side of the central maximum
(d) The amplitude of the maximum net emf induced are m1 and m2, respectively, then
(More than One Correct Option, 2014)
due to both the loops is equal to the amplitude of
maximum emf induced in the smaller loop alone (a) β 2 > β1
(b) m1 > m2
144. A source of constant voltage V is connected to a
resistance R and two ideal inductors L1 and L2 (c) from the central maximum, 3rd maximum of λ 2
through a switch S as shown. There is no overlaps with 5th minimum of λ1
mutual inductance between the two inductors. (d) the angular separation of fringes of λ1 is greater
The switch S is initially open. At t = 0, the than λ 2

THE TIME LINE


28 The Time Line~JEE Advanced Questions (2014 –2018)

147. A transparent thin film of n1 S1


uniform thickness and
d
refractive index n1 = 1.4 is x Air
Air n2
coated on the convex spherical d
S2 Water
surface of radius R at one end of
a long solid glass cylinder of
refractive index n 2 = 1.5, as shown in the figure.
150. Consider a concave mirror and a convex lens
Rays of light parallel to the axis of the cylinder (refractive index = 1.5) of focal length 10 cm
traversing through the film from air to glass get each, separated by a distance of 50 cm in air
focused at distance f1 from the film, while rays (refractive index = 1) as shown in the figure. An
of light traversing from glass to air get focused object is placed at a distance of 15 cm from the
at distance f2 from the film. Then mirror. Its erect image formed by this
(More than One Correct Option, 2014)
combination has magnification M1.When the
(a) | f1 | = 3R (b) | f1 | = 2.8R 7
set-up is kept in a medium of refractive index ,
(c) | f2 | = 2R (d) | f2 | = 1.4 R 6
the magnification becomes M 2. The magnitude
148. Four combinations of two thin lenses are given

M 2 is
in Column I.The radius of curvature of all
curved surfaces is r and the refractive index of M1 (Integer Type, 2015)
all the lenses is 1.5. Match lens combinations in
Column I with their focal length in Column II
and select the correct answer using the code 15 cm
given below the Columns.
50 cm
Column I Column II 151. Two identical glass rods S1 and S 2
(refractive index = 1.5) have one convex end of
P. 1. 2r radius of curvature 10 cm. They are placed with
the curved surfaces at a distance d as shown in
the figure, with their axes (shown by the
dashed line) aligned. When a point source of
Q. 2. r/2
light P is placed inside rod S1 on its axis at a
distance of 50 cm from the curved face, the light
rays emanating from it are found to be parallel
R. 3. −r to the axis inside S 2. The distance d is
(More than One Correct Option, 2015)

S1 P S2
S. 4. r 50 cm d
(a) 60 cm (b) 70 cm
(Matching Type, 2014) (c) 80 cm (d) 90 cm
Codes 152. A monochromatic beam of light is incident at
P Q R S P Q R S
60° on one face of an equilateral prism of
(a) 1 2 3 4 (b) 2 4 3 1
(c) 4 1 2 3 (d) 2 1 3 4 refractive index n and emerges from the
opposite face making an angle θ with the
149. A Young’s double slit interference arrangement normal (see figure). For n = 3 the value of θ is
with slits S1 and S 2 is immersed in water dθ
(refractive index = 4/ 3) as shown in the figure. 60° and = m. The value of m is
dn (Integer Type, 2015)
The positions of maxima on the surface of
water are given by x 2 = p2m 2λ 2 − d 2, where λ is
the wavelength of light in air (refractive index
= 1 ), 2d is the separation between the slits and 60° θ
m is an integer. The value of p is
(Integer Type, 2015)

THE TIME LINE


The Time Line~JEE Advanced Questions (2014 –2018) 29
P
Paragraph (Q. Nos. 153-154)
θ
Light guidance in an optical fibre can be
understood by considering a structure comprising α
of thin solid glass cylinder of refractive index n1
surrounded by a medium of lower refractive index
n2. The light guidance in the structure takes place
due to successive total internal reflections at the Q n=√2 R
interface of the media n1 and n2 as shown in the (a) 15° (b) 22.5° (c) 30° (d) 45°
figure. All rays with the angle of incidence i less
than a particular value im are confined in the 156. A plano-convex lens is made of material of
medium of refractive index n1. The numerical refractive index n. When a small object is placed
aperture (NA) of the structure is defined as sin im. 30 cm away in front of the curved surface of the
(Paragraph Type, 2015) lens, an image of double the size of the object is
153. 45 produced. Due to reflection from the convex
For two structures namely S1 with n1 = surface of the lens, another faint image is
4
observed at a distance of 10 cm away from the
3 8 7
and n 2 = , and S 2 with n1 = and n 2 = and lens. Which of the following statement(s) is (are)
2 5 5 true? (More then One Correct Option, 2016)
4
taking the refractive index of water to be and (a) The refractive index of the lens is 2.5
3
that to air to be 1, the correct options is/are (b) The radius of curvature of the convex surface is 45 cm
(c) The faint image is erect and real
n1>n2
Air θ (d) The focal length of the lens is 20 cm
Cladding n2
Core 157. A transparent slab of thickness d has a
i n1
refractive index n ( z ) that increases with z.
Here, z is the vertical distance inside the slab,
(a) NA of S1 immersed in water is the same as that of S2 measured from the top. The slab is placed
16 between two media with uniform refractive
immersed in a liquid of refractive index
3 15 indices n1 and n 2(> n1 ), as shown in the figure.
(b) NA of S1 immersed in liquid of refractive index
6 A ray of light is incident with angle θ i from
15 medium 1 and emerges in medium 2 with
is the same as that of S2 immersed in water refraction angle θ f with a lateral displacement l.
(c) NA of S1 placed in air is the same as that S2
4 θi
immersed in liquid of refractive index
15 n1=constant 1
(d) NA of S1 placed in air is the same as that of S2 n (z)
z
placed in water
d
154. If two structures of same cross-sectional area,
but different numerical apertures NA1 and
n2=constant 2
NA2( NA2 < NA1 ) are joined longitudinally, l θf
the numerical aperture of the combined
structure is Which of the following statement(s) is (are)
NA1NA2 true?
(a) (b) NA1 + NA2 (More then One Correct Option, 2016)
NA1 + NA2
(a) l is independent on n (z) (b) n1 sinθi = (n2 − n1) sinθf
(c) NA1 (d) NA2 (c) n1 sinθi = n2 sinθf (d) l is independent of n2
155. A parallel beam of light is incident from air at 158. A small object is placed 50 cm to the left of a
an angle α on the side PQ of a right angled thin convex lens of focal length 30 cm. A convex
triangular prism of refractive index n = 2. spherical mirror of radius of curvature 100 cm
Light undergoes total internal reflection in the is placed to the right of the lens at a distance of
prism at the face PR when α has a minimum 50 cm. The mirror is tilted such that the axis of
value of 45°. The angle θ of the prism is the mirror is at an angle θ = 30° to the axis of
(Single Correct Option, 2016) the lens, as shown in the figure.
(Single Correct Option, 2016)

THE TIME LINE


30 The Time Line~JEE Advanced Questions (2014 –2018)

f=30 cm (c) For this prism, the emergent ray at the second
surface will be tangential to the surface when the
angle of incidence at the first surface is
θ  A 
X
(– 50, 0) (0, 0) i1 = sin−1  sin A 4 cos2 − 1 − cos A
R=100 cm  2 
(d) For this prism, the refractive index µ and the angle
µ
prism A are related as A = cos−1  
1
50 cm (50 + 50√3, –50)  2
2
If the origin of the coordinate system is taken to
161. A monochromatic light is travelling in a
be at the centre of the lens, the coordinates (in
medium of refractive index n = 1 .6. It enters a
cm) of the point ( x , y ) at which the image is
stack of glass layers from the bottom side at an
formed are
angle θ = 30°. The interfaces of the glass layers
(a) (125 / 3, 25 / 3 ) (b) (50 − 25 3 , 25)
are parallel to each other.
(c) (0, 0) (d) (25, 25 3 )
The refractive indices of different glass layers
159. While conducting the Young’s double slit are monotonically decreasing as n m = n − m ∆n,
experiment, a student replaced the two slits where n m is the refractive index of the mth slab
with a large opaque plate in the xy-plane and ∆n = 0 . 1 (see the figure). The ray is
containing two small holes that act as two refracted out parallel to the interface
coherent point sources (S1 , S 2) emitting light of between the ( m − 1) th and mth slabs from
wavelength 600 mm. The student mistakenly
the right side of the stack. What is the value
placed the screen parallel to the xz-plane (for
of m ? (Integer Type, 2017)
z > 0) at a distance D = 3 m from the mid-point
of S1S 2, as shown schematically in the figure. m n–m∆n
m–1 n–(m–1)∆n
The distance between the source d = 0.6003 mm .
The origin O is at the intersection of the screen
and the line joining S1 S 2.
(More than One Correct Option, 2016) 3 n–3 ∆n
2 n–2 ∆n
1 n– ∆n
Screen

n
Z θ

O 162. Two coherent


y
1 P
S1 S2 x monochromatic point sources
d ∆θ
D S1 and S 2 of wavelength
λ = 600 nm are placed
Which of the following is (are) true of the
symmetrically on either side P2
intensity pattern on the screen? S1 S2
of the centre of the circle as d
(a) Semi circular bright and dark bands centered at
shown. The sources are
point O
separated by a distance
(b) The region very close to the point O will be dark
d = 1.8 mm. This
(c) Straight bright and dark bands parallel to the x-axis
arrangement produces interference fringes
(d) Hyperbolic bright and dark bands with foci
visible as alternate bright and dark spots on
symmetrically placed about O in the x-direction
the circumference of the circle. The angular
160. For an isosceles prism of angle A and refractive separation between two consecutive bright
index µ , it is found that the angle of minimum spots is ∆θ. Which of the following options
deviation δ m = A. Which of the following options is/are correct? (More than One Correct Option, 2017)
is/are correct? (More than One Correct Option, 2017) (a) The angular separation between two consecutive
(a) For the angle of incidence i1 = A, the ray inside the bright spots decreases as we move from P1 to P2
prism is parallel to the base of the prism along the first quadrant
(b) At minimum deviation, the incident angle i1 and the (b) A dark spot will be formed at the point P2
refracting angle r1 at the first refracting surface are (c) The total number of fringes produced between P1
related by r1 =  i 
i and P2 in the first quadrant is close to 3000
 2 (d) At P2 the order of the fringe will be maximum

THE TIME LINE


The Time Line~JEE Advanced Questions (2014 –2018) 31

163. A wire is bent in the shape of a right angled 167. Match the nuclear processes given in Column I
triangle and is placed in front of a concave with the appropriate option(s) in Column II.
mirror of focal length f as shown in the figure.
Which of the figures shown in the four options Column I Column II
qualitatively represent(s) the shape of the P. Nuclear fusion 1. absorption of thermal neutrons
image of the bent wire? (These figures are not by 235
92 U
to scale.) (More than One Correct Option, 2018)
60
Q. Fission in a 2. 27 Co nucleus
nuclear reactor
R. β-decay 3. Energy production in stars via
45° hydrogen conversion to helium
f f
— S. γ-ray emission 4. Heavy water
2
5. Neutrino emission

(Matching Type, 2015)

α>45° 168. For a radioactive material, its activity A and


(a) α (b) rate of change of its activity R are defined as
dN dA
A= − and R = − , where N ( t ) is the
∞ dt dt
(c) (d) number of nuclei at time t. Two radioactive
0<α<45°
α source P(mean life τ) and Q (mean life 2τ) have
the same activity at t = 0. Their rate of change
of activities at t = 2τ are RP and RQ ,
16. Modern Physics R n
respectively. If P = , then the value of n is
RQ e (Integer Type, 2015)
164. A metal surface is illuminated by light of two
different wavelengths 248 nm and 310 nm. The 169. An electron in an excited state of Li 2+ ion has
maximum speeds of the photoelectrons 3h
corresponding to these wavelengths are u1 and angular momentum . The de Broglie

u 2, respectively. If the ratio u1 : u 2 = 2 : 1 and wavelength of the electron in this state is pπa0
hc = 1240 eV nm, the work function of the (where a0 is the Bohr radius). The value of p is
metal is nearly (Integer Type, 2015)
(Single Correct Option, 2014)
(a) 3.7 eV (b) 3.2 eV (c) 2.8 eV (d) 2.5 eV 170. A fission reaction is given by
92 U → 54 Xe + + x + y,
236 140 94
38 Sr
165. If λ Cu is the wavelength of K α , X-ray line of
where x and y are two particles. Considering
copper (atomic number 29) and λ MO is the 236
92 U to be at rest, the kinetic energies of the
wavelength of the K α , X-ray line of
molybdenum (atomic number 42), then the products are denoted by K Xe , K Sr , K x (2 MeV)
ratio λ Cu / λ Mo is close to and K y (2 MeV), respectively. Let the binding
(Single Correct Option, 2014) energies per nucleon of 236 140 94
92 U, 54 Xe and 38 Sr be

(a) 1.99 (b) 2.14 (c) 0.50 (d) 0.48 7.5 MeV, 8.5 MeV and 8.5 MeV, respectively.
Considering different conservation laws, the
166. A nuclear power plant supplying electrical correct options is/are
power to a village uses a radioactive material of (More than One Correct Option, 2015)
half life T years as the fuel. The amount of fuel (a) x = n, y = n, K Sr = 129 MeV, K Xe = 86 MeV
at the beginning is such that the total power
(b) x = p, y = e − , K Sr = 129 MeV, K Xe = 86 MeV
requirement of the village is 12.5% of the
(c) x = p, y = n, K Sr = 129 MeV, K Xe = 86 MeV
electrical power available from the plant at
that time. If the plant is able to meet the total (d) x = n, y = n, K Sr = 86 MeV, K Xe = 129 MeV
power needs of the village for a maximum 171. In a historical experiment to determine Planck's
period of nT years, then the value of n is constant, a metal surface was irradiated with
(Integer Type, 2015)
light of different wavelengths. The emitted
photoelectron energies were measured by

THE TIME LINE


32 The Time Line~JEE Advanced Questions (2014 –2018)

applying a stopping potential. The relevant required for safe operation of the laboratory.
data for the wavelength (λ) of incident light and What is the minimum number of days after
the corresponding stopping potential (V 0 ) are which the laboratory can be considered safe for
given below: (Single Correct Option, 2016) use? (Single Correct Option, 2016)
(a) 64 (b) 90
λ (µm ) V0 (Volt )
(c) 108 (d) 120
0.3 2
176. The electrostatic energy of Z protons uniformly
0.4 1 distributed throughout a spherical nucleus of
Given that 3 Z ( Z − 1)e2
radius R is given by E = The
c = 3 × 108 ms −1 and e = 1.6 × 10−19 C, Planck's 5 4πε 0R
constant (in units of J-s) found from such an measured masses of the neutron, 11 H, 157 N and
experiment is) 15
8 O are 1.008665 u, 1.007825 u, 15.000109 u
(a) 6.0 × 10−34 (b) 6.4 × 10−34 and 15.003065 u, respectively. Given that the
(c) 6.6 × 10−34 (d) 6.8 × 10−34 radii of both the 157 N and 158 O nuclei are same, 1
172. Highly excited states for hydrogen-like atoms u = 931.5 MeV/c 2 (c is the speed of light) and
(also called Rydberg states) with nuclear charge e2 /( 4πε 0 ) = 1.44 MeV fm. Assuming that the
Ze are defined by their principle quantum
difference between the binding energies of 157 N
number n, where n >> 1. Which of the following
statement(s) is (are) true? and 158 O is purely due to the electrostatic energy,
(More than One Correct Option, 2016)
the radius of either of the nuclei is (1fm = 10− 15 m)
(a) Relative change in the radii of two consecutive (Single Correct Option, 2016)
orbitals does not depend on Z
(a) 2.85 fm (b) 3.03 fm (c) 3.42 fm (d) 3.80 fm
(b) Relative change in the radii of two consecutive
orbitals varies as 1/ n 177. 131
I is an isotope of Iodine that β decays to an
(c) Relative change in the energy of two consecutive isotope of Xenon with a half-life of 8 days. A
orbitals varies as 1 / n 3 small amount of a serum labelled with 131I is
(d) Relative change in the angular momenta of two injected into the blood of a person. The activity
consecutive orbitals varies as 1/n of the amount of 131I injected was 2 .4 × 105
173. A hydrogen atom in its ground state is becquerel (Bq). It is known that the injected
irradiated by light of wavelength 970Å. Taking serum will get distributed uniformly in the
hc / e = 1.237 × 10−6eVm and the ground state blood stream in less than half an hour. After
11.5 h, 2.5 ml of blood is drawn from the person’s
energy of hydrogen atom as − 13.6 eV, the
body, and gives an activity of 115 Bq. The total
number of lines present in the emission
volume of blood in the person’s body, in litres is
spectrum is
approximately (you may use e2 ≈ 1 + x for
(Integer Type, 2016)
|x|<< 1 and ln 2 ≈ 0.7). (Integer Type, 2017)
174. The isotopes 125 B having a mass 12.014 u
178. A photoelectric material having work-function
undergoes β-decay to 126 C. 126 C has an excited
φ0 is illuminated with light of wavelength λ
state of the nucleus (126 C* ) at 4.041 MeV above  hc
 λ <  . The fastest photoelectron has a
its ground state. If 125 B decays to 125 C *, the  φ0 
maximum kinetic energy of the β-particle in de-Broglie wavelength λ d . A change in
units of MeV is wavelength of the incident light by ∆λ results
(1u = 931.5MeV/ c2, where c is the speed of light ∆λ d
in a change ∆λ d in λ d . Then, the ratio is
in vacuum) (Integer Type, 2013)
∆λ
proportional to (Single Correct Option, 2017)
175. An accident in a nuclear laboratory resulted in λ2d λd
deposition of a certain amount of radioactive (a) (b)
λ 2
λ
material of half-life 18 days inside the
λ3d λ3d
laboratory. Tests revealed that the radiation (c) (d)
λ λ2
was 64 times more than the permissible level

THE TIME LINE


The Time Line~JEE Advanced Questions (2014 –2018) 33

179. In a radioactive decay chain, 232 Th nucleus cathode and the anode. All the emitted
90
electrons are incident normally on the anode
decays to 212
82 Pb nucleus. Let N α and N β be the and are absorbed. The anode experiences a
number of α and β- particles respectively, force F = n × 10−4N due to the impact of the
emitted in this decay process. Which of the electrons. The value of n is ........... . (Take mass
following statements is (are) true? of the electron, me = 9 × 10−31kg and
(More than One Correct Option, 2018) eV = 1.6 × 10−19 J) (Integer Type, 2018)
(a) N α = 5 (b) N α = 6 (c) N β = 2 (d) N β = 4
181. Consider a hydrogen-like ionised atom with
180. In a photoelectric experiment, a parallel beam atomic number Z with a single electron. In the
of monochromatic light with power of 200 W is emission spectrum of this atom, the photon
incident on a perfectly absorbing cathode of work emitted in the n = 2 to n = 1 transition has
function 6.25 eV. The frequency of light is just energy 74.8 eV higher than the photon emitted
above the threshold frequency, so that the in the n = 3 to n = 2 transition. The ionisation
photoelectrons are emitted with negligible energy of the hydrogen atom is 13.6 eV. The
kinetic energy. Assume that the photoelectron value of Z is ............ . (Integer Type, 2018)
emission efficiency is 100%. A potential
difference of 500 V is applied between the

ANSWERS
1 (4) 2 (c,d) 3 (b,.c) 4 (b,d) 5 (a,b,d) 6 (b) 7 (a) 8 (c) 9 (d) 10 (b)
11 (c) 12 (5) 13 (2) 14 (c) 15 (30) 16 (6.30) 17 (d) 18 (c,d) 19 (d) 20 (5)
21 (b) 22 (a) 23 (5) 24 (b) 25 (d) 26 (*) 27 (a,c,d) 28 (b,c) 29 (a,b) 30 (a)
31 (a,c) 32 (2.0) 33 (4) 34 (2) 35 (d) 36 (7) 37 (6) 38 (a,b,d) 39 (c,d) 40 (c)
41 (b) 42 (*) 43 (c) 44 (d) 45 (a,c) 46 (a,c,d) 47 (a,b) 48 (0.75) 49 (b) 50 (2)
51 (7) 52 (c) 53 (c) 54 (b) 55 (d) 56 (4) 57 (c) 58 (a) 59 (c) 60 (a)
61 (a,b) 62 (a) 63 (6) 64 (3) 65 (a,c) 66 (a,c,d) 67 (2) 68 (a) 69 (d) 70 (d)
71 (a,b,d) 72 (a,b,c) 73 (2) 74 (c) 75 (b) 76 (a) 77 (a) 78 (b) 79 (b) 80 (c)
81 (900) 82 (b,c,d) 83 (130.0) 84 (4) 85 (b,d) 86 (a,b,d) 87 (2.09) 88 (a,c,d) 89 (d) 90 (3)
91 (b,c,d) 92 (a,c,d) 93 (6) 94 (5) 95 (a,c) 96 (a,d) 97 (c,d) 98 (c) 99 (c) 100 (6)
101 (d) 102 (d) 103 (1.50) 104 (2) 105 (a,b) 106 (b) 107 (a,b,d) 108 (c) 109 (b) 110 (1)
111 (d) 112 (a,b,c,d) 113 (d) 114 (a) 115 (c,d) 116 (b) 117 (a) 118 (b,d) 119 (2) 120 (b)
121 (b) 122 (3) 123 (5) 124 (c) 125 (b) 126 (c) 127 (a,b,c) 128 (a,d) 129 (a,c) 130 (a,c)
131 (a,c) 132 (c) 133 (a) 134 (c) 135 (a) 136 (b,c) 137 (a,b,d) 138 (5.55) 139 (c,d) 140 (8)
141 (b,c) 142 (a,b) 143 (b,d) 144 (a,b,c) 145 (b,c) 146 (a,b,c) 147 (c) 148 (b) 149 (3) 150 (7)
151 (b) 152 (2) 153 (a) 154 (d) 155 (a) 156 (a,d) 157 (a,c,d) 158 (d) 159 (a,b) 160 (a,b,c)
161 (8) 162 (c.d) 163 (d) 164 (a) 165 (b) 166 (3) 167 (*) 168 (2) 169 (2) 170 (d)
171 (b) 172 (a,b,d) 173 (6) 174 (a) 175 (c) 176 (c) 177 (5) 178 (d) 179 (a,c) 180 (24)
181 (3)

(*) Answer given below


No one Correct Option

For Detailed Solutions, Visit https://goo.gl/SxpVW7

THE TIME LINE


TIME LINE
JEE Advanced 2014-18

CHEMISTRY
1. Some Basic Concepts of Chemistry
1. A compound H2 X with molar weight of 80 g is 5. To measure the quantity of MnCl 2 dissolved in
dissolved in a solvent having density of 0.4 g an aqueous solution, it was completely
mL−1. Assuming no change in volume upon converted to KMnO 4 using the reaction,
dissolution, the molality of a 3.2 molar solution MnCl 2 + K 2S 2O 8 + H 2O → KMnO 4 + H 2SO 4 + HCl
is (Integer Type, 2014) (equation not balanced).
2. The mole fraction of a solute in a solution is 0.1. Few drops of concentrated HCl were added to
At 298 K, molarity of this solution is the same this solution and gently warmed. Further,
as its molality. Density of this solution at 298 K oxalic acid (225 mg) was added in portions till
is 2.0 g cm−3 . The ratio of the molecular weights the colour of the permanganate ion
m  disappeared. The quantity of MnCl 2 (in mg)
of the solute and solvent,  solute  is ...
 msolvent  present in the initial solution is ……… .
(Integer Type, 2016) (Atomic weights in g mol −1: Mn = 55, Cl = 35.5)
3. The ammonia prepared by treating ammonium (Numerical Value Type, 2018)
sulphate with calcium hydroxide is completely
used by NiCl 2 ⋅ 6H2 O to form a stable 2. Atomic Structure
coordination compound. Assume that both the
reactions are 100% complete. If 1584 g of 6. In an atom, the total number of electrons
ammonium sulphate and 952 g of NiCl 2 ⋅ 6H2 O having quantum numbers (Integer Type, 2014)
are used in the preparation, the combined 1
n = 4,| ml | = 1 and ms = − is
weight (in grams) of gypsum and the 2
nickel-ammonia coordination compound thus 7. Not considering the electronic spin, the
produced is____ degeneracy of the second excited state ( n = 3) of
(Atomic weights in g mol −1 : H-atom is 9, while the degeneracy of the second
H = 1, N = 14, O = 16, S = 32, Cl = 35.5, Ca = 40, excited state of H− is (Integer Type, 2015)
Ni = 59 (Numerical Type, 2018)

4. Galena (an ore) is partially oxidised by passing Directions. (Q. Nos. 8-10)
air through it at high temperature. After some By appropriately matching the information given
time, the passage of air is stopped, but the in the three columns of the following table.
heating is continued in a closed furnace such The wave function, ψ n , l, ml is a mathematical
that the content undergo self-reduction. The function whose value depends upon spherical
weight (in kg) of Pb produced per kg of O 2 polar coordinates (r , θ , φ ) of the electron and
consumed is ……… . characterised by the quantum number n , l and ml .
(Atomic weights in g mol −1 : Here r is distance from nucleus, θ is colatitude
O = 16, S = 32, Pb = 207) (Numerical Type, 2018) and φ is azimuth. In the mathematical functions
given in the Table, Z is atomic number and a 0 is
Bohr radius (Matching Type, 2017)

THE TIME LINE


The Time Line~JEE Advanced Questions (2014 –2018) 35

Column 1 Column 2 Column 3


3  Zr 
(I) 1s-orbital (i) −  (P)
Z 2
ψn, l, ml ∝   e  0 
a

 a0  ψn, I, ml (r)

0
r/a0

(II) 2s-orbital (ii) One radial node (Q) 1


Probability density at nucleus ∝
a03
5  Zr 
(III) 2 pz -orbital (iii) (R) Probability density is maximum at nucleus
 Z  2 − a 
ψn, l ml ∝   re  0  cos θ
 a0 

(IV) 3 d z2 -orbital (iv) xy-plane is a nodal plane (S) Energy needed to excite electron from n = 2 state to
27
n = 4 state is times the energy needed to excite
32
electron from n = 2 state to n = 6 state

8. For He+ ion, the only INCORRECT combination 13. A list of species having the formula XZ 4 is given
is below XeF4 , SF4 , SiF4 , BF4− , BrF4− , [Cu(NH3 )4 ] 2+ ,
(a) (I) (i) (S) (b) (II) (ii) (Q)
[FeCl 4 ] 2− , [CoCl 4 ] 2− and [PtCl 4 ] 2−
(c) (I) (iii) (R) (d) (I) (i) (R)
Defining shape on the basis of the location of X
9. For the given orbital in Column 1, the Only and Z atoms, the total number of species having
CORRECT combination for any hydrogen-like a square planar shape is (Integer Type, 2014)
species is
(a) (II) (ii) (P) (b) (I) (ii) (S) 14. Hydrogen bonding plays a central role in which
(c) (IV) (iv) (R) (d) (III) (iii) (P) of the following phenomena?
(More than One Correct Option, 2014)
10. For hydrogen atom, the only CORRECT (a) Ice floats in water
combination is (b) Higher Lewis basicity of primary amines than
(a) (I) (i) (P) (b) (I) (iv) (R) tertiary amines in aqueous solutions
(c) (II) (i) (Q) (d) (I) (i) (S) (c) Formic acid is more acidic than acetic acid
(d) Dimerisation of acetic acid in benzene

3. Periodic Classfication and 15. Match the orbital overlap figures shown in
Column I with the description given in Column
Periodic Properties II and select the correct answer using the codes
given below the Columns. (Matching Type, 2014)
11. The option(s) with only amphoteric oxides
is(are) Column I Column II
(More than One Correct Option, 2017) A. 1. p-d π antibonding
(a) NO, B 2O3 , PbO, SnO2
(b) Cr2O3 , CrO, SnO, PbO
B. 2. d-d σ bonding
(c) Cr2O3 , BeO, SnO, SnO2
(d) ZnO, Al 2O3 , PbO, PbO2
C. 3. p-dπ bonding

4. Chemical Bonding
D. 4. d-d σ antibonding
12. Assuming 2s-2p mixing is not operative, the
paramagnetic species among the following is
(Single Correct Option, 2014) Codes
(a) Be2 (b) B 2 A B C D A B C D
(c) C2 (d) N2 (a) 4 3 2 1 (b) 1 2 3 4
(c) 2 3 1 4 (d) 4 1 2 3

THE TIME LINE


36 The Time Line~JEE Advanced Questions (2014 –2018)

16. Among the triatomic molecules/ions BeCl2 , N3− , 21. The experimental value of d is found to be
N 2O, NO+2 , O3 , SCl2, ICl−2 , I3− and XeF2 , the total smaller than the estimate obtained using
number of linear molecules(s)/ion(s) where the Graham’s law. This is due to
hybridisation of the central atom does not have (a) larger mean free path for X as a compared of that
contribution from the d-orbital(s) is [atomic of Y
number of S = 16, Cl = 17, I = 53 and Xe = 54] (b) larger mean free path for Y as compared to that of X
(Integer Type, 2015)
(c) increased collision frequency of Y with the inert gas
17. According to molecular orbital theory, which of as compared to that of X with the inert gas
the following statements is(are) correct? (d) increased collision frequency of X with the inert gas
(More Than One Correct Option, 2016) as compared to that of Y with the inert gas
(a) C2−
2 is expected to be diamagnetic 22. The value of d in cm (shown in the figure), as
(b) O2+
2 is expected to have a longer bond length than O2 estimated from Graham’s law, is
(c) N+2 and N−2 have the same bond order (a) 8 (b) 12 (c) 16 (d) 20
(d) He+2 has the same energy as two isolated He atoms
23. A closed vessel with rigid walls contains 1 mole
18. Among H2 , He+2 , Li 2 , Be 2 , B2 , C2 , N2 , O−2 and F2, of 238
92 U and 1 mole of air at 298 K. Considering
the number of diamagnetic species is (Atomic complete decay of 238 206
92 U to 82 Pb, the ratio of the
numbers : H = 1, He = 2, Li = 3, Be = 4, B = 5,
final pressure to the initial pressure of the
C = 6, N = 7, O = 8, F = 9) (Integer Type, 2017)
system at 298 K is (Integer Type, 2015)
19. The sum of the number of lone pairs of 24. One mole of a monoatomic real gas satisfies the
electrons on each central atom in the following
equation p(V − b ) = RT where, b is a constant.
species is (Integer Type, 2017)
The relationship of interatomic potential V(r)
[TeBr6 ]2 − , [BrF2 ]+ , SNF3 and [XeF3 ]− and interatomic distance r for gas is given by
(Atomic numbers : N = 7, F = 9, S = 16, Br = 35, (Single Correct Option, 2015)
Te = 52, Xe = 54 V(r) V(r)
(a) 0 r
(b) 0 r
5. States of Matter
20. If the value of Avogadro number is
6.023 × 10 23 mol −1 and the value of Boltzmann V(r) V(r)

constant is 1.380 × 10−23 JK −1 , then the number (c) 0 (d) 0


r r
of significant digits in the calculated value of
the universal gas constant is (Integer Type, 2014)
25. The qualitative sketches I, II and III given
Paragraph (Q. Nos. 21-22) below show the variation of surface tension
X and Y are two volatile liquids with molar with molar concentration of three different
weights of 10 g mol −1 and 40 g mol −1 respectively. aqueous solutions of KCl, CH3 OH and
Two cotton plugs, one soaked in X and the other CH3 (CH2 )11 OSO−3 Na + at room temperature.
soaked in Y, are simultaneously placed at the (Single Correct Option, 2016)
ends of a tube of length L = 24 cm, as shown in
Surface tension

Surface tension

Surface tension

I II III
the figure. The tube is filled with an inert gas at 1
atm pressure and a temperature of 300 K. Vapours
of X and Y react to form a product which is first
observed at a distance d cm from the plug soaked
in X. Take X and Y to have equal molecular
diameters and assume ideal behaviour for the Concentration Concentration Concentration
inert gas and the two vapours. The correct assignment of the sketches is
(Paragraph Type, 2014)
L = 24 cm
I II III
(a) KCl CH3OH CH3 (CH2 )11OSO−3 Na+
(b) CH3 (CH2 )11 CH3OH KCl
OSO−3 Na+
(c) KCl CH3 (CH2 )11OSO−3 Na+ CH3OH
d Initial formation Cotton wool
Cotton wool
soaked in X of the product soaked in Y (d) CH3OH KCl CH3 (CH2 )11OSO−3 Na+

THE TIME LINE


The Time Line~JEE Advanced Questions (2014 –2018) 37

26. The diffusion coefficient of an ideal gas is If this reaction is conducted at ( p , T1 ), with
proportional to its mean free path and mean T2 > T1 the % yield by of ammonia as a function
speed. The absolute temperature of an ideal gas of time is represented by
is increased 4 times and its pressure is
T2 T1
increased 2 times. As a result, the diffusion

% yield

% yield
T1 T2
coefficient of this gas increases x times. The (a) (b)
value of x is ... (Integer Type, 2016)
Time Time

6. Equilibrium T1 T2

% yield

% yield
27. A closed tank has two compartments A and B, T2 T1
(c) (d)
both filled with oxygen (assumed to be ideal
gas). The partition separating the two
compartments is fixed and is a perfect heat Time Time
insulator (Fig. 1). If the old partition is replaced 30. The molar conductivity of a solution of a weak
by a new partition which can slide and conduct acid HX (0.01 M) is 10 times smaller than the
heat but does not allow the gas to leak across molar conductivity of a solution of a weak acid
(Fig. 2), the volume (in m3 ) of the compartment HY (0.10 M). If λ 0X − ≈ λ 0Y − , the difference in
A after the system attains equilibrium is ____.
their pK a values, pK a ( HX ) − pK a ( HY ), is
(consider degree of ionisation of both acids to
1 m3, 5 bar be <<1). (Integer Type, 2015)
3 m3, 1 bar, 300 K
400 K
B
A
Paragraph (Q. Nos. 31-32)
Fig. 1 Thermal decomposition of gaseous X 2 to gaseous X
at 298 K takes place according to the following
equation:
X 2( g ) s 2X ( g )
A B The standard reaction Gibbs energy, ∆ rG°, of this
reaction is positive. At the start of the reaction,
there is one mole of X 2 and no X. As the reaction
proceeds, the number of moles of X formed is
Fig. 2 (Numerical Value Type, 2018) given by β. Thus, βequilibrium is the number of moles
28. MX 2 dissociates into M 2+ and X − ions in an of X formed at equilibrium. The reaction is carried
out at a constant total pressure of 2 bar. Consider
aqueous solution, with a degree of dissociation the gases to behave ideally. (Given, R = 0.083 L bar
(α ) of 0.5. The ratio of the observed depression K −1 mol −1) (Paragraph Type, 2016)
of freezing point of the aqueous solution to the
value of the depression of freezing point in the 31. The equilibrium constant K p for this reaction at
absence of ionic dissociation is 298 K, in terms of β equilibrium is
(Integer Type, 2014) 8β 2 equilibrium 8β 2 equilibrium
(a) (b)
29. The % yield of ammonia as a function of time in 2 − β equilibrium 4 − β 2 equilibrium
the reaction, N 2 ( g ) + 3H 2 ( g ) w 2 NH3 ( g ); 4β 2 equilibrium 4β 2 equilibrium
∆H < 0 (c)
2 − β equilibrium
(d)
4 − β 2 equilibrium
at ( p, T1 ) is given below.
(More than One Correct Option, 2015) 32. The incorrect statement among the following
for this reaction, is
(a) Decrease in the total pressure will result in the
formation of more moles of gaseous X
% yield

T1
(b) At the start of the reaction, dissociation of gaseous
X 2 takes place spontaneously
(c) β equilibrium = 0.7
Time (d) KC < 1

THE TIME LINE


38 The Time Line~JEE Advanced Questions (2014 –2018)

33. The solubility of a salt of weak acid ( AB) at pH 3 (c) ∆Ssystem < 0 and ∆Ssurrounding > 0
−3 −1 (d) ∆Ssystem < 0 and ∆Ssurrounding < 0
is Y × 10 mol L . The value of Y is_____
(Given that the value of solubility product of AB 36. An ideal gas in thermally insulated vessel at
( K sp ) = 2 × 10−10 and the value of ionisation internal pressure = p1, volume = V1 and
constant of HB ( K a ) = 1 × 10−8 absolute temperature = T1 expands irreversibly
against zero external pressure, as shown in
(Numerical Value Type, 2018)
the diagram.
34. Dilution processes of different aqueous The final internal pressure, volume and
solutions, with water, are given in List-I. The absolute temperature of the gas are p2 , V 2 and
effects of dilution of the solution on [H + ] are T2, respectively. For this expansion
given in List II. (Matching Type, 2018)

(Note Degree of dissociation (α ) of weak acid pext=0


and weak base is << 1; degree of hydrolysis of Irreversible
salt << 1;[H + ] represents the concentration of H + pext=0

ions)
List I List II
P. (10 mL of 0.1 M NaOH + 1. the value of [H+ ] p1,V1,T1 p2,V2,T2
20 mL of 0.1 M acetic does not change
acid) diluted to 60 mL on dilution
Thermal insulation
Q. (20 mL of 0.1 M NaOH + 2. the value of [H+ ]
20 mL of 0.1 M acetic changes to half of (a) q = 0 (More than One Correct Option, 2014)
acid) diluted to 80 mL its initial value on
(b) T2 = T1
dilution
(c) p2V2 = p1V1
R. (20 mL of 0.1M HCl + 20 3. the value of [H+ ]
(d) p2V2γ = p1V1γ
mL of 0.1 M ammonia changes to two
solution) diluted to 80 mL times of its initial 37. One mole of an ideal gas at 300 K in thermal
value on dilution. contact with surroundings expands
S. 10 mL saturated solution 4. the value of [H+ ] isothermally from 1.0 L to 2.0 L against a
of Ni(OH)2 in equilibrium changes to
1 constant pressure of 3.0 atm. In this process,
with exces solid Ni(OH)2 is 2 the change in entropy of surroundings ( ∆S surr )
diluted to 20 mL (solid times of its initial in JK −1 is (1 L atm = 101.3 J)
Ni(OH)2 is still present value on dilution (Single Correct Option, 2016)
after dilution). (a) 5.763 (b) 1.013
5. the value of [H+ ] (c) − 1.013 (d) − 5.763
changes to 2
times of its initial 38. The standard state Gibbs free energies of
value on dilution formation of C(graphite) and C(diamond) at
T = 298 K are
Match each process given in List I with one or
∆ f G ° [C (graphite)] = 0 kJ mol −1
more effect(s) in List II. The correct option is
(a) P → 4; Q → 2; R → 3; S → 1 ∆ f G ° [C (diamond)] = 2.9 kJ mol −1
(b) P → 4; Q → 3; R → 2; S → 3 The standard state means that the pressure
(c) P → 1; Q → 4; R → 5; S → 3 should be 1 bar, and substance should be pure
(d) P → 1; Q → 5; R → 4; S → 1 at a given temperature. The conversion of
graphite [C(graphite)] to diamond [C(diamond)]
reduces its volume by 2 × 10−6 m3 mol −1. If
7. Thermodynamics and C(graphite) is converted to C(diamond)
Thermochemistry isothermally at T = 298K, the pressure at
which C(graphite) is in equilibrium with
35. For the process, H2O ( l ) → H2O ( g) C(diamond), is (Single Correct Option, 2017)
at T = 100° C and 1 atmosphere pressure, the [Useful information : 1 J = 1 kg m 2s −2,
correct choice is (Single Correct Option, 2014) 1 Pa = 1 kg m −1 s −2; 1 bar = 10 5 Pa]
(a) ∆Ssystem > 0 and ∆Ssurrounding > 0
(a) 58001 bar (b) 1450 bar
(b) ∆Ssystem > 0 and ∆Ssurrounding < 0 (c) 14501 bar (d) 29001 bar

THE TIME LINE


The Time Line~JEE Advanced Questions (2014 –2018) 39

39. An ideal gas is expanded form ( p1, V1 , T1 ) to 42. For a reaction, A -


P , the plots of [A] and
( p2 , V 2 , T2 ) under different conditions. The [P] with time at temperatures T1 and T2 are
correct statement(s) among the following is given below.
(are) (More than One Correct Option, 2017)
10 10
(a) The work done by the gas is less when it is T1

[A]/(mol L–1)

[P]/(mol L–1)
expanded reversibly fromV1 toV2 under adiabatic T2
conditions as compared to that when expanded 5 5
reversibly fromV1 toV2 under isothermal conditions. T2
(b) The change in internal energy of the gas is (i) zero, if it T1
is expanded reversibly withT1 = T2 , and (ii) positive, if
Time Time
it is expanded reversibly under adiabatic conditions
withT1 ≠ T2 If T2 > T1, the correct statement(s) is are
(c) If the expansion is carried out freely, it is (Assume ∆H s and ∆S s are independent of
simultaneously both isothermal as well as adiabatic temperature and ratio of ln K at T1 to ln K at T2
(d) The work done on the gas is maximum when it is is greater than T2 / T1. Here H , S , G and K are
compressed irrversibly from (p2 , V2 ) to (p1, V1) against enthalpy, entropy, Gibbs energy and
constant pressure p1 equilibrium constant, respectively.)
(More than One Correct Option, 2018)
40. For a reaction taking place in a container in
equilibrium with its surroundings, the effect of (a) ∆H s < 0, ∆S s < 0 (b) ∆G s < 0, ∆H s > 0
temperature on its equilibrium constant K in (c) ∆G s < 0, ∆S s < 0 (d) ∆G s < 0, ∆S s > 0
terms of change in entropy is described by 43. The surface of copper gets tarnished by the
(More than One Correct Option, 2017)
formation of copper oxide. N 2 gas was passed to
(a) With increase in temperature, the value of K for prevent the oxide formation during heating of
endothermic reaction increases because copper at 1250 K. However, the N 2 gas contains
unfavourable change in entropy of the surroundings 1 mole % of water vapour as impurity. The
decreases water vapour oxidises copper as per the
(b) With increase in temperature, the value of K for reaction given below
exothermic reaction decreases because favourable
2Cu( g)+ H 2O( g) → Cu 2O( s)+ H 2( g)
change in entropy of the surrounding decreases
pH 2 is the minimum partial pressure of H 2 (in
(c) With increase in temperature, the value of K for
bar) needed to prevent the oxidation at 1250 K.
endothermic reaction increases because the
The value of ln( pH 2 ) is ……… .
entropy change of the system is negative
(d) With increase in temperature, the value of K for (Given : total pressure = 1 bar, R (universal gas
exothermic reaction decreases because the entropy constant) = 8 J K −1 mol −1, ln(10) = 2.30 Cu( s) and
change of the system is positive Cu 2O( s) are mutually immiscible.)
41. A reversible cyclic At 1250 K : 2Cu( s)+ 1 / 2 O 2( g) → Cu 2O( s);
process for an ideal ∆G s = − 78,000 J mol −1
gas is shown below. A( p1, V1, T1) C( p2, V1, T2) 1
H 2( g)+ O2( g) → H 2O( g);
Volume (V)

Here, p, V and T
are pressure, 2
volume and ∆G s = − 1,78,000 J mol −1; G is the Gibbs energy
temperature, (Numerical Value Type, 2018)
respectively. The B( p2, V2, T1)
thermodynamic
parameters q , w, H 8. Solid State
Temperature (T)
and U are heat, 44. If the unit cell of a mineral has cubic close
work, enthalpy and packed (ccp) array of oxygen atoms with m
internal energy, respectively. The correct fraction of octahedral holes occupied by
options is (are) (More than One Correct Option, 2018) aluminium ions and n fraction of tetrahedral
(a) qAC = ∆UBC and w AB = p2 (V2 − V1) holes occupied by magnesium ions, m and n
(b) wBC = p2 (V2 − V1) and qBC = ∆H AC respectively, are
(More than One Correct Option, 2015)
(c) ∆HCA < ∆UCA and qAC = ∆UBC 1 1 1 1 1 1 1
(d) qBC = ∆H AC and ∆HCA > ∆UCA (a) , (b) 1 , (c) , (d) ,
2 8 4 2 2 4 8

THE TIME LINE


40 The Time Line~JEE Advanced Questions (2014 –2018)

45. The correct statement(s) for cubic close packed


(ccp) three dimensional structure is (are) Z
(More than One Correct Option, 2016)
(a) The number of the nearest neighbours of an atom pL
present in the topmost layer is 12
(b) The packing efficiency of atom is 74%
(c) The number of octahedral and tetrahedral voids per 1 XM 0
atom are 1 and 2, respectively
(a) The point Z represents vapour pressure of pure liquid
(d) The unit cell edge length is 2 2 times the radius of
M and Raoult’s law is obeyed from xL = 0 to xL = 1
the atom
(b) Attractive intermolecular interactions between L - L in
46. A crystalline solid of a pure substance has a pure liquid L and M - M in pure liquid M are stronger
face-centred cubic structure with a cell edge of than those between L - M when mixed in solution
400 pm. If the density of the substance in the
(c) The point Z represents vapour pressure of pure
crystal is 8 g cm −3 , then the number of atoms
liquid M and Raoult’s law is obeyed when xL → 0
present in 256 g of the crystal is N × 1024. The
(d) The point Z represents vapour pressure of pure
value of N is (Integer Type, 2017)
liquid L and Raoult’s law is obeyed when xL → 1
47. Consider an ionic solid MX with NaCl structure.
51. Liquids A and B form ideal solution over the
Construct a new structure (Z) whose unit cell is
entire range of composition. At temperature T,
constructed from the unit cell of MX following
equimolar binary solution of liquids A and B
the sequential instruction given below. Neglect has vapour pressure 45 torr. At the same
the charge balance. temperature, a new solution of A and B having
(a) Remove all the anions (X) except the central one mole fractions xA and xB , respectively, has
(b) Replace all the face centered cations (M) by anions vapour pressure of 22.5 torr. The value of
(X) xA / xB in the new solution is ____.
(c) Remove all the corner cations (M) (Given that the vapour pressure of pure liquid
(d) Replace the central anion (X) with cation (M) A is 20 torr at temperature T)
The value of 
Number of anions  (Numerical Value Type, 2018)
 in Z is ___
 Number of cations 
52. The plot given below shows p −T curves (where
(Numerical Value Type, 2018)
p is the pressure and T is the temperature) for
two solvents X and Y and isomolal solution of
9. Solution and Colligative NaCl in these solvents. NaCl completely
dissociates in both the solvents.
Properties 1 2 3 4
48. If the freezing point of a 0.01 molal aqueous
solution of a cobalt (III) chloride-ammonia 760
complex (which behaves as a strong electrolyte)
is − 0.0558°C, the number of chloride(s) in the
Pressure (mmHg)

1. Solvent X
coordination sphere of the complex is 2. Solution of NaCl in solvent X
[K f of water = 1.86 K kg mol−1 ] (Integer Type, 2015) 3. Solvent Y
4. Solution of NaCl in solvent Y

49. Mixture(s) showing positive deviation from


Raoult’s law at 35°C is (are)
(More than One Correct Option, 2016)
(a) carbon tetrachloride + methanol
360

362

367
368

(b) carbon disulphide + acetone


Temperature (K)
(c) benzene + toluene (d) phenol + aniline
50. For a solution formed by mixing liquids L and On addition of equal number of moles of a
M, the vapour pressure of L plotted against the non-volatile solute S in equal amount (in kg) of
mole fraction of M in solution is shown in the these solvents, the elevation of boiling point of
following figure. Here xL and x M represent mole solvent X is three times that of solvent Y.
fractions of L and M, respectively, in the solution. Solute S is known to undergo dimerisation in
The correct statement(s) applicable to this these solvents. If the degree of dimerisation is
0.7 in solvent Y , the degree of dimerisation in
system is (are) (More than One Correct Option, 2017)
solvent X is ____. (Numerical Value Type, 2018)

THE TIME LINE


The Time Line~JEE Advanced Questions (2014 –2018) 41

The standard emf of the cell is 2.70 V at 300 K.


10. Electrochemistry When the concentration of Mg 2+ is changed to x
53. In a galvanic cell, the salt-bridge M, the cell potential changes to 2.67 V at 300 K.
(More than One Correct Option, 2014) The value of x is _____.
(a) does not participate chemically in the cell reaction F
(b) stops the diffusion of ions from one electrode to (Given, = 11500 K V−1 , where F is the
R
another Faraday constant and R is the gas contant, in
(c) is necessary for the occurrence of the cell reaction (10) = 2.30) (Numerical Value Type, 2018)
(d) ensures mixing of the two electrolytic solutions 59. Consider an electrochemical cell :
54. All the energy released from the reaction A( s)| An + ( aq , 2M)||B2n + ( aq , 1M)| B( s). The
−1
X → Y , ∆ r G ° = − 193 kJ mol is used for value of ∆H s for the cell reaction is twice of ∆G s
oxidising M + as at 300 K. If the emf of the cell is zero, the ∆S s
M + → M 3 + + 2e − , E ° = − 0. 25 V. (in J K −1 mol −1) of the cell reaction per mole of
B formed at 300 K is …… .
Under standard conditions, the number of
moles of M + oxidised when one mole of X is (Given : ln( 2) = 0.7, R (universal gas constant)
converted to Y is [F = 96500 C mol − ] = 8.3 J K −1 mol −1. H , S and G are enthalpy,
(Integer Type, 2015) entropy and Gibbs energy, respectively.)
(Numerical Value Type, 2018)
55. For the following electrochemical cell at 298 K,
Pt ( s)| H2( g, 1 bar)| H+ ( aq , 1 M) || M 4+ ( aq ),
M 2+ ( aq )| Pt ( s) 11. Chemical Kinetics
[M 2+ ( aq )] 60. For the elementary reaction, M → N , the
Ecell = 0.092 V when = 10x rate of disappearance of M increases by a factor
[M 4+ ( aq )]
RT of 8 upon doubling the concentration of M. The
Given : E °M 4+ / M 2+ = 0.151 V; 2.303 order of the reaction with respect to M is
F (Single Correct Option, 2014)
= 0.059 V (a) 4 (b) 3 (c) 2 (d) 1
The value of x is (Single Correct Option, 2016)
(a) − 2 (b) − 1 61. According to the Arrhenius equation,
(More than One Correct Option, 2016)
(c) 1 (d) 2
(a) a high activation energy usually implies a fast
56. For the following cell, reaction
Zn( s)|ZnSO4( aq )||CuSO4( aq )|Cu( s) (b) rate constant increases with increase in
when the concentration of Zn2+ is 10 times the temperature. This is due to a greater number of
concentration of Cu2+ , the expression for ∆G collisions whose energy exceeds the activation
(in J mol −1) is [F is Faraday constant; R is gas energy
constant; T is temperature; E° (cell) = 11. V] (c) higher the magnitude of activation energy, stronger
(Single Correct Option, 2017) is the temperature dependence of the rate constant
(a) 2.303 RT +11
. F (b) 1.1 F (d) the pre-exponential factor is a measure of the rate
(c) 2.303 RT − 2.2 F (d) −2 .2 F at which collisions occur, irrespective of their
57. The conductance of a 0.0015 M aqueous energy
solution of a weak monobasic acid was 62. In a bimolecular reaction, the steric factor P
determined by using a conductivity cell was experimentally determined to be 4.5. the
consisting of platinised Pt electrodes. The correct option(s) among the following is(are)
distance between the electrodes is 120 cm with (More than One Correct Option, 2017)
an area of cross section of 1 cm 2. The (a) The activation energy of the reaction is unaffected
conductance of this solution was found to be by the value of the steric factor
5 × 10−7 S. The pH of the solution is 4. The (b) Experimentally determined value of frequency factor
value of limiting molar conductivity ( Λ°m ) of this is higher than that predicted by Arrhenius equation
weak monobasic acid in aqueous solution is (c) The value of frequency factor predicted by
Z × 102 S cm −1 mol −1. The value of Z is Arrhenius equation is higher than that determined
(Integer Type, 2017) experimentally
58. For the electrochemical cell, (d) Since P = 4.5, the reaction will not proceed unless
2+ 2+ an effective catalyst is used
Mg( s )| Mg ( aq, 1 M) | | Cu ( aq , 1 M) | Cu( s )

THE TIME LINE


42 The Time Line~JEE Advanced Questions (2014 –2018)

63. For a first order reaction A( g) → 2B( g)+ C( g) (a) O2 is physisorbed


at constant volume and 300 K, the total (b) heat is released
pressure at the beginning ( t = 0) and at time t (c) occupancy of ∗ π 2 p of O2 is increased
are p0 and pt , respectively. (d) bond length of O2 is increased
Initially, only A is present with concentration
[ A]0, and t1/ 3 is the time required for the partial 67. The correct statement(s) about surface properties
pressure of A to reach 1/3 rd of its initial value. is(are) (More than One Correct Option, 2017)

The correct option(s) is (are) (a) The critical temperatures of ethane and nitrogen are
(Assume that all these gases behave as ideal 563 K and 126 K, respectively. The adsorption of
gases) ethane will be more than that of nitrogen of same
amount of activated charcoal at a given temperature
(b) Cloud is an emulsion type of colloid in which liquid
In(3p0–pt)

is dispersed phase and gas is dispersion medium


t1/3

(a) (b)
(c) Adsorption is accompanied by decrease in enthalpy
and decrease in entropy of the system
(d) Brownian motion of colloidal particles does not
Time [A]0
depend on the size of the particles but depends on
viscosity of the solution
Rate constant
In(p0–pt)

(c) (d)
14. s-Block Elements
68. Hydrogen peroxide in its reaction with KIO4
Time [ A ]0 and NH2 OH respectively, is acting as a
(Single Correct Option, 2014)
64. Consider the following reversible reaction,
(a) reducing agent, oxidising agent
A( g)+ B( g) - AB( g) (b) reducing agent, reducing agent
The activation energy of the backward reaction (c) oxidising agent, oxidising agent
exceeds that of the forward reaction by 2RT (in (d) oxidising agent, reducing agent
J mol −1). If the pre-exponential factor of the
forward reaction is 4 times that of the reverse
reaction, the absolute value of ∆G s (in J mol −1) 15. p-Block Elements-I
for the reaction at 300 K is ……… . 69. The correct statement(s) for orthoboric acid
(Given ; ln( 2) = 0.7 RT = 2500 J mol −1 at 300 K is/are (More than One Correct Option, 2014)

and G is the Gibbs energy) (a) It behaves as a weak acid in water due to self
(Numerical Value Type, 2018) ionisation
(b) Acidity of its aqueous solution increases upon
12. Nuclear Chemistry addition of ethylene glycol
(c) It has a three-dimensional structure due to hydrogen
65. A plot of the number of neutrons (n) against the bonding
number of protons (p) of stable nuclei exhibits (d) It is a weak electrolyte in water
upward deviation from linearity for atomic
number, Z > 20. For an unstable nucleus having 70. Three moles of B2 H6 are completely reacted
n/p ratio less than 1, the possible mode(s) of with methanol. The number of moles of boron
decay is (are) (More than One Correct Option, 2016) containing product formed is (Integer Type, 2015)
(a) β − - decay (β - emission) 71. The increasing order of atomic radii of the
(b) orbital or K-electron capture following Group 13 elements is
(Single Correct Option, 2016)
(c) neutron emission
(d) β + -decay (positron emission) (a) Al < Ga < In < Tl (b) Ga < Al < In < Tl
(c) Al < In < Ga < Tl (d) Al < Ga < Tl < In
72. The crystalline form of borax has
13. Surface Chemistry (More than One Correct Option, 2016)
66. When O2 is adsorbed on a metallic surface, (a) tetranuclear [B4O5 (OH)4]2 − unit
electron transfer occurs from the metal to O 2. (b) all boron atoms in the same plane
The true statement(s) regarding this adsorption (c) equal number of sp 2 and sp 3 hybridised boron atoms
is (are) (More than One Correct Option, 2015) (d) one terminal hydroxide per boron atom

THE TIME LINE


The Time Line~JEE Advanced Questions (2014 –2018) 43

73. Among the following, the correct statement(s) 79. The correct statement(s) about the oxoacids,
is(are) (More than One Correct Option, 2017) HClO4 and HClO, is (are)
(a) Al (CH3 )3 has the three-centre two-electron bonds in (More than One Correct Option, 2017)
its dimeric structure (a) The central atom in both HClO4 and HClO is
(b) The Lewis acidity of BCl3 is greater than that of AlCl 3 sp 3 -hybridised
(c) AlCl 3 has the three-centre two-electron bonds in its (b) HClO4 is formed in the reaction between Cl 2 and H2O
dimeric structure (c) The conjugate base of HClO4 is weaker base thanH2O
(d) BH3 has the three-centre two-electron bonds in its (d) HClO4 is more acidic than HClO because of the
dimeric structure resonance stabilisation of its anion
80. The colour of the X 2 molecules of group 17
16. p-Block Elements-II elements changes gradually from yellow to
violet down the group. This is due to
74. The product formed in the reaction of SOCl 2 with (More than One Correct Option, 2017)
white phosphorus is (Single Correct Option, 2014) (a) decrease in π * − σ * gap down the group
(a) PCl 3 (b) SO2 Cl 2
(b) decrease in ionisation energy down the group
(c) SCl 2 (d) POCl 3
(c) the physical state of X 2 at room temperature
75. Under ambient conditions, the total number of changes from gas to solid down the group
gases released as products in the final step of (d) decreases in HOMO-LUMO gap down the group
the reaction scheme shown below is
(Single Correct Option, 2014)
Paragraph (Q. Nos. 81-82)
Upon heating KClO3 in presence of catalytic
A(p1, V1, T1) C(p2, V1, T2) amount of MnO 2, a gas W is formed. Excess
Volume (V)

amount of W reacts with white phosphorus to give


X. The reaction of X with pure HNO3 gives Y and
Z. (Paragraph Type, 2017)
81. Y and Z are, respectively
B(p2, V2, T1)
(a) N2O4 and HPO3 (b) N2O4 and H3PO3
(c) N2O3 and H3PO4 (d) N2O5 and HPO3
Temperature (T)
82. W and X are, respectively
(a) 0 (b) 1 (a) O2 and P4O10 (b) O2 and P4O6
(c) 2 (d) 3 (c) O3 and P4O6 (d) O3 and P4O10
76. The compound(s) with two lone pairs of 83. The compound(s) which generate (s) N2 gas
electrons on the central atom is (are) upon thermal decomposition below 300° C is
(More than One Correct Option, 2016)
(are) (More than One Correct Option, 2018)
(a) BrF5 (b) ClF3
(a) NH4NO 3 (b) (NH4 )2 Cr2O 7
(c) XeF4 (d) SF4
(c) Ba(N3 )2 (d) Mg 3N2
77. The nitrogen containing compound produced in
84. Based on the compounds of group 15 elements,
the reaction of HNO3 with P4 O10
(More than One Correct Option, 2016) the correct statement(s) is (are)
(More than One Correct Option, 2018)
(a) can also be prepared by reaction of P4 and HNO 3
(a) Bi 2O 5 is more basic than N2O 5
(b) is diamagnetic
(b) NF3 is more covalent than BiF3
(c) contains one NN bond
(d) reacts with Na metal producing a brown gas (c) PH3 boils at lower temperature than NH3
(d) The N — N single bond is stronger than the P — P
78. The order of the oxidation state of the single bond
phosphorus atom in H3 PO2 , H3 PO4 , H3 PO3 and
H4 P2 O6 is (Single Correct Option, 2017) 85. The total number of compounds having at least
(a) H3PO4 > H3PO2 > H3PO3 > H4P2O6 one bridging oxo group among the molecules
given below is ……… .
(b) H3PO4 > H4P2O6 > H3PO3 > H3PO2
(c) H3PO2 > H3PO3 > H4P2O6 > H3PO4 N 2O 3, N 2O 5, P4O 6, P4O7 , H 4 P2O 5 ,
H 5P3O10, H 2S 2O 3, H 2S 2O 5
(d) H3PO3 > H3PO2 > H3PO4 > H4P2O6 (Numerical Value Type, 2018)

THE TIME LINE


44 The Time Line~JEE Advanced Questions (2014 –2018)

93. Among the species given below, the total


17. Transition and Inner number of diamagnetic species is____
Transition Elements H atom, NO2 monomer, O−2 (superoxide), dimeric
86. Consider the following list of reagents, acidified sulphur in vapour phase,
K 2 Cr2 O7 , alkaline KMnO4 , CuSO4 , H2 O2 , Cl 2, O3 , Mn 3 O4 ,( NH4 )2 [FeCl 4 ], ( NH4 )2 [NiCl 4 ],
FeCl3 , HNO3 and Na 2 S2 O3 . The total number of K 2 MnO4 , K 2 CrO4 (Numerical Value Type, 2018)
reagents that can oxidise aqueous iodide to
iodine is (Integer Type, 2014)
87. The correct statement(s) about Cr 2+ and
18. Coordination Compounds
3+
Mn is/are [atomic number of Cr = 24 and 94. Match each coordination compound in Column I
Mn = 25] (More than One Correct Option, 2015) with an appropriate pair of characteristics from
(a) Cr 2+ is a reducing agent
Column II and select the correct answer using
the codes given below the Columns
(b) Mn3+ is an oxidising agent
(en = H2 NCH2 CH2 NH2 ; atomic numbers : Ti = 22;
(c) both Cr 2+ and Mn3+ exhibit d 4 electronic Cr = 24; Co = 27; Pt = 78) (Matching Type, 2014)
configuration Column I Column II
(d) when Cr 2+ is used as a reducing agent, the (A) [Cr(NH3 )4 Cl 2 ]Cl 1. Paramagnetic and exhibits
chromiumion attains d 5 electronic configuration ionisation isomerism
88. Fe3 + is reduced to Fe2+ by using (B) [Ti(H2O)5 Cl](NO 3 )2 2. Diamagnetic and exhibits
(More than One Correct Option, 2015) cis-trans isomerism
(a) H2O2 in presence of NaOH (C) [Pt(en)(NH3 )Cl] 3. Paramagnetic and exhibits
(b) Na 2O2 in water NO 3 cis-trans isomerism
(c) H2O2 in presence of H2 SO4 (D) [CO(NH3 )4 (NO 3 )2 ] 4. Diamagnetic and exhibits
(d) Na 2O2 in presence of H2 SO4 NO 3 ionisation isomerism

89. In dilute aqueous H2 SO4 the complex Codes


diaquadioxalatoferrate (II) is oxidised by MnO–4 . A B C D A B C D
For this reaction, the ratio of the rate of change (a) 4 2 3 1 (b) 3 1 4 2
of [H+ ] to the rate of change of [MnO−4 ] is (c) 2 1 3 4 (d) 1 3 4 2
(Integer Type, 2015)
95. For the octahedral complexes of Fe3+ in SCN−
90. In neutral or faintly alkaline solution, 8 moles (thiocyanato-S) and in CN− ligand
of permanganate anion quantitative oxidise environments, the difference between the spin
thiosulphate anions to produce X moles of a only magnetic moments in Bohr magnetons
sulphur containing product. The magnitude of (when approximated to the nearest integer) is
X is ... . (Integer Type, 2016) [atomic number of Fe = 26 ] (Integer Type, 2015)
91. Which of the following combination will produce 96. In the complex acetylbromidodicarbonylbis
H 2 gas? (Single Correct Option, 2017) (triethylphosphine) iron (II), the number of Fe
(a) Fe metal and conc. HNO3  C bond (s) is (Integer Type, 2015)
(b) Cu metal and conc. HNO3 97. Among the complex ions,
(c) Au metal and NaCN (aq) in the presence of air
[Co(NH2  CH2  CH2  NH2 )2 Cl 2 ]+ ,
(d) Zn metal and NaOH (aq)
[CrCl 2 (C2 O4 )2 ]3− ,
92. The correct statement (s) regarding the binary
transition metal carbonyl compounds is (are) [Fe(H2 O)4 (OH)2 ]+ , [Fe(NH3 )2 (CN)4 ]− ,
(Atomic numbers : Fe = 26, Ni = 28) [Co(NH2  CH2  CH2  NH2 )2 (NH3 ) Cl]2+ and
(More Than One Option Correct, 2018) [Co(NH3 )4 (H2 O)Cl]2+ the number of complex ion(s)
(a) total number of valence shell electrons at metal that show(s) cis-trans isomerism is
centre in Fe(CO)5 or Ni(CO) 4 is 16 (Integer Type, 2015)
(b) these are predominantly low spin in nature 2−
98. Among [Ni(CO)4 ],[NiCl 4 ] , [Co(NH3 )4 Cl 2 ] Cl,
(c) metal-carbon bond strengthens when the oxidation Na 3 [CoF6 ], Na 2 O2 and CsO2 , the total number of
state of the metal is lowered paramagnetic compounds is
(d) the carbonyl C—O bond weakens when the (Single Correct option, 2016)
oxidation state of the metal is increased (a) 2 (b) 3 (c) 4 (d) 5

THE TIME LINE


The Time Line~JEE Advanced Questions (2014 –2018) 45

99. The possible number of geometrical isomers for 104. Copper is purified by electrolytic refining of blister
the complex [CoL2 Cl 2 ]−( L = H2 NCH2 CH2 O− ) is copper. The correct statement(s) about this
(are)... (Integer Type, 2016) process is/are (More than One Correct Option, 2015)
(a) impure Cu strip is used as cathode
100. Addition of excess aqueous ammonia to a pink
(b) acidified aqueous CuSO4 is used as electrolyte
coloured aqueous solution of MCl2 ⋅ 6H 2O( X )
and NH 4Cl gives an octahedral complex Y in (c) pure Cu deposits at cathode
the presence of air. In aqueous solution, (d) impurities settle as anode-mud
complex Y behaves as 1 : 3 electrolyte. The 105. Extraction of copper from copper pyrite (CuFeS 2)
reaction of X with excess HCl at room involves (More than One Correct Option, 2016)
temperature results in the formation of a blue (a) crushing followed by concentration of the ore by
colured complex Z. The calculated spin only froth-floatation
magnetic moment of X and Z is 3.87 B.M., (b) removal of iron as slag
whersas it is zero for complex Y . Among the
(c) self reduction step to produce ‘blister copper’
following options, which statement(s) is (are)
following evolution of SO2
correct? (More than One Correct Option, 2017)
(d) refining of ‘blister copper’ by carbon reduction
(a) The hybridisation of the central metal ion inY is d 2sp 3
(b) Addition of silver nitrate toY given only two 106. Match the anionic species given in Column I that
equivalents of silver chloride are present in the ore (s) given in Column II.
(c) When X and Y are in equilibrium at 0°C, the colour (Matching Type, 2015)
of the solution is pink Column I Column II
(d) Z is a tetrahedral complex
A. Carbonate P. Siderite
101. The correct option(s) regarding the complex B. Sulphide Q. Malachite
[Co(en)(NH 3 )3 ( H 2O)]3 + ( en = H 2NCH 2CH 2NH 2 ) is C. Hydroxide R. Bauxite
(are) (More Than One Correct Option, 2018) D. Oxide S. Calamine
(a) it has two geometrical isomers T. Argentite
(b) it will have three geometrical isomers, if bidentate
‘en’ is replaced by two cyanide ligands
(c) it is paramagnetic
20. Qualitative Analysis
(d) it absorbs light at longer wavelength as compared 107. Among PbS, CuS, HgS, MnS, Ag 2 S, NiS, CoS,
to [Co(en)(NH3 )4 ]3+ Bi 2 S3 and SnS2 the total number of black
coloured sulphides is (Integer Type, 2014)
102. Match each set of hybrid orbitals from List I
with complexes given in List II. 108. The pair(s) of ions where both the ions are
precipitated upon passing H2S gas in presence
List I List II
of dilute HCl, is (are)
P. dsp2 1. [FeF6 ]4− (More than One Correct Option, 2015)
(a) Ba 2+ , Zn2+ (b) Bi 3+ , Fe3+
Q. sp3 2. [Ti(H2O)3 Cl 3 ] (c) Cu2+ , Pb2+ (d) Hg2+ , Bi 3+
R. sp3d 2 3. [Cr(NH3 )6 ]3+ 109. In the following reaction sequence in aqueous
S. d 2 sp3 4. [FeCl 4 ]2− solution, the species X, Y and Z, respectively, are
(Single Correct Option, 2016)
5. [Ni(CO)4 ] Ag + Ag + With time
S2 O23 − → X → Y → Z
6. [Ni(CN)4 ]2− (Clear solution) (White ppt.) (Black ppt.)
(a) [Ag(S2O3 )2 ]3− , Ag2 S2O3 , Ag2 S
The correct option is (Matching Type , 2018)
(b) [Ag(S2O3 )3 ]5− , Ag2 SO3 , Ag2 S
(a) P → 5; Q → 4, 6; R → 2, 3; S → 1
(b) P → 5,6; Q → 4; R → 3; S → 1,2 (c) [Ag(SO3 )2 ]3− , Ag2 S2O3 , Ag
(c) P → 6; Q → 4, 5; R → 1; S → 2, 3 (d) [Ag(SO3 )3 ]3− , Ag2 SO4 , Ag
(d) P → 4,6; Q → 5, 6; R → 1,2; S → 3 110. The reagent(s) that can selectively precipitate
S2 − from a mixture of S2 − and SO24 − in
19. Extraction of Metals aqueous solution is (are)
103. Upon heating with Cu 2 S, the reagent(s) that (More than One Correct Option, 2016)
give copper metal is/are (a) CuCl 2 (b) BaCl 2
(More than One Correct Option, 2014) (c) Pb(OOCCH3 )2 (d) Na 2 [Fe(CN)5 NO]
(a) CuFeS2 (b) CuO (c) Cu2O (d) CuSO4

THE TIME LINE


46 The Time Line~JEE Advanced Questions (2014 –2018)

111. The correct option(s) to distinguish nitrate salts 116. For the given compound X, the total number of
to Mn 2+ and Cu 2+ taken separately is (are) optically active stereoisomers is ……… .
(More than One Correct Option, 2018)
HO
(a) Mn2+ shows the characteristic green colour in the HO
flame test
(b) only Cu2+ shows the formation of precipitate by
passing H2S in acidic medium
(c) only Mn2+ shows the formation of precipitate by HO
passing H2S in faintly basic medium HO
(d) Cu2+ /Cu has higher reduction potential than This type of bond indicates that the confguration at the
Mn2+ / Mn (measured under similar conditions) specific carbon and the geometry of the duble bond is fixed
This type of bond indicates that the confguration at the
specific carbon and the geometry of the duble bond is fixed

21. Organic Chemistry Basics (Numerical Value Type, 2018)

112. The correct combination of names for isomeric


alcohols with molecular formula C4 H10 O is/are 22. Hydrocarbons
(More than One Correct Option, 2014)
117. Isomers of hexane, based on their branching,
(a) tert-butanol and 2-methylpropan-2-ol can be divided into three distinct classes as
(b) tert-butanol and 1,1-dimethylethan-1-ol shown in the figure. (Single Correct Option, 2014)
(c) n-butanol and butan-1-ol
(d) iso-butyl alcohol and 2-methylpropan-1-ol I. and II. and
113. The total number(s) of stable conformers with
non-zero dipole moment for the following
III.
compound is/are (Integer Type, 2014)
Cl
Br CH3 The correct order of their boiling point is
Br Cl (a) I > II > III (b) III > II > I
(c) II > III > I (d) III > I > II
CH3
114. The total number of stereoisomers that can
exist for M is (Integer Type, 2015)
Paragraph (Q. No. 118-119)
H3C CH3 Pd-BaSO4 (i) B2H6
C8H6 C8H8 X
H2 (ii) H2O2,NaOH, H2O
H2O
HgSO4,H2SO4
(i) EtMgBr, H2O
C8H8O Y
(ii) H+, Heat (Paragraph Type, 2015)
118. Compound X is
H3C M OH
O
115. The correct order of acidity for the following (a) (b)
CH3 CH3
compounds is (Single Correct Option, 2016)]
CO2H CO2H
OH
HO OH (c) (d) CHO
I II
OH
CO2H 119. The major compound Y is
CO2H
(a) CH3
(b) CH3
OH
III IV CH3
CH3
CH3
OH (c) (d) CH3
(a) I > II > III > IV (b) III > I > II > IV
(c) III > IV > II > I (d) I > III > IV > II

THE TIME LINE


The Time Line~JEE Advanced Questions (2014 –2018) 47

120. The correct statement(s) for the following CH3 CH3


addition reactions is (are)
(More than One Correct Option, 2017) H3C C Br
Br Br Br
H3C H
Br2/CHCl3
CH3
(i) M and N I II III IV
H CH3 (a) compound IV undergoes inversion of configuration
H3C CH3 (b) the order of reactivity for I, III and IV is : IV > I > III
(ii)
Br2/CHCl3
O and P (c) I and III follow SN1mechanism
(d) I and II follow SN1mechanism
H H
(a) (M and O) and (N and P) are two pairs of 125. In the following reaction sequence, the correct
enantiomers structure (s) of X is (are)
(b) Bromination proceeds through trans-addition in Me N3
both the reactions (1) PBr3, Et2O
X
(c) O and P are identical molecules (2) NaI, Me2CO
(3) NaN3, HCONMe2
(d) (M and O) and (N and P) two pairs of diastereomers Enantiomerically
pure

23. Alkyl Halides Me OH


(b)
Me OH
(a)
121. The product of the reaction given below is
(Single Correct Option 2015)
(i) NBS/hν
X
(ii) H2O/K2CO3
(c) Me OH (d) Me OH
OH O CO2H (One Option Correct, 2018)
(a) (b) (c) (d)
24. Alcohols and Ethers
122. Compound(s) that on hydrogenation produce(s) 126. The acidic hydrolysis of ether (X) shown below
optically inactive compound (s) is/are is fastest when (Single Correct Option, 2014 )
(Single Correct Option, 2015)
H Br H Br
(a) (b) H2C CH3
H3 C CH3
Acid
H Br Br H OR OH + ROH
(c) H2 C (d) H2C CH3
CH3
CH3

123. In the following reaction, the major product is [X ]


(Single Correct Option, 2015)
CH3 (a) one phenyl group is replaced by a methyl group
CH2 1 equivalent HBr
(b) one phenyl group is replaced by a
H2C para-methoxyphenyl group
CH3 CH3
(c) two phenyl groups are replaced by two
CH3
(a) (b) para-methoxyphenyl groups
H2 C H3C
Br
Br (d) no structural change is made to X
CH3 CH3
127. The number of hydroxyl group(s) in Q is
(c) (d) (Integer Type, 2015)
H2 C Br H3 C Br
Aqueous dilute
124. For the following compounds, the correct H H+ KMnO4 (excess)
P Q
Heat 0°C
statement(s) with respect to nucleophilic HO
substitution reaction is(are) H3C CH3
(More than One Correct Option, 2017)

THE TIME LINE


48 The Time Line~JEE Advanced Questions (2014 –2018)

128. The correct statement(s) about the following 131. The major product of the following reaction
reaction sequence is (are) is
(More than One Correct Option, 2015)
(i) O2 CHCl 3 / NaOH
Cumene (C9 H12 )  → P     
→ O
(ii) H 3O+
(i) KOH, H2O
Q (major) + R (minor), Q   → S
NaOH
(ii) H+, Heat
PhCH 2 Br
(More than One Correct Option, 2016)
CH3
O
(a) R is steam volatile
CH3 CH3
(b) Q gives dark violet colouration with 1% aqueous O
O
FeCl 3 solution
(a) (b)
(c) S gives yellow precipitate with
2, 4-dinitrophenylhydrazine
O
(d) S gives dark violet colouration with 1% aqueous O CH3
FeCl 3 solution
(c) CH3 (d)
129. List I contains reactions and List II contains
major products.
List I List II 132. Reagent(s) which can be used to bring about the
P. 1. following transformation is (are)
+
(More than One Correct Option, 2016)
ONa Br OH
Q. + HBr 2. O O O O
OMe Br
C O C
R. + NaOMe 3.
Br OMe
O H O OH
S. + MeBr 4. COOH COOH
ONa
5. O (a) LiAlH 4 in (C2H5 )2 O
(b) BH 3 in THF
Match each reaction in List I with one or more (c) NaBH 4 in C2H5OH
products in List II and choose the correct (d) Raney Ni/H 2 in THF
option. (Matching Type, 2018)
(a) P → 1, 5; Q → 2; R → 3; S → 4
(b) P → 1, 4; Q → 2; R → 4; S → 3
26. Carboxylic Acids and Their
(c) P → 1, 4; Q → 1,2; R → 3,4; S → 4 Derivatives
(d) P → 4, 5; Q → 4; R → 4; S → 3,4
133. Different possible thermal decomposition
pathways for peroxyesters are shown below.
25. Aldehydes and Ketones Match each pathway from Column t I with an
appropriate structure from Column II and
130. The major product in the following reaction is select the correct answer using the code given
(Single Correct Option, 2014)
below the lists. (Single Correct Option, 2014)
O P
Cl (i) CH3MgBr, dry ether, 0°C R· + R′O·
–CO2
(ii) Aqueous acid
CH3
O Q
O R· + R′O· R· + X·
O –CO2
R O R′ + Carbonyl compound
(a) H3C (b) H2C CH3
CH3 CH3 (Peroxyester) R
RCO· 2+ R′O· R· + X′·
OH –CO2
+ Carbonyl
CH3 compound
(c) (d) S
CH2
CH3
RCO2· + R′O· R· + R′O·
O O –CO2

THE TIME LINE


The Time Line~JEE Advanced Questions (2014 –2018) 49

Column I Column II 135. The reactions, Q to R and R to S, are


P. Pathway P 1. O (a) Aromatic sulfonation and Friedel-Crafts acylation
O (b) Friedel-Crafts alkylation and Friedel-Crafts acylation
C6H5CH2 O CH3
(c) Friedel-Crafts alkylation, dehydration and
Q. Pathway Q 2. O Friedel-Crafts acylation
O (d) Dehydration and Friedel-Crafts acylation
C6H5 O CH3

R. Pathway R 3. O 27. Aliphatic Compounds


O CH3
C6H5CH2 O
CH3
Containing Nitrogen
CH2C6H5
Paragraph (Q. Nos. 136-137)
S. Pathway S 4. O
Treatment of compound O with KMnO 4 / H + gave P,
O CH3
C6H5 O which on heating with ammonia gave Q. The
CH3 compound Q on treatment with Br2 / NaOH
C6H5 produced R. On strong heating, Q gave S, which on
further treatment with ethyl 2-bromopropanoate
Codes
in the presence of KOH followed by acidification,
P Q R S P Q R S
gave a compound T. (Paragraph Type, 2016)
(a) 1 3 4 2 (b) 2 4 3 1
(c) 4 1 2 3 (d) 3 2 1 4

Paragraph (Q. Nos. 134-135)


The reaction of compound P with CH3MgBr (excess)
in (C2H5 )2 O followed by addition of H2O gives Q.
The compound Q on treatment with H2SO4 at 0° C ( O)
gives R. The reaction of R with CH3COCl in the 136. The compound R is
presence of anhydrous AlCl 3 in CH2Cl 2 followed by O
treatment with H 2O produces compound S. [Et in NH2
compound P is ethyl group] (Paragraph Type, 2017) Br
(a) (b)
(H3C)3C Br
NH2
CO2Et Q R S
O
P O O

134. The product S is NHBr


(c) (d) NBr
COCH3 NHBr
(a) (H3C)3C CH3
O O
137. The compound T is
H3 C CH3
(b) (H3C)3C (a) glycine
(b) alanine
(c) valine
(d) serine
COCH3
HO3S 138. The order of basicity among the following
(c) (H3C)3C O CH3 compounds is (Single Correct Option, 2017)
O NH2

H3C NH2 N NH HN N H 2N NH
COCH3
(I) (II) (III) (IV)
H3COC
(a) II > I > IV > III
(d) (H3C)3C HC CH3
3 (b) I > IV > III > I
(c) IV > I > II > III
(d) IV > I > II > III

THE TIME LINE


50 The Time Line~JEE Advanced Questions (2014 –2018)

141. Among the following reactions(s), which


28. Benzene and Alkyl Benzene gives(give) tert-butyl benzene as the major
139. Match the four starting materials given in Column I product? (More than One Correct Option, 2016)
with the corresponding reaction schemes Br Cl
provided in Column II and select the correct
(a) (b)
answer using the code given below the lists. NaOC2H5 AlCl3
Column I Column II
OH
A. H H p. Scheme I
(i) KMnO 4 , HO − , heat (ii) H+, H2O (c) (d)
H2SO4 BF3,OEt2
(iii) SOCl 2 (iv) NH3
?   →
C 7H6N2O 3 142. The reaction(s) leading to the formation of
B. OH q. Scheme II 1,3,5-trimethylbenzene is (are)
(More than One Correct Option, 2018)
(i) Sn/HCl (ii) CH 3 COCl (iii) Conc.
H2 SO 4 O Conc. H2SO4
(i) HNO 3 (v) Dil. H 2 SO 4 , heat (vi) HO − (a)

OH
?  → Heated iron tube
(b) Me —— H
C 6H6N2O 2 873 K
C. NO2 r. Scheme III O
(i) Red hot iron, 873 K (1) Br2, NaOH
(ii) fuming HNO 3 , H2SO 4 , heat (2) H3O+
(c)
(iii) H2S.SH3 (iv) NaNO 2 , H 2 SO 4 (3) Sodalime, ∆
(v) hydrolyisi
O O
?  →
CHO
C 6H5NO 3
D. NO2 s. Scheme IV Zn/Hg, HCl
(d)
(i) Conc. H2SO 4 , 60°C
(ii) Conc. HNO 3 , conc. H 2 SO 4 OHC CHO
(iii) Dil. H 2 SO 4 , heat
?   →
C 6H5NO 4
29. Aromatic Compounds
CH3
Containing Nitrogen
Codes (Matching Type, 2014)
143. In the reaction shown below, the major product(s)
A B C D A B C D
formed is/are (More than One Correct Option, 2014)
(a) p s q r (b) r p s q
(c) r s q p (d) s p r q NH2
140. The major product U in the following reaction is
(More than One Correct Option, 2015) Acetic
anhydride
Radical NH2 Product(s)
CH2==CH CH 3,H + initiator, O2 CH2Cl2
T U
High pressure, Heat
O
H N
O
H CH3
O H3 C CH3
CH3 O
O
O H (a)
(a) (b) NH2 + CH3COOH

H
O
NH2
O
O CH2
CH2 (b) H
O H N CH3 + CH3COOH
O
(c) (d)
O O

THE TIME LINE


The Time Line~JEE Advanced Questions (2014 –2018) 51
H Major product(s) (Single Option Correct, 2018)
N CH3
Br Br
O
(c) H Br Br
N CH3 + H2O (a) (b)
Br Br
O O
ρ σ Br Br
NH3CH2COO (d) Br
(c) Br

(d) H
N CH3
Br Br Br Br
O O Br

144. The major product of the following reaction is 147. In the following reaction sequence, the amount
(Single Correct Option, 2017) of D (in gram) formed from 10 moles of
OH acetophenone is …….
(i) NaNO2/HCl/0°C (Atomic weights in g mol −1 :
(ii) (aq. NaOH) H = 1, C = 12, N = 14, O = 16, Br = 80. The yield
(%) corresponding to the product in each step is
NH2
given in the parenthesis)
– +
OH ONa O

NaOBr NH3, ∆ Br2/KOH Br2(3 equivalent )


(a) (b) A B C D
H3O+ AcOH
(60%) (50%) (50%) (100%)
(Numerical Value Type, 2018)
N N NH2
N N OH OH
30. Aryl Halides and Phenols
(c) (d)
148. For the identification of β-naphthol using dye test,
Cl
it is necessary to use (Single Correct Option, 2014)
(a) dichloromethane solution of β-naphthol
145. The product(s) of the following reaction
(b) acidic solution of β-naphthol
sequence is (are)
(More than One Correct Option, 2017) (c) neutral solution of β -naphthol
NH2 (d) alkaline solution of β-naphthol
(i) Acetic anhydride/Pyridine 149. The number of resonance structures for N is
(ii) KBrO3/Br (Integer Type, 2015)
OH
(iii) H3O+, Heat
(iv) NaNO2/HCl, 273-278 K NaOH
N
(v) Cu/HBr
Br Br Br Br
Br Br Br Br 31. Aromatic Aldehydes, Ketones
Br and Acids
(b) Br (c) (d) Br
150. In the following reactions, the product S is
(a) (More than One Correct Option, 2015)
146. Aniline reacts with mixed acid (conc. HNO 3 and H3C
conc. H 2SO 4 ) at 288 K to give P (51%),Q (47%) (i) O3 (i) NH3
R S
and R (2%). The major product(s) of the (ii) Zn,H2O
following sequence is (are) H3C H 3C N
(1) Ac2 O, pyridine
(1) Sn /HCl
(a) N (b)
( 2) Br2 /H 2 O (excess)
( 2) Br2 , CH 3 CO 2 H
R        → S →
(3 ) H 3 O+ (3 ) NaNO 2 , HCl /273-278 K
( 4) H 3 PO 2 N
( 4) NaNO 2 , HCl /273-278 K
(c) N (d)
(5 ) EtOH, ∆

H 3C H3C

THE TIME LINE


52 The Time Line~JEE Advanced Questions (2014 –2018)

151. Among the following the number of reaction(s) and H3C


(b) H3C
that produce(s) benzaldehyde is
(Integer Type, 2015)
CH3

CO, HCl (c) H3C and


I.
Anhydrous AlCl3 / CuCl CH3
CHCl2 H3 C
H2O CH3 CH3
II.
100°C (d) and

COCl
H3C CH3
H2
III. 154. Positive Tollen’s test is observed for
Pd-BaSO4
(More than One Correct Option, 2017)
CO2Me
OH
DIBAL-H
IV. H O CHO Ph
Toluene, -78°C
Ph O
H2O H
H O
152. The major product of the following reaction Ph Ph
sequence is (Single Correct Option, 2016) Br
(a) (b) (c) (d)
O

(i) HCHO (Excess)/NaOH, Heat


Directions (Q. Nos. 155-156) Appropriately
+
matching the information given in the three columns
(ii) HCHO/H (Catalytic amount)
of the following table.
Column 1, 2 and 3 contain starting materials,
O O O O reaction conditions, and type of reactions,
OH respectively. (Matching Type, 2017)
Column 1 Column 2 Column 3
(a) (b)
(I) Toluene (i) NaOH/Br2 (P) Condensation
O OH (II) Acetophenone (ii) Br2 /hν (Q) Carboxylation
O O
HO (III) Benzaldehyde (iii) (CH3CO)2 O / (R) Substitution
CH3COOK
(c) (d)
(IV) Phenol (iv) NaOH/CO 2 (S) Haloform

155. The only CORRECT combination in which the


OH reaction proceeds through radical mechanism is
(a) (IV) (i) (Q) (b) (III) (ii) (P) (c) (II) (iii) (R) (d) (I) (ii) (R)
153. Compound P and R upon ozonolysis produce Q
and S, respectively. The molecular formula of Q 156. For the synthesis of benzoic acid, the only
and S is C8H 8O. Q undergoes Cannizzaro CORRECT combination is
reaction but not haloform reaction, whereas S (a) (II) (i) (S) (b) (I) (iv) (Q)
undergoes haloform reaction but not (c) (IV) (ii) (P (d) (III) (iv) (R)
Cannizzaro reaction. 157. The only CORRECT combination that gives two
(More than One Correct Option, 2017)
different carboxylic acids is
(i) O3 /CH 2Cl 2 (i) O3 /CH 2Cl 2
(i) P → Q (ii) R → S (a) (IV) (iii) (Q) (b) (II) (iv) (R)
(ii) Zn/ H 2O (C 8 H 8 O) (ii) Zn/ H 2O (C 8 H 8 O) (c) (I) (i) (S) (d) (III) (iii) (P)
The option(s) with suitable combination of P Paragraph (Q.Nos. 158-159)
and R, respectively, is(are)
Treatment of benzene with CO / HCl in the presence of
H 3C CH3 anhydrous AlCl 3 / CuCl followed by reaction with
CH3 CH3 Ac2 O / NaOAc gives compound X as the major product.
(a) and Compound X upon reaction with Br2 / Na 2 CO3 followed by
CH3 heating at 473 K with moist KOH furnishes Y as the major
CH3 product. Reaction of X with H2 / Pd - C, followed by H3 PO4
treatment gives Z as the major product. (Paragraph Type, 2018)

THE TIME LINE


The Time Line~JEE Advanced Questions (2014 –2018) 53

158. The compound Y is 161. The compound S is


OH
COBr Br
(a) (b) (a) (b)
HO O

NH2
Br HN
COBr
(c) (d)
NH2 H
Br (c) (d) N
159. The compound Z is

(a) (b) 162. The desired product X can be O


prepared by reacting the major Ph
O O product of the reactions in List I OH
Me Ph
with one or more appropriate
OH X
reagents in List II. (given, order
of migratory aptitude : aryl >
(c) (d)
alkyl > hydrogen)

O List I List II

P. Ph 1. I2 , NaOH
Paragraph (Q.Nos. 160-161) HO
Me
An organic acid P ( C11 H12 O2 ) can easily be oxidised to a Ph + H2SO4
OH
dibasic acid which reacts with ethylene glycol to produce a Me
polymer dacron. Upon ozonolysis, P gives an aliphatic ketone
Q. Ph 2. [Ag(NH 3 )2 ] OH
as one of the products. P undergoes the following reaction H2N
sequences to furnish R via Q. The compound P also undergoes H
Ph + HNO2
another set of reactions to produce S . (Paragraph Type, 2018) OH
Me
(1) H2/Pd-C
(2) NH3/∆ (1) H2/Pd-C (1) HCl
R. Ph 3. Fehling solution
(3) Br2/NaOH (2) SOCl2 (2) Mg/Et2O HO
S P Q R Ph
(4) CHCl3/KOH, ∆ (3) MeMgBr,CdCl2 (3) CO2(dry ice) Me + H2SO4
(5) H2/Pd-C (4) NaBH4 (4) H3O+ OH
Me
S. Ph 4. HCHO, NaOH
160. The Compound R is Br
H
HO2C Ph + AgNO3
OH
Me
CO2H
(a) (b) 5. NaOBr
The correct option is (Matching Type, 2018)
(a) P → 1; Q → 2, 3; R → 1, 4; S → 2,4
(b) P → 1, 5; Q → 3,4; R → 4, 5; S → 3
CO2H CO2H (c) P → 1, 5; Q → 3,4; R → 5; S → 2,4
(c) (d)
(d) P → 1, 5; Q → 2, 3; R → 1,5; S → 2,3

THE TIME LINE


54 The Time Line~JEE Advanced Questions (2014 –2018)

(a) ethylene-propylene copolymer


32. Biomolecules and Chemistry (b) vulcanised rubber
in Everyday Life (c) polypropylene (d) polybutylene
163. The total number of distinct naturally occurring 166. The Fischer presentation of D-glucose is given
amino acids obtained by complete acidic below.
hydrolysis of the peptide shown below is
(Integer Type, 2014)
CHO
O H OH
H HO H
O O H O H O N
H OH
N N N H OH
N N N

N CH2 O O H CH2 O CH2OH


H
H D-glucose
O The correct structure(s) of β-L-glucopyranose is
164. For ‘invert sugar’, the correct statement(s) is (are)
(are) H H
(Given: specific rotations of ( + ) - sucrose, (+) - O O
maltose, HO OH HO OH
L-( − ) -glucose and L-( + ) -fructose in aqueous (a) CH2OH (b) CH2OH
solution are + 66 °, +140 °, − 52 ° and 92° , H HO H H
respectively) H H H H
(More than One Correct Option, 2016)
(a) Invert sugar is prepared by acid catalysed OH H OH OH
hydrolysis of maltose
CH2OH H
(b) Invert sugar is an equimolar mixture of D-(+)
O O
-glucose and D-(−)- fructose H OH HO H
(c) Specific rotation of invert sugar is − 20 ° H CH2OH
(c) (d)
(d) On reaction with Br2 water, invert sugar forms OH HO H HO
saccharic acid as one of the products HO H H OH

165. On complete hydrogenation, natural rubber H H OH H


produces (Single Correct Option, 2016)

THE TIME LINE


The Time Line~JEE Advanced Questions (2014 –2018) 55

ANSWERS
1 (8) 2 (9) 3 (2992) 4 (6.47) 5 (126) 6 (6) 7 (9) 8 (c) 9 (a) 10 (a)
11 (c,d) 12 (c) 13 (4) 14 (a,b,d) 15 (c) 16 (4) 17 (a,c) 18 (6) 19 (6) 20 (4)
21 (d) 22 (c) 23 (9) 24 (c) 25 (d) 26 (4) 27 (2.22) 28 (2) 29 (b) 30 (3)
31 (b) 32 (c) 33 (4.47) 34 (d) 35 (b) 36 (a,b,c) 37 (c) 38 (c) 39 (a,b,d) 40 (a,b,d)
41 (b,c) 42 (a,c) 43 (−14. 6) 44 (a) 45 (b,c,d) 46 (2) 47 (3) 48 (1) 49 (a,b) 50 (b,c)
51 (19) 52 (0.05) 53 (a,b) 54 (4) 55 (d) 56 (c) 57 (6) 58 (10) 59 (−11.62) 60 (b)
61 (b,c,d) 62 (a,c) 63 (a,d) 64 (8500) 65 (b,d) 66 (b,c,d) 67 (a,c) 68 (a) 69 (b,d) 70 (6)
71 (b) 72 (a,c,d) 73 (a,b,d) 74 (a) 75 (c) 76 (b,c) 77 (b,d) 78 (b) 79 (a,c,d) 80 (d)
81 (d,) 82 (a) 83 (b,c) 84 (a,b,c) 85 (6) 86 (7) 87 (a,b,c) 88 (a,b) 89 (8) 90 (6)
91 (d) 92 (b,c) 93 (1) 94 (b) 95 (4) 96 (3) 97 (6) 98 (b) 99 (5) 100 (a,c,d)
101 (a,b,d) 102 (c) 103 (b,c,d) 104 (b,c,d) 105 (a,b,c) 106 (*) 107 (6 or 7) 108 (c,d) 109 (a) 110 (a,c)
111 (b,d) 112 (a,b,c,d) 113 (3) 114 (2) 115 (a) 116 (7) 117 (b) 118 (a) 119 (d) 120 (b,d)
121 (a) 122 (b,d) 123 (d) 124 (a,b,c) 125 (b) 126 (c) 127 (4) 128 (b,c) 129 (b) 130 (d,)
131 (a) 132 (c,d) 133 (a) 134 (b) 135 (c) 136 (a) 137 (b) 138 (c) 139 (c) 140 (b)
141 (b,c,d) 142 (a,b,d) 143 (a) 144 (a) 145 (b) 146 (d) 147 (495) 148 (d) 149 (9) 150 (a)
151 (4) 152 (a) 153 (a,b) 154 (a,b,c) 155 (d) 156 (a) 157 (d) 158 (c) 159 (a) 160 (a)
161 (b) 162 (d) 163 (1) 164 (a,b,c) 165 (a) 166 (d)

(*) Answer given below


106. A → P, Q, S; B → T; C →Q; D→ R

For Detailed Solutions, Visit https://goo.gl/eV36Ww

THE TIME LINE


TIME LINE
JEE Advanced 2014-18

MATHEMATICS
and centre  −
1 1 
1. Complex Numbers (b) the circle with radius −
2a
, 0 for
 2a 
 3kπ   2kπ  a < 0, b ≠ 0
1. Let z k = cos   + i sin   ; k = 1, 2, … ,9. (c) the X-axis for a ≠ 0, b = 0
 10   10 
(d) the Y-axis for a = 0, b ≠ 0
Column I Column II
4. Let a , b, x and y be real numbers such that
P. For each zk , there exists a z j such that (i) True a − b = 1 and y ≠ 0. If the complex number
kk ⋅ z j = 1  az + b
z = x + iy satisfies Im   = y, then which
Q. There exists a k ∈ {1, 2, …, 9} such that (ii) False  z +1 
z1 ⋅ z = zk has no solution z in the set of
of the following is(are) possible value(s) of x?
complex numbers
(More than One Correct Option, 2017)
R. |1 − z1||1 − z2| …|1 − z9| equal (iii) 1
10 (a) 1 − 1 + y 2
(b) − 1 − 1 − y 2

2 kπ  (c) 1 + 1+ y2 (d) − 1 + 1− y2
∑k = 1 cos 
S. 9 (iv) 2
1−  equals
 10 
5. For a non-zero complex number z, let arg( z )
Codes (Matching Type, 2014) denote the principal argument with
P Q R S − π < arg( z ) ≤ π. Then, which of the following
(a) (i) (ii) (iv) (iii) statement(s) is (are) FALSE ?
(b) (ii) (i) (iii) (iv) (More than One Correct Option, 2018)
π
(c) (i) (ii) (iii) (iv) (a) arg (−1 − i ) = , where i = −1
(d) (ii) (i) (iv) (iii) 4
 kπ   kπ  (b) The function f : R → (− π, π], defined by
2. For any integer k, let α k = cos   + i sin   , f (t ) = arg (−1 + it ) for all t ∈ R , is continuous at all
 7  7
points of R, where i = −1.
where i = −1. The value of the expression (c) For any two non-zero complex numbers z1 and z 2 ,
z 
12 arg  1  − arg (z1) + arg (z 2 ) is an integer multiple
∑|α k + 1 − α k|  z2 
k=1 of 2 π.
3
is
∑|α 4k − 1 − α 4k − 2|
(d) For any three given distinct complex numbers z1, z 2
and z 3 , the locus of the point z satisfying the
k=1 (Integer Type, 2015)  (z − z1) (z 2 − z 3 ) 
condition arg   = π, lies on a
3. Let a , b ∈ R and a 2 + b2 ≠ 0. Suppose  (z − z 3 ) (z 2 − z1) 
 1  straight line.
S = z ∈ C : z = , t ∈ R , t ≠ 0
 a + i bt  6. Let s, t, r be non-zero complex numbers and L
where i = − 1. If z = x + iy and z ∈ S , then ( x , y ) be the set of solutions z = x + iy ( x , y ∈ R , i = − 1 )
lies on (More than One Correct Option, 2016) of the equation sz + tz + r = 0, where z = x − iy.
and centre  , 0 for
1 1
(a) the circle with radius
2a  2a 
a > 0, b ≠ 0

THE TIME LINE


The Time Line~JEE Advanced Questions (2014 –2018) 57

Then, which of the following statement(s) is


(are) TRUE? (More than One Correct Option, 2018)
3. Sequences and Series
(a) If L has exactly one element, then | s | ≠ | t | 12. Let a,b,c be positive integers such that b/ a is an
(b) If | s | = | t |, then L has infinitely many elements integer. If a,b,c are in geometric progression
(c) The number of elements in L ∩ {z : | z − 1 + i | = 5 } is and the arithmetic mean of a,b,c is b + 2, then
at most 2 a 2 + a − 14
(d) If L has more than one element, then L has infinitely the value of is
a+1 (Integer Type, 2014)
many elements
13. Suppose that all the terms of an arithmetic
progression are natural numbers. If the ratio of
2. Theory of Equations the sum of the first seven terms to the sum of
7. In the quadratic equation p( x ) = 0 with real the first eleven terms is 6 : 11 and the seventh
term lies in between 130 and 140, then the
coefficients has purely imaginary roots. Then, common difference of this AP is
the equation p[ p( x )] = 0 has (Integer Type, 2015)
(Single Correct Option, 2014)
14. Let bi > 1 for i = 1, 2, ... , 101. Suppose loge b1 ,
(a) only purely imaginary roots
loge b2 , ... , loge b101 are in AP with the common
(b) all real roots
difference loge 2 . Suppose a1 , a2 , ... , a101 are in
(c) two real and two purely imaginary roots
AP, such that a1 = b1 and a51 = b51. If
(d) neither real nor purely imaginary roots
t = b1 + b2 + ... + b51 and s = a1 + a2 + ... + a51,
8. Let S be the set of all non-zero real numbers α then (Single Correct Option, 2016)
such that the quadratic equation αx 2 − x + α = 0 (a) s > t and a101 > b101 (b) s > t and a101 < b101
has two distinct real roots x1 and x2 satisfying
(c) s < t and a101 > b101 (d) s < t and a101 < b101
the inequality|x1 − x2|< 1. Which of the
following interual (s) is/are a subset of S? 15. The sides of a right angled triangle are in
(More than One Correct Type Option, 2015) arithmetic progression. If the triangle has area
24, then what is the length of its smallest side?
(a)  − ,−
1 
(b)  −
1 1  (Integer Type, 2017)
 ,0
 2 5  5  16. Let X be the set consisting of the first 2018
(c)  0,
1 
(d) 
1 1 terms of the arithmetic progression 1, 6, 11, ....,
 , 
 5  5 2 and Y be the set consisting of the first 2018
π π terms of the arithmetic progression 9, 16, 23, ....
9. Let − < θ < − . Suppose α1 and β1 are the . Then, the number of elements in the set
6 12
roots of the equation x 2 − 2x sec θ + 1 = 0 and α 2 X ∪ Y is .................. .
and β 2 are the roots of the equation
x 2 + 2x tanθ − 1 = 0. If α1 > β1 and α 2 > β 2, then 4. Permutations and Combinations
α1 + β 2 equals 17. Let n ≥ 2 be an integer. Take n distinct points on
(Single Correct Option, 2016) a circle and join each pair of points by a line
(a) 2(secθ − tanθ) (b) 2 secθ segment. Colour the line segment joining every
(c) −2 tanθ (d) 0 pair of adjacent points by blue and the rest by
red. If the number of red and blue line segments
Paragraph (Q. Nos. 10-11) are equal, then the value of n is (Integer Type, 2014)
Let p, q be integers and let α , β be the roots of 18. Let n1 < n 2 < n3 < n 4 < n5 be positive integers
the equation, x 2 − x − 1 = 0 where α ≠ β. For such that n1 + n 2 + n3 + n 4 + n5 = 20. The
number of such distinct arrangements
n = 0, 1, 2, …… , let an = pα + qβ .
n n
( n1 , n 2 , n3 , n 4 , n5 ) is (Integer Type, 2014 )
FACT : If a and b are rational numbers and 19. Six cards and six envelope are numbered 1,2,3,4,5,6
a + b 5 = 0, then a = 0 = b. (Paragraph Type, 2017) and cards are to be placed in envelopes so that
10. a12 = each envelope contains exactly one card and no
card is placed in the envelope bearing the same
(a) a11 + 2a10 (b) 2a11 + a10
number and moreover the card number 1 is always
(c) a11 − a10 (d) a11 + a10
placed in envelope numbered 2. Then, the number
11. If a4 = 28, then p + 2q = of ways it can be done is (Single Correct Option, 2014)
(a) 14 (b) 7 (c) 21 (d) 12 (a) 264 (b) 265 (c) 53 (d) 67

THE TIME LINE


58 The Time Line~JEE Advanced Questions (2014 –2018)

20. Let n be the number of ways in which 5 boys and (iii) Let α3 be the total number of ways in which
5 girls can stand in a queue in such a way that all the committee can be formed such that the
the girls stand consecutively in the queue. Let m committee has 5 members, at least 2 of them
be the number of ways in which 5 boys and 5 girls being girls.
can stand in a queue in such a way that exactly (iv) Let α 4 be the total number of ways in which
four girls stand consecutively in the queue. Then, the committee can be formed such that the
the value of m / n is (Integer Type, 2015) committee has 4 members, having at least 2
21. A debate club consists of 6 girls and 4 boys. A girls such that both M1 and G1 are NOT in the
team of 4 members is to be selected from this committee together.
club including the selection of a captain (from Column I Column II
among these 4 members) for the team. If the
team has to include atmost one boy, the number P. The value of α1 is 1. 136
of ways of selecting the team is Q. The value of α 2 is 2. 189
(Single Correct Option, 2016)
R. The value of α 3 is 3. 192
(a) 380 (b) 320 (c) 260 (d) 95
S. The value of α 4 is 4. 200
22. Words of length 10 are formed using the letters
A, B, C, D, E, F, G, H, I, J. Let x be the number of 5. 381
such words where no letter is repeated; and let y 6. 461
be the number of such words where exactly one
letter is repeated twice and no other letter is The correct option is (Matching Type, 2018)
repeated. Then, y / 9x = (Integer Type, 2017) (a) P → 4; Q → 6; R → 2; S → 1
23. Let S = { 1, 2, 3, …… , 9}. For k = 1, 2 , …… 5, let N k (b) P → 1; Q → 4; R → 2; S → 3
(c) P → 4; Q → 6; R → 5; S → 2
be the number of subsets of S, each containing (d) P → 4; Q → 2; R → 3; S → 1
five elements out of which exactly k are odd.
Then, N 1 + N 2 + N 3 + N 4 + N 5 =
(Single Correct Option, 2017) 5. Binomial Theorem
(a) 210 (b) 252 (c) 126 (d) 125 28. Coefficient of x11 in the expansion of (1 + x 2 )4
24. How many 3 × 3 matrices M with entries from (1 + x3 )7 (1 + x 4 )12 is (Single Correct Option, 2014)
{0, 1, 2} are there, for which the sum of the (a) 1051 (b) 1106
diagonal entries of M T M is 5 ? (c) 1113 (d) 1120
(Single Correct Option, 2017)
29. The coefficient of x 9 in the expansion of
(a) 198 (b) 162 (c) 126 (d) 135 (1 + x ) (1 + x 2 ) (1 + x3 ) ... (1 + x100 ) is
25. The number of 5 digit numbers which are (Integer Type, 2015)
divisible by 4, with digits from the set 30. Let m be the smallest positive integer such that
{ 1, 2, 3, 4, 5} and the repetition of digits is the coefficient of x 2 in the expansion of
allowed, is ..................... . (Integer Type, 2018)
(1 + x )2 + (1 + x )3 + K + (1 + x )49 + (1 + mx )50is
26. Let X be a set with exactly 5 elements and Y be ( 3n + 1) 51C3 for some positive integer n. Then,
a set with exactly 7 elements. If α is the number
the value of n is (Integer Type, 2016)
of one-one functions from X to Y and β is the
number of onto functions from Y to X, then the 31. Let X = (10C1 )2 + 2(10C2 )2 + 3(10C3 )2
1
value of ( β − α) is ......... . + ... + 10(10C10 )2, where 10Cr , r ∈{ 1, 2,... , 10}
5! (Integer Type, 2018) denote binomial coefficients. Then, the value of
27. In a high school, a committee has to be formed 1
X is .......... .
from a group of 6 boys M1, M 2, M3 , M 4, M5 , M 6 1430 (Integer Type, 2018)
and 5 girls G1, G2, G3 , G4, G5 .
(i) Let α1 be the total number of ways in which 6. Probability
the committee can be formed such that the
committee has 5 members, having exactly 3 32. Three boys and two girls stand in a queue. The
boys and 2 girls. probability that the number of boys a head of
(ii) Let α 2 be the total number of ways in which the every girl is atleast one more that the number
committee can be formed such that the of girls ahead of her, is (Single Correct Option, 2014 )
committee has at least 2 members, and (a) 1 / 2 (b) 1 / 3
having an equal number of boys and girls. (c) 2 / 3 (d) 3 / 4

THE TIME LINE


The Time Line~JEE Advanced Questions (2014 –2018) 59

Paragraph (Q. Nos. 33-34) produced in plant T2), where P ( E ) denotes the
probability of an event E.
Box I contains three cards bearing numbers 1, 2, 3 ; A computer produced in the factory is randomly
box II contains five cards bearing
selected and it does not turn out to be defective.
numbers 1, 2, 3, 4, 5; and box III contains seven
Then, the probability that it is produced in
cards bearing numbers
plant
1, 2, 3, 4, 5, 6, 7. A card is drawn from each of the
boxes. Let xi be the number on the card drawn T2, is (Single Correct Option, 2016)
36 47 78 75
from the i th box i = 1, 2, 3. (Paragraph Type, 2014) (a) (b) (c) (d)
73 79 93 83
33. The probability that x1 + x2 + x3 is odd, is
29 53 57 1
(a) (b) (c) (d) Paragraph (Q. Nos. 39-40)
105 105 105 2
Football teams T1 and T2 have to play two games
34. The probability that x1 , x2 and x3 are in an against each other. It is assumed that the
arithmetic progression, is outcomes of the two games are independent. The
9 10 11 7 probabilities of T1 winning, drawing and losing a
(a) (b) (c) (d)
105 105 105 105 1 1 1
game against T2 are , and , respectively. Each
2 6 3
Paragraph (Q. Nos. 35-36) team gets 3 points for a win, 1 point for a draw
and 0 point for a loss in a game. Let X and Y
Let n1 and n2 be the number of red and black
denote the total points scored by teams T1 and T2,
balls, respectively in box I. Let n3 and n4 be the
respectively, after two games. (Paragraph Type, 2016)
number of red and black balls, respectively in
box II. (Paragraph Type, 2015) 39. P ( X > Y ) is
1 5 1 7
35. One of the two boxes, box I and box II was (a) (b) (c) (d)
4 12 2 12
selected at random and a ball was drawn
randomly out of this box. The ball was found to 40. P ( X = Y ) is
be red. If the probability that this red ball was 11 1 13 1
(a) (b) (c) (d)
drawn from box II, is 1 / 3, then the correct 36 3 36 2
option(s) with the possible values of n1 , n 2, n3
1
and n 4 is/are 41. Let X and Y be two events such that P ( X ) = ,
(a) n1 = 3, n2 = 3, n3 = 5, n4 = 15 3
1 2
(b) n1 = 3, n2 = 6, n3 = 10, n4 = 50 P( X / Y ) = and P (Y / X ) = . Then
2 5
(c) n1 = 8, n2 = 6, n3 = 5, n4 = 20 (More than One Correct Option, 2017)
(d) n1 = 6, n2 = 12, n3 = 5, n4 = 20 (a) P (Y ) =
4
(b) P (X ′/Y ) =
1
15 2
36. A ball is drawn at random from box I and 2 1
transferred to box II. If the probability of (c) P (X ∪Y ) = (d) P (X ∩ Y ) =
5 5
drawing a red ball from box I, after this
1 42. Three randomly chosen nonnegative integers x,
transfer, is , then the correct option(s) with
3 y and z are found to satisfy the equation
the possible values of n1 and n 2 is/are x + y + z = 10. Then the probability that z is
(a) n1 = 4 and n2 = 6 (b) n1 = 2 and n2 = 3 even, is (Single Correct Option, 2017)
1 36 6 5
(c) n1 = 10 and n2 = 20 (d) n1 = 3 and n2 = 6 (a) (b) (c) (d)
2 55 11 11
37. The minimum number of times a fair coin
needs to be tossed, so that the probability of
getting atleast two heads is atleast 0.96, is
Paragraph (Q. Nos. 43-44)
(Integer Type, 2015) There are five students S1 , S 2, S3 , S 4 and S5 in a
38. A computer producing factory has only two music class and for them there are five seats
plants T1 and T2. Plant T1 produces 20% and R1 , R2, R3 , R4and R5 arranged in a row, where
initially the seat Ri is allotted to the student
plant T2 produces 80% of the total computers
Si , i = 1, 2, 3, 4, 5. But, on the examination day, the
produced. 7% of computers produced in the
five students are randomly allotted the five seats.
factory turn out to be defective. It is known that
P(computer turns out to be defective, given that (There are two questions based on Paragraph “A”,
it is produced in plant T1) = 10P (computer the question given below is one of them)
turns out to be defective, given that it is

THE TIME LINE


60 The Time Line~JEE Advanced Questions (2014 –2018)

43. The probability that, on the examination day, 1 0 0


the student S1 gets the previously allotted seat 49. Let P =  4 1 0 and I be the identity matrix
R1, and NONE of the remaining students gets  
the seat previously allotted to him/her is 16 4 1
(Paragraph Type, 2018) of order 3. If Q = [qij ] is a matrix, such that
3 1 7 1
(a) (b) (c) (d)
40 8 40 5 q31 + q32
P 50 − Q = I , then equals
44. For i = 1, 2, 3, 4, let Ti denote the event that the q21
students S i and S i +1 do NOT sit adjacent to (Single Correct Option, 2016)
each other on the day of the examination. Then, (a) 52 (b) 103 (c) 201 (d) 205
the probability of the event T1 ∩ T2 ∩ T3 ∩ T4 is
(Paragraph Type, 2018) −1 + 3i
50. Let z = , where i = −1, and
1 1 2
(a) (b)
15 10 r , s ∈ { 1, 2, 3}.
7 1
(c) (d) ( − z )r z 2s 
60 5 Let P =  2s  and I be the identity
 z zr 
matrix of order 2. Then, the total number of
7. Matrices and Determinants ordered pairs (r , s) for which P 2 = − I is
45. Let M and N be two 3 × 3 matrices such that (Integer Type, 2016)
MN = NM . Further, if M ≠ N 2 and M 2 = N 4, 51. The total number of distincts x ∈ R for which
then (More than One Correct Option, 2014) x x2 1 + x3
(a) determinant of (M + MN ) is 0
2 2
2x 4x 2 1 + 8x3 = 10 is
(b) there is a 3 × 3 non-zero matrix U such that
(M 2 + MN 2 ) U is zero matrix 3x 9x 2 1 + 27x3
(Integer Type, 2016)
(c) determinant of (M 2 + MN 2 ) ≥ 1
(d) for a 3 × 3 matrix U, if (M 2 + MN 2 ) U equals the zero 3 −1 −2
matrix, then U is the zero matrix 52. Let P = 2 0 α , where α ∈ R. Suppose
 
46. Let M be a 2 × 2 symmetric matrix with integer 3 −5 0 
entries. Then, M is invertible, if Q = [qij ] is a matrix such that PQ = kI , where
(More than One Correct Option, 2014)
(a) the first column of M is the transpose of the second k ∈ R , k ≠ 0 and I is the identity matrix of order 3.
row of M k k2
(b) the second row of M is the transpose of the first If q23 = − and det (Q ) = , then
8 2
column of M (More than One Correct Option, 2016)
(c) M is a diagonal matrix with non-zero entries in the (a) α = 0, k = 8 (b) 4α − k + 8 = 0
main digonal
(c) det (P adj (Q)) = 2 9 (d) det (Q adj (P )) = 213
(d) the product of entries in the main diagonal of M is
not the square of an integer 53. Which of the following is(are) NOT the square
of a 3 × 3 matrix with real entries?
47. Let X and Y be two arbitrary, 3 × 3, non-zero, (Single Correct Option, 2017)
skew-symmetric matrices and Z be an 1 0 0  1 0 0
arbitrary, 3 × 3, non-zero, symmetric matrix.
(a)  0 1 0  (b)  0 −1 0 
Then, which of the following matrices is/are    
skew-symmetric?  0 0 −1  0 0 −1
(More than One Correct Option, 2015) − 1 0 0  1 0 0
(a)Y 3 Z 4 − Z 4Y 3 (b) X 44 + Y 44 (c)  0 − 1 0  (d)  0 1 0
   
(c) X 4 Z 3 − Z 3 X 4 (d) X 23 + Y 23  0 0 −1  0 0 1
48. Which of the following values of α satisfy the 54. For a real number α, if the system
(1 + α )2 (1 + 2α )2 (1 + 3α )2  1 α α 2  x   1 
equation ( 2 + α )2 ( 2 + 2α )2 ( 2 + 3α )2 = − 648 α ?     
 α 1 α   y = −1
( 3 + α )2 ( 3 + 2α )2 ( 3 + 3α )2 α 2 α 1   z   1 
 
(a) −4 (b) 9 (c) −9 (d) 4 of linear equations, has infinitely many
(More than One Correct Option, 2015) solutions, then 1 + α + α 2 = (Integer Type, 2017)

THE TIME LINE


The Time Line~JEE Advanced Questions (2014 –2018) 61

55. Let P be a matrix of order 3 × 3 such that all


the entries in P are from the set { − 1, 0, 1}. π π 
58. Let f ( x ) = sin  sin sin x  for all x ∈ R and
Then, the maximum possible value of the 6 2 
π
determinant of P is ......... . (Integer Type, 2018) g( x ) = sin x for all x ∈ R. Let ( fog)( x ) denotes
2
 b1 
f { g( x )} and (gof ) (x) denotes g{f (x)}. Then,
56. Let S be the set of all column matrices b2 such
  which of the following is/are true?
b3  (More than One Correct Option, 2015)

(a) Range of f is  − , 
that b1, b2, b3 ∈R and the system of equations 1 1
 2 2 
(in real variables)
(b) Range of fog is  − , 
1 1
 2 2 
− x + 2 y + 5z = b1
f (x) π
2x − 4 y + 3z = b2 (c) lim =
x → 0 g (x) 6
x − 2 y + 2z = b3 (d) There is an x ∈ R such that (gof ) (x) = 1
has at least one solution. Then, which of the 59. Let E = { x ∈ R : x ≠ 1 and x
following system(s) (in real variables) has 1 > 0} and
x−1
 b1 
   x  
(have) at least one solution for each b2 ∈S ? E2 =  x ∈ E1 : sin− 1  loge    is a real number
    x − 1 
b3   
(More than One Correct Option, 2018)
(a) x + 2 y + 3 z = b1, 4 y + 5 z = b2 and x + 2 y + 6z = b3 (Here, the inverse trigonometric function
 π π
(b) x + y + 3 z = b1, 5 x + 2 y + 6z = b2 and sin− 1 x assumes values in − , .).Let
− 2 x − y − 3 z = b3  2 2
(c) − x + 2 y − 5 z = b1, 2 x − 4 y + 10z = b2 f : E1 → R be the function defined by
and x − 2 y + 5 z = b3  x 
f ( x ) = loge   and g : E2 → R be the function
(d) x + 2 y + 5 z = b1, 2 x + 3 z = b2 and x + 4 y − 5 z = b3  x − 1
  x 
defined by g( x ) = sin− 1  loge  .
8. Functions   x − 1 

Match the conditions/expressions in Column I Column I Column II


with statement in Column II.
 1   e 
57. Let f1 : R → R , f2 : [0, ∞ ] → R , f3 : R → R and  − ∞, ∪ , ∞
1− e   e − 1 
P. The range of f is 1.
f4 : R → [0, ∞ ) be defined by 

|x|, if x < 0 Q. The range of g contains 2. (0,1)


f1( x ) =  x ; f2( x ) = x 2 ;
 e , if x ≥ 0 − 1 , 1 
The domain of f
R. 3.  2 2 
 sin x , if x < 0 contains
f3 ( x ) = 
 x , if x ≥ 0 S. The domain of g is 4. (− ∞, 0) ∪ (0, ∞ )
 f [ f ( x )], if x < 0 5.  e 
and f4( x ) =  2 1  − ∞,
 f2 [ f1( x )] − 1, if x ≥ 0  e − 1

Column I Column II 6. 1 e 
(− ∞, 0) ∪  , 
P. f4 is 1. onto but not one-one  2 e − 1
Q. f3 is 2. neither continuous nor one-one (Matching Type, 2018)
R. f2 of1 is 3. differentiable but not one-one The correct option is
S. f2 is 4. continuous and one-one (a) P → 4; Q → 2; R → 1; S → 1
(b) P → 3; Q → 3; R → 6; S → 5
Codes (Matching Type, 2014)
(c) P → 4; Q → 2; R → 1; S → 6
P Q R S P Q R S
(a) 3 1 4 2 (b) 1 3 4 2 (d) P → 4; Q → 3; R → 6; S → 5
(c) 3 1 2 4 (d) 1 3 2 4

THE TIME LINE


62 The Time Line~JEE Advanced Questions (2014 –2018)

66. Let m and n be two positive integers greater


9. Limits, Continuity  ecos ( α n ) − e
than 1. If lim   = −  e  , then the
and Differentiability α→ 0
 α m 
  2
60. Let f : [a , b] → [1, ∞ ) be a continuous function m
and g : R → R be defined as value of is
n (Integer Type, 2015)

0 , if x< a 67. Let g : R → R be a differentiable function with
 x

g( x ) =  ∫ f ( t )dt , if a ≤ x ≤ b . g( 0) = 0, g′ ( 0) = 0 and g′ (1) =/ 0.
a
 b (More than One Correct Option, 2015)

 ∫a  x
f ( t )dt , if x> b
 g( x ), x =/ 0
Let f ( x ) = |x|
Then, (More than One Correct Option, 2014)
 0 , x=0
(a) g (x) is continuous but not differentiable at a
(b) g (x) is differentiable on R and h( x ) = e|x| for all x ∈ R. Let ( foh ) ( x )
(c) g (x) is continuous but not differentiable at b denotes f { h( x )} and ( hof )( x ) denotes h { f ( x )}.
(d) g (x) is continuous and differentiable at either a or b Then, which of the following is/are true?
but not both
(a) f is differentiable at x = 0
(b) h is differentiable at x = 0
Paragraph (Q. Nos. 61-62)
1−h −a
(c) foh is differentiable at x = 0
Given that for each a ∈ (0, 1), lim ∫h t (1 − t )a−1dt (d) h of is differentiable at x = 0
h→ 0 +
exists. Let this limit be g (a ). In addition, it is 68. Let f, g : [−1 , 2] → R be continuous functions
given that the function g (a ) is differentiable on which are twice differentiable on the interval
(0, 1). (Paragraph Type, 2014) ( −1, 2). Let the values of f and g at the points
 1 −1, 0 and 2 be as given in the following table:
61. The value of g   is
 2 x = −1 x =0 x =2
(a) π (b) 2 π (c) π / 2 (d) π / 4 f ( x) 3 6 0

62. The value of g′ (1/ 2) is g ( x) 0 1 −1


π π In each of the intervals ( −1, 0) and ( 0, 2), the
(a) (b) π (c) − (d) 0
2 2 function ( f − 3g)″ never vanishes. Then, the
63. The largest value of the non-negative integer a correct statement(s) is/are
1− x (More then One Correct Option, 2015)
 − ax + sin( x − 1) + a  1− 1 (a) f ′ (x) − 3g ′ (x) = 0 has exactly three solutions in
for which lim  
x
= is
x → 1  x + sin( x − 1) − 1  4 (−1, 0) ∪ (0, 2)
(Integer Type, 2014) (b) f ′ (x) − 3g ′ (x) = 0 has exactly one solution in (−1 , 0)
64. For every pair of continuous function (c) f ′ (x) − 3g ′ (x) = 0 has exactly one solution in (0, 2)
f , g : [0, 1] → R such that max { f ( x ): x ∈ [0, 1]} (d) f ′ (x) − 3g ′ (x) = 0 has exactly two solutions in (−1, 0)
= max { g( x ): x ∈ [0,1]}. The correct statement(s) is and exactly two solutions in (0, 2)
(are) (More than One Correct Option, 2014) 69. Let f : R → R be a continuous odd function,
(a) [f (c)]2 + 3f (c) = [g (c)]2 + 3g (c) for some c ∈[0,1] 1
which vanishes exactly at one point and f(1) = .
(b) [f (c)] + f (c) = [g (c)] + 3g (c) for some c ∈[0,1]
2 2 2
x
Suppose that F (x) = ∫ f (t ) dt for all x ∈ [−1 , 2] and
(c) [f (c)]2 + 3f (c) = [g (c)]2 + g (c) for some c ∈[0,1] –1
x
(d) [f (c)] = [g (c)] for some c ∈[0,1]
2 2
G (x) = ∫ t| f { f (t )}| dt for all x ∈ [−1 , 2]. If
–1
65. Let f : R → R and g : R → R be respectively given lim
F (x) 1  1
= , then the value of f   is
by f ( x ) =|x|+ 1 and g( x ) = x 2 + 1. Define h : R → R x→1 G (x) 14  2
 max{ f ( x ), g( x )}, if x ≤ 0 . (Integer Type, 2015)
by h( x ) = 
 min{ f ( x ), g( x )}, if x > 0 . x 2 sin (βx )
70. Let α , β ∈ R be such that lim =1.
The number of points at which h( x ) is not x → 0 αx − sin x
differentiable is (Integer Type, 2014) Then, 6 (α + β ) equals (Integer Type, 2016)

THE TIME LINE


The Time Line~JEE Advanced Questions (2014 –2018) 63

71. Let x
76. Let f : R → R be a differentiable function such
 n  π
 n  n that f ( 0) = 0, f   = 3 and f ′ ( 0) = 1.
 n n ( x + n )  x +  ...  x +    2
   n 
f ( x ) = lim 
2 π
,
n→ ∞ 
2  2
 n !( x + n ) x +
2 n   2 n2 
2
 ...  x + 2  
If g( x ) = ∫
x
2 [ f ′ ( t ) cosec t − cot t cosec t f ( t )] dt

  4  n    π
for x ∈  0, , then lim g( x ) =
for all x = 0. Then,  2 x→ 0 (Integer Type, 2017)
(More then One Correct Option, 2016)
77. Let [x ] be the greatest integer less than or
(a) f   ≥ f (1) (b) f   ≤ f  
1 1 2
 2  3  3 equals to x. Then, at which of the following
f ′ (3) f ′ (2) point(s) the function f ( x ) = x cos ( π( x + [x ])) is
(c) f ′ (2) ≤ 0 (d) ≥ discontinuous? (More then One Correct Option, 2017)
f (3) f (2)
(a) x = − 1 (b) x = 1 (c) x = 0 (d) x = 2
72. Let a , b ∈ R and f : R → R be defined by
78. Let f : R → R and g : R → R be two non-constant
f ( x ) = a cos (|x3 − x|) + b|x| sin(|x3 + x|). Then,
differentiable If f ′ ( x ) = ( e( f ( x ) − g ( x )) ) g′ ( x ) for all
f is (More than One Correct Option, 2016)
x ∈ R and f (1) = g( 2) = 1, then which of the
(a) differentiable at x = 0, if a = 0 and b = 1 following statement(s) is (are) TRUE?
(b) differentiable at x = 1, if a = 1 and b = 0 (More than One Correct Option, 2018)
(c) not differentiable at x = 0, if a = 1and b = 0 (a) f (2) < 1 − loge 2 (b) f (2) > 1 − loge 2
(d) not differentiable at x = 1, if a = 1and b = 1 (c) g (1) > 1 − loge 2 (d) g (1) < 1 − loge 2
73. Let f : R → R , g : R → R and h : R → R be 79. Let f : ( 0, π) → R be a twice differentiable
differentiable functions such that f ( x ) sin t − f ( t ) sin x
function such that lim
f ( x ) = x3 + 3x + 2, g( f ( x )) = x and h ( g( g( x ))) = x t→ x t−x
for all x ∈ R. Then,  π π
= sin x for all x ∈( 0, π). If f   = − , then
2

(More than One Correct Option, 2016)  6 12


1 which of the following statement(s) is (are)
(a) g ′(2) = (b) h ′(1) = 666 TRUE? (More than One Correct Option, 2018)
15
π π x4
(c) h(0) = 16 (d) h (g (3)) = 36 (a) f   = (b) f (x)< − x 2 for all x ∈ (0, π)
 4 4 2 6
 1   1 
74. Let f : − , 2 → R and g : − , 2 → R be (c) There exists α ∈(0, π) such that f ′ (α) = 0
 2   2  π π
functions defined by f ( x ) = [x − 3] and2 (d) f ′′  + f   = 0
 2  2
g( x ) =|x| f ( x ) +|4x − 7| f ( x ), where [ y ] denotes the
80. Let f : R → R, f :  − π , π  → R, f : ( − 1, eπ / 2 − 2)
greatest integer less than or equal to y for 2  
y ∈ R. Then, (More than One Correct Option, 2016)
1
 2 2 3

→ R and f4 : R → R be functions defined by


(a) f is discontinuous exactly at three points in  − , 2 
1
 2  2
(i) f1( x ) = sin( 1 − e− x ),
(b) f is discontinuous exactly at four points in  − , 2 
1
 2   |sin x|
 if x ≠ 0
(ii) f2( x ) =  tan− 1 x , where the inverse
(c)g is not differentiable exactly at four points in  − , 2 
1
 2   1 if x = 0
(d) g is not differentiable exactly at five points in  − , 2 
1 trigonometric function tan− 1 x assumes
 2   π π
values in  − ,  ,
1 − x (1 +|1 − x|)  1   2 2
75. Let f ( x ) = cos  
|1 − x|  1 − x (iii) f3 ( x ) = [sin(loge ( x + 2))], where for t ∈ R, [t ]
(More than One Correct option, 2017) denotes the greatest integer less than or
for x ≠ 1. Then equal
(a) limx → 1 + f (x) = 0
to t,
 2  1
(b) limx → 1− f (x) does not exist  x sin   if x ≠ 0
(iv) f4( x ) =   x
(c) limx → 1− f (x) = 0
 0 if x = 0
(d) limx → 1 + f (x) does not exist

THE TIME LINE


64 The Time Line~JEE Advanced Questions (2014 –2018)

Column I Column II minimum, when the inner radius of the


container is 10 mm, then the value of V / 250π is
P. The function f1 is 1. NOT continuous at x = 0 (Integer Type, 2015)
Q. The function f2 is 2. continuous at x = 0 and NOT
86. The least value of α ∈ R for which 4αx 2 + 1 / x ≥ 1,
differentiable at x = 0
for all x > 0, is (Single Correct Option, 2016)
R. The function f3 is 3. differentiable at x = 0 and its 1 1 1 1
derivative is NOT continuous (a) (b) (c) (d)
64 32 27 25
at x = 0
87. Let f : R → ( 0, ∞ ) and g : R → R be twice
S. The function f4 is 4. differentiable at x = 0 and its
differentiable functions such that f ′ ′ and g′ ′
derivative is continuous at
are continuous functions on R. Suppose
x= 0
f ′ ( 2) = g( 2) = 0, f ′ ′( 2) ≠ 0 and g′ ( 2) ≠ 0.
(Matching Type, 2018) f ( x )g( x )
If lim = 1, then
The correct option is x → 2 f ′ ( x ) g′ ( x )
(a) P → 2; Q → 3; R → 1; S → 4 (More than One Correct Option, 2016)
(b) P → 4; Q → 1; R → 2; S → 3 (a) f has a local minimum at x = 2
(c) P → 4; Q → 2; R → 1; S → 3 (b) f has a local maximum at x = 2
(c) f ′ ′ (2) > f (2)
(d) P → 2; Q → 1; R → 4; S → 3
(d) f (x) − f ′ ′ (x) = 0, for atleast one x ∈ R
88. If f : R → R is a twice differentiable function
10. Application of Derivatives  1 1
such that f ′′( x ) > 0 for all x ∈ R, and f   = ,
81. Let a ∈ R and f : R → R be given by  2 2
f ( x ) = x5 − 5x + a , f(1) = 1, then (Single Correct Option Type, 2017)

Then, (More than One Correct Option, 2014) (a) f ′ (1) ≤ 0 (b) f ′ (1) > 1
(a) f (x) has three real roots, if a > 4 1 1
(c) 0 < f ′ (1) ≤ (d) < f ′ (1) ≤ 1
(b) f (x) has only one real root, if a > 4 2 2
(c) f (x) has three real roots, if a < − 4
89. If f : R → R is a differentiable function such
(d) f (x) has three real roots, if −4 < a < 4
that f ′ ( x ) > 2 f ( x ) for all x ∈ R and f( 0) = 1, then
 π π (More then One Correct Option, 2017)
82. Let f :  − ,  → R be given by
 2 2 (a) f (x) > e 2 x in (0, ∞) (b) f ′ (x) < e 2 x in (0, ∞)
f ( x ) = [log(sec x + tan x )]3 . Then, (c) f (x) is increasing in (0, ∞) (d) f (x) is decreasing in (0 ∞)
(More than One Correct Option, 2014)
(a) f (x) is an odd function (b) f (x) is a one-one function Paragraph (Q.Nos. 90-92)
(c) f (x) is an onto function (d) f (x) is an even function By appropriately matching the information given in
83. The slope of the tangent to the curve the three columns of the following table.
( y − x5 )2 = x (1 + x 2 )2 at the point (1, 3) is Let f (x) = x + log e x − x log e x, x ∈ (0, ∞ )
(Integer Type, 2014) Column 1 contains information about zeros of
 1 f (x), f ′ (x) and f ′′ (x).
− t+ 
x  t dt
84. If f :( 0, ∞ ) → R be given by f ( x ) = ∫1/ xe t
. Column 2 contains information about the limiting
behaviour of f (x), f ′ (x) and f ′′ (x) at infinity.
Then, (More than One Correct Option, 2014)
Column 3 contains information about increasing/
(a) f (x) is monotonically increasing on [1, ∞) decreasing nature of f (x) and f ′ (x).
(b) f (x) is monotonically decreasing on [0,1)
Column 1 Column 2 Column 3
(c) f (x) + f   = 0, ∀x ∈ (0, ∞)
1
x (I) f( x) = 0 for some (i) lim f( x) = 0 (P) f is increasing in
x→ ∞
(d) f (2 x ) is an odd function of x on R x ∈(1, e 2 ) (0, 1)

85. A cylindrical container is to be made from certain (II) f ′( x) = 0 for (ii) lim f( x) = − ∞ (Q) f is decreasing
x→ ∞
solid material with the following constraints : It some x ∈(1, e ) in (e, e 2 )
has a fixed inner volume of V mm3 , has a 2 mm (III) f ′( x) = 0 for (iii) lim f ′( x) = − ∞ (R) f′ is increasing
thick solid wall and is open at the top. The some x ∈(0, 1)
x→ ∞
in (0, 1)
bottom of the container is a solid circular disc of
thickness 2 mm and is of radius equal to the (IV) f ′′( x) = 0 for (iv) lim f ′′( x) = 0 (S) f′ is decreasing
x→ ∞
outer radius of the container. If the volume of the some x ∈(1, e ) in (e, e 2 )
material used to make the container is (Matching Type, 2017)

THE TIME LINE


The Time Line~JEE Advanced Questions (2014 –2018) 65

96. The integral π / 2( 2 cosec x )17 dx is equal to


90. Which of the following options is the only
INCORRECT combination?
∫ π /4
(Single Correct Option, 2014)
(a) (I) (iii) (P) (b) (II) (iv) (Q) (c) (II) (iii) (P) (d) (III) (i) (R) log(1+ 2 ) log(1+ 2 )
(a) ∫0 2 (eu + e −u )16 du (b) ∫0 (eu + e −u )17 du
91. Which of the following options is the only
CORRECT combination? log(1+ 2 ) log(1+ 2 )
(c) ∫0 (eu − e −u )17 du (d) ∫0 2 (eu − e −u )16 du
(a) (I) (ii) (R) (b) (III) (iv) P) (c) (II) (iii) (S) (d) (IV) (i) (S)
1  d2 
92. Which of the following options is the only 97. The value of ∫ 4x3  (1 − x 2 )5  dx is
2
0
CORRECT combination?  dx 
(Integer Type, 2014)
(a) (III) (iii) (R)(b) (IV) (iv) (S)(c) (II) (ii) (Q) (d) (I) (i) (P)
93. cos( 2x ) cos( 2x ) sin( 2x ) 98. Let f : [0, 2] → R be a function which is continuous
If f ( x ) =− cos x cos x − sin x , then on [0, 2] and is differentiable on ( 0, 2) with f( 0) = 1.
  x2
 sin x sin x cos x  Let F ( x ) = ∫0 f ( t ) dt, for x ∈ [0, 2]. If F ′ ( x ) = f ′ ( x ),
(More than One Correct Option, 2017)
∀ x ∈( 0, 2), then F( 2) equals
(a) f (x) attains its minimum at x = 0 (Single Correct Option, 2014)
(b) f (x) attains its maximum at x = 0
(a) e 2 − 1 (b) e 4 − 1 (c) e − 1 (d) e 4
(c) f ′ (x) = 0 at more than three points in (− π, π)
(d) f ′ (x) = 0 at exactly three points in (− π, π)
Paragraph (Q. Nos. 99-100)
94. For every twice differentiable function
Let F : R → R be a thrice differentiable function.
f : R → [−2, 2] with ( f ( 0))2 + ( f ′ ( 0))2 = 85, which
Suppose that F (1) = 0, F (3) = − 4 and F ′ (x) < 0 for all
of the following statement(s) is (are) TRUE ? x ∈ (1, 3). Let f (x) = xF (x) for all x ∈ R.
(More than One Correct Option, 2018) (Paragraph Type, 2014)
(a) There exist r , s ∈ R , where r < s, such that f is
99. The correct statement(s) is/are
one-one on the open interval (r , s)
(b) There exists x 0 ∈ (−4, 0) such that | f ′ (x 0 )| ≤ 1 (a) f ′ (1) < 0
(c) lim f (x) = 1 (b) f (2) < 0
x→ ∞
(c) f ′ (x) =/ 0 for any x ∈ (1, 3)
(d) There exists α ∈ (−4, 4) such that f (α) + f ′′(α) = 0 and
(d) f ′ (x) = 0 for some x ∈(1, 3)
f ′ (α) ≠ 0
100. If 3 x 2 F ′ ( x ) dx = − 12 and 3
∫ ∫1 x F ′ ′ ( x ) dx = 40,
3
1
12. Definite Integration then the correct expression(s) is/are
3
95. Match the following (a) 9f ′ (3) + f ′ (1) − 32 = 0 (b) ∫ f (x) dx = 12
1
3
Column I Column II (c) 9f ′ (3) – f ′ (1) + 32 = 0 (d) ∫ f (x) dx = − 12
1
P. The number of polynomials f( x) with 1. 8
non-negative integer coefficients of 101. Let f ( x ) = 7 tan x + 7 tan x – 3 tan4 x – 3 tan2 x
8 6

degree ≤ 2, satisfying f(0) = 0 and  π π


1 for all x ∈  – ,  . Then, the correct expression(s)
∫0 f( x)dx = 1, is  2 2
Q. The number of points in the interval 2. 2 is/are (More Than One Correct Option, 2015)
[− 13, 13 ] at which π /4 1 π /4
f( x) = sin( x2 ) + cos( x2 ) attains its (a) ∫ x f (x) dx = (b) ∫ f (x) dx = 0
0 12 0
maximum value, is π /4 1 π /4
(c) ∫ x f (x) dx = (d) ∫ f (x) dx = 1
R. 2 3 x2 3. 4 0 6 0
∫−2 (1 + e x )dx equals
192x3  1
S.   1 + x  dx
102. Let f ′ ( x ) = for all x ∈ R withf   = 0.
1/ 2
 ∫−1/ 2 cos 2 xlog  
4. 0 2 + sin πx 4  2
  1− x   1
 1/ 2
 ∫0 cos 2 xlog
 1 + x  dx
 
equals If m ≤ ∫l/ 2 f ( x ) dx ≤ M , then the possible values of
  1− x   m and M are (More than One Correct Option, 2015)
1 1
Codes (Matching Type, 2014) (a) m = 13, M = 24 (b) m = ,M =
4 2
P Q R S P Q R S
(c) m = − 11, M = 0 (d) m = 1, M = 12
(a) 3 2 4 1 (b) 2 3 4 1
(c) 3 2 1 4 (d) 2 3 1 4

THE TIME LINE


66 The Time Line~JEE Advanced Questions (2014 –2018)

103. The option(s) with the values of a and L that 111. For each positive integer n, let
4π 1
∫ e (sin at + cos at)dt = L,
t 6 4
1
satisfy the equation 0π yn = (( n + 1) ( n + 2) ... ( n + n )) n .
n
∫0 e (sin at + cos at)dt
t 6 4
For x ∈ R , let [x ] be the greatest integer less
is/are (More than One Correct Option, 2015) than or equal to x. If lim yn = L, then the
n→∞
e 4π − 1 e 4π + 1 value of [L ] is ................. . (Integer Type, 2018)
(a) a = 2, L = π (b) a = 2, L =
e −1 eπ + 1
e 4π − 1 e 4π + 1 13. Area
(c) a = 4, L = (d) a = 4, L =
eπ − 1 eπ + 1
112. The common tangents to the circle x 2 + y 2 = 2
104. Let f : R → R be a function defined by and the parabola y 2 = 8x touch the circle at the
 [x ], x ≤ 2
f(x) =  , where [x ] denotes the points P,Q and the parabola at the points R, S.
0, x> 2 Then, the area (in sq units) of the quadrilateral
greatest integer less than or equal to x. If PQRS is (Single Correct Option, 2014)
2 xf ( x 2 ) (a) 3 (b) 6
I=∫ dx, then the value of ( 4I − 1)
−1 2 + f ( x + 1) (c) 9 (d) 15
is (Integer Type, 2015) 113. The area enclosed by the curves y = sin x + cos x
1 9x + 3 tan −1 x  12 + 9x 2   π
105. If α = ∫ ( e )  dx, and y =| cos x − sin x|over the interval 0,  is
0  1 + x2   2
where tan−1 x takes only principal values, then
(Single Correct Option, 2014)

 3π  (a) 4 ( 2 − 1) (b) 2 2 ( 2 − 1)
the value of  loge|1 + α| −  is
 4  (Integer Type, 2015) (c) 2 ( 2 + 1) (d) 2 2 ( 2 + 1)
2
π/ 2 x cos x 114. Let P and Q be distinct points on the parabola
106. The value of ∫ dx is equal to y 2 = 2x such that a circle with PQ as diameter
−π / 2 1+ e x
(Single Correct Option, 2016) passes through the vertex O of the parabola. If P
π2 π2 lies in the first quadrant and the area of ∆OPQ is
(a) −2 (b) + 2 (c) π 2 − e − π / 2 (d) π 2 + e π / 2
4 4 3 2, then which of the following is/are the
coordinates of P? (More than One Correct Option, 2015)
107. Let f : R → ( 0, 1) be a continuous function.
(c)  ,
1 1 
Then, which of the following function(s) has (a) (4 , 2 2 ) (b) (9 , 3 2 )  (d) (1, 2 )
4 2
(have) the value zero at some point in the
interval (0, 1) ? 115. Area of the region {( x , y )} ∈ R 2 : y ≥ |x + 3|,
(More than One Correct Option, 2017)
x
5 y ≤ ( x + 9) ≤ 15} is equal to
x (Single Option Correct, 2016)
(a) e x − ∫0 f (t ) sin t dt (b) f (x) + ∫0
2 f (t ) sint dt
1 4 3 5
x (a) (b) (c) (d)
−x 6 3 2 3
(c) x − ∫ (d) x − f (x)
9
2 f (t ) cos t dt
0
116. If the line x = α divides the area of region
k+1 k+1 R = {( x , y ) ∈ R 2 : x3 ≤ y ≤ x , 0 ≤ x ≤ 1} into two
∑k = 1 ∫k
98
108. If I = dx , then
x( x + 1) equal parts, then
(More than One Correct Option, 2017) (More than One Correct Option, 2017)
49 49
(a) I > loge 99 (b) I < loge 99 (c) I < (d) I > (a) 2α 4 − 4α 2 + 1 = 0 (b) α 4 + 4α 2 − 1 = 0
50 50 1 1
(c) < α < 1 (d) 0 < α ≤
sin( 2x )
∫sin x sin−1( t ) dt, then
109. If g( x ) = 2 2

(More than One Correct Option, 2017) 117. A farmer F1 has a land in the shape of a
π π triangle with vertices at P( 0, 0), Q(1, 1) and
(a) g′  −  = 2 π (b) g′  −  = − 2 π
 2  2 R( 2, 0). From this land, a neighbouring farmer
π π F2 takes away the region which lies between the
(c) g′   = 2 π (d) g′   = − 2 π sides PQ and a curve of the form y = x n ( n > 1).
 2  2
1 1 If the area of the region taken away by the
110. The value of ((log2 2 log 2 (log 2 9)
9) ) × ( 7) log 4 7
is farmer F2 is exactly 30% of the area of ∆PQR,
then the value of n is .................... .
............. . (Integer Type, 2018)

THE TIME LINE


The Time Line~JEE Advanced Questions (2014 –2018) 67

124. If y = y( x ) satisfies the differential equation


14. Differential Equations −1
 
118. The function y = f ( x ) is the solution of the 8 x  9 + x  dy =  4 + 9 + x  dx , x > 0
 
dy xy x + 2x 4
differential equation + = in and y( 0) = 7, then y( 256) =
dx x 2 − 1 1 − x2 (Single Correct Option, 2017)
3/ 2
( −1, 1) satisfying f( 0) = 0. Then, ∫ f ( x ) dx is (a) 16 (b) 3
3 (c) 9 (d) 80

2
(Single Correct Option, 2014) 125. Let f : R → R be a differentiable function with
π
(a) −
3 π
(b) −
3 π
(c) −
3 π
(d) −
3 f( 0) = 0. If y = f ( x ) satisfies the differential
3 2 3 4 6 4 6 2 dy
equation = ( 2 + 5 y )( 5 y − 2), then the value of
119. Let y( x ) be a solution of the differential dx
equation (1 + ex ) y ′ + yex = 1. If y( 0) = 2, then lim f ( x ) is ...... .
x→− ∞ (Integer Type, 2018)
which of the following statement(s) is/are true?
(Single Correct Option, 2015) 126. Let f : R → R be a differentiable function with
(a) y (−4) = 0 (b) y (−2) = 0 f( 0) = 1 and satisfying the equation
(c) y (x) has a critical point in the interval (−1 , 0) f ( x + y ) = f ( x ) f ′ ( y ) + f ′ ( x ) f ( y ) for all x , y ∈ R.
(d) y (x) has no critical point in the interval (−1 , 0) Then, the value of loge ( f ( 4)) is ....... .
(Integer Type, 2018)
120. Consider the family of all circles whose centres
lie on the straight line y = x. If this family of 127. Let f : [0, ∞ ) → R be a continuous function such
x
∫0 e
x− t
circles is represented by the differential that f ( x ) = 1 − 2x + f ( t ) dt for all x ∈ [0, ∞ ).
equation Py ′ ′+ Qy ′ + 1 = 0, where P , Q are the
functions of x, y and y′ (here, Then, which of the following statement(s) is
dy d2y (are) TRUE? (More than One Correct Option, 2018)
y′ = , y′ ′ = ), then which of the following (a) The curve y = f (x) passes through the point (1, 2)
dx dx 2
statement(s) is/are true? (b) The curve y = f (x) passes through the point (2, − 1)
(a) P =y+x (More than One Correct Option, 2015) (c) The area of the region
π−2
(b) P = y−x {(x, y) ∈ [0, 1] × R : f (x) ≤ y ≤ 1 − x 2 } is
4
(c) P + Q = 1 − x + y + y ′ + (y ′)2 (d) The area of the region
(d) P – Q = x + y – y ′ – (y ′)2 π −1
{(x, y) ∈ [0, 1] × R : f (x) ≤ y ≤ 1 − x 2 } is
4
121. Let f : ( 0, ∞ ) → R be a differentiable function
f(x)
such that f ′( x ) = 2 − for all x ∈ ( 0, ∞ ) and
x 15. Straight Line and Pair of
f(1) ≠ 1. Then, (More than One Correct Option, 2016)
Straight Lines
(a) lim f ′  = 1 (b) lim x f   = 2
1 1
x → 0+  x  x → 0+ x 128. For a point P in the plane, let d1( P ) and d2( P )
(c) lim x 2f ′(x) = 0 (d) | f (x)| ≤ 2 for all x ∈ (0, 2) be the distances of the point P from the lines
x → 0+
x − y = 0 and x + y = 0, respectively. The area of
122. A solution curve of the differential equation the region R consisting of all points P lying in
dy
( x 2 + xy + 4x + 2 y + 4) − y 2 = 0, x > 0, passes the first quadrant of the plane and satisfying
dx 2 ≤ d1( P ) + d2( P ) ≤ 4, is (Integer Type, 2014)
through the point (1, 3). Then, the solution
curve (More than One Correct Option, 2016) 129. Let a, λ, µ ∈ R. Consider the system of linear
(a) intersects y = x + 2 exactly at one point equations ax + 2 y = λ and 3x − 2 y = µ.
(b) intersects y = x + 2 exactly at two points Which of the following statement(s) is/are
(c) intersects y = (x + 2)2 correct? (More than One Correct Option, 2016)
(d) does not intersect y = (x + 3)2
(a) If a = − 3, then the system has infinitely many
13
1 solutions for all values of λ and µ
123. The value of ∑  π ( k − 1)π   π kπ  (b) If a ≠ − 3, then the system has a unique solution for
k=1 sin +  sin +  all values of λ and µ
is equal to 4 6  4 6
(Single Correct Option, 2016) (c) If λ + µ = 0, then the system has infinitely many
solutions for a = − 3
(a) 3 − 3 (b) 2 (3 − 3)
(d) If λ + µ ≠ 0, then the system has no solution fora = − 3
(c) 2 ( 3 − 1) (d) 2 (2 + 3)

THE TIME LINE


68 The Time Line~JEE Advanced Questions (2014 –2018)

135. Let P be a point on the circle S with both


16. Circle coordinates being positive. Let the tangent to S
130. A circle S passes through the point ( 0, 1) and is at P intersect the coordinate axes at the points M
orthogonal to the circles ( x − 1)2 + y 2 = 16 and and N . Then, the mid- point of the line segment
x 2 + y 2 = 1. Then, MN must lie on the curve (Paragraph Type , 2018)
(More than One Correct Option, 2014) (a) (x + y)2 = 3 xy (b) x 2 / 3 + y 2 / 3 = 2 4 / 3
(a) radius of S is 8 (b) radius of S is 7 (c) x 2 + y 2 = 2 xy (d) x 2 + y 2 = x 2 y 2
(c) centre of S is (−7,1) (d) centre of S is (−8,1) 136. Let T be the line passing through the points
131. Let RS be the diameter of the circle x 2 + y 2 = 1, P( − 2, 7) and Q( 2, − 5). Let F1 be the set of all
where S is the point (1, 0) . Let P be a variable pairs of circles (S1 , S 2 ) such that T is tangent to
point (other than R and S) on the circle and S1 at P and tangent to S 2 at Q, and also such
tangents to the circle at S and P meet at the that S1 and S 2 touch each other at a point, say
point Q. The normal to the circle at P intersects M. Let E1 be the set representing the locus of M
a line drawn through Q parallel to RS at point as the pair (S1 , S 2 ) varies in F1. Let the set of all
E. Then, the locus of E passes through the straight line segments joining a pair of distinct
point(s) points of E1 and passing through the point
( Single Correct Option, 2016) R(1, 1) be F2. Let E2 be the set of the mid-points
(a)  ,
1 1 
(c)  , −
 1 1 1 1  1 1
 (b)  ,   (d)  , −  of the line segments in the set F2. Then, which
3 3  4 2 3 3 4 2 of the following statement(s) is (are) TRUE?
(More than One Correct Option, 2018)
132. The circle C1 : x 2 + y 2 = 3 with centre at O
(a) The point (− 2, 7) lies in E1
intersects the parabola x = 2 y at the point P in
2

(b) The point  ,  does NOT lie in E 2


4 7
the first quadrant. Let the tangent to the circle  5 5
C1 at P touches other two circles C2 and C3 at R2
(c) The point  ,1 lies in E 2
1
and R3 , respectively. Suppose C2 and C3 have
2 
equal radii 2 3 and centres Q2 and Q3 ,
(d) The point  0,  does NOT lie in E1
3
respectively. If Q2 and Q3 lie on the Y-axis, then
 2
(a) Q2Q3 = 12 (More than One Correct Option, 2016)
(b) R 2R 3 = 4 6
(c) area of the ∆OR 2R 3 is 6 2 17. Parabola
(d) area of the ∆PQ2Q3 is 4 2
133. For how many values of p, the circle Paragraph (Q. Nos. 137-138)
x 2 + y 2 + 2x + 4 y − p = 0 and the coordinate Let a, r, s, t be non-zero real numbers. Let
axes have exactly three common points? P (at 2, 2at ), Q , R (ar 2, 2ar ) and S (as2, 2as) be distinct
(Integer Type, 2017)
point on the parabola y2 = 4ax. Suppose that PQ is
the focal chord and lines QR and PK are parallel,
Paragraph (Q. Nos. 134-135) where K is point (2a , 0). (Paragraph Type, 2014)
Let S be the circle in the XY -plane defined by the
equation x2 + y2 = 4.
137. The value of r is
1 t2 + 1 1 t2 −1
(There are two questions based on Paragraph “X”, (a) − (b) (c) (d)
t t t t
the question given below is one of them)
134. Let E1E2 and F1F2 be the chords of S passing 138. If st = 1, then the tangent at P and the normal
through the point P0 (1, 1) and parallel to the at S to the parabola meet at a point whose
X-axis and the Y -axis, respectively. Let G1G2 be ordinate is
the chord of S passing through P0 and having (t 2 + 1)2 a (t 2 + 1)2 a (t 2 + 1)2 a (t 2 + 2)2
(a) (b) (c) (d)
slope −1. Let the tangents to S at E1 and E2
3 3 3
2t 2t t t3
meet at E3 , then tangents to S at F1 and F2 meet
139. Let the curve C be the mirror image of the
at F3 , and the tangents to S at G1 and G2 meet
at G3 . Then, the points E3 , F3 and G3 lie on the parabola y 2 = 4x with respect to the line
curve (Paragraph Type , 2018) x + y + 4 = 0. If A and B are the points of
(a) x + y = 4 (b) (x − 4)2 + (y − 4)2 = 16 intersection of C with the line y = − 5, then the
(c) (x − 4) (y − 4) = 4 (d) xy = 4 distance between A and B is (Integer Type, 2015)

THE TIME LINE


The Time Line~JEE Advanced Questions (2014 –2018) 69

140. If the normals of the parabola y 2 = 4x drawn at Paragraph (Q. Nos. 146-147)
the end points of its latusrectum are tangents Let F1 (x1 , 0) and F2 (x2, 0), for x1 < 0 and x2 > 0, be
to the circle ( x − 3)2 + ( y + 2)2 = r 2, then the x2 y2
value of r 2 is (Integer Type, 2015)
the foci of the ellipse + = 1. Suppose a
9 8
141. Let P be the point on the parabola y 2 = 4x, parabola having vertex at the origin and focus at
F2 intersects the ellipse at point M in the first
which is at the shortest distance from the centre
quadrant and at point N in the fourth quadrant.
S of the circle x 2 + y 2 − 4x − 16 y + 64 = 0. Let Q (Paragraph Type, 2016)
be the point on the circle dividing the line 145. The orthocentre of ∆F1MN is
segment SP internally. Then,
(a)  − , 0 (b)  , 0 (c)  , 0 (d)  , 6 
9 2 9 2
(More than One Correct Option, 2016)
 10  3   10  3 
(a) SP = 2 5 (b) SQ : QP = ( 5 + 1) : 2
(c) the x-intercept of the normal to the parabola at P is 6 146. If the tangents to the ellipse at M and N meet
1 at R and the normal to the parabola at M meets
(d) the slope of the tangent to the circle at Q is
2 the X-axis at Q, then the ratio of area of ∆MQR
to area of the quadrilateral MF1NF2 is
142. If a chord, which is not a tangent, of the
(a) 3 : 4 (b) 4 : 5 (c) 5 : 8 (d) 2 : 3
parabola y 2 = 16x has the equation 2x + y = p,
and mid-point ( h , k), then which of the following 147. Consider two straight lines, each of which is
is(are) possible value(s) of p, h and k? tangent to both the circle x 2 + y 2 = (1/ 2) and the
(More than One Correct Option, 2017) parabola y 2 = 4x. Let these lines intersect at the
(a) p = − 1, h = 1, k = − 3 (b) p = 2, h = 3, k = − 4 point Q. Consider the ellipse whose centre is at
(c) p = − 2, h = 2, k = − 4 (d) p = 5, h = 4, k = − 3 the origin O( 0, 0) and whose semi-major axis is
OQ. If the length of the minor axis of this
ellipse is 2, then which of the following
18. Ellipse statement(s) is (are) TRUE?
(More than One Correct Option, 2014)
143. Let E1 and E2 be two ellipses whose centres are
(a) For the ellipse, the eccentricity is 1 / 2 and the
at the origin. The major axes of E1 and E2 lie
length of the latus rectum is 1
along the X-axis and Y-axis, respectively. Let S (b) For the ellipse, the eccentricity is 1/2 and the length
be the circle x 2 + ( y − 1)2 = 2. The straight line of the latus rectum is 1/2
x + y = 3 touches the curves S, E1 and E2 at P, Q (c) The area of the region bounded by the ellipse
and R, respectively. 1 1
between the lines x = and x =1is (π − 2)
2 4 2
2 2
Suppose that PQ = PR = . If e1 and e2 are the (d) The area of the region bounded by the ellipse
3 1 1
between the lines x = and x =1is (π − 2)
eccentricities of E1 and E2 respectively, then the 2 16
correct expression(s) is/are
(More than One Correct Option, 2015) 19. Hyperbola
43 7
(a) e12 + e22 = (b) e1 e2 = 148. Consider the hyperbola H : x 2 − y 2 = 1 and a circle
40 2 10
S with centre N ( x2 , 0). Suppose that H and S
5 3
(c) | e12 – e22 | = (d) e1 e2 = touch each other at a point P( x1 , y1 ) with x1 > 1
8 4 and y1 > 0. The common tangent to H and S at P
x2 y2 intersects the X-axis at point M. If ( l , m ) is the
144. Suppose that the foci of the ellipse + =1 centroid of ∆PMN , then the correct expression(s)
9 5
are ( f1 , 0) and ( f2 , 0), where f1 > 0 and f2 < 0. Let is/are (More than One Correct Option, 2015)
dl 1 dm x1
P1 and P2 be two parabolas with a common (a) = 1− for x1 > 1 (b) = for x1 > 1
vertex at ( 0, 0) with foci at ( f1 , 0) and (2 f2 , 0), dx1 3 x12 dx1 3 ( x 2 − 1)
1
respectively. Let T1 be a tangent to P1 which dl 1 dm 1
(c) = 1+ for x1 > 1 (d) = for y1 > 0
passes through ( 2 f2 , 0) and T2 be a tangent to dx1 3 x12
dy1 3
which passes through ( f1 , 0). If m1 is the slope of
T1 and m2 is the slope of T2, then the value of 149. If 2x − y + 1 = 0 is a tangent to the hyperbola
 1 2
x2 y2
 2 + m2  is 2
− = 1, then which of the following
 m1  a 16
(Integer Type, 2015)

THE TIME LINE


70 The Time Line~JEE Advanced Questions (2014 –2018)

CANNOT be sides of a right angled triangle? 155. Let a , b, c be three non-zero real numbers such
(More than One Correct Option, 2017) that the equation
(a) a, 4, 1 (b) 2a, 4, 1 (c) a, 4, 2 (d) 2a, 8, 1  π π
3 a cos x + 2b sin x = c, x ∈ − , , has two
150. x 2
y 2  2 2
Let H : −
= 1, where a > b > 0, be a π
a b2 2
distinct real roots α and β with α + β = . Then,
hyperbola in the XY -plane whose conjugate 3
b
axis LM subtends an angle of 60° at one of its the value of is .................. .
vertices N . Let the area of the ∆ LMN be 4 3. a (Integer Type , 2018)

Column I Column II 22. Inverse Circular Functions


P. The length of the conjugate 1. 8 156. If f : [ 0, 4π ] → [ 0, π ] be defined by
axis of H is
f ( x ) = cos−1(cos x ). Then, the number of points
2. 4 10 − x
Q. The eccentricity of H is x ∈ [0, 4π ] satisfying the equation f ( x ) = ,
3 10
2 is (Integer Type, 2014)
R. The distance between the foci 3.
of H is 3
157. If α = 3 sin−1  6  and β = 3 cos−1  4 , where the
   
S. The length of the latus rectum 4. 4  11  9
of H is
inverse trigonometric functions take only the
(Matching Type , 2018) principal values, then the correct option(s)
The correct option is is/are (More than One Correct Option, 2015)
(a) P → 4; Q → 2; R → 1; S → 3 (a) cos β > 0 (b) sin β < 0
(b) P → 4; Q → 3; R → 1; S → 2 (c) cos (α + β) > 0 (d) cos α < 0
(c) P → 4; Q → 1; R → 3; S → 2 158. The number of real solutions of the equation
(d) P → 3; Q → 4; R → 2; S → 1  ∞ ∞
 x 
i
sin−1  ∑ x i +1 − x ∑   
i = 1  
i =1 2 

21. Trigonometrical Equations π  ∞  x i ∞ 


= − cos−1  ∑  −  − ∑ ( − x )i  lying in the
151. For x ∈( 0, π ), the equation 2  i = 1 2 i =1 
sin x + 2 sin 2x − sin 3x = 3 has  1 1
(Single Correct Option, 2014) interval  − ,  is ................... .
(a) infinitely many solutions (b) three solutions
 2 2
(c) one solution (d) no solution (Here, the inverse trigonometric functions sin−1
 π π
152. The number of distinct solutions of the equation x and cos−1x assume values in − ,  and
5
cos2 2x + cos4 x + sin4 x + cos6 x + sin6 x = 2 in  2 2
4 [0, π ], respectively.) (Integer Type , 2018)
the interval [0, 2π ] is (Integer Type, 2015)
159. For any positive integer n, define fn :( 0, ∞ ) → R
 π n  1 
153. Let S =  x ∈ ( − π , π ): x =/ 0, ±  . The sum of all as fn ( x ) = Σ tan− 1   for all
 2 j =1  1 + ( x + j)( x + j − 1)
distinct solutions of the equation 3 sec x + cosec x x ∈ ( 0, ∞ ). (Here, the inverse trigonometric
+ 2(tan x − cot x ) = 0 in the set S is equal to  π π
(Single Correct Option, 2016) function tan− 1 x assumes values in  − ,  ).
 2 2
7π 2π 5π
(a) − (b) − (c) 0 (d) Then, which of the following statement(s) is
9 9 9
(are) TRUE? (More than One Correct Option, 2018)
154. Let α and β be non-zero real numbers such that
(a) ∑
5
2(cos β − cos α ) + cos α cos β = 1. Then which of tan2 (fj (0)) = 55
j =1
the following is/are true?
(b) ∑
10
(More than One Correct Option, 2017) (1+ f ′j (0)) sec2 (fj (0)) = 10
j =1
α β α β
(a) 3 tan  − tan  = 0 (b) tan  − 3 tan  = 0 (c) For any fixed positive integer n, lim tan(fn (x)) =
1
 2  2  2  2 x →∞ n
α β α β
(c) tan  + 3 tan  = 0 (d) 3 tan  + tan  = 0 (d) For any fixed positive integer n, lim sec2 (fn (x)) = 1
x →∞
 2  2  2  2

THE TIME LINE


The Time Line~JEE Advanced Questions (2014 –2018) 71

x , y and z respectively, then the value of


23. Properties of Triangles 2x + y + z is ((Integer Type, 2015)
160. In a triangle, the sum of two sides is x and the 166. Let u = u1i + u2 j + u3 k be a unit vector in R3
$ $ $
product of the same two sides is y. If 1 $ $
x 2 − c2 = y, where c is the third side of the and w = ( i + j + 2k
$ ). Given that there exists
6
triangle, then the ratio of the inradius to the
circumradius of the triangle is a vector v in R3 , such that|u + v| = 1 and
(Single Correct Option, 2014 ) w ⋅ ( u + v ) = 1. (More than One Correct Option, 2016)
3y 3y 3y 3y Which of the following statement(s) is/are correct?
(a) (b) (c) (d)
2 x (x + c) 2 c (x + c) 4 x (x + c) 4 c (x + c) (a) There is exactly one choice for such v
(b) There are infinitely many choices for such v
161. In a ∆XYZ, let x , y , z be the lengths of sides (c) If u$ lies in the XY-plane, then | u1| = | u2|
opposite to the angles X , Y , Z respectively and (d) If u$ lies in the XY-plane, then 2 | u1| = | u3 |
s−x s− y s−z
2s = x + y + z. If = = and area of
4 3 2 167. Let O be the origin and let PQR be an arbitrary
8π triangle. The point S is such that
incircle of the ∆XYZ is , then
3 OP ⋅ OQ + OR ⋅ OS = OR ⋅ OP + OQ ⋅ OS
(More Than Correct Option, 2016) = OQ ⋅ OR + OP ⋅ OS
(a) area of the ∆XYZ is 6 6 Then the triangle PQR has S as its
(Single Correct Option, 2017)
35
(b) the radius of circumcircle of the ∆XYZ is 6 (a) centroid (b) orthocentre
6
(c) incentre (d) circumcentre
X +Y 3
(d) sin2 
X Y Z 4
(c) sin sin sin =  =
2 2 2 35  2  5 Paragraph (Q. Nos. 168-169)
162. In a ∆ PQR, let ∠PQR = 30° and the sides PQ Let O be the origin and OX, OY, OZ be three unit
and QR have lengths 10 3 and 10, respectively. vectors in the directions of the sides QR, RP , PQ
Then, which of the following statement(s) is respectively, of a triangle PQR. (Paragraph Type, 2017)
(are) TRUE? (More than One Correct Option, 2018) 168. If the triangle PQR varies, then the minimum
(a) ∠QPR = 45 ° value of cos ( P + Q ) + cos (Q + R ) + cos( R + P ) is
(b) The area of the ∆PQR is 25 3 and ∠QRP = 120° (a) −
3
(b)
3
(c)
5
(d) −
5
(c) The radius of the incircle of the ∆PQR is 10 3 − 15 2 2 3 3
(d) The area of the circumcircle of the ∆PQR is 100 π 169. |OX × OY| =
(a) sin(P + Q) (b) sin(P + R) (c) sin (Q + R) (d) sin2R
24. Vectors 170. Let a and b be two unit vectors such that a ⋅ b = 0.
163. Let x , y and z be three vectors each of For some x , y ∈ R, let c = xa + yb + (a × b). If
magnitude 2 and the angle between each pair |c| = 2 and the vector c is inclined at the same
of them is π / 3. If a is a non-zero vector angle α to both a and b, then the value of 8 cos2 α
perpendicular to x and y × z and b is non-zero is ................ . (Integer Type , 2018)
vector perpendicular to y and z × x, then 171. Consider the cube in the first octant with sides
(Single Correct Option, 2014) OP, OQ and OR of length 1, along the X-axis,
(a) b =(b ⋅ z)(z − x ) (b) a =(a ⋅ y )(y − z) Y -axis and Z-axis, respectively, where O( 0, 0, 0)
(c) a ⋅ b = − (a ⋅ y )(b ⋅ z) (d) a =(a ⋅ y )(z − y )  1 1 1
is the origin. Let S  , ,  be the centre of the
 2 2 2
164. Let ∆PQR be a triangle. Let a = QR , b = RP and
c = PQ. If|a| = 12 ,| b| = 4 3 and b ⋅ c = 24, then cube and T be the vertex of the cube opposite to
the origin O such that S lies on the diagonal
which of the following is/are true? OT . If p = SP, q = SQ, r = SR and t = ST, then the
(More than One Correct Option, 2015) value of|( p × q ) × ( r × t )| is ..... .
2
|c | | c |2 (Integer Type , 2018)
(a) − | a | = 12 (b) + | a | = 30
2 2
(c) | a × b + c × a | = 48 3 (d) a ⋅ b = − 72 25. 3D Geometry
165. Suppose that p , q and r are three non-coplanar 172. From a point P( λ , λ , λ ), perpendiculars PQ and
vectors in R3 . Let the components of a vector s PR are drawn respectively on the lines y = x ,
along p , q and r be 4, 3 and 5, respectively. If z = 1 and y = − x , z = −1. If P is such that ∠QPR
is a right angle, then the possible value(s) of λ
the components of this vector s along
is (are) (More than One Correct Options, 2014)
( − p + q + r ), ( p – q + r ) and ( − p – q + r ) are x , y
(a) 2 (b) 1 (c) −1 (d) − 2

THE TIME LINE


72 The Time Line~JEE Advanced Questions (2014 –2018)

173. In R3 , consider the planes P1 : y = 0 and 178. Let P be a point in the first octant, whose image
P2 : x + z = 1. Let P3 be a plane, different from P1 Q in the plane x + y = 3 (that is, the line
and P2, which passes through the intersection of segment PQ is perpendicular to the plane
P1 and P2. If the distance of the point (0, 1, 0) x + y = 3 and the mid-point of PQ lies in the
from P3 is 1 and the distance of a point (α , β , γ ) plane x + y = 3) lies on the Z-axis. Let the
from P3 is 2, then which of the following distance of P from the X-axis be 5. If R is the
relation(s) is/are true? image of P in the XY -plane, then the length of
(More than One Correct Option, 2015) PR is ......... . (Integer Type , 2018)
(a) 2α + β + 2 γ + 2 = 0 (b) 2α − β + 2 γ + 4 = 0 179. Let P1 : 2x + y − z = 3 and P2 : x + 2 y + z = 2 be
(c) 2α + β − 2 γ − 10 = 0 (d) 2α – β + 2 γ − 8 = 0 two planes. Then, which of the following
3
174. In R , let L be a straight line passing through statement(s) is (are) TRUE?
(More than One Correct Option, 2018)
the origin. Suppose that all the points on L are
(a) The line of intersection of P1 and P2 has direction
at a constant distance from the two planes
ratios 1, 2, −1
P1 : x + 2 y − z + 1 = 0 and P2 : 2x − y + z − 1 = 0. 3x − 4 1 − 3y z
Let M be the locus of the foot of the (b) The line = = is perpendicular to the
9 9 3
perpendiculars drawn from the points on L to line of intersection of P1 and P2
the plane P1. Which of the following point(s) (c) The acute angle between P1 and P2 is 60°
lie(s) on M ? (d) If P3 is the plane passing through the point (4, 2, − 2)
(More than One Correct Option, 2015)
and perpendicular to the line of intersection of P1
(a)  0 , − , −  (b)  − , − , 
5 2 1 1 1
and P2 , then the distance of the point (2, 1, 1) from the
 6 3  6 3 6 2
 5
(c)  − , 0 , 
1
(d)  − , 0 , 
1 2 plane P3 is
3
 6 6  3 3
175. Consider a pyramid OPQRS located in the first 26. Miscellaneous
octant ( x ≥ 0, y ≥ 0, z ≥ 0) with O as origin, and 180. If 3x = 4x −1 , then x is equal to
OP and OR along the X-axis and the Y-axis, (More than One Correct Option, 2015)
respectively. The base OPQR of the pyramid is 2 log3 2 2
a square with OP = 3. The point S is directly (a) (b)
2 log3 2 − 1 2 − log2 3
above the mid-point T of diagonal OQ such that
1 2 log2 3
TS = 3. Then, (More than One Correct Option, 2016) (c) (d)
π 1 − log4 3 2 log2 3 − 1
(a) the acute angle between OQ and OS is
3 181. Match Column I with Column II and select the
(b) the equation of the plane containing the ∆OQS is correct answer using the codes given below the
x−y=0 lists.
(c) the length of the perpendicular from P to the plane Column I Column II
3
containing the ∆OQS is P. 3 1. 1
2 Let y ( x) = cos(3cos −1 x), x ∈ [−11
, ], x ≠ ± .
1  2
d y( x) xdy( x)  2
( x − 1) +
2
(d) the perpendicular distance from O to the straight Then,  equals
15 y ( x)  dx2 dx 
line containing RS is
2 Q. Let A1, A2 ,..., An ,(n > 2 ) be the vertices of a 2. 2
regular polygon of n sides with its centre at
176. Let P be the image of the point (3, 1, 7) with the origin. Let ak be the position vector of the
respect to the plane x − y + z = 3. Then, the n −1

equation of the plane passing through P and point Ak , k = 1,2,..., n. If ∑ (a k ⋅ a k + 1 )


x y z k =1
containing the straight line = = is n −1
1 2 1
(Single Correct Option, 2016)
= ∑ (a k ⋅ a k + 1 ) , then the minimum value of n is
k =1
(a) x + y − 3 z = 0 (b) 3 x + z = 0
R. If the normal form the point P(h,1) on the 3. 8
(c) x − 4 y + 7 z = 0 (d) 2 x − y = 0 x 2 y2
ellipse + = 1 is perpendicular to the line
177. The equation of the plane passing through the 6 3
point (1, 1, 1) and perpendicular to the planes x + y = 8, then the value of h is
2x + y − 2z = 5 and 3x − 6 y − 2z = 7 is S. Number of positive solutions satisfying the 4. 9
equation tan−1 
1  −1  1 
(Single Correct Option, 2017)  + tan  
 2 x + 1  4 x + 1
(a) 14 x + 2 y − 15 z = 1 (b) − 14 x + 2 y + 15 z = 3
= tan−1 (2 / x2 ) is
(c) 14 x − 2 y + 15 z = 27 (d) 14 x + 2 y + 15 z = 31

THE TIME LINE


The Time Line~JEE Advanced Questions (2014 –2018) 73

Codes (Matching Type, 2014) D. Suppose that F(α ) denotes the area of the S. 5
P Q R S P Q R S region bounded by x = 0, x = 2, y2 = 4 x
(a) 4 3 2 1 (b) 2 4 3 1
and y = |αx − 1| + |αx − 2| + αx,
(c) 4 3 1 2 (d) 2 4 1 3
where α ∈{0, 1}.
182. Match the given columns. Then,
Column I Column II 8
the value(s) of F(α ) + 2,
2 3
A. In R , if the magnitude of the projection P. 1
vector of the vector α $i + β$j on 3$i + $j is 3 when α = 0 and α = 1, is/are
and if α = 2 + 3 β, then possible value(s) T. 6
of|α| is/are
Let a and b be real numbers such that the Q 2 (Matching Type, 2015)
−3ax2 − 2, x < 1 π
B. function f( x) =  x +2

 bx + a , x≥1 184. Let F ( x ) = ∫ 2 cos2t dt for all x ∈ R and


2 6
x
is differentiable for all x ∈ R. Then, possible
 1
value(s) of a is/are f : 0,  → [0, ∞ ) be a continuous function. For
C. Let ω (=/ 1) be a complex cube root of unity. If R. 3
 2
(3 − 3ω + 2ω2 )4 n + 3 + (2 + 3ω − 3ω2 )4 n + 3  1
a ∈ 0, , if F ′ ( a ) + 2 is the area of the region
+ (−3 + 2ω + 3ω2 )4 n + 3 = 0, then the  2
possible value(s) of n is/are bounded by x = 0 , y = 0 , y = f ( x )
D. Let the harmonic mean of two positive real S. 4 and x = a, then f( 0) is (Integer Type, 2015)
numbers a and b be 4. If q is a positive real
number such that a, 5, q, b is in arithmetic 185. The total number of distincts x ∈ [0, 1] for which
progression, then the value(s) of|q − 2 a| x t2
is/are ∫0 1 + t4 dt = 2x − 1 is
(Integer Type, 2016)
T. 5
(Matching Type, 2015) Paragraph (Q.Nos. 186-188)
183. Match the given columns. By appropriately matching the information given
Column I Column II in the three columns of the following table.
A. In ∆ XYZ, let a, b and c be the lengths of the P. 1 Columns 1, 2 and 3 contain conics, equations of
sides opposite to the angles X, Y and Z, tangents to the conics and points of contact,
respectively. respectively.
sin ( X − Y )
If 2(a2 − b 2 ) = c 2 and λ = , Column 1 Column 2 Column 3
sin Z
(I) x2 + y2 = a2 (i my = m2 x + a (P)  a , 2 a 
 2 
then possible value(s) of n for which ) m m
cos (nπλ ) = 0, is/are
(II) x2 + a2 y2 (ii y = mx (Q)  − ma a 
B. In ∆XYZ, let a, b and c be the lengths of the Q. 2  , 
)  m2 + 1
=a + a m2 + 1 m2 + 1 
2
sides opposite to the angles X, Y and Z, 
respectively. If 1 + cos 2 X − 2 cos 2 Y = 2
a (III) y2 = 4ax (ii y = mx (R)  − a2 m 1 
sin X sin Y, then possible value(s) of is/are  , 
i)  a2 m2 + 1
+ a m −1 a2 m2 + 1 
b 2 2

C. In R 2, let 3$i + $j , $i + 3$j and β $i + (1 – β ) $j R. 3
be the position vectors of X, Y and Z with (IV) x2 − a2 y2 (i y = mx (S)  − a2 m 
 −1 
respect to the origin O, respectiveily. If the v) ,
= a2 + a2 m2 + 1  a2 m2 − 1 a2 m2 − 1 
distance of Z from the bisector of the acute 
3
angle of OX with OY is ,
2
then possible value(s) of|β| is/are (Matching Type, 2017)

THE TIME LINE


74 The Time Line~JEE Advanced Questions (2014 –2018)

186. For a = 2 , if a tangent is drawn to a suitable (a) (IV) (iv) (S) (b) (II) (iv) (R)
conic (Column 1) at the point of contact ( −1, 1), (c) (IV) (iii) (S) (d) (II) (iii) (R)
then which of the following options is the only 188. If a tangent to a suitable conic (Column 1) is
CORRECT combination for obtaining its found to be y = x + 8 and its point of contact is
equation? (Matching Type, 2017) (8, 16), then which of the following options is
(a) (I) (ii) (Q) (b) (I) (i) (P) the only CORRECT combination?
(c) (III) (i) (P) (d) (II) (ii) (Q) (a) (III) (i) (P) (b) (I) (ii) (Q)
187. The tangent to a suitable conic (Column 1) at (c) (II) (iv) (R) (d) (III) (ii) (Q)
 1 1 1
 3 ,  is found to be 3x + 2 y = 4, then which of
 2 189. The value of ((log2 9) ) 2 log 2 (log 2 9)
× ( 7) log 4 7
is
the following options is the only CORRECT (Integer Type, 2018)
combination?

ANSWERS
1 (c) 2 (4) 3 (a,c,d) 4 (b,d) 5 (a,b,d) 6 (a,c,d) 7 (d) 8 (a,d) 9 (c) 10 (d)
11 (d) 12 (4) 13 (9) 14 (b) 15 (6) 16 (3748) 17 (5) 18 (7) 19 (c) 20 (5)
21 (a) 22 (5) 23 (c) 24 (a) 25 (625) 26 (119) 27 (c) 28 (c) 29 (8) 30 (5)
31 (646) 32 (a) 33 (b) 34 (c) 35 (a,b) 36 (c,d) 37 (8) 38 (c) 39 (b) 40 (c)
41 (a,b) 42 (c) 43 (a) 44 (c) 45 (a,b) 46 (c,d) 47 (c,d) 48 (b,c) 49 (b) 50 (1)
51 (2) 52 (b,c) 53 (a,c) 54 (1) 55 (4) 56 (a,d) 57 (b) 58 (a,b,c) 59 (a) 60 (a,c)
61 (a) 62 (d) 63 (2) 64 (a,d) 65 (3) 66 (2) 67 (a,d) 68 (b,c) 69 (7) 70 (7)
71 (b,c) 72 (a,b) 73 (b,c) 74 (b,c) 75 (c,d) 76 (2) 77 (a,d) 78 (b,c) 79 (b,c,d) 80 (d)
81 (b,d) 82 (a,b,c) 83 (8) 84 (a,c,d) 85 (4) 86 (c) 87 (a,d) 88 (b) 89 (a,c) 90 (d)
91 (c) 92 (c) 93 (a,c) 94 (a,b,d) 95 (d) 96 (a) 97 (2) 98 (b) 99 (a) 100 (c)
101 (a,b) 102 (d) 103 (a,c) 104 (0) 105 (9) 106 (a) 107 (c,d) 108 (b,d) 109 (a,d) 110 (2)
111 (1) 112 (d) 113 (b) 114 (a,d) 115 (c) 116 (a,c) 117 (4) 118 (b) 119 (c) 120 (b,c)
121 (a) 122 (a,d) 123 (c) 124 (b) 125 (0.4) 126 (2) 127 (b,c) 128 (6) 129 (b,c,d) 130 (b,c)
131 (b,d) 132 (a,b,c) 133 (2) 134 (a) 135 (d) 136 (a,d) 137 (d) 138 (b) 139 (4) 140 (2)
141 (a,c,d) 142 (b) 143 (a,b) 144 (4) 145 (a) 146 (c) 147 (a,c) 148 (a,d) 149 (a,b,c,d) 150 (b)
151 (d) 152 (8) 153 (c) 154 (b,c) 155 (0.5) 156 (3) 157 (b,c) 158 (2) 159 (d) 160 (b)
161 (a,c,d) 162 (b,c,d) 163 (c) 164 (a,c,d) 165 (9) 166 (b,c) 167 (b) 168 (a) 169 (a) 170 (3)
171 (0.5) 172 (c,d) 173 (b,d) 174 (a,b) 175 (b,c,d) 176 (c) 177 (d) 178 (8) 179 (c,d) 180 (a,b,c)
181 (a) 182 (7) 183 (8) 184 (3) 185 (1) 186 (a) 187 (b) 188 (a) 189 (8)

For Detailed Solution, Visit https://goo.gl/oRDCN9

THE TIME LINE


PREP CATALYSIS
JEE Advanced

PRACTICE SET 1 (With Solutions)

Duration : 3 Hours Max. Marks . 360

Paper 1
PHYSICS
Section 1 (Maximum Marks : 24)
l This section contains SIX (06) questions.
l Each question has FOUR options for correct answer(s). ONE OR MORE THAN ONE of these four option(s) is (are)
correct options(s).
l
For each question, choose the correct options(s) to answer the question.
l
Answer to each question will be evaluated according to the following marking scheme:
Full Marks : + 4 If only (all) the correct option(s) is (are) chosen.
Partial Marks : + 3 If all the four options are correct but ONLY three options are chosen.
Partial Marks : + 2 If three or more options are correct but ONLY two options are chosen, both of which are
correct options.
Partial Marks : + 1 If two or more options are correct but ONLY one option is chosen and it is a correct option.
Zero Marks : 0 If none of the options is chosen (i.e. the question is unanswered).
Negative Marks : - 2 In all other cases.
l
For example: If first, third and fourth are the ONLY three correct options for a question with second option being an
incorrect option; selecting only all the three correct options will result in + 4 marks. Selecting only two of the three
correct options (e.g. the first and fourth options), without selecting any incorrect option (second option in this case), will
result in +2 marks. Selecting only one of the three correct options (either first or third or fourth option), without selecting
any incorrect option (second option in this case), will result in +1 marks. Selecting any incorrect option(s) (second
option in this case), with or without selection of any correct option(s) will result in -2 marks.

1. Potential energy of a particle is given by 2. Average height of a satellite of mass 220 kg


b c above earth’s surface is 640 km. It is losing its
U = a - + 2 , where x is position of particle mechanical energy at a rate of 1.42 ´ 105 J in
x x
each revolution.
and a , b and c are constants. Now, choose correct options.
Now choose correct options. [Take, mass of the earth = 6 ´ 1024 kg]
2c (a) Initial orbital speed of satellite is 7.56 ´ 103 ms-1
(a) Stable equilibrium occurs, when x =
b (b) Height of satellite after 1500 rounds above the
(b) Stable equilibrium occurs, when x is very large earth’s surface is 6808 km
b4 (c) Height of satellite after 1500 round above the
(c) For small oscillations, force constant is k =
8c 3 earth’s surface is 408 km
(d) Particle is in rectilinear motion (d) Average retarding force on satellite is 3.3 ´ 10-3 N

PREP CATALYSIS
78 JEE Advanced~Practice Set 1

3. A fully loaded 747 boeing aircraft has a mass of Now, choose the correct options.
3.3 ´ 105 kg. It’s total wing area is 500 m 2. It is in a (a) It is not possible, that images of both O1 and O2
level flight with speed of 960 km/h. Then, choose coincides for any position of L
the correct options. (b) Images of both O1 and O2 may coincides for
(a) There is no pressure difference between upper and some position of L
lower wing surfaces (c) Images of O1 and O2 coincides, when O1 is at
(b) Pressure difference between lower and upper wing 18 cm and O2 is at 6 cm from the lens L
. ´ 103 Pa
surfaces is 65 (d) Images of O1 and O2 coincides, when image is
(c) Speed of air on lower surface of wing is higher at 18 cm from L
(d) Speed of air on upper surface of wing is nearly 8% 1
higher than that below lower surface
6. A rod AB of length m, with 2 equal point
2
4. In given circuit, resistance of A B
masses attached to its ends is thrown in
wire AB is R = 6 W , length of air.
wire AB is l = 10 cm, number y vA
of density electrons in wire is + –
A
n = 1029 m -3 , cross-section 60°
6V
area of wire AB is A = 1 mm 2. C1

Now, choose the correct options. vB B


(a) Energy absorbed by electrons in wire AB = 2 ´ 10-17 J x
(b) Ohmic loss in wire AB =16 Js-1
(c) All kinetic energy of electrons will be lost in around Ground
3.4 ´ 10-18 s
(d) Energy lost per electron per second = 2 ´ 10-17 J At an instant rod AB makes an angle of
60° with the horizontal and velocity of its
5. Two point light sources O1 and O2 are kept 24 cm end A is vA = 2 $i + 3 $j ms -1. At same
apart. A biconvex lens L of focal length instant velocity of B is in negative
9 cm is kept between them x-direction.
L Now, choose the correct options.
(a) Rod AB has an angular speed of 4 rads-1
(b) Rod AB has an angular speed of 4 3 rads-1
O1 O2 (c) Velocity of COM C of rod is 3.5 $i + 2.6$j ms -1
1

(d) Velocity of COM C1 of rod is 2.6$i - 3.5 $j ms -1

Section 2 (Maximum Marks : 24)


l
This section contains EIGHT (08) questions. The answer to each question is a NUMERICAL VALUE.
l
Four each question, enter the correct numerical value (in decimal notation, truncated/rounded-off to the second
decimal place; e.g. 6.25, 7.00.- 0.33, -.30, 30.27, -127.30) using the mouse and the on-screen virtual numeric
keypad in the place designated to enter the answer.
l
Answer to each question will be evaluated according to the following marking scheme:
Full Marks : + 3 If ONLY the correct numerical value is entered as answer.
Zero Marks : 0 In all other cases.

$ } ´ 10-2 N moves
7. A force F = {( 2xy + z 2 ) i$ + x 2$j + 2xzk 9. A pair of parallel horizontal conducting
a particle from point ( 0, 1, 2) m to ( 5, 2, 7) m. Then, rails of negligible resistance shorted at one
find the work done by this force. end is fixed on a table. The distance
between the rails is L. A conducting
8. A stationary wave produced in a rod is expressed as,
. x ) sin ( 4.8 ´ 104 t ). What is the
y = 0.012 sin ( 015 massless rod of resistance R can slide on
the rails frictionlessly. The rod is tied to a
maximum particle speed at x = 3.5 cm? massless string which passes over a pulley
[Note that y is in cm]. fixed to the edge of the table.

PREP CATALYSIS
JEE Advanced~Practice Set 1 79

A mass m tied to the other end of the string 11. A solid glass cube, edge length 10 mm, refractive
hangs vertically. A constant magnetic field index 1.50 has a small air bubble at its centre.
B exists perpendicular to the table. If the What minimum fraction of cubes’ surface must be
system is released from rest, then calculate covered to prevent the bubble from being seen, no
matter what be the direction of viewing?
B
12. A cylindrical capacitor is made by using two
L R co-axial cylinders of radii 3 mm and 2mm as
shown in figure.

m 2mm

the acceleration of the mass at the instant, 3mm


when the velocity of the rod is half the
terminal velocity. (Take, g = 9.82 ms-2).
10. A thermocol vessel contains 0.5 kg of When this capacitor is fully charged, then half of
distilled water at 30°C. A metal coil of area potential energy lies in a region of radius, R =
5 ´ 10-3 m2, number of turns 100, mass 0.06 ................... (mm).
kg and resistance 1.6 W is lying horizontally 13. Two 50 g ice cubes are dropped in 200 g of water.
at the bottom of the vessel. A uniform time Initial temperature of ice cubes is
varying magnetic field is setup to pass -15°C and that of water is 25°C. Find final
vertically through the coil at time t = 0. The temperature of mixture, assuming no heat loss
field is first increased from 0 to 0.8 T at a and no heat is taken by the beaker in which
constant rate between 0 and 0.2 s and then mixing is done.
decreased to zero at the same rate between [Given, specific heat ice = 2220 J/kg-K,
0.2 and 0.4 s. specific heat water = 4190 J/kg-K and
The cycle is repeated 12000 times. Assume
latent heat of fusion of ice = 333 ´ 103 J/kg]
that no heat is lost to the vessel or the
surroundings. Determine the final 14. Consider that atmosphere consists only of nitrogen.
temperature of the water under thermal Assuming 1 atm pressure at 293 K, the collision
equilibrium. Specific heat of metal = 500 frequency of molecules is found nearly ......N....´ 109
J/kg-K and the specific heat of water = 4200 per second. Find value of N . [Take, diameter of a
J/kg-K. nitrogen molecule, d = 2 ´ 10-10m and their rms
Neglect the inductance of coil. speed, v = 473 ms-1].

PREP CATALYSIS
Section 3 (Maximum Marks : 12)
l This section contains TWO (02) paragraphs. Based on each paragraph, there are TWO(02) questions.
l Each question has FOUR options. ONLY ONE of these four options corresponds to the correct answer.
l Four each question, choose the option corresponding to the correct answer.
l Answer to each question will be evaluated according to the following marking scheme:
Full Marks : + 3 If ONLY the correct option is chosen.
Zero Marks : 0 If none of the options is chosen (i.e. the question is unanswered).
Negative Marks : - 1 In all other cases.

Paragraph X L = 0120
. ± 0.001 m, . ) ´ 10-3 m,
r = ( 3.4 ± 01
In an experiment, to determine charge to mass ratio I = 1.92 ± 0.02 A and V = 20 ± 1 V.
of electron using a cylindrical diode, we use
following equations 15. Error occurred in magnitude of magnetic field
is
e 8V
(i) = (a) 1% (b) 2% (c) 3% (d) 4%
m r 2B2
16. Error in value of e/m of electron is
V = potential difference across diode of (a) 2% (b) 4% (c) 6% (d) 8%
radius r
Paragraph A
B = magnitude of critical magnetic field, when A thick spherical shell of charge Q and uniform
electron pass undeviated. volumetric charge density r has inner and outer radii
m 0nI r1 and r2 , respectively. [Take, potential at infinity is
(ii) B =
(1 + D 2 / L2 )1/ 2 zero, also r1 = r and r2 = 3r].
n = number of turns per metre length of 17. Potential at a point distant 2r from centre is
solenoid, . r r 2 / e0
(a) 15 (b) 2.5 rr 2 / e0
(c) 3.5 rr 2 / e0 (d) 4.5 rr 2 / e0
I = current in solenoid,
D = diameter of solenoid and 18. Potential at a point inside the shell at distance
0.5 r from centre is
L = length of solenoid. (a) rr 2 / e0 (b) 2rr 2 / e0
Experimental observations are (c) 3 rr 2 / e0
(d) 4 rr 2 / e0
n = 3920 turns in 1 m, D = 0.035 ± 0.001 m,

CHEMISTRY
Section 1 (Maximum Marks : 24) 20. Which of the following statement(s) is/are correct?
(a) When FeCl 3 solution is added to K 4 [Fe(CN)6 ]
Instructions: Same as given in Physics. solution, in addition to FeIII [FeII (CN)6 ]- ,
FeII [FeIII (CN)6 ]- is also formed due to side redox
19. A freshly prepared aqueous solution of
reaction
Pd(NH3 )2Cl2 does not conduct electricity. It
suggests that (b) When FeCl 2 solution is added to K 3 [Fe(CN)6 ]
solution, in addition to FeII [FeIII (CN)6 ]- ,
(a) the structure of the compound involves covalent
FeIII [FeII (CN)6 ]- is also formed due to side redox
bonding only
reaction
(b) the chlorine atoms must be in coordination sphere
(c) FeIII [FeII (CN)6 ]- is paramagnetic while
(c) the van’t Hoff factor of the compound would be unity
FeII [FeIII (CN)6 ]- is diamagnetic
(d) on adding excess aqueous AgNO3 to 0.1 L of 0.1 M
solution of the compound, 0.02 mole of AgCl would (d) FeIII [FeII (CN)6 ]- is diamagnetic while FeII [FeIII (CN)6 ]-
be obtained is paramagnetic

PREP CATALYSIS
JEE Advanced~Practice Set 1 81

21. Based on the compounds of group 15 elements, P


the correct statement(s) is (are)

log 10 K
(a) ammonia has the lowest proton affinity θ
(b) KClO3 , KNO3 , sulphur and antimony contains the
head of match stick
(c) molecular nitrogen is more reactive than that of
oxygen O 1/T
(d) Thomas slag is a mixture of calcium phosphate and
calcium carbonate Graphically it is shown as given q = tan( 0.5)
and OP = 10. Thus,
22. In the following reaction sequence, the correct
(a) pre-exponential factor is 10
structure(s) of X is (are) (b) heat of reaction is 9.574 J mol - 1 at 298 K
Cl (c) pre-exponential factor is 1010
(i) Sn/HCl (d) heat of reaction is 2853 J mol - 1 at 298 K
(ii) CH3COCl
(iii) CH3—CH==CH 2/H + Section 2 (Maximum Marks : 24)
(iv) H3O+ /∆ Instructions: Same as given in Physics.
N
O O 25. The total number of compounds that are
Cl Cl diamagnetic among the molecular given below
is ..........
(a) (b) [Zn(OH)4 ]2 - , [Ni(NH3 )6 ]2 + , K 4 [Fe(CN)6 ],
NH2 K3 [Fe(CN)6 ], [Cu(NH3 )4 ]2 + , [PdBr4 ]2 - ,
NH2
[Ni(CO)4 ], [CoF6 ]3 -
Cl Cl 26. For the adsorption of a substance A from
NH2 aqueous solution by charcoal at 25° C, the
(c) (d) Freundlich constants are n = 3.0 and K = 0.50
x
(for in g per gram and C in g L - 1). The
NH2 m
weight of A will be adsorbed by 2g of charcoal
23. Which of the following reaction(s) will give from 1 L of a solution containing 2g of the
1, 3-dichlorobenzene as a significant organic substance is ..........
product? 27. The “fixing” of photographic film involves
Conc.HNO3 Cl2 SnCl2 NaNO2 CuCl dissolving unexposed silver bromide in a
(a) Conc.H2SO4 AlCl3 HCl,0°C thiosulphate (called hypo) solution
CH3 AgBr( s) + 2S2O32 - ( aq )- Ag(S2O3 )32 - ( aq )
+ Br- ( aq )
AlCl3 KMnO4 CaO Calculate the molar solubility of AgBr( s) in
(b) +Cl2(excess)
aq.NaOH NaOH, heat 0.10 M Na 2S2O3 .
heat
OH Given K sp( AgBr) = 5.4 ´ 1013 ;

Heat
K f [Ag(S2O3 )32 - ] = 2.9 ´ 1013
(c) + KCl
Cl 28. For the reaction, FeO( s) + CO( g) - Fe( s)
+ CO2( g), K p = 0.83 at 700 K and 0.48 at
Conc Zn/HCl NaNO2 CuCl
(d) + Conc.HNO3
H2SO4 HCl,0°C 1000 K. The total pressure at 850 K is 1.21
heat
bar. Calculate the partial pressure of CO2 at
24. Variation of equilibrium constant K with 850 K.
temperature, T is given by van’t Hoff equation
29. Hydrogen gas is produced by the following
DH °
log10 K = log A - reaction
2.303RT
Zn( s) + 2HCl( aq ) ¾®ZnCl2( aq ) + H 2( g)

PREP CATALYSIS
82 JEE Advanced~Practice Set 1

The gas is collected our water having vapour (b) B contains keto group and C contains aldehyde group
pressure 26.5 mm Hg at 22° C. If 128 mL of in addition to phenolic group
gas is collected at 22° C and 780 mm Hg (c) carbonyl reactivity in C is marked by intram
total pressure, then the mass of H 2 gas (d) carbonyl group is lost in hydrolysis step
collected will be .......... 34. Consider the following compounds
30. A sample of sea-water found to contain 3.5% OH OH OH
of NaCl and 0.13% MgCl2 by mass. The CH2OH COOH COOCH3
normal boiling point of water is 100° C and
K b (water) is 0.51 K kg mol - 1. Assuming
(I) (II) (III)
that both the salts are completely ionised,
calculate the normal boiling point of a COOCH3 OH COOH
sample of sea-water. COOH CHO

31. A galvanic cell consist of a Zn electrode in a


0.1 M Zn2 + solution and a second half-cell (IV) (V) (Vi)
with a Pt electrode in 0.1 M solution of HIO3 .
Calculate the cell potential if K a of HIO3 is Here, compound D, E and F respectively are
0.20. [Given : E° (Zn2 + / Zn) = - 0.76 V] (a) I, II and III (b) I, II and IV
(c) V, II and IV (d) II, III and VI
32. Under isothermal condition, variation of
æ1ö Paragraph A
log p with log ç ÷ is given below for H 2 gas,
èV ø When benzene reacts with ethanoyl chloride in
when pressure is 0.2 bar. presence of AlCl3 then compound A is formed that on
P
further reaction with PCl5 at 0° C gives B [C9H10Cl 2 ]. B
on reaction with 2NaNH 2 followed by heating gives C.
C on reaction with Li, liq. NH3 followed by hydrolysis
log p

45°
gives E (C9H10 ). C on reaction with H 2 , Ni 2 B catalyst
0.6990 bar dm3
gives D (C9H10 ).

+Me Cl AlCl3 PCl5 2 NaNH2


O (A) (B)
log 1
V ( ( O At 0°C
(C9H10Cl2)

(i) Li, liq.NH3


Calculate density of H 2 gas. (E) (C)
(ii) H2O
C9H10
Section 3 (Maximum Marks : 12) H2, Ni2B
catalyst

Instructions: Same as given in Physics.


(D)
Paragraph X (C9H10)
An organic compound ‘A’ gives positive
35. The compound (C) is
Libermann’s reaction and forms compounds B and
Me Me
C on treatment with trihalogen derivative of
(a) (b)
methange in alkaline medium followed by
hydrolysis. C is steam volatile and gives no Me
colouration with Schiff’s reagent, which is given (c) (d)
by compound B. C , when reduced with lithium Cl
aluminium hydride forms D with formula C 7 H 8O 2 . 36. The compound E is
D when oxidised forms E, which on reaction with
H H
anhydride in the presence of corresponding acid
forms a pain reliever F. Me
(a) (b) Me
33. B and C are isomeric but C does not give
colour with Schiff’s reagent. It is because of H Me
the fact that Me
(c) H (d)
(a) B contains an aldehyde group and C contains
keto group in addition to phenolic group Cl

PREP CATALYSIS
MATHEMATICS
Section 1 (Maximum Marks : 24) (a) The turning point of the curve y = f (x) is (2, 4)
(b) The area bounded by y = f (x), the X-axis and the
Instructions: Same as given in Physics. 5
ordinates x = 0 and x = 1 is sq units
æ 3 ö æ -3 ö æ- 3 ö 4
37. Let A ç , 2÷, B ç , 2÷, C ç , - 2 ÷ and (c) The turning point of the curve y = f (x) is (- 2, 4)
è 2 ø è 2 ø è 2 ø
(d) The area bounded by y = f (x), the X-axis and the
D( 3 cos q, 2 sin q ) are four points. If the area of
3
quadrilateral ABCD is maximum, where ordinates x = 0 and x = 1is sq units
4
æ 3p ö
q Îç , 2p ÷, then which of the following 42. Let a , b are the roots of ax 2 - bx + c = 0 and g, d
è 2 ø
statements is (are) TRUE? are the roots of px 2 - qx + r = 0. Then, which of
(a) The maximum area of quadrilateral is 10 sq units the following statements is (are) FALSE?
(b) Area of quadrilateral ABCD is maximum when (a) If a, b, g, d are in AP, then the common difference is
7p aq - bp
q=
4 4ap
(c) The maximum area of quadrilateral is 12 sq units (b) If a, b, g, d are in GP, then the common ratio is
(d) Area of quadrilateral ABCD is maximum when 1/ 4
æ cp ö
q = 2 p - sin-1 æç
3 ö ç ÷
÷ è ar ø
è 85 ø
(c) If a, b, g, d are in AP, then common difference is
38. Let the plane P : x - 2 y + 7z + 21 = 0. Then, bq - ap
which of the following statement(s) is (are) 4ap
TRUE? (d) If a, b, g, d are in GP, then the common ratio is
x+1 y-3 z+ 2 1/ 4
(a) The plane P contains the line = = æ ar ö
-3 2 1 ç ÷
è cp ø
(b) The plane P contains the point (0, 7, - 1)
x y z
(c) The plane P is perpendicular to the line = = Section 2 (Maximum Marks : 24)
1 -2 7
(d) The plane P is parallel to the plane x - 2 y + 7 z = 0 Instructions: Same as given in Physics.

39. Let f : ( - 1, 1) ® R be a continuous function 43. If a, b, c, d are the vertices of a cyclic


1 quadrilateral ABCD and A ¹ 90°, then
2 1 - x2
such that f ( x ) = x e . Then, which of the | a ´ b + b ´ d + d ´ a| | b ´ c + c ´ d + d ´ b|
following statements(s) is (are) TRUE? +
( b - a) × ( d - a) ( b - c) × ( d - c)
(a) The curve y = f (x) is decreasing in (- 1, 0)
(b) The curve f (x) = 1has two solution in (- 1, 1) 44. Let f : R ® R be a differentiable function
(c) The curve y = f (x) is increasing in (0, 1) satisfying f ( x ) = f ( y ) f ( x - y ) " x , y Î R and
(d) The curve f (x) = 1has no solution in (- 1, 1) 4
f ¢ ( 0) = ò { 2x } dx, where {. } denotes the
40. A circle of radius 4 cm is inscribed in DABC, 0
which touches the side BC at D. If BD = 6 cm,
fractional part function and f ¢ ( - 3) = aeb . Then,
DC = 8 cm. Then, which of the following
statements is (are) TRUE? |a + b| is equal to
(a) The perimeter of the DABC is 42 cm 45. Let f ( x ) be a polynomial with leading coefficient
(b) Area of DABC is 84 cm2
unity and satisfying the equation ( x - 16) f ( 2x )
(c) The DABC is an acute angled
= 16( x - 1) f ( x ), then f( 0) is equal to
(d) The DABC is an obtused angled
n 1
41. Let f : R ® R be a continuous function such 1 - P (cos rx ) r
x -t r=2
that f ( x ) = 3x 2 + ò0 e f ( x - t ) dt. Then, which of 46. If lim
x ®0 x2
= 10, then the value of n
the following statement is (are) TRUE? equals to

PREP CATALYSIS
84 JEE Advanced~Practice Set 1

47. If a 2 + b = 2, then maximum value of the term 51. The equation of the common tangent to
independent of x in the expansion of y 2 = 4ax and x 2 = 4ay is
1 1
- (a) x + y + a = 0 (b) x + y - a = 0
( ax 6 + bx 3 )9 ( a > 0, b > 0) is
(c) x - y + a = 0 (d) x - y - a = 0
48. There are 35 seats in a forum for all which 52. The equation of the parabola P is
2 2
three political parties are contesting the æ (1 + 3 ) ö æ (1 + 3 ) ö x + y - aö
2

number of ways in which no party gets (a) ç x - a÷ + çy - a ÷ = æç ÷


è 2 ø è 2 ø è 2 ø
absolute majority is ...... . 2 2
æ (1 + 3 ) ö æ (1 + 3 ) ö æ x + y - a ö
2
1 2 (b) ç x - a÷ + çy + a÷ = ç ÷
49. If sin a + cos b = and cos a + sin b = , then è 2 ø è 2 ø è 2 ø
2 3 2 2
a -b æ (1 + 3 ) ö æ (1 + 3 ) ö x + y + aö
2
the value of tan = a + b 3, then|a| +|b| is (c) ç x - a÷ + çy - a ÷ = æç ÷
2 è 2 ø è 2 ø è 2 ø
50. The sides of a DABC lie on the lines æ (1 + 3 ) ö æ
2 2
(1 + 3 ) ö æ x + y - a ö
2

3x + 4 y = 0, 4x + 3 y = 0 and x = 3. Let ( h , k) be (d) ç x + a÷ + çy - a÷ = ç ÷


è 2 ø è 2 ø è 2 ø
the centre of the circle inscribed in DABC. The
value of ( h + k) equals to.
Paragraph A
A four digit numbers are formed by using the digits
Section 3 (Maximum Marks : 12) 0, 1, 2, 3, 4, 5, 6, 7, 8, 9. For the above information,
answer the following questions.
Instructions: Same as given in Physics.
53. The probability that exactly three of digits are
Paragraph X same is
Let the 2 parabolas y 2 = 4ax and x 2 = 4ay intersect at 4 33 11
(a) (b) (c) (d) None of these
O and A (O being origin). Consider a parabola P 125 1000 270
whose directrix is the common tangent to the 54. The probability that two of the digits are equal
parabolas and whose focus is the point which and the other two digits are also equal is
divides OA internally in the ratio (1 + 3 ) : (7 - 3 ). (a)
3
(b)
3
(c)
27
(d) None of these
125 250 1000

Paper 2
PHYSICS
Section 1 (Maximum Marks : 24)
l
This section contains SIX (06) questions.
l
Each question has FOUR options for correct answer(s). ONE OR MORE THAN ONE of these four option(s) is (are)
correct options(s).
l
For each question, choose the correct options(s) to answer the question.
l
Answer to each question will be evaluated according to the following marking scheme:
Full Marks : + 4 If only (all) the correct option(s) is (are) chosen.
Partial Marks : + 3 If all the four options are correct but ONLY three options are chosen.
Partial Marks : + 2 If three or more options are correct but ONLY two options are chosen, both of which are
correct options.
Partial Marks : + 1 If two or more options are correct but ONLY one option is chosen and it is a correct option.
Zero Marks : 0 If none of the options is chosen (i.e. the question is unanswered).
Negative Marks : - 2 In all other cases.
l
For example: If first, third and fourth are the ONLY three correct options for a question with second option being an
incorrect option; selecting only all the three correct options will result in + 4marks. Selecting only two of the three correct
options (e.g. the first and fourth options), without selecting any incorrect option (second option in this case), will result in
+2 marks. Selecting only one of the three correct options (either first or third or fourth option), without selecting any
incorrect option (second option in this case), will result in +1marks. Selecting any incorrect option(s) (second option in
this case), with or without selection of any correct option(s) will result in -2 marks.

PREP CATALYSIS
JEE Advanced~Practice Set 1 85

1. Two men A and B, mA = 60 kg and mB = 80 kg are The U-tube is rotated at 10 rad s -1 about a
standing on a plank of mass 20 kg. Let A jumps to vertical axis at 0.1 m from one of the limb.
left with a velocity of 1 ms-1. Then, (g = 10 ms-2). Now, choose the correct options.
Z1 Z1
A B (a) = 0.684 (b) = 0513
.
Z2 Z2
(c) Zmin = 0.275 m (d) Zmin = 0135
. m
Plank
4. An ideal coil of 20 H is joined in series with
Smooth surface a resistance of 10 W and an ideal battery of
11 -1 10 V. After two seconds, the current flowing
(a) speed of A with respect to plank just after jump is ms (in ampere) in the circuit will be
8
5
(b) speed of A with respect to plank just after jump is ms-1 A
8
+ +
(c) speed of B with respect to A just after jump is 1 ms-1 + + +
3
(d) speed ofB with respect to ground after the jump is ms-1 + + + +
8 + + + +
2. Consider given arrangement + +O + +
+ + + ++
B + + + C
+ +

m2 (a) e (b) e - 1
(c) (1 - e - 1) (d) (1 - e)
2m
m1=10kg 5. In the figure shown there exists a uniform
time varying magnetic field
If friction is absent everywhere and hanging block B = [( 4 T/ s)t + 0.3 T ] in a cylindrical region of
strikes the ground in 2 s when released, then radius 4m. An equilateral triangular
choose the correct options. conducting loop is placed in the magnetic
(Take, g = 10 ms-2) field with its centroid on the axis of the field
and its plane perpendicular to the field.
(a) Acceleration of m2 is g
(a) emf induced in any one rod is 16 V
(b) Acceleration of m2 is 1ms-2
(b) emf induced in the complete DABC is 48 3 V
(c) Weight of m2 is 10 kg
(c) emf induced in the complete DABC is 48V
(d) Weight of m2 is not applicable to above problem
(d) emf induced in any one rod is 16 3 V
3. An upright narrow U-tube manometer with its
limbs 0.6 m high and spaced 0.3 m apart contains a
6. A lens of focal length f is placed in between
liquid upto a height of 0.4 m in each limb. an object and screen at a distance D. The
lens forms two real images of object on the
w
screen for two of its different positions, a
distance x apart. The two real images have
magnification m1 and m2, respectively
( m1 > m2 ).
Z2
x
(a) f = (b) m1m2 = 1`
0.4m

Z1
2 min m1 - m2
D2 - x 2
(c) f = (d) D ³ 4f
4D
0.1m 0.2m

PREP CATALYSIS
Section 2 (Maximum Marks : 24)
l This section contains EIGHT (08) questions. The answer to each question is a NUMERICAL VALUE.
l Four each question, enter the correct numerical value (in decimal notation, truncated/rounded-off to the second
decimal place; e.g. 6.25, 7.00.- 0.33, -.30, 30.27, -127.30) using the mouse and the on-screen virtual numeric
keypad in the place designated to enter the answer.
l Answer to each question will be evaluated according to the following marking scheme:
Full Marks : + 3 If ONLY the correct numerical value is entered as answer.
Zero Marks : 0 In all other cases.

7. A diatomic ideal gas is used in Carnot’s engine as 12. A mass m is undergoing SHM in the
working substance. During adiabatic expansion of vertical direction about the mean position
the cycle, if the volume of the gas increases from y0 with amplitude A and angular frequency
V to 32 V, then the efficiency of the engine is w . At a distance y from the mean position,
......... . the
8. An object of density 2000 kg-m -3 is hung from a
thin light wire. The fundamental frequency of the
transverse wave in the wire is 200 Hz. If object is
immersed in water such that its half of volume is
submerged, then the fundamental frequency of
the transverse wave in the wire is ............ Hz. y m

9. Magnetic field at two points on the axis of a mass detaches from the spring. Assume
circular coil at distances of 0.05 m and 0.2 m from
that the spring contracts and does not
the centre of coil are in the ratio 8 : 1. Find the
obstruct the motion of m. Find the distance
radius of coil in metres.
y (measured from the mean position) such
10. In a photoelectric effect, set-up a point of light of that the height h attained by the block is
power 3.2 ´ 10-3 W emits monoenergetic photons maximum. ( Aw2 > g).
of energy 5.0 eV. The source is located at a [Take, w = 0.5 rad/s and g = 9.8 ms-2]
distance of 0.8 m from the centre of a stationary
metallic sphere of work function 13. To make effective gravitational pull over a
3.0 eV and of radius 8.0 ´ 10-3 m. The efficiency of body at equator zero, angular speed of the
photoelectrons emission is one for every 106 earth is N ´ 10-3 rad-s -1. Find the value of N .
incident photons. Assume that the sphere is
isolated and initially neutral and that 14. A carpet of mass M made of inextensible
photoelectrons are instantly swept away after material is rolled along its length in the
emission. form of a cylinder of radius R and is kept on
If the number of photoelectrons emitted per a rough floor. The carpet starts unrolling
second is N ´ 105 photons per second, then find without sliding on the floor, when a
the value of N . negligibly small push is given to it. Let the
horizontal velocity of the axis of the
11. Plane waves of wavelength 6 ´ 10-5 cm falls cylindrical part of the carpet, when its
normally on a slit of width 0.2 mm. Calculate the radius reduces to R/2 is v. Then, find the
width of central maxima (in cm) on a screen 2m value of v 2. [Take, R = 1 m and g = 9.8 ms-2]
away.

PREP CATALYSIS
Section 3 (Maximum Marks : 12)
l This section contains FOUR (04) questions.
l Each question has TWO (02) matching lists : Column-I and Column-II.
l FOUR options are given representing matching of elements from Column-I and Column-II. ONLY ONE of these four
options corresponds to a correct matching.
l For each question, choose the option corresponding to the correct matching.
l For each question, marks will be awarded according to the following marking scheme.
Full Marks : + 3 If ONLY the option corresponding to the correct matching is chosen.
Zero Marks : 0 If none of the options is chosen (i.e. the question is unanswered).
Negative Marks : - 1 In all other cases.

15. Column II gives certain systems undergoing a process. Column I suggests changes in some of the
parameters related to the system. Match the statements in Column I to the appropriate process (es) from
Column II.
Column I Column II
A. The energy of the system is increased. p. System : A capacitor, initially uncharged.
Process : It is connected to a battery.
B. Mechanical energy is provided to the q. System : A gas in an adiabatic container fitted with an adiabatic piston.
system, which is converted into energy Process : The gas is compressed by pushing the piston.
of random motion of its parts.
C. Internal energy of the system is r. System : A gas in a rigid container.
converted into its mechanical energy. Process : The gas gets cooled due to colder atmosphere surrounding it.
D. Mass of the system is decreased. s. System : A heavy nucleus, initially at rest.
Process : The nucleus fissions into two fragments of nearly equal masses
and some neutrons are emitted.
t. System : A resistive wire loop.
Process : The loop is placed in a time varying magnetic field
perpendicular to its plane.

(a) A-p, B-q, C-rs, D-t (b) A-pt, B-qst, C-rst, D-s (c) A-pqst, B-q, C-s, D-s (d) A-s, B-t, C-p, D-q
16. Column II shows five systems in which two objects are labelled as X and Y . Also in each case a point P is
shown. Column I gives some statements about X and/or Y . Match these statements to the appropriate
system(s) from Column II.
Column I Column II
A. The force exerted by X onY p. Y
has a magnitude Mg
X

P
BlockY of mass M left on a fixed inclined plane X, slides on it with a constant velocity.
B. The gravitational potential q. P
energy of X is continuously Z
increasing.
Y
X

Two ring magnetsY and Z, each of mass M, are kept in frictionless vertical plastic
stand, so that they repel each other.Y rests on the base X and Z hangs in air in
equilibrium. P is the topmost point of the stand on the common axis of the two rings.
The whole system is in a lift that is going up with a constant velocity.

PREP CATALYSIS
88 JEE Advanced~Practice Set 1

C. Mechanical energy of the r. Y


P
system X + Y is continuously
decreasing. X

A pulley Y of mass m0 is fixed to a table through a clamp X. A block of mass M


hangs from a string that goes over the pulley and is fixed at point P of the table. The
whole system is kept in a lift that is going down with a constant velocity.
D. The torque of the weight ofY s.
about point P is zero.
Y
X
P
A sphereY of mass M is put in a non-viscous liquid X kept in a container at rest. The
sphere is released and it moves down in the liquid.
t. A sphereY of mass M is falling with its terminal velocity in a viscous liquid X kept in
a container.

(a) A-pt, B-qst, C-prst, D-q (b) A-pst, B-qst, C-rst, D-pqrst
(c) A-r, B-q, C-s, D-pt (d) A-pr, B-qs, C-st, D-qs
17. Six point charges, each of the same magnitude q, are arranged in different manners as shown in
Column II. In each case, a point M and a line PQ passing through M are shown. Let E be the electric
field and V be the electric potential at M (potential at infinity is zero) due to the given charge
distribution when it is at rest. Now, the whole system is set into rotation with a constant angular
velocity about the line PQ. Let B be the magnetic field at M and m be the magnetic moment of the
system in this condition. Assume each rotating charge to be equivalent to a steady current.
Column I Column II
A. E = 0 p. + – Q

– +
M
P+ –

Charges are at the corners of a regular hexagon. M is at the centre of the hexagon. PQ is perpendicular
to the plane of the hexagon.
B. V ¹ 0 q. P

– + – + – +
M

Charges are on a line perpendicular to PQ at equal intervals. M is the mid-point between the two
innermost charges.
C. B = 0 r. Q
+ – +
M
– –

P
+
Charges are placed on two coplanar insulating rings at equal intervals. M is the common centre of the
rings. PQ is perpendicular to the plane of the rings.

PREP CATALYSIS
JEE Advanced~Practice Set 1 89

D. m ¹ 0 s. – + –

P Q
M
– + –
Charges are placed at the corners of a rectangle of sides a and 2a and at the mid points of the longer
sides. M is at the centre of the rectangle. PQ is parallel to the longer sides.
t. +
P

+ + M –

Q

Charges are placed on two coplanar, identical insulating rings at equal intervals. M is the mid points
between the centres of the rings. PQ is perpendicular to the line joining the centres and coplanar to the
rings.

(a) A-p, B-q, C-r, D-st (b) A-st, B-q, C-pr, D-ps
(c) A- ps, B-rs, C-pq, D-t (d) A-prs, B-rs, C-pqt, D-rs
18. Two wires each carrying a steady current I are shown in four configurations in Column I. Some of the
resulting effects are described in Column II. Match the statements in Column I with the statements in
Column II.
Column I Column II
A. Point P is situated midway between the wires p. The magnetic fields (B) at P due to the currents in the
wires are in the same direction.
P

B. Point P is situated at the mid-point of the line joining the q. The magnetic fields (B) at P due to the currents in the
centres of the circular wires, which have same radii. wires are in opposite directions.

C. Point P is situated at the mid-point of the line joining the r. There is no magnetic field at P.
centres of the circular wires, which have same radi

D. Point P is situated at the common centre of the wires. s. The wires repel each other.

(a) A-s, B-q, C-r, D-p (b) A-p, B-q, C-r, D-s
(c) A-qr, B-p, C-qr, D-q (d) A-pr, B-p, C-s, D-qs

PREP CATALYSIS
CHEMISTRY
Section 1 (Maximum Marks : 24) OC2H5 F
Instructions: Same as given in Physics.
(c) and
19. The correct statement(s) regarding the
coordination compounds is/are NHCMe NO2
(a) [MnBr4 ]2 - have sp 3 hybridisation with magnetic
moment 5.9 BM. O
(b) [Cr(H2O)6 ]2 + and [Fe(H2O)6 ]2 + has same magnetic OC2H5 F
moment.
(c) the IUPAC name of [CoCl(NH3 )3 (H2O)2 ] Cl 2 is
(d) and
chlorodiaquatriammine cobalt (III) chloride.
(d) benzoylacetonatoberyllium exhibit optical
isomerism. NH2 NO2
20. An inorganic mixture gives yellow precipitate 22. The Fischer configuration of ( - )-gulose is given
on boiling with conc. HNO3 and ammonium below that undergoes following reaction
molybdate [(NH 4 )2 MoO4 ]. Thus, inorganic
CHO
mixture can be
(a) (NH4 )3 PO4 (b) (NH4 )3 AsO4 H OH
(c) Pb(NO3 )2 (d) (NH4 )2 C2O4 H OH Ruff Ruff HNO3
HO H degradation
‘ X’ degradation
‘ Y’ ‘Z’
21. p-nitrophenol reacts with C2H5I to give B. B on
reaction with HI gives G and H. B also reacts H OH
with Sn + HCl gives C. The following sequence CH2OH
of reactions also takes place as (–)-gulose
OH
The structure of ‘Z’ is
C2H5I Sn + HCl Ac2O+AcOH
(B ) (C ) (D) CHO COOH
Excess of (C10H13O3N) HO H H OH
(a) (b)
NO2 HI Analgesic and H OH H OH
(G) + (H) antipyretic
p-nitrophenol CH2OH COOH
(A )
COOH COOH
PhSO2Cl
NaF/DMSO HO H HO H
(c) (d)
(E ) (F) H OH HO H
The product D and F respectively are COOH COOH
OC2H5 F 23. The gas A2 in the left flask allowed to react
with gas B2 present in right flask as
(a) and A2( g) + B2( g) 2AB ( g), K C = 4 at 27° C
-
O
NHCMe 4 mol 8 mol
A2 B2

OH F
2L 4L
at 27°C at 27°C
(b) and
O What is the concentration of AB, when
equilibrium is established?
NHCMe NO2
(a) 0.59 L (b) 0.89 L (c) 0.69 L (d) 0.29 L

PREP CATALYSIS
JEE Advanced~Practice Set 1 91

24. Consider the following chemical reaction the surface and form a monolayer i.e. the layer
is only one molecule thick. The cross sectional
Pb(NO3 )2 + Na 2SO4 ¾® PbSO4 + 2NaNO3
area of each stearic acid molecule has been
[N: Pb(NO3 )2 = 330, Na 2SO4 = 142] measured to be 0-21 nm 2. In one experiment it
is found that 1.4 ´ 104 g of stearic acid is …… .
If a series of experiments are run maintaining
sum of the weights of two reactant constant needed to form monolayer over water in a dish
but varying the weight of reactants, which of of diameter 20 cm. Based on measurements
the following statement is/are true? value of Avogadro’s numbers is ..... ´ 1023 .
(a) In the experiment as the weight of Pb(NO3 )2
increases weight of precipitate increases. 30. Calculate the energy required for the
(b) In the experiment, as the weight of Na 2 SO4 excitation of 2 litres of hydrogen gas at 5atm
increases weight of precipitate increases. and 298 K to the first excited state of atomic
(c) Maximum weight of the precipitate PbSO4 will be hydrogen. [Given : Dissolution energy of H ¾ H
formed if equal weights of reactants are taken. bond = 512 kJ mol-1].
(d) Maximum weight of precipitate will be formed if
equal moles of reactants are taken. 31. Interconversion of boat form to chair form of
cyclohexane is a first order reaction (in both
Section 2 (Maximum Marks : 24) sides)
Instructions: Same as given in Physics. K1
K2
25. Among the species given below, the total
number of molecules that have non-zero value
of dipole moments? Equilibrium constant K C = 1010. Also energy of
CH 2Cl2, CH 4, CCl4, H 2O, CHCl3 , activation for conversion of chair to boat form
p-dichlorobenzene, o-cresol, p-xylene, SCl2, is 54 kJ mol- 1 in the following equation
BF3 , IBr, HCHO. K ( s- 1 ) = 1020 e- Ea/ RT . Calculate observed rate
26. Li and potassium crystallises in a body-centred constant of reaction at 298 K.
cubic lattice with their radii 152 and 230 pm, 32. The density of a pure substance containing P
respectively. Calculate the weight of solid atoms packed in fcc close pack arrangement is
potassium present in the volume equivalent to
2 g/cc. If Q-atoms can occupy tetrahedral voids
100 g of Li metal.
and if all the tetrahedral voids are occupied by
27. Consider the reaction at equilibrium: Q-atom. Calculate the density of resulting solid
LiCl × 3NH3 ( s) - LiCl × NH3 ( s) + 2NH3 ( g) in g/cc. [Given: Atomic mass of P = 60 g/mol
and atomic mass of Q = 100 g/mol]
A 8 litre vessel contains 0.2 mole of LiCl × NH3 .
If K p = 25 atm 2 at 40° C, then calculate the Section 3 (Maximum Marks : 12)
number of moles of NH3 that should be added Instructions: Same as given in Physics.
to the flask at this temperature to derive the
backward reaction for completion. 33. Match each set of reagents given in Column I
with its characteristic property given in
28. For the given compound ‘X’, the total number Column II.
of chiral centre(s) is/are ... .
H Column-I Column-II
I. K 2 CrO4 p. Baeyer’s reagent
Me
Me II. KMnO4 q. Transition element in + 6
H
oxidation state.

HO H III. K 2 Cr2O7 r. Oxidising agent in acidic


medium.
H H
IV. K 2MnO4 s. Manufactured from
29. To estimate the order of magnitude of pyrolusite
Avogadro’s number using stearic method t. Manufactured from
(C18H36O2 ) is described as follows. When stearic chromite ore.
acid is added to water, its molecules collect at

PREP CATALYSIS
92 JEE Advanced~Practice Set 1

Codes III. EtBr NBS r.


I II III IV AlCl3 CHO
(a) q,t p, r q, r, t q, s Alc.KOH
(b) q, p p, s q, p, s q, t
IV. MeCOCl s. CHO
(c) p q r s
AlCl3
(d) q s p t
NaBH4 H2SO4
34. The desired product X can be prepared by reacting
the major product of the reactions in list-I with one
or more appropriate reagents in list-II. t.
H3C N

H3C N CH3 (X )
H u.

List-I List-II
I. KMnO 4 p. O O
CH3 CH2OH ¾ ¾¾¾
¾®
½½ ½½ Codes
CH3 C ¾ C ¾ C2H5 I II III IV
II. CH3 CNH2 + NH3 ¾® q. NH3 (a) q, r p, s u t
½½ (b) p, q r, s u t
O (c) q, r p, s t u

O O O (d) q, s p, r t u
III. r.
½½ ½½ - + ½½
36. Relations between protons and neutrons
CH3 C Cl + CH3 ¾ C ONa ¾® CH3 C CH — Br
½ are given List-I. The calculated number
C2H5 are given in List-II.

s. NH2 O List-I List-II


½½ ½½
H3 C — CH — C— C2H5 P. Number of protons in the (i) 50
element with mass number 81
t. AcONH4 and having 31.7% more
neutrons than protons.
Codes
Q. Ionic mass of M 2 + which is (ii) 26
I II III I II III
isoelectronic of CO2 and has
(a) p, q r s, t (b) p, r q, s r, t (Z + 2) neutrons.
(c) p, q r, s s, t (d) p, q r, t s
R. Ionic mass of X - with 17 (iii) 35
35. List-I contains reactions and List-II contains major protons and having 11.1%
products. more neutrons than electrons.
S. Atomic number of M 3 + with (iv) 42
List-I List-II mass number 56 and having
I. p. OHC 30.4% more neutrons than

+
electrons.
CHO
(v) 37

II. q. Correct option is


∆ CHO
+ (a) P ® (iii); Q ® (i); R ® (v); S ® (ii)
CHO
(b) P ® (i); Q ® (ii); R ® (iii); S ® (iv)
(c) P ® (iv); Q ® (iii); R ® (ii); S ® (i)
(d) P ® (iii); Q ® (v); R ® (ii); S ® (i)

PREP CATALYSIS
MATHEMATICS
Section 1 (Maximum Marks : 24) 42. OACB is a square on XY -plane where O is
origin. A line through A intersects the
Instructions: Same as given in Physics. diagonal OC at D externally, side OB of E
37. For a point P on ellipse the circles with PS and internally and side CB at F externally. Given
PS ¢, where S, S ¢ represent foci of ellipse, as that AD : DE = 4 : 3, AD = 5 units and the
diameter intersect the auxiliary circle of ellipse square lies completely in first quadrant. Then
at A, A1 and B ,B1 respectively. Then which of which of the following is(are) TRUE?
the following is/are TRUE? (a) The area of square will be 49 sq units
8
(a) A and A1 coincide, B and B1 coincide (b) The abscissa of F is -
(b) Segment AB is tangent to ellipse at P 3
(c) Tangents at A and B on auxiliary circles are (c) The area of square will be 36 sq unit
7
perpendicular (d) The abscissa of F is -
(d) SA and S ¢B are parallel 3

x x2 0 Section 2 (Maximum Marks : 24)


38. Let f ( x , y ) = 1 ( 2x + y ) ( x + y )2 . Then, which Instructions: Same as given in Physics.
0 1 2x + 3 y 43. If the vector a = ( x , y , z ) makes equal angle q
of the following statements is (are) TRUE? with the vector b = (1, - 2, 3) and c = ( 2, 3, - 1)
and is perpendicular to the vector d = (1, - 1, 1)
(a) (x + y) is a factor of f (x, y)
(b) f (x, y) is divisible by (2x + y) and then 124 cos2 q is equal to
(c) (x + 2 y) is a factor of f (x, y) x2 + x + 1 50
(d) x is a factor of f (x, y) 44. If
1- x
= a0 + a1x + a2x 2 ...., then å ar is
r =1
39. In a DABC, tan A and tan B satisfy the
equal to .....
inequality 3x - 4x + 3 < 0. Then which of
2

the following expressions is(are) TRUE. 45. If f ( x ) + f ( y ) + f ( xy ) = 2 + f ( x ) × f ( y ), "x , y Î R


(where, symbols used have usual meanings)? and f ( x ) is a polynomial function with f(1) ¹ 1
(a) a + b - ab < c
2 2 2
(b) a + b > c
2 2 2 and f( 4) = 17, then the value of f( 5) is
(c) a 2 + b 2 < c 2 (d) a 2 + b 2 + ab > c 2 46. A set S contains 3n members. Let Pn be the
x 4 - 4x 2
40. Let f : [2, ¥ ) ® [1, ¥ ) defined by f ( x ) = 2 probability that S is partitioned into 3 disjoint
ép ù sin x + 4 subset with members in each subset such that
and g : ê , p ú ® A defined by g( x ) = be the three largest numbers in S are in different
ë2 û sin x - 2
subsets. If lim Pn is m : n where m
two invertible functions. Then, which of the n ®¥
following statements is(are) TRUE? and n are relatively prime, then the value of
(a) The inverse of f (x) is 2 + 4 - log2 x m + n is .......
47. If D is the area of any triangle and s its 2
(b) The range of g (x) is [5, 2] s
(c) The inverse of f (x) is 2 + 4 + log2 x semi-perimeter, then minimum value of is
D
(d) The range of g (x) is [- 5, - 2] 3 1 æ 1ö
48. The value of the integral ò cosec ç x - ÷ dx
41. Let z1 , z 2 and z3 be three distinct complex 1/ 3 x è xø
numbers satisfying|z1| =|z 2| =|z3| = 1, which of is .....
the following is(are) TRUE? 49. If O (origin) is a point inside the triangle such
æz ö p æ z - z1 ö p that OP + lOQ + m QR = 0, where l, m are
(a) If arg ç 1 ÷ = , then arg ç ÷ > where | z | > 1
è z2 ø 2 è z - z2 ø 4 area of DPQR
constants such that = 4, then the
(b) | z1z 2 + z 2 z 3 + z 3 z1| = | z1 + z 2 + z 3 | area of DOQR
æ (z + z 2 )(z 2 + z 3 )(z 3 + z1) ö value of l + m is
(c) Im ç 1 ÷=0
è z1 × z 2 × z 3 ø æ z 2 - iz1 ö
50. If A( z1 ), B( z 2 ) and C ç ÷ are the vertices
(d) If | z1 - z 2 | = 2 | z1 - z 3 | = 2 | z 2 - z 3 |, then è 1- i ø
æ z - z1 ö of DABC, such that|z 2 + iz1| =|z1| +|z 2|,|z1| = 6
Re ç 3 ÷=0
è z3 - z2 ø and|z 2| = 8, then the area of DABC is

PREP CATALYSIS
94 JEE Advanced~Practice Set 1

The correct option is


Section 3 (Maximum Marks : 12) (a) P ® 1; Q ® 3; R ® 5; S ® 2
Instructions: Same as given in Physics. (b) P ® 1; Q ® 3; R ® 5; S ® 4
(c) P ® 3; Q ® 5; R ® 6; S ® 2
51. Let M be the set of all 3 ´ 3 determinant has (d) P ® 2; Q ® 4; R ® 6; S ® 1
its entries as either 1 or - 1 and let N be the 1 2
set of all minus special determinant, which is
53. Let S1 º [ f ( x )]3 - [ f ( y )]3 = 0
defined below 2 2

“We call a 3 ´ 3 determinant with entries 1 or and S 2 º [ f ( x )]3 + [ f ( y )]3 = 12


- 1 as minus special if product of elements of are two curves, satisfying the relation
any row or column is - 1, for example ( f ( x + y ) - f ( x - y )) x - ( f ( x - y ) + f ( x + y )) y
1 -1 1
= 4xy ( x 2 - y 2 )
1 1 -1 is a minus special determinant.”
-1 1 1 List I List II
1
2p +
ò0 f (y)dy equals to
P. 1. 3
List I List II
P. n (M ) equals to 1. 24 Q. Area bounded by S1 and S2 is 2. 1
Q. n (N ) equals to 2. 2 3 4

R. Minimum value of minus special 3. - 22 R. Area bounded by S2 and 3. 12 p - 24


determinant is | x| + | y| = 12 is
S. Area bounded by S1 and y = x is 4. 1
S. Maximum value of a minus 4. 29
special determinant is 6

5. - 23 The correct option is


6. 22 (a) P ® 4; Q ® 2; R ® 1; S ® 3
(b) P ® 4; Q ® 3; R ® 1; S ® 2
The correct option is (c) P ® 2; Q ® 1; R ® 3; S ® 4
(a) P ® 4; Q ® 2; R ® 5; S ® 1 (d) P ® 2; Q ® 3; R ® 1; S ® 4
(b) P ® 4; Q ® 1; R ® 3; S ® 6
(c) P ® 2; Q ® 4; R ® 3; S ® 2 54. Consider the following functions which are
(d) P ® 1; Q ® 2; R ® 5; S ® 6 defined to be 0 at x = 0
f1( x ) = x 2sgn( x ) ; f2( x ) = x -1/ 3|sin x|
52. Let ABC be an acute angled triangle with
orthocentre H and D , E , F are the feet of f3 ( x ) = x3 [- x ], where for t Î R, [t ] denotes the
greatest integer less than or equal to t.
perpendicular from A, B and C, respectively, 2x dt
on opposite sides. Also, let R be the circum f4( x ) = ò
x t
radius of DABC. If AH × BH × CH = 3 and
Then, match the elements of List I with the
( AH )2 + ( BH )2 + (CH )2 = 7, then match the
elements of List II.
elements of List I with elements of List II.
List I List II
List I List II
P. The function f1 (x) is 1. Discontinuous at x = 0
P. Value of cos A × cosB.× cosC is 1. 3
Q. The function f2 (x) is 2. Continuous but not
8R 3
differentiable at x = 0
Q. Value of cos 2 A + cos 2 B + cos 2 C is 2. 9
8R 3 R. The function f3 (x) is 3. First derivative exists at
x = 0 but second
R. Value of R is 3. 7
derivative does not exist
4R 2
S. The function f4 (x) is 4. Second derivative
4. 5
2 exists at x = 0
4R
S. Value of HD × HE × HF is 5. 3 The correct option is
2 (a) P ® 2; Q ® 3; R ® 1; S ® 4
1 (b) P ® 3; Q ® 2; R ® 4; S ® 1
6.
2 (c) P ® 3; Q ® 2; R ® 1; S ® 4
(d) P ® 2; Q ® 3; R ® 4; S ® 1

PREP CATALYSIS
JEE Advanced~Practice Set 1 95

Answers
Paper 1
1. a,c 2. a,c,d 3. b,d 4. a,c 5. b,c,d 6. b,c 7. 2.95 8. 2.88 9. 4.91 10. 35.60
11. 0.63 12. 2.45 13. 2.50 14. 2.10 15. a 16. d 17. c 18. d 19. b,c 20. a,b,d
21. b 22. a 23. a,b,d 24. b,c) 25. 4 26. 1 27. 0.05 28. 0.23 29. 0.01 30. 373.65
31. 0.72 32. 0.08 33. c 34. b 35. b 36. c 37. b,c 38. a,b,c,d 39. a,b,c 40. a,b,c
41. b,c 42. b,c 43. 0 44. 4 45. 1024 46. 6 47. 84 48. 153 49. 11 50. 0
51. a 52. c 53. b 54. c

Paper 2
1. a,d 2. b,c 3. a,c 4. c 5. b 6. a,b,c,d) 7. 0.75 8. 173.20 9. 0.10 10. 1.00
11. 1.20 12. 39.20 13. 1.24 14. 45.73 15. c 16. a 17. d 18. c 19. a,b,c,d 20. a,b
21. c 22. c 23. b 24. d 25. 7 26. 159.41 27. 1.96) 28. 9 29. 3.03 30. 1620.55
31. 3.42 32. 8.66 33. a 34. a 35. c 36. a 37. a,b,d 38. a,c,d 39. a,d 40. c,d
´10 20
41. b,c,d 42. a,d 43. 7 44. 149 45. 26 46. 11 47. 4 48. 0 49. 3 50. 25
51. b 52. a 53. c 54. b

SCORE SHEET - Paper 1


Section No. of Marks from Marks from Marks Obtained
Correct Questions Correct Questions (A) Incorrect Questions (B) (A-B)
................ ............................... ............................... ............................... ...............................
................ ............................... ............................... ............................... ...............................
................ ............................... ............................... ............................... ...............................
Percentage Marks = Marks Obtain/Total Marks x 100

SCORE SHEET - Paper 2


Section No. of Marks from Marks from Marks Obtained
Correct Questions Correct Questions (A) Incorrect Questions (B) (A-B)
................ ............................... ............................... ............................... ...............................
................ ............................... ............................... ............................... ...............................
................ ............................... ............................... ............................... ...............................
Percentage Marks = Marks Obtain/Total Marks x 100
Note To expect your success marks in the test should be between 65%-70%.

PREP CATALYSIS
PREP CATALYSIS
JEE Advanced

PRACTICE SET 2 (With Solutions)

Duration : 3 Hours Max. Marks . 360

Paper 1
PHYSICS
Section 1 (Maximum Marks : 24)
l
This section contains SIX (06) questions.
l
Each question has FOUR options for correct answer(s). ONE OR MORE THAN ONE of these four option(s) is (are)
correct options(s).
l
For each question, choose the correct options(s) to answer the question.
l
Answer to each question will be evaluated according to the following marking scheme:
Full Marks : + 4 If only (all) the correct option(s) is (are) chosen.
Partial Marks : + 3 If all the four options are correct but ONLY three options are chosen.
Partial Marks : + 2 If three or more options are correct but ONLY two options are chosen, both of which are
correct options.
Partial Marks : + 1 If two or more options are correct but ONLY one option is chosen and it is a correct option.
Zero Marks : 0 If none of the options is chosen (i.e. the question is unanswered).
Negative Marks : - 2 In all other cases.
l
For example: If first, third and fourth are the ONLY three correct options for a question with second option being an
incorrect option; selecting only all the three correct options will result in + 4marks. Selecting only two of the three correct
options (e.g. the first and fourth options), without selecting any incorrect option (second option in this case), will result in
+2 marks. Selecting only one of the three correct options (either first or third or fourth option), without selecting any
incorrect option (second option in this case), will result in +1marks. Selecting any incorrect option(s) (second option in
this case), with or without selection of any correct option(s) will result in -2 marks.

1. Position of a particle in rectilinear motion is given 2. A liquid is flowing through horizontal pipes as
by s = t 3 - 6 t 2 + 9 t. shown in figure.
Correct options for this particle are All tubes AB , EB , BG , EF , GH, FC, CH and CD are
(a) total displacement of particle in first 5 s is 20 m identical and offers very less friction to the flow of
(b) total distance travelled by particle in first 5 s is 28 m
liquid through them.
(c) particle is moving at t = 1s and t = 3 s Lengths of tubes are
L L
(d) acceleration of particle is constant L AB = L CD = EF = GH
2 2

PREP CATALYSIS
JEE Advanced~Practice Set 2 97
R GH
Radii of tubes are R AB = R EF = R CD = 4. A spherical metal shell A of radius R A and a solid
2 metal sphere B of radius R B (< R A) are kept far
Pressure at A is 2 p0 and pressure at D is p0 . apart and each is given charge +Q. Now, they are
Volume flow rate through tube AB is Q. connected by a thin metal wire. Then,
Now, choose the correct options. (a) EAinside = 0 (b) QA > QB
E F s R
(c) A = B (d) EAon surface < EBon surface
sB R A
Q
A B C D 5. A closed container having a volume of 0.02 m 3
contains a mixture of neon and argon at 27°C and
G H pressure of 1 ´ 10 5 Nm -2. The mass of mixture is
(a) Volume flow rate through EF =
Q 28 g.
17 Now, choose the correct options.
16 (a) Mass of neon in mixture is 4.074 g
(b) Volume flow rate through GH is Q
17 (b) Mass of argon in mixture is 23.926 g
(c) Pressure at E is nearly æç
55 ö (c) Mass of neon in mixture is 23.926 g
p ÷
è 36 0 ø (d) Mass of neon in mixture is 4.074 g

(d) Pressure at E is nearly æç


47 ö 6. A particle of mass 0.01 kg is moving in region of
p ÷
è 36 0 ø æ Jö
potential energy function; V(x) = 0 .08 x 2 ç ÷ ,
3. For a H-like ion, 47.22 eV energy is required to è kg ø
excite the electron from n = 2 to n = 3 state. where x is displacement in metre. If total energy of
Assuming, Bohr’s model and mark the correct particle is 8 ´ 10 -4 J, then correct options are
options.
(a) angular frequency of oscillation of particle is 0.4 rad/s
(a) Number of photons in nucleus is 5 (b) amplitude of oscillation of particle is 1 m
(b) Total energy of ion in ground state is 340 eV (c) displacement of particle is x = sin (0.4t + f)
(c) Kinetic energy of electron in ground state is 340 eV
(d) displacement of particle is x = cos (0.4t + f)
(d) Potential energy of ion in ground state is - 680 eV

Section 2 (Maximum Marks : 24)


l
This section contains EIGHT (08) questions. The answer to each question is a NUMERICAL VALUE.
l
Four each question, enter the correct numerical value (in decimal notation, truncated/rounded-off to the second
decimal place; e.g. 6.25, 7.00.- 0.33, -.30, 30.27, -127.30) using the mouse and the on-screen virtual numeric
keypad in the place designated to enter the answer.
l
Answer to each question will be evaluated according to the following marking scheme:
Full Marks : + 3 If ONLY the correct numerical value is entered as answer.
Zero Marks : 0 In all other cases.

7. A point moves along a circle with velocity v = at, 9. Two smooth spheres A and B of equal radius, but
where a = 0.5 ms -2 . Find the total acceleration of the mass m and M are free to move on a horizontal
table. A is projected with speed u towards B, which
particle at the moment, it covered the nth fraction of
is at rest. On impact, the line joining their centres
circle (n = 0.10) after beginning of motion.
is inclined at an angle q to the velocity of A before
8. A ring of radius 4a is rigidly fixed in vertical impact. Let e is the coefficient of restitution
position on a table. A small disc of mass m and between the spheres. If A’s path after impact is
radius a is released as shown in perpendicular to its path before impact, then find
the figure. When the disc rolls the coefficient of restitution. [Take, m = 5 kg,
down without slipping to the M = 10 kg, q = 30 °)
lowest point of the ring, then its a
10. A student determines a dimensionless quantity,
speed (in m/s) will be 4a
en
(Take, g = 10 m / s 2 and B= .
Ze 0 hc
a = 0.4 m)
where, e = electric charge,

PREP CATALYSIS
98 JEE Advanced~Practice Set 2

e 0 = permittivity of vacuum, original volume. Taking air diatomic, find change


h = Plank’s constant in internal energy of air (in kJ).
and c = speed of light. 13. A uniform inextensible string of length l and total
Find the value of n. mass M is suspended vertically and tapped at the
top, so that a transverse impulse runs down it. At
11. A copper piece of mass 10 kg is suspended by a the same moment, a body is released from rest and
vertical spring. The spring elongates 1 cm over its it falls freely from top of the string. How far from
natural length to keep the piece in equilibrium. A the bottom does the body pass the impulse?
beaker containing water is now placed below the (Take, length of string, l = 1m)
piece, so as to immerse the piece completely in
water. If the elongation of the spring is 14. A planet is orbiting a massive star in a nearly
89 ´ 10 - n cm, then find the value of n. (Take, circular orbit. The star undergoes an internal and
spherically symmetric explosion. Due to explosion,
density of copper = 9000 kg/m and g = 10 m/s ) 3 2
1% of mass of star is ejected and thrown into space
12. A diesel engine takes in 5 mol of air at 20°C and 1 well beyond planet’s orbit. Assume that planet
1 does not affected by explosion. Find the ecentricity
atm; and compress it adiabatically th of its of new orbit.
10

Section 3 (Maximum Marks : 12)


l
This section contains TWO (02) paragraphs. Based on each paragraph, there are TWO(02) questions.
l Each question has FOUR options. ONLY ONE of these four options corresponds to the correct answer.
l
Four each question, choose the option corresponding to the correct answer.
l
Answer to each question will be evaluated according to the following marking scheme:
Full Marks : + 3 If ONLY the correct option is chosen.
Zero Marks : 0 If none of the options is chosen (i.e. the question is unanswered).
Negative Marks : - 1 In all other cases.

Paragraph X Paragraph A
Rolling is the combination of translation and rotation. The Consider given parallel L-C-R circuit,
point of contact plays a crucial role as it decides the (Take, tan -1 ( 4 /3) = 53° )
direction of friction, a uniform disc of mass M and radius R Box-1
is given in linear velocity v 0 and angular speed w0 as R1=3Ω XL=4Ω
shown in the diagram on a rough horizontal surface.
2v
(Take, at t = 0 , w = w0 = 0 )
R XC=10Ω
ω0
v0

R
Rough V=200√2 sin(ωt+π/4)
15. The graph of angular speed of disc about its centre
is best represented by 17. Instantaneous current in branch having capacitor C
ω ω ω ω will be
(a) (b) (c) ω0 (d) (a) 20 2 sin (wt + 3p / 4) (b) 40 2 sin (wt + 3p / 4)
(c) 60 2 sin (wt - p / 4) (d) None of these
t t t t
t0 t0 t0 t0
18. Angle between current through inductor and
16. The velocity of centre of mass of disc when pure capacitor will be
rolling starts will be equal to (a) 143° (b) 90°
1 5v 0 v0 (c) 53° (d) None of these
(a) v0 (b) (c) (d) zero
3 6 7

PREP CATALYSIS
JEE Advanced~Practice Set 2 99

CHEMISTRY
Section 1 (Maximum Marks : 24) (b) O CON3
∆ H 3O
+

Instructions : Same as given in Physics


O
19. ‘M’ is a first row transition metal. MCl 2 on OH
– +
H 3O
(c) O C NHOH
treatment with aqueous ammonia gives a blue
coloured solution of complex ‘N’. A solution of
MCl2 gives a bright red precipitate of complex ‘O’ O
+ +
with ethanolic dmg. The correct statement N3H/H H 3O
(d) O COH ∆
regarding the given reactions are
(a) structure of ‘N’ is [Ni(NH3 )6 ]Cl 2
24. 100 mL samples of two different acids X and Y are
(b) hybridisation of ‘M ’ is sp3d 2 in complex ‘N ’
titrated with 0.1 M NaOH giving the following
(c) ‘N ’ is a diamagnetic complex titration curve
(d) metal ‘M ’ is iron
10 X
20. The correct statement(s) regarding interhalogen 8 Y
compounds is/are
pH 6
(a) the bonds are covalent as the electronegativity 4
difference between them is small 2
(b) on hydrolysis gives halide and oxohalide ions
(c) ICl and IBr is used as aqueous ionising solvents O 10 20 30 40
(d) ClF3 is trigonal bipyramid in shape
mL of NaOH
21. Which of the following statement is correct for If both X and Y are monobasic acids, then the
group 14 elements? correct statement(s) is / are
(a) CO2 is more acidic than H2O (a) X has higher concentration than Y
(b) SiO2 is a neutral oxide (b) X is a stronger acid
(c) PbI4 is least stable and has doubtful existence (c) Molarity of acid X and Y are 0.01 and 0.02 respectively
(d) Pb4+ compounds are stronger oxidising agents than (d) Salts of acid Y will have greater pH in aqueous solution if
Sn2+ compounds their concentrations are same
22. In the following sequence of reaction, (X) is
O Section 2 (Maximum Marks : 24)
(1) S O,
Instructions : Same as given in Physics
O
Cl 25. A list of species is given below:
Pyridine
X –
(2) AcO , DMF
O H [Ni(CN)4 ]2- , PF5, [Pt(Cl)4 ]2- , BrF5, SF6 , NH +4 , PCl 5,
[CrF6 ]3- , [Co(NH 3)6 ]3+ , CH 4 , [Ni(CO)4 ]
(R)
H OH H OH If X is metal and Y is ligand then define the shape
on the basis of the location of X and Z atoms, the
(a) (b)
total number of species having a square planar
shape is ........
(c) OH 26. At 27°C, two balloons of equal volume and porosity
(d) H
are filled to a pressure of 3 atm, one with 14 kg N 2
H
H and other with 1 kg of H 2 . The N 2 balloon leaks to
23. The reactions leading to the formation of a pressure of 1/2 atm in 30 mins. The time required
for H 2 balloon to reach a pressure of 1/2 atm will
O NH2 is/are be ……… mins.

O
27. The figure given below shows the unit cell of
σ +
compound which is a mixed oxide of zirconium,
Br2/OH H3 O
(a) O C NH2 calcium and nickel. The formula of mixed oxide is
ZraCa b NicOd . The value of (a + b + c + d) is……

PREP CATALYSIS
100 JEE Advanced~Practice Set 2

Ni Ni 32. The concept of kinetic versus thermodynamic


O control of reaction products has frequently been
Ni
Cube 1 ⇒ Ni employed in organic synthesis to direct product
formation, for example, in sulphonation diels -
Ca Ni alder isomerisation and addition reactions. It is
Ni Ni ⇒ At corner
⇒ Oxygen (edge normally represented as a concurrent reaction
Ni O
Cube 2 ⇒ Ni centered) scheme, as shown below for the case where the
Zr Ca ⇒ Body center reaction of A proceeds competitively towards B
Ni Ni
Zr ⇒ Body center and C
Ni k1 k2
Cube 3 ⇒ Ni B ¾®
¾®
A ¾®
¾®
C
k -1 k -2
Ni Ni
The energy profile for the reaction is depicted in the
figure below:
Ni Ni

28. The concentration of Ca 2+ in sea water is


determined using a Ca 2+ selective electrode. A
∆E
10 mL sample of sea water is diluted to B
100 mL and its 50 mL aliquot is placed in a beaker
A
with Ca 2+ ion selective electrode and a refrence C
electrode. The emf was found to be -0 .053 V. A
1.0 mL aliquot of 0.05 M Ca 2+ ion is added and In first 4 min of the reaction, the product ratio of
emf was found to be -0 .0422 V. The concentration B/C is 1/0.1. Using the same rate constants, the
product ratio [B]/[C] when the reaction time
of Ca 2+ in sea water will be ………
exceed 4 days will be. [Given, k1 =1, k -1 = 0 .01,
Note Ignore volume change due to addition of second aliquot.
k 2 = 0 .1, k -2 = 0 .0005 min -1 ]
æ 3 ö
29. 3 moles of an ideal gas çCV , m = R ÷ are initially in
è 2 ø
an insulated piston - fitted cylinder at a constant Section 3 (Maximum Marks : 12)
pressure. The volume of the gas is initially 40 L. Instructions : Same as given in Physics
The cylinder is then placed in contact with a
constant temperature bath at 500 K. Bringing the Paragraph X
bath in contact with the cylinder causes 6.236 kJ An organic compound ‘A’ gives positive Libermann’s
of heat to transfer into the cylinder and causes the reaction and forms compound ‘B’ and ‘C’ on treatment
gas to expand. The value of DS total for this process with trihalogen derivative of methane in alkaline medium
is ……… followed by hydrolysis. C is steam volatile and gives no
colouration with Schiff’s reagent, which is given by
Constant temp.

compound ‘B’. ‘C’ when reduced with lithium aluminium


500 K

∆Stotal =?
Gas Constant Gas Constant hydride forms ‘D’ with formula C 7 H 8 O 2 . ‘D’ when
oxidised forms ‘E’ which on reaction with anhydride in the
presence of corresponding acid forms a pain reliever ‘F’.
33. ‘B’ and ‘C’ are isomeric but ‘C’ does not give colour
30. The density of an equilibrium mixture of N 2O 4 and with Schiff ’s reagent. It is because of the fact that
NO2 at 1.0 atm is 3.62 g/L at 288 K and 1.84 g/L at (a) ‘B’ contains an aldehyde group and ‘C’ contains keto
348 K. The enthalpy of reaction for group in addition to phenolic group
N 2O 4 - 2NO 2 will be ……… kJ. (b) ‘B’ contains keto group and ‘C’ contains aldehyde group
in addition to the phenolic group
31. For the indicator thymol blue, pH is 2.0 when half (c) carbonyl reactivity in ‘C’ is marked by intermolecular
of the indicator is in unionised form. The H-bonding
percentage indicator in unionised form in a (d) carbonyl group is lost in hydrolysis step
solution with [H + ] = 4 ´ 10 -3 M is ………

PREP CATALYSIS
JEE Advanced~Practice Set 2 101

34. Compound D, E and F respectively are: O

OH OH OH
CH2OH COOH COOCH3 (c) (d)

O Et Me

36. The structures of products (Q) and (R) respectively


I II III
are:
OCOCH3 OH COOH O
COOH CHO
H
(a) ;
Me COMe

IV V VI Me Me Me Me
O
The correct option is
(a) I, II and III (b) I, II and IV H
(c) V, II and IV (d) II, VI and III (b) ;
CHO
Paragraph A Me Me Me Me
A carbonyl compound (P) which gives positive iodoform
test, undergoes reaction with MeMgBr followed by O
dehydration to give an olefin (Q). Ozonolysis of (Q) leads
to a dicarbonyl compound (R) which undergoes H
(c) ;
intramolecular aldol reaction to give predominantly (S). CHO
1. MeMgBr 1. O3 1.OH - Me Et Me Et
P ¾¾¾¾¾® Q ¾¾¾® R ¾¾¾® S
+
2. H , H 2O 2. Zn, H 2O 2. D Me O
3. H 2SO4 , D

35. The structure of the carbonyl compound (P) is CH3


(d) ;
Me CHO
(a) (b) Me Et
Me
O Me O Me

MATHEMATICS
x x
Section 1 (Maximum Marks : 24) æ5ö æ12 ö
38. Let f (x) = ç ÷ + ç ÷ - 1 for all x Î R. Then,
è13 ø è13 ø
Instructions : Same as given in Physics
which of the following statement(s) is (are) true?
37. Let P1 denotes the equation of a plane to which the
(a) The equation f( x) = 0 has exactly one solution in R
vector $i + $j is normal and which contains the line L (b) f( x) is strictly increasing for all x Î R
whose equation is r = $i + $j + k
$ + l($i - $j - k
$) and P (c) The equation f( x) = 0 has exactly two solution in R
2
(d) f( x) is strictly decreasing for all x Î R
denotes the equation of the plane containing the
line L and a point with position vector $j. Which of 39. The sides of a right triangle T1 are 20, x and
the following holds good? hypotenuse y. The sides of another right triangle T2
(a) The equation of P1 is x + y = 2
are 30, x - 5 and hypotenuse y + 5. If P1 , P2 are the
radii of the circles inscribed in the triangles T1 , T2
(b) The equation of P2 is r × ($i - 2j$ + k$ ) = 2
respectively, then which of the following is/are
(c) The acute angle between P1 and P2 is cot - 1( 3 ) TRUE?
Area of triangle T1 5
(d) The angle between the plane P2 and the line L is (a) = (b) Ratio of P1 to P2 is 1 : 2
tan- 1( 3 ) Area of triangle T2 6

PREP CATALYSIS
102 JEE Advanced~Practice Set 2

644444448

844444446
(c) Radii of both circles are equal x
Area of triangle T1 7 47. lim 3
equals to
(d) = x ®¥ x
Area of triangle T2 8 x+ 3
x
x x+
40. Let g (x) = x e andf (x) = ò0 te (1 + 3 t )
a ax 2t 2 1/ 2 3
dt. If x
x+
f ¢ (x) .... infinity
L = lim is non-zero finite number. Then,
x ® ¥ g ¢ (x)
48. The product of all values of x satisfying the
which of the following is(are) TRUE? equation,
3 3
(a) The value of a is (b) The value of L is æ 2 x 2 + 10 |x| + 4 ö ì -1 æ 2 - 18 |x|öü
2 2 sin - 1cos ç 2 ÷ = cot ícot ç ÷ý
3 è x + 5 |x| + 3 ø î è 9 |x| øþ
(c) The value of a is 2 (d) The value of L is
4 p
+ is
41. Let f (x) = ax 2 + bc + c , a > 0 and f (2 - x) = f (2 + x), 2
"x Î R and f (x) = 0 has 2 distinct real roots. Then, 49. A natural number is chosen at random from the
which of the following is(are) TRUE? first 100 natural numbers. The probability that
100 m
(a) At least one root must be positive x+ > 50 is , then m + n is equal to ....
(b) f(2 ) < f(1) < f(0) x n
(c) Minimum value of f( x) is negative (where, m and n are coprime).
(d) Vertex of graph of y = f( x) lies in IIIrd quadrant.
50. If f (x) = ax 2 + bx + c is such that|f (0)| £ 1,|f ()|
1 £1
42. If point M moved on the circle
and|f (- 1)| £ 1, then the maximum value of|f (x)| if
(x - 4)2 + (y - 8)2 = 20 , then, it broke away from it
x Î[ - 1, 1].
and moving along a tangent to the circle, cuts the
X-axis at the point (- 2 , 0). The coordinates of a
point on the circle at which the moving point Section 3 (Maximum Marks : 12)
broke away, is Instructions : Same as given in Physics
(a) æç , ö÷ (b) æç - , ö÷ (c) (6, 4)
42 36 2 44
è 5 5ø è 5 5ø
(d) (2, 4) Paragraph X
If z = r (cos q + i sin q ) and n is a positive integer, then
é (2kp + q ) (2kp + q) ù
Section 2 (Maximum Marks : 24) z 1/n = r 1/n ê cos + i sin ú where, k = 0, 1,
ë n n û
Instructions : Same as given in Physics
2, 3,. K (n - 1), gives n, nth roots of the complex number z.
43. A man has 3 friends. If N is the number of ways he
can invite one friend everyday for dinner on 51. If 1, a 1 , a 2 , ... , a n - 1 are the distinct nth roots of
6 successive nights so that no friend is invited unity, then
more than 3 times, then the value of N is ....... 1 + (- 1)n
(a) (1 + a1 )(1 + a2 )...(1 + an - 1 ) =
44. Let f : R ® R be a function defined by 2
1 - (- 1)n
(b) (1 + a1 )(1 + a2 ) ... (1 + an - 1 ) =
ì[ x], x £ 2 2
f (x) = í , where [x] is the greatest integer
æ pö æ 2p ö æ pö æ (n - 1)p ö 1 + (- 1)n
î 0, x > 2 (c) cosç ÷ cosç ÷ cosç 3 ÷ ... cosç ÷ =
è nø è nø è nø è n ø 2
2 xf (x 2)
less than or equal to x. If I = ò- 1 2 + f (x + 1) dx, then (d) cos 4° cos 8° cos 12° .... cos 88° = 23
2
1

the value of I is equal to ………


52. If 1, w, w2 , ... , wn - 1 are the n, nth root of unity and
-l
45. Let l = log 5 (log 53). If 3k + 5 = 405, then the value z1 and z 2 are any two complex numbers, then
n -1
of k is equal to
å | z1 + wk z2 |2 is equal to
46. Let|a| = 1,|b| = 1 and|a + b| = 3. If c is a vector k=0

such that c = a + 2 b - 3 (a ´ b) and l = |(a ´ b) ´ c|, (a) n[| z1 |2 + | z2 |2 ] (b) (n - 1) [| z1 |2 + | z2 |2 ]


then the value of 4 l2 is equal to
(c) (n + 1) [| z1 |2 + | z2 | 2 ] (d) (n + 2 ) [| z1 |2 + | z2 |2 ]

PREP CATALYSIS
JEE Advanced~Practice Set 2 103

Paragraph A 54. Let l(x - h) + m (y - k) = 1 represents the chord AB


Consider the conic whose equation is p
of the above conic such that ÐACB = , where
4 x 2 - 9y 2 - 8x - 36y + 4 = 0. 2
53. The eccentricity of the conic is c (h , k) is the centre of conic, then l + m 2 is
2

13 13 1 1 1 1 5
(a) (b) (c) (d) 2 (a) (b) (c) (d)
3 2 3 9 12 18 36

Paper 2
PHYSICS

Section 1 (Maximum Marks : 24)


l This section contains SIX (06) questions.
l Each question has FOUR options for correct answer(s). ONE OR MORE THAN ONE of these four option(s) is (are)
correct options(s).
l For each question, choose the correct options(s) to answer the question.
l Answer to each question will be evaluated according to the following marking scheme:
Full Marks : + 4 If only (all) the correct option(s) is (are) chosen.
Partial Marks : + 3 If all the four options are correct but ONLY three options are chosen.
Partial Marks : + 2 If three or more options are correct but ONLY two options are chosen, both of which are
correct options.
Partial Marks : + 1 If two or more options are correct but ONLY one option is chosen and it is a correct option.
Zero Marks : 0 If none of the options is chosen (i.e. the question is unanswered).
Negative Marks : - 2 In all other cases.
l
For example: If first, third and fourth are the ONLY three correct options for a question with second option being an
incorrect option; selecting only all the three correct options will result in + 4marks. Selecting only two of the three correct
options (e.g. the first and fourth options), without selecting any incorrect option (second option in this case), will result in
+2 marks. Selecting only one of the three correct options (either first or third or fourth option), without selecting any
incorrect option (second option in this case), will result in +1marks. Selecting any incorrect option(s) (second option in
this case), with or without selection of any correct option(s) will result in -2 marks.

1. Students of class XIth are determining Young’s (c) Maximum error in Young’s modulus is due to
modulus of a wire in their school laboratory. Their incorrectness of diameter
observations are (d) Maximum fractional error in measured value of
Young’s modulus is 0.0489
(i) A weight of 50 N produces an extension of
0.125 cm. This is measured by a micrometer of 2. A H-like atom has its ground state binding energy
least count 0.001 cm of 122.4 eV.
(ii) Diameter of wire is 0.050 cm. This is measured Now, choose the correct options.
by a screw gauge of least count 0.001 cm (a) Its atomic number is 3
(b) An electron of 90 V can excite it, when atom is in ground
(iii) Length of wire is 110 cm. It is measured by a state
scale of least count 0.1 cm (c) An electron of 91.8 eV can be brought almost to rest by
Now, choose the correct options. this atom
(a) Maximum error in observation of Young’s modulus is (d) An electron of kinetic energy of 2.6 eV may emerge,
due to incorrectness of length when electron of kinetic energy of 125 eV collides with
(b) Maximum error in Young’s modulus is due to this atom
incorrectness of extension

PREP CATALYSIS
104 JEE Advanced~Practice Set 2

3. A 100 mW laser beam falls over a surface of Now, choose the correct options.
1 m 2. (a) L2 = L1 [1 + a(T2 - T1 )], whereT2 - T1 and a are not large
Now, choose the correct options. (b) L2 = L1e a( T2 - T1 ) , where a is constant
T2
(a) If it is reflected from surface, then force on surface ò a dT
is 6.67 ´ 10-10 N (c) L2 = L1e T1 , where a is a function of temperature
(b) If it is reflected from surface, then force on surface (d) Formula in option (a) is valid in all conditions
is 3.33 ´ 10-10 N 6. A vertical square loop of copper wire with sides of
(c) If it is absorbed by surface, then force on surface length a is allowed to fall through a region, where
is 3.33 ´ 10-10 N the magnetic field is horizontal and into a region,
(d) If it is absorbed by surface, then force on surface
where field is zero.
×
is 6.67 ´ 10-10 N × ×

4. A current of 1 mA extens at point A of given ×


× ×
B=1.2T
network of resistances.
× ×
2kW

A C D B
1kW 1kW 1kW a B=0

2kW Magnitude of magnetic field is B and diameter of


wire is d.
Now, choose the correct options. [Take, resistivity of copper is r c in W-m and density
(a) Current through 2kW resistors is
2
mA of copper is r m in kgm -3 ]
5
3 Now, choose the correct options.
(b) Current through branch AC is mA
5 (a) In loop, current induced flows in clockwise sense
1 pBvd 2
(c) Current through branch CD is mA (b) Magnitude of induced current in loop is I =
5 16rc
3
(d) Current through branch DB is mA (c) Magnetic force on loop has a magnitude of
5 pB2 vd 2 a
F=
5. A rod of original length L1 changes its temperature 16rc
16rc r m g
from T1 to T2 . Let a is temperature coefficient of (d) Terminal speed of loop is v =
B2
linear expansion and L 2 is length of temperature
T2 .

Section 2 (Maximum Marks : 24)


l
This section contains EIGHT (08) questions. The answer to each question is a NUMERICAL VALUE.
l
Four each question, enter the correct numerical value (in decimal notation, truncated/rounded-off to the second
decimal place; e.g. 6.25, 7.00.- 0.33, -.30, 30.27, -127.30) using the mouse and the on-screen virtual numeric
keypad in the place designated to enter the answer.
l
Answer to each question will be evaluated according to the following marking scheme:
Full Marks : + 3 If ONLY the correct numerical value is entered as answer.
Zero Marks : 0 In all other cases.

7. A rocket mass 50 kg contains a fuel of 450 kg. 8. A bullet (m = 0.012 kg) and horizontal speed of
Exhaust velocity of expelled gases is 2 km/s. 70 ms-1 strikes a block of wood of mass 0.4 kg
Minimum rate of fuel consumption for vertical (which is suspended by a string) and gets embeded
take off is .................... . in it. Heat produced will be ...................... .

PREP CATALYSIS
JEE Advanced~Practice Set 2 105

9. A spring having a spring constant 1200 Nm -1 is 11. A block of mass 1 kg and density 0.8 g/cm 3 is held
mounted on a horizontal table as shown in figure. stationary with the help of a string as shown in
A mass of 3.0 kg is attached to the free end of the figure. Let the tank is accelerating vertically
spring. The mass is then pulled sideways to a upwards with an acceleration a = 10 . m / s2 .
distance of 0.2 cm and released. Determine the
maximum acceleration of the mass.
a

If the string is now cut, then find the acceleration of


block.
10. General theory of relativity by Einstein (in 1916) (Take, g = 10 m / s2 and density of water
gives principle of equivalence. It follows from = 10 3 kg/ m 3 )
equivalence principle that light should subjected to 12. Two separate air bubbles (radii 0.004 m and
gravity. This prediction is confirmed by taking 0.002 m) formed of the same liquid (surface tension
photographs of distant stars. 0.07 N/m) come together to form a double bubble.
Light from a distant pulsar, when received on earth If R is the radius of curvature of the internal film
seems to come identically from two distant sources surface common to both the bubbles (in cm), then
as shown in figure. find the value of 10R.
Apparent 13. A thermodynamic system is taken from an initial
position state D to an intermediate state E by linear process as
of pulsar shown in figure. Its volume is then reduced to the
origin value from E to F by an isobaric process.
Calculate the total work done by the gas from D to E to
E Marking Pulsar F.
Earth object
D

Apparent
position F E

Now, if mass of sun is 2 ´ 10 30 kg and its radius is


7 ´ 10 8 m and it causes approximate red shift of N 14. An a-particle collides head on with a stationary
pm in the light emitted by sun, find the value of N. proton elastically. Find the ratio of de-Broglie’s
wavelengths associated with a-particle and photon
[Take, wavelength of light emitted by
after collision.
sun = 500 nm]

PREP CATALYSIS
106 JEE Advanced~Practice Set 2

Section 3 (Maximum Marks : 12)


l This section contains FOUR (04) questions.
l Each question has TWO (02) matching lists : List-I and List-II.
l FOUR options are given representing matching of elements from List-I and List-II. ONLY ONE of these four options
corresponds to a correct matching.
l For each question, choose the option corresponding to the correct matching.
l For each question, marks will be awarded according to the following marking scheme.
Full Marks : + 3 If ONLY the option corresponding to the correct matching is chosen.
Zero Marks : 0 If none of the options is chosen (i.e. the question is unanswered).
Negative Marks : - 1 In all other cases.

15. A right angled prism of refractive index m1 is placed 17. One mole of a monoatomic ideal gas is taken along
in a rectangular block of refractive index m 2 , which two cyclic processes E ® F ® G ® E and
is surrounded by a medium of refractive index m 3 , E ® F ® H ® E as shown in the p-V diagram.
as shown in the figure. A ray of light e enters the p
rectangular block at normal incidence. Depending F
32p0
upon the relationships between m1 , m 2 and m 3 , it
takes one of the four possible paths ef, eg, eh or ei.
f

e 45° g
p0 G
H
m1 E
h V
i V0

m2 m3 The processes involved are purely isochoric,


isobaric, isothermal of adiabatic.
Match the paths in List I with conditions of Match the paths in List I with the magnitudes of
refractive indices in List II and select the correct the work done in List II and select the correct
answer using the codes given below the lists. answer using the codes given below the lists.
List I List II List I List II
P. G®E 1. 160 p0V0 ln2
P. e ®f 1. m1 > 2 m 2
Q. G®H 2. 36 p0V0
Q. e ®g 2. m 2 > m1 and m 2 > m 3
R. F ®H 3. 24 p0V0
R. e ®h 3. m1 = m 2
S. F ®G 4. 31 p0V0
S. e ®i 4. m 2 < m1 < 2 m 2 and
m2 > m3 Codes
PQRS PQRS PQRS PQRS
Codes (a) 4 3 2 1 (b) 4 3 1 2 (c) 3 1 2 4 (d) 1 3 2 4
PQRS PQRS PQRS PQRS
(a) 2 3 1 4 (b) 1 2 3 4 (c) 4 1 2 3 (d) 2 3 4 1 18. Match List I of the nuclear process with List II
containing parent nucleus and one of the end
16. Match List I with List II and select the correct products of each process and then select the correct
answer using the codes given below the lists. answer using the codes given below the lists.
List I List II List I List II
P. Boltzmann constant 1. [ML2T -1 ] P. Alpha decay 1. 8 O ® 7 N + .....
15 17

Q. Coefficient of viscosity 2. [ML-1T-1 ] Q. b + decay 2. 92 U ® 90 Th + ....


238 234
-3 -1
R. Planck constant 3. [MLT K ]
83 Bi ® 82 Pb + .....
185 184
R. Fission 3.
S. Thermal conductivity 4. [ML2T -2K -1 ]
94 Pu ® 57 La + ....
239 140
S. Proton emission 4.

Codes Codes
PQRS PQRS PQRS PQRS PQRS PQRS PQRS PQRS
(a) 3 1 2 4 (b) 3 2 1 4 (c) 4 2 1 3 (d) 4 1 2 3 (a) 4 2 1 3 (b) 1 3 2 4 (c) 2 1 4 3 (d) 4 3 2 1

PREP CATALYSIS
JEE Advanced~Practice Set 2 107

CHEMISTRY
O
Section 1 (Maximum Marks : 24) O O
OHC
CHO
Instructions : Same as given in Physics (c) ; OHC C C
19. One of the most general reaction exhibited by CHO
coordination compounds is that of substitution or
replacement of one ligand by another. When O O O
tetrachloroplatinate (II) ion reacted with ammonia CHO OHC
yield X and tetrammineplatinum (II) ion reacted (d) ; OHC C C
with chloride ion yield Y, here X and Y are CH2OH
(a) X is cis-diamminedichloroplatinum (II)
Y is trans-diamminedichloroplatinum (II) 22. Consider the following compound:
(b) X is trans-diamminedichloroplatinum (II) HOH2C H
Y is cis-diamminedichloroplatinum (II) O
(c) Y is trans-diamminedichloroplatinum (II) HH H OH
X is triamminechloroplatinum (II)
OH OH
(d) Y is cis-diamminedichloroplatinum (III)
X is triamminechloroplatinum (II) The correct structure of its anomer is
HOH2C H HOH2C OH
20. A soluble compound of a poisonous element M, O O
when heated with Zn/ H 2SO4 gives a colourless (a) (b)
H OH H OH HH HH
and extremely poisonous gaseous compound N
which on passing through a heated tube gives a H OH OH OH
silvery mirror of element M. The correct statement
is/are H OH HOH2C H
O O
(a) ‘N’ is CdH3 (c) (d)
(b) ‘M ’ is arsenic, silvery in colour HOH2C
OH OH H HH OH OH

(c) on heating XH3 , hydrogen is evolved H H OH H


(d) the given element (M) is insoluble in water
23. When the solids Ba(OH)2 and NH 4SCN are mixed,
21. Benzaldehyde reacts with cyclopentanone in a solution in produced and the temperature drops.
presence of a base to give (A) that on oxidation
Ba(OH)2(s) + 2NH 4SCN(s) ¾®
produces (B). (A) also produces C and D on
ozonolysis followed by reduction. The reaction Ba(SCN)2(aq) + 2NH3 (g) + 2H 2O()
l
sequence(s) are as follows: Which statement about the energies of this reaction

is / are incorrect?
(i) OH KMnO4/H+ (a) The reaction is endothermic and DH is negative
PhCHO + O (A) B
(ii) H+ Only one predict (b) The reaction is endothermic and DH is positive
(c) The reaction is exothermic and DH is negative
(d) The reaction is exothermic and DH is positive
(C) + (D)
Two products
24. One mole of an ideal gas defined by the state A
(300 K, 2 atm) is subjected to the following change
The two products (C) and (D) are of state:
O Adiabatic rev.
Isochoric
A ¾¾¾¾¾® B (5.0atm) ¾¾¾¾®
CHO heating expansion
(a) ;
Isothermal rev.
CHO C ¾¾¾¾¾® A, CV =15
. R
compression
O O
H
CHO Which of the following p - V diagram describes the
(b) ; OHC C O above-mentioned cyclic process most appropriately?
CHO

PREP CATALYSIS
108 JEE Advanced~Practice Set 2

B A B 30. Two buffers (X) and (Y) of pH 6.0 and 4.0


V V respectively are prepared from acid HA and the salt
(a) (b) Na A. Both the buffers are 0.50 M in HA. The pH of
A the solution obtained by mixing equal volumes of
C C two buffers will be ……… (Given, K HA -10 . ´ 10 -5)
p p 31. 64 g of methanol at a temperature of 60°C are
C C poured into 128 g of methanol kept in an insulated
V V vessel. The net change in entropy is ………
(c) (d) 32. In a given first order reaction, A n + is getting
converted to A(n + 4)+ in solution. The rate constant
A B A B of this reaction is measured by titrating a volume
p p of the solution with a reducing agent which reacts
only with A n + and A(n + 4)+ . In this process, it
converts A n + to A(n - 2)+ and A(n + 4) + to A(n - 1) + .
Section 2 (Maximum Marks : 24) At t = 0, the volume of reagent consumed is 30 mL
Instructions : Same as given in Physics and at t = 10 min, the volume used is 42 mL. The
value of rate constant for the conversion of A n + to
25. Among the following, the number of compounds A(n + 4)+ is ………
that can react with ClF to give Cl 2 is ………
Al, SF4, Co, U, S, SO 2 Section 3 (Maximum Marks : 12)
26. 1.22 g sample of Na 2CO3 and K 2CO3 was dissolved Instructions : Same as given in Physics
in water to form 100 mL of a solution. 20 mL of 33. Match the molecules given in List I with their
this solution required 40 mL of 0.1 N HCl for properties given in List II.
complete neutralisation. If another 20 mL of this
solution is treated with excess of BaCl2, then the List I List II
mass of the precipitate will be ………… I. H 2N 2O 2 p. s and p-bonds are present.
II. N 2O 4 q. Exists as cis, trans isomers.
27. Consider the given half-cell reactions,
III. N 2O 5 r. N¾N bond length is greater than
Co(CN)36- + e- ¾® Co(CN)6+ , E°RP = - 0 .83V theoretical value.
IV. HNO 2 s. Oxidising, reducing and
Co3 + + e- ¾® CO2+ , E°RP = 1.82V complexing agent.
The overall formation constant for the reaction of 6 The correct option is
moles of CN - with cobalt (II) is1 ´ 10 19 . (a) I ® p,q ; II ® p,r ; III ® p ; IV ® p,s
The formation constant for the reaction of 6 moles (b) I ® q ; II ® r,s ; III ® p ; IV ® q,s
of CN - with cobalt (III) is ……… ´ 1063 . (c) I ® p ; II ® q ; III ® r ; IV ® s
(d) I ® s ; II ® r ; III ® q ; IV ® p
28. The total number of stereocenters in the given
34. The desired product ‘X’ can be prepared by reacting
structures are.
the major product of the reactions in List I with one
or more appropriate reagents in List II.
(X)
O

List I List II
HO P. C 15H10O 5 (corncobs) +10% 1. Pd/C, 200°C
H 2SO 4 , NaCl+ heat
29. 1.5 g of a hydrated organic compound ‘A’, 2. Acid
containing C, H and O, on mild heating rendered
Q. Ribose - 5- phosphate + 3. H 2SO 4
anhydrous and leave behind 1.07 g of solid residue ribonucleotide reductase
‘B’. 0.675g of ‘B’ on complete combustion produced (enzyme)
0.66 g of CO2 gas and 0.135 g of water vapour. If
4. O 2, 200°C
1.12g compound A was burnt in excess of O2 (g) the
weight of water molecules produced will be. R. HO ¾(CH 2 )4 ¾OH + PCC 5. HCl + HCN

PREP CATALYSIS
JEE Advanced~Practice Set 2 109

(a) P®1 ; Q ® 2,4 ; R ® 3 Codes


(b) P ® 2 ; Q ® 2, 3 ; R ® 1 (i) (ii) (iii) (iv)
(c) P ® 3 ; Q ® 3,4 ; R ® 2 (a) p-r, p, q-s ,q
(d) P ® 1; Q ® 2 ; R ® 5 (b) t-s, r, p, q
(c) p-r, p-r, q-s, q-s
35. Match the reactants given in List-I with step (d) s-p, q, r, t
involved in List-II and mark the correct option
36. The mixture of solution are given in Column I with
List I List II its characterisation in Column II.
Ph Me Column I Column II
1 2 H+
Ph p. Carbocation p. 300 mL of 0.1M (i) pH < 7 at 298K
I. Ph C C
is formed at C-1 CH 3COONa + 100 mL of
OH OH 0.1M HCl
OH OH q. 0.1 M CH 3COOH (ii) Buffer solution
(pK a = 4.74) + 01
. M NH 4OH
H+ q. Carbocation is
II. MeO CH CH Ph (pK b = 4.74)
formed at C-2
r. 0.1M CH 3COOH (iii) pH = 7 at 298 K
X (pK a = 4.74)
Me Ph r. Ph group + 01. M CH 3COONa
mirgrates s. 0.1M CH 3COOH +01
. M HCl (iv) Basic buffer its
III. MeO C C
manimum
capacity
OH OH
X (v) Acidic buffer at its
H+ maximum
MeO capacity.

Match the given solutions in Column I with its


Y
characterstics in Column II and choose the correct
Me Ph option.
IV. Cl C C Me
H+ (a) p® (ii) ; q ® (iii) ; r ® (v); s ® (i)
(b) p® (iii) ; q ® (iv) ; r ® (ii); s ® (i)
OH OH s. X migrates (c) p® (iv) ; q ® (i) ; r ® (ii); s ® (iii)
X Y
(d) p® (ii) ; q ® (v) ; r ® (i) s ® (iv)
t. Y migrates

MATHEMATICS
(c) Between any two roots of e x cos x = 1, there exists at least
Section 1 (Maximum Marks : 24) one root of e x sin x = 1
Instructions : Same as given in Physics (d) Between any two roots of e x sin x = 1, there exists at least
one root of e x cos x = 1
37. For 3 ´ 3 matrices M and N, which of the following
statements is(are) FALSE? 39. Let ABC be a triangle with incentre at I. If P and Q
T
(a) N MN is symmetric or skew symmetric, according to M is are foot of the perpendiculars from A to BI and CI
symmetric or skew symmetric respectively, then which of the following results
(b) MN - NM is skew symmetric for all symmetric matrices are correct?
M and N B C C B
sin cos sin cos
AP 2 2 AQ 2 2
(c) MN is symmetric for all symmetric matrices M and N (a) = (b) =
(d) (adjM ) (adjN) = adj(MN) for all invertible matrices M and BI A CI A
sin sin
N 2 2
C B
sin cos
38. Which of the following is/are correct? (c)
AP
= 2 2 (d)
AP AQ
+ = 3, if ÐA = 60°
(a) Between any two roots of the e x cos x = 1, there exists at BI sin
A BI CI
least one root of tan x = 1 2
(b) Between any two roots of e x sin x = 1, there exists at least
one root of tan x = - 1

PREP CATALYSIS
110 JEE Advanced~Practice Set 2

40. A line which makes an acute angle q with the 47. Let r = (a ´ b) sin x + (b ´ c)cos y + 2(c ´ a), where
positive direction of X-axis is drawn through the a, b , c are non-zero and non-coplanar vectors. If r
point P(3 , 4) to meet the line x = 6 at R and y = 8 is orthogonal to a + b + c , then the minimum
at S. Then, which of the following is/are TRUE? 4 (x 2 + y 2)
value of is equal to
(a) PR = 3sec q p2
(b) PS = 4 cosec q
2 (3 sin q + 4 cos q) 48. If the equation of curve on reflection of ellipse
(c) PR + PS =
sin 2 q (x - 4)2 (y - 3)2
+ = 1 about the line x - y - 2 = 0 is
9 16 16 9
(d) + =1
(PR )2 (PS )2 16 x 2 + 9 y 2 + lx - 36 y + m = 0 , then the value of
[ x] + 1 é 5ö æ1 ù l + m is equal to .....
41. f (x) = forf : ê0 , ÷ ® ç , 3 ú, where [×] and {×}
{ x} + 1 ë 2ø è2 û
é 1 -3 ù
represents greatest integer function and fractional 49. If A is the matrix ê ú, then
ë -1 1 û
part of x respectively. Then, which of the following n
1 2 1 3 æ 1ö 3 é1 9 ù
is (are) TRUE? A- A + A ... + ç - ÷ A n - 1 + ... ¥ = êë b 1 úû ,
3 9 è 3ø 13
(a) f( x) is injective discontinuous function
(b) f( x) is surjective non-differentiable function then|a + b| is equal to
æ ö
(c) min ç lim f( x), lim f( x)÷ = f(1) ì 1
è x ® 1- x ® 1+ ø
ï [ x], -2£ x £ -
(d) max (x values of point of discontinuity) = f(1) 50. Let f (x) = í 2 and
1
ï2 x - 1, - < x £ 2
2
42. If the locus of the circumcentre of a variable î 2
triangle having side Y-axis, y = 2 and lx + my = 1, g (x) = f (|x|) + |f (x)|, where [×] represents the
where (l , m) lies on the parabola y 2 = 4 ax is a curve greatest integer function. The number of points,
C. Then, which of the following is (are) TRUE? where g (x) is non-differentiable, is
(a) Coordinate of the vertex of this curve C is æç - 2 a, ö÷
3
è 2ø Section 3 (Maximum Marks : 12)
(b) The length of smallest focal chord of this curve C Instructions : Same as given in Physics
1
is 51. Let a seven-digit number, made up of all distinct
8a
3 digits 8, 7, 6, 4, 2, x and y, is divisible by 3. Then,
(c) The curve C is symmetric about the line y = match the elements of List I with the elements of
2
(d) The locus of curve C is a parabola. List II
List I List II
P. Maximum value of x + y is 1. 0
Section 2 (Maximum Marks : 24) Q. Minimum value of x + y is 2. 12
Instructions : Same as given in Physics R. Maximum value of x - y is 3. 9
43. Let C(a) be the coefficient of x 2018 of the binomial S. Minimum value of xy is 4. 3
expansion (1 + x)a . Then, the value of given The correct option is
integral, (a) P ® 2; Q ® 3; R ® 4; S ® 1
1 æ 1 1 1 ö (b) P ® 3; Q ® 2; R ® 1; S ® 4
ò0 C(- y - 1) çè y + 1 + y + 2 + ... y + 2018 ÷ø dy is equal (c) P ® 2; Q ® 4; R ® 3; S ® 1
(d) P ® 3; Q ® 1; R ® 2; S ® 4
to ….
dn y
52. If y = e- x cos x and y n + k n y = 0, where y n =
44. If|z| = min {|z - 1|,|z + 1|}, then the value of|z + z|is dx n
and k n , n Î N are constants. Then, match the
45. Number of ordered pair (a , x) satisfying the elements of List I with the elements of List II.
equation sec 2 (a + 2) x + a 2 - 1 = 0 ,- p < x < p is List I List II
P. The value of k12 is 1. 4
æ x +yö
46. If f (x) + f (y) = f ç ÷, " x , y Î R (xy ¹ 1) and Q. The value of k16 is 2. - 256
è1 - xy ø
R. The value of k 8 is 3. 64
f (x)
lim = 2, then the value of f ¢()
1 is S. The value of k 4 is 4. - 16
x ®0 x

PREP CATALYSIS
JEE Advanced~Practice Set 2 111

The correct option is The correct option is


(a) P ® 3; Q ® 4; R ® 2; S ® 1 (a) P ® 1; Q ® 2; R ® 4; S ® 3
(b) P ® 3; Q ® 2; R ® 4; S ® 1 (b) P ® 5; Q ® 1; R ® 2; S ® 3
(c) P ® 2; Q ® 3; R ® 1; S ® 4 (c) P ® 2; Q ® 3; R ® 4; S ® 6
(d) P ® 2; Q ® 1; R ® 3; S ® 4 (d) P ® 2; Q ® 1; R ® 3; S ® 6
53. A1 , A 2 , A 3 , ...., A n be n arithmetic means between 54. Match the given function in List I to the number of
- 1 and 971, and G1 , G 2 , G 3 , ..., G m be m geometric integers in its range given in List II
means between 1 and 256. If product of geometric
List I List II
means is 2 28 and sum of arithmetic means is
970 ´ 121. Then, match the elements of List I with P. f( x ) = 2 cos 2 x + sin x - 8 1. 6
the elements of List II. Q. f( x ) = sin2 x + 3 cos 2 x + 5 2. 5

List I List II R. f( x ) = 4 sin x cos x - sin2 x + 3 cos 2 x 3. 4

P. The value of n is 1. 12 S. f( x ) = cos(sin x ) + sin(sin x ) 4. 3

Q. The value of m is 2. 242 5. 2


6. 1
R. The value of G1 + G 2+ G 3 + ... + G m is 3. 7
S. The common difference of the 4. 254 The correct option is
progression A1, A 3, A 5, ...., A n - 1 is (a) P ® 3; Q ® 1; R ® 2; S ® 5
(b) P ® 3; Q ® 4; R ® 1; S ® 6
5. 6
(c) P ® 3; Q ® 4; R ® 2; S ® 5
6. 8 (d) P ® 3; Q ® 2; R ® 1; S ® 5

Answers
Paper 1
1. a, b 2. a, b, c 3. a, c, d 4. a,b,c,d 5. a, b 6. a,b,c,d 7. 0.80 8. 4.00 9. 1.00 10. 2.00
11. 2.00 12. 46.00 13. 0.11 14. 0.01 15. d 16. a 17. a 18. a 19. a, b 20. a, b
21. a, c, d 22. b 23. a,b,c,d 24. b, c, d 25. 4 26. 8 27. 13 28. 0.08 29. 13.90 30. 74.93
31. 28.57 32. 0.5 33. c 34. b 35. b 36. a 37. a, c 38. a, d 39. c, d 40. b, c
41. a, b, c 42. b, c 43. 510 44. 0.25 45. 4 46. 21 47. 1 48. 9 49. 33 50. 1.25
51. b 52. a 53. b 54. d

Paper 2
1. c, d 2. a, c, d 3. a, c 4. a,b,c,d 5. a, b, c 6. b, c, d 7. 2.45 8. 28.54 9. 8.00 10. 1.00
11. 3.75 12. 0.04 13. 450.00 14. 0.67 15. d 16. b 17. a 18. c 19. a 20. b, c
21. c 22. b 23. a, c, d 24. c 25. 2 26. 0.39 27. 8.23 28. 8 29. 0.48 30. 5.70
31. 0.85 32. 0.03 33. a 34. a 35. c 36. a 37. c, d 38. a, b, c 39. a, b, d 40. a,b,c,d
41. a, b, d 42. a,b,c,d 43. 2018 44. 1 45. 3 46. 1 47. 5 48. 132 49. 12 50. 3
51. c 52. b 53. c 54. c

SCORE SHEET - Paper 1


Section No. of Marks from Marks from Marks Obtained
Correct Questions Correct Questions (A) Incorrect Questions (B) (A-B)
................ ............................... ............................... ............................... ...............................
................ ............................... ............................... ............................... ...............................
................ ............................... ............................... ............................... ...............................
Percentage Marks = Marks Obtain/Total Marks x 100

SCORE SHEET - Paper 2


Section No. of Marks from Marks from Marks Obtained
Correct Questions Correct Questions (A) Incorrect Questions (B) (A-B)
................ ............................... ............................... ............................... ...............................
................ ............................... ............................... ............................... ...............................
................ ............................... ............................... ............................... ...............................
Percentage Marks = Marks Obtain/Total Marks x 100
Note To expect your success marks in the test should be between 65%-70%.

PREP CATALYSIS
PREP CATALYSIS
JEE Advanced

PRACTICE SET 3 (With Solutions)

Duration : 3 Hours Max. Marks . 360

Paper 1
PHYSICS

Section 1 (Maximum Marks : 28)


l
This section contains SEVEN questions.
l
Each question has FOUR options (a), (b), (c) and (d). ONE OR MORE THAN ONE of these four options is (are) correct.
l
For each question, darken the bubble(s) corresponding to all the correct option(s) in the ORS.
l
For each question, marks will be awarded in one of the following categories:
Full Marks : + 4 If only the bubble(s) corresponding to all the correct option(s)
is (are) darkened.
Partial Marks : + 1 For darkening a bubble corresponding to each correct option, provided NO
incorrect option is darkened
Zero Marks : 0 If none of the bubbles is darkened.
Negative Marks : - 2 In all other cases.
l
For example, if [a], [c] and [d] are all the correct options for a question, darkening all these three will get +4 marks;
darkening only [a] and [d] will get +2 marks; and darkening [a] and [b] will get-2 marks, as a wrong option is also
darkened

1. A thin rod of length


f
is placed along the Number of a and b-particles emitted during
3 this process are
optical axis of a concave mirror of focal length (a) N a = 8 (b) Nb = 5
f , such that its image which is real and (c) N a = 3 (d) Nb = 6
elongated just touching the rod.
3. A particle of mass m moves over a circular
Now, choose the correct options. path of constant radius, such that its
(a) Length of image is half of focal length centripetal acceleration is ac = k2rt 2, where
(b) Image is one and half times magnified
k = constant and t is instantaneous time.
(c) Image formed is erect
(d) One of the end of image is at a distance of 5f / 2 Now, choose the correct options.
from mirror (a) Force on the particle is F = mkr
(b) Force on the particle is F = mkr 1 + k 2t 4
2. In the uranium radioactive series, the initial
nucleus is 238
and the final nucleus is 206 (c) Power delivered to the particle is P = mk 2r 2t 4
92 U 82 Pb.
(d) Power delivered to the particle is P = mk 2r 2t

PREP CATALYSIS
JEE Advanced~Practice Set 3 113

4. A metal sphere of radius 1mm and mass 3


(c) Volume of A reduces by a fraction of
50 mg falls vertically in glycerine. 5
3
(Take, density of glycerine = 1260 kgm -3 , (d) Volume of B increases by a fraction of
2
coefficient of viscosity of glycerine = 8 poise)
6. A stretched string is vibrating in its fifth
Now, choose the correct options. harmonic, according to equation
(a) Acceleration of sphere is constant
(b) Hydrostatic force on sphere is 5.2 ´ 10- 5 N
y ( x , t ) = 0. 01 × sin( 62 . 8x ) cos( 62 . 8t ).
(c) Speed of sphere is always less than 33 ms- 1 (Take, p = 314 . )
(d) Acceleration of sphere reduces to zero with Now, choose the correct options
increase in its speed (where, x is in metre options and t is in second)
(a) Number of nodes on string is 5
5. A cylindrical vessel contains a gas (b) Length of string is 0.25 m
æ Cp ö
çg = = 1.5÷, which is divided into two (c) Maximum displacement of mid-point of string is 0.01 m
è CV ø (d) Fundamental frequency of string is 100 Hz
parts A and B by a piston. 7. A particle of mass m is attached to one end of a
A B massless spring of spring constant k lying on a
horizontal frictionless table. Other end of spring
p, 8 p, is fixed, particle starts moving horizontally from
5V V its equilibrium position at t = 0 with an initial
velocity u 0. When speed of particle is 0.5u 0, it
collided elastically with a rigid wall. After this
collision,
Initially, piston is kept fixed such that part A (a) the speed of particle is u0 when it returns back t 0 its
has pressure p and volume 5V and the part equilibrium position
B has pressure 8 p and volume V . (b) time at which particle passes through equilibrium
When piston is released, then the gas m
position, for first time is t = p
undergoes adiabatic process. k
Now, choose the correct options. (c) maximum compression of spring occurs at time,
1 4p m
(a) Volume of chamber A reduces by a fraction of t =
3 3 k
(b) Volume of chamber B increases by a fraction (d) time for which particle comes to equilibrium position
5 5p m
of for second time is t =
3 3 k

Section 2 (Maximum Marks : 15)


l
This section contains FIVE questions
l
The answer to each question is a SINGLE DIGIT INTEGER ranging from 0 to 9, both inclusive.
l
For each question, darken the bubble corresponding to the correct integer in the ORS.
l
For each question, marks will be awarded in one of the following categories:
Full Marks : + 3 If only the bubble corresponding to all the correct answer is darkened.
Zero Marks : 0 In all other cases.

8. A particle is executing SHM. The time taken 9. One mole of an ideal gas undergoes through
æ 3ö process
for ç ÷ th of oscillation from extreme position
è 8ø æ æV ö ö
3

is T . The time taken for the particle to p = p 0 ç1 - a ç ÷ ÷


ç èV0 ø ÷ø
æ 5ö è
complete ç ÷ th of oscillation from mean
è 8ø
where p, V are pressure and volume of gas and
position isNT / 4, then find the value of N . p0, V 0 and a, are constants.

PREP CATALYSIS
114 JEE Advanced~Practice Set 3

Maximum attainable temperature of the gas is O S


3 p0V 0
× , then the value of 4a is
4 R
S1 S2
(where, R = universal gas constant) A A
10. Consider a thick biconvex lens, which is 5 cm thick The blocks A and B are simultaneously
at centre and radii of curvature of it’s surfaces is displaced through a distance of 0.5m and
5 cm. An object is located at a distance of 20 cm then left to oscillate. Angular velocity of
from its front surface on principal axis. each block is 40 rad s–1. If maximum
Let d is the distance of image in centimetres from frequency observed by observer is f, then
4 find the value of f / 90. (Take, velocity of
its rear surface, then find the value of d.
5 sound in air = 340 ms –1).
(Take, refractive index of medium of lens is 3/2) 12. Radius of curvature of a biconvex glass lens
400 is R = 40 cm.
11. A source S emitting sound of frequency Hz is
l Its one of surface is silvered, then find the
fixed on block B which is attached to spring S 2 and value of ( F - 1) , where F = focal length and
an observer O is on block A which is attached to F is in metre of this system
free end of spring S1 as shown in the figure. 3
(Take, refractive index of glass = )
2

Section 3 (Maximum Marks : 18)


l
This section contains SIX questions of matching type.
l The section contains TWO tables (each having 3 columns and 4 rows)
l
Based on each table, there are THREE questions.
l
Each question has FOUR options [a], [b], [c] and [d]. ONLY ONE of these four options is correct.
l
For each question, darken the bubble corresponding to the correct option in the ORS.
l
For each question, Marks will be awarded in one of the following categories:
Full Marks : + 3 If only the bubble corresponding to the correct option isdarkened
Zero Marks : 0 If none of the bubbles is darkened
Negative Marks : -1 In all other cases

Direction (Q. Nos. 13-15) Matching information is given in following table. In Column I, an arrangement of 3
blocks is given (m1 = 1 kg, m2 = 2 kg, m 3 = 3 kg). A force F is applied on blocks as shown in the each
arrangement. In Column II, force between first and second block ( f ) is given. In Column III, acceleration of
blocks is given.
Column I Column II Column III
I. A. f = 3N P. a = 0 ms-2
F m2 m3
m1

m = 0, F = 6N
II. F B. f =5N Q. a = 1 ms-2
m3 m1 m2

m = 0, F = 6N
III. F C. f = 12.4 N R. a = 0.17 ms-2
m3 m1 m2

m = 0.2, F = 13N
IV. D. f = 12 N S. a = 2 ms-2
F m3 m2 m1
m = 0.2, F = 12N

PREP CATALYSIS
JEE Advanced~Practice Set 3 115

13. In this case, f is least II. N0 B. 0.1% Q. 3


(a) I AQ (b) II CR (c) III BP (d) IV DS 8 4
III. N0 C. 50% R. 7
14. In this case, force on third block (from left) is 2 8
highest
IV. N0 D. 25% S. 1
(a) I AP (b) II BQ (c) III CR (d) IV DS
210
15. In this case, f is largest
(a) I AP (b) II BQ (c) III CR (d) IV DS 16. This is the sample after 2 half-lives
(a) I DQ (b) II AR (c) III BP (d) IV CS
Direction Q.16 to 18 Matching information is given in 1
table in three columns. Consider a radioactive sample 17. This is sample, when its activity is th of
8
consisting of N 0 atoms at t = 0. initial activity
Column I Column II Column III (a) I BP (b) II AR
(Number of (Number of remaining (Fraction of (c) III CQ (d) IV DS
undecayed atoms) active atoms) atoms decayed) 18. This is sample, after 10 half-lives
I. N0 A. 12.5% P. 1 (a) I AP (b) II CQ
4 2 (c) III DR (d) IV BS

CHEMISTRY
Section 1 (Maximum Marks : 28) (a) A represents formation of slag and B represents
reduction of zinc
Instructions: Same as given in Physics. (b) A represents formation of flux and B represents
19. Which of the following is optically inactive? reduction of zinc
(a) Trioxalatochromate (III) ion (c) A represents formation of slag and C represents
(b) trans-dichlorobisethylenediamineplatinum (II) Hall-Heroult process
chloride (d) B represents reduction of zinc and C represents
(c) trans-diaminedichlorobisethylenediaminechromium (I) Hall-Heroult process
(d) Both (a) and (c) 23. Which of the following reactions complete with
20. Derived name of iso-octane is racemisation?
(a) dimethyl n-pentylmethane Br2 H⊕, H2O
(b) iso-butyltrimethylmethane (a) (b)
UV
(c) trimethyl iso-butylmethane
(d) iso -propyltrimethylmethane CHO
HCN
(c)
21. Decomposition of 3 A ( g) ¾® 2B ( g) + 2 C ( g) Basic medium

follows first order kinetics. Initially, only A is O


present in the container. Pressure developed
after 20 min and infinite time are 3.5 atm and C—NH2
Br2 + KOH
4 atm respectively. Which of the following is (d)
true?
(a) t 50% = 20 min (b) t 75% = 40 min
24. Out of the following the correct statement(s)
(c) t 99% = 64 / 3 min (d) t 87.5% = 60 min
is/are
22. Which of the following statement(s) is/are (a) the structure of diamond and corrundum is same
correct regarding given equations written (b) a mixture of 5 -10% CO 2 and O 2 is known as
below? carbogen which is used for artificial respiration in
(A) FeO + SiO 2 ¾® FeSiO 3 pneumonia patient
(c) SnCl 2 is a strong oxidising agent
(B) ZnO + C ¾® Zn + CO (d) PbO is a yellow orange coloured powder
(C) 2Al 2 O 3 + 3 C ¾® 4Al + 3CO 2 commonly known as litharge

PREP CATALYSIS
116 JEE Advanced~Practice Set 3

25. Consider the following sequence of reactions :


CHO
Section 3 (Maximum Marks : 18)
NaBH4 H2/Pt
(A) Me (C ) Instructions: Same as given in Physics.
H2,Pt/C Directions (Q. No. 31-33) By appropriately matching the
(B) information given in the three columns of the following
table. Column I, II and III contain shape, hybridisation
Which of the following statement(s) is/are correct?
and bond angle respectively.
(a) The compounds (A) and (B) are OH
Me Column I Column II Column III
and , respectively
Me OH Shape Hybridisation Bond angle
(b) The compounds (A) and (B) are A. Pentagonal bipyramidal I sp 2 P. 120° and 90°
Me OH
3 3
and , respectively B. Square planar II sp d Q. sp 2
Me OH
C. Trigonal bipyramidal III dsp 2 R. 72° and 90°
(c) The compound (B) and (C)
Me OH D. Triangular planar 3
IV sp d S. 90°
CHO
are and , respectively
Me 31. Which of the combination is correct for IF7 ?
(d) Both compound (B) and (C) are (a) C IV P (b) A II R (b) D I R (d) B II Q
Me OH
32. Which of the following combination is incorrect
Section 2 (Maximum Marks : 15) for zero lone pair?
Instructions: Same as given in Physics. (a) B III S (b) D I R (c) C IV P (d) A II R
26. Consider the equation A( g) + 2B( g) s C ( g) 33. Which of the following combination is correct for
when the reaction was carried out at 120°C, CCl 4?
the equilibrium concentrations of A and B were (a) B III S (b) B II R
3 M and 4 M respectively. When the volume of (c) D I R (d) None of these
the vessel was doubled and system is allowed
Directions (Q. No. 34-36) By appropriately matching
to reach equilibrium, the concentration of B
was found to be 3 M. The original the information given is the three columns of the following
concentration of C will be ……… . table. Columns I, II and III contain chemical reaction,
reactant and product respectively.
27. Electrons in a sample of H-atom make
transition from state n = x to some lower Column I Column II Column III
excited state. The emission spectrum from the A. Favorskii I Formaldehyde P. Methylene group
sample is found to contain only the lines rearrangement
belonging to a particular series. If one of the
B. Mannich reaction II a-haloketone Q. Amino methylated
photons had energy of 0.6375 eV. Then, find
product
the value of x. (Take 0.6375 eV = 3 / 4 ´ 0.85 eV)
C. Pinacol-Pinacolone III Carbonyl R. Ketone
28. What will be the change in oxidation state rearrangement group
during transformation of KMnO 4 to manganese
dioxide? D. Clemmenson IV 1, 2-diol S. Ester
reduction
29. Oxidation state of Ti in Zieglar-Natta catalyst
is ……… . 34. Which of the following combination has
alkoxide ion as a reacting species?
30. In the acid-base titration [H 3 PO 4 (0.1M)
(a) A II R (b) B III Q (c) A II S (d) C III P
+ NaOH(0.1M)] emf of the solution is measured
by coupling this electrode with suitable 35. Which of the following combination has
reference electrodes. When alkali is added, pH of imonium ion as intermediate?
the solution is in accordance with the equation, (a) B I Q (b) A II S
(c) C II Q (d) D III R
° ´ 0.059 pH
Ecell = Ecell
36. Which of the following combination involves
For H 3 PO 4 , K 1 = 10-3 , K 2 = 10- 8 , K 3 = 10-13
the alkyl migration?
What is the cell emf at the 2nd end point of (a) B I Q (b) C IV R
° (c) C I Q (d) B II S
titration if Ecell at this stage is 1.3805 V ?

PREP CATALYSIS
MATHEMATICS
Section 1 (Maximum Marks : 28) a- x ex log e a x2
Instructions: Same as given in Physics. 42. If g ( x ) = a - 3 x e3 x log e a x 4 , then
- 5x 5 x log e a
37. An urn contain five tickets having numbers a e 1
112, 121, 211, 222, 221 written on them. If one (a) graphs of g (x) is symmetrical about origin
ticket is drawn at random and Ai ( i = 1, 2, 3) be (b) graphs of g (x) is symmetrical aboutY-axis
the event that the ith digit from left on the d 4g (x)
number of ticket drawn is 1, then (c) =0
dx 4 x = 0
1 1
(a) P (A1 Ç A2 Ç A3 ) = (b) P (A2 Ç A3 ) =
2 5 æa - x ö
(d) f (x) = g (x) × log ç ÷ is an odd function
3
3 4 èa + x ø
(c) å P (Ai Ç Aj ) = (d) P (A1 È A2 È A3 ) =
1£i < y 5 5 x2
43. Let E1 and E2 be two ellipses + y 2 = 1 and
a2
38. A function f is defined by y2
3p x2 + = 1 (where a is a parameter). Then, the
f(x) = ò0 cos z cos( x - z ) dz, 0 £ x £ 4p. Then, a2
which of the following hold(s) good? locus of the points of intersection of the ellipses
3p E1 and E2 is a set of curves comprising
(a) Maximum value of f is
2 (a) two straight lines
3p (b) one straight lines
(b) Minimum value of f is -
2 (c) one circle
(c) f (x) is continuous but not differentiable in (0, 4p) (d) one parabola
(d) There exist at least one c Î(0, 4p) such that f ¢ (c) = 0
39. If from a point P representing the complex
Section 2 (Maximum Marks : 15)
number z1, on the curve|z| = 2, two tangents Instructions: Same as given in Physics.
are drawn from P to the curve|z| = 1 meeting 44. If N is the number of ways in which 3 distinct
at points Q ( z 2 ) and R( z3 ) , then numbers can be selected from the set
z + z2 + z3 { 31 , 32 , 33 , ... , 310 } so that they form a GP, then
(a) complex number 1 will be on the curve
3 the value of N is
|z| = 1
æ4 1 1öæ4 1 1ö 45. DABC is inscribed in a unit circle. The three
(b) ç + + ÷ç + + ÷=9 bisectors of angles A, B and C are extended to
è z1 z 2 z 3 ø è z1 z 2 z3 ø
intersect the circle of A1 , B1 and C1 respectively.
æ z ö 2p
(c) arg ç 2 ÷ = Then the value of
è z3 ø 3
A B C
(d) orthocentre and circumcentre of DPQR will AA1 cos + BB1 cos + CC1 cos
2 2 2 is
coincide
sin A + sin B + sin C
40. If a pair of variable straight lines 1
x 2 + 4 y 2 + a x y = 0 (where, a is a real 29 ò (1 - x 4 )7 dx
0
46. The value of is
parameter) cuts the ellipse x 2 + 4 y 2 = 4 at two 1
4ò (1 - x 4 )6 dx
points A and B, then locus of the point of 0
intersection of tangents at A and B is/are 47. Two perpendicular chords AB and AC are
(a) x - 2 y = 0 (b) 2 x - y = 0 drawn through the vertex A of the parabola
(c) x + 2 y = 0 (d) 2 x + y = 0 y 2 = 4ax. The locus of circumcentre of the
x 2 - 9x + 20 DABC is y 2 = 2ax - la 2. Then, the value of l
41. Let f ( x ) = , where [x ] is the greatest must be
x - [x ]
integer not greater than x, then 48. If 100C0 . 100C2 + C2 . 100C4 + 100C4 . 100C6
100

(a) lim f (x) = 0 (b) lim f (x) = 1 1


x ® 5- x ® 5+ + .... + 100C98 . 100C100 = [ 200C98 - 100C49 ], then
l
(c) lim f (x) does not exist (d) None of these
x ®5 the value l is

PREP CATALYSIS
118 JEE Advanced~Practice Set 3

Section 3 (Maximum Marks : 18)


Direction (Q. Nos. 49 to 51) By appropriately matching the information given in the three columns of the following
table.
Columns I, II and III contains functions, their graph and nature respectively
Column I Column II Column III
Y
A. 1 I. P. One-one
f (x) = -3
x

0 X

Y
B. f (x) = [x] + x - [x ] II. Q. Onto

X
0
Y
C. f (x) = min{| x |, | x - 1 |, | x + 1 | } III. R. Into

X
0

D. 2x IV. Y
S. Many-one
f (x) = [x ]
2

49. When x Î [0, 1] and f ( x ) Î [0, 1), then which of


the following is correctly matched? . Column I Column II Column III
(a) B I P (b) A III P (c) C IV P (d) D II P I. 1 i. 1 P.
by L1, L2 by L1, L2 L1, L2 are
50. When x Î [- 1, 1], and f ( x ) Î [0, 2], then which of 6 6 perpendicular
the following is correctly matched? II. 5
by L1, L2 ii. 1
by L3 , L4 Q. L5 L6 are parallel
(a) A III S (b) B I P (c) C IV S (d) D II P 6 6
5 5
51. When x Î [- 2, 2], and f ( x ) Î [1, 2), then which of III. by L3 , L4 iii. by L3 , L4 R. L3 L4 are neither
6 6 parallel nor
the following is correctly matched?
perpendicular
(a) B I P (b) B II S (c) D II R (d) D II Q
IV. 1 iv. 5 S. L3 L4 are parallel
by L5 , L6 by L5 , L6
Direction (Q. Nos. 52 to 54) Answer by appropriately 6 6
matching the information given in the three columns of
the following table. 52. Which of the following options is the only
Let the lines given by 6x 2 + 5xy - 6y 2 - x + 5y - 1 = 0 correct combination?
(a) (I) (i) (P) (b) (II) (ii) (Q)
are L1 , L 2 , by 30x 2 + 36xy + 6y 2 - 35x - 11y + 5 = 0
(c) (III) (iii) (R) (d) (IV) (iv) (S)
are L 3 , L 4 and by 6x 2 + 12xy + 6y 2 - 7 x - 7 y + 1 = 0 are
L5 , L 6 . 53. Which of the following option is the only
Column 1 Contains information about the intercepts correct combination?
(a) (I) (ii) (R) (b) (II) (iii) (S)
made by the lines L1 , L 2 ; L 3, L 4 and L5 , L 6 on X-axis.
(c) (III) (iv) (P) (d) (IV) (iv) (S)
Column 2 Contains information about the intercepts
54. Which of the following options is the only
made by the lines L1 , L 2 ; L 3 , L 4 and L5 , L 6 on Y-axis.
incorrect combination?
Column 3 Contains information about the lines (a) (II) (i) (Q) (b) (III) (ii) (R)
L1 , L 2 ; L 3 , L 4 ; L5 , L 6 are parallel, perpendicular, neither (c) (III) (i) (P) (d) (II) (iv) (S)
parallel nor perpendicular.

PREP CATALYSIS
Paper 2
PHYSICS

Section 1 (Maximum Marks : 21)


l This section contains SEVEN questions.
l Each question has FOUR options (a), (b), (c) and (d). ONLY ONE of these four options is correct.
l For each question, darken the bubble corresponding to the correct option in the ORS.
l For each question, marks will be awarded in one of the following categories.
Full Marks : + 3 If only the bubble corresponding to the correct option is darkened.
Zero Marks : 0 If none of the bubbles is darkened.
Negative Marks : - 1 In all other cases

1. One mole of argon is expanded polytropically


( n = 1 . 50)
In the process, temperature of gas reduces by
26 K.
Heat absorbed by gas (in kilojoule) is
(a) 0.11 (b) 0.22 (c) 0.33 (d) 0.50
2. Fluid flowing through a tube has a velocity
æ r2 ö
v = v0 ç1 - 2 ÷
è R ø To go up, she increases her force by 15% by
where, r = distance from centre of tube, pulling on the rope. (Take, mass of person and
bucket as 60 kg and g = 10 ms- 2.)
R = radius of tube = 10 cm,
Acceleration of the window washer is
v0 = 0.5 ms- 1
(a) 2.5 ms- 2 (b) 9.8 ms- 2 (c) 10 ms- 2 . ms- 2
(d) 150
l = length of tube = 1 m
h = coefficient of viscosity of fluid = 12 .5. 6. A 1 m long heavy steel cable passes over a
massless frictionless pulley.
The pressure difference (Nm -2 ), required across
Let two third length of cable is on one side of
ends of tube to maintain a steady flow is
pulley and it is released to fall on the ground.
. Nm-2
(a) 075 (b) 0.25 Nm-2
Velocity of free end of cable at the moment, then
. Nm-2
(c) 10 (d) 2.0 Nm-2
the whole cable has fallen from the pulley is
3. Percentage uncertainity in the volume of a ball (a) 2.1 ms-1 (b) 4.5 ms-1 (c) 3.2 ms-1 (d) 12
. ms-1
with radius r = 0.84 ± 0.04 m is
7. Two identical C1
(a) nearly 39% (b) nearly 50%
capacitors are
(c) nearly 29% (d) nearly 70%
connected in parallel
4. As snow started to fell vertically downwards, a and a charge Q is given
B
car driver hurried on highway at 25 m s- 1
A
to this combination
speed. At instant t = 0, plates C2
Snow appears to fall at an angle of 37º from the of capacitor C1 are
car window. moved apart with speed
Speed of snowflakes relative to the ground is v and of capacitor C2 are moved towards each
(a) 18.75 ms- 1 (b) 28.32 ms- 1 other with speed v. Find the intensity of
(c) 0.98 ms- 1 (d) 5.42 ms- 1 current which flows from A to B.
Take,|Q| = d0 = initial distance between plates
5. A window washer is holding onto the rope of of capacitors and v = 1 m s- 1.
pulley-rope arrangement as shown in figure.
(a) 1 A (b) 2.5 A (c) 0.5 A (d) 0 A

PREP CATALYSIS
120 JEE Advanced~Practice Set 3

Section 2 (Maximum Marks : 28)


l This section contains SEVEN questions.
l Each question has FOUR options (a), (b) (c) and (d). ONE OR MORE THAN ONE of these four option(s) is (are)
correct.
l For each question, darken the bubble(s) corresponding to all the correct option(s) in the ORS.
l For each questions, marks will be awarded in one of the following categories :
Full Marks : + 4 If only the bubble(s) corresponding to all the correct option(s) is (are)
darkened.
Partial Marks : + 1 For darkening a bubble corresponding to each correct option, provided NO
incorrect option is darkened.
Zero Marks : 0 If none of the bubbles is darkened.
Negative Marks : - 2 In all other cases.
l For example, if (a), (c) and (d) are all the correct options for a question, darkening all these three will get in + 4 marks ;
darkening only (a) and (d) will get + 2 marks and darkening (a) and (b) will result in - 2 marks, as a wrong option is also
darkened.

8. A block of mass 2 kg is free to move along (b) If disc is rotating clockwise with angular speed w,
X-axis. Force on the particle varies as then its angular momentum about origin is
F æ MR 2 w ö
(N) 4 N L=ç + Mvd ÷ (k$ )
è 2 ø
(c) If disc is rotating anti-clockwise with angular speed
4.5s w, then its angular momentum about origin is
t(s)
3s æ MR 2 w ö
L=ç - Mvd ÷ k$
–2 N è 2 ø
(d) If disc is rotating clockwise with angular speed w,
then its angular momentum is
Now, choose the correct options. æ MR 2 w ö
(a) Area under above graph = Kinetic energy of particle L=ç + Mvd ÷ (- k$ )
è 2 ø
(b) Area under above graph = Momentum of particle
(c) Kinetic energy of particle after 4.5 s = 4.50 J 10. A time dependent force F = 2t$i + 2$j started to
(d) Kinetic energy of particle after 4.5 s = 5.06 J
act on a body of mass 1 kg at t = 0 s.
9. Consider a disc (radius R, mass M) placed in Now, choose the correct options.
xoy-plane with its axis of rotation parallel to (a) Torque on the body about origin is anti-clockwise
Z-axis. (b) Magnitude of torque about origin at t = 1s is 4 N.m
Y (c) Torque on body at t = 3 s is 36 N.m
1
(d) Angular momentum at t = 1s is (- k$ ) kg - m2 s- 1
3
11. A capillary tube with a narrow bore is dipped
in water. Let radius of tube is r and water rises
d in capillary upto height h.
X If surface tension of water is T , then choose the
O
C correct options.
2T 2
Disc is translating with velocity v$i and is (a) Potential energy of water column in capillary is
rg
rotating on its axis.
(a) If disc is rotating anti-clockwise with angular speed 4 pT 2
(b) Work done by force of surface tension is
w, then its angular momentum about origin is rg
æ MR 2 w ö (c) Half of work done by surface tension is dissipated
L=ç + MvC ÷ (- k$ )
è 2 ø (d) Work done by surface tension is stored in form of
potential energy of water column

PREP CATALYSIS
JEE Advanced~Practice Set 3 121

12. In given circuit, L = 400 mH, R1 = 2 W, 14. An infinitely long thin non-conducting wire is
R2 = 2 W, E = 12 V. placed parallel to Z-axis. Charge density of wire
is l cm - 1. It passes through a cube of side length
+
E 1 m, centred at origin with its faces parallel to

R1 L axes.
S Wire subtends an angle of 90º at origin as shown
in the figure.
R2 Y
Switch S is closed at t = 0 s. Now, choose the
correct option.
(a) Potential drop across L = 12e - 5t volts
(b) Current in the inductor is i = 6(1 - e - 5t )
(c) Steady state current through R 2 = 4 A
(d) When, switch is opened again, current through R1
90°
decreases exponentially with time 1m X
O
13. A sample of ideal gas is carried through a
cycle as shown in the figure. Cube
p C

λ
Size

B A If permittivity of medium in cube is e. Now,


V
choose the correct options.
(a) Flux through cube is l / e0
Process CA is isothermal.
(b) z-component of electric field at all points on surface
Now, choose the correct options. of cube is zero
(a) In process AB, DU, DQ and DW all are negative (c) Flux through cube is l / e
(b) In process BC, DW = 0 (d) Electric field is perpendicular at all points on surface
(c) In process CA, DQ is positive of cube
(d) In process BC, DU is positive

Section 3 (Maximum Marks : 12)


l
This section contains TWO paragraphs.
l
Based on each paragraph, there are TWO questions.
l
Each question has FOUR options (a), (b) (c) and (d). ONLY ONE of these four options is correct.
l
For each question, darken the bubble corresponding to all the correct option in the ORS.
l
For each questions, marks will be awarded in one of the following categories.
Full Marks : + 3 If only the bubble corresponding to all the correct answer is darkened.
Zero Marks : 0 In all other cases.

Paragraph X é 1 -1 1 ù
(c) [c1/ 2 G h] (d) aêc 2 G 2 h 2 ú
Let speed of light ( c), gravitational constant (G) and êë úû
Planck’s constant ( h) are chosen as fundamental
16. In given system, dimensional formula for
quantities.
momentum is
15. In given system, dimensional formula for é 1 1 1ù
mass will be (a) êc 2 G 2 h 2 ú (b) [c G 2 h]
êë úû
é 1 1 1ù
(a) êc 2 G 2 h 2 ú (b) [c G h 2 ] é 1 ù é 3 -1 1ù
êë úû (c) êc 2 G h 2 ú (d) êc 2 G 2 h 2 ú
êë úû êë úû

PREP CATALYSIS
122 JEE Advanced~Practice Set 3

Paragraph A 17. Fractional error in X is


Let a physical quantity X is related to four (a) 0.28 (b) 0.25 (c) 0.24 (d) 0.26
measurable quantities X = a 2b 3 c 5/2 d - 5 18. If value of X is found 2.763, then it is rounded
Percentage errors in measurements of a, b, c and d are off to
1%, 2%, 3% and 4%, respectively. (a) 2.7 (b) 2.76 (c) 2.77 (d) 2.80

CHEMISTRY
Section 1 (Maximum Marks : 21) 24. CH 3 ¾ CH 2 ¾ CH == CH 2 + CBrCl 3
( C 6 H 5 CO) 2 O 2
Instructions: Same as given in Physics. ¾¾¾¾® M, M will be
19. Among the hydroxides of period 3 elements, (a) CH3 ¾CH2 —CH ¾ CH2 ¾CBrCl 2
the element whose oxide is amphoteric has ½
Cl
value of quantum numbers are
(a) n = 3, l = 1, s = 1/ 2 (b) n = 3, l = 1, s = 3 / 2 (b) CH3 ¾CH2 ¾CH ¾ CH2 ¾Br
(c) n = 3, l = 1, s = 1 (d) n = 3, l = 0, s = 1/ 2 ½
CCl 3
20. An aromatic organic compound A having (c) CH3 ¾CH2 ¾CH ¾CH2 ¾CCl 3
molecular mass 112.5 containing C, H and ½
chlorine only, have 4 degree of unsaturation Br
what will be the product when A undergoes (d) CH3 ¾CH2 ¾CH ¾CH2 ¾Cl
reaction with sodium hydroxide? ½
OH Cl CBrCl 2
OH
(a) (b) 25. Which of the following will produce a stable
non-planar carbocation?
Cl (a) Addition of antimonypentafluoride to
cycloheptatrienyl fluoride
(b) Addition of bromonium ion to propene
(c) (d) None of these
(c) Addition of acid to propene
(d) Addition of argentium ion to triphenylmethyl
OH chloride
21. x mol KIO 3 is treated with excess of KI, Section 2 (Maximum Marks : 28)
liberates I 2 which was dissolved in freshly
Instructions: Same as given in Physics.
prepared starch solution and it is neutralised
by 60 mL 0.1 N Na 2S2O3 until white ppt. was 26. The number of electrons present in e g set of
obtained. What is x? orbital of triaquatrifluorocobalt (III) is
(a) 10-3 mol (b) 10-5 mol (c) 5 mol (d) 6 mol (a) 2 (b) 3 (c) 4 (d) 0
22. Some oxides of p-block elements show HOCl/ H +
colour while some do not, among the following 27. Ph ¾ C ºº CH ¾¾® A, then A will be
which of the following pair of oxides are (a) Ph ¾C ¾ CHCl 2 (b) Ph ¾C ¾ CH2 Cl
coloured? ½½ ½½
O O
(a) NO 2 and N2O 3 (b) NO 2 and N2O 5
(c) N2O 4 and NO2 (d) NO and N2O (c) Ph ¾CH ¾ C ¾H (d) Ph ¾C == CH
½ ½½ ½ ½
23. 1 mole of an ideal gas A (CV = 3R ) and 2 moles Cl O OH Cl
æ 3 ö 28. A radioactive nuclide is produced at a
of an ideal gas B çCV = R ÷ taken in a
è 2 ø constant rate of a per second and its decay
constant is l . If N0 be the number of nuclei at
container and expanded reversibly and
time t = 0, then maximum number of nuclei
adiabatically from 1 L to 4 L starting from initial possible are
temperature of 320 K. DE for the process is a l
(a) N 0 (b) a / l (c) N 0 + (d) + N0 S
(a) - 240R (b) 240R (c) 480R (d) - 960R l s

PREP CATALYSIS
JEE Advanced~Practice Set 3 123

29. Which of the following 3° alcohol cannot be 33. Which of the following statements is/are
prepared from reaction of an ester with excess correct regarding CrO 5 ?
of Grignard reagent? (a) It has two types of oxide linkage, dioxide and
OH OH oxygen peroxide linkage
(a) (b) (b) Oxidation state of Cr is + 10
(c) Formation of CrO5 takes place by oxidation of
Cr2O2-
OH OH 7

(c) (d) (d) All the statements written above are correct
34. A deep red coloured vapour is obtained when
Cr2O72- is heated with Cl - and H 2SO 4, the
30. In a solid having rock salt structure, if all the oxidation state of chromium in deep red
atoms touching one body diagonal planes are coloured vapour is
removed (except at body centre), then the (a) + 4 (b) + 3
formula of unit cell is (c) + 6 (d) + 7
(a) A 3.5 B 2.5 (b) A 7 B 3 (c) A 5 B 3 (d) A 3 B 5
Paragraph A
31. Choose correct options for below sequence of In the given figure, A and B are isomeric liquids and
reaction. form an ideal solution at T K. The vapour pressures
OH of A and B are p°A and p°B . The vapours of A and B are
(i) CO2, NaOH (CH3CO)2O AlCl3 collected in volume V, first container is maintained
A B C 3T
at 2T K and second container is at K.
(ii) H+ H2O 2 both A and B
At the temperature, greater than T K,
OH
OH exist only in gaseous form p t = p°A x A + p°B x B , where
x A and x B are molecular formula of A and B in liquid
(a) (b) B =
A= state.
C—OAc TK
COOH
(major) O
OH A+B

OAc —COOH In container I Collected gas behave ideally at 2T K and


C= isomerisation takes place
(c) (d)
—C—OH
B=
C—CH3
A ¾® k
B (first order)
O
3T
O In container II At , A and B are ideal in nature and
2
32. The separation of SO 2 and CO 2 can be done by non–mixing in nature. A small pin hole is made into
using the reagent(s ) kp
(a) lime water
container, rate of effusion is given by r = .
M
(b) BaCl 2 solution
where, p = pressure difference between system and
(c) H2O2 + BaCl 2 solution
surrounding, k = constant M = molecular weight.
(d) acidified dichromate paper
35. The vapour is collected and passed into
Section 3 (Maximum Marks : 12) container of volume 8.21 L maintained at
100 K and after 5 min, number of moles of B
Instructions: Same as given in Physics. 8
equals to . Then, calculate the pressure
Paragraph X 3
When the qualitative analysis is done for chromium, developed in the container after two half-lives.
the presence of chromium is confirmed by the (a) 2 atm (b) 4 atm (c) 1 atm (d) 0.5 atm
formation of CrO5 (peroxo species) which has blue 36. If vapour is collected in the container of
colour. Formation of CrO5 takes place when a volume 8.21 L maintained at 75 K, then the
solution of H2 O2 is added to an acidic solution ratio of initial rates of effusion of gas A and B
is given as
containing Cr2 O72- . (a) 2 : 1 (b) 1 : 2 (c) 4 : 1 (d) 1 : 4

PREP CATALYSIS
124 JEE Advanced~Practice Set 3

MATHEMATICS
Section 1 (Maximum Marks : 21) 43. Let, m k > 0 and z k = m k (cos a k + i sin a k ) for
k = 1, 2, 3 be such that
Instructions: Same as given in Physics.
37. The sum of the series 1 1 1
+ + =0
99+1 z1 z 2 z3
å ( - 1)r ( 99Cr + 99Cr - 1 ) Let the affix of the point Ak be
r=0
é1 3r 7r ù cos 2a k + i sin 2a k
ê r + 2r + 3 r + ¼ + 10 terms ú wk = for k = 1, 2, 3, then the
êë 2 2 2 úû zk
origin O is
1 é 21000 + 1ù 1 é 21000 - 1ù
(a) ê 100 ú (b) ê 100 ú (a) centroid of D A1A2 A3 (b) circumcentre of D A1A2 A3
21000
ë2 + 1û 21000
ë2 + 1û (c) incentre of D A1A2 A3 (d) orthocentre of D A1A2 A3
1 é 21000 - 1ù
(c) ê 100 ú (d) None of these Section 2 (Maximum Marks : 28)
21000 ë2 - 1û
Instructions: Same as given in Physics.
38. The pair of lines whose direction cosines are
44. If the tangent at any point P ( 4m 2 , 8m3 ) of
given by the equations 3l + m + 5n = 0 and
x 3 - y 2 = 0 is also a normal to the curve
6mn - 2nl + 5 ml = 0 are
x3 - y 2 = 0, then the value of m is
(a) parallel (b) perpendicular
- 1 æ 1ö
2 2 3 3
(c) inclined at cos ç ÷ (d) None of these (a) (b) - (c) (d) -
è 6ø 3 3 2 2
1 dx
39. A 2n digit number starts with 2 and all its 45. If I n = ò0 (1 + x 2 )n , where n Î N , then which of
digits are prime, then the probability that the
sum of all 2 consecutive digits of numbers is the following statements hold good?
p 1
prime, is (a) 2nIn + 1 = 2 -n + (2n - 1)In (b) I 2 = +
8 4
(a) 4 ´ 2 3n (b) 2 3n
p 1 3p 1
(c) 4 ´ 2 - 3n
(d) None of these (c) I 2 = - (d) I 3 = +
8 4 32 4
40. If the function f ( x ) satisfies the relation 46. Let f : ( -1, 1) ® R be such that
æ1 - x ö
2 f ( x - 1) - f ç ÷ = x is true, " x Î R, then 2 æ pö æp pö
è x ø f(cos 4q ) = for q Î ç 0, ÷ È ç , ÷.
2 - sec2 q è 4ø è 4 2ø
f ( x ) is equal to æ1ö
2 (1 + x)2 + 1 (1 - x) Then, the values(s) of f ç ÷ is
(a) (b) 2 (x - 1) - è 3ø
3 (1 + x) x
3 3 2 2
1 1 ì 1 ü (a) 1 - (b) 1 + (c) 1 - (d) 1 +
(c) 2 (x + 1) + (d) í(x + 2) + ý 2 2 3 3
(x + 1) 4 î (x + 2) þ
47. A vector a has components 2 p and 1 with
1x 1 dt
41. If f ( x ) = ò dt ( x > 0) and ò0 = 2 , then respect to a rectangular cartesian system. This
0 f ( t) f ( t) system is rotated through a certain angle
f( 200) will be about the origin in the counter clockwise sense.
(a) 20 (b) 10 (c) 30 (d) 40 If, with respect to the new system, a has
components p + 1 and 1, then
42. Let a, b and c be three non-coplanar unit 1 1
(a) p = 0 (b) p = 1 (c) p = (d) p = -
vectors equally inclined to one another at an 3 3
acute angle q. Then|[a b c]| in terms of q is
equal to 48. If a curve y = a x + bx passes through the
(a) (1 + cos q) cos 2 q point (1, 2) and the area bounded by the curve,
(b) (1 + cos q) 1 - 2 cos 2 q line x = 4 and X-axis is 8 square units, then
(c) (1 - cos q) cos 2 q (a) ab = - 3 (b) | a - b| = 4
(d) (1 - cos q) 1 + 2 cos q (c) a = 3 (d) b = - 1

PREP CATALYSIS
JEE Advanced~Practice Set 3 125

49. The plane lx + my = 0 is rotated about its line 52. The value of ‘a’ for which f ( x ) has local minima
of intersection with the plane z = 0, through an at some negative real x
angle a, then equation of plane in its new (a) (-¥,1) È (9, ¥) (b) (-¥,1] È [9, ¥)
position may be (c) (0, 1) (d) (1, 9)
(a) lx + my + z l 2 + m 2 tana = 0 Paragraph A
(b) lx + my - z l 2 + m 2 tana = 0 Let A be a m ´ n matrix. If there exists a matrix L of
(c) lx + my + z l + m tana = 0 type n ´ m such that LA = I n , then L is called left
(d) Data is not sufficient inverse of A. Similarly, if there exists a matrix R of
50. A function f ( x ) satisfies the relation type n ´ m such that AR = I m , then R is called right
f ( x + y ) = f ( x ) + f ( y ) + xy( x + y ) " x , y Î R. If inverse of A.
f ¢ ( 0) = - 1, then 53. Which of the following matrices is not left
(a) f (x) is a polynomial function é 1 - 1ù
(b) f (x) is an exponential function inverse of matrix êê 1 1 úú ?
(c) f (x) is twice differentiable for all x ÎR êë 2 3 úû
(d) f ¢ (3) = 8
é 1 1
0ù - 7 3ù
ê ú é 2
Section 3 (Maximum Marks : 12) (a) ê 2 2
ú (b) ê 1 1 ú
ê-
1 1
0ú êë - 2 2

û
Instructions: Same as given in Physics. ë 2 2 û
Paragraph X é- 0ù
1 1
é 0 3 - 1ù
If f ( x) = 2x - 3( a - 3) x 2 + 6ax + a + 2 is cubic equation
3
ê
(c) ê 2 2 ú
(d) ê 1 1 ú
ú
in x. Then give the answer of the following ê-
1 1
0ú êë - 2 2

û
ë 2 2 û
51. The value of ’a’ for which f ( x ) has exactly one
54. The number of right inverse for the matrix
point of local maxima and one point of local é 1 - 1 2ù
minima ê 2 - 1 1 ú is
(a) (-¥,1) È (9, ¥) (b) (-¥,1] È [9, ¥) ë û
(c) [1, 9] (d) (1, 9) (a) 0 (b) 1 (c) 2 (d) infinite

PREP CATALYSIS
126 JEE Advanced~Practice Set 3

Answers
Paper 1
1. a,b,d 2. a,d 3. b,d 4. b,c,d 5. a,b 6. b,c 7. a,d 8. 7 9. 1 10. 5
11. 5 12. 9 13. a 14. b 15. c 16. a 17. b 18. d 19. b 20. b
21. a,b,d 22. a,c,d 23. a 24. a,b,d) 25. a,c 26. 4 27. 8 28. 3 29. 4 30. 2
31. b 32. a 33. d 34. c 35. a 36. b 37. b,c,d 38. a,b,d 39. a,b,c,d 40. a,c
41. a,b,c 42. a,c 43. a,c 44. 20 45. 2 46. 7 47. 8 48. 2 49. a 50. c
51. d 52. b 53. c 54. d

Paper 2
1. a 2. b 3. c 4. a 5. d 6. a 7. c 8. b,d 9. c,d 10. c,d
11. b,c 12. a,b 13. a,b,c,d 14. b,c,d 15. d 16. d 17. c 18. d 19. a 20. d
21. a 22. a 23. d 24. c 25. d 26. a 27. b 28. b 29. b,c 30. a
31. c,d 32. c,d 33. a 34. c 35. b 36. a 37. c 38. c 39. c 40. a
41. a 42. d 43. a 44. a,b 45. a,b,d 46. a,b 47. b,d 48. a,b,c,d 49. a,b 50. a,c,d
51. a 52. c 53. c 54. d

SCORE SHEET - Paper 1


Section No. of Marks from Marks from Marks Obtained
Correct Questions Correct Questions (A) Incorrect Questions (B) (A-B)
................ ............................... ............................... ............................... ...............................
................ ............................... ............................... ............................... ...............................
................ ............................... ............................... ............................... ...............................
Percentage Marks = Marks Obtain/Total Marks x 100

SCORE SHEET - Paper 2


Section No. of Marks from Marks from Marks Obtained
Correct Questions Correct Questions (A) Incorrect Questions (B) (A-B)
................ ............................... ............................... ............................... ...............................
................ ............................... ............................... ............................... ...............................
................ ............................... ............................... ............................... ...............................
Percentage Marks = Marks Obtain/Total Marks x 100
Note To expect your success marks in the test should be between 65%-70%.

PREP CATALYSIS
PREP CATALYSIS
JEE Advanced

PRACTICE SET 4 (With Solutions)

Duration : 3 Hours Max. Marks . 360

Paper 1
PHYSICS
Section 1 (Maximum Marks : 24)
l
This section contains SIX (06) questions.
l
Each question has FOUR options for correct answer(s). ONE OR MORE THAN ONE of these four option(s) is (are)
correct options(s).
l
For each question, choose the correct options(s) to answer the question.
l
Answer to each question will be evaluated according to the following marking scheme:
Full Marks : + 4 If only (all) the correct option(s) is (are) chosen.
Partial Marks : + 3 If all the four options are correct but ONLY three options are chosen.
Partial Marks : + 2 If three or more options are correct but ONLY two options are chosen, both of which are
correct options.
Partial Marks : + 1 If two or more options are correct but ONLY one option is chosen and it is a correct option.
Zero Marks : 0 If none of the options is chosen (i.e. the question is unanswered).
Negative Marks : - 2 In all other cases.
l
For example: If first, third and fourth are the ONLY three correct options for a question with second option being an
incorrect option; selecting only all the three correct options will result in + 4 marks. Selecting only two of the three
correct options (e.g. the first and fourth options), without selecting any incorrect option (second option in this case), will
result in +2 marks. Selecting only one of the three correct options (either first or third or fourth option), without selecting
any incorrect option (second option in this case), will result in +1 marks. Selecting any incorrect option(s) (second
option in this case), with or without selection of any correct option(s) will result in -2 marks.

1. For lead isotopes Pb208, Pb206 and Pb206, the ‘ka ’ æx ö


2. Let y = a3 sin-1 ç - 1÷, where y is in metres.
wavelengths are l1 , l 2 and l3 , respectively. èb ø
Now, choose the correct options. Now, choose the correct options.
(a) Dimensions of a are [M0L1/ 3 T 0 ]
(a) l1 > l 2 > l 3
(b) SI unit of sin-1 æç - 1ö÷ is radians
x
(b) l1 < l 2 < l 3 èb ø
(c) l1 = l 2 = l 3 (c) Dimensions of x and b are same
(d) Dimensions of sin-1 æç - 1ö÷ are [M0L0 T 0 ]
x
(d) l1 = l 2l 3
èb ø

PREP CATALYSIS
128 JEE Advanced~Practice Set 4

3. A cyclic process is shown on p-T diagram in 5. A small mirror of mass m is suspended


figure. Which of the curve(s) show(s) the same by a light thread of length l. The angle
process on V -T diagram? through which the thread will be
p deflected when a short pulse of laser of
a b
power E falls normally on the mirror is

2E mc gl
(a) (b)
d c mc 2gl 2E
T
mc 2gl 2E
V V (c) (d)
a b b a 2E mc gl

(a) (b) 6. A hemispherical bowl of radius, R = 01


. m is
rotating about its own axis.
d c c d
T T ω
V V
c
c
(c) (d) b h
d
b
d
a a
T T A particle of mass 10-2 kg on the frictionless
inner surface of the bowl is also rotating with
4. µ=0.5
5 Kg 30 N the same angular speed w. Particle is at height
µ=0.5 h from the bottom of the bowl.
10 Kg
Now, choose the correct options.
Consider a system of blocks (masses and (a) h is always zero
coefficients of friction are given in figure g
(b) h = R - 2
shown). If upper block is pulled with a force of w
30 N horizontally, then the correct options are (c) Particle drops to bottom of bowl if w is below 9.8
(a) only 5 kg block moves rads-1
(b) both 5 and 10 kg blocks moves g
(c) acceleration of 5 kg block is 1 ms-2 (d) If h is non zero, then R > 2
w
(d) acceleration of 10 kg block is 1 ms-2

Section 2 (Maximum Marks : 24)


l
This section contains EIGHT (08) questions. The answer to each question is a NUMERICAL VALUE.
l
Four each question, enter the correct numerical value (in decimal notation, truncated/rounded-off to the second
decimal place; e.g. 6.25, 7.00.- 0.33, -.30, 30.27, -127.30) using the mouse and the on-screen virtual numeric
keypad in the place designated to enter the answer.
l
Answer to each question will be evaluated according to the following marking scheme:
Full Marks : + 3 If ONLY the correct numerical value is entered as answer.
Zero Marks : 0 In all other cases.

7. In the given figure, the blocks A, B and C have horiontal surface to left at a constant speed.
masses 3 kg, 4 kg and 8 kg, respectively. The
coefficient of sliding friction between any two
surfaces is 0.25. A is held at rest by a massless
rigid rod fixed to the wall, while B and C are A
connected by a light flexible cord passing
around a fixed frictionless pulley. Find value of B
F F C
, where F is force necessary to drag C along
10

PREP CATALYSIS
JEE Advanced~Practice Set 4 129

8. Three objects A, B and C are kept in a The latent heat of evaporation of water is
straight line on a frictionless horizontal 540 calg -1 and latent heat of melting of ice is 80
surface. These have masses m, 2m and m, cal-g -1. If the point P is at a distance of lx from
respectively. The object A moves towards B the ice end A, find the value of l.
with a speed 9ms -1 and makes an elastic (Neglect any heat loss to the surrounding.)
collision with it.
11. A pin is placed 10 cm in front of a convex lens of
Thereafter, B makes completely inelastic focal length 20 cm and made of a material of
collision with C. All motions occur on the refractive index 1.5.
same straight line. Find the final speed The convex surface of the lens farther away
(in ms -1) of the object C. from the pin is silvered and has a radius of
curvature of 22 cm. If image is at a distance of
v cm from the pole (centre of lens), then find the
m 2m m value of v.
A B C 12. A spherical ball of radius 3.0 ´ 10-4 m and density
9. Two blocks of mass 2 kg and M are at rest 104 kg/m3 falls freely under gravity through a
on an inclined plane and are separated by a distance h before entering a tank of water. If
distance of 6.0 m as shown. The coefficient of after entering the water, the velocity of the ball
friction between each block and the inclined æ h ö
does not change, find ç 3 ÷. Viscosity of water is
plane is 0.25. è 10 ø
The 2 kg block is given, a velocity of 10.0 m/s 9.8 ´ 10-6 Ns/m 2.
up the inclined plane. It collides with M, 13. Consider a thin concave-convex lens of glass
comes back and has a velocity of 1.0 m/s (m = 1.5), convex surface of lens has a radius of
when it reaches its initial position. The other curvature of 20 cm and concave surface has a
block M after the collision moves 0.5 m up radius of curvature of 60 cm. The convex side of
and comes to rest. What is the velocity of lens is silvered.
block of 2 kg just after collision?
Water
[Take, sin q » tan q = 0.05 and g = 10 m/ s2]
Glass lens
M
m
0 A pin is placed on optical axis of lens such that
6.
it’s image is formed at same place.
2 kg
If concave part of lens is now filled with water
(m = 4 / 3), then find the distance by which pin
θ must be moved, so that it’s image again coincides
with pin.
10. A metal rod AB of length 10x has its one end
14. A steel wire has a length of 12.0 m and a mass of
A in ice at 0°C and the other end B in water
æ tension ö
at 100°C. If a point P on the rod is 2.10 kg. What should be the value of ç ÷ in
è 104 ø
maintained at 400°C, then it’s found that
equal amounts of water and ice evaporate and the wire, so that speed of a transverse wave on
melt per unit time. the wire equals the speed of sound in dry air at
20°C = 343 ms-1?

PREP CATALYSIS
Section 3 (Maximum Marks : 12)
l This section contains TWO (02) paragraphs. Based on each paragraph, there are TWO(02) questions.
l Each question has FOUR options. ONLY ONE of these four options corresponds to the correct answer.
l Four each question, choose the option corresponding to the correct answer.
l Answer to each question will be evaluated according to the following marking scheme:
Full Marks : + 3 If ONLY the correct option is chosen.
Zero Marks : 0 If none of the options is chosen (i.e. the question is unanswered).
Negative Marks : - 1 In all other cases.

Paragraph X Paragraph A
A man wants to cross a river of width d. He wants Three long parallel straight current carrying wires are
to reach at the opposite point B. If velocity of man placed as shown in the figure. Two of them are fixed
is v m = 10 m/s and velocity of river is v r = 12 m/s, and the middle wire is free and it could be displaced.
then solve the questions given below Solve the questions given below on the basis of this
situation.
B
A B C
vm I I′ I
d = 10 m
θ
L
A vr

d d
15. The value of angle made by the velocity of Fixed Fixed
man by line AB
Three long straight parallel wires
(a) q = sin-1 æç ö÷
10
è 12 ø
17. Choose the correct options.
(b) q = sin-1 æç ö÷
12
è 10 ø (a) The wire B is in stable equilibrium
(b) The wire B is in unstable equilibrium
(c) 60 °
(c) The wire B is not in equilibrium
(d) None of the above
(d) None of the above
16. If vm become 24 m/s, then the time taken to
18. Now, we have reversed the direction of current in
reach the exactly opposite point B B. If we displaced the wire B by a small amount
5 n towards C, then time taken by it to again reach
(a) 3 m/s
6 at the same position
6
(d) 3 m/s
5 m 0LI ¢ mp
(a) 2 p (b) 2p
(c) 3 3 m/s 2mp m 0LII ¢
(d) None of the above m pI m 0LI
(c) 2 p (d) 2 p
2m 0LI ¢ m pI ¢

PREP CATALYSIS
CHEMISTRY
Section 1 (Maximum Marks : 24)
Instructions: Same as given in Physics.

19. A metal salt on heating with a mixture of KCl NO2


and conc. H 2SO4 yields a deep red vapour J.
The vapour on passing through an aqueous (b)
solution of KOH gives a yellow solution of
compound K . Passing SO2 gas through Cl
acidified solution of ‘K ’ leads to green O(CH2)2CH3
colouration of the solution due to the formation NO2
of ‘M’. The compound ‘M’ is
(a) K 2 CrO4 (b) Cr2 (SO4 )3
(c) Cr2O2- (d) CrO2- (c)
7 4

20. Which of the following statement(s) is/are CH3(CH2)2O


correct regarding the variation in the Cl
ionisation energies? NO2
(a) In general, first ionisation energy < second
ionisation energy < third ionisation energy
(b) The noble gases He, Ne, Ar, Kr, Xe and Rn have (d)
the highest ionisation energies in their respective O(CH2)2CH3
periods
Cl
(c) The group 1 metals Li, Na, K and Rb have the
lowest ionisation energies in their respective 23. The reaction(s) leading to the formation of
periods 3-hydroxypentanal is/are
(d) There is a general trend in ionisation energy within (i) Br
a horizontal period (a) CH2 == CH2 ¾ ¾¾
¾
2
®
(ii) KCN
21. Based on the compounds of group 15 elements, (iii) H 3 O+
(i) H2O, H+ O
the correct statement is/are (b) CH2 CH2 X
(ii) C 5 H 5 N . CrO3 . HCl OH–/H2O
(a) Al(CH3 )3 has three centre two electron bond in its
dimeric form O OH–/H2O
(b) The Lewis acidity of BCl 3 is greater than that of (c) +
O
AlCl 3 (i) Br2, CCl4
(c) AlCl 3 has the three-centre two electron bonds in its (d) CH2 CH2 + –
dimeric structure (ii) KOH, N R4Cl , Hexane-2, 6-diol
(iii) NaNH2, NH3
(d) BH3 has the three-centre two electron bonds in its
dimeric structure O (i) NH3, ether
X+ (ii) NH4Cl, H2O
22. In the following reaction sequence, the correct
structure of X is/are 24. A mixture of NaHCO3 and Na 2CO3 required
O a mL of a HCl solution to reach the
(i) Cl2/FeCl3 phenolphthalein end point and b mL of same HCl
CH2CH2CH3 (ii) Na-Hg/HCl is required to reach the methyl orange point.
‘X’ Which of the following statements is/are correct?
(iii) HNO3/H2SO4
Cl (iv) CH2 CHCH2O – Na+ ,H2O (a) Volume of HCl required for complete neutralisation
(v) H2/Pd/C
of Na 2 CO3 is 2a
NO2 (b) Volume of HCl required for complete neutralisation
of NaHCO3 is (b - a)
(c) It methyl orange is added at phenolphthalein end
(a) point, volume of HCl required to reach the end
point would be (b - a)
Cl
(d) If the mixture is titrated against NaOH of same
O(CH2)3CH3 strength as that of HCl, (b - 2a) mL of solution
would be required

PREP CATALYSIS
132 JEE Advanced~Practice Set 4

32. 1.0 mole of an ideal gas initially at 500 K and


Section 2 (Maximum Marks : 24) 20.30 atm is expanded to a final state at 400
Instructions: Same as given in Physics. K and 1.0 atm. To achieve the above change a
reversible path is constructed that involves an
25. Among the following complexes (K to P)
adiabatic expansion in beginning followed by
K3 [Fe(CN)6 ] ( K ), [Co(NH3 )6 ]Cl3 ( L ), an isothermal expansion to final state. The
Na3 [Co( ox)3 ] ( M ) net work down by the system will be ………
[Ni(H 2O)6 ] Cl2( N ), K 2 [Pt(CN)4 ](O ) kJ. (Given, CV , M = 1.5 R )
and [Zn(H 2O)6 ] (NO3 )2( P )
The number of diamagnetic complexes are ……
Section 3 (Maximum Marks : 12)
Instructions: Same as given in Physics.
26. Two flasks of equal volume
connected by a narrow tube of I II Paragraph X
negligible volume (as shown in An organic compound X(C10 H12 O 3 ) is not soluble
the figure given below) are at 27°C 117°C
in water or NaHCO 3 . A solution of Br2 in CCl 4 is
27°C and contain 0.60 mole of H 2 at 0.8 atm.
One of the flask is then immersed into a bath decolourised by X forming C10 H12 O 3 Br2 . ‘X’ on
kept at 117°C, while the other remains at 27°C. controlled ozonolysis followed by the treatment
The final pressure of H 2 will be …… . with (CH 3 ) 2 S gives ‘Y’ ( C 8 H 8 O 3 ) and C 2 H 4 O 2 . ‘Y’
can also be obtained by reaction between
27. Two weak acid solutions HA1 and HA2 each with
ortho-methoxy phenol with CHCl 3 in KOH solution
the same concentration and having pK a values followed be acid hydrolysis.
3 and 5 are placed in contact with hydrogen
electrode (1 atm at 25°C) and are 33. The correct structure of ‘X’ is
interconnected through a salt bridge. The, e. m. f CH3O
the of cell is …… .
28. 0.96 g of HI were heated to attain equilibrium (a) HO
2HI - H 2 + I2. The reaction mixture on
titration requires 15.7 mL of N/10 hypo solution. CH CH CH2OH
The degree of dissociation of HI is …… .
29. 1.4 g of acetone dissolved in 100 g of benzene OCH3
gave a solution which freezes at 277.12 K. Pure
benzene freezes at 278.4 K. 2.8 g of a solid (A) (b) HO
dissolved in 100 g of benzene gave a solution
which froze at 277.76 K. The molar mass of (A) CH CH CH2OH
is …… .
CH3O
30. The gaseous decomposition of Cl2O7 ( g) according
to the following reaction follows a first order (c) HO CH CH CH2
kinetics.
7 OH
Cl2O7 ( g) ® Cl2( g) + O2( g)
2 CH3O
When the decomposition is carried out in a
closed container, pressure measured after (d) HO CH CH CH2OH
15 min and after a very long time were found to
be 240 and 900 mm of Hg respectively. The
pressure that would be observed in the flask 34. What would be the major product if X is
after 140 min under identical conditions of treated with cold concentrated H 2SO4?
temperature will be …… .
CH3O CH3O
31. The entropy change in kJ when 1 kg of water is (a) (b)
heated from 27°C to 200°C forming superheated
HO HO
steam under constant pressure will be …… .
(Given, specific heat of water = 4180 J kg-1K -1 OCH3 OCH3
and specific heat of steam HO HO
= 1670 + 0.49 J kg-1K -1 and latent heat of (c) (d)
vaporisation =23 ´ 105 J kg-1.

PREP CATALYSIS
JEE Advanced~Practice Set 4 133

Paragraph A O
CH3
A solid organic ‘P’ of formula C15 H15 ON, was found
(c) CH3 C NH
to be insoluble in water, dil. HCl or dil. NaOH in
cold. After prolonged heating of ‘A’ with aqueous
NaOH, a liquid ‘R’ was found to be floating on the O
surface of alkaline mixture. ‘R’ did not solidified on
(d) C NH
cooling to room temperature. It was steam distilled
and separated. Also, acidification of alkaline mixture
with hydrochloric acid caused precipitation of a CH3 H 3C

white solid ‘S’ (C8H 8O 2 ). Some additional


36. It S is treated with SOCl2 followed by addition
informations are as follows :
of benzene and some AlCl3 would produce
NaNO2 b-naphthol
Highly coloured O
HCl/0°C
solid
R (a) C
Br2
C7H6NBr3
H 2O CH3
O
S on treatment with Br2 / FeBr 3 in CCl 4 produced a
single isomer C 8 H 7 O 2 Br while heating ‘S’ with soda lime (b) C
gave toluene.
CH3
35. The most likely structural formula of ‘P’ is
O
O
(c) C CH2
(a) CH3 C NH CH3

O
CH3
O
(d) C CH3
(b) C NH

CH3

MATHEMATICS
Section 1 (Maximum Marks : 24) 38. Consider the equation
æ 17 ö p
Instructions: Same as given in Physics. sin-1 ç x 2 - 6x + -1
÷ + cos k = . Then, which
è 2ø 2
37. For given complex number z1 = 5 + 12i and
of the following statements is (are) TRUE?
|z 2| = 4. Then, which of the following is (are)
(a) The largest value of k is 1 for which equation has
TRUE? two distinct solution is one.
(a) The maximum value of | z1 + iz 2 | = 17 (b) The equation must have real root if k Î æç - , 1ö÷
1
(b) The minimum value of | z1 + (1 + i )z 2 | = 13 - 9 2 è 2 ø

(c) The equation must have real root if k Î æç - 1, ö÷


1
z1 13
(c) The minimum value of = è 2ø
z2 + 4 / z2 4
1
(d) The equation has unique solution if k =
z1 13 2
(d) The maximum value of =
z2 + 4 / z2 3

PREP CATALYSIS
134 JEE Advanced~Practice Set 4

39. The graph of the function y = f ( x ) passing 44. Number of ways in which the letters of the
through the point (0, 1) and satisfying the word ‘ABBCABBC’ can be arranged such
dy dy that the word ABBC does not appear in
differential equation tan2 x = sec x (1 - y ) - .
dx dx word is less than or equal to ...... .
Then, which of the following statements is(are) 3n
Cn
TRUE? 45. For each positive integer n. Let Y n = 2n
.
Cn
(a) It is continuous function for all x 1
(b) It is periodic function a
If lim (Y n ) n = , where a and b are
(c) It is differential function for all x n ®¥ b
(d) It is neither an even nor an odd function relatively prime, then the value of ( a + b) is
40. A line with direction cosine proportional to equal to ......
( 2, 7, - 5) is drawn to intersect the lines 46. The ordered pair ( x , y ) satisfying the
x-5 y-7 z+2 x-3 y-3 z-6
= = and = = . equation x 2 = 1 + 6 log4 y and
3 -1 1 -3 2 4
Then, which of the following is(are) TRUE? y 2 = 2x y + 22x + 1 are ( x1 , y1 ) and ( x2 , y2 ), then
(a) The coordinates of point of intersection are (2, 8, - 3) the value of|x1x2 y1 y2| is ......
and (0,1, 2) 1^ 3$
(b) The length of intercepted on it is 78 47. Let a = 3 $i - $j and b = i + j and
2 2
(c) The length of intercepted on it is 68
(d) The equation of intersecting straight line be x = a + ( q 2 - 3) b, y = - pa + qb. If x and y
x-2 y-8 z+ 3 are orthogonal vector, such that p = f ( q ) ,
= =
2 7 -5 ( p ¹ 0, q ¹ 0), and the interval of f ( q ) is
41. Let f ( x ) =|x 2 - 4| x |+3|, where decreasing when q Î( a , b), then the value of
ì + x , if x > 0ü |a + b| is equal to ......
f ( x ) =|x|= í ý. Then, which of the
î - x , if x < 0þ 48. Let a = sin 10° , b = sin 50° , c = sin 70°, then
following is (are) TRUE?
æa + bö æ 1 1 1ö
(a) f (x) = 0 has four real roots 8abc ç ÷ ç + - ÷ is equal to .......
(b) Sum of the roots f (x) = 0 is 0 è c ø è a b cø
(c) f (x) = a has 8 real roots if 0 < a < 1 and product of
49. A and B are two square matrices. Such that
roots for this case is > 0
A2B = BA and if ( AB)10 = AK × B10, then the
(d) f (x) = a, has no real roots for a < 0
value of K is equal to .....
42. Let f ( x ) be a non-constant twice differentiable
function defined on ( - ¥ , ¥ ) such that 50. Let f ( x ) = ax + cos 2x + sin x + cos x is defined
æ1ö for " x Î R and a Î R and is strictly
f ( x ) = f (1 - x ) and f ¢ ç ÷ = 0. Then, which of the
è 4ø increasing
following is (are) TRUE? ém ö
(a) f ¢ ¢ (x) vanishes at least twice in [0, 1] function of the range of a is ê , ¥ ÷, then
ë n ø
(b) f ¢ æç ö÷ = 0
1
the minimum value of ( m + n ) is equal to.
è2ø
(c) ò f æç x + ö÷ sin x dx = 0
1/ 2 1
-1/ 2 è 2ø Section 3 (Maximum Marks : 12)
1/ 2 1
(d) ò f (t )e sin pt dt = ò1/ 2 f (1 - t )e
sin pt
dt Instructions: Same as given in Physics.
0
Paragraph X
Section 2 (Maximum Marks : 24) There are four boxes B1 , B2 , B 3 and B4 . Box Bi has
Instructions: Same as given in Physics. i cards and each card a number is printed, the
numbers are from 1 to i. A box is selected
43. In a DABC , AB = 3, BC = 4, CA = 5. P is any point i
inside the DABC such that
randomly, the probability of selecting box Bi is
10
ÐPAB = ÐPBC = ÐPCA = a, then tana is equal to
m and then a card is drawn.
, then the value of m + n is equal to.... (where, Let Ei represent the event that a card with number
n
m and n are relatively prime) ‘i’ is drawn. Then,

PREP CATALYSIS
JEE Advanced~Practice Set 4 135

51. P ( E1 ) is equal to 53. The chords in which the circle C cuts the
1 1 2 1 members of the family S of circle passing
(a) (b) (c) (d)
5 10 5 4 through A and B are concurrent at
(b) æç 2,
23 ö
(c) æç 3,
23 ö
52. P ( B3 / E2 ) is equal to (a) (2, 3) ÷ ÷ (d) (3, 2)
è 3ø è 2ø
1 1 1 2
(a) (b) (c) (d)
2 4 3 3 54. Equation of the member of the family of
circle S that bisects the circumference of C is
Paragraph A (a) x 2 + y 2 - 5x - 1 = 0
Let A( 3, 7) and B ( 6, 5) are two points (b) x 2 + y 2 - 5 x + 6y + 1 = 0
(c) x 2 + y 2 - 5 x - 6y - 1 = 0
C = x 2 + y 2 - 4x - 6y - 3 = 0 is a circle (d) x 2 + y 2 + 5 x - 6y - 1 = 0

Paper 2
PHYSICS
Section 1 (Maximum Marks : 24)
l This section contains SIX (06) questions.
l Each question has FOUR options for correct answer(s). ONE OR MORE THAN ONE of these four option(s) is (are)
correct options(s).
l For each question, choose the correct options(s) to answer the question.
l
Answer to each question will be evaluated according to the following marking scheme:
Full Marks : + 4 If only (all) the correct option(s) is (are) chosen.
Partial Marks : + 3 If all the four options are correct but ONLY three options are chosen.
Partial Marks : + 2 If three or more options are correct but ONLY two options are chosen, both of which are
correct options.
Partial Marks : + 1 If two or more options are correct but ONLY one option is chosen and it is a correct option.
Zero Marks : 0 If none of the options is chosen (i.e. the question is unanswered).
Negative Marks : - 2 In all other cases.
l
For example: If first, third and fourth are the ONLY three correct options for a question with second option being an
incorrect option; selecting only all the three correct options will result in + 4marks. Selecting only two of the three correct
options (e.g. the first and fourth options), without selecting any incorrect option (second option in this case), will result in
+2 marks. Selecting only one of the three correct options (either first or third or fourth option), without selecting any
incorrect option (second option in this case), will result in +1marks. Selecting any incorrect option(s) (second option in
this case), with or without selection of any correct option(s) will result in -2 marks.

1. A dichromatic beam of light with two 2. There are two massless springs A and B of
wavelengths; 6500 A° and 5200 A° are used in a spring constants kA and kB respectively,
Young’s double slit experiment. where kA > kB . If W A and W B are denoted as
Distance between slits is 2 mm and distance of work done on A and B respectively, then
screen from slits is 120 cm. [assume very slow compression]
Now choose the correct options. (a) if they are compressed by same distance,
(a) For l = 6500 A°, distance of third bright fringe from WA > WB
central maxima is 1.17 mm (b) if they are compressed by same force (upto
(b) For l = 5200 A°, distance of 3rd bright fringe from equilibrium state),WA < WB
central maxima is 1.17 cm (c) if they are compressed by same distance,
(c) Bright fringes of both wavelengths meet at a distance WA = WB
of 2mm from central maxima (d) if they are compressed by same force (upto
(d) Bright fringe of both wavelengths meet at a distance equilibrium state),WA > WB
of 1.56 mm from central maxima

PREP CATALYSIS
136 JEE Advanced~Practice Set 4

3. Two progressive transverse waves are described (a) The product of p and V at A is equal to the product
by of p and V at B
y1 = ( 3.0 cm ) sin [4x - 700 t ] (b) Pressure at B is greater than the pressure at A
(c) Work done by the gas during the process AB is zero
y2 = ( 3.0 cm ) sin [4x - 700 t - p/ 3]
(d) The change in internal energy from A to B is positive
(a) amplitude resulting wave is 3 3 cm
(b) resultant is a standing wave 6. A thin conducting rod AB is introduced in
(c) resultant is a progressive wave between the two point charges + q1 and - q2 as
(d) the particle will be oscillating in xy-plane shown in figure. For this situation, mark the
4. A light cylindrical vessel is kept on a correct statement(s).
horizontal surface. Its base area is A. A hole of A B
cross-sectional area a is made just at its +q1 –q2
bottom side. The minimum coefficient of
friction necessary for sliding of the vessel due
to the impact force of the emerging liquid is (a) The total force experienced by q1 is vector sum of
( a << A) electric force experienced by q1 due to q2 and by
(a) constant (b) a /A due to induced charges on rod
(c) 2 a /A (d) None of these (b) The end A will become negatively charged
5. The density (r ) of an ideal gas ρ (c) The total force action on + q1 will be greater than as
B
compared to the case without rod
varies with temperature T as
(d) The total force acting on - q2 will be greater than as
shown in figure. Select the wrong
A compared to the case without rod
statement/statement(s). T

Section 2 (Maximum Marks : 24)


l
This section contains EIGHT (08) questions. The answer to each question is a NUMERICAL VALUE.
l
Four each question, enter the correct numerical value (in decimal notation, truncated/rounded-off to the second
decimal place; e.g. 6.25, 7.00.- 0.33, -.30, 30.27, -127.30) using the mouse and the on-screen virtual numeric
keypad in the place designated to enter the answer.
l
Answer to each question will be evaluated according to the following marking scheme:
Full Marks : + 3 If ONLY the correct numerical value is entered as answer.
Zero Marks : 0 In all other cases.

7. A vernier calipers has 1 mm marks on the


main scale. It has 20 equal divisions on the 6
vernier scale, which match with 16 main scale 5
divisions. For this vernier calipers, the least
In|dN(t)|

count (in mm) is ..................... . 4

8. A hollow pipe of length 0.8 m is closed at one 3


end. At its open end a 0.5 m long uniform
string is vibrating in its second harmonic and 2
it resonates with the fundamental frequency 1
of the pipe. If the tension in the wire is 50 N 2 3 4 5 6 7 8
and the speed of sound is 320 ms-1, the mass of years

the string (in grams) is .................... . 10. A diatomic ideal gas is compressed adiabatically
1
9. To determine the half-life of a radioactive to of its initial volume. If the initial
dN ( t ) 32
element, a student plots a graph of ln temperature of the gas is Ti (in kelvin) and the
dt
dN ( t ) final temperature is aTi , the value of a is
versus t. Here, is the rate of radioactive
dt 11. At time t = 0, a battery of 10 V is connected
decay at time t. If the number of radioactive across points A and B in the given circuit. If
nuclei of this element decreases by a factor of p the capacitors have no charge initially, at what
after 4.16 yr, the value of p is time (in second) does the voltage across them
become 4 V? (Take, ln 5 = 1.6, ln 3 = 11
.)

PREP CATALYSIS
JEE Advanced~Practice Set 4 137
2 µF
2MΩ connects two blocks of masses
0.36 kg and
A B 0.72 kg. (Take, g = 10 ms-2). Find
the work done (in joule) by string
2MΩ 2 µF on the block of mass 0.36 kg
during the first second after the
12. A steady current I goes through a wire loop system is released from rest.
PQR having shape of a right angle triangle
14. A solid sphere of radius R has a charge Q
with PQ = 3x, PR = 4x and QR = 5x. If the
magnitude of the magnetic field at P due to distributed in its volume with a charge density
æ m I ö r = kr a , where k and a are constants and r is
this loop is k ç 0 ÷, find the value of k. the distance from its centre. If the electric field
è 48px ø
R 1
at r = is times that at r = R, find the value
13. A light inextensible string that goes over a 2 8
smooth fixed pulley as shown in the figure of a.

Section 3 (Maximum Marks : 12)


l This section contains FOUR (04) questions.
l Each question has TWO (02) matching lists : Column-I and Column-II.
l FOUR options are given representing matching of elements from Column-I and Column-II. ONLY ONE of these four
options corresponds to a correct matching.
l For each question, choose the option corresponding to the correct matching.
l For each question, marks will be awarded according to the following marking scheme.
Full Marks : + 3 If ONLY the option corresponding to the correct matching is chosen.
Zero Marks : 0 If none of the options is chosen (i.e. the question is unanswered).
Negative Marks : - 1 In all other cases.

15. One mole of a monoatomic ideal gas is taken 16. Column I shown four systems, each of the
through a cycle ABCDA as shown in the p-V same length L, for producing standing waves.
diagram. Column II gives the characteristics The lowest possible natural frequency of a
system is called its fundamental frequency,
involved in the cycle. Match them with each of whose wavelength is denoted as l f . Match
the processes given in Column I. each system with statements given in
p Column II describing the nature and
wavelength of the standing waves.
B A
3p
Column I Column II
A. Pipe closed at one end p. Longitudinal
waves
1p
C D O L
B. Pipe open at both ends q. Transverse
0 1V 3V V
9V waves

Column I Column II O L
C. Stretched wire clamped r. lf = L
A. Process A ® B p. Internal energy decreases at both ends
B. Process B ® C q. Internal energy increases
C. Process C ® D r. Heat is lost O L

D. Process D ® A s. Heat is gained D. Stretched wire clamped s. lf = 2L


at both ends and at
t. Work is done on the gas mid-point
(a) A-prt, B-pr , C-qs, D-rt (b) A-p, B-q, C-r, D-s
(c) A-pt, B-qs, C-rs, D-p (d) A-pr, B-pr, C-qs, D-rt O L
L/2
t. lf = 4L

PREP CATALYSIS
138 JEE Advanced~Practice Set 4

(a) A-pt, B-ps, C-qs, D-qr 18. Two transparent media of refractive indices m1
(b) A-qr, B-ps, C-qs, D-pt and m3 have a solid lens shaped transparent
(c) A-p, B-q, C-r, D-s
material of refractive index m 2 between them as
(d) A-pq, B-qr, C-rs, D-st
shown in figures in Column II. A ray
17. You are given many resistances, capacitors and traversing these media is also shown in the
inductors. These are connected to a variable figures. In Column I different relationships
DC voltage source (the first two circuits) or an between m1, m 2 and m3 are given. Match them to
AC voltage source of 50 Hz frequency (the next the ray diagram shown in Column II.
three circuits) in different ways as shown in
Column II. When a current I (steady state for Column I Column II
DC or rms for AC) flows through the circuit,
A. m1 < m 2 p.
the corresponding voltage V1 and V 2 (indicated µ1
in circuits) are related as shown in Column I.
Column I Column II µ2
A. I ¹ 0, V1 is p. V1 V2 µ3
proportional to I
6mH 3µF B. m1 > m 2 q.
µ1

V µ3 µ2
B. I ¹ 0, V2 > V1 q. V1 V2

6mH 2Ω
C. m2 = m3 r.

µ1
V
C. V1 = 0, V2 = V r. V1 V2 µ3
µ2
6mH 2Ω
D. m2 > m3 s.

V
µ1
D. I ¹ 0, V2 is s. V1 V2
proportional to I µ3
6mH 3µF µ2

t.
V
t. V1 V2 µ1
µ3
1kΩ 3µF
µ2

V
Codes
Codes (a) A-pr, B-q, C-r, D-s,
(a) A-p, B-q, C-r, D-st (b) A-q, B-p, C-rs, D-t
(b) A-pq, B-qr, C-rs, D-t (c) A-pr, B-qst, C-prt, D-qs
(c) A-pqs, B-qrs, C-rst, D-pq (d) A-qr, B-st, C-pq, D-qs
(d) A-rst, B-qrst, C-qp, D-qrst

PREP CATALYSIS
CHEMISTRY
Section 1 (Maximum Marks : 24) HO
OH
O
Instructions: Same as given in Physics.
(c)
,
19. The correct option( s) regarding the complex
[Pt (NH3 )2Cl2 ] is
CHO
(a) It is diamagnetic
O OH
(b) Hybridisation of metal ion involved is dsp 2
(c) The compound undergoes ionisation isomerism in (d)
,
presence of AgNO3
(d) It is tetrahedral in shape
22. PhCH == CHCOOH (Q ) is produced on reaction
20. If an electron can have three values of spin with acetic anhydride and sodium acetate
æ 1 1ö followed by hydrolysis. (Q ) further goes in the
quantum numbers ç + , 0 and - ÷, which of
è 2 2ø following sequence of reactions.
the following statement would be correct? H 2,Pd/C (i) SOCl2
(a) Li would have in the first period of the periodic (Q ) ¾¾¾¾® R ¾¾¾¾¾® S
MeOH (ii) AlCl3 , heat
table
(b) The total number of elements is the second period The major product( s) is
would have been ten
(a) (b)
(c) There would have been 27 elements in the fourth
period of periodic table
(d) Number of periods would have been less than the CH CH COOH
number of periods present in modern periodic table (c) CH3 (d)

21. C10H16( A) on ozonolysis followed be reduction


23. The lattice energy of solid NaCl is 180 kcal mol -1.
O
O The dissolution of the solid in H 2O is
with Zn - H 2O gives ( H ). endothermic to the extent of 1.0 kcal mol -1. If
the hydration energies of Na + and C - ions are
The major product( s) of the following sequence in the ratio of 6 : 5. What is the enthalpy of
is/are hydration of sodium ion?
NaOH (dilute) (a) - 8 kcal mol -1 (b) - 98 kcal mol -1
B
D (c) + 82 kcal mol -1 (d) + 100 kcal mol -1
H 24. A weak acid or base is titrated against a strong
(i) HO CH2CH2OH/H+ base or acid and the volume of titrant added, is
C
(ii) Li AlH4 plotted against pH as shown below :
(iii) H3O+
12
The products B and C are
10
CHO
O 8
OH pH
6
(a)
, 4
2
CHO OH 0
V1 V2
O
V
(b)
,
Which of the following statements regarding
above titration is/are correct?

PREP CATALYSIS
140 JEE Advanced~Practice Set 4

(a) The substance in a beaker is a solution of weak 30. Consider the following reaction :
dibasic acid
2NOBr( g) - 2NO(g) + Br (g) 2
(b) V2 = 2V1
(c) The pH corresponding to (V1 / 2) corresponds to If nitrosyl bromide (NOBr) is 34% dissociated at
pKa1 of the weak dibasic acid being titrated 25°C and a total pressure of 0.25 atm, then K p
(d) The pH corresponding to (3 / 2) V1 corresponds to for dissociation is Y ´ 10-3 mol L-1.
pKa2 of the weak dibasic acid The value of Y is …… .
31. An alloy weighing 1.05 g of Pb - Ag was
Section 2 (Maximum Marks : 24) dissolved in desired amount of HNO3 and the
Instructions: Same as given in Physics. volume was made 350 mL. An Ag electrode was
dipped in solution and Ecell of the cell
25. The number of compounds that produce Pt H 2|H +||Ag+|Ag was 0.503 V at 298 K. The
dioxygen on heating are …… . percentage of lead in alloy is …… .
KClO3 , (NH 4 )2 Cr2O7 , NH 4NO3 , NaNO3 , KMnO4 o
(Given, EAg = 0.80V)
+
|Ag
K 2Cr2O7 , H 2O2 , Pb3O4 , NH 4NO2
32. O2 undergoes photochemical dissociation to
26. The freezing point of 0.04 mole fraction of normal oxygen atom and 1 oxygen atom
acetic acid in benzene is 282.4 K. Acetic acid 1.967 eV more energetic than normal. The
exists partly as dimer. The value of dissociation of O2 into 2 normal oxygen atom is
equilibrium constant for dimerisation is …… . known to require 498 k J/mol O2.
[Given : Heat of fusion of benzene = 11.062 The maximum wavelength effective for the
kJ/mol; Freezing point of benzene = 284.7 K] chemical dissociation of O2 is …… .
27. K C for the reaction, N 2O4 2NO2 in -
chloroform at 219 K is 1.14. The free energy
Section 3 (Maximum Marks : 12)
change of the reaction when the concentration Instructions: Same as given in Physics.
of the two gases are 0.5 mol dm -3 each at the
33. Match the species given in List I with the
same temperature is …… .
shape given in List II.
28. For the given compound X, the total number
List I List II
of optically active stereoisomers is …… .
HO I. OSF2 p. Tetrahedral
O O O II. O2SF2 q. Linear
HO OH
III. XeF4 r. Square planar
HO OH IV. ClO-4 s. Trigonal pyramidal
OH OH
OH V. I3- t. See-saw
29. One mole of compound A (molar mass 171)
The correct option is
reacts with one mole of 4-hydroxy phenol and
one half of sodium carbonate in aqueous (a) I ® s; II ® p; III ® r ; IV ® p; V ® q
ethanol to produce compound B (molar mass (b) I ® s; II ® p; III ® r ; IV ® q; V ® t
200) in 87% yield. Spectral analysis indicate (c) I ® p; II ® s; III ® r ; IV ® p; V ® t
that both A and B has at least one phenyl (d) I ® q; II ® r ; III ® s; IV ® t; V ® p
ring. Deduce the structure of A and B. The 34. The desired product X can be prepared by
molecular mass of B is …… . reacting the major product of the reactions
OH given in List I with one or more appropriate
reagents in List II.
HO
A B
(ii) Na2CO3, H2O
CH3CH2OH (X)

PREP CATALYSIS
JEE Advanced~Practice Set 4 141

List I List II S. 4. (i) PCl 5


P. 1. CH2Br2 H 2C O (ii) 2NaNH2
(i) Anhy . AlCl3
+ CH3Cl OH
(ii) Cl2, hν
H2C
H
2. AlCl 3
Q. Red hot iron tube 3. C6H6 The correct option is
CH CH 873 K (a) P ® 2; Q ® 1; R ® 3; S ® 4
(b) P ® 2; Q ® 1; R ® 4; S ® 3
4. H2 SO4 , Zn
(c) P ® 1; Q ® 2; R ® 3; S ® 4
R. O 5. But-1, 3-diene (d) P ® 3; Q ® 2; R ® 4; S ® 1

C OH 36. Graphically Boyle’s law can be represented by


D various options depending on the coordinates.
Match the graph in List I with their
C OH coordinates in List II and select the correct
alternate from the codes given below:
O
List I List II
S. 6. CrO3 , Zn
CrO3
P. i. Plot of pressure versus
CH3COOH (glacial)
volume

The correct option is


(a) P ® 5; Q ® 2, 3; R ® 3, 4, 5; S ® 1, 6
(b) P ® 2; Q ® 1, 2; R ® 3, 2, 4; S ® 5, 6 Q. ii. Plot of log pressure
1
(c) P ® 1; Q ® 2, 6; R ® 2, 4; S ® 1, 5 versus log
(d) P ® 2; Q ® 1, 3; R ® 3, 4; S ® 5, 6 volume

35. Match each reaction given in List I with the


reagents given in List II and choose the correct R. iii. Plot of log pressure
option. versus log volume

List I List I
P. O 1. (i) Glycol + HCl
S. iv. Plot of 1
(ii) SOCl 2 versus
pressure
CHO (iii) DIBAL-H 1
CHO
(iv) H3O+ volume
Q. CHO CHO 2. (i) LiAlH4
v. Plot of pressure versus
(ii) Conc.H2 SO4 / D 1
(iii) O3 /Zn - AcOH volume

Codes
COOH CHO P Q R S
R. O 3. (i) BH3 /THF (a) i ii iv v
Me (ii) H2O2 /OH- (b) v i iii ii
Ph (c) v iii ii i
Ph Me (d) i iv ii iii

PREP CATALYSIS
MATHEMATICS
Section 1 (Maximum Marks : 24) x2 y2
42. Given ellipse + = 1 and the hyperbola
16 7
Instructions: Same as given in Physics.
x2 y2 1
- = . If the ordinate of one of the points
37. Let q and f Î [0, 2p ] be such that 144 81 25
æ q qö of intersection is produced to cut asymptote at P.
2 cos q (1 - sin f ) = sin2 q ç tan + cot ÷ cos f - 1, Then, which of the following is (are) TRUE?
è 2 2ø
(a) They have the same foci
3
tan(2p - q) > 0 and - 1 < sinq < - . Then, (b) Square of the ordinate of point of intersection is
63
2 25
which of the following is(are) FALSE? (c) Sum of the square of coordinate of P is 16
p p 4p (d) P lies on the auxiliary circle formed by ellipse.
(a) 0 < f < (b) <f<
2 2 3
(c)
4p
<f<
3p
(d)
3p
< f < 2p
Section 2 (Maximum Marks : 24)
3 2 2 Instructions: Same as given in Physics.
38. Consider the circle x 2 + y 2 - 10x - 6 y + 30 = 0. 43. The number of solution of the equation
Let O be the centre of the circle and tangent at 1
A ( 7, 3) and B (5, 1) meet at C. Let S = 0 e- |ln{ x }| - { x } |ln{ x }| = [sgn( x )]
represents family of circles passing through A (where, [×] and { x } are greatest integer and
and B. Then, which of the following statements fractional part function respectively) is/are
is (are) TRUE? cos( x + a ) sin( x + a ) sin(b - g )
(a) The area of quadrilateral OACB is 4 sq units
(b) The radical axis for the family of circle S = 0 is
44. If f ( x ) = cos( x + b ) sin( x + b) sin( g - a ) and
x + y = 10 cos( x + g) sin( x + g ) sin(a - g )
(c) The smallest possible circle of family S = 0 is 25
x 2 + y 2 - 12 x - 4 y + 38 = 0` f( 2) = 6, then å f (r ) is equal to
r =1
(d) The coordinate of point C are (7, 1)
39. Let M and N be two 3 ´ 3 matrices such that 45. f is a real valued, continuous function such
MN = NM . Further, if M ¹ N and M = N . 2 2 4 æp ö æp ö p
that f ç + x ÷ + f ç - x ÷ = . Then, the value of
è6 ø è3 ø 2
Then, which of the following is(are) TRUE? p/ 2
(a) The value of determinant of (M 2 + MN 2 ) is zero integral ò0 (cos( f ( x )))2 dx is kp, then the value
(b) There is a 3 ´ 3 non-zero matrix È such that of k is .....
(M 2 + MN 2 ) È is a zero matrix ® ® ®
(c) The value of determinant (M 2 + MN 2 ) ³ 1 46. If a , b and c are non-coplanar unit vectors
(d) For a 3 ´ 3 matrix È, of (M 2 + MN 2 ) È equals the ®®® ® ® ® ®® ®
such that [a b c ] = [b ´ c c ´ a a ´ b ],then the
zero matrix, then È is the zero matrix, ® ® ® ®
projection of ( b + c ) on a ´ b is equal to .......
40. Let f ( x ) = a5 x + a4x + a3 x + a2x + a1x , where
5 4 3 2
47. The number of ordered pair of ( x , y ) such that
ai ’s are real and f ( x ) = 0 has a positive root a 0.
( x + iy )2018 = x - iy.
Then, which of the following is (are) TRUE?
(a) f ¢(x) = 0 has a root a1 such that 0 < a1 < a0 48. The tangent at a point P on the curve
(b) f ¢¢(x) = 0 has at least one real root æ 2 + 4 - x2 ö
(c) f ¢(x) = 0 has at least one real root y = ln ç ÷ - 4 - x 2 meets the Y -axis
ç 2 - 4 - x2 ÷
(d) None of the above è ø
at T , then the value of PT 2 is equal to ....
41. If a and b are two numbers such that
a 2 + b2 = 7 and a3 + b3 = 10. Then, which of the 49. The distance of the point P ( - 2, 3, - 4) from the
x + 2 2 y + 3 3z + 4
following is(are) TRUE? line = = measured parallel
3 4 5
(a) The maximum value of | a + b| is 5 to the plane 4x + 12 y - 3z + 1 = 0 is d, then the
(b) The maximum value of (a + b) is 4 value of d is equal to .....
(c) The minimum value of (a + b) is 1 1
50. The value of SS i . j is equal to .....
(d) The minimum value of | a + b| is 1 15 1 £ i £ j £ 9

PREP CATALYSIS
JEE Advanced~Practice Set 4 143

The correct option is


Section 3 (Maximum Marks : 12) (a) P ® 5; Q ® 4; R ® 2; S ® 6
Instructions: Same as given in Physics. (b) P ® 6; Q ® 3; R ® 1; S ® 5
(c) P ® 1; Q ® 4; R ® 2; S ® 5
51. f ( x ) is polynomial function of degree 6, which (d) P ® 1; Q ® 3; R ® 2; S ® 5
1/ x
æ f(x)ö 53. In the parallelogram ABCD with angle A = 60°,
satisfies lim ç1 + 3 ÷ = e2 and has local
x ®0 è x ø the bisector of angle B is drawn which cuts the
maximum at x = 1 and local minimum at x = 0 side CD at a point E. A circle S1 of radius ‘r’ is
and x = 2. inscribed in the DECB. Another circle ‘S 2’ is
List I List II inscribed in the trapezoid ABED.
P. 6
The coefficient of x is 1. - 2/3 List I List II
Q. The coefficient of x 5 is 2. 0 P. The value of radius of S2 is 1. 2 3 r

R. The coefficient of x 4 is 3.
Q. The value of distance between 2. 3
2 r
the centres of S1 and S2 is 2
S. The coefficient of x 3 is 4. - 12 / 5
R. The value of the length of 3. 7r
5. 2/3 internal common tangent of S1
6. 12 and S2 is
5 S. The value of length CE is 4. 3 r
2
The correct option is
(a) P ® 5; Q ® 4; R ® 3; S ® 2 The correct option is
(b) P ® 1; Q ® 4; R ® 2; S ® 3 (a) P ® 4; Q ® 2; R ® 1; S ® 3
(c) P ® 5; Q ® 6; R ® 3; S ® 2 (b) P ® 4; Q ® 1; R ® 2; S ® 3
(d) P ® 1; Q ® 6; R ® 3; S ® 2 (c) P ® 4; Q ® 3; R ® 2 ; S ® 1
æ 30 ö æ 20 ö
m æ pö p (d) P ® 4; Q ® 3; R ® 1; S ® 2
52. f ( m ) = S ç ÷ç ÷, where ç ÷ = Cq.
i=0è 30 - i ø è m - i ø èqø 54. Given a function ‘f’ which has a derivative
Then, match the elements of List-I with the f ¢ ( x ) for every real x and which satisfy f ¢ ( 0) = 2
elements of List-II. and f ( x + y ) = e y f ( x ) + ex f ( y ) for all x and y
List I List II List I List II
50 P. 1. 0
P. Maximum value of f (m) is 1. C25 The domain of f (x) is
Q. 50
2. 100
C50 Q. The range of f (x) is 2. (- ¥, ¥)
The value of S f (m) is
m =0 1
ò
R. The value of f (x) dx is 3. 2
50 0
R.
The value of S (f (m)) is 2 3. 2 50
m =0 S. The value of lim f (x) is 4. é - 2 , ¥ö
x ® 0 êë e ÷ø
S. The value of 3f (0) - 8f (1) 4. 2100
+ 13f (2) - 18f (3) + .... The correct option is
+ 253 f (50) is (a) P ® 2; Q ® 4; R ® 1; S ® 3
5. 0 (b) P ® 4; Q ® 2; R ® 1; S ® 3
(c) P ® 2; Q ® 4; R ® 3; S ® 1
6. 5
(d) P ® 4; Q ® 2; R ® 3; S ® 1

PREP CATALYSIS
Answers
Paper 1
1. c, d 2. a,b,c,d 3. c 4. a, c 5. d 6. b, c, d 7. 8.00 8. 4.00 9. 5.00 10. 9.00
11. 11.00 12. 1.65 13. 1.16 14. 2.06 15. d 16. d 17. b 18. b 19. b 20. a,b,c,d
21. a, b, c 22. b 23. b, c 24. a, c, d 25. 4 26. 0.90 27. 0.06 28. 0.21 29. 232 30. 495.75
31. 7.52 32. –1.13 33. d 34. a 35. c 36. d 37. a, d 38. a, b, d 39. a, b, c 40. a, b, d
41. a,b,c,d 42. a,b,c,d 43. 37 44. 361 45. 43 46. 128 47. 0 48. 6 49. 1023 50. 25
51. c 52. c 53. b 54. b

Paper 2
1. a, d 2. a, b 3. a, c, d 4. a, c 5. c, d 6. a,b,c,d 7. 0.20 8. 10.00 9. 8.00 10. 4.00
11. 2.00 12. 7.00 13. 8.00 14. 2.00 15. a 16. a 17. d 18. c 19. a, b, c 20. a, c, d
21. b 22. b 23. b 24. a, c, d 25. b 26. 1.17 27. –19.17 28. 512 29. 200 30. 0.01
31. 99.97 32. 174 33. a 34. b 35. b 36. c 37. a, c, d 38. a, c, d 39. a, b 40. a, b, c
41. a, b, d 42. a,b,c,d 43. 0 44. 150 45. 0.25 46. 1 47. 2020 48. 4 49. 85 50. 77
51. a 52. d 53. c 54. c

SCORE SHEET - Paper 1


Section No. of Marks from Marks from Marks Obtained
Correct Questions Correct Questions (A) Incorrect Questions (B) (A-B)
................ ............................... ............................... ............................... ...............................
................ ............................... ............................... ............................... ...............................
................ ............................... ............................... ............................... ...............................
Percentage Marks = Marks Obtain/Total Marks x 100

SCORE SHEET - Paper 2


Section No. of Marks from Marks from Marks Obtained
Correct Questions Correct Questions (A) Incorrect Questions (B) (A-B)
................ ............................... ............................... ............................... ...............................
................ ............................... ............................... ............................... ...............................
................ ............................... ............................... ............................... ...............................
Percentage Marks = Marks Obtain/Total Marks x 100
Note To expect your success marks in the test should be between 65%-70%.

PREP CATALYSIS
PREP CATALYSIS
JEE Advanced

PRACTICE SET 5 (With Solutions)

Duration : 3 Hours Max. Marks . 360

Paper 1
PHYSICS
Section 1 (Maximum Marks : 24)
l
This section contains SIX (06) questions.
l
Each question has FOUR options for correct answer(s). ONE OR MORE THAN ONE of these four option(s) is (are)
correct options(s).
l
For each question, choose the correct options(s) to answer the question.
l
Answer to each question will be evaluated according to the following marking scheme:
Full Marks : + 4 If only (all) the correct option(s) is (are) chosen.
Partial Marks : + 3 If all the four options are correct but ONLY three options are chosen.
Partial Marks : + 2 If three or more options are correct but ONLY two options are chosen, both of which are
correct options.
Partial Marks : + 1 If two or more options are correct but ONLY one option is chosen and it is a correct option.
Zero Marks : 0 If none of the options is chosen (i.e. the question is unanswered).
Negative Marks : - 2 In all other cases.
l
For example: If first, third and fourth are the ONLY three correct options for a question with second option being an
incorrect option; selecting only all the three correct options will result in + 4 marks. Selecting only two of the three
correct options (e.g. the first and fourth options), without selecting any incorrect option (second option in this case), will
result in +2 marks. Selecting only one of the three correct options (either first or third or fourth option), without selecting
any incorrect option (second option in this case), will result in +1 marks. Selecting any incorrect option(s) (second
option in this case), with or without selection of any correct option(s) will result in -2 marks.

1. A fine capillary tube of (a) Mass of liquid in capillary tube is pr 2h r


(b) Mass of liquid in capillary tube is pr 2 æçh + ö÷ r
r
radius r dipped in a beaker
è 3ø
containing water. The water hrrg
rises in capillary upto height h (c) Surface tension of water isT =
2 cos q
h. Let q = angle of contact (d) Surface tension of water in glass capillary is
and r = density of water.
r æçh + ö÷ rg
r
è 3ø
Now, choose the correct 2r approximately T =
options. 2

PREP CATALYSIS
146 JEE Advanced~Practice Set 5

2. To study a relation of frequency and length of (a) mass will executes SHM with time period inversely
a given wire under constant tension using a proportional to radius of cylinder
sonometer, let you are performing an pr 2 g
(b) mass will executes SHM with time period
experiment in your school lab. 2m
Now, choose the correct options. (c) mass will not execute SHM
(a) Tuning fork of known frequency is touched to wire 1 pr 2gr
(d) mass will executes SHM with frequency
for observing resonance 2p m
(b) Vibrations of wire are observed by paper riders
5. For the given figure, two blocks of equal mass
1 æT ö
(c) Frequency of vibration of wire is f = . ç ÷ m are connected by a light string that passes
LD è pr ø
over a smooth pulley. The block A is connected
(where, L = length of wire between bridges, by spring constant k. Initially spring is relaxed.
D = diameter and r = density of wire ) Then, the block A is slowly pulled down a
(d) Frequency of tuning forks remains constant with distance x and released. The magnitude of
temperature of lab accelerations of the left and the right blocks
3. A hollow copper tube is filled completely using immediately after releasing are a1 and a2,
a steel rod. The rod and tube are joined by two respectively.
rivet pins as shown in the figure.

d d/2 Steel m B
A m
Copper
Steel k
rivets
Temperature of assembly is raised by DT after
Now, choose the correct options.
setting the rivets. 2 mg
(a) a1 = a2 (b) a1 = a2 , if k £
Now, choose the correct options. x
1 k 2 mg
(a) Stress in copper (sCu ) tube = - Stress in (c) If a1 = x,a2 = 0 (d) a1 < a2 , if k >
3 m x
steel (ssteel ) tube
6. Hydrogen like atom of atomic number 2 is in an
(b) Stress in copper tube = - Stress in steel tube
excited state of quantum number 2n. It can emit
(c) Strain in steel tube = asteel DT a maximum energy proton of 204 eV. If it makes
s a transition to quantum state n, a photon of
(d) Strain in copper tube = Cu + a Cu DT
YCu energy 40.8 eV is emitted. Calculate the
minimum energy (in eV) that can be emitted by
4. A cylindrical shaped wooden having mass m is this atom during deexcitation. Ground state
floating inside a liquid with its axis is energy of hydrogen atom is –13.6 eV.
parallel to the vertical. The mass in (a) 12.42 eV (b) 10.58 eV
depressed a little by a force and then force is (c) 15.42 eV (d) 14.89 eV
removed, then

PREP CATALYSIS
Section 2 (Maximum Marks : 24)
l This section contains EIGHT (08) questions. The answer to each question is a NUMERICAL VALUE.
l Four each question, enter the correct numerical value (in decimal notation, truncated/rounded-off to the second
decimal place; e.g. 6.25, 7.00.- 0.33, -.30, 30.27, -127.30) using the mouse and the on-screen virtual numeric
keypad in the place designated to enter the answer.
l Answer to each question will be evaluated according to the following marking scheme:
Full Marks : + 3 If ONLY the correct numerical value is entered as answer.
Zero Marks : 0 In all other cases.

7. A spherical ball of mass m is kept at the The masses move such that the portion of the
highest point in the space between two fixed, string between P1 and P2 is parallel to the
concentric spheres A and B, (see figure). The inclined plane and the portion of the string
smaller sphere A has a radius R and the space between P2 and M3 , is horizontal. The masses
between the two spheres has a width d. The M 2 and M3 are 4.0 kg each and the coefficient
ball has a diameter very slightly less than d. of kinetic friction between the masses and the
All surfaces are frictionless. The ball is given a
surfaces is 0.25. The inclined plane makes an
gentle push (towards the right in the figure).
The angle made by the radius vector of the ball angle of 37° with the horizontal. If the mass M1
with the upward vertical is denoted by q. moves downwards with a uniform velocity and
T
tension in string is T , then find value of .
g
Sphere B
10. A particle is suspended vertically from a point O
θ by an inextensible massless string of length L.
A vertical line AB is at a distance L / 8 from O
d
R O as shown in figure. The object is given a
horizontal velocity u. At some point, its motion
Sphere A ceases to be circular and eventually the object
passes through the line AB.
Let N A and N B denote the magnitudes of the Let velocity of particle at the instant of crossing
normal reaction forces on the ball exerted by 2 3v 2
AB is v, then value of will be
the spheres A and B, respectively. What will be gL
æ 2ö
the value of N A for q ³ cos-1 ç ÷ ? O
è 3ø L /8
A

8. At a given instant there are 25% undecayed


radioactive nuclei in a sample. After 10 s the
number of undecayed nuclei reduces to 12.5%. L
If the time in which the number of undecayed
nuclei will further reduce to 6.25% of the
t B
u
reduced number is t1, then find the value of 1 .
10
11. A rock is 1.5 ´ 109 yr old. The rock contains
9. Masses M1 , M 2 and M3 are connected by 238
U, which disintegrates to form 206Pb.
strings of negligible mass, which passes over Assume that there was no 206Pb in the rock
massless and frictionless pulleys P1 and P2 as initially and it is the only stable product
shown in figure. formed by the decay. If the ratio of number of
P1 nuclei of 238U to that of 206Pb in the rock is
closest to integer Z , then find Z. Half-life of
238
U is 4.5 ´ 109 yr. (Take, 21/ 3 = 1.259)
M
2
P2 12. A uniform circular disc of mass 50 kg and
M1 radius 0.4 m is rotating with an angular
37° M3
velocity of 10 rad/s about its own axis, which is
vertical. Two uniform circular rings, each of

PREP CATALYSIS
148 JEE Advanced~Practice Set 5

mass 6.25 kg and radius 0.2 m are gently (Take, k1 = 1.8 N/ m, k2 = 3.2 N / m, m = 200 g)
placed symmetrically on the disc in such a
manner that they are touching each other
along the axis of the disc and are horizontal. 60 cm
Assume that the friction is large enough such k1 k2
m v
that the rings are at rest relative to the disc
and the system rotates about the original axis.
A C D B
The new angular velocity (in rads -1) of the
system is
13. Two light springs of force constants k1 and k2 and 14. A column of mercury of length 10 cm is
a block of mass m are in one line AB on a smooth contained in the middle of a horizontal tube of
horizontal table such that one end of each spring length 1 m, which is closed at both the ends.
is fixed on rigid supports and the other end is The two equal lengths contain air at standard
free as shown in the figure. The distance CD atmospheric pressure of 0.76 m of mercury.
between the free ends of the spring is 60 cm. If The tube is now turned to vertical position. By
the block moves along AB with a velocity what distance will the column of mercury be
120 cm/s in between the springs. Calculate the displaced? Assume, temperature to be
period of oscillation of the block. constant.

Section 3 (Maximum Marks : 12)


l This section contains TWO (02) paragraphs. Based on each paragraph, there are TWO(02) questions.
l Each question has FOUR options. ONLY ONE of these four options corresponds to the correct answer.
l Four each question, choose the option corresponding to the correct answer.
l Answer to each question will be evaluated according to the following marking scheme:
Full Marks : + 3 If ONLY the correct option is chosen.
Zero Marks : 0 If none of the options is chosen (i.e. the question is unanswered).
Negative Marks : - 1 In all other cases.

Paragraph X
D
One mole of an ideal monoatomic gas undergo a 4p0
thermodynamic cyclic process ABCDA as shown in p
2p0 C
internal energy (U) and density (r) graph. (b) A
p0 B

2U0 D C
V0 / 2 V V0

U C
4p 0
U0 A B
p
2p 0 B
ρ0 ρ 2ρ0 (c) D
p0 A
15. The corresponding pressure ( p) versus volume
(V ) graph is V0 / 2 V V0

C D
4p0 4p 0
p p C
2p0 D 2p0 A
(a) B (d)
p0 A p0
B

V0 / 2 V V0 V0 / 2 V V0

PREP CATALYSIS
JEE Advanced~Practice Set 5 149

16. The corresponding temperature (T ) versus Here, m = refractive index for yellow colour and A = angle
pressure ( p) graph is of prism.
D
2T0 C
T
(a)
A δR δv
A
T0
B
Red
White
p0 2p 0 p 4p 0
Violet
D Also, dispersive power of a prism is
2T 0 C
T Angular dispersion æ mV - m R ö
w= =ç ÷
(b) Mean deviation è m-1 ø
T0
A B Here, mV = refractive index for violet colour
and m R = refractive index for red colour.
p0 2p 0 p 4p 0
17. If, we use two prisms of different materials
D (refractive index, m1 and m 2) and different angles (A1
2T0 C and A2 ) as shown in figure.
T
(c)
A1 µ2
T0 t
A B ligh
ite
Wh µ1
A2
p0 2p 0 p 4p 0
D C
2T0 If emergent light rays are parallel to incident ray, then
T (a) A1 = A2 (b) m1 = m 2
(c) A1 (m1 - 1) = A2 (m 2 - 1) (d) A1 (m1 - 1) = A2 (1 - m 2 )
(d)
T0 B 18. If for a prism combination, we get following picture
A
ite
Wh
µ2 am
p0 p 2p 0 be
A1
Paragraph A
A prism disperses white light into it’s ite µ1
Wh m A2
bea
constituent colours.
Deviation produced by a thin prism is
Then, choose the correct options.
d = (m - 1) A (a) w1d1 = w2 d 2 (b) w1w2 = d1d 2
(c) w1d 2 = - w2 d1 (d) w1d1 = - w2 d 2

PREP CATALYSIS
CHEMISTRY
Section 1 (Maximum Marks : 24) 23. The reaction(s) leading to the formation of
Instructions: Same as given in Physics. 2 - methylbutan - 2-ol is/are
O
19. Which of the following statement is correct (a) (i) C2H5MgBr
about PCl5 ? (ii) NH4Cl, H2O
(a) Three (P—Cl) equatorial bonds lie in one plane and O
(i) CH3MgBr
make an angle of 120° with each other (b)
(ii) NH4Cl, H2O
(b) Two (P—Cl) axial bonds one lying above and the
other lying below the equatorial plane make an O
(i) CH3MgBr
angle of 90° with the plane (c) (ii) NH4Cl, H2O
(c) Axial bonds are slightly longer and slightly weaker OCH3
than equatorial bonds O
(d) PCl 5 exists as [PCl+4 ] [PCl -6 ] in aqueous state (d) (i) THF, ∆
+ 2C2H5MgBr +
(ii) H3O
20. NiCl2 in the presence of dimethyl glyoxime CH3 NH2
(DMG) gives a complex which precipitates in
the presence of NH 4OH, giving a bright red 24. Consider two liquids A and B having pure
colour. The correct statement regarding the vapour pressure p°A and p°B forming an ideal
complex formed is/are 1 1
-
(a) CNO is a strong field ligand that forces the solution. The plot of versus (when X A
XA YA
electrons to pair up
(b) Complex formed is diamagnetic and Y A are the mole fraction of liquid A in
(c) The complex is stabilised because two six liquid and vapour phase respectively) is linear
membered chelate rings are formed with slope and intercepts respectively.
(d) The complex formed is tetrahedral
21. Which of the following reaction is correct about
compounds of group 15 elements?
(a) PCl 3 is formed when white phosphorus reacts with 1
SOCl 2 XA
(b) NO is paramagnetic in liquid state
(c) The percentage of p - character in the orbitals 1
YA
forming P—P bonds in P4 is 75%
(d) SO2 cannot be collected over water p°A - pB° (pB° - p°A )
(a) p°A / pB° and (b) p°A / pB° and
22. In the following reaction sequence, the correct pB° pB°
structure(s) of ‘A’ is/are (p° A - p°B ) p°B - p° A
(c) p°B / p° A and (d) p°B / p° A and
p°B p°B
O3 Dil. NaOH
A(C8H14) (CH3)2S
B(C8H14O2)
Optically active
O Section 2 (Maximum Marks : 24)
(C)
Instructions: Same as given in Physics.

25. Total number of chemical species having bent


(a) (b) shape is ……
O3 , NO2 , NO2+ , N 2O, SO2 , N3- , CS2 , C3O2 , H 2S
26. The number of moles of AgI, which may be
(c) (d) dissolved in 1.0 L of 1.0 M CN - solution is ……
(Given, Ksp for AgI and K C for Ag(CN)-2 are
1.2 ´ 10-17 M2 and 71
. ´ 1019 M-2 respectively.)

PREP CATALYSIS
JEE Advanced~Practice Set 5 151

27. CuSO4 has rock salt structure. A cubic crystal Y

of CuSO4 of edge length 17.1 mm is dissolved


in water to make 500 mL solution of pH 5. k
h X
The edge length of FCC unit cell of CuSO4 is l
…………
(Given, Z
Cu(H 2O)2+
6 + H 2O 3 [Cu(H 2O)5 (OH)]+ d112 spacing for cystalline k is 0.3080 nm. A
-5
+ H3O , K = 10 and 3 1.5 = 114
+
. ) wavelength of 0.542 nm falls on this plane. The
value of sinq for first order reflection in Bragg’s
28. The standard enthalpy of formation of FeO diffraction is …………
and Fe2O3 is - 85 kcal mol-1 and - 209 kcal
mol-1 respectively. A mixture of two oxides
contains FeO and Fe2O3 in the mole ratio
Section 3 (Maximum Marks : 12)
3 : 1. If on oxidation, it is changed into the Instructions: Same as given in Physics.
mixture of 1 : 3 mole ratio, then the amount of Paragraph X
thermal energy released per mole of mixture An organic compound A (C11H16O) is optically active
will be …….
but does not effervesces with NaH. A on refluxing
29. For the reversible reaction in equilibrium: with dilute H 2SO 4 gives B(C9H12O) as one product. B
K1
P - Q. The values of K and K 2 and changes orange colour of acidic dichromate solution
1
K2 to blue-green and a new compound C(C9H10O) is
4 ´ 10-3 mol L-1 sec-1 and 5 ´ 10-3 sec-1 formed. C on treatment with C2H5 MgBr followed by
respectively. If we add 0.9 mole of Q in the hydrolysis results in an achiral alcohol. Also ‘A’ on
equilibrium mixture, initially containing treatment with HI gives a chiral compound D(C9H11I)
4 moles of P, then the time required for as one of the organic product.
3
concentration of Q to become of the 33. The compounds ‘A’ and ‘C’ are
8
concentration of P at initial equilibrium will CH3
be ……
Note The volume of mixture is 1L and remains (a) ,
O O
constant.
30. Iron forms a series of compounds of type CH3 O
Fex (CO) y . In air, they are oxidised to Fe2O3
and CO2 gas. After heating 90.142 g sample in
(b) ,
air, you isolate the CO2 in a 1.50L flask at O
25°C. The pressure of the gas is 44.9 mm Hg.
y CH3
The value of in the simplest empirical O
x
formula of Fex (CO) y will be .........
(c) ,
31. A balloon is 0.5 m in diameter and contains O
air at 25°C and 1 bar pressure. It is then
filled with air isothermally and reversibly O
until the temperature reaches 5 bar.
Assuming that pressure is proportional to the H
(d) ,
diameter of the balloon, the work done in the O
process is x ´ 106J. The value of x is………
32. For a cubic unit cell, the distance dh kl
34. Consider the following reaction,
between planes with Miller indices hkl is - +
a2 D + C2H5 OK / C2H5OH ¾ X ; Here, X is
2
given by dhkl = 2 where, a is the (a) a pair of enantiomers
h + k2 + l 2 (b) a single pure enantiomer
length of cubic unit cell. (c) a pair of diastereomers
(d) does not show stereoisomerism

PREP CATALYSIS
152 JEE Advanced~Practice Set 5

Paragraph A 35. Compound A and B are


The —NO 2 group in an aromatic ring deactivates (a) homologues
the ortho and para positions for an electrophilic (b) identical
attack. When —NO 2 group is present at ortho or (c) functional isomers
(d) different compounds
para positions of a leaving group, it activates the
ring for nucleophilic attack. The reduction of —NO 2 36. Compound C is
group by metal in acid causes its reduction to —NH 2 Cl
group and then the ring becomes strongly activated
for electrophilic attack. The strong activation of
(a) (b)
—NH 2 group is moderated by its acylation with
CH3 COCl to — NHAc group. Deacylation is carried
out by hydrolysis with H3 O + or OH - . NH2 NH2
Cl Cl COCH3
(i) KCN, D
A
(i) Sn/HCl (ii) Br2/Fe
(ii) CH3COCl
C (c) (d)
(iii)CH3 CH CH2/H+
(iv) H3O+/D (i) Br2/Fe
(ii) KCN, D
B COCH3
N NH2 NH2
O O

MATHEMATICS
Section 1 (Maximum Marks : 24) (c) Their exists a > 1, such that | f ¢ (x)| < | f (x)| for all
x Î (a, ¥)
Instructions: Same as given in Physics. (d) Their exists b > 0, such that | f (x)| + | f ¢ (x)| £ b for all
x Î (0, ¥)
37. Let z1 , z 2 , z3 be three non-zero complex
numbers, such that z 2 ¹ 1, a =|z1|, b =|z 2|and
39. If ( x - a ) cos q + y sin q = ( x - a ) cos f
æq ö æ fö
a b c + y sin f = a and tanç ÷ - tanç ÷ = 2b, then
è 2ø è 2ø
c =|z3|. Let b c a = 0. Then, which of the which of the following is (are) TRUE?
1
c a b (a) y 2 = 2ax - (1 - b 2 ) x 2 (b) tan q/ 2 =
(y + bx)
x
following is(are) TRUE? f 1
2
(c) y 2 = 2bx - (1 - a 2 )x 2 (d) tan = (y - bx)
æz ö æ z - z1 ö 2 x
(a) arg ç 3 ÷ = arg ç 3 ÷ 40. For every pair of continuous function
è z1 ø è z 2 - z1 ø
f1 g = [0, 1] ® R such that max { f ( x ) : x Î [0, 1]} =
(b) Orthocentre of triangle formed by z1, z 2, z 3 is max{ g( x ) : x Î [0, 1]}. Then, which of the
z1 + z 2 + z 3 following statements is(are) TRUE?
(c) If triangle formed by z1, z 2 , z 3 is equilateral, then its (a) (f (c))2 + 3f (c) = (g (c))2 + 3g (c) $ c Î[0,1]
area is
3 3
| z1| 2 (b) (f (c))2 + f (c) = (g (c))2 + 3g (c) $ c Î[0,1]
4 (c) (f (c))2 + 3f (c) = (g (c))2 + g (c) $ c Î[0,1]
(d) If triangle formed by z1, z 2 , z 3 is equilateral, then (d) (f (c))2 = (g (c))2 $ c Î[0,1]
z1 + z 2 + z 3 = 0 41. a and b are two given vectors, on these vectors
38. Let f be a real valued function defined on the as adjacent sides, a parallelogram is
x constructed. The vector (s) which is altitude of
interval (0, ¥) by f ( x ) = log x + ò0 1 + sin t dt. parallelogram and its perpendicular to a is
b ´ (a ´ b) æa × b ö
Then, which of the following statements is(are) (a) 2
(b) çç 2 ÷÷ a - b
TRUE? | b| è|a | ø
æ® ® ® ö
ç a ´ (a ´ b ) ÷ 1 ® ® ® ® ®
(a) f ¢ ¢ (x) exists for all x Î (0, ¥) (c) (d) {| a | 2 b - (a × b ) a }
ç ® ÷ ® 2
(b) f (x) exists for all x Î (0, ¥) and f ¢ is continuous on è | a |2 ø |a |
(0, ¥) but not differentiable on (0, ¥)

PREP CATALYSIS
JEE Advanced~Practice Set 5 153

42. If the tangent drawn at point ( t 2 , 2t ) on the 50. The number of positive integral solution of the
parabola y = 4x is same as the normal drawn
2 y3 + 1 y 2z y 2x
at point ( 5 cos q, 2 sin q) on the ellipse equation yz 2 z3 + 1 z 2x = 11
4x 2 + 5 y 2 = 20. Then, which of the following is yx 2
x 2z x3 + 1
(are) TRUE?
-1
(a) q = cos-1 æç ö÷ (b) q = cos-1 æç ö÷ Section 3 (Maximum Marks : 12)
1
è 5ø è 5ø
Instructions: Same as given in Physics.
(c) t = - 2 / 5 (d) t = - 1/ 5
Paragraph X
Section 2 (Maximum Marks : 24) The probability that a family has exactly n children is
Instructions: Same as given in Physics. ap n , n ³ 1. All sex distribution of n children in a
family have the same probability.
43. Positive number x , y and z satisfy xyz = 1081
and (log10 x ) (log10 yz ) + (log10 y ) (log10 z ) = 468, 51. The probability that a family contains exactly k
then the value of boys is (where, k ³ 1)
(log10 x ) + (log10 y ) + (log10 z ) is equal to
2 2 2 (a) apk (1 - p)-k - 1
(b) 2 apk (2 - p)-k - 1
44. Consider the set of eight vectors (c) 2apk (2 - p)-k
V = { a $i + b$j + ck$ ; a , b, c Î { - 1, 1}}. Three (d) 2 apk - 1 (2 - p)-k - 1
non-coplanar vectors can be chosen from V is l 52. The probability that a family includes at least
ways, then l is equals to one boy is
45. The value of limit a2 p ap 2
(a) (b)
11( k - 9)
(2 - p) (1 - p) (2 - p) (1 - p)
æ ö ap 2 ap
çn + 9 2 n n -1
58 ÷ (c) (d)
lim ç S - S ÷ is (2 - p) (1 - p) (2 - p) (1 - p)
n ® ¥ k = 10 log e n / 11 k = 0 p ( n - k) ( n + k)
ç 2 ÷ Paragraph A
è ø
Let f and g be two functions such that f ( x) and g( x)
46. The number of real solution of the equation
are continuous in [a, b] and differentiable in ( a, b).
æ ¥ ( -1)n - 1 x 2n ö -1 æ
¥ ( - 1)n - 1 x 4n ö
sin -1
ç S ÷ + cos ç S ÷ Then, there exists atleast one c Î( a, b) such that
èn = 1 3n - 1 ø èn = 1 3n - 1 ø f (b) - f ( a)
p f ¢( c) =
= , where 0 £ x £ 3, is b-a
2
1. If f ( a) = f (b), then f ¢( c) = 0 (RMVT)
47. Let x, y and z be unit vectors such that 2. If f ( a) ¹ f (b) and a ¹ b (LMVT)
x + y + z = a , x ´ ( y ´ z ) = b, ( x ´ y ) ´ z = c, f (b) - f ( a) f ¢( c)
3 7 3. If g ¢( x) ¹ 0, then = (Cauchy
a ×x = , a ×y = and|a| = 2, if y = - l c. Then, l g(b) - g( a) g ¢( c)
2 4
equals to theorem)
53. Which of the following is true?
48. One side of square ABCD is on the line
(a) | sin x - sin y | ³ | x - y |, "x, y ÎR
y = 2x - 17 and other two vertices are on
parabola y = x 2. If the minimum area of square (b) | tan- 1 x - tan- 1 y | ³ | x - y |, " x, y ÎR
is A, then the value of A is (c) | tan- 1 x - tan- 1 y | £ | x - y |
(d) None of the above
49. A solid cylinder of height 6 m has a conical
1 p sin a - sin b
portion of same height and radius rd of 54. Let 0 < a < q < b < , then is equal to
3 2 cos a - cos b
height removed from it. Rain water is falling in (a) tan q
the cylinder with rate equal to p times the (b) - cot q
instantaneous radius of water surface inside (c) cot q
hole, the time after which hole will fill up with (d) - tan q
water is .....

PREP CATALYSIS
Paper 2
PHYSICS

Section 1 (Maximum Marks : 24)


l This section contains SIX (06) questions.
l Each question has FOUR options for correct answer(s). ONE OR MORE THAN ONE of these four option(s) is (are)
correct options(s).
l For each question, choose the correct options(s) to answer the question.
l Answer to each question will be evaluated according to the following marking scheme:
Full Marks : + 4 If only (all) the correct option(s) is (are) chosen.
Partial Marks : + 3 If all the four options are correct but ONLY three options are chosen.
Partial Marks : + 2 If three or more options are correct but ONLY two options are chosen, both of which are
correct options.
Partial Marks : + 1 If two or more options are correct but ONLY one option is chosen and it is a correct option.
Zero Marks : 0 If none of the options is chosen (i.e. the question is unanswered).
Negative Marks : - 2 In all other cases.
l For example: If first, third and fourth are the ONLY three correct options for a question with second option being an
incorrect option; selecting only all the three correct options will result in + 4marks. Selecting only two of the three correct
options (e.g. the first and fourth options), without selecting any incorrect option (second option in this case), will result in
+2 marks. Selecting only one of the three correct options (either first or third or fourth option), without selecting any
incorrect option (second option in this case), will result in +1marks. Selecting any incorrect option(s) (second option in
this case), with or without selection of any correct option(s) will result in -2 marks.

1. A block is connected to a spring in an elevator. (a) The final pressure in the two compartments are
Elongation in spring is x1 = 4 2 mm and equal
3V
x2 = 3 2 mm, when it moves up and down with (b) Volume of compartment I is
5
constant acceleration, respectively. Then, 12 V
g (c) Volume of compartment II is
(a) acceleration of elevator is m/ s2 5
7 5p
g (d) Final pressure in compartment I is
(b) acceleration of elevator is m/ s2
3 3
(c) if same elevator moves horizontally with same 3. Radiations of monochromatic waves of
acceleration, then elongation will be 5 mm
wavelength 400 nm are made incident on the
(d) if same elevator moves horizontally with same
surface of metals Zn, Fe and Ni of work
acceleration, then elongation will be 10 mm
functions 3.4 eV, 4.8 eV and 5.9 eV,
2. A partition divides a container having respectively. Which of the following is (are)
insulated walls into two compartments I and correct?
II. The same gas fills the two compartments,
(a) Maximum KE associated with photoelectrons from
whose initial parameters are given. The
the surface of any metal is 0.3 eV
partition is a conduction wall, which can move
freely without friction. Which of the following (b) No photoelectrons are emitted from the surface
statements is/are correct with reference to the of Ni
final equilibrium position? (c) If the wavelength of source radiation is doubled,
then KE of photoelectrons is also doubled
(d) Photoelectrons will be emitted from the surface of
p, V, T 2p, 2V, T
all the three metals, if the wavelength of incident
I II
radiations < 200 nm

PREP CATALYSIS
JEE Advanced~Practice Set 5 155

4. A semicircular wire of O P 40 20
(a) m (b) m
v 3 13
radius r is supported in its
own vertical plane by a θ 13 13
(c) m (d) m
hinge at O and smooth peg 20 40
P as shown in figure. If
peg starts from O and 6. A small hole of area a is at the bottom of a
moves with constant speed v along horizontal container of area A. The liquid is filled up to
axis through O. The angular speed of wire is height h from base. As liquid comes out, then
é Aù
(a)
v
(b)
v êë a = 3 úû .
4r - v t
2 2 2 2r
v
(c) (d) None of these
2r 2 - v 2 t
h
5. A ball is projected from point A with a velocity
10 m/s perpendicular to the plane as given.
The range of the ball on plane is
u = 10 m/s

gh
A (a) level of liquid in container falls at rate of
2
(b) acceleration of top surface of liquid is g / 4 m/ s2
30°
(c) acceleration of top surface of liquid is g/3
(d) None of the above

Section 2 (Maximum Marks : 24)


l
This section contains EIGHT (08) questions. The answer to each question is a NUMERICAL VALUE.
l
Four each question, enter the correct numerical value (in decimal notation, truncated/rounded-off to the second
decimal place; e.g. 6.25, 7.00.- 0.33, -.30, 30.27, -127.30) using the mouse and the on-screen virtual numeric
keypad in the place designated to enter the answer.
l
Answer to each question will be evaluated according to the following marking scheme:
Full Marks : + 3 If ONLY the correct numerical value is entered as answer.
Zero Marks : 0 In all other cases.

7. A binary star consists of two stars A (mass 10. Degree of freedom for a gas is 6. This gas does
2.2 M S ) and B (mass 11 M S ), where M S is the 25 J of piston work at constant pressure. Net
mass of the sun. They are separated by heat absorbed by the gas during this process
distance d and are rotating about their centre is ……… .
of mass, which is stationary. The ratio of the
11. A thin plano-convex lens of focal length f is
total angular momentum of the binary star to
split into two halves. One of the halves is
the angular momentum of star B about the
shifted along the optical axis. The separation
centre of mass is
between object and image planes is 1.8 m. The
8. A hydrogen atom is in n = 5 state makes a magnification of the image formed by one of
transition to it’s ground state. Assuming, H-atom the half lens is 2. Find the separation between
is initially at rest, find the recoil speed of H-atom. the halves.
9. A 5 m long cylindrical steel wire with radius
2 ´ 10-3 m is suspended vertically from a rigid
support and carries a bob of mass 100 kg at O
the other end. Calculate the energy stored in
the wire upto two decimal places. (For the steel
wire : Young’s modulus = 2.1 ´ 1011 Pa, density
1.8 m
= 7860 kg-m3 .

PREP CATALYSIS
156 JEE Advanced~Practice Set 5

12. A cat can run at a speed of 5 ms-1 and is of side a. The surface tension of the soap film
heading towards a mouse running at a is g. The system of charges and planar film are
constant speed of 3 ms-1. 1/ N
é q2 ù
Mouse 3 ms–1 in equilibrium and a = k ê ú , where k is a
ë g û
constant. Then, N is ……… .
16 m
14. A beam of light has three wavelengths 4144 Å,
5 ms–1 4972 Å and 6216 Å with a total intensity of
3.6 ´ 10-3 Wm-2 equally distributed amongst the
During persuasion cat and mouse are moving three wavelengths. The beam falls normally on
perpendicularly at t = 0. If initial distance an area 1.0 cm2 of a clean metallic surface of
between them is 16 m, find the time taken by work function 2.3 eV. Assume that there is no
cat to catch the mouse. Assume that cat is loss of light by reflection and that each
always headed towards the mouse. energetically capable photon ejects one electron.
13. Four point charges, each of + q are rigidly fixed If the number of photoelectrons liberated in two
at the four corners of a square planar soap film seconds is N ´ 1012, then find N .

Section 3 (Maximum Marks : 12)


l This section contains FOUR (04) questions.
l Each question has TWO (02) matching lists : Column-I and Column-II.
l FOUR options are given representing matching of elements from Column-I and Column-II. ONLY ONE of these four
options corresponds to a correct matching.
l For each question, choose the option corresponding to the correct matching.
l
For each question, marks will be awarded according to the following marking scheme.
Full Marks : + 3 If ONLY the option corresponding to the correct matching is chosen.
Zero Marks : 0 If none of the options is chosen (i.e. the question is unanswered).
Negative Marks : - 1 In all other cases.

15. Match physical constants from Column I with (a) A-p, B-s, C-r, D-q (b) A-p, B-q, C-r,D-s
their dimensions in Column II. (c) A-s, B-q, C-p, D-r (d) A-s, B-r, C-q, D-p
17. Column I gives a list of possible set of
Column I Column II
parameters measured in some experiments.
A. Boltzmann’s constant p. [ML2 T -1] The variations of the parameters in the form of
B. Coefficient of viscosity q. [ML-1T -1] graphs are shown in Column II. Match the set
of parameters given in Column I with the
C. Planck’s constant r. [MLT -3K -1]
graphs given in Column II.
D. Thermal conductivity s. [ML2 T -2K -1]
Column I Column II
(a) A-p, B-q, C-r, D-s (b) A-s, B-p, C-q, D-r
(c) A-s, B-q, C-p, D-r (d) A-s, B-q, C-r, D-p A. Potential energy of a p. Y

simple pendulum
16. In the following, Column I lists some physical
(Y-axis) as a function of
quantities and the Column II gives approximate
energy values associated with some of them. displacement (X-axis).
Choose the appropriate value of energy from O X
Column II for each of the physical quantities in
Column I and write the corresponding letters A, B. Displacement (Y-axis) q. Y
B, C etc. Against the number (i), (ii) and (iii), etc. as a function of time
of the physical quantity. (X-axis) for a one
dimensional motion at
Column I Column II zero or constant
A. Energy of thermal neutrons p. 0.025 eV acceleration, when the
O X
B. Energy of X-ray q. 3 eV body is moving along
C. Binding energy per nucleon r. 8 MeV the positive X-direction.
D. Photoelectric threshold of a metal s. 10 keV

PREP CATALYSIS
JEE Advanced~Practice Set 5 157

C. Range of a projectile r r. Y B. The object moves on the q. The object


(Y-axis) as a function of X-axis in such a way that its does not
its velocity (X-axis) when velocity and its displacement change its
projected at a fixed from the origin satisfy direction.
angle. v = - kx, where k is a positive
O X constant.

D. The square of the s. C. The object is attached to one r. The kinetic


Y
time(s) period (Y-axis) of end of a massless spring of energy of the
a simple pendulum as a a given spring constant. The object keeps
function of its length other end of the spring is on
(X-axis). attached to the ceiling of an decreasing.
elevator. Initially everything
O X
is at rest. The elevator starts
going upwards with a
(a) A-p s B-q r s, C-s, D-p constant acceleration a. The
(b) A-p s, B-q r s, C-s, D-q motion of the object is
(c) A-p, B-q, C-r, D-s observed from the elevator
(d) A-p q, B-r s, C-p s, D-q s during the period it
maintains this acceleration.
`18.`Column I describes some situations in which a
small object moves. Column II describes some D. The object is projected from s. The object
characteristics of these motions. Match the the earth’s surface vertically can change
situations in Column I with the upwards with a speed its direction
characteristics in Column II. GMe only once.
2 , where Me is the
Re
Column I Column II
mass of the earth and Re is
A. The object moves on the p. The object
X-axis under a conservative executes a the radius of the earth.
force in such a way that its simple Neglect forces from objects
speed and position satisfy harmonic other than the earth.
v = c1 c2 - x 2 , where c1 and motion.
(a) A-q, B-p, C-r, D-s (b) A-r, B-q, C-p, D-s
c2 are positive constants. (c) A-p, B-q r, C-p, D-q r (d) A-p, B-q, C-r, D-s

CHEMISTRY
Section 1 (Maximum Marks : 24) (a) CuSO4 (b) CuI
(c) CuO (d) CuCO3
Instructions: Same as given in Physics.
21. Benzyl chloride reacts with KCN in presence of
19. Which of the following statement is correct for DMF to give A. (A) further reacts in the
[Fe(H 2O)5 NO]SO4 complex? following manner:
(a) Fe is in +1oxidation state –+
C 2H5 ON a/ C 2H5 OH H3 O+ SOCl 2
(b) The complex is paramagnetic ( A) ¾¾¾¾¾¾®( B) ¾¾®(C ) ¾¾¾®( D )
(c) In this complex, Fe has three unpaired electrons C 6 H5 CHO/ D D CH3 NH 2
(d) NO is a weak field ligand The major product (D) is
O NH CH3
20. An aqueous blue coloured solution of a O
transition metal sulphate reacts with H 2S in Ph
acidic medium to give a black precipitate ‘A’
(a) NHCH3 (b)
which is insoluble in warm aqueous solution of
KOH. The blue solution on treatment with KI
in weakly acidic medium, turns yellow and
produces a white precipitate B. Here, Ph Ph
compound ‘B’ is

PREP CATALYSIS
158 JEE Advanced~Practice Set 5

O O K p = 2.00 atm -1? (Assume ideal gas behaviour


NH CH3
of all gases.)
(c) NHCH3 (d) (a) pA (eq) < 100
. atm
Ph
(b) 1.00 atm £ pA(eq) < 150
. atm
(c) 1.50 atm £ pA(eq) < 2.00 atm
Ph (d) 2.00 atm £ pA(eq) < 2.50 atm

22. Mycarose is a rare C7H14O4 sugar found in


Section 2 (Maximum Marks : 24)
some antibiotic natural products. Mycarose
gives a positive Tollen’s test, does not form an Instructions: Same as given in Physics.
osazone derivative and on exhaustive HIO4 25. Among the following, the number of compounds
oxidation gives HCO2H, CH3CHO and that can react with PCl5 to give POCl3 is .......
CH3COCH 2CHO. Which of the following
O2 , CO2 , SO2 , H 2O, H 2SO4 , P4O10
structures might be mycarose?
26. An acidic solution of Cu2+ salt is electrolysed
CH3
HO untill all the Cu is deposited. The electrolysis is
(a) continued for 8 more minutes with the volume
O OH
OH of solution kept at 100 mL and the current at
H3 C 1.4 ampere. The volume of gases evolved at
H NTP during entire electrolysis will be .......
OH (Given, mass of Cu2+ ion = 0.8 g , atomic mass
CH3 of Cu = 63.5)
HO
O 27. In the Lindemann theory of unimolecular
(b) reactions, it is shown that apparent rate constant
OH kC
CH3 for such a reaction is kapp = 1 ,
OH 1 + aC
OH
O where, C is the concentration of the reactant k1 and
(c)
a are constants. Calculate the value of C for which
CH3 kapp has 90% of its limiting value at C tending to
H3C infinetly large values, given a = 9 ´ 105 . If answer
OH is x then (log x) equals to..…… .
CH3 O
HO
28. Total number of chiral centres in testosterone
CH3 is ..…… .
(d)
HO OH
OH O

23. A dilute solution contains contains ‘m’ moles of O OH

solute B in 1kg solvent (molal elevation H

constant is K b). The solute B undergoes


dimerisation as 2B 5 B2. The equilibrium H H
constant ( K C ) for dimerisation is O
Kb[Kb ´ m - DTb] Kb[DTb - Kbm] 29. A human patient suffering from a duodenal ulcer
(a) KC = (b) KC =
[ 2 DTb - Kb ´ m] 2 [ 2 DTb - Kb ´ m] 2 may show a hydrochloric acid concentration of
Kb[Kbm - DTb] [Kbm - DTb ] 0.80 mol/L in his gastric juice. It is possible to
(c) KC = (d) KC = neutralise this acid with aluminium hydroxide,
[DTb - Kb ´ m] 2 [ 2 DTb - Kb ´ m] 2 Al(OH)3 , which reacts with HCl according to the
24. 2.00 moles of compound A (g) and 1.00 mole of chemical reaction shown below:
compound B (g) are introduced in a 18.0 L Al(OH)3 + HCl ¾® AlCl3 + H 2O
container. Compound C (g) is formed according
If the patient’s stomach receives 3.0 L of
to A ( g) + B ( g) C( g). What will be the
- gastric juice per day, how much aluminium
equilibrium partial pressure of A(g) at 25°C hydroxide must be consume per day to
knowing that at this temperature counteract the acid?

PREP CATALYSIS
JEE Advanced~Practice Set 5 159

30. Pure O2 diffuses through an aperture in List I List II


224 seconds, whereas mixture of O2 and P. 2NH3 + CO2 1. Ethyl acrylate,
another gas containing 80% O2 diffuses from 130 -150 °C 210°C
¾¾¾®
the same in 234 seconds. The molar mass of 35 atm pressure
gas is ………
Q. D 2. Br2 , AcOH
NH4NCS ¾¾¾®
31. Consider the following reaction, 170 °C

[Ag(CN)2 ]- - Ag+ + 2CN - , the K C at 25°C is R. a -bromosuccinic acid 3. Boil in C5H5N


4 ´ 10 . The concentration of Ag+ is x ´ 10-18
-19 Ag2 O
¾¾®
M in solution which was originally 0.1 M in 4. Sodioformyl
KCN - and 0.03 M in AgNO3 . The value of x is acetic ester
……… . 5. aq. ClCH2 COOH,
32. 1.5 L He gas is taken at 500 K and 560 mm boil
pressure in which 90% of He is converted into 6. H2 SO4
He+ ions. Spectral line analysis shows that
7. NH2 CONH2
85% ions exist in the 3rd level, 10% in the 2nd
level and rest in the ground level. The total The correct option is
energy evolved when all the ions return back (a) P ® 1,2,3 ; Q ® 4,5 ; R ® 6,7
to ground state is ……… . (b) P ® 1,4 ; Q ® 2,3 ; R ® 5,6
(c) P ® 1,2,4 ; Q ® 1,3 ; R ® 5,6
Section 3 (Maximum Marks : 12) (d) P ® 1,2 ; Q ® 3,4 ; R ® 5,6,7
Instructions: Same as given in Physics.
35. List I contains reactant and reagent and List II
33. Match the following metals given in List I with contains product.
extracted processes given in List II. List I List II
List I List II I CH3 ¾ CH2 ¾ Cºº CH p. CH3 ¾ C == C ¾ CH3
A. Silver p. Fused salt electrolysis NBS + CCl 4 +hn
¾¾¾¾® ½ ½
H OCH3
B. Calcium q. Carbon reduction
C. Zinc r. Carbon monoxide reduction II CH3 ¾ C ºº C ¾H q. CH3 ¾ C ºº C ¾ CH3
Sia 2BH
D. Iron s. Amalgamation ¾¾¾¾®
E. Copper t. Self-reduction
III Potassium succinate r. CH3 ¾ C == C ¾H
The correct option is Electrolysis ½ ½
¾¾¾¾¾® H H
(a) A ® s ; B ® p ; C ® q ; D ® q,r ; E ® t
IV CH3 ¾ C ºº C ¾ CH3 s. CH3 ¾ CH ¾ C ºº C ¾H
(b) A ® t ; B ® q ; C ® p ; D ® r,s ; E ® s _
CH 3 O ½
(c) A ® p ; B ® q ; C ® r ; D ® s ; E ® t ¾¾¾¾® Br
(d) A ® p ; B ® q,r ; C ® r ; D ® q,s ; E ® t
The correct option is
34. The desired product ‘X’ can be prepared by
(a) I ® s ; II ® q ; III ® r ; IV ® p
reacting the major product of the reactions in
(b) I ® p ; II ® q ; III ® r ; IV ® s
List I with one or more appropriate reagents in
(c) I ® s ; II ® s ; III ® q ; IV ® p
list II.
(d) I ® r ; II ® p ; III ® q ; IV ® s
O
36. The standard potential of the following cell is
HN 0.23 V at 288 K and 0.21 V at 308 K.
°
EAg +
/ Ag
= 0.80 at 298 K
O N
H
Pt(H 2 )|HCl( aq )||AgCl (s) |Ag (s)
(X)

PREP CATALYSIS
160 JEE Advanced~Practice Set 5

Match the parameters in Column I with their (iv) Solubility product of AgCl in s. -10
.
values in Column II and select the answer water at 298 K in terms of (log).
from the codes given below:
t. –96.5
Column I Column II
Codes
(i) Temperature coefficient of emf of p. -9.81
the cell ( in mv) K -1
(i) (ii) (iii) (iv)
(a) p q r s
(ii) Entropy change (in JK -1) in the q. -80.2
temperature range 288 K and (b) q s p r
308 K. (c) s t r p
(iii) Enthalpy change (in kJ mol -1) in r. -50.0 (d) s p r t
the temperature range 288 K
and 308 K.

MATHEMATICS
-3e -6 ö
(c) If P is æç 6, ö÷, then Q is æç
3
Section 1 (Maximum Marks : 24) è 2ø
, ÷, where e is
è 2 e ø
Instructions: Same as given in Physics. eccentricity of the given ellipse
(d) If O is origin, then product of slopes of OP andOQ
37. If f ( x ), g( x ) are thrice differentiable function on is positive.
[0, 3] satisfying
æ1ö æ1ö 40. If the angles of a triangle are 30° and 45°, and
f ¢ ¢ ¢ ( x ) = g¢ ¢ ¢ ( x ), f ¢ ¢ ç ÷ = g¢ ¢ ç ÷ = 0, the included side is ( 3 + 1) cm, then which of
è 2ø è 2ø
f ¢ (1) = 4, g¢ (1) = 2 and f ( 3) = 12, g( 3) = 3, then the following is(are) correct?
which of the following is (are) TRUE? 3 +1
(a) The area of triangle is sq unit
(a) f (2) - g (2) = 7 2
(b) | f (x) - g (x)| < 5 Þ x Î] - 4, 1[ 3 +1
(b) Ratio of greater side of smaller side is
(c) | f (x) - g (x)| > 5 Þ x Î] - ¥, - 4 [È] 1, ¥ [ 2
(d) f (x) - g (x) = 2 x - 3 (c) The area of triangle is 2 - 3
3 +1
38. Let P ( x ) and Q ( x ) be two polynomials. Suppose (d) Ratio of greatest side to smaller side is
3 -1
that f ( x ) = P ( x3 ) + x Q ( x3 ) is divisible by
x 2 + x + 1, then which of the following is(are) 41. Let z1 and z 2 be two distinct complex number
TRUE? and let z = (1 - t )z1 + tz 2 for some real number t
with 0 < t < 1, if arg ( w) denotes the principal
(a) P (x) is divisible by (x - 1) but Q (x) is not divisible
by x - 1
argument of a non-zero complex number w,
(b) Q (x) is divisible by (x - 1) but P (x) is not divisible then which of the following is (are) FALSE?
by x - 1 (a) | z - z1| + | z - z 2 | = | z1 - z 2 |
(c) Both P (x) and Q (x) are divisible by x - 1 (b) (z - z1) = (z - z 2 )
(d) f (x) is divisible by x - 1 z - z1 z - z1
(c) =0
39. The normal at one extremity of latusrectum (in z 2 - z1 z 2 - z1
x2 y2 (d) arg (z - z1) = arg (z 2 - z1)
first quadrant) of the ellipse =1 2
+
a b2 42. If 4a 2 - 5b2 + 6a + 1 = 0 and the line
a > b > 0 meets the rectangular hyperbola
ax + by + 1 = 0 touches a fixed circle, then
xy = 9 at points P and Q, then which of the
which of the following is (are) TRUE?
following is(are) TRUE?
(a) Centre of circle is at (3, 0)
æ-3 2 ö
(a) If P is æç 6, ö÷ then, Q is ç
3
, - 3 2÷ (b) The radius of circle is 5
è 2ø è 2 ø (c) The radius of circle is 3
(b) Eccentricity of hyperbola is 2 (d) The circle passes through (1, 1)

PREP CATALYSIS
JEE Advanced~Practice Set 5 161

Section 2 (Maximum Marks : 24) Section 3 (Maximum Marks : 12)


Instructions: Same as given in Physics. Instructions: Same as given in Physics.

43. For each positive integer n, 51. Let a function f ( x ) = [x ] { x } -|x|, where [×], {×}
1/ n are greatest integer and fractional part
Let yn = n 2 ò-1/ n ( 2018 sin x + 2019 cos x )|x| dx respectively.
where,|x| is modulus function of x. If List I List II
lim yn = k, then the value of k is
n ®¥ P. f (x) is continuous at x equal to 1. 0
44. Let the equation of the plane containing line
x - y - z - 4 = 0 = x + y + 2z - 4 and parallel to Q. 4 3 2. 1
the line of intersection of the planes 3 ò2 f (x) dx is equal to

2x + 3 y + z = 1 and x + 3 y + 2z = 2 be R. If g (x) = x - 1and if f (x) = g (x), where 3. 2


x + Ay + Bz + C = 0, then the value of x Î (- 3, ¥), then number of solutions is
| A + B + C| is equal to ..... equal to
45. If the differential equation of a curve, passing S. If L = lim f (x), the | L | is equal to 4. 3
æ pö x ® 4+
through ç 0, - ÷ and ( t , 0) is
è 4ø 5. 4
æ dy -x ö æ dy ö e-x
cos y ç + e ÷ + sin y ç e- x - ÷ = e , then 6. 5
è dx ø è dx ø
-t
the value of tee is equal to The correct option is
(a) P ® 3; Q ® 2; R ® 4; S ® 5
é 2 1ù é 3 4ù
46. Let three matrices A = ê (b) P ® 2; Q ® 3; R ® 4; S ® 5
ú, B = ê 2 3ú and
ë 4 1û ë û (c) P ® 1; Q ® 3; R ® 5; S ® 6
é 3 - 4ù (d) P ® 2; Q ® 3; R ® 5; S ® 6
C=ê ú , then
ë- 2 3 û 52. If y = x + 1 is axis of parabola, y + x = 4 is a
æ ABC ö æ A( BC )2 ö æ A( BC )3 ö tangent of some parabola at its vertex and
tr ( A) + tr ç ÷ + tr ç ÷ + tr ç ÷
è 2 ø è 4 ø è 8 ø y = 2x + 3 is one of its tangent.
é A ( BC )4 ù
+ tr ê ú + ... ¥ is equal to
List I List II
ë 16 û P. If equation of directrix of parabola 1. 9
47. If sin q + cos q = 1, then the minimum value of ax + by - 29 = 0, then a + b =

(1 + cosec q ) (1 + sec q ) is equal to .... Q. If length of latusrectum of parabola is 2. 18


a 2
, where a and b are relatively
48. ABCD is a convex quadrilateral and 3 4, 5 and b
6 points are marked on the sides AB, BC , CD prime natural number, then a + b =
and DA, respectively, then the number of
R. Let extremities of latusrectum are 3. 23
triangle with vertices on different side is equal
(a1, b1) and (a2 , b2 ), then
to ......
[a1 + b1 + a2 + b2 ] = (where, [.]
49. If the curve given by parametric equation denotes greatest integer functions)
x = t - t3 , y = t - t 4 forms a loop for all values of S. 4.
If equation of parabola is 37
m a (x - y + 1)2 = b (x + y - 4)2 , where a
t Î [- 1, 1]. If the area of loop is , (where m
n and b are coprime natural numbers,
and n are relatively prime), then the value of then a + b =
m + n is equal to ....
The correct option is
1008 2018 !
50. If S = a , 2b, where a (a) P ® 3; Q ® 1; R ® 4; S ® 2
n = 1 ( 2018 - 2n )!( 2n - 2)! (b) P ® 2; Q ® 3; R ® 4; S ® 1
and b are positive integer such that gcd (c) P ® 2; Q ® 3; R ® 1; S ® 4
( a , 2) = 1, then the value of b is equal to ..... . (d) P ® 3; Q ® 2; R ® 1; S ® 4

PREP CATALYSIS
162 JEE Advanced~Practice Set 5

1 1 1 1 The correct option is


53. Let f ( n ) = 1 + + + + .... + , such that
2 3 4 n (a) P ® 4; Q ® 1; R ® 6; S ® 5
P ( n ) f ( n + 2) = P ( n ) f ( n ) + q( n ), where P ( n ), (b) P ® 4; Q ® 3; R ® 6; S ® 2
Q( n ) are polynomials to least possible degree (c) P ® 1; Q ® 4; R ® 6; S ® 5
(d) P ® 1; Q ® 4; R ® 3; S ® 2
and P ( n ) has leading coefficient is unity.
lnx
54. Consider the function f ( x ) = - ax + x 2 and
List I List II 8
a ³ 0 is a real number.
P. mp (n) - 2 1. m (m + 1)
S is equal to List I List II
n =1 n 2 P. f (x) gives a local 1. a = 1; x = 1 / 4
m q (n) - 3 3m (m + 1) maxima at
Q. S is equal to 2.
n =1 2 2 Q. f (x) gives a local 2. a - a2 - 1
minima at a > 1;x =
4
R. mp (n) + q 2 (n) - 11 3. 5m (m + 1)
S is equal to R. f (x) gives a point of 3. 0£a <1
n =1 n 2
inflection for
S. m q (n) - p (n) - 7
2
4. m (m + 7) S. f (x) is strictly 4. a+ a2 - 1
S is equal to increasing for all a > 1; x =
n =1 n 2 4
x ÎR +
5. 3m (m + 7)
The correct option is
2
(a) P ® 3; Q ® 1; R ® 4; S ® 2
6. 5m (m + 7) (b) P ® 2; Q ® 4; R ® 3; S ® 1
2 (c) P ® 4; Q ® 2; R ® 1; S ® 3
(d) P ® 2; Q ® 4; R ® 1; S ® 3

Answers
Paper 1
1. b, c, d 2. b, c 3. a, d 4. a, d 5. c, d 6. b 7. 0.00 8. 4.00 9. 1.00 10. 3.00
11. 4.00 12. 8.00 13. 2.82 14. 2.95 15. b 16. a 17. d 18. d 19. a, b, c 20. a, b
21. a, c, d 22. c 23. a, b, c 24. b 25. 4 26. 0.49 27. 150 28. –5.85 29. 196.20 30. 5
31. 3.06 32. 0.36 33. c 34. c 35. b 36. b 37. a,b,c,d 38. b, c 39. a, b, d 40. a, d
41. b, c, d 42. a, d 43. 75 44. 32 45. 2018 46. 3 47. 4 48. 80 49. 6 50. 3
51. b 52. c 53. c 54. b

Paper 2
1. a, c 2. a,b,c,d 3. b, d 4. a 5. a 6. a, b 7. 6.00 8. 4.20 9. 0.95 10. 75.00
11. 0.60 12. 5.00 13. 3.00 14. 1.10 15. c 16. a 17. b 18. c 19. a, b, c 20. b
21. b 22. a 23. a 24. c 25. 4 26. 258.05 27. 5 28. 6 29. 6.24 30. 46.60
31. 7.50 32. 106 33. a 34. a 35. c 36. c 37. a, b, c 38. c, d 39. b, c, d 40. a, b
41. b 42. a, b, d 43. 2019 44. 11 45. 1 46. 6 47. 9 48. 342 49. 51 50. 1
51. b 52. c 53. a 54. d

SCORE SHEET - Paper 1


Section No. of Marks from Marks from Marks Obtained
Correct Questions Correct Questions (A) Incorrect Questions (B) (A-B)
................ ............................... ............................... ............................... ...............................
................ ............................... ............................... ............................... ...............................
................ ............................... ............................... ............................... ...............................
Percentage Marks = Marks Obtain/Total Marks x 100

SCORE SHEET - Paper 2


Section No. of Marks from Marks from Marks Obtained
Correct Questions Correct Questions (A) Incorrect Questions (B) (A-B)
................ ............................... ............................... ............................... ...............................
................ ............................... ............................... ............................... ...............................
................ ............................... ............................... ............................... ...............................
Percentage Marks = Marks Obtain/Total Marks x 100
Note To expect your success marks in the test should be between 65%-70%.
PREP CATALYSIS
JEE Advanced

PRACTICE SET 6 (With Solutions)

Duration : 3 Hours Max. Marks . 360

Paper 1
PHYSICS

Section 1 (Maximum Marks : 28)


l
This section contains SEVEN questions.
l
Each question has FOUR options (a), (b), (c) and (d). ONE OR MORE THAN ONE of these four options is (are) correct.
l
For each question, darken the bubble(s) corresponding to all the correct option(s) in the ORS.
l
For each question, marks will be awarded in one of the following categories:
Full Marks : + 4 If only the bubble(s) corresponding to all the correct option(s)
is (are) darkened.
Partial Marks : + 1 For darkening a bubble corresponding to each correct option, providedNO
incorrect option is darkened
Zero Marks : 0 If none of the bubbles is darkened.
Negative Marks : - 2 In all other cases.
l
For example, if [a], [c] and [d] are all the correct options for a question, darkening all these three will get +4 marks;
darkening only [a] and [d] will get +2 marks; and darkening [a] and [b] will get-2 marks, as a wrong option is also
darkened

1. The viscosity h of a gas depends on mass of its 2. A ray of light travelling in a transparent
molecules, their effective diameter and their medium falls on a surface separating the
mean speeds. medium from air at an angle of incidence 45°.
Now, choose the correct options. The ray undergoes total internal reflection. If n
is the refractive index of the medium with
[Given, at 27°C, h (methane) = 2 ´ 10-5 , respect to air, select the possible value (s) of n
SI units, h (helium) = 11 . ´ 10-5 , SI units, from the following
. ´ 10-10 m]
diameter of helium atom is 21 (a) 1.3 (b) 1.4 (c) 1.5 (d) 1.6
kmv
(a) h = 3. A composite block is Heat 0 1L 5L 6L
d2 made of slabs B 3K
1L A E
mv 2 A, B, C , D and E of
(b) h= k
d different thermal 2K C 4K 6K
(c) diameter of methane molecule is nearly thrice that conductivities (given 3L
of helium molecule in terms of a constant D 5K
4L
(d) diameter of methane molecule is nearly 4 ´ 10-10 m K ) and sizes (given in
terms of length L) as shown in the figure.

PREP CATALYSIS
164 JEE Advanced~Practice Set 1

All slabs are of same width. Heat Q flows only S 2 are illuminated with coherent microwave
from left to right through the blocks. Then, in sources, each of frequency 106 Hz. The sources
steady state are synchronised to have zero phase
(a) heat flow through A and E slabs are same difference. The slits are separated by a
(b) heat flow through slab E is maximum distance, d = 150.0 m. The intensity I (q ) is
(c) temperature difference across slab E is smallest measured as a function of q, where q is defined
(d) heat flow through C = heat flow through B + heat flow as shown in figure. If I 0 is the maximum
through D intensity, then I(q ) for 0 £ q £ 90° is given by
4. A student is taking a measurement using a
screw gauge. He found that in 10 complete S1
rotations of vernier scale, the distance moved d /2
over linear scale is 1 cm. There are 100 θ
divisions on circular scale. When screw gauge
is closed fully, the 95th division of circular d /2
scale meets with the reference line while S2
measuring diameter of a wire, linear scale
reads 2 mm and 45th division on circular scale (a) I (q) = I 0 / 2 for q = 30° (b) I (q) = I 0 / 4 for q = 90°
coincides with the reference line. (c) I (q) = I 0 for q = 0° (d) I (q) is constant
Now, choose the correct options.
(a) Error in instrument is (- 0.05 mm) 7. A particle is moving over a curved path with
$ ( ms-1 ) and acceleration,
velocity, 2$i + 2$j + k
(b) Error in instrument is + 0.95 mm
(c) Diameter of wire is 2.50 mm $ (ms-2).
$i + 2$j - 2k
(d) Diameter of wire is 1.50 mm Now, choose the correct options.
5. A microwave oven heats up a food item, 4 -2
(a) Tangential acceleration of particle is ms
because 3
65
(a) frequency of e/m waves used is around 2450 MHz (b) Tangential acceleration of particle is ms-2
(b) quantum energy of photons of microwaves is about 3
4
1 ´ 10-5 eV (c) Normal acceleration of particle is ms-2
(c) water molecules present in food rotates and 3
65
vibrates in resonance with microwaves (d) Normal acceleration of particle is ms-2
(d) only complex food molecules vibrates and rotates 3
in resonance with microwaves
6. In an interference arrangement similar to
Young’s double-slit experiment, the slits S1 and

Section 2 (Maximum Marks : 15)


l
This section contains FIVE questions
l
The answer to each question is a SINGLE DIGIT INTEGER ranging from 0 to 9, both inclusive.
l
For each question, darken the bubble corresponding to the correct integer in the ORS.
l
For each question, marks will be awarded in one of the following categories:
Full Marks : + 3 If only the bubble corresponding to all the correct answer is darkened.
Zero Marks : 0 In all other cases.

8. There is an isolated planet having mass 2M 9. A spring of force constant k = 300 N / m


and radius 2R, where M and R are the mass connects two blocks having masses 2 kg and
and radius of the earth. A simple pendulum 3 kg, lying on a smooth horizontal plane. If the
having mass m and length 2R is made to small spring block system is released from a
oscillations on the planet. If the time period of stretched position, then find the number of
SHM of pendulum in second is 1692 T , then find complete oscillations in 1/4 minute.
T . (Take, p = 3.00, g = 10 m / s2 , 2 = 1.41) (Take, p = 10)

PREP CATALYSIS
JEE Advanced~Practice Set 1 165

10. In the shown figure, a particle 12. A rod AB of length 2m is hinged at point A and
of mass M / 10 strikes the M it’s other and B is attached to a platform on
block of mass M with velocity 10 A which a block of mass m is kept. Rod rotates
v0 and gets attached to it. For M K about point A maintaining angle q = 30° with
what velocity v0 (in ms- 1), the the vertical in such a way that platform
block B just able to leave the B remains horizontal and revolves on the
2M
ground? horizontal circular path. If the coefficient of
(Given, M = 100 gm, K = 880 N / m ) static friction between the block and platform
is m = 01
. , then find the maximum angular
11. In figure, if m = 01
. is the coefficient of friction velocity in rad/s of rod, so that block does not
between the wedge and the horizontal surface, slip on the platform. (Take, g = 10 m /s2)
then for what maximum value of m in kg, the
wedge remains at rest?
(Take, M = 18 kg and q = 45°) B

Smooth m
θ

M A
θ µ

Section 3 (Maximum Marks : 18)


l
This section contains SIX questions of matching type.
l
The section contains TWO tables (each having 3 columns and 4 rows)
l
Based on each table, there are THREE questions.
l
Each question has FOUR options [a], [b], [c] and [d]. ONLY ONE of these four options is correct.
l
For each question, darken the bubble corresponding to the correct option in the ORS.
l
For each question, Marks will be awarded in one of the following categories:
Full Marks : + 3 If only the bubble corresponding to the correct option isdarkened
Zero Marks : 0 If none of the bubbles is darkened
Negative Marks : -1 In all other cases

Direction (Q. Nos. 13-15) An object of circular 14. Which combination belongs to an object, which
cross-section, is allowed to roll down an inclined roll down latest?
plane. In Column I acceleration of COM of object is (a) A II P (b) B I R (c) C IV Q (d) D III S
given, in Column II value of friction force acting on 15. Which combination belongs to an object, which
object is given and in Column III value of coefficient roll down and comes IInd in race?
of friction to prevent slipping is given. (a) A II P (b) B I R (c) C IV Q (d) D III S
(Take, m = mass of object and q = angle of inclination) Direction (Q. Nos. 16-18) In Column I initial (ni )
and final (n f ) stationary states of an electron in
Column I Column II Column III
Hydrogen atom are given. In Column II maximum
A. . g sin q
050 I. 05
. macom P. 05
. tanq possible emitted wavelength is given and in
B. 0.67 g sin q II. macom Q. 0.286 tanq Column III minimum possible wavelength is given.
(where, R = Rydberg constant)
C. . g sin q
071 III. 07
. macom R. 0.33 tanq
Column I Column II Column III
D. . g sin q
053 IV. 0.4 macom S. 0.45 tanq
A. nf = 1 I. 900 P. l = 1
lmax = min
13. Which of the combination is correct for an ni = 2, 3, 4 K ¥ 11R R
object, which roll down earliest? 144
B. nf = 5 II. lmax = Q. l = 25
(a) A II P (b) B I R (c) C IV Q (d) D III S min
ni = 6, 7, 8 K ¥ 7R R

PREP CATALYSIS
166 JEE Advanced~Practice Set 6

C. nf = 3 III. l 400 R. l = 9 17. Radiation which are in Pfund series are


max = min
ni = 4, 5, 6 K ¥ 9R R (a) A IV P (b) B I Q
(c) C II R (d) D III S
D. nf = 4 IV. l 4 S. l = 16
max = min
ni = 5, 6, 7 K ¥ 3R R 18. Radiations which are in Brackett series are
(a) A IV P
16. Radiation which are observed mainly in (b) B I Q
ultraviolet region are (c) C II R
(a) A IV P (b) B I Q (c) C II R (d) D III S (d) D III S

CHEMISTRY
Section 1 (Maximum Marks : 28) CHO CHO
H OH H OH
Instructions: Same as given in Physics.
HO H HO H
19. What is the Ligand-Field Stabilisation Energy (c) A = , B=
HO H H OH
(LFSE) for octahedral ions of the following H OH H OH
configurations?
CH2OH CH2OH
A. d3
CHO CHO
B. High-spin d5 H OH H OH
C. d 9 HO H HO H
(a) A = 1.42D 0 ,B = 0.68D 0 ,C = 0.84D 0 (d) A = H OH
, B=
HO H
(b) A = 1.2D 0 ,B = 0.6D 0 ,C = 0.8D 0
H OH H OH
(c) A = 1.2D 0 ,B = 0, C = 0.6D 0
(d) A = 1.4D 0 ,B = 0, C = 0.88D 0 CH2OH CH2OH
20. Two sugars A and B are known to be glucose
21. Nickel is estimated by reaction of dyglyoxime
and galactose but it is not mentioned on the
with nickel. Which of the following statement (s)
bottles. is/are correct regarding the complex obtained
On treatment with nitric acid, A gives an during the reaction?
optically inactive aldaric acid while B gives an (a) It is a chelating complex
optically active aldaric acid. Which sugar is
(b) It is a paramagnetic complex
glucose and which is galactose?
(c) It is a diamagnetic complex
CHO CHO (d) Magnetic moment of complex is 2.83 BM
HO H H H 22. Suitable reagent(s) for conversion of
HO H HO H 2, 2-dichloropropane to propyne is/are
(a) A = H OH
, B=
HO H Me Cl
H OH H OH CH3  C ≡≡ CH
Me Cl
CH2OH CH2OH (a) aq KOH (b) alc. KOH (c) NaNH2 (d) KNH2
CHO CHO 23. CsBr crystallises in a body centered cubic unit
H OH HO H lattice with an edge length of 4.287 Å.
HO H HO H Calculate the angle(s) at which the second
(b) A = H OH
, B=
H OH
order reflection maxima may be expected for
(2, 0, 0), (1, 1, 0) and (1, 1, 1) planes when
H OH H OH X-rays of l = 0.50 Å are used
CH2OH CH2OH

PREP CATALYSIS
JEE Advanced~Practice Set 6 167

(a) q1 = 13° 29¢, q2 = 9° 60¢, q3 = 19° 49¢ 28. How many of the following polymers are step
(b) q1 = 15° 29¢, q2 = 10° 30¢, q3 = 23° 83 ¢ growth polymer? Buna-S, Protein, Cellulose,
(c) q1 = 23° 49¢, q2 = 10° 30¢, q3 = 13° 49¢ Terylene, Bakelite, nylon-6, PVC, polythene,
(d) q1 = 13° 49¢, q2 = 9° 49¢, q3 = 23° 83 ¢ teflon, malamine, Formaldehyde resin

24. Which of the following is/are correct 29. At 25°C, the inversion of sucrose proceeds
statement(s) regarding boron? with a constant half-life of 500 min at pH = 5
(a) Boron has two isotopes and 50 min at pH = 4. If the rate law for
(b) Boron is the 3rd most abundant element in earth crust
-d
inversion of sucrose is given by [sucrose]
(c) Kernite is an ore of boron dt
+ b
(d) Crystalline boron has atomicity equal to 12 = K [sucrose] ( H ) . Find the value of (b).
a

25. Choose the correct option(s) among the 30. How many number of planes of symmetry
following statement(s). present in antimony pentafluoride?
I I
(a) will react more readily than for SN 2 Section 3 (Maximum Marks : 18)
reaction Instructions: Same as given in Physics.
I
(b) will react more readily than
Cl for Directions (Q. Nos. 31-33) By appropriately
SN1reaction matching the information given in the three columns
of the following tables. Columns, I, II and III contain
reagent or catalyst, molecular formula of reagent or
CH2—Br
(c) will react more readily than catalyst, reactant respectively.

Br Column I Column II Column III


for SN 2 reaction
-
A. Wilkinson’s catalyst I. (i ¾ C3H7 )2 N Li +
P. ketone
Cl I
(d) will react more readily than for E 2 B. Gilman reagent II. C2H5MgCl Q. alkene
reaction
C. LDA III. Me2 CuLi R. aldehyde
Section 2 (Maximum Marks : 15) D. Grignard reagent IV. RhCl[(Ph3P)3 ] S. alkyl halide
Instructions: Same as given in Physics.
31. Which of the following combination gives an
26. The molecule MLx is planar with six pairs of alcohol as a product?
electrons around M in the valence shell. The (a) D II R (b) D III Q (c) A II R (d) C I S
value of x is ……… .
32. Which of the following results in homogenous
27. How many of the following possess both centre catalytic hydrogenation?
of symmetry and Plane of Symmetry (PoS)? (a) A I Q (b) A IV Q
(c) B III R (d) D I Q
Me H Me
(i) (iv)
Me 33. Which of the following combination is used to
Me H remove a less acidic proton?
H
(a) A IV Q (b) B III R
H NO2
(c) D II P (d) C I P
(ii) Cl Cl (v)
Directions (Q.Nos. 34-36) By appropriately matching
the information given in the three columns of the
H following table. Columns I, II and III contain orbitals,
Me D H COOH quantum numbers, radical probability curves.
H (vi)
(iii)
Column I Column II Column II
HOOC
D H Me COOH H A. 1s I. n = 3, l = 1 P. 1.5
H 1.0 M
ψ2

0.5
H 0
COOH 5 10
r(Å)

PREP CATALYSIS
168 JEE Advanced~Practice Set 6

Column I Column II Column II 34. Which of the following combination is correct


for hydrogenation?
B. 2 p II. n = 3, l = 2 Q. 1.5
M (a) A IV R (b) C II Q
1.0

ψ2
0.5 (c) D III P (d) B III S
0 35. Which of the following combination have 2
5 10
r (Å) node system?
8 (a) C II Q
C. 3 p III. n = 2, l = 1 R. M
(b) C I P
4
ψ2

(c) D I R
0 (d) B III S
r(Å)
36. Which of the following combination has
D. 3d IV. n = 1, l = 0 S. 3 M highest energy?
2
1 (a) A IV R
ψ2

0 (b) B II S
2.116 5 10 (c) D I Q
r(Å) (d) D II Q

MATHEMATICS
Section 1 (Maximum Marks : 28) 41. If the two lines represented by
Instructions: Same as given in Physics. x 2(tan2 q + cos2 q ) - 2xy tan q + y 2 sin2 q = 0
make angles a , b with the X-axis, then
37. If x + y + z = 5 and xy + yz + zx = 3 ( x , y , z Î R ),
(a) tan a + tanb = 4 cosec 2 q
then (b) tan a × tanb = sec2 q + tan2 q
(a) maximum value of x, y and z are same (c) tan a - tanb = 2
(b) minimum value of x, y and z are same tan a 2 + sin 2 q
(d) =
(c) maximum value of x = 13 / 3 and minimum value of x = -1 tanb 2 - sin 2 q
(d) Probability for x is positive, is 13 / 16.
42. If f ( 2 - x ) = f ( 2 + x ) and f ( 4 - x ) = f ( 4 + x ) and
38. Let a = x$i + x 2$j + 2k$ , b = - 3$i + $j + k$ , 2
c = ( 3x + 11)$i + ( x - 9)$j - 3k$ be three vectors. f ( x ) is a function for which ò0 f ( x ) dx = 5, then
Then, angle between a and b is acute angle and 50

between c and a is obtuse, if x lies in ò0 f ( x ) dx is equal to


46
(a) (-¥, 1) È (2, 3) (b) (- ¥, 1) (a) 125 (b) ò f (x)dx
-4
(c) (2, 3) (d) None of these 51 52
(c) ò f (x) dx (d) ò f (x) dx
39. For the parabola y = 4ax, a normal chord AB
2 1 2

is drawn at a point A such that AB is shortest, 43. If ( a sec q, b tan q) and ( a sec f , b tan f ) are the
then the circumcentre of DOAB is x2 y2
ends of a focal chord of - = 1, then
(a) (5a, 2a) (b) ( 2a, 5a) a2 b2
(c) (5a, - 2a) (d) (- 2a, 5a) q f
tan tan equals to
40. For the equation x + y + z + w = 19, the number 2 2
e -1
of positive integral solution is equal to (a)
e+1
(a) the number of ways in which 15 identical things can
1- e
be distributed among 4 persons. (b)
(b) the number of ways in which 19 identical things can 1+ e
be distributed among 4 persons 1+ e
(c)
(c) Coefficient of x19 in (x 0 + x1 + x 2 + ...... + x19 )4 1- e
e+1
(d) Coefficient of x19 in (x + x 2 + x 3 + .... + x19 )4 (d)
e -1

PREP CATALYSIS
JEE Advanced~Practice Set 6 169

( xi +1 , 0). Now, again a tangent is drawn at


Section 2 (Maximum Marks : 15) ( xi + 1 , yi + 1 ) on the curve which intersects the
Instructions: Same as given in Physics. X-axis at ( xi + 2 , 0) and the process is repeated n
times, i.e. i = 1, 2, 3, ... n. Let x1 , x2 , x3 , ... , xn
44. If the planes x - cy - bz = 0, cx - y + az = 0 and forms an arithmetic progression with common
bx + ay - z = 0 pass through a line, then the difference equal to log2 e and the curve passes
value of a 2 + b2 + c2 + 2abc is through (0, 2). Now, if curve passes through
p the point ( - 2, k), then the value of k is
45. In DABC, ÐC = and
2
æ ax ö æ bx ö Section 3 (Maximum Marks : 18)
sin-1( x ) = sin-1 ç ÷ + sin-1 ç ÷, where a , b, c
è c ø è c ø Instructions: Same as given in Physics.
are the sides of triangle, then total number of Direction (Q. No. 49, 50 and 51) by appropriately
different values of x is matching the information given in the three columns
ì minimum of { 3t 4 - 8t3 - 6t 2 + 24t ; of the following table.
ï
46. If f ( x ) = í ì 1 ü n (n + 1) ... (n + k)
ïî
maximum of í 3t + sin2 pt + 2; ý k, n Î N, let B ( k , n) = and
î 4 þ k +1
1 £ t £ x } 1 £ x < 2ü Sk (n) = 1 k + 2 k + ... + n k .
ý
2 £ t £ x } 2 £ x £ 4þ Column I contains information about B( 2, n); B( 3, n)
Then, the greatest value of f ( x ) is …… . and B( 4, n)

47. A point P moves in XY -plane in such a way Column II contains information about
that [|x|] + [| y|] = 1, where [×] denotes the S1 (n), S2 (n), S 3 (n) and S4 (n).
greatest integer function. Area of the region Column III contains information about the value of
representing all possible positions of the point æ k + 1ö æ k + 1ö
P is equal to …… . ç ÷ Sk (n) + ç ÷ Sk - 1 (n) + ...
è 1 ø è 1 ø
48. Tangent is drawn at the point ( xi , yi ) on the
æ k + 1ö æ k + 1ö
curve y = f ( x ) which intersects the X-axis at +ç ÷ S1 (n) + ç ÷ S0 (n)
è k ø è k + 1ø
Column I Column II Column III
n
S1 (n) = B (1, n) (n + 1)k - 1
Sk
I. i. P.
B (2, n) = 2
k =1

n n
S2 (n) = B (2, n) + B (1, n)
S k (k + 1) (k + 2) S éêæçèk 1+ 1ö÷ør æk + 1ö k - 1
II. ii. Q.
B (3, n) = + ç k
÷r +
k =1 r =1
ë è 2 ø
æk + 1ö æk + 1ö ù
K+ ç ÷ r .... + ç ÷ú
è k ø èk + 1ø û
n n
S3 (n) = B (3, n) - 3B (2, n) + B (1, n)
S k (k + 1) S [(r + 1)
III. iii. R.
B (2, n) = k
- rk ]
k =1 r =1

n +1
S4 (n) = B (4, n) - 6B (3, n)+ 7B (2, n) - B (1, n) (n + 1)k -1
S k (k + 1) (k + 2) (k + 3)
IV. iv. S.
B (4, n) =
k =1

49. Which of the following options is the only correct combination?


(a) (I) (i) (P) (b) (II) (ii) (Q) (c) (III) (iii) (R) (d) (IV) (iv) (S)
50. Which of the following option is the only correct combination?
(a) (I) (ii) (R) (b) (II) (iii) (S) (c) (III) (iv) (P) (d) (IV) (ii) (R)
51. Which of the following option is the only incorrect combination?
(a) (II) (iii) (Q) (b) (III) (iv) (S) (c) (III) (iv) (Q) (d) (IV) (iii) (R)

PREP CATALYSIS
170 JEE Advanced~Practice Set 6

Directions (Q.Nos. 52, 53 and 54) by the appropriately matching the information given in the three column of
the following table. Column I, II and III contain equation of curves, one of their points of intersection and the
area bounded by them, respectively.

Column I Column II Column III

x 2 = 4ay, y 2 = 4bx (b - a, 2 ab ) tan-1 æç ö÷


I. i. P. 4 a
ab èb ø

II. y 2 = 4a (x + a), y 2 = 4b (b - x) ii. æ 1 1 ö Q. 16


(ab)
ç , ÷
ç a2 + b 2 a2 + b 2 ÷ 3
è ø

III. x2 y2 1 x2 y2 1 iii. (4a 2 / 3b1/ 3 , 4a1/ 3b 2 / 3 ) R. 8a 2


+ = , + =
b2 a2 a 2b 2
a2 b2 a 2b 2 3b 3
IV. y 2 = 4ax and y = bx iv. æ 4a , 4a ö S. 8
ab (a + b)
ç 2 ÷
èb b ø 3

52. For a = 2 and b = 3, the area bounded by the curves is 32 sq units, then which of the following options is
the only CORRECT combination?
(a) (I) (iv) (P) (b) (I) (iii) (Q) (c) (IV) (iv) (R) (d) (II) (i) (S)
27
53. If the area bounded by the curves is sq units and one of their point of intersection is (9, 9), then
2
which of the following options is the only correct combination?
(a) (III) (iii) (P) (b) (IV) (iv) (R) (c) (II) (ii) (S) (d) (I) (iii) (Q)
54. If the points of intersection of the curves with X-axis are ( - 3, 0) and (3, 0), then which of the following
options is the only correct combination?
(a) (I) (iii) (Q) (b) (III) (ii) (S) (c) (IV) (iv) (R) (d) (II) (i) (S)

Paper 2
PHYSICS

Section 1 (Maximum Marks : 21)


l
This section contains SEVEN questions.
l
Each question has FOUR options (a), (b), (c) and (d). ONLY ONE of these four options is correct.
l
For each question, darken the bubble corresponding to the correct option in the ORS.
l
For each question, marks will be awarded in one of the following categories.
Full Marks : + 3 If only the bubble corresponding to the correct option is darkened.
Zero Marks : 0 If none of the bubbles is darkened.
Negative Marks : - 1 In all other cases

2 1
1. A block B is attached to two unstretched M2 M1
springs S1 and S 2 with spring constants k and S2 S1
4 k, respectively. The other ends are attached B
to two supports M1 and M 2 not attached to the 2 1
walls. The springs and supports have M2 x M1
negligible mass. There is no friction anywhere. S2 S1
B

PREP CATALYSIS
JEE Advanced~Practice Set 6 171

The block B is displaced towards wall 1 by a 4. A biconvex lens of focal length 15 cm is in front
small distance x and release(d). The block returns of a plane mirror. The distance between the
and moves a maximum distance y towards lens and the mirror is 10 cm. A small object is
wall 2. Displacements x and y are measured kept at a distance of 30 cm from the lens. The
with respect to the equilibrium position of the final image is
y (a) virtual and at a distance of 16 cm from the mirror.
block B. The ratio is (b) real and at a distance of 16 cm from the mirror.
x
1 1 (c) virtual and at a distance of 20 cm from the mirror.
(a) 4 (b) 2 (c) (d) (d) real and at a distance of 20 cm from the mirror.
2 4
2. Components of a force of 12 N along directions 5. A long insulated copper wire is closely wound
making angles of 45° and 15° with it, on as a spiral of N turns. The spiral has inner
opposite sides of it are radius a and outer radius b. The spiral lies in
the xy-plane and a steady current I flows
Q
through the wire. The z-component of the
magnetic field at the centre of the spiral is
Y
F=12N
15°
I a
45°
P b X
(a) P = 2 N, Q = 10 N
(b) P = 6 N, Q = 11 6 N
(c) P = 4 6 N, Q = 2 (3 2 - 6) N
m0 N I m 0 NI æb + a ö
ln æç ö÷
b
(d) P = 2 (3 2 - 6) N, Q = 4 6 N (a) (b) ln ç ÷
2 (b - a) è a ø 2 (b - a) è b - a ø
3. A light ray travelling in glass medium is m NI m 0 NI æ b + a ö
(c) 0 ln æç ö÷
b
incident on glass-air interface at an angle of (d) ln ç ÷
2b èa ø 2b èb - a ø
incidence q. The reflected ( R ) and transmitted
(T ) intensities, both as function of q, are 6. A particle starts from origin and moves along
plotte(d). The correct sketch is path y = 2x3/ 2. Displacement of particle when it
100% 100% reaches point (7m, y) is
T T 1022 3592 723
(a) m (b) m (c) 285 m (d) m
27 54 11
Intensity

Intensity

(a) (b)
7. Two bodies are projected from the same point
R R with equal speeds in such direction that both
0 θ 90° 0 θ 90° strike the same point on a plane, whose angle
of incitation is ‘a’. If first body is projected at
100% 100% angle ‘q’ with the horizontal, then the ratio of
T T
their time of high is
Intensity

Intensity

(c) cos(q - a) sin(q - a)


(d) (a) (b)
cosb cos q
R R sin(q - a) cos(q - a)
(c) (d)
0 θ 90° 0 θ 90° sinb sin q

PREP CATALYSIS
172 JEE Advanced~Practice Set 6

Section 2 (Maximum Marks : 28)


l This section contains SEVEN questions.
l Each question has FOUR options (a), (b) (c) and (d). ONE OR MORE THAN ONE of these four option(s) is (are)
correct.
l For each question, darken the bubble(s) corresponding to all the correct option(s) in the ORS.
l For each questions, marks will be awarded in one of the following categories :
Full Marks : + 4 If only the bubble(s) corresponding to all the correct option(s) is (are)
darkened.
Partial Marks : + 1 For darkening a bubble corresponding to each correct option, provided NO
incorrect option is darkened.
Zero Marks : 0 If none of the bubbles is darkened.
Negative Marks : - 2 In all other cases.
l For example, if (a), (c) and (d) are all the correct options for a question, darkening all these three will get in + 4 marks ;
darkening only (a) and (d) will get + 2 marks and darkening (a) and (b) will result in - 2 marks, as a wrong option is also
darkened.

8. Two difference coils have self-inductances 11. A particle of charge + q and mass m moving
L1 = 8 mH and L2 = 2 mH. The current in one under the influence of a uniform electric field
coil is increased at a constant rate. The current E $i and uniform magnetic field B k$ follows a
in the second coil is also increased at the same trajectory from P to Q as shown in figure. The
constant rate. At a certain instant of time, the velocities at P and Q are v $i and - 2 $j. Which of
power given to the two coils is the same. At the following statements(s) is/are correct?
that time, the current, the induced voltage and
the energy stored in the first coil are i1, V1 and Y
P v E
W1 respectively. Corresponding values for the
second coil at the same instant are i2, V 2 and
B
W 2, respectively. Then, a
i1 1 i1
(a) = (b) =4
i2 4 i2
W 1 V Q X
(c) 1 = (d) 1 = 4 2a 2v
W2 4 V2
3 é mv 2 ù
9. A proton moving with a constant velocity (a) E = ê ú
passes through a region of space without any 4 ë qa û
change in its velocity. If E and B represent the (b) Rate of work done by the electric field at P is
electric and magnetic fields respectively, this 3 é mv 3 ù
ê ú.
region of space may have 4ë a û
(a) E = 0, B = 0 (b) E = 0, B ¹ 0 (c) Rate of work done by the electric field at P is zero.
(c) E ¹ 0, B = 0 (d) E ¹ 0, B ¹ 0 (d) Rate of work done by both the fields at Q is zero.

10. A simple pendulum of length L and mass (bob) 12. A reference frame attached to the earth
M is oscillating in a plane about a vertical line (a) is an inertial frame by definition.
between angular limits - f and + f. For an (b) cannot be an inertial frame because the earth is
angular displacement q (|q | < f ), the tension in revolving round the sun.
(c) is an inertial frame because Newton’s laws are
the string and the velocity of the bob are T and
applicable in this frame.
V , respectively. The following relations hold
(d) cannot be an inertial frame because the earth is
good under the above conditions. rotating about its own axis.
(a) T cos q = Mg.
(b) T - Mg cos q = MV 2 / L. 13. H + , He+ and O 2 + all having the same kinetic
(c) The magnitude of the tangential acceleration of the energy pass through a region in which, there is
bob | aT | = g sin q. a uniform magnetic field perpendicular to their
(d) T = Mg cos q. velocity.

PREP CATALYSIS
JEE Advanced~Practice Set 6 173

The masses of H + , He+ and O 2 + are 1 amu, 4 14. Time for 20 oscillations of a pendulum is
amu and 16 amu respectively. Then, measured as t1 = 3.96 s, t2 = 39.9 s and
(a) H+ will be deflected most. t3 = 39.5 s. Now, choose the correct options.
(b) O2 + will be deflected most. (a) Precision in measurement is 0.005 s.
(c) He+ and O2 + will be deflected equally. (b) Precision in measurement is 0.015 s.
(d) All will be deflected equally. (c) Accuracy of measurement is 0.005 s.
(d) Accuracy of measurement is 0.015 s.

Section 3 (Maximum Marks : 12)


l This section contains TWO paragraphs.
l Based on each paragraph, there are TWO questions.
l Each question has FOUR options (a), (b) (c) and (d). ONLY ONE of these four options is correct.
l For each question, darken the bubble corresponding to all the correct option in the ORS.
l For each questions, marks will be awarded in one of the following categories.
Full Marks : + 3 If only the bubble corresponding to all the correct answer is darkened.
Zero Marks : 0 In all other cases.

Paragraph X Md Md md
(a) d (b) (c) (d)
A horizontal shot of mass m penetrate distance d in a m m+M M +m
fixed plate of mass M. Paragraph A
Consider the nuclear reaction,
M a + 14
7 N ¾® X + 1 p
1

m u In above reaction, KE of incoming a-particle, which


strikes stationary nitrogen nucleus is 7.7 MeV.
d Q-value of reaction is negative, Q = - 1.26 MeV.
[Take, m( p) = 1.00814u, m (N) = 14.00752 u , m
(a ) = 4.0038 u]
15. The average force applied by plate on shell,
when plate is fixed 17. Element X in reaction is
mu 2 mu 2 mu 2 mu 3 (a) 17 (b) 16 (c) 16 (d) 16
(a) (b) (c) (d) 8 O 8 O 7 N 8 N
d 2 2d 2d
18. Angle between direction of incoming a-particle
16. The distance of penetration, when plate is free and outgoing proton (11 p) is
to move during collision is
(a) 0° (b) 30° (c) 45° (d) Above 60°

CHEMISTRY
OH OH
Section 1 (Maximum Marks : 21)
Br Br
Instructions: Same as given in Physics. (a) A= B=
19. An aromatic compound A (C 7 H 8 O) on reaction CH3 CH3
with Br2 + H 2 O gives a white ppt. of compound Br
B (C 7 H 5 OBr3 ). The compound A is soluble in OCH3 OCH3
NaOH. Compound C, an isomer of A also gives
Br Br
the same reaction and gives a white ppt. of
compound D (C 7 H 5 OBr3 ). The compound C is C= D=
insoluble in NaOH. Identify A, B, C and D .
Br

PREP CATALYSIS
174 JEE Advanced~Practice Set 6

OH OH Br
Br (b) A= CCl3
(b) A= B=
B = R—CHCl—CH2—CCl3
Br CH3
CH3 C = R—CH—CH2—CH—R
OCH3 OCH3 CH2CCl3 Cl
Br Br H
C= D= (c) A= CCl3
Br
Br B = R—CH(Cl)—CH2—CCl3
OH OH
C = R—CH—CH2—CH2—R
(c) A= B= CH2CCl3
CH3
Br
CH3 Br (d) A= CCl3

OH OCH3 B =R—CH2—CH(Cl)—CCl3
C = R—CH—CH2—CH—R
C= D=
CH2CCl3 Cl
Br Br Br
OH OH 21. 138 g of N 2O4( g) is placed in 8.2 L container at
300 K. The equilibrium vapour density of
CH3
mixture was found to be 30.67. Then
(d) A= B=
(R = 0.082 L atm mol -1 K -1)
CH3
(a) total pressure at equilibrium = 675
. atm
Br (b) degree of dissociation of N2O5 = 0.25
(c) the density of equilibrium mixture = 15.83 g/L
OH OCH3 (d) K p of N2O4 - 2NO2 (g) will be 2 atm
Br
22. One mole of an ideal gas is carried through the
C= D=
reversible cyclic process as shown. The maximum
Br temperature attained by the gas during the
Br cycle is

20. Consider the following reactions: 4 atm B

(i) 1, 3-butadiene + BrCCl 3 + Peroxide ¾® A p


1atm C
Peroxide
(ii) RCH == CH 2 + CCl 4 ¾¾¾® B + C
Identify A to C. 1L 2L
V
H 7 12 49
(a) CCl3 (a) (b) (c) (d) None of
A= 6R 49R 12R
Br these
B = R—CH2—CHCl—CCl3
23. What will be the correct trend of aqua acid
C = R—CH—CH2—CH2—R strength of the following compound? [Al(H 2O)6 ]3 + ,
[Fe(H 2O)6 ]3 + , [Fe(H 2O)6 ]2+ , [Hg(H 2O)]2+
CH2CCl3
(a) [Fe(H2O)6 ]2+ » [Fe(H2O)6 ]3+ < [Al(H2O)6 ]3+
» [Hg(H2O)]2+
(b) [Fe(H2O)6 ] » [Fe(H2O)6 ] < [Al(H2O)6 ]3+
2+ 3+

< [Hg(H2O)]2+

PREP CATALYSIS
JEE Advanced~Practice Set 6 175

(c) [Fe(H2O)6 ]2+ < [Fe(H2O)6 ]3+ < [Al(H2O)6 ]3+ O O


» [Hg(H2O)]2+
CHO ½½ ½½
(a) ½ (b) CH3 — C— C—H
(d) [Fe(H2O)6 ]2+ < [Fe(H2O)6 ]3+ < [Al(H2O)6 ]3+ CHO
<[Hg(H2O)]2+ O

24. H 2SO4is electrolysed by using Pt electrodes in


the presence of high current density due to which
at the cathode 2.52 L of H 2 at STP was generated (c) O (d) All of these
along with 1.2 L of O 2 at anode along with 30. Choose the correct statement(s).
H 2S2O 8 . Calculate weight of H 2S2O 8 generated. (a) —N == O, — SO2R, — NO2 , — X (set of
(a) 1.039 g deactivating groups)
(b) 2 g
F Cl Br I
(c) 3.0039 g
(d) 1.029 g
(b) > > >
25. The product obtained by heating zinc blende in
air is dissolved in dil. H 2SO4 produces [A].
A mixture of [ A] and BaS is used as white (order of rate of electrophilic substitution)
pigment. Choose the incorrect statement among
(c) All deactivating substituents are meta directing
following statements.
⊕ σ
(a) [A] is known as white vitriol when it contain 7 moles
of water of crystallisation (d)
(b) [A] can also be prepared by reaction of zinc , ,
carbonate with dil. sulphuric acid (set of aromatic species)
(c) A mixture of [A] and barium sulphate is known as
lithophone
31. The chemical reaction(s) involved in
(d) [A] is zinc oxide and amphoteric in nature
development of photographic plate is/are
Section 2 (Maximum Marks : 28) (a) AgBr + 2 NH 3 (aq) ¾® [Ag(NH 3 ) 2 ]Br
(b) AgBr + 2 Na 2 S 2 O 3 ¾® Na 3 [Ag(S2O3 )2 ] + NaBr
Instructions: Same as given in Physics.
(c) 2 AgBr + Na 2 S 2 O 3 ¾® Ag 2 S 2 O 3 +2 NaBr
26. A coordination compound having molecular (d) C6H4 (OH)2 + 2 AgBr ¾® 2 Ag + C6H4O2 + 2HBr
formula Pd C14H10S2N 2 which contains phenyl
and SCN as a ligand shows 32. Which of the following statement(s) is/are
(a) coordination isomerism correct?
(b) linkage isomerism (a) All real gases are less compressible than ideal gas
(c) geometrical isomerism at high pressure.
(d) ionisation isomerism (b) H2 and He are more compressible than ideal gases
for all values of pressure.
27. Which of the following is a meso compound? (c) Except H2 and He, the compressibility factor
Me æ Z = pV < 1ö, for all gases at low pressure.
ç ÷
Me Me Me è nRT ø
(a) (b) (c) (d) All of these
Me (d) The compressibility factor of real gases is
H H independent of temperature.
Me

28. The activity per mL of solution of radioactive Section 3 (Maximum Marks : 12)
substance is x. How much water should be Instructions: Same as given in Physics.
added to 200 mL of this solution so that the
activity falls to x/ 20 per mL after 4 half-lives? Paragraph X
(a) 80 mL (b) 50 mL Compound (A) with molecular formula C16 H16
(c) 10 mL (d) 100 mL undergoes ozonolysis to give 2 moles of C 8 H 8 O (B).
O3 Compound (B) gives iodoform test and produces
29. o-xylene ¾¾¾® Product(?). Which of the sodium benzosate (B) also react with N 2 H 4 / KOH
Zn + H2O
following product is formed in the given reaction? to give C 8 H10 .

PREP CATALYSIS
176 JEE Advanced~Practice Set 6

O3
(A) C16H16 (ii) the stability of X s
Zn + H2O
(Alkene) (iii) the degree of stabilisation through solvation of X
Na
OH Sodium benzoate (iv) the strength of ( R ¾ X) bond
I2 +
(B) C8H8O N The leaving group tendency is called fugacity.
2H
(2 moles) KO 4
H
(C) C8H10 35. Which of the following statement is correct?
Hydrocarbon
(a) EtO- is a strong base and therefore a good leaving
33. The structure of B is group.
OH OH O (b) The amine group in ArNH2 is converted into a good
leaving group by reacting ArNH2 with
(a) (b) (c) (d) OH NaNO 2 + HCl at 0° C.
(c) The (OH) group is converted into a good leaving
34. Which isomer of A gives a racemic mixture on group by reacting alcohols with TsCl (p -toluene
catalytic hydrogenation? sulphonyl chloride)
O
Ph Ph Ph Me
(a) (b)
Me S — Cl
Me Me Ph Me
O
Ph H Ph Me
(c) (d) (d) The amine group in R NH2 is converted into poor
H Ph Me Ph leaving group by reacting R NH2 with NaNO 2 + HCl
at 0° C.
Paragraph A
36. Point out the wrong order of fugacity.
The leaving group is that functional group which is
ejected with electron of the s-bond in a reaction. (a) SbH3 > AsH3 >PH3 >NH3
s s s
Better the leaving group, faster is the reaction. The (b) F s >OH >NH2 >CH3
relative leaving ability of the leaving group X in (c) NH3 >NH2 — NH2 >MeNH —NHMe>Me 2N — NMe 2
( R ¾ X) is increased by s s s s
(d) CN >OH >OMe >CH3
(i) the polarisability of ( R ¾ X) bond

MATHEMATICS
$i + $j
Section 1 (Maximum Marks : 21) r= + l( $i + $j + k$ ), l Î R. If ai , bi , ci Î I,
2
Instructions: Same as given in Physics.
ai < ai + 1 and| Pi¢¢( x )| £ 9, then
n 1
37. A non-zero vector a is parallel to the line of
intersection of the plane determined by the å ò ( - 1)i Pi ( x ) dx is equal to
i = 1- 1
vectors $i, $i + $j and the plane determined by the
vectors $i - $j, $i + k
$ . The angle between a and (a) 1 (b) 0 (c) - 1 (d) 2
i$ - 2$j + 2k is
$ 40. Let z be a complex number satisfying equation
(a)
p
(b)
p
(c)
p
(d)
p z 2 - ( 3 + i )z + m + 2i = 0, where m Î R. Suppose
4 3 6 2 the equation has a real root. Then, the non real
38. If three 1 ´ 1 squares are selected at random root is
(a) 1 - i (b) 1 + i (c) - 1 - i (d) - 1 + i
from a 8 ´ 8 chess board, then the probability
that they form the letter ‘L’ is 41. If r1, r2, r3 are ex-radii of a DABC and R is
196 49 radius of circumcircle, then minimum value of
(a) 64
(b) 64
C3 C3 bc ca ab
36 93 + + will be
(c) (d) r1 r2 r3
64 64
C3 C3 1 1
(a) 6R (b) (c) 3R (d)
39. Let curves y = Pi ( x ) = ai x + bi x + ci , i = 1, 2, 3,
2 6R 3R
..., n on XY -plane intersect the line

PREP CATALYSIS
JEE Advanced~Practice Set 6 177

42. There are three coplanar parallel lines. If any 48. The solution(s) of differential equation
2
p points are taken on each of the line, the æ dy ö dy
x ç ÷ + ( y - 2x ) - 2 y = 0 is/are
maximum number of triangles with vertices at è dx ø dx
these points is (a) y = 2 x + c (b) y = - 2 x - c
(a) 3 p 2 (p - 1) (b) 3 p 2 (p - 1) + 1 (c) x = c / y (d) x 2 y 2 = y + c
(c) p 2 (4 p - 3) (d) None of these
49. The equation of a plane is 2x - y - 3z = 5 and A
43. If f ( x ) = 3|x| - x - 2 and g( x ) = sin x, then (1, 1, 1), B (2, 1, - 3), C (1, - 2, - 2) and
domain of definition of fog( x ) is D ( - 3, 1, 2) are four points. Which of the
p following line segment are intersected by the
(a) ìí2n p + üý, x ÎI plane?
î 2þ
(a) AD (b) AB
7p 11p ù
(b) U é 2np + , 2n p + (c) AC (d)BC
ê
n ÎI ë 6 6 ûú
7p ü 50. Let A, G and H are the AM, GM and HM
(c) ìí2np + ý, n ÎI
î 6þ respectively of two unequal positive integers.
ì p ü Then, equation Ax 2 -|G| x - H = 0 has
(d) í(4m + 1) m ÎI ý U é 2np + 7 p , 2np + 11p ù
êë 6 úû (a) both roots as fractions
î 2, þ n ÎI 6
(b) at least one root which is a negative fraction
Section 2 (Maximum Marks : 28) (c) exactly one positive root
(d) at least one root which is an integer.
Instructions: Same as given in Physics.
æ 1 ö
Section 3 (Maximum Marks : 12)
44. If (1 + tan2 2x ) ç cos2 y + ÷ ( 3 + sin 3z ) = 4, Instructions: Same as given in Physics.
è cos y ø
2
Paragraph X
then
p Suppose a function f ( x) satisfies the following
(a) x is an integral multiple of
2 f ( x) + f ( y)
conditions f ( x + y) = , " x, y and f ¢ ( 0) = 1.
(b) y is an integral multiple of p 1 + f ( x) × f ( y)
2np p Also, - 1 < f ( x) < 1, " x Î R. Then,
(c) z = + , x ÎI
3 2
(d) z is a multiple of p 51. f ( x ) is differentiable over the set
(a) R (b) R - {- 1, 0, 1} only
45. If ò sec 2x dx = f [g( x )] + c, then (c) {- 1, 0,1} only (d) None of these
(a) dom f (x) = R - {0} 52. the value of the limit lim [ f ( x )]x is
x ®¥
(b) range of g (x) = R
1 (a) 0 (b) 1 (c) - 1 (d) 2
(c) f ¢ (x) = , " x ÎR +
2x Paragraph A
p
(d) g ¢ (x) = - cosec2 æç - x ö÷ Let A( 0, 5), B(1, 3), C( 4, 9) are three points on the
è4 ø
parabola y = ax 2 + bx + c. Then,
46. The critical points of the function f ( x ),
| x - 2| 53. the point D on the curve which is nearest from
where f ( x ) = 2
is the line AB is
x
(a) æç , ö÷ (b) æç , ö÷ (c) æç , ö÷
1 15 1 11 3 11
(a) 0 (b) 2 è2 4 ø è2 4 ø è2 4 ø
(c) 4 (d) 6
(d) æç , ö÷
3 15
n n è2 4 ø
47. The value of the sum å å ( 3n + 2n ) n Cm mCr
r =1 m = r 54. the lowest point on the curve y = ax 2 + bx + c is
(a) æç , ö÷ (b) æç , 2 ö÷
must be 3 11 3
(a) 9n + 4n (b) 9n - 4n è2 2 ø è2 ø
(c) 36n (c) æç , ö÷ (d) æç , ö÷
(d) independent of r 1 19 3 11
è2 4 ø è2 4 ø

PREP CATALYSIS
178 JEE Advanced~Practice Set 6

Answers
Paper 1
1. a, d 2. c, d 3. a, c, d 4. a, c 5. a, b, c 6. a, c 7. a, d 8. 4 9. 6 10. 1
11. 4 12. 1 13. c 14. a 15. b 16. a 17. b 18. d 19. c 20. c
21. a, c 22. c, d 23. d 24. a, c, d 25. a, c, d 26. 4 27. 1 28. 6 29. 1 30. 4
31. a 32. b 33. d 34. a 35. b 36. d 37. a,b,c,d 38. a, b, c 39. a, c 40. a, d
41. a, c, d 42. a, b, d 43. b, c 44. 1 45. 3 46. 14 47. 8 48. 8 49. d 50. b
51. d 52. b 53. b 54. d

Paper 2
1. c 2. d 3. c 4. b 5. a 6. a 7. b 8. a, c, d 9. a, c, d 10. b, c
11. a, b, d 12. b, d 13. a, c 14. a, d 15. c 16. c 17. a 18. d 19. a 20. b
21. a, c, d 22. c 23. c 24. a 25. d 26. b 27. c 28. b 29. d 30. c
31. b, d 32. a, b, c 33. c 34. d 35. c 36. d 37. a 38. a 39. b 40. b
41. a 42. c 43. d 44. a, b, c 45. a, b, c 46. a, b, c 47. b, d 48. a, c 49. b, c 50. b, c
51. a 52. b 53. a 54. d

SCORE SHEET - Paper 1


Section No. of Marks from Marks from Marks Obtained
Correct Questions Correct Questions (A) Incorrect Questions (B) (A-B)
................ ............................... ............................... ............................... ...............................
................ ............................... ............................... ............................... ...............................
................ ............................... ............................... ............................... ...............................
Percentage Marks = Marks Obtain/Total Marks x 100

SCORE SHEET - Paper 2


Section No. of Marks from Marks from Marks Obtained
Correct Questions Correct Questions (A) Incorrect Questions (B) (A-B)
................ ............................... ............................... ............................... ...............................
................ ............................... ............................... ............................... ...............................
................ ............................... ............................... ............................... ...............................
Percentage Marks = Marks Obtain/Total Marks x 100
Note To expect your success marks in the test should be between 65%-70%.

PREP CATALYSIS
PREP CATALYSIS
JEE Advanced

PRACTICE SET 7 (With Solutions)

Duration : 3 Hours Max. Marks . 360

Paper 1
PHYSICS
Section 1 (Maximum Marks : 24)
l This section contains SIX (06) questions.
l
Each question has FOUR options for correct answer(s). ONE OR MORE THAN ONE of these four option(s) is (are)
correct options(s).
l
For each question, choose the correct options(s) to answer the question.
l
Answer to each question will be evaluated according to the following marking scheme:
Full Marks : + 4 If only (all) the correct option(s) is (are) chosen.
Partial Marks : + 3 If all the four options are correct but ONLY three options are chosen.
Partial Marks : + 2 If three or more options are correct but ONLY two options are chosen, both of which are
correct options.
Partial Marks : + 1 If two or more options are correct but ONLY one option is chosen and it is a correct option.
Zero Marks : 0 If none of the options is chosen (i.e. the question is unanswered).
Negative Marks : - 2 In all other cases.
l
For example: If first, third and fourth are the ONLY three correct options for a question with second option being an
incorrect option; selecting only all the three correct options will result in + 4 marks. Selecting only two of the three
correct options (e.g. the first and fourth options), without selecting any incorrect option (second option in this case), will
result in +2 marks. Selecting only one of the three correct options (either first or third or fourth option), without selecting
any incorrect option (second option in this case), will result in +1 marks. Selecting any incorrect option(s) (second
option in this case), with or without selection of any correct option(s) will result in -2 marks.

1. A horizontal force F is applied on a body of I


(b) If h = , then body rolls without any slip only, if
circular cross-section. Let h = height of line of MR
action of force, M = mass of body and R = surface is rough.
I
radius of cross-section of body. (c) If h > , then friction acts in forward direction.
MR
Now, choose the correct options. I
(d) If h < , then friction acts in reverse direction to
F MR
that of F .
M h0
R
2. A particle is executing SHM with amplitude A.
At displacement, x 2 = - A / 4, force acting on
I
(a) If h = , then body rolls without any slip even, the particle is F, potential energy of the
MR
particle is U, velocity of particle is v and
when there is no friction.
kinetic energy is K.

PREP CATALYSIS
180 JEE Advanced~Practice Set 7

Assuming, potential energy to be zero at mean (d) will not be valid, because isotropy is lost.
position. At displacement, x = A / 2
5. In Davisson-Germer experiment, which of
(a) force acting on the particle will be 2F . these are correct?
(b) potential energy of the particle will be 4U.
(a) Scattering of electrons is from many individual
4
(c) velocity of the particle must be v. crystals in the Ni sample.
5 (b) Scattering of electrons is from atoms of only one
(d) kinetic energy of the particle will be 0. 8 K. crystal in the Ni sample.
3. Two bodies A and B have thermal emissivities (c) Kinetic energy of a missile electron decreases as it
of 0.01 and 0.81, respectively. The outer enters Ni crystal.
surface areas of the two bodies are the same. (d) Kinetic energy of a missile electron increases as it
enters Ni crystal.
The two bodies emit total radiant power at the
same rate. 6. For the given circuit, E1 = 3 V, R1 = 100 W,
The wavelength l B corresponding to maximum R2 = 300 W and R3 = 250 W, cell and ammeter
spectral radiancy in the radiation from B is are ideal. Voltmeter resistance is 5 kW.
shifted from the wavelength corresponding to
maximum spectral radiancy in the radiation +
3V – R2=300Ω
from A by 1.00 mm. If the temperature of A is
5802 K, then R1=100Ω
A
(a) the temperature of B is 1934 K.
(b) lB = 1.5 mm. A V R3=250Ω
(c) the temperature of B is 11604 K.
B
(d) the temperature of B is 2901 K.
B C
4. ABCDEFGH is a hollow cube +P
made of an insulator. Face A Now, choose the correct options.
D
(a) Voltmeter reads 3% lower value of potential drop
ABCD has positive charge on
across AB.
it. Inside the cube, we have G (b) Voltmeter reads 3% more value of potential drop
F
ionised hydrogen. across AB.
E H
(c) IfVAB is measured by using a potentiometer of
The usual kinetic theory expression for pressure
potential gradient 0.01 V/cm, then the balance
(a) will be valid.
length is 115 cm.
(b) will not be valid, since the ions would experience
(d) IfVAB is measured by using a potentiometer of
forces order than due to collisions with the walls.
potential gradient 0.01 V/cm, then the balance
(c) will not be valid, since collisions with walls, would
length is 112 cm.
not be elastic.

Section 2 (Maximum Marks : 24)


l
This section contains EIGHT (08) questions. The answer to each question is a NUMERICAL VALUE.
l
Four each question, enter the correct numerical value (in decimal notation, truncated/rounded-off to the second
decimal place; e.g. 6.25, 7.00.- 0.33, -.30, 30.27, -127.30) using the mouse and the on-screen virtual numeric
keypad in the place designated to enter the answer.
l
Answer to each question will be evaluated according to the following marking scheme:
Full Marks : + 3 If ONLY the correct numerical value is entered as answer.
Zero Marks : 0 In all other cases.

7. Two blocks A and B of equal masses are


released from an inclined plane of inclination 2m
A
45° at t = 0. Both the blocks are initially at rest
B
and the coefficient of kinetic friction between
the block A is 0.2, while it is 0.3 for block B.
Initially the block A is 2 m behind the block
B
B. When their front faces will come in a line? A
45°
(Take, g = 10 m/s2 )

PREP CATALYSIS
JEE Advanced~Practice Set 7 181

8. In the figure, masses m1, m2 and M are released from rest, then find the acceleration
20 kg, 5 kg and 50 kg, respectively. of A.
P1 m1

P2 A B
M
M F o o
m2 m 45 45 2m

The coefficient of friction between M and ground 11. A block of mass 2 kg slides on an inclined
is zero. The coefficient of friction between m1 and plane, which makes an angle of 30° with the
M and that between m2 and ground is 0.3. The horizontal. The coefficient of friction between
pulleys and the strings are massless. The string is the block and the surface is 3 / 2 .
perfectly horizontal between P1 and m1 and also If Fup = force required to move block up the
between P2 and m2.
plane without acceleration and Fdown = force
The string is perfectly vertical between P1 and P2. required to move the block down the plane
An external horizontal force F is applied to the without acceleration. Then, find the value of
mass M. (Take, g = 10 m/ s2) (Fup - Fdown )/10.
Let the magnitude of the force of friction between
12. A wire has a mass, m = 0.3 ± 0.003 g and
m1 and M be f1 and that between m2 and ground
be f2. For a particular force F, it is found that radius, r = 0.5 ± 0.005 mm and length,
f1 = 2 f2. If T = tension in the string, then find the l = 6 ± 0.006 cm. Find the maximum
value of T / 2. percentage error in measurement of its
9. Nuclei of a radioactive element A are being density.
produced at a constant rate a. The element has 13. A train is moving along a straight line with a
a decay constant l. At time t = 0, there are N 0 constant acceleration. A boy standing in train
nuclei of the element. If a = 2N 0 l, then find the throws a ball forward with a speed of 10 ms-1
N
value of as t ® ¥. at an angle of 60° to horizontal. The boy has
N0 to move forward by 1.15 m inside train to
10. Block A of mass m and block B of mass 2m are catch ball at the initial height. Find the
placed on a fixed triangular wedge by means of acceleration of train.
a massless, in extensible string and a 14. A body of mass 1 kg initially at rest explodes
frictionless pulley as shown in the figure. The and brakes into three fragments of masses in
wedge is inclined at 45° to the horizontal on both ratio 1 : 1 : 3. Two pieces of equal mass fly off
sides. The coefficient of friction between block A perpendicularly with a speed of 30 ms-1 each.
and the wedge is 2/3 and that between block B What is the velocity of heavier segment?
and the wedge is 1/3. If the blocks A and B are (Take, 2 = 1.411).

PREP CATALYSIS
182 JEE Advanced~Practice Set 7

Section 3 (Maximum Marks : 12)


l This section contains TWO (02) paragraphs. Based on each paragraph, there are TWO(02) questions.
l Each question has FOUR options. ONLY ONE of these four options corresponds to the correct answer.
l Four each question, choose the option corresponding to the correct answer.
l Answer to each question will be evaluated according to the following marking scheme:
Full Marks : + 3 If ONLY the correct option is chosen.
Zero Marks : 0 If none of the options is chosen (i.e. the question is unanswered).
Negative Marks : - 1 In all other cases.

Paragraph X (d) No fringe is formed


Two glass plates are two microscope slides 10 cm Paragraph A
long. They are kept as shown in the figure. If pulleys are frictionless and massless and strings
are massless, then answer the following questions on
the basis of given situation.

0.02mm

10 cm
At one end slides are in contact and at other end they C aC
are 0.02 mm apart. Interference is observed using 500 aA A B aB
nm light.
17. The relation between aA , aB and aC
15. Spacing between interference fringes, when
(a) aA + aC = aB (b) aA + aC = 2 aB
seen from top of first slide is
(c) aA + aB = 2 aC (d) aA + 2aB = aC
(a) 0.02 mm (b) 2.50 mm (c) 1.25 mm (d) 0.25 mm
18. The relation between their accelerations of the
16. Fringe formed at the line of contact is three blocks will depend on
(a) dark (b) bright (a) mass of A (b) mass of B
(c) sometimes dark sometimes bright (c) mass of C (d) None of these

CHEMISTRY
Section 1 (Maximum Marks : 24)
Instructions: Same as given in Physics. (c) A = [Cr(H2O)5 ] Cl 3 × H2O, B = [Cr(H2O)6 ] Cl 3 ,
C = [Cr(H2O)4 Cl 2 ] Cl × H2O
19. A, B and C are three complexes of chromium (d) A = [Cr(H2O)6 ] Cl 3 , B = [Cr(H2O)4 Cl 2 ] Cl × 2H2O,
(III) with empirical formula H12O6Cl3Cr. All the C = [Cr(H2O)5 Cl] Cl 2 × H2O
three complexes have water and chloride ion as
ligands. Complex A does not react with 20. NiCl2 [P(C2H5 )2(C6H5 )]2 exhibits temperature
concentrated H 2SO4, whereas complexes B and dependent magnetic behaviour
C lose 6.75% and 13.5% of their original mass, (paramagnetic/diamagnetic) the coordination
respectively, on treatment with concentrated geometries of Ni2+ in the paramagnetic and
H 2SO4. The compound A, B and C respectively diamagnetic states respectively are
are (a) tetrahedral and tetrahedral
(a) A = [Cr(H2O)5 Cl] Cl 2 × H2O, B = [Cr(H2O)4 ] Cl 3 , (b) square planar and square planar
C = [Cr(H2O)6 Cl 2 ] (c) tetrahedral and square planar
(b) A = [Cr(H2O)6 ] Cl 3 , B = [Cr(H2O)5 Cl] Cl 2 × H2O, (d) square planar and tetrahedral
C = [Cr(H2O)4 Cl 2 ]Cl × 2H2O

PREP CATALYSIS
JEE Advanced~Practice Set 7 183

21. Which of the following statement(s) is/are correct? (a) DSx ¾® y = 95.74 J K -1
(a) The degree of hydration decreases on moving (b) DSx ¾® z = 0
down the group from Li+ to Cs+ (c) DSx ¾® y = DSx ¾® z + DSz ¾® y = 95.76JK -1
(b) Li 2O2 is prepared by adding hydrogen peroxide (d) DSx ¾® z ¾® y = 0
and alcohol to a concentrated solution of lithium
hydroxide
(c) Lattice energy of ionic solids having the same anion Section 2 (Maximum Marks : 24)
decreases with increase in the size of the cation Instructions : Same as given in Physics.
(d) Lithium nitrate evolves nitrogen monoxide and
oxygen on heating 25. Among the given molecules, the number of
metal sulphides precipitated by passing H 2S in
22. In the following reaction sequence, the correct
acidic medium are ..........
structure of X is/are
O ZnS, PbS, CuS, NiS, MnS, CdS, CoS, HgS, SnS
Br
æ 7 ö
Mg
A, then A +
(i) Et2O
B 26. Three moles of an ideal gas çCV , m = R ÷ are
Et2O (ii) H3O+ è 2 ø
CH3 initially at a pressure of 1 bar and temperature
OH of 300 K. It is then taken through a number of
compression and expansion steps.
OH CH3
A. It is compressed isothermally and reversibly
(a) (b)
CH3 until the pressure is 20 bar.
B. It is expanded adiabatically against a constant
O OH external pressure of 2 bar.
CH3 C. Next it is taken through a process that is
(c) (d) neither adiabatic nor isothermal until the
OH volume is four times the original value and the
temperature is 400 K.
The entropy change of the gas after all three
23. Among the following, which reaction will give processes were completed is ……… .
N-ethylcyclopentylamine as major product?
H2/Pt
27. A cube shaped crystal of an alkali metal,
(a) + NH2 1.62 mm on an edge, was vaporised in a
500.00 mL flask. The pressure of the resulting
vapour was 12.50 mm at 802° C. The structure
H2/Pt
(b) —NH2 + H of the solid metal is known to be body centred
cubic. The atomic radius of the metal atom in
picometre is …… .
CH3C Cl (i) LiAlH4, diethylether 28. The ionisation constant of benzoic acid is
(c) —NH2
Pyridine (ii) H2O
6.46 ´ 10-5 and K sp for silver benzoate is
2.5 ´ 10-13 . The number of times silver
CH3CH2 NH2 (i) LiAlH4, diethylether
(d) —CCl benzoate is more soluble in a buffer of pH 3.19
Pyridine (ii) H2O
compared to its solubility in pure water will be
………
24. Consider the following process and choose the
correct answer(s) from the options given below: 29. A spherical glass bulb of radius 1.0 m contains
a concentric rubber balloon that contains some
5 moles x Isothermal y N 2 gas and the remaining space in the flask
of argon P=10 P=1
at 298K atm atm contains 50 g H 2. In the given conditions,
radius of the rubber balloon was found to be
q=0
40 cm. Now seal of the glass bulb was opened
and 25g H 2( g) was further added and released.
Assuming constant temperature throughout,
5 moles the radius of the rubber balloon in the new
of argon z
at 118.6K conditions is ……… .

PREP CATALYSIS
184 JEE Advanced~Practice Set 7

30. Hydrogen atoms wheighing 4.8 g were excited


to radiations. The study of spectra showed that
18% of the atoms were in third energy level; (c)
10% of atoms were in second energy level and HOOC
the rest were in ground state. The total energy
evolved when all the atoms return to the
ground state would be ……….
(d)
[Given: Ionisation potential of H = 13.6 eV]
COOH
31. Consider the reaction involved in discharging
of lead storage battery. 34. The compound ‘D’ is
Pb( s) + PbO2 + 2H 2SO4( l ) ¾® 2PbSO4( s) + 2H 2O
On discharging, the concentration of H 2SO4
electrolyte changes from 38.2% by mass (density (a) (b)
1.261 g m L-1 at 25° C) to 22% by mass. The H2 N
N H2 N
original volume of electrolyte is one litre. If the
water is produced by the cell reaction as H 2SO4
is used up, then the amount of Faraday left the
anode of the battery will be ……… .
32. Two solid compounds A and B dissociates into (c) (d)
gaseous products at 293 K as N N

(i) A (s) - X ( g) + H S( g); K = KA N N


2 eq
(ii) B (s) -Y ( g) + H S( g); K
2 eq = KB Paragraph A
Under the given condition, pressure over An acyclic hydrocarbon ‘P’ having molecular
excess solid A is 68 mm and that over excess formula C 6 H10 , gave acetone as the only organic
solid B is 50 mm. The total pressure if the product through the following sequence of reactions,
above decomposition takes place in the same
in which Q is an intermediate organic compound.
vessel is ……… .
P (i) Dil. H2SO4
O
(i) Dil. H2SO4/HgSO4
Section 3 (Maximum Marks : 12) (C6 H10 ) (ii) NaBH4/ethanol
Q (ii) O3/ethanol
2
(iii) Dil. acid (iii) Zn/H2O
H3C CH3
Instructions: Same as given in Physics.

Paragraph X 35. The structure of compound ‘P ’ is


An organic compound A, C8H 4O3 , in dry benzene in (a) CH3 CH2 CH2 CH2 ¾ C ºº C ¾H
the presence of anhydrous AlCl 3 , gives compound (b) H3 C H2 C ¾ C ºº C ¾ CH2 CH3
B. The compound B on treatment with PCl5 followed H3C
by reaction with H 2 /Pd, BaSO 4 gives compound C, (c) H—C—C≡≡C—CH3
which on reaction with hydrazine gives a cyclised H3C
compound D(C14H10N 2 ). H3C
(d) H3C—C—C≡≡C—H
33. The compound ‘B’ is H3C

36. The structure of compound ‘Q’ is


OH OH
H3C H3C
(a)
(a) H—C—C—CH2CH3 (b) H3C—C—C—CH3
H3C H3C
H H
COOH
OH OH
H3C
COOH (c) H—C—CH2 CH CH3 (d) CH3 CH2 CH2 CH CH2CH3
(b) H3C

PREP CATALYSIS
JEE Advanced~Practice Set 7 185

MATHEMATICS
Section 1 (Maximum Marks : 24) event that the ship is operational and let
Instructions: Same as given in Physics. X1 , X 2 and X3 denote, respectively, the events
that the engine E1 , E2 and E3 are functioning.
37. Given a real-valued function f, such that Which of the following is (are) TRUE?
3
ì tan2 { x } (a) P (X1¢ / X ) =
ï 2 , for x > 0 16
ï ( x - [x ] )
2 5
(b) P (X / X 2 ) =
f(x) = í 16
1 for x = 0 7
ï (c) P (X / X1) =
ï { x } cot { x } for x < 0 16
î (d) P (exactly two engine of ship are functioning/X)
=7/8
where, [x ] is the integral part and { x } is the
fractional part of x, then which of the following 1æ 1ö 1æ 1ö
41. If cosa = ç x + ÷ and cosb = ç y + ÷,
is(are) TRUE? 2è xø 2è yø
(a) lim f (x) = 1 (b) lim f (x) = cot 1 ( xy > 0); x , y , a, b, Î R. Then, which of the
x ® 0+ x ® 0-
following is (are) TRUE?
2
æ ö æ ö p (a) sin(a + b + g) = sin g, " y ÎR
(c) cot - 1 ç lim f (x)÷ = 1 (d) tan- 1 ç lim f (x)÷ =
èx ® 0- ø èx ® 0+ ø 4 (b) cos a cosb = 1, " a, b ÎR
(c) (cos a + cosb)2 = 4, " a, b ÎR
38. Let PM be the perpendicular from the point (d) sin(a + b + g) = sin a + sinb + sin g,"a, b, g ÎR
P(1, 2, 3) to the XY -plane. If OP makes an
angle q with the positive direction of the Z-axis 42. If f ( x ) is continuous in [0, 2] and f ( 0) = f ( 2),
and OM makes an angle f with the positive then the equation f ( x ) = f ( x + 1) has (where,
direction of X-axis, where O is the origin and q f ( 0) ¹ f (1))
and f are acute angles. Then, which of the (a) non-real root in [0, 2]
following is (are) TRUE? (b) at least one real root in [0, 1]
1 (c) at least one real root in [0, 2]
(a) cos q cos f = (b) sin q sin f = 2 / 14
14 (d) at least one real root in [1, 2]
(c) tanf= 2 (d) tan q = 5 / 3
Section 2 (Maximum Marks : 24)
æp æp öö
39. Let f ( x ) = sinç sinç sin x ÷ ÷ for all x Î R and Instructions: Same as given in Physics.
è6 è2 øø
p 43. A 2 ´ 2 real matrix A has determinant of 2018.
g( x ) = sin x, " x Î R. Let ( fog)( x ) denote f ( g( x ))
2 Let A-2 = ( A-1 )2. If ( tr( A-1 ))2 - tr( A-2 ) is equal
and ( gof )( x ) denote ( g f ( x )). Then, which of the m
following is(are) TRUE? to (where, m and n are relatively prime),
n
(a) Range of f is é - , ù
1 1
êë 2 2 úû
then the value of m + n is equal to ...
(where, tr( B) = The trace of a square matrix B
(b) Range of fog is é - , ù
1 1
êë 2 2 úû , which is defined as sum of the elements on
p the main diagonal)
(c) lim g (x) >
x ® 0 6
(d) There is an x ÎR , such that (gof ) (x) = 1
44. The number of solution of the equation
sin-1(|log26(cos x ) - 1|) + cos-1(|3 log26 (cos x ) - 7|)
40. A ship is fitted with three engines E1, E2 and
= p / 2 is k, if x Î [0, 4p ], then the value of k is
E3 . The engines function independently of each equal to …….
1 1 1
other with respective probabilities , and . 45. A closed has 5 pairs of shoes. The number of
2 4 4
For the ship to be operational at least two of ways in which 4 shoes can be drawn from it
its engine must function.Let X denotes the such that there will be no complete pair is

PREP CATALYSIS
186 JEE Advanced~Practice Set 7

46. A function f : Z ® Z is defined as follows, variable point on this semi-circle such that
ìï n + 3, if n is odd ÐPBD = q, 0 £ q £ p. Let ‘R’ is the region bounded by
f(n ) = í n . arc AP the straight line PD and line AD, and ‘L’ is
, if n is even
îï 2 perimeter of region ‘R’.
If k is an odd integer and f [ f { f ( k)}] = 27, then 51. The maximum possible area of region ‘R’ is
the sum of the digits of the number k is equal to 2p - 3 2p + 3 3 2p + 6 3 2p + 3 3
(a) (b) (c) (d)
12 6 12 12
47. The number of integral values of a , a Î ( 6, 100)
for which the equation [tan x ] 2 + tan x - a = 0 52. The non-negative difference of greatest and
has real roots; where [×] denotes the greatest least value of ‘L’ is,
p p
integer function, is (a) 3 - 3 + (b) 3 - 3 +
3 3
48. In DABC, D divides BC in the ratio 3 : 2 and E 2p
(c) 3 - 3 + (d)p - 2
divides CA in the ratio 1 : 3. The lines AD and 3
BE meet at H and CH meet AB in F. If the Paragraph A
AF k Let z = a + ib = ( a, b) be any complex number, a, b Î R
ratio = , then the numerical quantity k
FB 1
and -1 = i. Let z ¹ 0 + 0i , arg (z) = tan -1 æç
Im z ö
should be ÷,
è Re z ø
1
49. Evaluate ò [x (1 + sin px ) + 1] dx, where [. ] is where - p < arg (z) £ p and arg (z) + arg ( -z)
-1
the greatest integer function. ì p , if arg (z) < 0

50. If a , b, c are three numbers between 1 to 20 î - p , if arg (z) > 0
such that their sum is 18 and the number On the basis of above information answer the
1, a , b are consecutive term of AP and the following questions.
number b, c, 20 are consecutive terms of GP,
then the value of a + b + c is ....... 53. If arg( z ) > 0, then arg( - z ) - arg( z ) is equal to,
(a) p (b) -p
Section 3 (Maximum Marks : 12) (c) p/ 2 (d) - p/ 2
Instructions: Same as given in Physics. 54. If arg( z ) < 0, then arg( z ) - arg( - z ) = 3 l1 and if
Paragraph X arg( z ) > 0 then arg( - z ) - arg( z ) = 2 l 2 then
Let A , B, C , D lie on a line such that (a) l1 - l 2 = 5 p / 6
(b) 3 l1 - 2 l 2 = 0
AB = BC = CD = 1. The points A and C are also joined
(c) l 2 - l1 = – p / 6
by a semi-circle with AC as diameter and P is a (d) 2 l1 - 3 l 2 = 0

PREP CATALYSIS
Paper 2
PHYSICS
Section 1 (Maximum Marks : 24)
l This section contains SIX (06) questions.
l Each question has FOUR options for correct answer(s). ONE OR MORE THAN ONE of these four option(s) is (are)
correct options(s).
l For each question, choose the correct options(s) to answer the question.
l Answer to each question will be evaluated according to the following marking scheme:
Full Marks : + 4 If only (all) the correct option(s) is (are) chosen.
Partial Marks : + 3 If all the four options are correct but ONLY three options are chosen.
Partial Marks : + 2 If three or more options are correct but ONLY two options are chosen, both of which are
correct options.
Partial Marks : + 1 If two or more options are correct but ONLY one option is chosen and it is a correct option.
Zero Marks : 0 If none of the options is chosen (i.e. the question is unanswered).
Negative Marks : - 2 In all other cases.
l For example: If first, third and fourth are the ONLY three correct options for a question with second option being an
incorrect option; selecting only all the three correct options will result in + 4marks. Selecting only two of the three correct
options (e.g. the first and fourth options), without selecting any incorrect option (second option in this case), will result in
+2 marks. Selecting only one of the three correct options (either first or third or fourth option), without selecting any
incorrect option (second option in this case), will result in +1marks. Selecting any incorrect option(s) (second option in
this case), with or without selection of any correct option(s) will result in -2 marks.

1. An a-particle is moving through a region, (a) The region AB represents ice and water in thermal
equilibrium.
where electric field, E = 4i$ - $j - 2k
$ V/m and
(b) At B water starts boiling.
also a magnetic field, B = 2$i + 4$j + k$ exists. (c) At C all the water gets converted into steam.
(d) C to D represents water and steam in equilibrium at
Velocity of a-particle as it enters this region is
boiling point.
v = 2i$ + 3$j - k
$ m/s.
3. Three students perform an experiment to find
Now, choose the correct options. acceleration due to gravity using a simple
(a) Velocity of a-particle is constant with time. pendulum. Their observations are
(b) Acceleration of a-particle is perpendicular to Z-axis.
(c) de-Broglie’s wavelength associated with a -particle Length of Number of Time for (n ) Time
is decreasing with time. Student pendulum l oscillations oscillations period T
(d) Particles’ trajectory is a circle. (cm) (n ) (t s) (s)
A 64 8 128 16
2. Refer to the plot of temperature versus time
B 64 4 64 16
showing the changes in the state of ice on
heating (not to scale). Which of the following is C 20 4 36 9
correct?
Now choose the correct options.
Temperature (°C)

E
(a) Observation of student A is most accurate.
(b) Observation of student C is most accurate.
100 (c) Observation of student B is least accurate.
C D
(d) Observation of student C is least accurate.

A B 4. A convex lens of focal length 30 cm is placed


0 tm time (min) 8 cm away from a concave lens of focal length
20 cm.

PREP CATALYSIS
188 JEE Advanced~Practice Set 7

(a) Force on the body is zero at x = ± a.


f=30cm f=20cm c
(b) Force constant for motion of body is k = .
2a 3
(c) Angular frequency of oscillation of body is
c
w=
8cm 2a 3m
(d) Potential energy of body is always negative.
Now, choose the correct options.
(a) Equivalent focal length of system is (100 / 7) cm. 6. The displacement of a string is given by
(b) A parallel beam from left, when incident over y( x , t ) = 0.06 sin ( 2px/3) cos(120 pt ), where
convex lens, seems to diverge from a point 216 cm x and y are in m and t in s. The length of
from centre of 2 lens system.
the string is 1.5m and its mass is 3.0 ´ 10-2 kg.
(c) When a parallel beam is made to incident over
concave lens, then the parallel beam appears to Choose the correct options.
diverge from a point 416 cm from the centre of two (a) Wave over string is a progressive wave of
lens system. frequency 60 Hz.
(d) Notion of effective focal length is not meaningful for (b) IWave over string is a stationary wave of frequency
this system. 60 Hz.
5. A body of mass m has potential energy, (c) Wave over string is the result of superposition of
- cx two waves of wavelength 3 m, frequency 60 Hz
U = , where c and a are positive
x2 + a2 each travelling with a speed of 180 m/s in opposite
constants. If the body is moving only along direction.
X-axis, then choose the correct options. (d) Amplitude of this wave is constant.

Section 2 (Maximum Marks : 24)


l
This section contains EIGHT (08) questions. The answer to each question is a NUMERICAL VALUE.
l
Four each question, enter the correct numerical value (in decimal notation, truncated/rounded-off to the second
decimal place; e.g. 6.25, 7.00.- 0.33, -.30, 30.27, -127.30) using the mouse and the on-screen virtual numeric
keypad in the place designated to enter the answer.
l
Answer to each question will be evaluated according to the following marking scheme:
Full Marks : + 3 If ONLY the correct numerical value is entered as answer.
Zero Marks : 0 In all other cases.

7. Two satellites S1 and S 2 revolves around a 10. Two rods of different materials having
planet in coplanar circular orbits in same coefficient of thermal expansion a 1 , a 2 and
sense. Radius of orbit of S1 = 104 km. When S 2 Young’s modulus Y1, Y 2 are fixed between two
massive walls. The rods are heated such that
is closest to S1, let the angular speed of S 2 as
they undergo the same increase in temperature.
observed by an astronaut in S1 be x ´ 10-4 If a 1 : a 2 = 2 : 3, then thermal stresses in rods
radians per second. Find the value of x. are equal, then the ratio Y1 : Y 2 is ...................... .
8. A thin fixed ring of radius 1 m has a positive 11. In a cyclotron, magnetic field applied is of 0.7
charge of 1 ´ 10-5 C uniformly distributed over T and radius of its dees is 1.8 m, then energy
it. A particle of mass 9 g with a negative of emergent photon beam in MeV is ……… .
charge of 1 ´ 10-6C is placed on axis at a 12. A resistor of 200 W and a capacitor of 15 mF are
distance of 1 cm from centre of ring. When the connected in series to a 220 V, 50 Hz source.
particle is released, it oscillates with a period Find current in the circuit.
of ……… seconds.
9. Three SHM in the same direction having same
amplitude and same period are superposed. If
each differ in phase from the next by 45°.
Then, find the energy associated with resulting
motion taking energy associated with each of
given SHM as 1 J.

PREP CATALYSIS
JEE Advanced~Practice Set 7 189

13. A disc over which a thread is cm -3 is poured till the upper surfaces of oil and
wound unrolls itself, while falling glycerine are in same level.
vertically.
Acceleration of disc’s centre is
............... .
Glycerine
(Take, g = 9.8 ms-2)
Oil

14. A vertical U -tube of uniform inner


cross-section contains some mercury. A
glycerine column of length 10 cm Mercury
(density = 1.3 gcm -3 ) is introduced in one of
Length of oil column is ....................... .
limbs and in other limb oil of density 0.8 gcm -3

Section 3 (Maximum Marks : 12)


l This section contains FOUR (04) questions.
l Each question has TWO (02) matching lists : Column-I and Column-II.
l FOUR options are given representing matching of elements from Column-I and Column-II. ONLY ONE of these four
options corresponds to a correct matching.
l For each question, choose the option corresponding to the correct matching.
l For each question, marks will be awarded according to the following marking scheme.
Full Marks : + 3 If ONLY the option corresponding to the correct matching is chosen.
Zero Marks : 0 If none of the options is chosen (i.e. the question is unanswered).
Negative Marks : - 1 In all other cases.

15. Consider an L-C-R series circuit match (a) A ® p, B ® q, C ® r, D ® s


quantity given in Column I with its’ variation (b) A ® s, B ® r, C ® q, D ® p
with frequency in Column II. (c) A ® q, B ® p, C ® s, D ® r
Column I Column II (d) A ® s, B ® q, C ® r, D ® p

A. Resistance p. 16. Match type of electromagnetic wave given in


Column I with the possible way of producing
it in Column II.

f Column I Column II

B. Inductive reactance q. A. Microwaves p. Molecular vibrations

B. X-rays q. Electron transitions

C. Ultraviolet r. Collision of electron


f
D. Infrared s. Gunn diode
C. Capacitive reactance r.
(a) A ® s, B ® r, C ® q, D ® p
(b) A ® p, B ® q, C ® r, D ® s
(c) A ® s, B ® r, C ® p, D ® q
f (d) A ® p, B ® r, C ® q, D ® s

D. Impedence s. 17. Match process of Column I with its change in


entropy per unit mass in Column II. A gas is
used for each of the process. In each process,
initial values are p1 , V1 and final values are
f
p2, V 2.

PREP CATALYSIS
190 JEE Advanced~Practice Set 7

18. Considering conduction process in solids,


liquids and gases, match Column I and
Column I Column II Column II.
A. V = constant p. æV ö Column I Column II
R loge ç 2 ÷
è V1 ø A. Conduction in solids p. Phenon vibration
æV ö at low temperature
B. p = constant q.
Cp loge ç 2 ÷
è V1 ø B. Conduction in solids q. Drift of electrons
at high temperature
C. T = constant r. æp ö
CV loge ç 2 ÷ C. Conduction in gases r. Molecular collisions
è p1 ø
D. Conduction in liquids s. Molecular
D. DQ = 0 s. 0 transformation

(a) A ® r, B ® q, C ® p, D ® s (a) A ® p, B ® q, C ® r, D ® s
(b) A ® s, B ® p, C ® q, D ® r (b) A ® s, B ® p, C ® q, D ® r
(c) A ® p, B ® q, C ® r, D ® s (c) A ® s, B ® q, C ® p, D ® s
(d) A ® r, B ® p, C ® q, D ® s (d) A ® p, B ® p q, C® r, D ® q, r

CHEMISTRY
Section 1 (Maximum Marks : 24) The major product(s) of the given sequence is
CH3
Instructions: Same as given in Physics.
(a) (CH3)3C(CH2)2NH CH3 (b) (CH3)3CCH2—N
19. Ammonia forms the complex ion [Cu(NH3 )4 ]2+ CH3
with copper ions in the alkaline solutions but
CH3 CH3
not in acidic solutions. The reason is
(a) in acidic solutions, hydration protects copper ions (c) (CH3)3CCH2 CH2N (d) (CH3)2CHCH2CH2N
(b) in acidic solutions, protons co-ordinate with CH3 CH3
ammonia molecules forming NH+4 ions and NH3
molecules are not available 22. Which product will be formed in the following
(c) in alkaline solutions, insoluble Cu(OH)2 is reaction?
precipitated which is soluble in excess of any alkali OH
(d) copper hydroxide is an amphoteric substance O
HO NaBH4
OH
20. Colourless salt (A), on heating with NaOH, H 2O
HO
release gas (B) that can also be obtained when OH
Mg3N 2 reacts with H 2O. When reaction of (A) CH2OH CH2OH
with NaOH get complete, solution obtained on
reaction with FeSO4 and conc. H 2SO4 gave a H OH H OH
brown coloured ring (C) between two layers. HO H HO H
(a) (b)
(A) on heating strongly forms (D) and (E). (E) H OH H OH
is a neutral oxide of a dibasic acid (F). Which H OH HO H
of the following statement is / are correct?
CH2OH CH2OH
(a) ‘A’ is a nitrate salt
(b) ‘F’ exist as cis and trans isomer
(c) The molecular formula of brown coloured ring (C) is CH2OH CH2OH
[Fe(H2O)5 NO]SO4
HO H H OH
(d) ‘D’ is a gas with molecular formula NH3
HO H HO H
(c) (d)
21. 3,3-dimethylbutanol reacts with H OH HO H
K 2Cr2O7 , H 2SO4 followed by heat gives (A). (A) H OH H OH
also undergoes the following transformations.
(i) LiAlH4, CH2OH CH2OH
SOCl2 (CH3)NH diethylether
A B (2 mol)
C D
H2O

PREP CATALYSIS
JEE Advanced~Practice Set 7 191

23. The two equilibrium, AB + B - and


-A +
After 15 minutes, a leak developed in the flask.
-
AB + B - AB 2-
are simultaneously On analysis of the gaseous mixture coming out
maintained in a solution with equilibrium initially, mole fraction of NO2 was found to be
constants, K 1 and K 2 respectively. The ratio of 0.66. The half-life for the decomposition
A+ to AB 2- in the solution is reaction is ……… .
(a) directly proportional to the concentration of B -
27. A solid surface has N 2 to the extent of 20% on
(b) inversely proportional to the concentration of B -
(c) directly proportional to the square of the
its sites. On heating these sites at 298 K and
concentration of B - at 0.001 atm, 2.46 cm3 of N 2 is collected. If
(d) inversely proportional to the square of density of the surface site is 6.023 ´ 1023 / cm 2
concentration of B - and surface area is 103 cm 2, then the number of
surface sites occupied by one N 2 molecule is
24. Nitrogen at 0ºC ……… ´108.

1.6 Hydrogen at 0ºC 28. Total number of stereocenters present in the


given molecule is/are ……… .
1.4 Carbon dioxide at 40ºC OH

1.2
N
1.0
Ideal gas, pV/nRT=1 OH
0.8
OH
0.6

0.4

0 29. An organic compound containing C, H, O, N


100

200

300

400

500

600

700

800

and Cl was analysed and 0.15 g of sample on


Pressure/atm combustion produced 0.138 g of CO2 and 0.054 g
Based on the isotherms given above, which of of H 2O. All the nitrogen in different 2.0 g
the following statements are correct? sample of compound was converted into NH3
(a) For CO2 and N2 , two effects compensate each other which was found to weigh 0.238 g. Finally, the
at a particular pressure where, z = 1. chlorine in a 0.125 g sample of compound was
(b) For H2 at 0° C, molecular attractive forces are weak converted to Cl- and by reacting with AgNO3 ,
and size effect dominates its behaviour resulting 0.251g AgCl was obtained. The molecular mass
that (z > 1). of emperical formula is ……… .
(c) For N2 at 0°C, attractive forces are large enough to
cause negative deviation upto 150 atm thus (z < 1) 30. 0.4 mole of monoatomic gas fills a 1 dm3
after which size effect dominates thus (z > 1.) container to a pressure of 1.0 kPa. It is
(d) For CO2 , intermolecular attraction is large even at expanded reversibly and adiabatically until a
40° C resulting (z < 1) even at high pressure. pressure of 0.1 kPa is reached. The work done
during the expansion is ……… .
Section 2 (Maximum Marks : 24) 31. At 25°C, l 0(H + ) = 3.4982 ´ 10-2S m 2 mol-1
Instructions: Same as given in Physics.
and l 0(OH - ) = 1.98 ´ 10-2S m 2 mol-1.
25. The total number of chemical species among
the given molecules that have dp- pp bonds in The pH of the solution is ……… .
their structure are …… . (Given : K (specific conductivity for H 2O)
NO3- , SO3 , XeO3 , H3PO4 , ClO4- , CO32- SO24- , = 5.7 ´ 10-6S m -1 )
(CN)2 , XeOF2 32. Zinc granules are added in excess to 500 mL of
26. Gaseous decomposition of N 2O5 follows a first 1.0 M nickel nitrate solution at 25° C until the
order kinetics. Pure N 2O5 gas is taken in a equilibrium is reached. If the standard
flask, sealed and finally heated to 100°C where reduction potentials of Zn2+ / Zn and Ni2+ / Ni
decomposition occurs as follows: are -0.75 and -0.24 Volt respectively. The
concentration of Ni2+ ions in solution at
N 2O5 ¾® 2NO2 + O2
1 equilibrium will x ´ 10-18 M . The value of x is
2 ……… .

PREP CATALYSIS
192 JEE Advanced~Practice Set 7

35. Match the reaction is given in List I with name


Section 3 (Maximum Marks : 12) of the reactions given in List II
Instructions: Same as given in Physics. List I List II
- + CCl 4
A. p. Simmons-
33. All the compounds given in List I react with PhCOOAg + Br2 ¾¾®
D Smith
water. Match the result of the respective reaction
PhBr + CO2 + AgBr
reactions with appropriate options given in -+
B. C H COOAg + I2 ¾® q. Borodine
List-II. 2 5
Hunsdiecker
C2H5 COOC2H5 + 2AgI
List I List II reaction
C. Zn/Cu
r. Birnbaum
A. (CH3 )2 SiCl 2 p. Hydrogen halide formation + CH2I2
-Simonini
B. XeF4 q. Redox reaction reaction
D. s. Carbene
C. Cl 2 r. Reacts with glass POCl3
–HOPOCl2 NH
D. VCl 5 s. Polymerisation NH
C Me
t. O2 formation
Me
The correct option is E. NH2 OH t. Bischler -
(a) A ® p, q, s; B ® p; C ® q, r, s; D ® t Napieralski
(b) A ® p, r, s; B ® q; C ® p, t; D ® p, q, r aq. NaHSO3 reaction
(c) A ® p, s; B ® p, q, r, t; C ® p, q, t; D ® p
(d) A ® p, q; B ® p, r, t; C ® p, s; D ® q
u. Bucherer
34. The desired product X can be prepared by reacting
reaction
the major product of the reactions given in List I
with one or more appropriate reagents in List II. The correct option is
(a) A ® q; B ® r; C ® p, s; D ® t; E ® u
(X) (b) A ® q; B ® r; C ® s; D ® t, u; E ® p
(c) A ® q; B ® s; C ® t; D ® r, u; E ® p
(d) A ® p; B ® q; C ® r; D ® s; E ® t, u
List-I List-II
P. – + 1. Heat 36. 1 L buffer of 1.0 M NaH 2PO4 and 1.0 M
COO Na
Na 2HPO4 are taken in two electrolytic vessel
CaO
separately. Electrolysis was carried using
+ NaOH ∆ platinum electrodes for 201 min with constant
current of 1.25 A. Assume that electrolysis of
Q. 2. Cu water takes place.
NH2
H3PO4 H 2PO-4 + H + , pK a = 215
- .
(i) NaNO2 + HCl Match the parameters given in Column I with
(ii) KI, ∆ their values in Column II and select the
answer from the codes given below:
R. 3. NaNO2 Column I Column II
(i) Conc. H2SO4 + Conc. HNO3
(ii) Sn/HCl (i) pH of the electrolytic cell p. 2.29
before current is passed.
S. NO2 4. HCl (ii) H+ ion formed (in terms of q. 0.156
molarity).
Zn, HCl
(iii) pH in the cathodic side r. 2.01
after current is passed.
(iv) pH in the anodic side after s. 3.51
5. H3PO2 current is passed.
The correct option is t. 2.15
(a) P ® 3; Q ® 4; R ® 2; S ® 1, 5 Codes
(b) P ® 1; Q ® 2; R ® 3; S ® 3, 4, 5 (i) (ii) (iii) (iv) (i) (ii) (iii) (iv)
(c) P ® 2; Q ® 3; R ® 4; S ® 1, 5 (a) p q r s (b) t q p r
(d) P ® 1; Q ® 2; R ® 3, 4; S ® 3, 4, 5 (c) q p r t (d) s r q p

PREP CATALYSIS
JEE Advanced~Practice Set 7 193

MATHEMATICS
Section 1 (Maximum Marks : 24) (a) t 2 = 2t1 (b) t 2 + 2t1 = 0
(c) t1 + 2t 2 = 0 (d) 6t12 = t 2 (t1 + 2t 2 )
Instructions: Same as given in Physics.
42. Let f : R ® R be a continuous function such
sin x cos x sin x that f ( x + y ) = f ( x ) + f ( y ) + 3xy( x + y ), " x , y Î R
37. Iff ( x ) = cos x sin x cos x . Then, which of the and f ¢ ( 0) = - 4. Then, which of the following
cos x sin x sin x statements is(are) TRUE?
(a) The curve y = f (x) is passes through (2, 0)
following is(are) TRUE? (b) The curve y = f (x) is passes through (-1, 3)
(a) f (x) = 0 has exactly 2 real solution in [0, p] (c) The number of real roots of the equation f (x) = 0 is 3
(b) f (x) = 0 has exactly 3 real solution in [0, p] (d) The domain of the f (x) is (- ¥, ¥)
f (x)
(c) Range of function is [- 2 , 2 ]
1 - sin 2 x Section 2 (Maximum Marks : 24)
f (x)
(d) Range of function is [- 3, 3] Instructions: Same as given in Physics.
sin2 x - 1
43. In a certain test, there are n questions. In this
38. Consider the circle x 2 + y 2 + 2x + 4 y - 20 = 0 test 2n - i , students gave wrong answers to
and x + y + 6x - 8 y + 10 = 0. Then, which of
2 2
atleast i questions, where i = 1, 2, 3, ¼ , n. If the
the following statements is(are) TRUE? total numbers of wrong answer given is 2047,
(a) The number of common tangent of circle is 2. then n equals
(b) The circles are orthogonal éxù éxù éxù 31
1/ 4 44. If 0 < x < 1000 and ê ú + ê ú + ê ú = x,
(c) Length of their common tangent is 5 æç ö÷
12
ë 2 û ë 3 û ë 5 û 30
è5 ø
where, [x ] is greatest integer function less than
(d) Length of their common chord is 5 3 / 2 or equal to x, then the number of possible
values of x is
39. Let f ( n ) denote the number of different ways in
which the positive integer n can be expressed 45. The number of points P ( x , y ) lying inside or on
as the sum of 1s and 2s . For example, f( 4) = 5, the circle x 2 + y 2 = 9 and satisfying the
equation tan4 x + cot4 x + 2 = 4 sin2 y is
Since, 4 = 2 + 2 = 2 + 1 + 1 = 1 + 2 + 1 = 1 + 1 + 2
= 1 + 1 + 1 + 1. Note that order of 1s and 2s is 46. A line L1 with direction ratios ( - 3, 2, 4) passes
important. Then, which of the following is(are) through the point A (7, 6, 2) and a line L2 with
TRUE? direction ratios (2, 1, 3) passes through the
(a) The value of f (6) is 13 point B (5, 3, 4). A line L3 with direction ratios
(b) The value of f (f (6)) is 377 (2, - 2, - 1) intersects L1 and L2 at C and D,
(c) The function f : N ® N is one-one and onto then the value of CD is equal to ....
(d) The function f : N + N is one-one and onto x 1

40. In DABC , P , Q , R are the feet of angle bisector


47. Let f ( x ) = ò-1 4 + t 2 dt and g( x ) = òx 4 + t 2 dt,

from the vertices A, B and C respectively to then the value of ( f × g)¢ ( 0) is ....
their opposite sides and DPQR is constructed. 48. Let A be the set of all 3 ´ 3 symmetric matrices
Then, which of the following is(are) TRUE?
all of whose entries are either 0 or 1. Five of
(a) If ÐBAC = 120° , then measure of ÐRPQ will be 60°
these entries are 1 and four of them are 0, then
(b) If AB = 7 units, BC = 8 units and AC = 5 units, then
28
the number of matrices in A is equal to ....
the side PQ will be units
3 ® ® ®
49. Let a , b and c be three non-coplanar unit
(c) If ÐBAC = 120° , then measure of ÐRPQ will be 90°
vectors such that the angle between every pair
(d) If AB = 7 units, BC = 8 units and AC = 5 units, then
84
of them is p / 3.
the side PQ will be units ® ® ® ® ® ® ®
3 If ( a ´ b ) + ( b ´ c ) = p a + q b + r c,
41. The chord AB of the parabola y 2 = 4ax cuts the where p, q are r are scalars, then the value of
axis of the parabola at C. If A = ( at12 , 2at1 ) and p2 + 2q 2 + r 2
B = ( at22 , 2at2 ) and AC : AB = 1 : 3. Then, which is equal to .....
q2
of the following is(are) TRUE?

PREP CATALYSIS
194 JEE Advanced~Practice Set 7

50. A reservoir contains 50 L of pure water The correct option is


initially. Salted water flows into the reservoir (a) P ® 2; Q ® 4; R ® 1; S ® 6
at the rate of 2 L per minute. It contains 2 gm (b) P ® 5; Q ® 2; R ® 2; S ® 1
of salt per litre. The mixture is kept uniform (c) P ® 6; Q ® 1; R ® 4; S ® 2
by stirring and if flows out of the bottom of the (d) P ® 4; Q ® 2; R ® 5; S ® 3
reservoir at the same rate. The time taken for
the quantity of salt in the reservoir to increase
53. Consider the D ABC, a circle is drawn touching
the in circle of the D ABC, side AC extended,
from 40 gm to 80 gm is a loge b min, then the
and side BC. Let the radius of this circle be r
value of a + b is equal to
similarly r and rc are radii of circles touching in
circle. BA extended and CB extended as well as
Section 3 (Maximum Marks : 12) AC and AB respectively r is the radius of in
Instructions: Same as given in Physics. circle and r1 , r2 and r3 are exradii of DABC.
sin 2x + 2 tan x List I List II
51. If ò dx
cos x + 6 cos x + 4
6 2 r
P. a is equal to 1. æ cot A + cot B + cot C ö
2
ç ÷
r è 2 2 2ø
1 a3 a4
= log a2 + + +C
a1 cos x cosa3 x
a4 Q. rb rc is equal to 2. cot 2 C / 2
2
r
where, C is constant of integration and R. ra rb rc is equal to 3. r1 + r2 + r3
a1 , a2 , a3 , a4 and N and a3 < a4. r3
List I List II S. ra rb + rb rc + 4. A B
cot 2 cot 2
P. 1. rc rb is equal to 2 2
a1 is equal to 6
Q. a4 is equal to 2. 4 The correct option is
R. a2 a3 is equal to 3. 14 (a) P ® 1; Q ® 3; R ® 2; S ® 4
(b) P ® 3; Q ® 2; R ® 4; S ® 1
S. a1 a3 is equal to 4. 12
(c) P ® 2; Q ® 4; R ® 1; S ® 3
5. 48 (d) P ® 1; Q ® 3; R ® 2; S ® 4
6m
The correct option is, 54. Consider the identity (1 + x )6m = S 6m
Cr × x r ,
(a) P ® 1; Q ® 3; R ® 2; S ® 4 r=0
(b) P ® 4; Q ® 1; R ® 2; S ® 5 By putting different values of x on both sides,
(c) P ® 3; Q ® 2; R ® 4; S ® 1 as we can get summation of several series
(d) P ® 2; Q ® 4; R ® 1; S ® 3 involving binomial coefficients. For example,
6m
52. Consider the polynomial 1 6m 1 æ 4ö
by putting x = , we get S 6mCr . r = ç ÷ .
f ( x ) = 1 + 2x + 3x 2 + 4x3 . Let S be the sum of all 3 r=0 3 è 3ø
distinct real roots of f ( x ) and let|S| = t List I List II
6m
List I List II P.
The value of S 6m
Cr 2 r /2
is 1. 0
r =0
P. f ¢¢ (x) is increasing in 1. æ- 3 , - 1 ö
ç ÷
è 4 2ø Q. 3m
2. 64m
The value of S (- 1)r 6m
C2r
r =0
Q. f ¢¢ (x) is decreasing in 2. æ- 1 , t ö
ç ÷ if m is even
è 4 ø
R. 6m
3. (3 + 2 2 )3m
R. The real number S lies on 3. æ 4, 5 ö The value of S 6m
Cr is
ç ÷ r =0
è 3ø
S. The value of 4. (- 1)m / 2 2 3m
S. The area bounded by the 4. æ - t, - 1 ö
ç ÷ 3m
r - 1 6m
curve y = f (x) and the lines è 4ø S (- 3) C2r - 1 is
r =1
x = 0, y = 0 and x = t
5. æ- 1 , t ö The correct option is
ç ÷
è 3 ø (a) P ® 2; Q ® 3; R ® 1; S ® 4
æ 3 , 3ö (b) P ® 4; Q ® 1; R ® 2; S ® 3
6.
ç ÷ (c) P ® 4; Q ® 3; R ® 1; S ® 2
è4 ø
(d) P ® 3; Q ® 4; R ® 2; S ® 1

PREP CATALYSIS
JEE Advanced~Practice Set 7 195

Answers
Paper 1
1. a,b,c,d 2. b, d 3. a, b 4. b, d 5. b, d 6. a, c 7. 2.00 8. 9.00 9. 2.00 10. 0.00
11. 2.00 12. 3.10 13. 5.00 14. 14.11 15. c 16. a 17. b 18. d 19. b 20. c
21. a, b, c 22. a 23. a, b, c 24. a, b, c 25. 5 26. 36.84 27. 230.09 28. 3.20 29. 35.13 30. 1480.20
31. 2.54 32. 84.42 33. c 34. c 35. d 36. b 37. a, c, d 38. b, c, d 39. a, b, c 40. c, d
41. a,b,c,d 42. b, c 43. 1010 44. 4 45. 80 46. 6 47. 7 48. 2 49. 2 50. 18
51. b 52. c 53. b 54. b

Paper 2
1. b, c 2. a, d 3. a, d 4. b, c, d 5. a, b, c 6. b, c 7. 3.00 8. 0.62 9. 5.82 10. 1.50
11. 76.00 12. 0.75 13. 6.53 14. 9.60 15. d 16. a 17. a 18. d 19. b 20. a, b, c
21. d 22. a 23. d 24. a,b,c,d 25. 6 26. 11.36 27. 2.02 28. 4 29. 143 30. 0.90
31. 6.98 32. 5.62 33. c 34. b 35. a 36. b 37. a, c 38. a,b,c,d 39. a, b, d 40. c, d
41. b, d 42. a, b, c 43. 11 44. 33 45. 8 46. 9 47. 0 48. 12 49. 4 50. 28
51. b 52. a 53. c 54. d

SCORE SHEET - Paper 1


Section No. of Marks from Marks from Marks Obtained
Correct Questions Correct Questions (A) Incorrect Questions (B) (A-B)
................ ............................... ............................... ............................... ...............................
................ ............................... ............................... ............................... ...............................
................ ............................... ............................... ............................... ...............................
Percentage Marks = Marks Obtain/Total Marks x 100

SCORE SHEET - Paper 2


Section No. of Marks from Marks from Marks Obtained
Correct Questions Correct Questions (A) Incorrect Questions (B) (A-B)
................ ............................... ............................... ............................... ...............................
................ ............................... ............................... ............................... ...............................
................ ............................... ............................... ............................... ...............................
Percentage Marks = Marks Obtain/Total Marks x 100
Note To expect your success marks in the test should be between 65%-70%.

PREP CATALYSIS
PREP CATALYSIS
JEE Advanced

PRACTICE SET 8 (With Solutions)

Duration : 3 Hours Max. Marks . 360

Paper 1
PHYSICS
Section 1 (Maximum Marks : 24)
l
This section contains SIX (06) questions.
l
Each question has FOUR options for correct answer(s). ONE OR MORE THAN ONE of these four option(s) is (are)
correct options(s).
l
For each question, choose the correct options(s) to answer the question.
l
Answer to each question will be evaluated according to the following marking scheme:
Full Marks : + 4 If only (all) the correct option(s) is (are) chosen.
Partial Marks : + 3 If all the four options are correct but ONLY three options are chosen.
Partial Marks : + 2 If three or more options are correct but ONLY two options are chosen, both of which are
correct options.
Partial Marks : + 1 If two or more options are correct but ONLY one option is chosen and it is a correct option.
Zero Marks : 0 If none of the options is chosen (i.e. the question is unanswered).
Negative Marks : - 2 In all other cases.
l
For example: If first, third and fourth are the ONLY three correct options for a question with second option being an
incorrect option; selecting only all the three correct options will result in + 4marks. Selecting only two of the three correct
options (e.g. the first and fourth options), without selecting any incorrect option (second option in this case), will result in
+2 marks. Selecting only one of the three correct options (either first or third or fourth option), without selecting any
incorrect option (second option in this case), will result in +1marks. Selecting any incorrect option(s) (second option in
this case), with or without selection of any correct option(s) will result in -2 marks.

1. A small pulley B over which a


string passes and attached to (a) horizontal velocity of A is equal to velocity of B
xv B
a mass A is of negligible (b) velocity of A in vertical direction is
diameter is pulled θ x2 + h2
h
B
horizontally over the ground vB v B cos 2 q
(c) rate of change of q angle is
as shown in the figure. At an y h
x
instant, if velocity of B is v B , (d) velocity of A (total) will be v A2 + v B2 , where v A is upward
A
then (all surfaces are
velocity of A
smooth)

PREP CATALYSIS
JEE Advanced~Practice Set 8 197

2. 2 mol of He is mixed with 2 mol of H 2 in a closed ON the magnetic field, the ring is just about to
adiabatic container. Initially, the mixture occupies rotate about vertical axis through its centre. Then,
3 L at 27°C. The volume is suddenly decreased to (a) friction coefficient between the ring and the surface is
2 B0 RQ
3/2 L.
mg
Now, choose the correct options. (b) friction coefficient between the ring and the surface is
(When H 2 and He can be treated as ideal gases) B 0 RQ
(a) g for mixture is 3/2 mg
(b) Final temperature = 300 2 K (c) if magnetic field is switched OFF after 4 s, then the
(c) Value of g will change with higher temperature angle rotated by the ring before coming to stop after
(d) Work done in compression is totally converted into 2B Q
switching OFF magnetic field is 0
internal energy m
(d) if magnetic field is switched OFF after 4 s, then the
3. A man pushes a cylinder of mass M with the help angle rotated by the ring before coming to stop after
of a plank of mass m as shown in the figure. There BQ
switching OFF the magnetic field is 0
is no slipping at any contact. The horizontal m
component of the force applied by the man is F0 ,
then 5. A point source of light is placed at a distance of 25
cm from the centre of a glass sphere (n = 15
. ) of
m
F0 radius 10 cm. Then, choose the correct options.
(a) Image formed by first surface is real
(b) Image formed by first surface is at a distance of 100 cm
from centre of sphere
M (c) Image formed by second surface is real
(d) Image formed by second surface is at a distance of
28.5 cm from the centre of sphere
(a) acceleration of the cylinder and the plank are equal
8F0 6. Two blocks are placed on a fixed rough inclined
(b) acceleration of the plank is
3M + 8m plane. The coefficient of frictions for two blocks are
4F0 m1 and m 2 as shown in the figure. The normal
(c) acceleration of the cylinder will be
3M + 8m reaction between the blocks will be zero, if
8F0
(d) acceleration of plank and the cylinder will be
8M + 3m
m
5F0 µ2
and , respectively. M
3M + 8m µ1

4. A non-conducting ring of mass m and radius R has θ


a charge Q uniformly distributed over its Fixed
circumference. The ring is placed on a rough (a) M > m
horizontal surface such that axis of the ring is (b) m > M
parallel to the surface. A vertical magnetic field
(c) m1 > m 2
B = B 0 t 2 X is switched ON. After 2 s from switching (d) m 2 > m1

PREP CATALYSIS
198 JEE Advanced~Practice Set 8

Section 2 (Maximum Marks : 24)


l This section contains EIGHT (08) questions. The answer to each question is a NUMERICAL VALUE.
l Four each question, enter the correct numerical value (in decimal notation, truncated/rounded-off to the second
decimal place; e.g. 6.25, 7.00.- 0.33, -.30, 30.27, -127.30) using the mouse and the on-screen virtual numeric
keypad in the place designated to enter the answer.
l Answer to each question will be evaluated according to the following marking scheme:
Full Marks : + 3 If ONLY the correct numerical value is entered as answer.
Zero Marks : 0 In all other cases.

The temperature coefficient of resistance of the coil


Y is x ´ 10 -2 . Find the value of x.
7. The moment of inertia about X-axis of the solid
spherical segment of mass M (see figure) is 10. Two concentric thin
53 MR 2 conducting shells of radii a
. Then, the value of r is and b are as shown in the
40 X a
figure. The region inside the
shells is filled with a medium r b
of resistivity, r = 2 W-m.
O The conducting sphere are
x given equal and opposite
charges. The electric flux through the surface of a
spherical region (indicated by dotted region) of
radius r is f = 10 N-m 2 / C. Find the current crossing
R/2 R/2 the dotted spherical surface of radius r.
11. Find the quantum number n corresponding to
8. Two blocks A and B of B
7 kg excited state of He + ion, if on de-excitation to the
masses 14 kg and 7 kg ground state the ion emits two photons in succession
respectively, connected by a with wavelengths 110.0 nm and 30.0 nm. The
flexible inextensible cord
A ionisation energy of H-atom is 13.6 eV.
rest on a smooth plane
inclined at 45° with the 14 kg 12. A circular disc of a ω
horizontal as shown in diameter d is slowly
figure. 45° rotated in a liquid of
large viscosity m at a Liquid h
When the masses are released, what will be the small distance h from a
tension T in the cord? Assume, the coefficient of fixed surface. An expression for torque T necessary
friction between the plane and block A is 0.25 and m p d 4w
to maintain an angular velocity w is . Find
between the plane and block B is 3/8. Xh
the value of X.
9. Figure shows a meter bridge wire AC has a
uniform cross-section. The length of wire AC is 100 13. A rolling body is initially F
cm. X is a standard resistor of 4 W and Y is a coil. placed on a rough surface.
When Y is immersed in melting ice the null point A horizontal force is r
m
is at 40 cm from point A . When the coil Y is heated applied on the uppermost
to 100°C, a 12 W resistor has to be connected in point. Calculate the friction
parallel with Y in order to keep the bridge balanced force, when force F is equal Rough surface
to 14 N.
at the same point.
X=4Ω Y 14. A set of parallel f1 = +15 cm f2 = – 20 cm
rays incident on Convex
the bi-convex lens. mirror
If after two
G refractions and one
reflection final
A C image formed at X
D
l 15 cm left of lens,
X
then the value of is
9

PREP CATALYSIS
JEE Advanced~Practice Set 8 199

Section 3 (Maximum Marks : 12)


l This section contains TWO (02) paragraphs. Based on each paragraph, there are TWO(02) questions.
l Each question has FOUR options. ONLY ONE of these four options corresponds to the correct answer.
l Four each question, choose the option corresponding to the correct answer.
l Answer to each question will be evaluated according to the following marking scheme:
Full Marks : + 3 If ONLY the correct option is chosen.
Zero Marks : 0 If none of the options is chosen (i.e. the question is unanswered).
Negative Marks : - 1 In all other cases.

Passage X Passage A
A conducting loop (as shown in the figure) is pulled In a Searle’s experiment, following are the observations.
outwards with a constant speed as shown in the figure. Least count of micrometer of Searle’s apparatus = 0.001 cm.
Magnetic field into the plane of
Pitch of screw gauge = 0.1 cm.
paper and plane of loop is perpendicular to B Number of divisions on circular scale = 100.
There is no zero error in screw gauge.
× × × × × × × × A ×
While measuring diameter of wire, the linear scale reads
× × × × × × × × × 0 mm and 50th division on circular scale coincides with
Semicircular R 2R v the reference line.
part × × × × × × × × × Scale used for measuring length has a least count of 0.1 cm.
× × × × × × × × B× Length of wire recorded by scale is 110 cm.
Following load(w) versus extension (l ) graph is made using
15. The induced emf between the points A and B, just
experimental observations.
after the motion start is
(a) vB(2pr ) (b) vBR l(×10–3m)
vB2 R
(c) (d) 2v BR
2
1.25
16. In the given passage, if the loop is just pushed
with speed v 0 and left, then the correct graph
between v and t is
w(N)
v0 v0 40 50 60

17. Young’s modulus of the wire is


(a) v (b) v (a) 25 ´ 1011 N / m2
(b) 2.50 ´ 1011 N / m2
O t O t (c) 0.56 ´ 1011 N / m2
v0 v0 (d) 5.60 ´ 1011 N / m2

18. Maximum percentage error in the measurement of


(c) v (d) v Young’s modulus is around
(a) 10% (b) 1%
(c) 5% (d) 8%
O t O t

PREP CATALYSIS
CHEMISTRY
NO2
Section 1 (Maximum Marks : 24)
Instructions : Same as given in Physics Cl2 aq. NaOH HCl
(b)
19. Compound ‘X’ on reduction with LiAlH 4 gives a AlCl3 600°C

hydride ‘Y’ containing 21.72% hydrogen alongwith


NO2
other products. The compound ‘Y’ reacts with air
explosively resulting in boron trioxide. The structure
(i) HNO3/H2SO4 (i) HNO2, 10°C
of Y is (c)
(ii) NH4HS (ii) H2O
(a) BH-4 (b) B 2H6 (c) B 4H10 (d) B10H14
20. A freshly prepared aqueous solution of NO2
Pd(NH 3)2 Cl 2 does not conduct electricity. It
suggests that SO3 NaOH (s) HCl
(d) Fusion
H2SO4
(a) the structure of the compound involves covalent
bonding only
(b) the chlorine atoms must be in coordination sphere 24. When intermolecular forces in a real gas are taken
(c) the van’t-Hoff factor of the compound would be unity zero, variation of pV with pressure p at a given
(d) on adding excess aqueous AgNO3 to 0.1L of 0.1M temperature is shown by following isotherm.
solution of the compound, 0.02 mole of AgCl would be
obtained
2.6
21. Which of the following statement(s) is/are correct 2.0
about group 2 elements salt?
(bardm3mol–1)

(a) Thermal stability of sulphates and carbonates increases


from Be 2+ to Ba 2+
pV

(b) Mg(NO 3 )2 and LiNO 3 differ in the nature of


decomposition products
(c) The lattice energies of group 2 salts are much higher 0 2 5 p(bar)
than those of group 1 salts
(d) Hydration energies of group 2 salts are higher than If TC = 100 K for this gas, then the value of pC is
those of group 1 salts (a) 41. 5 bar (b) 13.8 bar (c) 124.5 bar (d) 55.3 bar
22. In the following reaction sequence, the correct
structure(s) of X is/are Section 2 (Maximum Marks : 24)
(i) Br2/CCl4 Instructions : Same as given in Physics
(ii) NaNH2
(X) 25. The total number of compounds that contain
(iii) HgSO4/H2SO4
(iv) NaOD/D2O (excess)
(SiO 4-
4 ) anion among the molecules given below is
…… .
O Orthosilicate, pyrosilicate, pyroxene, amphibole,
(a)
mica, zeolite, single chain silicate, double chain
CH2OD (b) C CD3
silicate, sheet like silicate.
26. A sample of sugar D-ribose (C5H10O 5) weighing
0.727g was placed in a calorimeter and then
(c) COOD (d) CH2D ignited in the presence of excess of oxygen. The
temperature rose by 0.91K.
In a separate experiment in the same calorimeter,
23. The reaction(s) leading to the formation of the combustion of 0.825 g of benzoic acid, for
meta-nitrophenol are
which the internal energy of combustion is -3251
NO2 kJ/mol, gave a temperature rise of 1.94K. The
Electrolytic
enthalpy of formation of D-ribose will be ……… ´
(a)
reduction Rearranges 10 6 J.
[Given : DH°f CO2 = -394 kJ/ mol,
(Strongly acidic
medium)
H 2O( l ) = -286 kJ/ mol]

PREP CATALYSIS
JEE Advanced~Practice Set 8 201

27. A drop of 1.5 M HCl is made to fall on 1.0mm thick water in the right column is 77.8 mm in excess of
aluminium sheet and a hole in the sheet is the height of the solution in the left column at
developed. The radius of hole is x cm, if the density 298K.
of aluminium is 2.7 g/mL and 6.0 drops of HCl are The molar mass of haemoglobin is …… .
equivalent to 1.0 mL. The value of x is …… .
[Given : Atomic mass of Al is 27.] Section 3 (Maximum Marks : 12)
28. Tritium (13 T) isa radioactive isotope of H and a Instructions : Same as given in Physics
b - emitter. It combines with fluorine to form a Paragraph X
weak acid TF which ionises to give T + . The Hydrocarbon ‘A’ has molecular formula C 8 H12 . ‘A’ on
equivalent of pH of a freshly prepared dilute treatment with aqueous NaNH 2 followed by reaction
aqueous solution of TF is 1.8 which freezes at with CH 3 Br gives B (C 9 H14 ). B on oxidation with acidic
-0 .383° C. If 500 mL of freshly prepared solution solution of KMnO 4 gives C (C 7 H12 O 2 ) as one of the
were allowed to stand for 23.4 years, then the product. C on heating with sodalime followed by
value of charge carried by b-particles emitted by chlorination of product in presence of light yielded a
tritium in Faraday will be …… . single monochloro derivative C 6 H11 Cl.
[Given : K f for H 2O =186
. , t1 / 2 (T) = 117
. years]
33. The compound A is
29. At high temperature CO2 dissociates according to C C CH3
the equation. (a) C C (b)

2CO2(g) = 2CO(g ) + O (g )
2

At 1.0 atm pressure the percentage of oxygen at


equilibrium is 2.05 ´ 10 -5 at 1000K and at 1400K is CH2 C CH
. ´10 -2 . Assuming the enthalpy change of the
127 (c) (d)
reaction to be independent of temperature, the
standard entropy change at 1000 K will be …… . 34. Which of the statement is correct regarding B?
30. It is said that the finely divided catalyst has greater (a) B on treatment with Lindlar’s catalyst followed by
catalytic activity than the solid in the compact reaction with alkaline KMnO 4 will gave a meso diol
form. For a catalysed reaction, a total surface area (b) B on treatment with 1.0 equivalent of Br2 will give a
mixture of geometrical isomers in comparable amount
of 768 cm 2 is required for the adsorption of
(c) B on treatment with Na/NH3 (l ) followed by addition of Br2
gaseous reactants. The number of splits that will give a racemic mixture of dibromides
should be made of cube of exactly 1cm in length is (d) B on treatment with HgSO 4 (aq ) / H2SO 4 will give a
…… . methyl ketone as major product
31. Heat of combustion of sucrose under standard Paragraph A
state is -5640 kJ mol-1 . A healthy person takes 20
breaths per minute, and each breath consumer 480 Toluene(A) in presence of Cl 2 /Fe gives q and p-isomers.
mL of air. Inhaled and exhaled air contains 20% o-isomer gives (F) in presence of Mg/THF followed by
and 15% oxygen by volume. If oxygen retained in CO 2 / H 3 O + . Following transformation also occurs as
the body is used in the combustion of sucrose, follows :
then heat evolved as a result of combustion of Cl2/Fe (i) Mg/THF
PhMe (o-isomer) +( p-isomer)
sucrose is …… . (A ) (B) (C) (ii) CO2/H3O+

(i) Conc.H2SO4
32. Consider the (ii) Cl2/Fe (i) Mg/THF
following (ii) CO2/H3O+ (D)
Semiper- (iii) H3O+, ∆
arrangement in meable Cl2/Fe
77.8 mm Conc.H2SO4 [O]
which a solution membrane (F ) (G )
KMnO4, ∆
(H )
(E)
containing 20 g of Br2/Fe
haemoglobin in 1L of (I) (J )
Major
the solution is placed KMnO4, ∆
in the right Water Solution
compartment and (K )
pure water is placed 2 Br2/FeBr3 (i) Mg/ether KMnO4
(L ) (M ) (N )
in the left (i) CO2/H3O+ ∆

compartment. At equilibrium, the height of the

PREP CATALYSIS
202 JEE Advanced~Practice Set 8

35. The structural formula of compound K is 36. The structural formula of compound H is
COOH COOH Me Me
Br Br Br Br

(a) (b) (a) (b)

NO2 CH3

NO2 COOH

COOH COOH COOH COOH


NO2 Br COOH COOH

(c) (d) (c) (d)

Br NO2
SO3H CH3

MATHEMATICS
z - 1- i ö -p
(a) argæç ÷= where i = - 1
Section 1 (Maximum Marks : 24) è z ø 4
z-2
Instructions : Same as given in Physics (b) is purely imaginary number
z
[ x] + 1 é 5ö æ1 ù z -2
37. f (x) = for f : ê0 , ÷ ® ç , 3 ú, where [×] (c) is purely real number
{ x} + 1 ë 2 ø è2 û z
2
represents the greatest integer function and {×} (d) if arg(z) = q, and q is acute, then 1 - = i tanq
z
represents the fractional part of x , then which of
the following is true? 41. Let r be a position vector of a variable point in
(a) f( x) is surjective non-differentiable function cartesian OXY plane such that r . (10 $j - 8 $i - r) = 40
(b) f( x) is injective discontinuous function and l = max {|r + 2 i$ - 3 $j|2 },
æ ö
(c) min ç lim f( x), lim f( x)÷ = f(1 ) m = min {|r + 2 $i - 3 $j|2 }. A tangent line is drawn to
è x ® 1- x ® 1+ ø
8
(d) maximum point of discontinuity = 2 the curve Y = 2 at point A with abscissa 2. The
x
38. If in a D ABC, cos ( A - C) cos B + cos 2B = 0, then
line cuts the X-axis at a point B. Then, which of the
which of the following options are correct? following is(are) TRUE?
(a) sin2 A + sin2 C = 2 sin2 B (b) a2 , b 2 and c 2 are in AP (a) l is equal to 2 2 + 1
(c) a2 , b 2 and c 2 are in GP (b) AB × OB is equal to 3
(d) sin A sin C = sin2 B (c) l + m is equal to 18
(d) AB × OB is equal to 2
39. Two real numbers x and y are selected at random.
Given that 0 £ x £ 1, 0 £ y £ 1. Let A be the event 42. Let a function f (x) be continuous for all x Î R,
that y 2 £ x , B be the event that x 2 £ y , then which satisfies the relation f (x + y) - f (x - y) = 4 xy
"x , y Î(R) and f(0) = 0 . Then, which of the following
(a) A and B are exhaustive events
(b) P( A Ç B) = 1/ 3
is (are) TRUE?
(c) A and B are mutually exclusive (a) f( x) is bounded
(b) f( x) + f æç ö÷ = f æç x +
(d) A and B are independent events 1 1ö
÷+2
è xø è xø
40. For a non-zero complex number z, let arg(z)
(c) f( x) + f æç ö÷ = f æç x -
1 1ö
÷+2
denotes the principle argument with è xø è xø
- p < arg(z) < p and|z - 1| = 1. Then, which of the
(d) f( x) = x2
following statements is (are) TRUE?

PREP CATALYSIS
JEE Advanced~Practice Set 8 203

Section 2 (Maximum Marks : 24) Section 3 (Maximum Marks : 12)


Instructions : Same as given in Physics Instructions : Same as given in Physics
æ æ 1 ö æ
ç 3 öö
÷ Paragraph X
ç ç ÷ ç log ÷÷
43. The value of ç(81)è log 5 9 ø + (3) è 6 ø
3
÷ A ball moving around the circle
ç ÷ 14 x 2 + 14y 2 + 216x - 69y + 432 = 0 in clockwise direction.
è ø
Leaves it tangentially at the point P( -3, 6). After getting
æ æ
ç
2 ö
÷ ö reflected from a straight line L = 0 it passes through the
ç è log 25 7 ø log 25 6 ÷
ç ( 7 ) - (
125) ÷ centre of the circle.
è ø The perpendicular distance of this straight line L = 0 from
the point P is 11 130. You can assume that the angle of
44. Let u be a vector on rectangular cartesian system 13
with angle 60° with X-axis. Suppose that|u - $i| is incidence is equal to the angle of reflection.
geometric mean of|u| and|u - 2 $i| where, $i is the 51. The equation of tangent to the circle at P is
unit vector along X-axis. Then, the value of (a) 2 x - y + 12 = 0
( 2 + 1)|u| is (b) 4 x + 3 y - 6 = 0
(c) 3 x - 2 y + 21 = 0
45. The number of solution of the equation
(d) 2 x + 5 y - 24 = 0
x p
ò-2|cos x| dx = 0,0 < x < 2 will be
52. If angle between the tangent at P and the line
through P perpendicular to the line L = 0 is q, then
46. Suppose a matrix A satisfies A - 5A + 7I = 0, if
2
tanq is
A 6 = aA + bI, then the value of a - b is equal to ..... (a)
2
(b)
3
11 11
47. The are 10 girls and 8 boys in a classroom 4
(c) (d) None of the above
including Ravi, Rani and Radha. A list of speakers 11
consisting of 8 girls and 6 boys has to prepared.
Ravi refuses to speak if Rani is speaker. Rani Paragraph A
refuses to speak if Radha is a speaker. The total If a1 , a 2 , a 3 , a 4 ¼ an - 1 , an are different numbers, then AM
number list can be prepared is N. Then, N is equal a + a2
to of two numbers will be 1 , AM of three numbers
2
48. Let Pi and Pi ¢ be the feet of the perpendicular a + a2 + a3
= 1 , similarly GM of two numbers is a1a 2 ,
drawn from foci S, S¢ on a tangent Ti to an ellipse 3
whose length of semi-major axis is 20. If GM of three numbers will be 3 a1 a 2 a 3 and GM of
10
S (SPi)(S¢ Pi ¢) = 2560. Then, eccentricity of ellipse is
i =1
n numbers will be n a1a 2 a 3 a 4 ¼ an .

equal to 53. Let a + b + c + d = 8 and abcd = 16


p where, a , b, c , d are positive numbers, then value of
49. If tan is the root of equation x 4 - ax 2 + 1 = 0 , a + b is
8
(a) 2 (b) 4
then the value of a is equal to (c) 7 (d) 6
50. Let f : R ® R be a twice differentiable function 54. The minimum value of the sum of real numbers
satisfying f ¢ ¢ - 5f ¢ (x) + 6 f (x) ³ 0 , " x ³ 0 if a -5 , a -4 , 3a -3 , 1, a 8 and a 10 with a > 0 is
f (0) = 1, f ¢ (0) = 0 if f (x) satisfies
f (x) ³ ah(bx) - bh(ax), "x ³ 0, then the value of (a) 4 (b) 6
(a + b) h (0) is equal to ...... (c) 8 (d) 10

PREP CATALYSIS
Paper 2
PHYSICS

Section 1 (Maximum Marks : 24)


l This section contains SIX (06) questions.
l Each question has FOUR options for correct answer(s). ONE OR MORE THAN ONE of these four option(s) is (are)
correct options(s).
l For each question, choose the correct options(s) to answer the question.
l Answer to each question will be evaluated according to the following marking scheme:
Full Marks : + 4 If only (all) the correct option(s) is (are) chosen.
Partial Marks : + 3 If all the four options are correct but ONLY three options are chosen.
Partial Marks : + 2 If three or more options are correct but ONLY two options are chosen, both of which are
correct options.
Partial Marks : + 1 If two or more options are correct but ONLY one option is chosen and it is a correct option.
Zero Marks : 0 If none of the options is chosen (i.e. the question is unanswered).
Negative Marks : - 2 In all other cases.
l For example: If first, third and fourth are the ONLY three correct options for a question with second option being an
incorrect option; selecting only all the three correct options will result in + 4marks. Selecting only two of the three correct
options (e.g. the first and fourth options), without selecting any incorrect option (second option in this case), will result in
+2 marks. Selecting only one of the three correct options (either first or third or fourth option), without selecting any
incorrect option (second option in this case), will result in +1marks. Selecting any incorrect option(s) (second option in
this case), with or without selection of any correct option(s) will result in -2 marks.

1. Two capacitors of 3mF and 6 mF are in series in a


A0
circuit with a resistance of 0 .5 ´ 10 6 W. A battery is (a) Decay constant of the radioactive sample is
2t0
connected across arrangement which provides a
(b) Number of radioactive nuclei at the start of the
constant supply of 240 V. t
experiment was 0 e A 0
Now, choose the correct options. A0
(a) Time constant of arrangement is 1 s t A
(c) Number of radioactive nuclei after one half-life is 0 e 0
(b) Potential drops across both capacitors are same A0
(c) At t = 1s, potential drop across capacitor of 3mF (d) Half-life of the radioactive sample is t 0
is 101.1 V
(d) Charge on capacitor of 6 mF at t = 1s is around 3. Consider three concentric B
shells A , B and C having
303 mC
charge densities of s, - s rb
2. A radioactive nuclei decays with a decay constant and s respectively, where ra C
A
dN dN s = 8 .6 nCm -2. Also,
l. Let A = log , where is the number of
dt dt radii of the shells
radioactive decays taking place per second. respectively, are ra = 1 m,
rb = 15
. m and rc = 2 m rc
A
(Take, e 0 = 8 .6 ´ 10 -12 SI
unit)
A0
Now, choose the correct options.
A0/2
(a) Potential on surface of A = 1.5 ´ 103 V
O t (s) (b) Potential on surface of B = 1.16 ´ 103 V
Suppose the experiment starts at t = 0 and the (c) Potential on surface of C = 1.375 ´ 103 V
curve between A and t is shown in the adjacent (d) To make potential on surface of A and C equal, radius of
figure. Choose the correct statement(s) from the C must be 2.5 m
following.

PREP CATALYSIS
JEE Advanced~Practice Set 8 205

4. A light beam (l = 6600 Å) is incident over face AB A


k
B v0
of prism ABC. Given, angle A = 30 ° and angle of m 2m
incidence of light beam = 60 °. On face AC of prism,
a thin film of a transparent medium (n = 2.2) is 2m
coated. (a) Time period of SHM is 2 p
3k
A (b) Maximum speed of A w.r.t. ground is 4v 0 / 3
(c) Kinetic energy in the frame of CM is mv 02 / 3
30°
Transparent (d) Maximum speed of A w.r.t. ground is 2 v 0 / 3
60° coating
6. A wire of length 4.8 m and mass 0.06 kg is joined
with another wire of length 2.56 m and mass 0.2
kg. This composite wire is taut under a tension of
80 N. A sinusoidal wave pulse of amplitude 3.5 cm
is sent along the wire from one of it’s end A as
B Prism C
shown in the figure.
Refractive index of material of prism is 3.
3.5
Now, choose the correct options. A cm B C
(a) Angle of emergence of light beam is 30°
(b) Angle of emergence of light beam is 0°
AB=4.8m BC=2.56m
(c) Intensity of emergent ray is maximum when film
Mass (AB)=0.06 kg Mass (BC) = 0.2 kg
thickness is 3300Å
(d) Intensity of emergent ray is maximum when film Assuming that no power is dissipated.
thickness is 1500 Å Now, choose the correct options.
5. Two blocks spring system is placed on a smooth (a) Time taken by wave pulse to reach end C = 0.07 s
horizontal surface as shown in figure. If 2m mass (b) Time taken by wave pulse to reach C = 0.14 s
is given, velocity v 0 towards right, then find correct (c) Amplitude of wave in region BC is 1.5 cm
statements about motion. (d) Amplitude of wave in region BC is 2.0 cm

Section 2 (Maximum Marks : 24)


l
This section contains EIGHT (08) questions. The answer to each question is a NUMERICAL VALUE.
l
Four each question, enter the correct numerical value (in decimal notation, truncated/rounded-off to the second
decimal place; e.g. 6.25, 7.00.- 0.33, -.30, 30.27, -127.30) using the mouse and the on-screen virtual numeric
keypad in the place designated to enter the answer.
l
Answer to each question will be evaluated according to the following marking scheme:
Full Marks : + 3 If ONLY the correct numerical value is entered as answer.
Zero Marks : 0 In all other cases.

7. A uniform vertical cylinder a


of friction for AB is 0.3. Find the compression of
l =80 cm
of cross-sectional area a and other spring (in cm)
length 80 cm floats, 90% l l/10
submerged in a liquid inside k k
1 kg 2m
a tank with cross-sectional
area four times that of A µ = 0.3 B
µ=0 µ=0
cylinder. When cylinder is
pushed down gently and 9. The prism spectrum is spread out more at violet
released, it performs SHM. end than at the red end. Angular dispersion is
The maximum possible dd
4a (Cross-sectional area) defined as f = , if deviation of small prism d is
amplitude (in cm) for this SHM is dl
8. Massless springs, each with k = 1350 Nm - 1 are defined as d = (m - 1) A 0 and wavelength is defined
B
attached to the left and right walls as shown in the as m = A + 2 , where A and B are constants and A 0
figure. A 1 kg block is initially held against the l
1
left-hand spring, compressing the spring by 0.1 m. is prism angle, then dispersion f µ N . Find the
The block is released to move. The floor is l
frictionless except for the section AB and coefficient value of N.

PREP CATALYSIS
206 JEE Advanced~Practice Set 8

10. In an ammeter, 0.5% of main current passes Before After


through galvanometer coil. If resistance of m v
galvanometer coil is 12 W, then find resistance of X a
θ
the ammeter. u
ω
l
11. A hydrogen like atom (atomic number Z) is in a
higher excited state of quantum number n. The M
excited atom can make a transition to the first
excited state by successively emitting two photons 13. The electric field on two sides of a charged plate is
of energy 10.2 eV and 17.0 eV, respectively. shown in the figure. If the charge density on the
Alternatively, the atom from the same excited state plate is ne 0 , then n is
can make a transition to the second excited by +
+
successively emitting two photons of energies E1=8 V/m +
+ E2=12 V/m
4.25 eV and 5.95 eV, respectively. Determine the +
+
value of n. +
+
(Take, ionisation energy of H-atom = 13.6 eV)
14. A long circular tube of length 10 m and
12. A long thin cord of length l = 90 cm and mass radius 0.3 m carries a current I along its
600 g, is rotating clockwise on a frictionless curved surface as shown in figure. A
horizontal table about its centre of mass, which is wire-loop of resistance 0.005 W and of
stationary. A small ball of mass 300 g is placed on radius 0.1 m is placed inside the tube
the table at distance x from the centre of mass of with its axis coinciding with the axis of
the rod collides elastically with the ball. After the the tube. The current varies as ⇒
collision, the rod is translating to the left without I = I 0 cos 300 t, where I 0 is constant. If I
rotating and the ball moves to the right. What is the magnetic moment of the loop is
the distance ( x / 5 ) (in cm)? N m 0I 0 sin(300 t), then N is ……… .

Section 3 (Maximum Marks : 12)


l
This section contains FOUR (04) questions.
l
Each question has TWO (02) matching lists : Column-I and Column-II.
l
FOUR options are given representing matching of elements from Column-I and Column-II. ONLY ONE of these four
options corresponds to a correct matching.
l
For each question, choose the option corresponding to the correct matching.
l
For each question, marks will be awarded according to the following marking scheme.
Full Marks : + 3 If ONLY the option corresponding to the correct matching is chosen.
Zero Marks : 0 If none of the options is chosen (i.e. the question is unanswered).
Negative Marks : - 1 In all other cases.

15. A very small circular loop of radius a is initially


coplanar and concentric with a much larger C. Induced emf in the large r. m 20wp 2a 4 I 2
sin2 wt
circular loop of radius b (>> a). A constant current I loop due to current in 4 b 2R
is passed in the large loop which is kept fixed in smaller loop as a
space and the small loop is rotated with angular function of time
velocity w about a diameter. The resistance of the D. Flux as a function of s. m 20p 2a 4 w 2I
cos wt
small loop is R and the inductance is negligible. time 4b 2
Match the Column I with Column II and mark t. m 0p a 2I
cos wt
the correct option from the codes given below. 2b
Column I Column II u. m 0wp 2a 4 I 2
sin2 wt
A. Current in the small loop p. m 0pa I
2 8 b 2R
cos wt
as a function of time 4b Codes
B. Torque exerted on the q. m 0wpa 2I sin wt A B C D A B C D
small loop to rotate it 2 bR (a) q r q t (b) t r q q
(c) q u s p (d) t u p s

PREP CATALYSIS
JEE Advanced~Practice Set 8 207

16. In the given arrangement, the square loop of area


10 cm 2 rotates with an angular velocity w about its C
diagonal. The loop is connected to a inductance of
L = 100 mH with a capacitance of 10 mF in series.
The lead wires have a net resistance of 10 W .
(Take, B = 0 .1 T and w = 63 rad/s). Match the D B
Column I with Column II and mark the correct
option from the codes given below.
u
× × × × × × A
× × × × × ×
× × × × × ×
ω
× × × × × × Column I Column II
L
× × × × × × A. Total energy p. 5
mgr
2
C B. Kinetic energy q. 3
mgr
2
Column I Column II C. Potential energy r. mgr
A. rms current p. 4 ´ 10 -4 A D. Angular momentum s. 3m2gr 3
B. Energy dissipated in 50 s q. 5.6 ´ 10 -4 A
Codes
C. Frequency of rotation of the coil r. 8.12 ´ 10 -5 J
AB CD AB CD
D. Peak value of the current s. 8.14 ´ 10 -6 J (a) p q r s (b) r p q s
t. 31.6 rad/s (c) s p q r (d) p q s r
u. 31.9 rad/s 18. Match gases given in Column I with degree of
Codes freedom of their molecules at room temperature in
A B C D Column II and mark the correct option from the
(a) p r t q codes given below.
(b) p s u q
Column I Column II
(c) q u s p
A. He, Ne p. 7
(d) q r t q
B. H 2, O 2 q. 6
17. Consider a particle moving in a vertical loop. C. H2O, SO 2 r. 5
Let u = velocity imparted to particle at point A and D. CO 2 , BeCl 2 s. 3
r = radius of circle. Now, match the Column I with Codes
Column II, considering particle at point B and AB C D AB CD
mark the correct option from the codes given (a) s r p q (b) s r q p
below. (c) s p q r (d) s q p r

CHEMISTRY
Section 1 (Maximum Marks : 24) (a) [Co(NH3 )5 NO2 ]Cl 2 , ionisation isomerism
Instructions : Same as given in Physics (b) [Co(NH3 )5 NO2 ]Cl 2 , linkage isomerism
(c) [Co(NH3 )5 NO2 ]Cl 2 , both linkage and ionisation
19. A coordination compound of cobalt has the isomerism
molecular formula containing five ammonia (d) [Co(NH3 )5 Cl 2 ]NO2 , ionisation isomerism
molecules, one nitro group and two chlorine atoms
20. Roasting of sulphides gives the gas ‘X’ as a
for one cobalt atom. One mole of this compound
byproduct. This is a colourless gas with choking
produce three ions in an aqueous solution. The smell of burnt sulphur and causes great damage to
aqueous solution on treatment with an excess of respiratory organs as a result of acid rain. Its
AgNO3 gives two moles of AgCl as a precipitate. aqueous solution is acidic, acts as reducing agent
The formula of the complex and the isomerism and its acid is never isolated. The gas X is
shown by this is (a) H2 S (b) SO2 (c) CO2 (d) SO3

PREP CATALYSIS
208 JEE Advanced~Practice Set 8

21. Compound ‘X’ with molecular formula C18H 20O (a) X is a reducing sugar and Y is a non-reducing sugar
(b) X is a non-reducing sugar and Y is a reducing sugar
upon ozonolysis gives compound A (C10H10O) and
(c) The glucosidic linkages in X and Y are b and g
B (C8H 8O 2). Compound A produced shows positive respectively
iodoform test and in presence of NH 2OH gives (d) The glucosidic linkages in X and Y are b and a,
oxime (C10H13ON) (C) and the compound C gives respectively
compound D (C8H11N) on reaction with PCl 5 / ether
23. A unit cell comprises of atoms P, Q and R
followed by hydrolysis. respectively.
Following transformations also occur as: (1) Atom(P) occupies all the corners of the lattice.
O 3 / Red
Compound ( X ) ¾® C10H10O(A) + C8H 8O 2 (B) (2) Atoms(Q) occupies all the edges and are also
(Highest mpt.) present at the alternate faces.
(i) PCl5 / ether
NH 2OH (3) Atoms(R) are present at face centres left from
Compound( A ) ¾® Oxime(C) ¾¾¾® atoms (Q) and one at each body diagonal at a
(Shows positive (C10 H13 ON) (ii) H3 O+
iodoform test distance of 1/4th of body diagonal from the
HNO2 at 0°C corner.
C8H11N + CH3COOH ¾¾¾® If a four-fold axis is passed from the given unit cell
(D)
in such a way that all atoms touching the axis are
Aromatic alcohol(E) ¾® Phthalic acid
[O] removed, then the possible formula of the
(C 8 H10 O) remaining compound will be
(a) PQ3 R 6 (b) P3Q6 R 7 (c) PQR 2 (d) PQ4 R 5
Mild
Compound (B)Oxidation Acid (F) [C8H8O3]
24. A piston is cleverly designed so that it extracts the
HI
maximum amount of work out of a chemical
reaction, by matching pext to the pint at all times.
HOOC COOH + CH3I
This 8 cm diameter piston initially holds back 1
mole of gas occupying 1L and comes to rest after
The compound (B) is being pushed out a further 2 L at 25°C. After exactly
half of the work has been done, the piston travelled
(a) MeO COOH out a total of
(a) 20.0 cm (b) 9.8 cm (c) 11.2 cm (d) 14.6 cm

(b) MeO CH2OH


Section 2 (Maximum Marks : 24)
Instructions : Same as given in Physics
(c) MeO CHO 25. Among the species given below, total number of
species that obey Sidgwick EAN rule are
(D) HO CHO
K3[Fe(CN)6],[ Ru(CO)5 ],[Cr(NH3 )6]3+ ,
[Co(NH3 )6]3+ ,[Ni(NH3 )6]2+ ,[Fe(CO)5 ],[ W(CO)6].

22. The correct statement(s) about the following 26. The complex ion of Fe2+ with the chelating agent
sugars X and Y is/are dipyridyl has been studied kinetically in both the
CH2OH forward and backward directions. For the complex
O H HOH2C O formation reaction.
H H
H Fe2+ (aq) + 3 dipy ¾® [Fe(dipy)3 ]2 + (aq)
OH H O (X)
HO H HO CH2OH If the rate of the formation of the complex at 298K
H OH is given by = (1.45 ´ 10 13 mol-3L3 s-1)
OH H
[Fe2+ ][dipy]3 and for the reverse reaction, the rate
CH2OH
CH2OH of disapperance of the complex is;
H
O H
H H
(Y)
rate = (x)[Fe(dipy)2+
3 ].
H O
H HO
OH H H OH Then, the half-life period of the backward reaction
HO
OH H is …… min.
H OH
[Given : k eq = 119
. ´ 10 17 ]

PREP CATALYSIS
JEE Advanced~Practice Set 8 209

27. At 2 atm and 23°C, the solubility of N 2 is List I List II


8 .4 ´ 10 -4 mol L-1 of air (80% N 2 + 20% O2). A deep A Li 1 Diagonal relationship.
sea diver inside the sea (at 23°C) breaths air at 10 B Be 2 Maximum standard reduction
atm, when total volume of blood is 8 L. After a electrode potential (E°).
while, the diver comes on the surface of water C Mg 3 Cation is maximum hydrated.
where air pressure is 2 atm. At 2atm and 23°C, the D Al 4 Cation has maximum ionic mobility
amount of volume of N 2 come out from the blood in aqueous solution.
will be …… . E Ca 5 Fluoride and carbonate least
soluble in water.
28. For the given compound X, the total number
6 Coordination number six in the
optically active or chiral centres is/are …… . complexes.
7 Ions are involved in the transmission
of electrical impulses is human
body.

The correct option is/are


(a) A®1,2; B ®13, ; C®167
, , ; D®16, ; E ®4,5,67,
(b) A ®2,3; B ®67 , ; D®5,6; E ®12
, ; C®13 , ,3,4
29. The cyanate content of a pesticide was determined (c) A ®12
, ; B ®3,5; C®14
, ; D®67, ; E ®12, ,3,4
by dissolving a 4.112 g sample in water and (d) A ®3,4; B ® 5,6; C®12 , , ; E ®4,5,67
, ; D®167 ,
treating with semicarbazide hydrochloride. 34. The desired product ‘X’ can be prepared by reacting
H 2NCONHNH3 + CNO- ¾® the major product of the reactions in List I with
one or more appropriate reagents in List II.
H 2NCONHNHCONH 2(s)
O
The solid formed in this reaction, when filtered and H
dried, weighed 0.209 g. Then percentage of KCNO N
HN
in the pesticide is …… . O (X)
30. An excess of liquid mercury is added to an O N N
H
acidified solution of 10 . ´ 10 -3 M Fe3+ . It is found H
3+
that 5% of Fe remains at equilibrium at 25°C. The List I List II
value of E° Hg 2+ / Hg is X volt, assuming that the only P NH2 EtO2C (i) H2SO4, heat 1 Br2,H 2O
2 (ii) Fuming HNO3
reaction that occurs is OC + CH2 (iii) H2O, heat
2Hg + 2Fe3 + ¾® Hg22+ + 2Fe2+ NH2 O C (iv) Sn, HCl

The value of X is …… . Me
[Given: E° Fe3 + / Fe 2+ = 0 .77 V] Q (i) POCl3 2 Urea
NH2 HO2C
(ii) HNO3
31. One mole of a monoatomic gas initially at a OC + CH2 (iii) NH4HS
pressure of 2.00 bar and a temperature of 273K is NH2 HO2C
taken to a final pressure of 4.00 bar by a reversible
path defined by p/V = constant. Taking CV to be R NH2 EtO2C 3 H 2SO 4 ,
(i) POCl Heat
equal to 12.5 Jmol-1K -1 , the value of DU / w for OC + CH2 (ii) NaOH3
NH2 NC (iii) HNO2
this process is calculated to be …… . (iv) NH4HS

32. 10 L polluted air at a temperature of 25°C and 4 KNCO,H 2O


atmospheric pressure was passed through
5 20% HCl,
lime-water giving 0.065 g CaCO3 . If Henry law heat
constant for CO2 is 3.35 ´ 10 -4 Matm -1 and
6 ClCO 2Et,
K a1 = 4 .2 ´ 10 -7 , then the value of pH of lake water NaOH
that is located on the polluted site is …… .
7 180°C

Section 3 (Maximum Marks : 12) The correct option is


, ,5; Q ®12
(a) P®13 , ; R® 67,
Instructions : Same as given in Physics
, ,3; Q ®4,5; R® 67
(b) P®12 ,
33. Match each set of names from List I with their , ,4; Q ®12
(c) P®13 , ,3; R ® 5,67
,
corresponding property (in List II). , ,3; Q ®13
(d) P®12 , ; R® 57, ,8

PREP CATALYSIS
210 JEE Advanced~Practice Set 8

35. List 1 contain reactions and List II contain major Match each reaction in List I with one or more
products. products in List II and choose the correct option.
(a) P®1; Q ®2; R®3; S®4; T ®5
List I List II (b) P®4; Q ®1; R®2; S®5; T ®3
P O 1 OH (c) P®2; Q ®1; R®4; S®3; T ®5
HO HO (d) P®3; Q ®4; R®1; S®2; T ® 5
1mol of
LAH/ether
36. pK a of different acids are given below:
OH
CHO Acid pK a
Q 2
NH +4 9.26
O OH
HO HO
CH 3COOH 4.74
NaBH4/MeOH C6H 5COOH 4.20
HCN 9.21
CHO CHO C6H 5NH +3 4.62
R O 3 HO Match the species (salt) given in Column I with
HO OH their corresponding pH of 0.1M solutions given in
(i) 1 mol of
OH Column II.
(ii) NaBH4/MeOH (iii) H3O+ Me
Column I Column II
CHO - +
i p 7.00
CH3 COONH4
S 4 - +
O O ii q 2.81
C6H5 COOK
HO HO iii C6H5NH+3 Cl - r 10.01
Zn,Hg/HCl
iv NaCN s 8.60
OH
t 11.10
CHO
Codes
T O 5 i ii iii iv
HO (a) p s q t
PhNHNH2+glycol (b) t p r s
Me
+KOH (c) p q r s
CHO (d) r p s q

MATHEMATICS
f (x)
Section 1 (Maximum Marks : 24) 38. Given, lim
x ® 0 2 = 2, where [×] denotes greatest
x
Instructions : Same as given in Physics integer function, then which of the following is
37. For any positive integer n (are) TRUE?
(a) lim [f( x)] = 0 (b) lim [f( x)] = 1
æ1ö æ2ö æ1ö
Sn = tan -1 ç ÷ + tan -1 ç ÷ + tan -1 ç ÷ x® 0 x® 0
è2ø è9 ø è8 ø
(c) lim éê
f ( x) ù
does not exists (d) lim éê
f ( x) ù
exists
x® 0 ë x ú û x® 0 ë x ú û
æ2ö
+ tan -1 ç ÷ + ... n term.
è 25 ø x2 y2
39. Consider the ellipse +
= 1 is such that it has
Then, which of the following is (are) TRUE? a 2 b2
æ 2 ö the least area but contains the circle
(a) The nth term of the series S n is tan-1 çç 2 ÷÷ (x - 1)2 + y 2 = 1. Then, which of the following
è n + n + 1 ø
1 statements is (are) TRUE?
(b) tan(S ¥ ) = 1
3 (a) The eccentricity of the ellipse is
1 3
(c) cot(S ¥ ) = (b) The length of the latusrectum is 2 units
3
(d) 1 + tan2 (S ¥ ) = 10 (c) The equation of the auxiliary circle of ellipse is
x 2 + y2 = 5
(d) The foci of the ellipse is (0,± 3)

PREP CATALYSIS
JEE Advanced~Practice Set 8 211

40. Let X and Y be two arbitrary, 3 ´ 3 non-zero, 47. The number of integral values of k for which
skew-symmetric matrices and Z be an arbitrary exactly three roots of the equation
3 ´ 3, non zero symmetric matrix. Then, which of sin 2 x + (cos x - 1)sin x - cos x - k sin x + k = 0 ,
the following matrices is(are) skew symmetric? x Î(0 ,2p) are real, is equal to
(a) Y 3 Z 4 - Z 4 X 3 (b) X 44 + Y 44
48. If a 1 , a 2 , a 3, ...., a n , n distinct odd natural numbers
(c) X 4 Z 3 - Z 3 X 4 (d) X 23 + Y 23
not divisible by any prime number
æ a3 a2 - 3ö 1 1 1
41. If the point ç , ÷, greater than 5. Then, + + ... + is less than
èa - 1 a - 1 ø a1 a 2 an
m
æ b3 b2 - 3 ö æ c 3 c 2 - 3 ö , then the value of (m + n), (where m and n are
ç , ÷, ç , ÷, where a , b, c are n
è b - 1 b - 1 ø èc - 1 c - 1 ø relatively prime)
different from 1, lie on the line lx + my + n = 0.
Then, which of the following is(are) TRUE? 49. If the distance between the plane
m Ax - 2 y + z = d and the plane containing the
(a) a + b + c = -
l x -1 y - 2 z - 4
n lines = = and
(b) ab + bc + ca = z 3 4
l x -2 y-3 z- 4
m+ n = = is 6 , then the value of|d| is
(c) abc = 3 4 5
l
equal to
(d) abc - (ab + bc + ca) + 3 (a + b + c ) = 0
42. p and q are non-zero constants. The equation 50. If m is the slope of the tangent to the circle
x 2 + y 2 = 5 at a point of intersection with the curve
x + px + q = 0 has roots u and v, then which of the
2
y = [|sin x| + |cos x|], where [ ×] denotes the greatest
following is(are) TRUE? integer function, then|m| is equal to
1 1
(a) q x2 + px + 1 = 0 has roots ,
u v
(b) ( x - p)( x - q ) = 0 has roots u + v and u × v
Section 3 (Maximum Marks : 12)
(c) x2 - ( p2 - 2q )x + q 2 = 0 has roots u 2 and v 2 Instructions : Same as given in Physics

(d) x2 + q x + p = 0 has roots and


u v 51. Let ABCDEF is a regular hexagonA(z1), B(z 2),C(z 3),
v u D(z 4), E(z 5),F(z 6) in argand plane where A , B , C, D, E
and F are taken in anticlock wise manner. If
Section 2 (Maximum Marks : 24) z1 = - 2 , z 3 = 1 - 3
Instructions : Same as given in Physics
List I List II
43. Let f (x) be a differentiable function such that P. If z2 = a + ib, then 2 a 2 + b 2 is equal to 1. 8
f ¢ (x) + f (x) = 4 xe- x × sin 2 x and f(0) = 0 , Q. The square of the inradius of hexagon is 2. 7
p
n
- lpe R. The area of region formed by point P( z) 3. 6
if lim
n ®¥
å f (kp) = (ep - 1)2 , then the value of l is .... lying inside the incircle of hexagon and
p 5p m
k =1 satisfying £ arg( z) £ is p, where
3 6 n
44. The minimum value of the expression p = | z | 2 + m and n are coprime natural number,
m + n, m th is equal to
| z - 3 | 2 + | z - 6 i | 2 is q, then the value of q is
S. The value of z42 - z12 - z22 - z32 - z52 - z62 4. 5
45. If A ,B , C and D are four points in spaces, then is equal to
AB ´ CD + BC ´ AD + CA ´ BD = k 5. 4
(area of DABC), where k is equal to 6. 3

46. If f (x) is an even function and satisfies the relation The correct option is
(a) P ® 4; Q ® 6; R ® 2; S ® 1
æ1ö (b) P ® 3; Q ® 6; R ® 1; S ® 2
x 2f (x) - 2f ç ÷ = g (x), where g (x) is an odd function,
èxø (c) P ® 5; Q ® 3; R ® 2; S ® 1
then f(5) equals (d) P ® 4; Q ® 6; R ® 1; S ® 2

PREP CATALYSIS
212 JEE Advanced~Practice Set 8

52. The function f (x) = ax 3 + bx 2 + cx + d has its The correct option is


non-zero local minimum and maximum values at (a) P ® 5; Q ® 2; R ® 3; S ® 6
x = - 2 and x = 2 respectively. If ‘a’ is one of the (b) P ® 6; Q ® 1; R ® 4; S ® 3
(c) P ® 6; Q ® 4; R ® 2; S ® 3
root of x 2 - x - 6 = 0
(d) P ® 5; Q ® 1; R ® 3; S ® 1
List I List II
54. Let E, F , G , H be 4 distinct points inside square
P. The value of ‘a’ is 1. 0
ABCD whose area is square units such that
Q. The value of ‘b’ is 2. 24 ÐEDC = ÐECD = ÐHDA = ÐHAD =
R. The value of ‘c’ is 3. > 32 ÐGAB = ÐGBA = ÐFBC = ÐFCB = 15°
S. The value of ‘d’ is 4. -2 List I List II
The correct option is P. If area of quadrilateral EFGH is equal to 1. 1
(a) P ® 3; Q ® 1; R ® 4; S ® 2 a - b , where a, b Î N, then a + b =
(b) P ® 3; Q ® 1; R ® 2; S ® 4
(c) P ® 3; Q ® 2; R ® 1; S ® 4 Q. ÐAEB = p , then R = 2. 3
(d) P ® 3; Q ® 4; R ® 2; S ® 1 k

53. All letters of the word BREAKAGE are to be R. The radius of circle circumscribing the 3. 5
DAHD is equal to
jumbled. The number of ways of arranging them so
that S. Let the lengths of perpendiculars from 4. 6
vertices G, A, H to opposite sides of
List I List II DAHG be h1, h2, h3
1 1 1
P. The two ‘A’s are not 1. 720 respectively. Let 2 + 2 + 2 = a + b
together h1 h2 h3
b
Q. The two ‘E’s are together 2. 1800 where, a, b, Î N then =
a
but not two A’s
R. Neither two A’s nor ‘E’s are 3. 5760 The correct option is
together (a) P ® 3; Q ® 2; R ® 1; S ® 4
(b) P ® 3; Q ® 1; R ® 4; S ® 2
S. No two vowel are together 4. 6000
(c) P ® 3; Q ® 2; R ® 4; S ® 1
5. 7560 (d) P ® 2; Q ® 3; R ® 4; S ® 1
6. 2400

PREP CATALYSIS
JEE Advanced~Practice Set 8 213

Answers
Paper 1
1. a,b,c,d 2. a,b,c,d 3. b, c 4. a, d 5. b, c, d 6. d 7. 5.00 8. 4.00 9. 1.00 10. 5.00
11. 5.00 12. 4.00 13. 6.00 14. 4.00 15. d 16. a 17. c 18. c 19. b 20. b, c
21. a, c, d 22. b, c, d 23. b, c, d 24. a 25. 6 26. – 1.27 27. 0.05 28. 0.048 29. 188.70 30. 9
31. –1327 32. 64.93 33. d 34. c 35. b 36. c 37. a, b, d 38. a, b 39. a, b 40. a, b, d
7.69
41. a, b, c 42. c, d 43. 409 44. 1 45. 0 46. 1403 47. 308 48. 0. 6 49. 6 50. 5
51. b 52. b 53. b 54. c

Paper 2
1. a, c, d 2. a, c 3. a,b,c,d 4. b, d 5. a, b, c 6. b, d 7. 6.00 8. 3.33 9. 3.00 10. 0.06
11. 6.00 12. 9.00 13. 4.00 14. 6.00 15. a 16. a 17. a 18. b 19. c 20. b
21. c 22. b 23. a, d 24. d 25. 4 26. 94.67 27. 0.33 28. 4 29. 0.1434 30. 0.79
31. 3.01 32. 6.48 33. b 34. b 35. a 36. a 37. c, d 38. a, c 39. b 40. c, d
41. a, b, d 42. a, c 43. 2 44. 30 45. 4 46. 0 47. 2 48. 23 49. 6 50. 2
51. a 52. b 53. d 54. a

SCORE SHEET - Paper 1


Section No. of Marks from Marks from Marks Obtained
Correct Questions Correct Questions (A) Incorrect Questions (B) (A-B)
................ ............................... ............................... ............................... ...............................
................ ............................... ............................... ............................... ...............................
................ ............................... ............................... ............................... ...............................
Percentage Marks = Marks Obtain/Total Marks x 100

SCORE SHEET - Paper 2


Section No. of Marks from Marks from Marks Obtained
Correct Questions Correct Questions (A) Incorrect Questions (B) (A-B)
................ ............................... ............................... ............................... ...............................
................ ............................... ............................... ............................... ...............................
................ ............................... ............................... ............................... ...............................
Percentage Marks = Marks Obtain/Total Marks x 100
Note To expect your success marks in the test should be between 65%-70%.

PREP CATALYSIS
PREP CATALYSIS
JEE Advanced

PRACTICE SET 9 (With Solutions)

Duration : 3 Hours Max. Marks . 360

Paper 1
PHYSICS

Section 1 (Maximum Marks : 28)


l
This section contains SEVEN questions.
l
Each question has FOUR options (a), (b), (c) and (d). ONE OR MORE THAN ONE of these four options is (are) correct.
l
For each question, darken the bubble(s) corresponding to all the correct option(s) in the ORS.
l
For each question, marks will be awarded in one of the following categories:
Full Marks : + 4 If only the bubble(s) corresponding to all the correct option(s)
is (are) darkened.
Partial Marks : + 1 For darkening a bubble corresponding to each correct option, providedNO
incorrect option is darkened
Zero Marks : 0 If none of the bubbles is darkened.
Negative Marks : - 2 In all other cases.
l
For example, if [a], [c] and [d] are all the correct options for a question, darkening all these three will get +4 marks;
darkening only [a] and [d] will get +2 marks; and darkening [a] and [b] will get-2 marks, as a wrong option is also
darkened

1. A student performed the experiment of 2. Two coherent P1


determination of focal length of a concave mirror monochromatic point
by u-v method using an optical bench of length sources S1 and S2 of ∆θ
1.5 m. The focal length of the mirror used is 24 cm. wavelength l = 600 nm
The maximum error in the location of the image are placed symmetrically
P2
can be 0.2 cm. The 5 sets of (u , v) values recorded on either side of the S1 S2
by the student (in cm) are (42, 56), (48, 48), centre of the circle as d
(60, 40), (66, 33), (78, 39). The data set(s) that shown in the figure. The
cannot come from experiment and is (are) sources are separated by a
distance d = 18
. mm. This
incorrectly recorded, is (are)
arrangement produces interference fringes visible
(a) (42, 56) (b) (48, 48) as alternate bright and dark spots on the
(c) (66, 33) (d) (78, 39) circumference of the circle.

PREP CATALYSIS
JEE Advanced~Practice Set 9 215

The angular separation between two consecutive


bright spots is Dq. Which of the following options
is/are correct? Q1 Q2
(a) The angular separation between two consecutive
bright spots decreases as we move from P1 to P2 along
the first quadrant (a) | Q1| > | Q2| (b)| Q1| < |Q2|
(b) A dark spot will be formed at the point P2
(c) at a finite distance to the left of Q1, the electric field is
(c) The total number of fringes produced between P1 and P2
zero
in the first quadrant is close to 3000
(d) at a finite distance to the right of Q2 , the electric field is
(d) At P2 the order of the fringe will be maximum
zero
210
3. The polonium isotope 84 Po is unstable. It emits an
6. Specific heats of a gas varies with temperature.
a-particle with energy of 5.30 MeV. The atomic Now, choose the correct options, let hydrogen gas
mass of 210 4
84 Po is 209.9829u and atomic mass of 2 He sample is taken in observation.
is 4.0026 u, then choose the correct options. 3
(a) At temperature around 60 K, C V = R
206 2
(a) Daughter nucleus is 82 Po
5
(b) Disintegration energy is 5.30 MeV (b) About 250 K, C V = R
2
(c) Disintegration energy is 5.40 MeV
5
(d) Mass of daughter nucleus is 205.9745u (c) About 750 K, C V = R
2
4. An ideal monoatomic gas is confined in a 7
(d) About 4000 K, C V = R
horizontal cylinder by a spring loaded piston (as 2
shown in the figure). Initially, the gas is at 7. A uniform magnetic field exists in a square of side
temperature T1 , pressure p1 and volume V1 and the 3a as shown in the figure. A square loop of side 2a
spring is in its relaxed state. The gas is then heated enters in the field with constant speed along the
very slowly to temperature T2 , pressure p2 and diagonal and leave the field. Choose the correct
volume V2 . During this process, the piston moves curve between induced emf and distance along the
out by a distance x. Ignoring the friction between diagonal, is let x 0 .
the piston and the cylinder, then choose the correct
statements. × × × × ×

× × × × ×
2a
2a
× × × × ×

× v× × × ×
(a) If V2 = 2 V1 and T2 = 3T1, then the energy stored in the
spring is 1 / 4( p1V1 )
× × × × ×
(b) If V2 = 2 V1 and T2 = 3T1, then the change in internal
ε ε
energy is 3 p1V1
(c) If V2 = 3V1 and T2 = 4T1, then the work done by the gas is
7
p1V1 (a) x0 (b) x0
3
(d) If V2 = 3V1 and T2 = 4T1, then the heat supplied to the gas
1
is ( p1V1 ) ε
4 ε

5. A few electric field lines for a system of two


charges Q1 and Q 2 fixed at two different points on (c) x0 (d) x0
the X-axis are shown in the figure. These lines
suggest that

PREP CATALYSIS
216 JEE Advanced~Practice Set 9

Section 2 (Maximum Marks : 15)


l This section contains FIVE questions
l The answer to each question is a SINGLE DIGIT INTEGER ranging from 0 to 9, both inclusive.
l For each question, darken the bubble corresponding to the correct integer in the ORS.
l For each question, marks will be awarded in one of the following categories:
Full Marks : + 3 If only the bubble corresponding to all the correct answer is darkened.
Zero Marks : 0 In all other cases.

8. A uniform disc of radius r and mass m is charged [Take, b = 50 cm, c = 10 cm, A = 40 cm 2 ,


uniformly with the charge q. This disc is placed flat on a L = 20 cm and g = 10 m /s 2 ].
rough horizontal surface having coefficient of friction m
as shown in figure. A uniform magnetic field is present F
Find the value of .
in a circular region (a > r), but varying as kt 3 . 25
Find the time in second after, which the disc begins to 10. A neutron collides head on elastically with
rotate. an initially stationary deuteron. If
(Take, r = 1 m, m = 18 kg, q = 1 C, m = 0.1, K = 4 and h = fraction of kinetic energy lost by neutron,
g = 10 m /s 2 ) then find the value of 9h.
11. A chain of mass m = 0.80 kg and length l = 15
.
θB m rests on a rough surfaced table, so that one
Disc
of its ends hangs over the edge. The chain
a starts sliding off the table all by itself
r 1
provided the over hanging part equals h =
3
of the chain length. If W is the total work
performed by the friction forces acting on
9. A vessel with a symmetrical hole in its bottom is the chain by the moment, it is starting to
fastened on a cart. The mass of the vessel and the cart is slide and till it slides completely off the table
1 kg. With force F (in N) the cart is pulled, so that a æWö
then find value of ç ÷.
maximum amount of water remains in the vessel. The è 13
. ø
dimensions of the vessel are as shown in the figure.
12. The minimum and maximum distance of a
L
satellite from the centre of the earth are 2R
and 4R respectively, where R is the radius of
b earth. If radius of curvature at the point of
c æR ö
maximum distance is N ç ÷, then find the
F è3ø
value of N.

PREP CATALYSIS
JEE Advanced~Practice Set 9 217

Section 3 (Maximum Marks : 18)


l This section contains SIX questions of matching type.
l The section contains TWO tables (each having 3 columns and 4 rows)
l Based on each table, there are THREE questions.
l Each question has FOUR options [a], [b], [c] and [d]. ONLY ONE of these four options is correct.
l For each question, darken the bubble corresponding to the correct option in the ORS.
l For each question, Marks will be awarded in one of the following categories:
Full Marks : + 3 If only the bubble corresponding to the correct option isdarkened
Zero Marks : 0 If none of the bubbles is darkened
Negative Marks : -1 In all other cases

Direction (Q.Nos. 13-15) Consider a particle of mass m Direction (Q.Nos. 16-18) In Column I, a system of two
moving in a vertical circle by a string of length l under the masses m1 and m 2 is given. All surfaces are frictionless.
influence of gravity of earth. Let a1 = acceleration of mass m1 and a 2 = acceleration of
C mass m 2 . T 1 and T 2 indicates tensions in part of strings.
v String is inextensible and light.
O
D B Column I Column II Column III
θ P a2 ( m2 - m1 )g P.
A. T1 I. a1 = T1 = T2
m2 m1 + m2 2 m1m2g
A =
4m1 + m2
Let A is the lowest point of trajectory and AC and BD are T2
two diameters.
m1
Column I Column II Column III a1
A. Velocity at point I. Tension at P. Total energy of
A is 5gl point C is particle is 3gl B. T1 II. a1 = a2 Q. T = 4m1m2g
a2 1
equals to T2 m2g m1 + m2
m a1 = m + m
weight of 1 2
θ m1
particle
B. Velocity at point II. Tension at Q. Total energy of
C. a2 III a1 = a2 and less R. T2 =
m1m2g
A is less than point D is ‘3mg particle is 3/2 gl
2 gl
m2 T2 . than m1 + m2
m1g / m1 + m2
C. Velocity at A is III. Tension is R. Total energy of T1
between 2 gl non-zero 5
particle is mgl
and 5gl everywhere 2 m1
a1
D. Velocity at point IV. Tension in S. Total energy of
A is 6 gl string is zero particle is
some, where 3 D. IV a1 ¹ a2 S. T1 =
2 m1m2g
gl
between B and 4 . 4m1 + m2
C
T2 T1
13. In which case, particle keeps on rotating along the m2
circular path? m1
(a) A II R (b) B III S
(c) C IV Q (d) D I P
16. In which case, T1 = T2 = m 2(g - a)?
14. In which case, particle leaves circular path and
(a) A II R (b) B III S (c) C IV P (d) D I Q
follows a parabolic trajectory?
(a) A II R (b) B III S 17. In which case, T1 = T2 = m 2a ?
(c) C IV Q (d) D I P (a) A II R (b) B III S (c) C IV P (d) D I Q
15. In which case, motion of particle is simple æg aö
18. In which case, T1 = T2 = m 2 ç - ÷?
harmonic? è2 4ø
(a) A II R (b) B III S (c) C IV Q (d) D I P (a) A II R (b) B III S (c) C IV P (d) D I Q

PREP CATALYSIS
CHEMISTRY
Reagents used in step I, II and III are given below
Section 1 (Maximum Marks : 28) which of the following is/are incorrect set of reagents?
Instructions : Same as given in Physics Step I Step II Step III
19. Which of the following order represent correct (a) Na 2CO 3 / air, D C C
order of polymerising tendency? (b) NaOH /air, D C, D Al, D
(a) Si—O >P —O >Cl —O > S —O (c) Na 2CO 3 / air, D C, D C, D
(b) P—O >S—O >Cl—O >Si—O (d) Conc. H2SO 4 , D NH4Cl, D C, D
(c) Cl—O >S—O >P—O >Si—O
(d) Si—O >P—O >S—O >Cl—O 25. Which of the following is the correct value of pH for
solution obtained by adding 4 g of NaOH to 1 L of
20. If the slope of Z versus p curve is constant solution containing 60 g of acetic acid?
æ p ö [K a of acetic acid = 0.000018]
çslope = atm - 1 ÷ at a particular temperature
è 492.6 ø Choose the correct choice/choices.
(300 K) and at very high pressure, then diameter 1.1
(a) log (0.18 ´ 10-4 ) + log
of the molecules is 0.9
(a) 7.5 Å (b) 5 Å (c) 2.5 Å (d) 1.25 Å 1.1
(b) - log (0.018 ´ 10-5 ) + log
0.9
21. On adding AgNO 3 solution into KI solution, a
-7 1.1
negatively charged solution is obtained when their (c) - log (0.018 ´ 10 ) + log
0.9
proportion is in (d) 4.83
(a) 100 mL of 0.1 M AgNO 3 + 100 mL of 0.1 M KI
(b) 100 mL of 0.2 M AgNO 3 + 100 mL of 0.1 M KI
(c) 100 mL of 0.1M AgNO 3 + 100 mL of 0.2 M KI
Section 2 (Maximum Marks : 15)
(d) 100 mL of 0.15 M AgNO 3 + 100 mL of 0.25 M KI Instructions : Same as given in Physics
26. The radial wave function for 1s orbit of a single
22. Which of the following compound undergoes
substitution reaction? electron species is given by y r = Ke- s / 2 where
2Zr
Cl Cl s= and K is constant [a 0 = radius of Ist Bohr
na 0
orbit in H-atom = 52.9 pm, n is principal quantum
(a) (b) (c) (d) number]. Calculate most probable distance of
1s-electron in Be3+ ion. Give answer in the form of
NO2 most probable distance in pm?
23. 0.0755 g of selenium vapours occupying a volume of 27. 100 mL 0.05 M CuSO 4 is electrolysed by using
185 0.965 A for 100 min. Find pH of the solution at the
114.2 mL at 973 K exerts a pressure atm.
760 end of electrolysis.
For the reaction Se6 (g) r 3Se2 (g) , 28. Ferric hydroxide obtained during qualitative
Degree of dissociation of Se is 0.59. analysis of Fe3+ is insoluble in NaOH but when
Which of the following statement(s) is/are correct dissolved in HCl produces (B) which is when
regarding above statement? dissolved in potassium ferrocyanide produces a blue
(a) Value of K p is 0.1687 ppt. when a large excess of reagent is added, it
(b) Value of KC = 0.2645 ´ 10-4 converts into prussian blue colour. What is the
0.1687 coordination number of Fe in complex obtained?
(c) Value of KC =
(0.0821 ´ 973)2 29. If there are 3 moles of atoms present in the packing
(0.0821 ´ 973)2 of pattern, ABC ABC ABC… . The number of moles of
(d) Value of K p =
973 tetrahedral void is equal to… .

24. Conversion of ferrous chromate to chromium can 30. PhCH 2CONH 2, CH 3CONH 2, CH 3CH 2CONH 2 are
be done by using following steps. Ferrous chromite mixed together and then this mixture is treated
I II III with Br2 + KOH, then number of 1° amine formed
¾® Na 2 CrO4 ¾® Cr2O 3 ¾® Cr. is/are … .

PREP CATALYSIS
JEE Advanced~Practice Set 9 219

33. Which of the following combination takes place in


Section 3 (Maximum Marks : 18) presence of base?
Instructions : Same as given in Physics
(a) B I R (b) A IV Q (c) A III Q (d) A III P
Direction (Q.Nos. 31-33) By appropriately matching the
information given in the three columns of the following Direction Q.Nos. 34-36) By appropriately matching the
table. information given in the three columns of following table.
Columns 1, 2 and 3 contains radius ratio, coordination
Columns I, II and III contain name reaction product and number and shape respectively.
intermediate respectively.
Column I Column II Column III
Column I Column II Column III
A. 0.732-1.000 I. 3 P. Octahedral
A. Claisen I. Diazoketone P. Carbene
B. 0.225-0.414 II. 4 Q. Body centered
condensation
cubic
B. Baeyer-villiger II. Ester Q. Carbanion C. 0.414-0.732 III. 6 R. Planar triangular
rearrangement
D. 0.155-0.225 IV. 8 S. Tetrahedral
C. Fries III. b-keto ester R. Alkyl
rearrangement migration 34. Which of the following combination is correct for
D. Wolf IV. Hydroxy S. Acyl CsCl?
rearrangement acetophenone migration (a) A III Q (b) B II P (c) D I R (d) A IV Q

31. Which of the following combination takes place in 35. Which of the following combination has a bond
presence of AlCl 3 catalyst? angle 90°?
(a) C IV S (b) C IV R (c) B II R (d) A I P (a) C III P (b) C II R (c) D III P (d) A III Q

32. Which of the following combination involves 36. Which of the following combination involves sp3
ketone as a reactant? hybridisation?
(a) B I R (b) B II R (c) D III P (d) A IV Q (a) C II Q (b) B II S (c) C I P (d) D II R

MATHEMATICS
40. Given z = f (x) + ig (x), where f, g :(0 , 1) ® (0 , 1) are real
Section 1 (Maximum Marks : 28) valued functions. Then, which of the following
Instructions : Same as given in Physics
does not hold good?
37. Let f (x) = max{1 + sin x , 1, 1 - cos x},x Î[0 , 2p] and 1 æ 1 ö 1 æ 1 ö
(a) z = + iç ÷ (b) z = + iç ÷
g (x) = max{1,| x - 1|}. x Î R, then 1 - ix è 1 + ix ø 1 + ix è 1 - ix ø
(a) g (f(0)) = 1 (b) g (f(1)) = 1 1 æ 1 ö 1 æ 1 ö
(c) z = + iç ÷ (d) z = + iç ÷
(c) f(f(1)) = 1 (d) f(g (0)) = 1 + sin1 1 + ix è 1 + ix ø 1 - ix è 1 - ix ø
38. In a DABC, A º (a, b), B º (1, 2), C º (2, 3) and point ‘
3 x 2 + ax + a + 1
A’ lies on the line y = 2 x + 3 where a, b Î integer 41. If f (x) = , then which of the
and area of the triangle is S such that [S] = 2, where x2 + x - 2
[.] denotes the greatest integer function. Then, all following can be correct?
possible coordinates of A are (a) lim f( x) exists Þ a = - 2
x®1
(a) (- 7, - 11) (b) (- 6, - 9)
(b) lim f( x) exists Þ a = 13
(c) (2, 7 ) (d) (3, 9) x ® -2
4 1
39. Consider the functions f (x) and g (x), both defined (c) lim f( x) = (d) lim f( x) = -
x®1 3 x ® -2 3
from R ® R and are defined as f (x) = 2 x - x 2 and
42. For which of the following value of x, 5th term is
g (x) = x n , where n ÎN. If the area between f (x) and the numerically greatest term in the
g (x) is 1/2, then n is a divisor of 10
æ xö
(a) 12 (b) 15 expansion of ç1 + ÷ .
è 3ø
(c) 20 (d) 30
(a) - 2 (b) 1.8 (c) 2 (d) - 1.9

PREP CATALYSIS
220 JEE Advanced~Practice Set 9

43. Let x, y be real variable satisfying the area of triangle is 15 3. If ÐACB is obtuse and if r
x + y + 8 x - 10 y - 40 = 0. Let
2 2 denotes the radius of the incircle of the triangle,
then the value of r is ……… .
a = max( (x + 2)2 + (y - 3)2 ) and b = min
( (x + 2)2 + (y - 3)2 , then
46. Tangents are drawn from the point (a , b) to the
hyperbola 3 x 2 - 2 y 2 = 6 and are inclined at angle q
(a) a + b = 18
(b) a + b = 4 2 and f to the X-axis. If tan q × tan f = 2, then the
(c) a - b = 4 2 value of 2a 2 - b 2 is ……… .
(d) ab = 73
6n
47. If the value of lim (n - 3/ 2)
n ®¥
å r is equal to l,
Section 2 (Maximum Marks : 15) r =1

Instructions : Same as given in Physics then the value of l is ……… .


44. Let A be the set of 3 ´ 3 skew symmetric matrices 48. A drawer contains a mixture of white and black
whose entries are either - 1, 0, or 1. If there are socks, at most 15 in all. It so happens that when
exactly three 0’s, three 1’s and three- 1’s, then the two socks are selected randomly without
number of such matrices is ……… . replacement, there is a probability of exactly 1 / 2
that both are white or both are black. The largest
45. Consider a D ABC and let a, b and c denote the possible number of black socks in the drawer that
lengths of the sides opposite to the vertices A, B is consistent with this data is ……… .
and C respectively. Suppose a = 6 , b= 10 and the

Section 3 (Maximum Marks : 18)


Instructions : Same as given in Physics
Direction Answers (Q. No. 49, 50 and 51) by appropriately matching the information given in the three columns of the
following table.
x 2 + bx + c
Let f ( x ) = 2 . Also, let a , b be the real roots of x 2 + bx + c = 0 and a 1 , b 1 are the real roots of
x + b1 x + c 1
x 2 + b1 x + c 1 = 0
Column I Contains the information about the graph of f ( x ).
Column II Contains the information about the increasing/decreasing nature of f ( x ).
Column III Contains the information about the maxima/minima of f ( x ).
Column I Column II Column III
I. When, a1 < a < b1 < b i. f( x ) is increasing in ( a1, b1 ), P. f( x ) has a maxima in [a1, b1 ], when
when a1 < a < b1 < b a1 < b1 < a < b

asymptote
asymptote

asymptote

y=1
β1
X
α1 α β

PREP CATALYSIS
JEE Advanced~Practice Set 9 221

II. When a1 < b1 < a < b ii. f( x ) is decreasing in ( a, b ), Q. f( x ) has minima in [a, b ], when
when a1 < b1 < a < b a1 < b1 < a < b

y=1
X
y=0 α1 β1 α β

III. When a1 < b1 < a < b iii. f( x ) is increasing in (b1, b ) , R. f( x ) has a minima in [a1, b1 ], when
when a1 < a < b1 < b a1 < a < b1 < b

y=1
X
y=0 α1 β1 α β

IV. When, a1 < a < b1 < b iv. f( x ) is decreasing in ( a1, a), S. f( x ) has a maxima in [a, b ], when
when a1 < b1 < a < b a1 < a < b1 < b

asymptote
asymptote

asymptote

y=1
β1
X
α1 α β

49. Which of the following options is the only correct Column I Column II Column III
combination?
C. x = - 4ay
2
III. ( am , - 2 am)
2
R. y = mx + 2 a + a
(a) (I) (i) (P) (b) (II) (ii) (Q) (c) (III) (iii) (R) (d) (IV) (iv) (S) m2
50. Which of the following options is the only correct D. x 2 = 4ay IV. æ - 2 a , a ö S. y = mx + 2 am + am3
ç ÷
combination? è m m2 ø
(a) (I) (ii) (Q) (b) (III) (iii) (Q) (c) (IV) (i) (S) (d) (II) (iv) (P)
52. For a = 1 if a normal is drawn to a suitable parabola
51. Which of the following options is the only
at (- 4 , 4) is y = 2 x + 12, then which of following
incorrect combination?
option is correct combination?
(a) (I) (iii) (P) (b) (III) (i) (P) (c) (I) (iii) (S) (d) (III) (i) (Q)
(a) A I S (b) B I S (c) D IV R (d) B IV S
Direction (Q. No. 52, 53 and 54) by appropriate matching
the information given in the three columns of the 53. If a normal to a suitable parabola (Column I) is
following table. found to be x - 3 y - 33 = 0 and its point of contact
Columns I, II and III contains equation of parabolas, point is (6 , - 9), then which of the following option is
of contact in terms of slope, equation of normals in terms only correct combination?
of slope, respectively. (a) B II Q (b) A III P (c) C II Q (d) B III Q

Column I Column II Column III 54. If the focus of the suitable parabola (Column-I) is
(0, 2) and normal is drawn to the parabola at
A. y = 4ax
2
I. ( - am ,2 am),
2
P. y = mx - 2 am - am 3
(- 4 , 2) is y = x + 6, then which of the following
B. y 2 = - 4ax II. æ2 a , - a ö Q. y = mx - 2 a - a option is correct combination?
ç ÷
è m m2 ø m2 (a) A I R (b) C IV R (c) D IV R (d) C I S

PREP CATALYSIS
Paper 2
PHYSICS
Section 1 (Maximum Marks : 21)
l This section contains SEVEN questions.
l Each question has FOUR options (a), (b), (c) and (d). ONLY ONE of these four options is correct.
l For each question, darken the bubble corresponding to the correct option in the ORS.
l For each question, marks will be awarded in one of the following categories.
Full Marks : + 3 If only the bubble corresponding to the correct option is darkened.
Zero Marks : 0 If none of the bubbles is darkened.
Negative Marks : - 1 In all other cases

1. A thin wire of length L and a uniform linear mass P


density r is bent into a circular loop with centre at
O as shown in figure. The moment of inertia of the 4R
loop about the XX¢-axis is
X X´ 3R
90° 4R

O
2 GM 2 GM
(a) (4 2 - 5) (b) - (4 2 - 5)
7R 7R
GM 2 GM
(c) (d) ( 2 - 1)
rL3 rL3 5 rL3 3 rL3 4R 5R
(a) (b) (c) (d)
8p 2
16p 2
16p 2
8p 2 5. A glass tube of uniform
internal radius (r) has a valve
2. A particle is projected from the ground in such a
separating the two identical
way that it passes through a given point A(a , b).
ends. Initially, the valve is in
The minimum speed is required for it is 2 1
a tightly closed position. End
(a) u ³ bg + g a2 + b 2 (b) u £ bg + g a2 + b 2 1 has a hemispherial soap
bubble of radius r. End 2 has sub-hemispherical
(c) u = ag + g a2 - b 2 (d) u £ bg - g a2 + b 2
soap bubble as shown in figure. Just after opening
3. Polonium has five isotopes in which three isotopes the valve
decay by a-emission, b-emission and deuteron (a) air from end 1 flows towards end 2. No change in the
volume of the soap bubbles
emission, respectively. Their half lives are T1 = 400
(b) air from end 1 flows towards end 2. Volume of the soap
s, T2 = 800 s and T3 = 1600 s, respectively. At t = 0 bubble at end 1 decreases
s, probability of getting a, b and deuteron from (c) no change occurs
polonium nuclide are equal. The probability of (d) air from end 2 flows towards end 1. Volume of the soap
a-emission at bubble at end 1 increases
t = 1600 s will be
6. Water is filled upto a height h in a beaker of radius
(a) 19/13 (b) 8/13 R as shown in the figure. The density of water is r,
(c) 4/13 (d) 1/13 the surface tension of water is T and the
4. A thin uniform angular disc (see figure) of mass M atmospheric pressure is p0 . Consider a vertical
has outer radius 4R and inner radius 3R. The work section ABCD of the water column through a
required to take a unit mass from point P on its diameter of the beaker. The force on water on one
axis to infinity is side of this section by water on the other side of
this section has magnitude

PREP CATALYSIS
JEE Advanced~Practice Set 9 223

7. A sonometer wire resonates with a given tuning


2R
fork forming standing waves with five antinodes
B between the two bridges, when a mass of 9 kg is
A suspended from the wire. When this mass is
C h replaced by mass M. The wire resonates with the
same tuning fork forming three antinodes for the
D same positions of the bridges. The value of M is
(a) | 2 p0Rh + pR2rgh - 2 RT | (b) | 2p0Rh + Rrgh2 - 2 RT | (a) 25 kg (b) 5 kg
(c) | p0 pR + Rrgh - 2 RT| (d)| p0 pR + Rrgh + 2 RT|
2 2 2 2 (c) 12.5 kg (d) 1/25 kg

Section 2 (Maximum Marks : 28)


l This section contains SEVEN questions.
l Each question has FOUR options (a), (b), (c) and (d). ONE OR MORE THAN ONE of these four options is (are) correct.
l For each question, darken the bubble(s) corresponding to all the correct option(s) in the ORS.
l For each question, marks will be awarded in one of the following categories:
Full Marks : + 4 If only the bubble(s) corresponding to all the correct option(s)
is (are) darkened.
Partial Marks : + 1 For darkening a bubble corresponding to each correct option, providedNO
incorrect option is darkened
Zero Marks : 0 If none of the bubbles is darkened.
Negative Marks : - 2 In all other cases.
For example, if [a], [c] and [d] are all the correct options for a question, darkening all these three will get +4 marks;
darkening only [a] and [d] will get +2 marks; and darkening [a] and [b] will get-2 marks, as a wrong option is also
darkened

8. In the circuit shown in the figure R = 50 W, (d) Force acting on a particle for equal displacements can
produce different change in kinetic energy, but same
E1 = 25 3 V and E2 = 25 6 sin(wt) V, where change in momentum
w = 100 ps- 1 . The switch is closed at t = 0 and
10. Two bodies of masses m1 and m 2 are placed on a
remains closed for 14 min, then it is opened.
horizontal table with coefficient of friction m and
R are joined by a spring. Initially, the spring has its
natural length. If F is minimum force which, when
applied on m1 will make the other block m 2 just
move. (k is the spring constant) and x is
S elongation in spring at that instant.
E1 E2
µ m2 m1 F
(a) The amount of heat produced in the resistor is 63000 J
(b) The amount of heat produced in the resistor is 7000 J
(c) If total of heat produced is used to heat 3 kg of water at mm1g mm2g
(a) F = mm2g + (b) x =
20°C, the final temperature will be 25°C 2 k
(d) The value of direct current that will produce same mm2 g 2mm2 g
(c) F = mm1 g + (d) x =
amount of heat in same time through same resistor will 2 k
be 1.5 A
11. An ideal gas undergoes a thermodynamic cycle as
9. Choose the correct statement (s) of the following. shown in the figure. Which of the following graphs
(a) Force acting on a particle for equal time intervals can represent the same cycle?
produce the same change in momentum, but different
change in kinetic energy C
B
(b) Force acting on a particle for equal displacements can
produce same change in kinetic energy, but different V
change in momentum
(c) Force acting on a particle for equal time intervals can A
produce different change in momentum, but same
change in kinetic energy T

PREP CATALYSIS
224 JEE Advanced~Practice Set 9

(b) during time interval (0 £ t < ¥) readings of B and A are


A
B B changing
(a) V V (c) reading of A and B will be equal at t = RC ln2
(b) (d) None of the above
C A C
13. An image of a bright square is obtained on a screen
T with the aid of a convergent lens. The distance
T
between the square and the lens is 40 cm. The area
A B A B
of the image is nine time large than that of the
(c) p p square. Select the correct statement(s).
(d)
(a) Image is formed at a distance 120 cm from lens
C C (b) Image is formed at a distance 360 cm from lens
(c) Focal length of lens is 30 cm
V V
(d) Focal length of lens is 36 cm
12. A capacitor of A 14. The magnetic flux f linked with a conducting coil
capacitance C is depends on time as f = 4 t n + 6 , where n is positive
connected to two B
constant. The induced emf in the coil is e.
C
voltmeters A and B. A (a) If 0 < n < 1, e ¹ 0 and| e | decreases with time
is an ideal voltmeter (b) If n = 1, e is constant
having infinite (c) If n > 1,| e | increases with time
ε Sw
resistance, while B has (d) If n > 1,| e | decreases with time
resistance R. The
capacitor is uncharged and then the switch Sw is
closed at t = 0,
(a) readings of B and A will be e and zero at t = 0

Section 3 (Maximum Marks : 12)


l
This section contains TWO paragraphs.
l
Based on each paragraph, there are TWO questions.
l
Each question has FOUR options (a), (b) (c) and (d). ONLY ONE of these four options is correct.
l
For each question, darken the bubble corresponding to all the correct option in the ORS.
l
For each questions, marks will be awarded in one of the following categories.
Full Marks : + 3 If only the bubble corresponding to all the correct answer is darkened.
Zero Marks : 0 In all other cases.

Paragraph X 15. Angular velocity of system after collision about an


A thin rod of length L and mass m moves on smooth axis passing through common centre of mass of
horizontal surface with speed v 0 $j in the direction of its system is
3v 0 $ 3v 0 $
length. The end of the rod hits the end of second rod of (a) - k (b) - k
p 2L 4L
same mass and length place angle to first one. Two ends 3v 3v
2 (c) - 0 k$ (d) - 0 k$
5L 2L
stick together after collision, then
16. Maximum speed attained by end A of system after
collision
C D
x v0
(2 + 3/ 5 )
v0 æ3 + 2 ö
(a) (b) ç ÷
B 2 2 è 5ø
v0 æ1 + 3 ö v0 æ1 + 3 ö
(c) ç ÷ (d) ç ÷
v0 2 è 10 ø 2 è 5ø

PREP CATALYSIS
JEE Advanced~Practice Set 9 225

Paragraph A one third of the initial distance between plates. Also, the
In the given circuit, two 2k A B k
electric potential energy stored in the capacitor is found to
identical parallel conducting be 7.5 ´ 10 -4 J.
plates A and B are connected 17. Initially, when the plates are uncharged,
to a 25 V battery by metal
capacitance of the capacitor is
springs of spring constant 2k
and k, respectively. Initially, (a) 1.2 mF (b) 1mF (c) 0.8 mF (d) 0.6 mF
the switch S is open and the S 18. Extension of the spring connected to plate B is
plates are uncharged. In fact, V
(a) 2.67 mm (b) 3.22 mm
the two plates form a capacitor. When the switch S is (c) 4.33 mm (d) 1.33 mm
closed, distance between the plates becomes 2 mm and it is

CHEMISTRY
23. Product (W ) will be
Section 1 (Maximum Marks : 21)
Instructions : Same as given in Physics (i) NaH, THF
(W )
(ii) Br
19. The half-life of Tc99 is 6 h. The total residual activity
in a patient 30 h after receiving an injection
containing Tc99 must be more than 0.01 mCi . What
is the maximum activity (in mCi ) that the sample
injected can have? (a) (b)
(a) 0.32 (b) 0.16
(c) 0.08 (d) 0.64
20. When copper oxide is treated with sulphuric acid it
produces a compound [X] which exist as blue
coloured aqueous solution when it is heated its (c) (d)
colour changes to white. When [ X] is treated with
excess of aqueous NH 3 , it again produces a deep
blue coloured solution of [Y]. 24. 7 moles of tetra atomic non-linear gas A at 10 atm
Identify the correct statement about [ X] and [ Y]. and T K are mixed with 6 moles of another gas B
(a) [X] is Cu(SO 4 ) × 5H2O and [Y ] is [Cu (NH3 )6 ]
2+ T
at K and 5 atm in a closed vessel without energy
(b) [ X ] is [Cu(H2O)4 ]SO 4 × H2O and [Y ] is [Cu (NH3 )4 ]SO 4
3
transfer with surroundings. If final temperature of
(c) [ X ] is [Cu(H2O)4 ]SO 4 × H2O and [Y ] is 5T
[Cu (NH3 )4 (H2O)2 ]2+ mixture was K then, gas B is
6
(d) [ X ] is [Cu(H2O)6 ]SO 4 and [Y ] is [Cu (NH3 )4 ]SO 4 (a) monoatomic (b) diatomic
21. Arrange the following polymers in increasing order (c) triatomic (d) tetraatomic
of their intermolecular forces. 25. What is the oxidation state and hybridisation of
Nylon-6, 6, Polythene, Bakelite, Buna-S central metal ion in Wilkinson catalyst?
(G) (A) (U) (R) (a) +I, dsp2 (b) +III, dsp2 (c) +III, sp3 (d) +I, sp3
(a) A < R < G < U (b) R < A < G < U
(c) U > G > A > R (d) A < R < U < G Section 2 (Maximum Marks : 28)
22. Which of the following group of metals do not Instructions : Same as given in Physics
involve self reduction method during its 26. The vapour pressure of two pure liquids P and
extraction? Fe, Cu, Sn, Pb, Al and Mg Q,that form an ideal solution are 100 torr and
(a) Fe,Sn, Al and Mg 400 torr respectively at a temperature T. This liquid
(b) Fe, Sn, Pb, Al and Mg solution of P and Q is composed of 1 mole of P and
(c) Cu, Sn, Pb, Al and Mg 1 mole of Q. What will be the pressure, when 1
mole of mixture has been vapourised?
(d) Fe, Pb, Cu and Mg
(a) 200 torr (b) 300 torr (c) 500 torr (d) 700 torr

PREP CATALYSIS
226 JEE Advanced~Practice Set 9

27. Which of the following represents correct relation 31. Correct reagents to convert E into A is/are
between standard electrode potential and
equilibrium constant and choose the correct ⊕
O3 Mg + Hg H LiAlH4
options from the following codes given below: B C D E
Zn + H2O H2O ∆ H2O
nFE°
I. log K =
2.303RT A
nFE °
(a) Al 2O 3 (b) conc. H2SO 4
II. K = e RT
(c) H3 PO 4 (d) ThO 2 , D
- nFE°
III. log K = 32. Which of the following statement(s) is/are correct
2.303RT
regarding oxygen?
nFE°
IV. log K = 0 .4342 (a) It can be prepared by thermal decomposition of KClO 3
RT (b) According to molecular orbital theory, it contains two
Codes unpaired electrons in its bonding molecular orbital
(a) I, II and III are correct (b) II and III are correct (c) On reaction with aluminium it produces basic oxide
(c) I, II and IV are correct (d) I and III are correct (d) On reaction with sulphur it produces acidic oxide
Å Å
28. H 3 N ¾ CH ¾ COOH r H 3 N ¾ CH ¾ COO -
pK a = 9. 7
2 ½ pK a = 2.3
1 ½ Section 3 (Maximum Marks : 12)
Instructions : Same as given in Physics
R R
(A) (B ) Paragraph X
- Compound ( A ) on reaction with BH 3 , THF followed by
r H 2 N ¾ CH ¾ COO
½ OH - , H 2 O gives compound ( X ), that on further reaction
gives (Y ). Compound ( A ) on reaction with H 2 SO 4 , H 2 O
R
gives (Z ). It gives W on reaction with
(C ) ( CH 3 CO ) 2 O, Hg, H 2 O followed by NaBH 4 .
Which of the following statements is/are correct?
(a) pH = 6 is called iso-electric point
(b) At pH = 6, (B) is at its maximum
H2SO4, H2O (i) BH3, THF HBr
(c) At pH = 6, the solution having the above equilibrium (Z) (X) (Y)
become non-conducting –
(ii) OH , H2O
(d) Structure (A) is called Zwitter ion (A )
29. One mole of an ideal diatomic gas (CV = 5 Cal) was (i) (CH3CO)2O, Hg, H2O
(ii) NaBH4
transformed from initial 25°C and 1L to the state
when temperature is 100°C and volume 10 L. Then (W )
for this process (R = 2 Cal/mol/K) (take calories as
unit of energy and Kelvin for temperature) 33. Formation of Z is an example of
(a) electrophilic substitution reaction
(a) DH = 525 cal
373 (b) nucleophilic addition reaction
(b) DS = 5 ln + 2 ln 10 (c) electrophilic addition reaction
298
(d) free radical addition reaction
(c) DE = 525 cal
(d) DG of the process can be calculated using given 34. Choose the correct statement.
information. (a) Y is a secondary alkyl halide
30. The inorganic compounds show colour mainly due (b) product Z is a rearranged product
to three reasons i.e. d-d transition, charge transfer (c) Z and W are same product
and defect in crystal structure. Which of the (d) X, Z,W gives different alkene on dehydration
following shows colour due to charge transfer?
(a) KMnO 4
Paragraph A
(b) KO 2 Hybridisation is the measure to express the molecular
(c) K 2Cr2O 7
shapes reasonably. Hybridisation, although theoretical,
but it is highly correlated with molecular shapes or inter
(d) AgI
orbital angles.

PREP CATALYSIS
JEE Advanced~Practice Set 9 227

If there is any variation in geometry (on the basis of between two equivalent (same f s and same f p ) hybrid
VSEPR theory), the apparent variation in hybridisation -1
orbitals is given by cos q = .
can be observed and the following characteristic i
relationship is the easiest way to interpret that variation. 35. In BFClBr molecule which of the following bond
Any hybrid orbital made by s and p-orbitals is have minimum 1% s character provided to the
characterised by the following relationship. central atom?
1 i fp (a) B ¾ Cl (b) B ¾ F
fs = or f p = and i = (c) B ¾ Br (d) Cannot be predicted
i +1 i +1 fs
36. What percentage of s character is used by the
for sp 3 , i = 3 and sp 2 , i = 2,for an orbital f s + f p = 1 C-atom in the orbitals of CH 2 F2 molecule directed
while all the sp-hybrids of given atom must satisfy the towards H-atom?
condition, Sf s = 1.00. d and the inter orbital angle (a) 37.29% (b) 27.1% (c) 29% (d) 29.99%

MATHEMATICS
42. If u = cos x (sin x + sin 2x + sin 2 q ) where q is a
Section 1 (Maximum Marks : 21)
Instructions : Same as given in Physics given constant, then the maximum value of u is
(a) 1 + cos 2 q
ì 1+ x if x < 0
37. If f (x) = í (b) 1 + sin2 q
î(1 - x) (px + q) if x ³ 0
(c)| sin q|
satisfies the assumptions of Rolle’s theorem in the (d)| cos q|
interval [ - 1, 1], then the ordered pair (p, q) is
(a) (2, - 1) (b) (- 2, - 1) (c) (- 2, 1) (d) (2, 1) 1 æ p3 p5 p7 (- 1)n - 1 p 2n + 1 ö
43. ç - + K ÷ is equal to
2p è1!3 3 !5 5 !7 (2 n - 1)! (2 n + 1) ø
38. The system of linear equations x - y + 2z = a ,
1 1
x + 2 y - z = b, 2 y - 2 z = c is inconsistent, when (a) (b)
(a , b, c) equals 2 3
(c) 1 (d) None of these
(a) (2, 2, 0) (b) (2, 2, 1)
(c) (0, 3, 2 ) (d) (2, - 1, - 2 )
Section 2 (Maximum Marks : 28)
39. If a, b , c, d are unit vectors such that
Instructions : Same as given in Physics
® ®
1
(a ´ b) × (c ´ d) = 1 and a × c = , then 44. If the equation px 2 + y 2 + qz 2 + 2yz + zx + 3xy = 0
2
represents a pair of perpendicular planes, then
(a) a, b, c are non-coplanar
p - q is equals to
(b) b, c, d are non-coplanar
5 5
(c) b, d are non-parallel (a) 5 (b) - 5 (c) (d) -
(d) a, d are parallel and b, c are parallel 2 2

40. The function f (x) = ax 2 - c satisfies - 4 £ f()


1 £ -1 x dx + y dy a 2 - x 2 - y2
45. The solution of = is
and - 1 £ f(2) £ 5. Which of the following x dy - y dx x 2 + y2
statements is true?
(a) x2 + y2 = a sin (tan-1 y / x) + C
(a) - 7 £ f(3) £ 26 (b) - 4 £ f(3) £ 15
28 35 (b) x2 + y2 = a (cos (tan-1 y / x)) + C
(c) - 1 £ f(3) £ 20 (d) - £ f(3) £
3 3 (c) x2 + y2 = a (tan(sin-1 y / x) + C
(d) y = x tan æçsin-1 x + y2 ö÷ + C
41. The probability that a teacher will give an 1 2
1 è a ø
unannounced test during any class meeting is . If
5
46. If a, b, c are three positive number are in AP and
a student is absent twice, then the probability that
a + 8b c + 8b
the student will miss at least one test is x= + , then a value of x 2 can be
4 2 7 9 2b - a 2b - c
(a) (b) (c) (d)
5 5 25 25 (a) 256 (b) 324 (c) 361 (d) 400

PREP CATALYSIS
228 JEE Advanced~Practice Set 9

47. The parabola y = x 2 + px + q cuts the straight line and a circle with equation x 2 + y 2 = r 2 . If in the first
y = 2 x - 3 at a point with abscissa 1. quadrant, the common tangent to a circle of this family
and the above ellipse meets the coordinate axes at A and B.
If the distance between the vertex of the parabola
and the X-axis is least, then 51. Let P be a variable point on the ellipse with foci at S
and S¢. If D is the area of DPSS¢, then the
(a) p = 0 and q = - 2
(b) p = - 2 and q = 0 maximum value of D, is
(c) least distance between the parabola and X-axis is 2 (a) 7 sq unit
(d) least distance between the parabola and X-axis is 1 (b) 2 7 sq unit
(c) 3 7 sq unit
48. If non-zero vector a and b are equally inclined to
(d) 4 7 sq unit
coplanar vector c, then c can be
|a| |b| 52. If mid-point of A and B is (x1 , y1) and slope of
(a) a + b
|a| + 2 |b| |a| + 2 |b| common tangent be m, then
| b| |a| (a) 2 mx1 + y1 = 0
(b) a+ b
|a| + | b| |a| + | b| (b) 2 my1 + x1 = 0
|a| | b| (c) my1 + x1 = 0
(c) a+
|a| + 2 | b| ®
| a| + 2 | b| (d) mx1 + y1 = 0

(d)
| b|
a+
|a|
b Paragraph A
2|a| + | b| 2|a| + | b| Using three basic colours (Red, Blue, Green) different
colours are made by mixing these two or more colours in
49. If f ¢ (x) = g (x) (x - a) , where g (a) ¹ 0 and g is
2
equal proportion. Then, vertical stripes are painted by all
continuous at x = a , then colours available, such that no two consecutive stripes
(a) f is increasing in the neighbourhood of a, if g (a) > 0 have the same colours.
(b) f is increasing in the neighbourhood of a, if g (a) < 0
(c) f is decreasing in the neighbourhood of a, if g (a) > 0 53. The number of ways of painting the n stripes be
(d) f is decreasing in the neighbourhood of a, if g (a) < 0 t n (n ³ 1), then
(a) t n + 1 = 6t n
1
50. Let f (x) = cos(a 1 + x) + cos (a 2 + x) (b) t n + 1 = 7t n
2
1 1 (c) t n + 1 = 3t n
+ 2 cos (a 3 + x) + ... + n - 1 cos(a n + x), where a 1 , (d) t n + 1 = 2t n
2 2
a 2 , a 3 , ..., a n Î R. If f (x1) = f (x 2) = 0 , then| x 2 - x1 | 54. If n stripes are painted such that no two
may be equal to
consecutive stripes have any basic colour common,
p
(a) p (b) 2p (c) 3p (d) then number of ways of painting n stripes is
2
1
(a) [(1 + 2 )n - 1 + (1 - 2 )n - 1 ]
2
Section 3 (Maximum Marks : 12) 1
(b) [(1 + 2 )n - 1 - (1 - 2 )n - 1 ]
Instructions : Same as given in Physics 2
Paragraph X 1
(c) [(1 + 2 )n + (1 - 2 )n ]
Consider the standard equation of an ellipse whose focus 2
3
æ 16 ö (d) [(1 + 2 )n - (1 - 2 )n ]
and corresponding foot of directrix are ( 7 , 0) and ç , 0÷, 2
è 7 ø

PREP CATALYSIS
JEE Advanced~Practice Set 9 229

Answers
Paper 1
1. b, d 2. c, d 3. a, c, d 4. a, b 5. b, c 6. a,b,c,d 7. a 8. 2 9. 3 10. 8
11. 1 12. 8 13. d 14. c 15. b 16. a 17. b 18. c 19. d 20. b
21. c, d 22. a,b,c,d 23. a, b, c 24. a, c, d 25. b, d 26. 13. 23 27. 1 28. 6 29. 6 30. 3
31. a 32. b 33. c 34. d 35. a 36. b 37. a, b, d 38. a,b,c,d 39. b, c, d 40. a, c, d
41. a,b,c,d 42. a,b,c,d 43. a, c, d 44. 8 45. 1.73 46. 7 47. 96 48. 6 49. 9 50. b
51. c 52. b 53. c 54. c

Paper 2
1. d 2. a 3. d 4. a 5. b 6. b 7. a 8. a, c, d 9. a, b 10. b, c
11. a, c 12. b, c 13. a, c 14. a, b, c 15. c 16. c 17. c 18. a 19. a 20. b
21. b 22. a 23. c 24. b 25. a 26. a 27. c 28. a, b, c 29. a, c, d 30. a, c, d
31. b, c 32. a, b, d 33. c 34. c 35. a 36. b 37. d 38. b 39. c 40. c
41. d 42. b 43. a 44. a, d 45. a, d 46. b, c, d 47. b, d 48. b, d 49. a, d 50. a, b, c
51. c 52. d 53. a 54. d

SCORE SHEET - Paper 1


Section No. of Marks from Marks from Marks Obtained
Correct Questions Correct Questions (A) Incorrect Questions (B) (A-B)
................ ............................... ............................... ............................... ...............................
................ ............................... ............................... ............................... ...............................
................ ............................... ............................... ............................... ...............................
Percentage Marks = Marks Obtain/Total Marks x 100

SCORE SHEET - Paper 2


Section No. of Marks from Marks from Marks Obtained
Correct Questions Correct Questions (A) Incorrect Questions (B) (A-B)
................ ............................... ............................... ............................... ...............................
................ ............................... ............................... ............................... ...............................
................ ............................... ............................... ............................... ...............................
Percentage Marks = Marks Obtain/Total Marks x 100
Note To expect your success marks in the test should be between 65%-70%.

PREP CATALYSIS
PREP CATALYSIS
JEE Advanced

PRACTICE SET 10 (With Solutions)

Duration : 3 Hours Max. Marks . 360

Paper 1
PHYSICS
Section 1 (Maximum Marks : 24)
l This section contains SIX (06) questions.
l
Each question has FOUR options for correct answer(s). ONE OR MORE THAN ONE of these four option(s) is (are)
correct options(s).
l
For each question, choose the correct options(s) to answer the question.
l
Answer to each question will be evaluated according to the following marking scheme:
Full Marks : + 4 If only (all) the correct option(s) is (are) chosen.
Partial Marks : + 3 If all the four options are correct but ONLY three options are chosen.
Partial Marks : + 2 If three or more options are correct but ONLY two options are chosen, both of which are
correct options.
Partial Marks : + 1 If two or more options are correct but ONLY one option is chosen and it is a correct option.
Zero Marks : 0 If none of the options is chosen (i.e. the question is unanswered).
Negative Marks : - 2 In all other cases.
l
For example: If first, third and fourth are the ONLY three correct options for a question with second option being an
incorrect option; selecting only all the three correct options will result in + 4 marks. Selecting only two of the three
correct options (e.g. the first and fourth options), without selecting any incorrect option (second option in this case), will
result in +2 marks. Selecting only one of the three correct options (either first or third or fourth option), without selecting
any incorrect option (second option in this case), will result in +1 marks. Selecting any incorrect option(s) (second
option in this case), with or without selection of any correct option(s) will result in -2 marks.

1. Two smooth identical tracks of equal length have 2. The ring shown in figure is given a
bumps A up and B down both of same curvature and æ g ö
constant horizontal acceleration ç a0 = ÷.
size. If two balls starts simultaneously with same è 3ø
initial speed, then (bumps are completely identical) Maximum deflection of string from vertical
A v is q 0, then
m
B v END
Smooth
horizontal
(a) Both will reach end point at same time. l
(b) B will reach early than A.
(c) average velocity of B is greater than A.
m
(d) A takes less time to cover the bump than B.

PREP CATALYSIS
JEE Advanced~Practice Set 10 231

(a) q0 = 30°. Pressure


(p )
(b) q0 = 60°. C
(c) at maximum deflection tension in string is equal 3p0
to mg.
p0 B
(d) at maximum deflection, tension in string is equal
2mg p0/2 A
to .
3
Volume (V)
3. A leaky parallel plate capacitor is filled Vc V0
completely with a material having dielectric ln 6 ln 5
(a) g = (b) g =
constant, k = 5 and electrical conductivity ln 3 ln 3
s = 7.4 ´ 10-12 W-1m -1. (c) BC is adiabatic (d) AC is adiabatic
If the charge on the capacitor at instant, t = 0 is 6. A small solid cylinder of mass M and radius R
q = 8.85 mC, then calculate leakage current at the slides down a smooth curve from height h. It
(instant), t = 12 s. gets onto plank of mass M, which is resting on
(a) 0.249 m A (b) 0.150 m A a smooth surface. If m is coefficient of friction
(c) 1.02 m A (d) 0.198 m A between cylinder and plank, then
4. A perfectly reflecting solid sphere of radius r M, R
is kept in the path of a parallel beam of light of
large aperture. If the beam carries an intensity h
I, then the force exerted by the beam on the
surface is
Smooth
pIr 2 pIr 2
(a) (b) Negligible
C 2C
pIr 2 3 pIr 2 (a) cylinder is performing impure rolling, initially on
(c) (d) plank and finally performs pure rolling motion.
4C C
(b) friction on cylinder is initially backward and finally, it
5. One mole of an ideal gas is carried through a will be zero.
thermodynamic cycle as shown in the figure. (c) velocity of COM of plank + cylinder at moment of
The cycle consists of an isochoric an isothermal v
pure rolling is 0 , where v 0 = 2gh .
and an adiabatic processes. The adiabatic exponent 2
of the gas is g. (d) minimum length required to attain pure rolling on
Now, choose the correct option(s). plank is 3v 02 /16 mg, where v 0 = 2gh .

Section 2 (Maximum Marks : 24)


l
This section contains EIGHT (08) questions. The answer to each question is a NUMERICAL VALUE.
l
Four each question, enter the correct numerical value (in decimal notation, truncated/rounded-off to the second
decimal place; e.g. 6.25, 7.00.- 0.33, -.30, 30.27, -127.30) using the mouse and the on-screen virtual numeric
keypad in the place designated to enter the answer.
l
Answer to each question will be evaluated according to the following marking scheme:
Full Marks : + 3 If ONLY the correct numerical value is entered as answer.
Zero Marks : 0 In all other cases.

7. A 100 kg gun fires a ball of 1 kg horizontally 3x


If efficiency of cycle is 1 - , then
from a cliff of height 500 m. It falls on the 12 ln 2 + 15
ground at a distance of 400 m from the bottom find the value of x.
of the cliff. If recoil velocity of gun is v, then
find the value of v. (Take, g = 10 ms -2) 2 × 103
A B

8. Two moles of a monoatomic ideal gas is taken p


(N/m2 )
through a cyclic process shown in p-T diagram. 103 C
The process CA is represented as pT = constant.
T (K)
300 K 600 K

PREP CATALYSIS
232 JEE Advanced~Practice Set 10

9. A short semicircular 12. In a modified y


right cylinder of YDSE, the source
radius r and weight w S of wavelength S1 P
put on a horizontal 5000 Å oscillates S d = 1mm
θ B x
surface and is pulled by P about axis of
a horizontal force P set up according S2
applied at the point B of A to the equation, 1m D = 2m
the front edge. The flat face make angle q with æpö
y = 0 . 5 sin ç ÷ t,
the horizontal plane just before sliding begins è 6ø
and coefficient of friction at the line of where y is in millimeter and t in second At
contact with surface is m. Find the value of what time t will the intensity at P, a point
7 sin q , if m is 1/ p. exactly in front of slit S1, be maximum for the
10. A shaft of diameter D is F first time?
resting on a support as 13. A certain quantity of ideal gas takes up 56 J of
shown in the figure. The heat in the process AB and 360 J in the process
M
shaft exerts a total axial AC. What is the number of degree of freedom
force F. Assuming the of the gas?
µ
coefficient of friction is m, p
the twisting moment M necessary to cause 4p0 C
rotary motion to impend is m FD / N . Find the
value of N.
11. A photoelectric plate is initially exposed to a p0 A
B
spectrum of hydrogen gas excited to second
energy level. Later when the same V
V0 3V0 4V0
photoelectric plate is exposed to radiation from
some unknown hydrogen like gas, excited to 14. A uniformly charged thin
second energy level, it is found that the spherical shell of radius
de-Broglie wavelength of the photoelectrons, R carries uniform surface F F

now ejected has decreased 61 . times. For this charge density 6 per unit
new gas difference of energies of first Lyman area. It is made of two
series and Balmer series, limit is found to be hemispherical shells, held together by pressing
them with force F. The value of F will be
two times, the ionisation potential of the
(Take, ps 2R 2 = 4e 0 ).
hydrogen atom. Find the value of Z for
unknown gas.

Section 3 (Maximum Marks : 12)


l
This section contains TWO (02) paragraphs. Based on each paragraph, there are TWO(02) questions.
l
Each question has FOUR options. ONLY ONE of these four options corresponds to the correct answer.
l
Four each question, choose the option corresponding to the correct answer.
l
Answer to each question will be evaluated according to the following marking scheme:
Full Marks : + 3 If ONLY the correct option is chosen.
Zero Marks : 0 If none of the options is chosen (i.e. the question is unanswered).
Negative Marks : - 1 In all other cases.

Paragraph X
A circular disc of mass M and radius R is connected
to spring and placed on rough surface. The friction is k
sufficient for pure rolling. When disc is slightly
displaced and released, it executes SHM.

PREP CATALYSIS
JEE Advanced~Practice Set 10 233

across capacitor is æç
15. If m is friction coefficient and A is amplitude of 640 ö
÷ V and switch is opened.
oscillation for which no slipping occur is è 3 ø
4 mMg 3 mMg When switch is closed for (2.5 ln 4) min, then gas
(a) 0 < A < (b) A >
k k expands isobarically and its temperature increases
mMg 3 mMg
(c) 0 < A < (d) 0 < A < by 72 K. Heat loss through the wires is negligible
k k
(Take , R = 8.3 J mol -1 K-1 ).
16. If mass of disc is 1kg and spring constant of
spring is 50N/m, then angular frequency of C=75 mF
simple harmonic motion will Piston

(a) 5 / 3 (b) 10/ 3 (c) 1/ 3 (d) 2 / 3 R=2 kΩ


S
Paragraph A
A cylinder fitted with a piston, which can slide Cylinder
without friction contains one mole of an ideal gas. 17. Work done by the gas is approximately
The walls of the cylinder and piston are adiabatic. (a) 0.2 kJ (b) 0.4 kJ (c) 1.6 kJ (d) 0.8 kJ
The cylinder contains a resistor of resistance,
18. The increment in internal energy of gas is
R = 2 k W, which is connected to a capacitor of
(a) 1 kJ (b) 2 kJ (c) 3 kJ (d) 4 kJ
capacity, C = 75 mF. Initially, potential difference

CHEMISTRY
Section 1 (Maximum Marks : 24) (a) a zero of energy when two H - atoms are separated
by a great distance
Instructions: Same as given in Physics. (b) a drop in potential energy (net attraction) as the two
H - atoms approach each other
19. The reactions of Cl2 gas with cold dilute and (c) when stable H2 molecule is formed, internuclear
hot conc. NaOH in water give sodium salts of distance is about (74 pm)
two (different) oxoacids of chlorine, P and Q (d) an increase in potential energy as the atoms
respectively. The Cl2 gas reacts with SO2 gas in approach more closely
the presence of charcoal, to give product R. R 21. Which of the following statement is / are
reacts with white phosphorus to give a correct for group 16 elements?
compound S. On hydrolysis, S gives on oxoacid
(a) In OF2 , oxidation state of oxygen is +2
of phosphorus, T . The compounds R, S and T
(b) Sulphur shows strong tendency to catenation while
respectively are
oxygen shows this tendency to a limited extent
(a) SOCl 2 , PCl 5 and H3PO 4 (c) The transition temperature of rhombic and
(b) SO 2Cl 2 , PCl 3 and H3PO 4 monoclinic sulphur is 369° C
(c) SO 2Cl 2 , PCl 5 and H3PO2 (d) In 16th group oxygen is most abundant element in
(d) SO2 Cl 2 , PCl 5 and H3PO 4 the earth’s crust followed by sulphur

20. As two H - atoms approach each other, 22. In the following reaction sequence, the correct
potential energy change in the formation of a structure(s) of X is/are
16
covalent compound H 2 is shown. (i) Bz NHOH
NaOH, DMF, 80%
(X)
Energy (kJ/mol)

BnO 2 OTs
(ii) HCl/EtOH reflux
End
NHCbz
product
B D
O
Distance of (a) (b)
separation BnO 2 ONH2 BnO 2 O —NH2

NH2 NHCbz
435.8 C
Bond 74pm Internuclear
length distance
(c) (d) 16 NH2
Bn 2 ONHB2 BnO 2 O
Thus, the graph shows
NHCbz NH2

PREP CATALYSIS
234 JEE Advanced~Practice Set 10

23. Which of the following reaction sequence


is/are correct? Section 2 (Maximum Marks : 24)
F OH F Instructions: Same as given in Physics.
OH F
(a)
O2N O2N 25. Number of alloys that contain nickel among
CH3 CH3 N 2O
the following

No solder, gun metal, German silver, nichrome,


OH
release monel metal, constanton, bell metal,
of F
duralumin, type metal, invar, alnico.
26. At 1200°C mixture of Cl2 and Cl atoms (both in
(b) 3H2, Pd/C gaseous state) effuses 1.16 times as fast as
Krypton effuses under identical conditions.
The fraction of chlorine molecules dissociated
into atoms is ……… .
[Given, molar mass of Krypton = 83.8 g mol-1]

27. An ore in which only oxidisable ion is Sn2+ is


but not
titrated with dichromate solution containing
1.8 g of K 2Cr2O7 in 0.38 L.
If 15 cm3 of titrant was required by 0.25 g
sample of ore to reach equivalence point, then
N N N
the percentage of Sn is the ore is ……
Conc.H2SO4 NO2
(c)
+conc.HNO3
+ [Given: Molar mass of K = 39.1 g mol-1
Cr = 52 g mol-1, Sn = 118.7 g mol-1 ]
NO2
NO2 NO2
28. A cube shaped crystal of an alkali metal,
NO2 1.62 mm on an edge, was vaporised in a
(d)
Conc.H2SO4+conc.HNO3 500 mL flask. The pressure of the resulting
vapour was 12.50 mm at 802° C.
If the structure of the solid metal is known to
24. A quantity of 200 mL of 0.862 M HCl is mixed
with 200 mL of 0.431 Ba(OH)2 in a constant be body centred cubic, then the atomic radius
pressure calorimeter that has a heat capacity of the metal atom in picometre is ……
of 453 JK -1. 29. The molar conductivity of a 0.01 M KCl
The initial temperature of HCl and Ba(OH)2 solution, at 25° C is 150 S cm 2 mol-1 and
solution is the same at 20.48° C. For the ressistance of solution is 525 ohms.
process the heat of neutralisation for Ressistance of the same cell when filled with
H + ( aq ) + OH - ( aq ) - H O (aq ) is -56.2 kJ mol
2 2
-1 0.1 M NH 4OH solution is 2030 ohms. Also,
ionic conductivities of NH +4 and OH - are
Select the correct statement(s).
73.4 and 198.6 S cm 2 mol -1 respectively. If the
(a) Enthalpy change of the process is -9.69 kJ
(b) Final temperature is 25.12°C degree of ionisation of NH 4OH is X ´ 10-2, then
(c) Some HCl is left unreacted the value of X is ……… .
(d) Some Ba(OH)2 is left unreacted

PREP CATALYSIS
JEE Advanced~Practice Set 10 235

30. 3 g of activated charcoal was added to 50 mL of 33. The structural formula of compound B is
0.06 M acetic acid solution in a flask. After an Me
hour, 10 mL of residual CH3COOH required (a) (b) Me
Me
14 mL of 0.03 M NaOH. If Freundlich
x 1/ 2
adsorption isotherm is followed, i.e. = KC Me Me
m
(c) (d) —C2H5
then the value of K is ……… . COOH

[x is the amount of adsorbate in grams 34. The structural formula of compound C is


adsorbed per gram of adsorbent and C is the
equilibrium concentration in millimoles.] (a) (b)

31. 2.0 moles of an ideal monoatomic gas (c) (d)


( CV = 1.5 R) at 500 K and 5.0 atm was first
Paragraph A
expanded reversibly and adiabatically till the An ester (A) having molecular formula C7 H14O 2 reacts
pressure falls to 2.0 atm and then expanded at with MeMgBr followed by hydrolysis form an alcohol
constant pressure to some extent and finaly (B). (B) alcohol in presence of conc. H 2SO 4 gives
compressed isothermally and reversibly to compound (C). (C) further undergoes the following
5.0 atm. Depicting the above change on a p-V sequence of reaction.
diagram, the value of net work done will be H3O+
Ester (A)
……… . + 2 MeMgBr Alcohol (B)
C7H14O2
32. A 500 mL flask was charged with 1.0 mole of H3O+ Conc. H2SO4
COCl2( g) and heated to some temperature (T)
Acid (E) PCC O3/Red
where it decomposes partially as: + (D) (C)
Alcohol (F)
COCl2( g) - CO (g) + Cl (g) ; K
2 C (T ) = 1.5 M
I2 + Ca(OH)2 CHI3 +
Now the above flask is connected to another Alcohol (F) or compound (D) Calcium salt

flask containing some pure chlorine gas at the of acid (G)
same temperature and pressure, by a narrow ∆
tube of negligible volume. When the
Acetone
equilibrium was resorted, the concentration of
COCl2( g) was found to be 0.694 M. The volume 35. The structural formula of compound A is
of Cl2( g) flask is ……
Me
(a) Me
Section 3 (Maximum Marks : 12) Me
O Me

Instructions: Same as given in Physics. Me


(b) Me
Paragraph X Me O
Three compounds A , B and C are isomers of the Me
formula C5 H 8 . All of them decolourise bromine in (c) Me Me
O
CCl 4 and give a positive test with Baeyer’s reagent.
All three compounds dissolve in conc. H 2SO 4 . (d) Me
Me O Me
Compound A gives a white precipitate with Me
ammoniacal silver nitrate, whereas compounds B
36. The structural formula of compound E is
and C do not react with it. On hydrogenation, in the
(a) Me (b)
presence of platinum catalyst, both compounds A Me COOH Me COOH
and B yield n - pentane, whereas compound C gives a Me
product of formula C5 H10 . On oxidation with hot Me
(a) COOH (b) Me COOH
acidified KMnO 4 , B gave acetic acid and Me
CH3 CH 2COOH.

PREP CATALYSIS
236 JEE Advanced~Practice Set 10

MATHEMATICS
4bc
Section 1 (Maximum Marks : 24) (c) EF =
b+c
sin A / 2 (d) DAEF is isosceles

Instructions: Same as given in Physics.


42. A bag initially contains one red and two blue
37. Let g( x ) = ( f ( x ))2 + ( f ¢ ( x ))2, g( 0) = 6, where f ( x ) balls. An experiment consisting of selecting a
is a thrice differentiable function, such that ball of random, noting its colours and replacing
| f ( x )| £|, " x Î [- 1, 1]. Then, which of the it together with an additional ball of the same
following is (are) TRUE? colour. If three such trials are made. Then,
(a) There is at least one point in each of the intervals which of the following statements is(are) TRUE?
(- 1, 0) and (0, 1,
) where | f ¢ (x)| £ 2 (a) Probability that at least one blue ball is drawn is 0.9
(b) There is atleast one point in each of the intervals (b) Probability that exactly one blue ball is drawn is 0.2
(- 1, 0) and (0, 1), where g (x) £ 5 (c) Probability that all the drawn balls are red given that
(c) There is no point of local maxima of g (x) is (- 1, 1) all the drawn balls are of same colour is 0.2
(d) For some c Î (- 1, 1), g (c) ³ 6, g ¢ (c) = 0 and g ¢ ¢ (c) £ 0 (d) Probability that atleast one red ball is drawn is 0.6

38. Given that the complex numbers which satisfy Section 2 (Maximum Marks : 24)
the equation|zz 3| +|z3 z| = 350 form a rectangle
Instructions: Same as given in Physics.
in the arg and plane with the length of its
diagonal having an integral number of units. 43. The polynomial f ( x ) = x 4 + ax3 + bx 2 + cx + d
Then, which of the following is(are) TRUE? has real coefficient, andf ( 2i ) = f ( 2 + i ) = 0, then
(a) Area of rectangle is 48 sq units the value of a + b + c + d is equal to
(b) If z1, z 2 , z 3 , z 4 are vertices of rectangle, then
3p
z1 + z 2 + z 3 + z 4 = 0 44. If cos-1 p + cos-1 1 - p + cos-1 1 - q = .
(c) Rectangle is symmetrical about the real axis. 4
p 3p Then, the value of q is equal to ....
(d) arg (z1 - z 3 ) = or
4 4
ìï x üï 45. Let f : R ® R be a positive decreasing function
39. Let f ( x ) = í ò0 (1 +|1 - t|) dt , x > 2ý, then which of æ x - x3 ö
ïî 5x + 1, x £ 2 ïþ fç ÷
è 6 ø f (sin x )
the following is(are) TRUE? with lim = 1, then lim is
x ®¥ f(x) x ®¥ f(x)
(a) f (x) is not continuous at x = 2
(b) f (x) is continuous but not differentiable at x = 2 ¾® ¾®
(c) f (x) is differentiable everywhere 46. Let PR = 3i$ + $j - 2k
$ and SQ = $i - 3$j - 4k
$
(d) The right derivative of f (x) at x = 2 does not exist determine diagonals of a parallelogram PQRS,
¾®
40. A ray of light comes along the line L = 0 and and PT = i$ + 2$j + 3k
$ be another vector. Then,
strikes the plane mirror kept along the plane the volume of the parallelopiped determined by
P = 0 at B. A (2, 1, 6) is a point on the line ¾® ¾® ¾®
L = 0, whose image about P = 0 is A¢. It is given the vectors PQ , PT and PS is equal to ....
x- 2 y-1 z - 6
that, L = 0 is = = and P = 0 is 47. A pack contains n cards numbered from 1 to n,
3 4 5 two consecutive numbered cards are removed
x + y - 2z = 3. Then, which of the following from the pack and the sum of the numbers on
is(are) TRUE? the remaining cards is 1224. If the smaller of
(a) The coordinate of A¢ is (6, 5, 2) the numbers of the removed cards is k, then
(b) The coordinate of B is (- 10, - 15, - 14) the value of k is equals to ......
(c) The coordinate of A¢ is (6, 5, - 2) 1 4x3 (1 + ( x 4 )2016 )
(d) The coordiate of B is (10, 15, 11) 48. The value of the integral ò dx
0 (1 + x 4 ) 2018
41. In DABC, internal angle bisector of ÐA meets m
= where, m and n are relatively prime positive
side BC in D. DE ^ AD meets AC in E and AB n
in F, then which of the following is(are) TRUE? integers, then the value of ( m + n ) is equal to .....
(a) AE is the harmonic mean of b and c 49. The number of values of x in [- 2p, 2p ]
2bc
(b) AD = cos A / 2 satisfying the equation
b+c
|sin 2x|+|cos 2x| =|sin x| are ..... .

PREP CATALYSIS
JEE Advanced~Practice Set 10 237

102 æ r 2 - b2 ö
÷ x + r a -b
1 10 2 2
- C1 + (a) y = ç 2
2n 2n
50. The value of C2
81 n
81 n
81n ç a -r2 ÷ a -r
2 2
è ø
103 102n
- 2n
C3 + ... + is æ r 2 - b2 ö
÷ x + r a -b
2 2
81n 81n (b) y = - ç 2
ç a -r2 ÷ a -r
2 2
è ø
Section 3 (Maximum Marks : 12) æ r 2 - b2 ö
÷ x -r a -b
2 2

Instructions: Same as given in Physics. (c) y = ç 2


ç a -r ÷ 2
a -r
2 2
è ø
Paragraph X æ r 2 - b2 ö
÷ x -r a -b
2 2
(d) y = - ç 2
x2 y2 ç a -r ÷ 2
a -r
2 2
Consider the ellipse 2 + 2 = 1 ( a > b) and circle è ø
a b
Paragraph A
x + y = r . Now, any tangent of ellipse will be
2 2 2

If g ( x + y) = g( x) + g( y) + 3xy ( x + y) "x, y Î R and


y = mx ± a 2m 2 + b 2 and any tangent of circle will be
g ¢ ( 0) = - 4, then
y = mx ± r 1 + m 2 ,
53. Number of real roots of the equation g( x ) = 0
51. The number of common tangents to the ellipse
and circle will be, is
(a) at most 4 (b) exactly 4 (a) 2 (b) 0 (c) 1 (d) 3
(c) at least 4 (d) exactly 2
54. The domain of g ( x ) is
52. The equation of common tangent in 4th quadrant (a) [- 2, 0] È [2, ¥) (b) [0, 1]
will be, (c) [- 1, 1] (d) None of these

Paper 2
PHYSICS
Section 1 (Maximum Marks : 24)
l
This section contains SIX (06) questions.
l
Each question has FOUR options for correct answer(s). ONE OR MORE THAN ONE of these four option(s) is (are)
correct options(s).
l
For each question, choose the correct options(s) to answer the question.
l
Answer to each question will be evaluated according to the following marking scheme:
Full Marks : + 4 If only (all) the correct option(s) is (are) chosen.
Partial Marks : + 3 If all the four options are correct but ONLY three options are chosen.
Partial Marks : + 2 If three or more options are correct but ONLY two options are chosen, both of which are
correct options.
Partial Marks : + 1 If two or more options are correct but ONLY one option is chosen and it is a correct option.
Zero Marks : 0 If none of the options is chosen (i.e. the question is unanswered).
Negative Marks : - 2 In all other cases.
l
For example: If first, third and fourth are the ONLY three correct options for a question with second option being an
incorrect option; selecting only all the three correct options will result in + 4marks. Selecting only two of the three correct
options (e.g. the first and fourth options), without selecting any incorrect option (second option in this case), will result in
+2 marks. Selecting only one of the three correct options (either first or third or fourth option), without selecting any
incorrect option (second option in this case), will result in +1marks. Selecting any incorrect option(s) (second option in
this case), with or without selection of any correct option(s) will result in -2 marks.

PREP CATALYSIS
238 JEE Advanced~Practice Set 10

1. A smooth track is form of a quarter circle of 4. A charged particle having a positive charge q
radius 6 m lies in vertical plane. A particle approaches a grounded metallic neutral sphere
moves from P1 to P2 under action of force F1 , F2 of radius R with a constant speed v as shown
and F3 . Force F1 is always towards P2 and is in figure.
always 20 N in magnitude. Force F2 is always
act horizontally and is always 30 N in Now, choose the correct options.
magnitude. F3 acts tangentially and has
magnitude 15 N. q
R v
Now, choose the correct options.
O P2

F1 F3
(a) As, the charged particle draws nearer to the
F2 surface of the sphere, a current flows into the
ground.
P1
(b) As, the charged particle draws nearer to the
(a) Work done by F1 is 120 2 J. surface of the sphere, a current flows out of the
(b) Work done by F2 is 180 J. ground into the sphere.
(c) Work done by F3 = 45 pJ. (c) As, the charged particle draw nearer, the
(d) F3 is conservation in nature. magnitude of current flowing in the connector
joining the shell to the ground increases.
2. A pendulum is constructed from two
(d) As, the charged particle draws nearer, the
identical uniform thin rods A and B
magnitude of current flowing in the connector
each of length L and mass m joining the sphere shell to the ground decreases.
L
connected at right angle forming T
shape. It suspended free end and 5. A reflecting surface is represented by the
swings in vertical plane. 2L æp ö
L equation, y = sin ç x ÷ , 0 £ x £ L. A ray of
(a) Moment of inertia of theT about axis of rotation is p èL ø
17 light moving horizontally becomes vertical
ml 2 .
12 after reflecting. Find coordinates of points,
(b) Moment of inertia a ofT about axis of rotation is where ray is incident.
y
13 /12 ml 2 .
(c) Time period of small angular oscillation of T is
17l
2p .
18g
18g x
(d) Angular frequency ofT for small oscillation is .
17l æ L 2L ö æ L 3L ö
(a) ç , ÷ (b) ç , ÷
3. Variation of y-coordinate of two projectiles è4 p ø è3 p ø
with time is given in figure. If initial speed of æ 3L 2L ö æ 2L 3L ö
both is same, then (c) ç , ÷ (d) ç , ÷
è4 p ø è3 p ø
y
6. A cone of radius r and height h rests on rough
horizontal surface, the coefficient of friction
between the cone and the surface being m. A
gradually increasing horizontal force F is
applied to the vertex of the cone. The largest
t
2 1
value of m for, which cone will slide before it
topples is
(a) horizontal speed of 1 is less than 2. r 2r
(a) m = (b) m =
(b) horizontal speed of 1 is greater than 2. 2h 5h
(c) horizontal range of 1 is greater than 2.
r r
(d) Both has same horizontal range. (c) m = (d) m =
h h

PREP CATALYSIS
JEE Advanced~Practice Set 10 239

Section 2 (Maximum Marks : 24)


l This section contains EIGHT (08) questions. The answer to each question is a NUMERICAL VALUE.
l Four each question, enter the correct numerical value (in decimal notation, truncated/rounded-off to the second
decimal place; e.g. 6.25, 7.00.- 0.33, -.30, 30.27, -127.30) using the mouse and the on-screen virtual numeric
keypad in the place designated to enter the answer.
l Answer to each question will be evaluated according to the following marking scheme:
Full Marks : + 3 If ONLY the correct numerical value is entered as answer.
Zero Marks : 0 In all other cases.

7. A classroom is maintained at 20°C by a heater 11. The mean lives of a radioactive substance are
of resistance 20 W connected to 200 V mains. 1000 and 400 years for a-emission and
The temperature is uniform throughout the b-emission, respectively. Find out the time
classroom and heat is transmitted through a during, which three-fourth of a sample will
glass window of area A m 2 and thickness 0.2 cm. decay, if it is decaying both by a-emission and
So, outside temperature maintained is 15.24°(C). b-emission simultaneously and fill the number
Find the value of (A). of significant digits of your answer in OMR.
(Take, thermal conductivity of glass is 12. Water is filled in a
0.2 cal-m -1 s-1(°C) -1 and mechanical equivalent uniform container A
of heat is 4.2 J/cal) of area of
a
cross-section A. A
8. Water is flowing in varying cross-section pipe. hole of cross-section 20 m
The areas of cross-sections 1, 2 and 3 are area a (<<A) is
1 cm 2 , 2 cm 2 and A cm 2, respectively. Water made in the
levels are shown in different vertical tubes of container at a height of 20 m above the base.
manometers. The speed of water at Water streams out and hits a small block
1 placed at some distance from container. With
cross-section 3 is m/s.
x what speed (in ms - 1), the block should be
moved such that water streams always hits the
The value of x is
æ a 1ö -2
block? ç Given, = ÷ (Take, g = 10 ms )
è A 20 ø
35 cm
37.5 cm 13. A particle is suspended by a light vertical
200 cm
inelastic string of length l from a fixed support.
At its equilibrium position, it is projected
1 horizontally with a speed 6gl. What is the
3 2
ratio of the tension in the string in its horizontal
position to that in string, when the particle is
9. A block is moving with speed v towards a vertically above the point of support?
system of two blocks system. The first block
hits, the second block elastically. What will be 14. The cross-section of a prism as given in figure.
the common velocity by which two blocks One of the refracting surface is given by y = x 2.
system will move together after sometime. A ray of light travelling parallel to X-axis is
Given, the maximum compression x is 1 m. incident normally on face AB and refracted.
v=2 m/s
Find the minimum distance of incidence ray
1 2 k= 2N/m 3 from surface AD. Refractive index for prism
m m m
is 3.
Plane surface y

B C
10. If the K a radiation of Mo ( Z = 42) has a y = x2
wavelength of 0.71 Å, calculate wavelength of
the corresponding radiation of Cu. i.e. K a for h
Cu ( Z = 29). Find the value of nearest integer x
value (in Å) of wavelength. A D

PREP CATALYSIS
240 JEE Advanced~Practice Set 10

Section 3 (Maximum Marks : 12)


l This section contains FOUR (04) questions.
l Each question has TWO (02) matching lists : Column-I and Column-II.
l FOUR options are given representing matching of elements from Column-I and Column-II. ONLY ONE of these four
options corresponds to a correct matching.
l For each question, choose the option corresponding to the correct matching.
l For each question, marks will be awarded according to the following marking scheme.
Full Marks : + 3 If ONLY the option corresponding to the correct matching is chosen.
Zero Marks : 0 If none of the options is chosen (i.e. the question is unanswered).
Negative Marks : - 1 In all other cases.

15. A conducting rod PQ of mass M rotates 16. A resistance coil of resistance r connected to an
without friction on a horizontal plane about O external battery is placed inside an adiabatic
on circular rails of diameter l. The centre O cylinder fitted with a frictionless piston of
and the periphery are connected by resistance mass m and some area A. Initially, cylinder
R. The system is located in a uniform magnetic contains one mole of ideal gas He. A current I
field perpendicular to the plane of the loop. At flows through the coil such that temperature of
t = 0, PQ starts rotating clockwise with angular gas varies as T = T0 + at + bt 2 keeping pressure
velocity w0. Neglecting the resistance of the constant with time t. Atmospheric pressure
rails and rod as well as self-inductance. above piston is p0.

B p0
θ ω0
R m
P O

I He
Match the Column I and Column II as shown r
below.

Column I Column II Match the Column I and Column II as shown


A. Heat generated in the P. B wl 2 below.
circuit by t ® ¥ 18 R Column I Column II
B. Current as a function Q. Ml 2
w20 A. Current I flowing P. R
(2bt + a)
of time 24 through the coil as a pA
function of time
C. Total charge flown through R. Ml 2 w20
1/ 2
the resistance 12 B. Speed of piston as Q. é 5R (2bt + a)ù
function of time êë 2r úû
D. Angular velocity as a S. w0M
function of time 3B C. Acceleration of the R. R
(2b)
piston pA
T. w0 e -k t ,
R
where, k = D. Cp for the gas S. (bt 2 + a)
constant pA

T. 5R
U. B wl 2
2
8R
1/ 2
U. é 5R (2bt + a)ù
Codes êë 3r úû
(a) A-Q, B-U, C-R, D-T
(b) A-Q, B-U, C-S, D-T Codes
(c) A-P, B-T, C-S, D-U (a) A-U, B-S, C-R, D-T (b) A-Q, B-P, C-R, D-T
(d) A-P, B-U, C-S, D-T (c) A-U, B-P, C-T, D-R (d) A-U, B-P, C-R, D-T

PREP CATALYSIS
JEE Advanced~Practice Set 10 241

17. On the basis of Bohr’s model, match the Match the Column I and Column II for this
Column I and Column II for a hydrogen like particle and select correct option from the
atom with atomic number Z. codes given below.
Column I Column II
Column I Column II
A. Acceleration in ms- 2 p. 5.88
A. Radius of orbit p. proportional to Z 1 of particle at t = 1s
B. Current due to q. proportional to Z - 1 B. Radius of curvature q. 6.00
motion of electron in metre of particle at
C. Magnetic field at r. proportional to Z 2 t = 1s
centre of atom C. Normal acceleration r. 6.32
D. Velocity of electron s. proportional to Z 3 in ms-2 at t = 1s
D. Tangential s. 2.33
Codes acceleration in ms-2
A B C D at t = 1s
(a) q r s p
Codes
(b) q r p s
A B C D
(c) q s r p
(a) p q r s
(d) q p r s
(b) r q s p
18. Instantaneous position of a particle is given by (c) p r q s
r = t3 $i + t 2$j + tk$ (metre) (d) r s p q

CHEMISTRY
Section 1 (Maximum Marks : 24) 21. Vanillin (C8H 8O3 ) reacts with OH, MeI (1 mol)
to form A (C9H10O3 ). ‘A’ further reacts with
Instructions: Same as given in Physics.
KMnO4 to form D (C9H10O4 ). D is soluble in
19. A violet compound of manganese A NaHCO3 and in presence of conc. HI, heat
decomposes on heating to give compounds B gives 3, 4- dihydroxy benzoic acid. Vanillin also
and C along with the liberation of a gaseous undergoes following transformations:
substance D. Compound C reacts with KOH in FeCl3
Intense colour
the presence of KNO3 to give compound B. On [Ag (NH3)2]+ 2Br2
heating C with conc. H 2SO4 and NaCl, greenish Vanillin Ag + B C (C7H6Br2O2)
(C8H8O3) HCl/steam
yellow gas E is liberated and a compound of No reaction
manganese F is formed along with other
products. The compounds A and C in the above The structure of product C is
reaction are OH
(a) KMnO 4 and MnO 2 (b) K 2MnO 4 and MnO 2 C—OH C==O
(c) KMnO 4 and K 2MnO4 (d) KMnO 4 and Mn2O 3
Br Br Br
20. Black coloured (insoluble in H 2O) solid (A) does (a) (b)
not dissolve in dil. HNO3 . Aqua-regia can Br OMe OMe
dissolve (A) forming (B). (B) gives yellow ppt. OH OH
(C) with NaOH. (B) also gives orange Br Br
precipitate (D) with KI. (D) dissolves in excess Br Br
of KI forming (E). (E) gives brown ppt. with (c) (d)
NH +4 salt in presence of NaOH. (A) is Br OMe OH
precipitated if H 2S gas is passed into solution OH OMe
of (B) in dil. HCl. Then, compound E is
(a) HgCl 2 (b) HgO (c) K 2HgI4 (d) HgI2

PREP CATALYSIS
242 JEE Advanced~Practice Set 10

22. An organic compound A (C5H10O4 ) is oxidised 26. For the proton transfer reaction between
by Br2 - H 2O to an acid B (C5H10O5 ). A forms a piperidine (C5H10NH) and water:
triacetate and it is reduced by HI to n-pentane. C5H10NH + H 2O -C H 5
+
10NH 2 + OH -
Oxidation of A with HIO4 gives, among other
products, 1.0 molecule of CH 2O, 1.0 molecule of The following galvanic cell was constructed
HCOOH. What is the possible structure of A? Pt, H 2 (0.20 atm) / C5H10NH (0.10M),
CHO CHO
C5H10NH +2 (0.05 M) // Hg2Cl2( s), KCl(s) / Hg
H H H OH
(a) (b)
If the voltage of the cell is 0.902 V, then the
HO H H H
value of K b is ………… ´10-3
H OH H OH
CH2OH CH2OH [ Given : E° (Hg2Cl2 / Hg, Cl- = 0.242V)]
CHO CHO 27. Iron crystallises in several modifications. At
H H HO H about 910°C, ‘bcc’ form (called a - form)
(c) H OH (d) H OH undergoes transition to r-form with fcc lattice.
HO H H H
Assuming that the distance between the
nearest neighbours is the same in the two
CH2OH CH2OH forms at the transition temperature, the ratio
of the density of r-iron to that of
23. Which of the following statements is / are
a - iron at the transition temperature is ………
correct?
(a) Different types of velocities are related by equation,
u mp< uav < u rms
(b) Average kinetic energy of 1 g H2 gas and 8 g O 2 r
gas at 300 K are equal 2r
(c) Kinetic energy of 1 mole of a gas in terms of Ump is r
3
M (Ump)2
4
(d) Kinetic energy of 1 mole of a gas in terms of Urms is 28. For the given compound X, the total number of
(Urms )2 M
chiral centres present are ………
2 OH
24. X and Y both are soluble in given solvent
(shown in the graph below). Molar mass of the
polymer Y is
(a) 50 g mol -1 (b) 250 g mol -1 H H
(c) 110 g mol -1 (d) 200 g mol -1 HO
1.0 atmospheric pressure 29. n- butane is produced by the mono bromination
nt of ethane by Wurtz reaction. The volume of
Vapour pressure

o lve lX Y
es ola of
ethane at NTP to produce 55 g in butane, if the
u r m n
P 0 tio bromination takes place with 90% yield and the
I 1. lu
so Wurtz reaction with 85% yield is ……… .
II %
III 10 30. A sample weighing 0.30 g contained
K3 [Fe(C2O4 )3 ] × 3H 2O, FeCl3 × 6H 2O and the
90 100 110 120 inert impurity is dissolved in dilute sulphuric
T/ºC acid and volume made up to 100 mL. A 20 mL
portion of this solution required 3.75 mL of
Section 2 (Maximum Marks : 24) 0.005 M acidified KMnO4 solution to reach the
equivalence point. In an another experiment,
Instructions: Same as given in Physics. 50 mL sample of the same stock solution is
treated with Zn amalgum and the resulting
25. The total number of chemical species having
solution required 17.5 mL of permanganate
pyramidal shape among these species are
solution of same strength. The mass
……… .
percentage of FeCl3 × 6H 2O present in the
H3O+ , SO2 , H 2S, OSF2 , SO3 , SO32– , S2O32– , NF3 original sample is………

PREP CATALYSIS
JEE Advanced~Practice Set 10 243

31. For a first order List I List II


process A ® P, the P. NH2 EtO2C 1. Br2 AcOH

P. E
Heat
following activation A 155 kJ OC + CMe
energy diagram 35 kJ
NH2 H 2C
applies appropriately. P Q. NH2 EtO2C 2. C5H5N
Also, at 300 K, SC + CMe
half-life is 0.693 hrs Reaction coordinate + –
NH2 NaOHC
and equilibrium constant is 0.16. The value of
3. CH2 ClCOOH
pre-exponential factor for backward reaction
comes out to be x ´ 1014 h-1. The value of x is R. NH2 HO2C
Ac2O
4. H2,Pt
O==C + CHMe
……
NH2 CN
32. When suspected drunk drivers are tested with
a breathalyser, the alcohol (ethanol) in the The correct option is
exhaled breath is oxidised to ethanoic acid (a) P ® 1,2 ; Q ® 3 ; R ® 4
with an acidic solution of potassium (b) P ® 1,3 ; Q ® 2 ; R ® 1,4
dichromate. (c) P ® 1,2 ; Q ® 3 ; R ® 3,4
3CH3 , CH 2OH + 2Cr2O72- + 16H + (d) P ® 1,3 ; Q ® 2,3 ; R ® 3,4
35. List I contains reactions and List II contains
¾¾¾® 3CH3COOH + 4Cr3 + + 11H 2O
major products.
Under the condition of unit activity of each List I List II
species, emf recorded in the breathalyser at pH NO2 Cl
4 would be ……… . OCH3
NaOH CH3Cl
[Given: E°CH3 COOH|CH3 CH 2OH = 0.06V, P. 150ºC i.
E°Cr 2- 3+ = 1.33V] Cl Cl O2N
2O7 /Cr
NO2
(i) Cl2, AlCl3
Section 3 (Maximum Marks : 12) (ii) Br2, FeBr3
OH
Q. (iii) CH3ONa, 130ºC ii.
Instructions: Same as given in Physics.
OH
33. Match the species given is Column I with the OH OCH3
shape given in Column II. (i) NaOH, CH3Cl
Cl
(ii) H2SO4,+HNO3
Column I Column II R. (iii) Cl2, AlCl3 iii.
O2N
I. OSF2 p. Tetrahedral NO2
OH
II. O2 SF2 q. Linear (i) NaOH, H2O2
S. (ii) H+, H2O iv.
III. XeF4 r. Square planar
CHO Cl OCH3
IV. ClO–4 s. Trigonal pyramidal
V. I–3 t. See-saw OH
(i) CHCl3, NaOH
The correct option is T. (ii) H2O2
I II III IV V
(a) s p r p q NO2
(b) s p r q t (i) Cl2, AlCl3
(ii) CH3O–N+a
(c) p s r p t U. (iii) Cl2, AlCl3
(d) q r s t p

34. The desired product X can be The correct option is


Me
prepared by reacting the major HN (a) P ® i ; Q ® ii ; R ® iii ; S ® iv ; T ® ii ; U ® iii
product of the reactions in List (X)
N
(b) P ® iii ; Q ® ii ; R ® i ; S ® iv T ® ii ; U ® i
I with one or more appropriate (c) P ® iv ; Q ® i ; R ® i ; S ® ii T ® ii ; U ® iii
H
reagents in list II. (d) P ® i ; Q ® ii ; R ® ii ; S ® iii ; T ® iii ; U ® iv

PREP CATALYSIS
244 JEE Advanced~Practice Set 10

36. Match the graphs in Column I with their (iii) r. If OP = 0.3010, then
related properties in Column II and select the half-life = 1

[(A)0–(A)t]
answer from the codes given. at [A]0 = 0.5 M
p
[ A]0 = Initial concentration of reactant
[ A]t = Concentration of reactant at time t
Time (t)
Column I Column II
(iv) s. If OP = 0.3010, rate = 4
(i) p. Rate constant is equal at [A]0 = 2 M

log [A]t
to rate of the reaction.
[A]t–1 p
p

Time (t)
Time (t)
t. Half - life is
(ii) q. If OP = 0.5, rate = 4 K independent of initial
conc.
log (dx/dt)

p u. Rate becomes four


times on doubling [A]0 .

Time (t)

MATHEMATICS
Section 1 (Maximum Marks : 24) d × (a + c )
(b) =2
[a bc ]
Instructions: Same as given in Physics.
p2
(c) minimum value of x 2 + y 2 is
37. If x > m, y > n, z > r , ( x , y , z > 0) such that, 4
x n r 5p 2
(d) minimum value of x + y is
2 2

m y r = 0. Then,which of the following 4


m n z 39. Let P be the point on the parabola y 2 = 4x
is(are) TRUE? which is at the shortest distance from the
(a) The value of
x
+
y
+
z
is 2 centre S of the circle x 2 + y 2 - 4x
x -m y -n z -r - 16 y + 64 = 0. Let Q be the point on the circle
m n r dividing the line segment SP internally. Then,
(b) The value of + + is - 1
x -m y -n z -r which of the following is (are) TRUE?
xyz 8 (a) The length of line segment SP is 2 5
(c) The greatest value of is
(x - m) (y - n) (z - r ) 27 (b) The ratio of SQ to QP is ( 5 + 1) : 2
xyz 64 (c) The x-intercept of the normal to the parabola at P is 6.
(d) The greatest value of is
(x - m) (y - n) (z - r ) 27 1
(d) The slope of the tangent to the circle at Q is
2
38. Let a, b and c be three non-coplanar vectors
and d be a non-zero vector, which is 40. Let f ( x ) be a real valued continuous function
perpendicular to (a + b + c). Now, on R defined as f ( x ) = x 2e-|x|. Then, which of
d = (a ´ b ) sin x + ( b ´ c) cos y + 2 ( c ´ a ). Then, the following is (are) TRUE?
which of the following is (are) FALSE? (a) y = f (x) has two points of maxima
d × (a + c ) (b) Number of points of inflection for y = f (x) is 4
(a) =-2
[a bc ] (c) f ¢ (x) = 0 has three real roots
(d) y = f (x) has only one asymptote

PREP CATALYSIS
JEE Advanced~Practice Set 10 245

41. In a DXYZ, let x , y , z be the lengths of sides 46. If A is an idempotent matrix (non-zero) and I is
opposite to the angles X , Y , Z respectively, an identity matrix of the same order, then find
S-x S-y S-z the value of n , n Î N , such that
and x + y + z = 2S if, = =
4 3 2 ( A + I )n = I + 127 A.
8p
and area of incircle of DXYZ is . Then, 47. The minimum distance of the point (1, 1, 1) from
3 the plane x + y + z = 1 measured parallel to the
which of the following is (are) TRUE? x- 2 y - 3 z - 4 a
(a) Area of DXYZ is 6 6
line = = is , where a and b
1 2 3 b
35 6
(b) The radius of circumcircle of the DXYZ is are positive integer and a is not divisible by the
6 square of any prime,then ( a + b) is ……… .
X Y Z
(c) sin sin sin = 4 / 35 48. If range of the function
2 2 2
X +Yö f ( x ) = sin- 1 x + 2 tan- 1 x + x 2 + 4x + 1 is [ p, q ],
(d) sin2 æç =
÷ 3 /5
è 2 ø then the value of ( p + q ) is ……… .
49. If a , b (a < b ) are the two roots of the equation
42. Consider the hyperbola H : x 2 - y 2 = 1 and a
1 - 8 (log10 x )2
circle S with centre N ( x2 , 0). Suppose that H = 1, then the value of
and S touch each other at a point P ( x1 , y1 ) log10 x - 2 (log10 x )2
with x1 > 1 and y1 > 0. The common tangent to 1 é (a 2b3 + 1)2 ù
H and S at P intersects the X-axis at point ê - 30000 a 4 ú is ……… .
10 ë a 4
û
M. If, ( l , m ) is the centroid of the DPMN , then
the correct expression is (are) 50. Total number of solution of sin { x } = cos { x },
dl 1 where { ×} denotes the fractional part, in [0, 2p] is
(a) = 1- for x1 > 1
dx1 3 x1 2 equal to ……… .
dm x1
(b) = for x1 > 1
dx1 3 x 2 - 1
1
Section 3 (Maximum Marks : 12)
dl 1 Instructions: Same as given in Physics.
(c) = 1+ for x1 > 1
dx1 3 x1 2
dm 1 51. Let a contest consists of ranking 10 songs of
(d) = for x1 > 0 which 6 are classic and 4 are western songs.
dy1 3
Then, match the elements of List I with the
elements of List II.
Section 2 (Maximum Marks : 24)
List I List II
Instructions: Same as given in Physics.
10
P. Number of ways of ranking, so that 1. P4
43. The number of three digits numbers from 100 there are exactly 3 classic songs in
to 999 inclusive which have any one digit top 5, is
that is average of the other two is ...... Q. Number of ways of ranking, so that 2. 10
P6
44. Circle C1 and C2 touches externally and top rank goes to classic song, is
circles C1 and C2 touches internally to the R. Number of ways of ranking, so that 3. (5!) 3
circle C3 . The radii of C1 and C2 are 4 and 10 ranks of all western songs are
respectively and the centres of the three consecutive, is
circles are collinear. A chord of C3 is also a S. Number of ways of ranking, so that 6 4. 6(9!)
common transverse tangent of C1 and C2.
classic songs are in a specified order, is
a b
Given that, the length of the chord is , 5. 4! 7!
c
where a , b, c are positive integers a and c are 6. 5! 6!
relatively prime and b is not divisible by the
square of any prime, then the value of The correct option is
a + b + c is ..... (a) P ® 2; Q ® 3; R ® 4; S ® 5
(b) P ® 3; Q ® 4; R ® 5; S ® 1
1/ n
45. The value of lim n 2 ò x 2018x + 1 dx is .... . (c) P ® 1; Q ® 6; R ® 4; S ® 3
n ®¥ 0
(d) P ® 1; Q ® 3; R ® 5; S ® 4

PREP CATALYSIS
246 JEE Advanced~Practice Set 10

1 R. If a = 1and b = 2, then 3. lies on y = x


52. Let y = mi x + ( i = 1, 2, 3) represent three
mi (x, y)
straight lines whose slopes are the roots of the S. If a = 2 and b = 2, then 4. lies on
equation 2m3 - 3m 2 - 3m + 2 = 0, then (x, y) (4 x 2 - 1) (y 2 - 1) = 0

List I List II The correct option is


(a) P ® 1; Q ® 2; R ® 3; S ® 4
P. Algebraic sum of the 1. (4 2 + 9 5 ) / 4
intercepts made by lines (b) P ® 2; Q ® 3; R ® 1; S ® 4
on X-axis, is (c) P ® 1; Q ® 2; R ® 1; S ® 4
Q. Algebraic sum of the 2. 3 (d) P ® 2; Q ® 1; R ® 1; S ® 4
intercepts made by the 2 54. Let f1 : R ® R, f2 : [0, ¥ ) ® R, f3 : R ® R and
lines on Y-axis, is
f4 : R ® [0, ¥ ) be define by
R. Sum of the distances of 3. 21
- ì|x|, if x < 0
the lines from the origin, 4 f1( x ) = í x
is î e , if x ³ 0
S. Sum of the lengths of the 4. (5 2 + 9 5 ) / 10 f2( x ) = x 2
lines intercepted ì sin x , if x < 0
between the coordinate f3 ( x ) = í
î x, if x ³ 0
axes, is
ì f ( f ( x )), if x < 0
5. (5 2 + 9 5 ) / 4 f4( x ) = í 2 1
5 î 2 1
f ( f ( x )) - 1, if x ³ 0
6.
2 Then, match the statements of List I with
the statement of List II.
The correct option is
(a) P ® 1; Q ® 5; R ® 3; S ® 6 List I List II
(b) P ® 2; Q ® 3; R ® 4; S ® 1 P. f4 is 1. Onto but not one-one
(c) P ® 3; Q ® 2; R ® 4; S ® 1 Q. f3 is 2. neither continuous nor
(d) P ® 3; Q ® 2; R ® 1; S ® 6 one-one
53. Let ( x , y ) be such that,sin-1( ax ) + cos-1( y ) R. f2 of f1 is 3. differentiable but not
-1 p one-one
+ cos ( bxy ) = . Then, match the statement of
2 S. f2 is 4. continuous and one-one
List I with the statement of List II.
The correct option is
List I List II (a) P ® 3; Q ® 1; R ® 4; S ® 2
P. If a = 1and b = 0, then 1. lie on the circle (b) P ® 1; Q ® 3; R ® 4; S ® 2
(x, y) x 2 + y 2 =1 (c) P ® 3; Q ® 1; R ® 2; S ® 4
(d) P ® 1; Q ® 3; R ® 2; S ® 4
Q. If a = 1and b = 1, then 2. lie on
(x, y) (x 2 - 1)(y 2 - 1) = 0

PREP CATALYSIS
JEE Advanced~Practice Set 10 247

Answers
Paper 1
1. b,c 2. a,d 3. d 4. a 5. a,d 6. a,b,c,d 7. 0.40 8. 7.00 9. 3.00 10. 3.00
11. 2.00 12. 1.00 13. 5.00 14. 2.00 15. d 16. b 17. c 18. a 19. d 20. a,b,c,d
21. a,b,d 22. d 23. a,b,c 24. a,b 25. 6 26. 0.14 27. 34.16 28. 230 29. 1.43 30. 0.01
31. 1239.96 32. 200 33. b 34. c 35. a 36. c 37. a,b,d 38. a,b,c 39. a,d 40. b,c
41. a,b,c,d 42. a,b,c,d 43. 9 44. 0.5 45. 1 46. 10 47. 25 48. 2018 49. 4 50. 1
51. b 52. c 53. d 54. a

Paper 2
1. a,b,c 2. a,c 3. b,c 4. a,c 5. b,d 6. c 7. 1.00 8. 2.00 9. 1.00 10. 2.00
11. 4.00 12. 1.00 13. 4.00 14. 2.00 15. b 16. b 17. a 18. b 19. a 20. c
21. c 22. b 23. a,b,c,d 24. a 25. 4 26. 1.58 27. 1.09 28. 5 29. 55.43 30. 67.36
31. 4.04 32. 0.96 33. a 34. a 35. c 36. b 37. a,b,c 38. b,c 39. a,c,d 40. a,b,c,d
41. a,c,d 42. a,b,d 43. 121 44. 19 45. 0.5 46. 7 47. 17 48. 4 49. 10 50. 6
51. b 52. c 53. c 54. d

SCORE SHEET - Paper 1


Section No. of Marks from Marks from Marks Obtained
Correct Questions Correct Questions (A) Incorrect Questions (B) (A-B)
................ ............................... ............................... ............................... ...............................
................ ............................... ............................... ............................... ...............................
................ ............................... ............................... ............................... ...............................
Percentage Marks = Marks Obtain/Total Marks x 100

SCORE SHEET - Paper 2


Section No. of Marks from Marks from Marks Obtained
Correct Questions Correct Questions (A) Incorrect Questions (B) (A-B)
................ ............................... ............................... ............................... ...............................
................ ............................... ............................... ............................... ...............................
................ ............................... ............................... ............................... ...............................
Percentage Marks = Marks Obtain/Total Marks x 100
Note To expect your success marks in the test should be between 65%-70%.

PREP CATALYSIS
PREP CATALYSIS
JEE Advanced

PRACTICE SET 11 (With Solutions)

Duration : 3 Hours Max. Marks . 360

Paper 1
PHYSICS
Section 1 (Maximum Marks : 24)
l
This section contains SIX (06) questions.
l
Each question has FOUR options for correct answer(s). ONE OR MORE THAN ONE of these four option(s) is (are)
correct options(s).
l
For each question, choose the correct options(s) to answer the question.
l
Answer to each question will be evaluated according to the following marking scheme:
Full Marks : + 4 If only (all) the correct option(s) is (are) chosen.
Partial Marks : + 3 If all the four options are correct but ONLY three options are chosen.
Partial Marks : + 2 If three or more options are correct but ONLY two options are chosen, both of which are
correct options.
Partial Marks : + 1 If two or more options are correct but ONLY one option is chosen and it is a correct option.
Zero Marks : 0 If none of the options is chosen (i.e. the question is unanswered).
Negative Marks : - 2 In all other cases.
l
For example: If first, third and fourth are the ONLY three correct options for a question with second option being an
incorrect option; selecting only all the three correct options will result in + 4 marks. Selecting only two of the three
correct options (e.g. the first and fourth options), without selecting any incorrect option (second option in this case), will
result in +2 marks. Selecting only one of the three correct options (either first or third or fourth option), without selecting
any incorrect option (second option in this case), will result in +1 marks. Selecting any incorrect option(s) (second
option in this case), with or without selection of any correct option(s) will result in -2 marks.

1. SI unit of inductance, the henry is equivalent E0


(a) the electric field after inserting the slab is
to K
(a) weber/ ampere (b) volt-second/ampere (b) the capacitance after inserting the slab is K C0
(c) the induced charge on the slab is Q æç1 - ö÷
(c) joule/(ampere) 2 (d) ohm-second 1
è Kø
2. A parallel plate capacitor with plate area A U
(d) the energy stored in the capacitor becomes 0 , U 0
and separation d has charge Q. A slab of K
dielectric constant K is inserted in space being the energy of the capacitor before inserting
between the plates almost completely fills the slab
the space. If E0 and C0 be the electric field
and capacitance before inserting the slab, then 3. A uniform wire of resistance R is bent in the
form of a ring. Four points A, B, C and D are

PREP CATALYSIS
JEE Advanced~Practice Set 11 249

marked on the ring as shown in the figure. A 5. In given Young’s double slit experiment,
battery of emf E may be connected across the separation of slits is b and distance of slits
ring in two ways from screen is D.
C
S1
P
B D b
S2

A
D
Case I Battery is connected between B and D.
Case II Battery is connected between A and B.
When a source of white light is used, at point
(a) In each case, same current flows out of the battery
P(directly in front of S1), certain wavelengths are
(b) In case I, more current comes out of the battery
missing. Some of the missing wavelengths are
(c) In case II, more current comes out of the battery
b2 2b 2 b2 2b 2
(d) In case I, resistance is more than in case II (a) l = (b) l = (c) l = (d) l =
D D 3D 3D
4. Consider following experiment of
photoemission. In this, a point source of 6. In the circuit shown, A and B are equal
3.2 ´ 10-3 W power emitting photons of 5 eV resistances. When S is closed, the capacitor C
charges from the cell of emf e reaches a steady
energy is kept at a distance of 0.8 m from a state. The incorrect option is/are
metal sphere.
C
Source
Metal
sphere B

Work function of metallic sphere is 3 eV and it’s + A S


radius is 8 ´ 10-3 m. Assuming 100% absorption E

of photons by metal sphere. (a) during charging, more heat is produced in A than in
Now, choose the correct options. B
(b) in the steady state, heat is produced at the different
(a) In first 3 min, sphere emits nearly 107 electrons
rate in A and B
(b) In first 3 min, sphere emits nearly 108 electrons 1
(c) Photoemission stops after 1 s (c) in the steady state, energy stored in C is Ce2
8
(d) Photoemission stops after 180 s
(d) in the steady state, energy stored in C is 1/ 2Ce2

Section 2 (Maximum Marks : 24)


l
This section contains EIGHT (08) questions. The answer to each question is a NUMERICAL VALUE.
l
Four each question, enter the correct numerical value (in decimal notation, truncated/rounded-off to the second
decimal place; e.g. 6.25, 7.00.- 0.33, -.30, 30.27, -127.30) using the mouse and the on-screen virtual numeric
keypad in the place designated to enter the answer.
l
Answer to each question will be evaluated according to the following marking scheme:
Full Marks : + 3 If ONLY the correct numerical value is entered as answer.
Zero Marks : 0 In all other cases.

7. Moment of inertia of a Find the moment of inertia of remaining disc


disc (radius R), about an about same axis.
axis through its centroid R/3
and perpendicular to its 8. A solid cylinder rolls without any slip on an
plane is 0.81 kg-m 2. A inclined plane with angle of inclination 30°.
portion of radius R / 3 is Linear acceleration of centre of mass of cylinder
is .................... ms-2. (Take, g = 9.81 ms-2)
removed from disc as R
shown in the figure.

PREP CATALYSIS
250 JEE Advanced~Practice Set 11

9. A small block is connected to one end of a 11. A train moves towards a stationary observer
massless spring of unstretched length 4.9 m. with speed of 34 ms-1. The train sounds a
The other end of spring is attached to a rigid whistle and its frequency registered by
wall at O. observer is f1. If the train’s speed is reduced to
17 ms-1, the frequency registered is f2.
The block is stretched by 0.2 m and released at
t = 0; and it then executes SHM with frequency If speed of sound is 340 ms-1, then the ratio of
p f1 : f2 is ……… .
of rads-1. Simultaneously, at t = 0, a small
3 12. A room is maintained at 20°C by a heater of
pebble is projected with speed v from point P at resistance 20 W connected to 200 V mains. The
an angle of 45° as shown in figure (distance temperature is uniform throughout the room
OP = 10 m). and heat is transmitted through a glass
V window of area 1 m 2 and thickness 0.2 cm.
m 45° Calculate the temperature outside in °C.
O 10 m P Thermal conductivity of glass is 0.2 cal
m -1s-1. ° C-1 and mechanical equivalent of heat
If pebble hits the block at t = 1 s, the value of v is is 4.2 J cal-1.
....................... ms-1.
13. When an ideal diatomic gas is heated at
(Take, 2 = 1.414 and g = 10 ms-2)
constant pressure, the fraction of the heat
10. A steel wire of length 1m, mass 0.1 kg and of energy supplied, which increases the internal
uniform cross-sectional area of 10-6 m 2 is kept energy is ……… .
taut and is rigidly fixed at both ends. Then, 14. The size of the image of an object at infinity
the wire is cooled by 20°C. Fundamental mode formed by a convex lens of focal length 30 cm
of vibration of wire is ...................... . is 2 cm. If a concave lens of focal length 20 cm
(Take, Ysteel = 2 ´ 1011 Nm -2 and is placed between the convex lens and the
a steel = 1.21 ´ 10-5 °C -1) image at a distance of 26 cm from the convex
lens, the new image size will be ……… cm.

Section 3 (Maximum Marks : 12)


l
This section contains TWO (02) paragraphs. Based on each paragraph, there are TWO(02) questions.
l
Each question has FOUR options. ONLY ONE of these four options corresponds to the correct answer.
l
Four each question, choose the option corresponding to the correct answer.
l
Answer to each question will be evaluated according to the following marking scheme:
Full Marks : + 3 If ONLY the correct option is chosen.
Zero Marks : 0 If none of the options is chosen (i.e. the question is unanswered).
Negative Marks : - 1 In all other cases.

Paragraph X Initially, stone, water and tank are at room


A stone of mass 20 kg is dropped in a tank temperature.
containing 200 kg water from a height of 15 m. 15. DQ , DW and DU , when the stone is about to
Consider stone and water with tank as a single
enter the water are respectively
system.
(a) 2943 J, - 2943 J, 0 J
System (b) - 2943 J, 2943 J, 0 J
boundary (c) 0 J, 2943 J, 0 J
(d) 0 J, 0 J, 0 J

Stone
16. DQ , DW and DU , when the stone comes to rest
15 m at bottom of tank are respectively
Water (a) - 2943 J, 2943 J, 0 J
tank (b) 0 J, - 2943 J, 2943 J
(c) 0 J, 2943 J, 0 J
(d) 0 J, 0 J, 2943 J

PREP CATALYSIS
JEE Advanced~Practice Set 11 251

Paragraph A Y Y
A particle moves in XOY-plane such that its x and
(c) (d)
y-coordinates varies with time as, x = 3t and y = 4 sin 3t X X

17. Trajectory of the particle is


Y Y 18. Magnitude of acceleration of the particle at
p
t= s is
(a) (b) 6
X X (a) 10 ms-2 (b) 0 (c) 36 ms-2 (d) 18 ms-2

CHEMISTRY
Section 1 (Maximum Marks : 24) 23. Primary alcohol can easily be prepared from
primary alkyl halide via SN 2 reaction with
Instructions: Same as given in Physics.
aqueous NaOH. However, similar method does
19. Potassium superoxide finds use in breathing not work for the preparation of tertiary alcohol.
equipment and safeguards the user to breathe Which of the following reaction can be used for
in oxygen generated internally in the the efficient preparation of tertiary alcohol
apparatus without being exposed to toxic (tertiary butanol) from tertiary butyl bromide?
C H OH H 2 SO 4
fumes outside. The supply of oxygen is due to (a) (CH3 )3 CBr + KOH ¾¾®
2 5
¾¾®
Heat
(a) slow decomposition of KO2
H 3 O+
(b) reaction of superoxide with moisture in the exhaled air (b) (CH3 )3 CBr + CH3 COOAg(aq) ¾® ¾¾®
(c) reaction of KO2 with CO2 in the exhaled air Mg (i) O
(d) fast decomposition of KO2 (c) (CH3 )3 CBr ¾¾® ¾¾®
2

(C 2H 5 ) 2 O (ii) H 3 O+

20. The formation of O+2 [PtF6 ]- is the basis for the KOH/Ethanol
(d) (CH3 )3 CBr ¾¾¾¾® ¾¾¾¾¾®
2 6 (i) B H

formation of xenon fluorides. This is because (ii) H 2 O 2 /NaOH

(a) O2 and Xe have comparable sizes 24. The pycnometric density of NaCl crystal is
(b) Both O2 and Xe are gases .
2165 ´ 103 kg m -3 while its X-ray density is
(c) O2 and Xe have comparable ionisation enthalpies. .
2178 ´ 10-3 kg m -3 . From the given
(d) O2 and Xe have comparable electronegativities
information, the correct option(s) is/are
21. Which of the following statement is/are correct weight w
(a) molar volume of NaCl crystal = m3
for group 17 elements? 2.165 ´ 103
(a) O2 product is formed when HOCl reacts with H2O2 w
(b) molar volume from X-ray density = m3
in acidic medium 2.178 ´ 103
(b) Perchloric acid is a peroxyl acid
(c) volume occupied by NaCl crystal = 0.013 w ´ 10-3
(c) Cl 2 has the highest bond dissociation energy
(d) fraction of unoccupied sites of NaCl = 5.96 ´ 10-3
(d) Fluorine reacts with water to give HF, O2 and O3
22. In the following reaction sequence, the correct Section 2 (Maximum Marks : 24)
structure(s) of Y is/are:
- + (CH 3 ) 3COH trans
Instructions: Same as given in Physics.
CHCl3 + (CH3 )3 COK ¾¾® X ¾¾® Y
Unstable 25. Among the given species, the total number of
Cl chemicals that produce dioxygen on heating
are ……….
CCl3
(a) (b)
KClO3 , (NH 4 )2Cr2O7 , NH 4NO3 , NaNO3 , KMnO4,
K 2Cr2O7 , H 2O2, Pb3O4 , NH 4NO2
CH2Cl
26. A battery was used to supply a constant current
Cl Cl Cl Cl
of what was believed to be exactly 0.45 A as read
on a meter in the external circuit. The cell was
(c) H H (d) H CH3 based on the electrolysis of a copper sulphate
solution. During the 30 min, that current was
H 3C CH3 H3C H
allowed to flow, a total of 0.3 g of copper metal

PREP CATALYSIS
252 JEE Advanced~Practice Set 11

was deposited at the cathode. The extent to


which the meter was inaccurate is ……… %. Section 3 (Maximum Marks : 12)
27. Sulphide of cobalt metal has a cubic structure Instructions: Same as given in Physics.
with four formula weight per unit cell. If
density is 4.269 g/cc and edge length of unit Paragraph X
cell is 6.93 Å. The mass of sulphur required to An organic compound X(C 7 H11 Br) shows optical
produce 1.5 kg of this compound is ……… . isomerism as well as decolourises brown colour of
28. In a unit cell, atom A are present at all corners bromine water solution. ‘X’ on treatment with HBr in
of lattice, B atom are present at alternate faces the absence of a peroxide forms a pair of
and all edge centres. Atom C are present at diasteromers both of them are optically active. Also,
face centres left from B and at each body X with C 2 H5 ONa in C 2 H5 OH gives a single product
diagonal at a distance of (1/4)th of the body Y(C 7 H10 ). Y on treatment with ozone followed by
diagonal from corner. The total fraction of reduction with (CH 3 ) 2 S gives 1,3-cyclopentanedione
voids occupied is ……… . as one product.
29. Moles of an ideal gas with an unknown heat
33. The structure of compound X is
capacity are in an insulated piston fitted
cylinder at an initial temperature of 300 K. (a) CH3 (b) CH2Br
The piston has a mass on top of it as shown in
Br H
the diagram. The insulation in the bottom of
Br
the piston is removed and it brought into
contact with a constant temperature bath of an (c) (d)
unknown temperature. Heat flows into the
system causing the gas to both expand and
change temperature. After the temperature Br
has equilibrated, the work for this process is 34. The correct statement concerning product(s)
found to be, W = - 3.325 kJ. The temperature formed when Y is treated with excess of HCl is
of bath will be ……… . (a) a pair of enantiomers is formed in equal amount
(b) two pairs of diastereomers are formed
Vacuum Vacuum
m m (c) only a meso-dichloride is formed
Piston Piston (d) only one pair of diastereomers is formed
gas Gas
T=380K T=?
Paragraph A
C9H 8O 2 ( A) can be formed on reaction with
T=? benzaldehyde¨ with acetic anhydride in presence of
T=?
AcONa and OH/ H 2O. The product ( A) can easily
give the following transformations:
30. Decomposition of NH 4NO2( s) into N 2 and water CHO
at 500K follow a first order kinetics and the (i) (CH3CO)2O
– +
CH3COONa H2, Pd/C (i) SOCl2
total pressure inside the sealed reaction vessel C9H8O2 (B) (C)
(ii) H+ MeOH (ii) AlCl3, heat
measured after 20 min and a very long time (A)
are 120 and 750 mm Hg. The total pressure
inside the reaction vessel after 1.0 h is ……… . 35. The compound ( A) is
-1 (a) C6H5 CH == CHCOOH (b) C6H5 (CH2 )2 OH
31. 30 mL of a solution containing 9.15 g L of an
O
oxalate K x H y (C2O4 )z × nH 2O required 27 mL of ||
0.12 N NaOH in titration I and 36 mL of 0.12 (c) C6H5 CH2 ¾ C ¾OH (d) C6H5 (CH2 )2 COOH
N KMnO4 in acidic medium in titration II.
36. The compound (C) is
The value of ( x + y + z ) is …… .
32. Adsorption of ethanoic acid on wood charcoal
follows Freundlich isotherm. If x g of ethanoic (a) (b)
acid is absorbed by 500 g of wood charcoal at
320 K from 4 litres of 0.50 M ethanol solution,
then the molarity of left ethanol in solution
will be …… . (c) (d)
(Given, value of constant k = 0.20 and n = 2.45)

PREP CATALYSIS
JEE Advanced~Practice Set 11 253

MATHEMATICS
[x ] + 1 é 5ö æ1 ù
Section 1 (Maximum Marks : 24) 42. If f ( x ) = for f : ê 0, ÷ ® ç , 3ú, where [×]
{ x} + 1 ë 2ø è2 û
Instructions: Same as given in Physics.
represents greatest integer function and { ×}
37. Consider the family of all circles whose centres represents fractional part of x, then which of
lie on the straight line y = x. If this family of the following is true?
circles is represented by the differential (a) f (x) in injective discontinuous function.
equation Py ¢¢ + Qy ¢ + 1 = 0, where P and Q are (b) f (x) is surjective non-differentiable function
the functions of x , y and y¢, then which of the (c) min( lim f (x), lim f (x) = f (1)
x ®1-1 x ®1+
following statement(s) is/are true?
(d) max (x value of point of discontinuity) = f (1)
(a) P =y+ x
(b) P =y-x
(c) P + Q = 1 - x + y + y ¢ + (y ¢)2 Section 2 (Maximum Marks : 24)
(d) P - Q = x + y - y ¢ - (y ¢)2
Instructions: Same as given in Physics.
38. Let n be a positive integer with
f ( n ) = 1 ! + 2 ! + ¼ + n ! and P ( x ), Q( x ) be 43. Let 2z + 1 = 3i, where i = -1 and
polynomials in x such that é( - z )r z 2s ù
f ( x + 2) = P ( x ) f ( x + 1) + Q( x ) f ( x ) for all x ³ 1, r , s Î { 1, 2, 3}. Let P = ê 2s ú and I be the
ë z zr û
then which of the following is (are) TRUE?
(a) P (x) = - x - 2 (b) Q (x) = x + 3 identity matrix of order 2. Then, the total
(c) P (x) = x + 3 (d) Q (x) = - x - 2 number of ordered pairs (r , s) for which
P 2 + I = 0 is ……
39. For the cubic function f ( x ) = 2x3 + 9x 2 + 12x + 1,
ì |x - [x ]|, [x ] is odd
which one of the following statements(s) hold 44. Let f : R ® R, f ( x ) = í ,
good? î| x - [x + 1]|, [x ] is even
(a) f (x) is non-monotonic where [×] denotes greatest integer function,
4
(b) f (x) increases in (-¥, - 2) È (-1, ¥) and decreases in then ò f ( x ) dx is equal to …… .
(-2, - 1) -2

(c) f : R ® R is bijective. 45. The value of (1 + tan 5° )(1 + tan 10° )


(d) Inflection point occurs at x = - 3/2
(1 + tan 15° ) … (1 + tan 45° ) is equal to ……
40. If the sides a , b and c of DABC is such that 46. If logk x log5 k = logx 5, k ¹ 1, k > 0, then sum of
a b c
= = , then all values of x is equal to ……
1 + a 2b 2 a 2 + b 2 (1 - a 2 )(1 + b 2 )
which of the following is(are) True?
47. If r = a( m ´ n) + b( n ´ l ) + c( l ´ m ) and `
æ aö [l m n] = 4 where, a , b, c are scalar, then the
(a) A = 2 tan-1 ç ÷ a+ b+ c
èb ø value of is …… .
r × ( l + m + n)
(b) B = 2 tan-1 (ab)
abab
(c) Area of DABC = 48. If the tangent to the curve y = 1 - x 2 at x = a,
a2 + b 2
where 0 < a < 1, meets the axes at P and Q. If a
abbc
(d) Area of DABC = varies and the minimum value of the area of
a2 + b 2 the D OPQ is k times the area bounded by the
axes and the part of the curve for 0 < x < 1,
41. Let the unit vector a$ and b$ be perpendicular
then the value of 3k is equal to …
and the unimodular vector vector c inclined at
an angle a to both a$ and b.
$ If 49. Number of integral values of l such that the
c = la + mb + n(a ´ b ), then which of the
$ $ $ $ equation cos-1 x + cot-1 x = l possesses solution
following is(are) TRUE? is …
(a) l = m (b) n 2 = 1 - 2l 2 30
(c) n 2 = - cos 2 a (d) m 2 = 2 sin2 a 50. Number of zeroes at the end of P ( n )n + 1, is
n =5
equal to ……

PREP CATALYSIS
254 JEE Advanced~Practice Set 11

Section 3 (Maximum Marks : 12) Paragraph A


An ellipse has semi-major axis of length 2 and
Instructions: Same as given in Physics.
semi-minor axis of length 1. If slides between the
Paragraph X coordinate axes in the first quadrant, while
The rule of an ‘‘obstacle course’’ specifies that at the maintaining with both X-axis and Y-axis.
nth obstacle a person has to toss a fair 6 sides die n 53. The locus of the centre of ellipse is
times. If the sum of points in these n tosses is bigger (a) x 2 + y 2 = 3 (b) x 2 + y 2 = 5
than 2 n , the person is said to have crossed the obstacle. (c) (x - 2)2 + (y - 1)2 = 5 (d) (x - 2)2 + (y - 1)2 = 3
51. The probability that a person crosses the first 54. The locus of the foci of the ellipse is
three obstacles 1 1
(a) x 2 + y 2 + + = 16
143 100 216 100 x2 y2
(a) (b) (c) (d)
216 243 243 216 1 1
(b) x 2 + y 2 + + =4+ 2 3
52. The probability that a person crosses the first x2 y2
two obstacles but tails to cross third obstacles. 1 1
(c) x 2 + y 2 - - = 16
36 116 35 143 x2 y2
(a) (b) (c) (d) 1 1
243 216 243 243 (d) x 2 + y 2 - - =4+ 2 3
x2 y2

Paper 2
PHYSICS
Section 1 (Maximum Marks : 24)
l
This section contains SIX (06) questions.
l
Each question has FOUR options for correct answer(s). ONE OR MORE THAN ONE of these four option(s) is (are)
correct options(s).
l
For each question, choose the correct options(s) to answer the question.
l
Answer to each question will be evaluated according to the following marking scheme:
Full Marks : + 4 If only (all) the correct option(s) is (are) chosen.
Partial Marks : + 3 If all the four options are correct but ONLY three options are chosen.
Partial Marks : + 2 If three or more options are correct but ONLY two options are chosen, both of which are
correct options.
Partial Marks : + 1 If two or more options are correct but ONLY one option is chosen and it is a correct option.
Zero Marks : 0 If none of the options is chosen (i.e. the question is unanswered).
Negative Marks : - 2 In all other cases.
l
For example: If first, third and fourth are the ONLY three correct options for a question with second option being an
incorrect option; selecting only all the three correct options will result in + 4marks. Selecting only two of the three correct
options (e.g. the first and fourth options), without selecting any incorrect option (second option in this case), will result in
+2 marks. Selecting only one of the three correct options (either first or third or fourth option), without selecting any
incorrect option (second option in this case), will result in +1marks. Selecting any incorrect option(s) (second option in
this case), with or without selection of any correct option(s) will result in -2 marks.

1. A particle moves along positive branch of the (c) the acceleration of particle at t = 1s is 2 $i + $j
3
x t (d) the acceleration of particle at t = 2 s is $i + 2 $j
curve y = , where x = x and y are measured
2 3
2. A block of mass m is held 2a
in metres and t in seconds, then
1$
stationary against a rough
a
(a) the velocity of particle at t = 1s is $i + j wall by applying a force F as
2 F
1 shown in figure. Which one of
(b) the velocity of particle at t = 1s is $i + $j the following statement(s)
2
is/are correct?

PREP CATALYSIS
JEE Advanced~Practice Set 11 255

(a) Frictional force, f = mg 5. When a monochromatic light source is kept at


(b) Normal reaction, N = F a distance of 0.2 m from a photoelectric cell,
(c) F will not produce a torque the cut off voltage and the saturation current
(d) N will not produce any torque are 0.6 V and 18 mA, respectively.
3. The figure shows a container When distance of source is made 0.6 m from the
filled with a liquid of density r. A photocell, then
Four points A, B, C and D lie on (a) stopping potential is 0.2 V
the vertices of a vertical square. D B (b) stopping potential is 0.6 V
Points A and C lie on a vertical C (c) saturation current is 18 mA
line and points B and D lies on a (d) saturation current is 2 mA
horizontal line. Choose the correct statement(s)
6. In the arrangement shown in the adjacent
about the pressure at the four points.
figure, the two pulleys are fixed and the two
(a) pD = pB (b) pA < pB = pD < pC blocks A and B are made to move downwards,
pC - pA p + pA so that they accelerates at 10 m/ s2. The
(c) pD = pB = (d) pD = pB = C
2 2 arrangement achieving the above is shown in
4. One quarter of a glass cylinder of radius R and the diagram. The block C, which is fixed to the
refractive index 3/2 is placed on a flat surface. middle of the string moves upward with a
constant velocity u.
At a certain instant, q (shown in the figure)
P A æ dq ö
= 30° , if ç ÷ = +1 rad/s, then it can be
mR R è dt ø
concluded that
A point source P is kept at distance mR from the
cylinder piece.
Now, choose the correct options. θ θ
4
(a) Emergent ray is parallel to surface, if m =
3
3 B C A
(b) Emergent ray is parallel to surface, if m = u
4
(c) Image of P by first surface is formed at distance of (a) u = 5 m/s (b) u = 10 m/s
2R from A (c) u = 20 m/s (d) u = 15 m/s
(d) Image of P by first surface is formed at distance 3R
from A

Section 2 (Maximum Marks : 24)


l
This section contains EIGHT (08) questions. The answer to each question is a NUMERICAL VALUE.
l
Four each question, enter the correct numerical value (in decimal notation, truncated/rounded-off to the second
decimal place; e.g. 6.25, 7.00.- 0.33, -.30, 30.27, -127.30) using the mouse and the on-screen virtual numeric
keypad in the place designated to enter the answer.
l
Answer to each question will be evaluated according to the following marking scheme:
Full Marks : + 3 If ONLY the correct numerical value is entered as answer.
Zero Marks : 0 In all other cases.

qn In taking measurement of a diameter of a


7. In the dimensionless ratio , find the
cylinder, main scale reading is 10 divisions and
2e 0hc
Vernier scale reading is 6 divisions. Dia of
value of n, where q is electric charge, e 0 is
cylinder is ............ .
permittivity of air, h is Planck’s constant and c
is speed of light. 9. A ball is thrown up with velocity v0. Air
8. In a Vernier scale, when both jaws touch each resistance is bv, where b = 10 s-1 and v is
other, zero of Vernier scale lies right to zero of instantaneous velocity of ball.
main scale. In this condition, 4th mark on Time taken by ball to reach maximum height is
Vernier scale coincides with one of the main bv
t0. If 0 = 2.25 units, then find the value of ebt0 .
scale division. g

PREP CATALYSIS
256 JEE Advanced~Practice Set 11

10. Consider the following experimental set up 12. A particle of mass × × ×


Lens Screen m = 1.6 ´ 10-27 kg and
charge q = 1.6 ´ 10-19 C × × ×
0.5 mm A enters a region of a B
Source A
O uniform magnetic field × × ×
S
of strength 1 T 45°
perpendicularly into
q × × ×
plane of paper, as
0.15 m 1.30 m
shown in the figure at
In this two halves of a convex lens (f = 010. m) point A.
are placed symmetrically about the central axis
SO with a gap of 0.5 mm. Distance of S from lens If speed of particle is 107 ms-1 and it emerges
halves is 0.15 m and a screen is kept 1.30 m away from some point B (from the region of magnetic
from the lens halves. field), then find the distance AB.
Source S emits light of 500 nm. Let the third 13. A body of mass 2 kg is dragged with a uniform
intensity maxima occurs at point A, then find the velocity of 2 ms-1 on a rough floor. Coefficient
distance OA (in cm). of friction is 0.2, j = 4.2 J/cal and g = 9.8 ms-2.
4 7 Heat generated in 5 s is ……… cal.
11. In the given arrangement, cosa = , cosq = ,
5 11 14. In the given arrangement, friction is abscent
mass of A = 3 kg, mass of B = 8 kg and mass of everywhere and spring is ideal. Minimum
C = 4 kg. All spheres are smooth. Normal mass of block B, so that it can lift A when
reaction of wall on sphere C is .................... . released from rest is ……… .
O3
C
θ
O2
B mB

α
O1 A 10 kg
A

Section 3 (Maximum Marks : 12)


l
This section contains FOUR (04) questions.
l
Each question has TWO (02) matching lists : Column-I and Column-II.
l
FOUR option are given representing matching of element from Column-I and Column-II. ONLY ONE of these four
options corresponds to a correct matching.
l
For each question, choose the option corresponding to the correct matching.
l
Answer to each question will be evaluated according to the following marking scheme:
Full Marks : + 3 If ONLY the correct option is chosen.
Zero Marks : 0 If none of the options is chosen (i.e. the question is unanswered).
Negative Marks : - 1 In all other cases.

15. Charge Q is distributed on two identical


capacitors in parallel. Separation of plates in Column I Column II
each capacitor is d0. If the first plate of A. Charge on capacitor C1 as P. Reduces
capacitor C1 and second plate of capacitor C2 a function of time
starts moving to left with a constant speed v. B. Charge on capacitor C2 as Q. Q (d0 + vt ) / 2d0
Match the options in Column I with those in a function of time
Column II and mark the correct option from C. Current in circuit R. Q (d0 - vt ) / 2d0
the codes given below.
D. Energy of the system S. Qv / 2d0

PREP CATALYSIS
JEE Advanced~Practice Set 11 257

Codes Column I Column II


(a) A-P, B-Q, C-S, D-R A. Monoatomic P. 7 RT
(b) A-R, B-Q, C-S, D-P 2
(c) A-P, B-S, C-Q, D-R B. Diatomic Q. 3RT
(d) A-Q, B-S, C-P, D-R C. Triatomic non-linear R. 5 RT
16. Consider the given Wheatstone bridge, let 2
current in branch AD is I 2. D. Triatomic linear S. 3 RT
2
B
Q

Codes
=
40

10
P=

(a) A-P, B-Q, C-R, D-S (b) A-P, B-Q, C-S, D-R

I1 (c) A-S, B-R, C-Q, D-P (d) A-S, B-R, C-P, D-Q
I2
A C
18. When a gas is taken from a to c along the
R=

curved path, the work done by the gas (W ac ) is


15
60

S=

- 35 J and heat added to the gas is - 63 J.


D
Along path abc, work done (W abc) is - 54 J.
+ –
p
E 10Ω
b a
Match the Column I with Column II and mark
the correct option from the codes given below.
Column I Column II
A. Heat produced in t second in P. 15 l 22t c d
branch AB
V
B. Heat produced in t second in Q. 60 l 22t 1
branch BC
Also, pc = pb and ( DU int )d - ( DU int )c = 12 J.
2
C. Heat produced in t second in R. 22.5 l 22t Now, match the entries of Column I with
branch AD Column II and mark the correct option from the
D. Heat produced in t second in S. 90 l 22t codes given below.
branch DC
Column I Column II
Codes A. Wcda P. 16 J
(a) A-S, B-R, C-Q, D-P (b) A-P, B-Q, C-R, D-S
B. (DUint )cda Q. 28 J
(c) A-S, B-P, C-Q, D-R (d) A-S, B-R, C-P, D-Q
C. DUint, a - DUint, a R. 55 J
17. Match the Column I with value of kinetic
D. Qda S. 27 J
energy of 1 mol of gas given in Column II and
mark the correct option from the codes given Codes
below. (a) A-R, B-Q, C-S, D-P (b) A-R, B-S, C-Q, D-P
(c) A-S, B-R, C-Q, D-P (d) A-S, B-R, C-P, D-Q

CHEMISTRY
Section 1 (Maximum Marks : 24) concentrated ammonia. Which of the following
statement is correct regarding the given
Instructions: Same as given in Physics. informations?
19. A metal complex having composition (a) The complexes A and B are [Cr(NH3 )4 Br2 ]Cl and
Cr(NH3 )4Cl2Br has been isolated in two forms [Cr(NH3 )4 BrCl]Br
A and B. The form ‘A’ reacts with AgNO3 to (b) Hybridisation of chromium is d 2sp 3
give a white precipitate readily soluble in (c) Magnetic moment is 3
dilute aqueous ammonia, whereas ‘B ’ gives a (d) The electronic configuration of chromium is 3d 6 4s 0
in the complex
pale yellow precipritatereadily soluble in

PREP CATALYSIS
258 JEE Advanced~Practice Set 11

20. For the given aqueous reaction which of the OH


HO O
statement(s) is/are true?
Dil. H 2SO4 (c)
Excess KI +K3 [Fe(CN)6 ] ¾¾® (Brown-yellow H
HO OH
solution )
ZnSO4 OMe
OH
¾® (White precipitate + Brownish yellow OH
Na2S 2O3 O
filtrate) ¾¾® Colourless solution
(a) The first reaction is a redox reaction H
(d)
(b) White precipitate is Zn3 [Fe(CN)6 ]2 HO OH
(c) Addition of filtrate to starch solution gives blue colour OMe
(d) White precipitate is soluble in NaOH solution
21. Malonic ester react with NaOEt to produce ( A). 23. Distribution of molecular velocities was
studied by Maxwell and is given by
The major product ( A) undergoes the following
1 æ dN ö 2æ M ö
3/ 2
æ Mu 2 ö
sequence. These are ç ÷ = 4pu ç ÷ exp. ç - ÷;
CH3 CH(Br)COOC 2H5
N è du ø è 2pRT ø è 2RT ø
NaOEt
` CH 2(COOC2H5 )2 ¾¾®( A) ¾¾¾¾® ( B) 1 æ dN ö
ç ÷ is also called probability ‘P’. Based on
(i) OH - N è du ø
¾¾® CH3 ¾ CH ¾ COOH
(ii) H + the graph given below, the increasing order of
½
CH 2 ¾ COOH temperatures (T1 , T2 and T3 ) is
The compound ( B) is
(a) H5 C2OOC ¾ CH ¾ COOC2H5
| 1 dN
H3 C ¾ CH ¾ CH3 N du
T1
T2
(b) HOOC ¾ CH ¾ CH3
|
T3
H3 C ¾ CH ¾ COOH
Velocity, u
(c) H3 C ¾ CH ¾ COOC2H5
| (a)T3 < T2 < T1 (b)T1 = T2 = T3
H5 C2OOC ¾ CH ¾ COOC2H5 (c)T1 < T2 < T3 (d)T3 < T2 <<< T1
(d) H5 C2OOC ¾ CH ¾ CH3 24. The reaction between phosphorus (P4 ) and
|
H3 C ¾ CH ¾ COOC2H5 hydrogen (H 2 ) can result in the formation of
phosphorus (PH3 ) as shown:
22. The fischer presentation of D-galactose is
P4( s) + 6H 2( g) 4PH3 ( s) -
CHO
OH
The graph shows the change in concentration
H
HO H of hydrogen for this reaction in which the
HO H system was disturbed after four hours.
H OH
Concentration of

CH2OH
H2(g) mol L–1

D- galactose

The correct structure(s) of methyl a-D-galacto


pyranoside is/are
HO
HO O O
1 2 3 4 5 6 7
(a) Time (hours)
OMe
HO OH
H
Which of the following could explain the change
OH
OH in hydrogen concentration at time = 4 h?
O
(a) The volume of reaction vessel was decreased
(b) (b) A catalyst was added
OMe
HO OH (c) The pressure on the reaction mixture was decreased
(d) More phosphorus was added
H

PREP CATALYSIS
JEE Advanced~Practice Set 11 259

31. When ammonium nitrite (NH 4NO2 ) is heated,


Section 2 (Maximum Marks : 24) it decomposes to give nitrogen gas. This
Instructions: Same as given in Physics. property is used to inflate some tennis balls.
The quantity of NH 4NO2 needed to inflate a
25. The maximum number of electrons that can tennis ball to a volume of 86.2 mL at 1.2 atm
have principal quantum number n = 3 and spin and 22°C is ……… .
1
quantum number, ms = - is …. 32. A process of electrolysis of AgNO3 solution was
2
carried out between silver electrodes. Before
26. The pressure of saturated vapour over solid electrolysis 10 g of solution contained
benzene at 268.2 K is 2279.8 Pa and that of a 0.01848 g of AgNO3 . After getting electrolysed,
saturated vapour over super cooled benzene 20.09 g of the solution (anodic) contained
(liq) is 2639.7 Pa. The change in Gibb’s free 0.08327 g of AgNO3 . If at the same time,
energy in the course of solidification of one 0.009239 g of Cu was deposited in the copper
mole of super-cooled benzene at the above coulometer placed in series, then the value of
temperature is … .
transport number of NO3- will be ……… .
27. A laboratory supervisor measured the contents
of a partially filled 22.0 gallon acetone Section 3 (Maximum Marks : 12)
container on a day when the temperature was
Instructions: Same as given in Physics.
20°C and atmospheric pressure 720 mm Hg.
He found that the amount of solvent remained 33. Match each set of unbalanced chemical
was 11.8 gallon. After tightly sealing the reactions from List-I with reagent or condition
container, while carrying it upstairs to the given in List-II.
laboratory, the supervisor assistant dropped
the container. The container was dented and List-I List-II
its internal volume was decreased to p. PbO2 + H2 SO4 ¾® PbSO4 + O2 + 1. NO
16.9 gallon. The total pressure inside the other product
container after the accident will be …… . q. Na 2 S2O3 + H2O ¾® NaHSO4 2. I2
(Given, vapour pressure of acetone at 20°C is + other product
380 mm Hg)
r. N2H4 ¾® N2 + other product 3. Warm
28. For the given compound X, the total number of
s. XeF2 ¾® Xe + other product 4. Cl 2
chiral centre in the compound is ……… .
CH3 The correct option is
(a) p ® 4; q ® 2; r ® 3; s ® 1
H3C (b) p ® 3; q ® 2; r ® 1; s ® 4
(c) p ® 1; q ® 4; r ® 2; s ® 3
(d) p ® 3; q ® 4; r ® 2; s ® 1

O 34. The desired product ‘X’ can be prepared by


reacting the major product of reactions in List-I
with one or more appropriate reagents in List-II.
29. A chemist synthesizes a new acidic compound O C6H5
which has two acidic hydrogen atoms per
molecule. He dissolves a sample of 1.26 g of
pure compound in water and titrates with a
0.100 mol/L NaOH. If 75.0 mL of NaOH are O
required to reach the end point, what is the (X)
molar mass of the new acid?
List-I List-II
30. 20 mL of 0.05 N iodine is required for the
titration of a mixture containing As2O3 and p. OH 1. C5H11ONO, HCl
As2O5 . The solution obtained is then acidified OHσ
+ C6H5CHO
and excess of KI was added. To complete the
reaction liberated iodine required 1.0 g of COCH3
Na 2S2O3 × 5H 2O. The weight of the mixture is
……… .

PREP CATALYSIS
260 JEE Advanced~Practice Set 11

q. OH 2. H2 SO4 r. O (iii) Dehydration


(i) OHσ
+ C6H5CHO 18
(ii) Ac2O
CH2CH2CH2OH
COCH3
18
O
r. CHCl2 3. Br2
H2SO4
(i) H2O, NaHSO3
(ii) Na2CO3

s. CH2CH2CH2C(CH3)2 (iv) Nucleophilic


4. KOH, EtOH addition
OH
5. C6H5 (OH) ¾
×COCH2 Cl H2SO4

6. NaOH, EtOH

The correct option is H 3C CH3


(a) p ® 1, 3; q ® 2, 4; r ® 5
(v) Carbanion
(b) p ® 3; q ® 4; r ® 1, 4
(c) p ® 5; q ® 1, 2; r ® 3, 4 The correct option is/are.
(d) p ® 1, 2; q ® 4, 5; r ® 5, 6 (a) p ® iii, iv, v; q ® i, iv; r ® iii, iv; s ® ii, iii
(b) p ® i, ii; q ® v; r ® i, iv; s ® iii, iv
35. Match the reactions from List-I with (c) p ® iii; q ® ii; r ® i; s ® ii, iii
appropriate types of steps/reactive (d) p ® iii, v; q ® iii; r ® i; s ® i, iv, v
intermediate involved in these reactions as
given in List-II. 36. A sugar syrup (density = 1.2 g mL -1) of weight
214.2 g contains 34.2 g of sugar (C6H12O6 ).
List-I List-II Match the concentration parameters of
p. H3C O (i) Nucleophilic Column I with their values in Column II and
O substitution select the answer from the codes given below
the Columns.
Column I Column II
O (i) Kilomoles of water p. 5.55 ´ 10-4
(ii) Mole fraction of q. 9.9 ´ 10-3
aq. NaOH
sugar
(iii) Molality of solution r. 0.80
q. O (ii) Electrophilic (iv) Molarity of solution s. 056
.
substitution t. 0.01
CH2CH2CH2Cl
Codes
i ii iii iv
O
(a) t q p s
CH3MgI (b) p q s r
CH3
(c) p r t s
(d) r p s q

PREP CATALYSIS
JEE Advanced~Practice Set 11 261

MATHEMATICS
Section 1 (Maximum Marks : 24) 41. Two consecutive numbers from n natural
numbers 1, 2, 3, … n are removed arithmetic
Instructions: Same as given in Physics. 105
mean of remaining number is . then, which
37. Let a , l , m Î R. Consider the system of linear 4
of the following is(are) TRUE?
equations ax + 2 y = l and 3x - 2 y = m.
(a) The product of removed number is 72
Which of the following statement(s) is/are correct? (b) The value of n is 50
(a) If a = - 3, then the system has infinitely many (c) The value of sum of removed number and n is 65
solution for all values of l and m (d) The arithmetic mean of n number is 25
(b) If a ¹ - 3, then the system has a unique solution for
all values of l and m 42. If the ellipse x 2 + k2 y 2 = k2a 2 is confocal with
(c) If l + m = 0, then the system has infinitely many the hyperbola x 2 - y 2 = a 2 (k > 1). Then, which
solution for a = - 3 of the following statements is(are) TRUE?
(d) If l + m ¹ 0, then the system has no solution for
(a) Ratio of eccentricities of ellipse and hyperbola is
a = -3 1
38. Let P ( a , b) be variable point satisfying 3
4 £ a 2 + b2 £ 9 and b2 - 4ab + a 2 £ 0. Let R be (b) Ratio of major axis of ellipse and transverse axis of
hyperbola is 3
the complete equation represented in XY -plane
(c) Ratio of minor axis of ellipse and conjugate axis of
in which P can lie. Then, which of the
hyperbola is 3
following is (are) TRUE?
2p (d) Ratio of length of latusrectum of ellipse and
(a) The area of region R is 1
3 hyperbola is
3
(b) Minimum value of | a + b| for all position of P lying in
region R is 6
5p Section 2 (Maximum Marks : 24)
(c) The area of region R is
3 Instructions: Same as given in Physics.
(d) Minimum value of | a + b| for all position of P lying in
region R is 2 3 43. Let P be a matrix of order 3 ´ 3 such that all
æ1ö æ1ö
x tan ç ÷ ln ç ÷ the entries in P are from set { -1, 1}. Then, the
ènø ènø
39. If f ( x ) = lim e and maximum value of the determinant of P is ……
n ®¥ 2 2
f(x) 44. Let ò ex +x
( 4x3 + 4x 2 + 5x + 1)dx = ex +x
f ( x ) + c,
ò 3 sin11 x cos x dx = g( x ) + C, then which of the
then the value of f(1) is equal to ……
following is(are) true? 1 1 1 1 1 1
p
(a) g æç ö÷ =
3
(b) g (x) is continuous for all x
45. If 1 + 2
+ 2
+ 1+ 2
+ 2
+ 1+ 2
+
è4ø 2 1 2 2 3 3 42
p -15
1 1 1 1
(c) g æç ö÷ = + 1+ + + … + 1+ +
è4ø 8 42 52 ( 2018)2 ( 2019)2
p
(d) g (x) is not differentiable at infinitely many points = p + . Then, the value of p + q is
q
40. Consider the equation,
æ 17 ö p x2 y2
sin-1 ç x 2 - 6x + -1
÷ + cos K = , then which of 46. With one focus of the hyperbola - = 1 as
è 2ø 2 9 16
the following is(are) TRUE? the centre a circle is drawn which is tangent to
the hyperbola with no part of the circle being
(a) The largest value of K for which equation has two
distinct solution is one.
outside the hyperbola. Then, the radius of the
-1 circle is equal to ……
(b) The equation must have real root, if K Î æç , 1ö÷
è2 ø 47. The ratio in which the curve
-1
(c) The equation has unique solution, if K = é 25 1 ù
2 x=ê - sin y - cos 2 y ú, y Î [0, p ] divides the
(d) The equation must have real root, if K Î æç -1, ö÷
1 ë 12 2 û
è 2ø area bounded by the curves y = x3 - x 2 and

PREP CATALYSIS
262 JEE Advanced~Practice Set 11

m 52. AB, AC and AD are three adjacent edges of a


y = x 2 is (where, m and n are relatively
n parallelopiped. The diagonal of the
prime and [×] denotes the greatest integer parallelepiped passing through A and directed
function), then the value of m + n is ……… away from it is vector a. The vector area of the
x- 2 y+1 z -1 faces containing vertices A, B, C and A, B, D
48. The line = = intersects the are b and c, respectively, i.e. AB ´ AC = b and
3 2 -1
AD ´ AB = c. The projection of each edge AB
curve xy = c , z = 0 if c equals to
2 2
|a|
and AC on diagonal vector a is .
49. A real number a is chosen randomly and 3
uniformly from the interval [-20, 18]. The
List I List II
probability that the roots of the polynomial
x 4 + 2ax3 + ( 2a - 2)x 2 + ( -4a + 3)x - 2 are all P. AB - AD is 1. a a ´ (b - c) 3 (b ´ a )
+ +
m 3 |a |2 |a |2
real can be written in the form , where m
n Q. AB is 2. a a ´ (b + c) 2 (b ´ a )
and n are relatively prime positive integer, + +
3 |a |2 |a |2
then m + n is equal to ………
R. AC is 3. a a ´ (b - c)
50. For x > 0, the minimum value of +
6
3 |a |2
æ 1ö æ 6 1ö
çx + ÷ - çx + 6 ÷ - 2 S. AD is c ´a
è xø è x ø
4. 3×
3
is equal to …… |a |2
æ 1ö æ 3 1ö
ç x + ÷ + ç x + ÷ a ´c
è xø è x3 ø
5. 2× r 2
|a |
Section 3 (Maximum Marks : 12) 6. a a ´ (b - c) 3 (c ´ a )
+ r -
Instructions: Same as given in Physics. 3 |a |2 |a |2

51. Let y = f ( x ) and y = g( x ) be the pair of curves The correct option is


such that, (a) P ® 1; Q ® 4; R ® 6; S ® 3
(b) P ® 5; Q ® 3; R ® 2; S ® 1
(i) the tangents at point with equal abscissae
(c) P ® 4; Q ® 3; R ® 1; S ® 6
intersect of Y -axis.
(d) P ® 2; Q ® 1; R ® 2; S ® 5
(ii) the normal drawn at points with equal
abscissae intersect on X-axis and 53. Let the equations x3 + 2x 2 + px + q = 0 and
x3 + x 2 + px + r = 0 have two roots in common
(iii) one curve passes through (1, 1) and the other
one passes through (2, 3), then and the third root of each equation are
represented by a and b respectively.
List I List II ì ex log1 + x|a + b|, -1 < x < 0
ï
P. The value of f (2) is equal to 1. 3 If f ( x ) = í log( ex 2 + ab x ) is
ïb × , 0< x<1
Q. The value of g(4) is equal to 2. -6 î tan x
R. The number of positive integral 3. 9.2
continuous at x = 0
solution for f (x) = g (x) are List I List II
2 P. The value of a + b is
2 2
1. 27
S. The value of ò (g (x) - f (x))dx is 4. 4.5
1
Q. The value of lim f (x) is 2. 6
x ® 0-
5. 0
R. The value of 4b is 3. 5
6. -1
S. The value of 6(a + b) is 4. 3
The correct option is The correct option is
(a) P ® 6; Q ® 4; R ® 5; S ® 1 (a) P ® 2; Q ® 3; R ® 1; S ® 2
(b) P ® 4; Q ® 6; S ® 1; S ® 3 (b) P ® 4; Q ® 2; R ® 3; S ® 1
(c) P ® 2; Q ® 1; S ® 3; S ® 5 (c) P ® 1; Q ® 3; R ® 1; S ® 2
(d) P ® 1; Q ® 3; R ® 6; S ® 2 (d) P ® 3; Q ® 4; R ® 2; S ® 1

PREP CATALYSIS
JEE Advanced~Practice Set 11 263

54. For x , y , z , t Î R, sin-1 x + cos-1 y + sec-1z ³ The correct option is


t - 2p t + 3p
2
(a) P ® 3; Q ® 4; R ® 2; S ® 1
(b) P ® 4; Q ® 1; R ® 4; S ® 3
List I List II
(c) P ® 1; Q ® 3; R ® 1; S ® 2
P. The value of x + y is 1. p/ 2
(d) P ® 2; Q ® 3; R ® 1; S ® 3
Q. The value of xyz is 2. p
-1
R. The value of cos (min{x, y, z }) is 3. 0
S. The principal value of cos-1 (cos5t 2 ) is 4. 1

Answers
Paper 1
1. a,b,c,d 2. a,b,c,d 3. b, d 4. a, c 5. a, c 6. a, c 7. 0.80 8. 3.27 9. 7.00 10. 11.00
11. 1 : 05 12. 15.24 13. 0.71 14. 2.50 15. d 16. d 17. a 18. c 19. b, c 20. a, c
21. a, c, d 22. d 23. b 24. a, b, c 25. 6 26. 11 27. 673 28. 0.58 29. 500 30. 305.40
31. 6 32. 0.09 33. b 34. b 35. a 36. b 37. b, c 38. b, c, d 39. a, b, c 40. a, b, d
41. a, b, c 42. a, b, d 43. 1 44. 3 45. 32 46. 5.2 47. 0.25 48. 12 49. 5 50. 137
51. b 52. c 53. b 54. a

Paper 2
1. a, c 2. a,b,c,d 3. a, b, d 4. a, c 5. b, d 6. c 7. 2.00 8. 10.20 9. 3.25 10. 0.10
11. 33.00 12. 0.14 13. 9.33 14. 5.00 15. b 16. a 17. c 18. c 19. a, b, d 20. a, c, d
21. c 22. d 23. c 24. a 25. a 26. –327 27. 1060 28. 5 29. 336 30. 0.28
31. 0.27 32. 0.94 33. d 34. d 35. a 36. a 37. b, c, d 38. b, c 39. a, b, d 40. a, b, c
41. b, c 42. a, b, d 43. 4 44. 3 45. 4037 46. 3 47. 6 48. 5 49. 37 50. 6
51. a 52. c 53. d 54. a

SCORE SHEET - Paper 1


Section No. of Marks from Marks from Marks Obtained
Correct Questions Correct Questions (A) Incorrect Questions (B) (A-B)
................ ............................... ............................... ............................... ...............................
................ ............................... ............................... ............................... ...............................
................ ............................... ............................... ............................... ...............................
Percentage Marks = Marks Obtain/Total Marks x 100

SCORE SHEET - Paper 2


Section No. of Marks from Marks from Marks Obtained
Correct Questions Correct Questions (A) Incorrect Questions (B) (A-B)
................ ............................... ............................... ............................... ...............................
................ ............................... ............................... ............................... ...............................
................ ............................... ............................... ............................... ...............................
Percentage Marks = Marks Obtain/Total Marks x 100
Note To expect your success marks in the test should be between 65%-70%.

PREP CATALYSIS
PREP CATALYSIS
JEE Advanced

PRACTICE SET 12 (With Solutions)

Duration : 3 Hours Max. Marks . 360

Paper 1
PHYSICS

Section 1 (Maximum Marks : 28)


l
This section contains SEVEN questions.
l
Each question has FOUR options (a), (b), (c) and (d). ONE OR MORE THAN ONE of these four options is (are) correct.
l
For each question, darken the bubble(s) corresponding to all the correct option(s) in the ORS.
l
For each question, marks will be awarded in one of the following categories:
Full Marks : + 4 If only the bubble(s) corresponding to all the correct option(s)
is (are) darkened.
Partial Marks : + 1 For darkening a bubble corresponding to each correct option, providedNO
incorrect option is darkened
Zero Marks : 0 If none of the bubbles is darkened.
Negative Marks : - 2 In all other cases.
l
For example, if [a], [c] and [d] are all the correct options for a question, darkening all these three will get +4 marks;
darkening only [a] and [d] will get +2 marks; and darkening [a] and [b] will get-2 marks, as a wrong option is also
darkened

1. An aeroplane is flying at constant speed of 100 m/s 2. Energy released, Q values for various modes of
in a vertical circle of radius 1000 m. The force decays of 232
U are listed below.
exerted by the seat on the pilot of mass 100 kg at
lowest point of the trajectory is N A and that at the Emitted particle Energy released
highest point is N B . Then, (MeV)
1
(a) NA = 0, NB = 2000N 0n – 7.26
1
(b) NA = NB = 2000N 1H – 6.12
(c) NA = 2000N 4
2 He + 5.14
(d) NB =0 2
1H – 10.70

PREP CATALYSIS
JEE Advanced~Practice Set 12 265

Now, choose the correct options. (b) Potential energy of charges, which are at a distance of
(a) Spontaneous decay is possible only for a-particles 3 32 ke 2
a from each other is, U = - ×
(b) Neutron decay is spontaneously possible 2 3 a
(c) Spontaneous proton decay is not possible (c) Potential energy of charge pairs separated by distance
12
(d) All reactions except a-decay are endothermic 2 a is; U = ke 2
2
3. A container contains 20 g of oxygen. Translational (d) Potential energy of charge pairs which at a distance
energy of this oxygen mass is found around 2202 J. 4 ke 2
3 a from each other is, U = ×
Now, choose the correct options. 3 a
(a) Rotational energy of oxygen sample is 1468 J
6. Ten identical rods of length L, area A made of same
(b) Total energy of given sample is 3600 J material with conductivity K are joined as shown
(c) Oxygen molecule has 7 degree of freedom in the figure. Ends A and E are maintained at
(d) Temperature of sample is 10°C constant temperatures of 100°C and 0°C.
Assuming, no radiative and convective heat loss.
4. Consider two tunnels through earth as shown in
Now, choose the correct options.
figure, let a small body is dropped in any of tunnel.
5
2 6
Tunnel 1 C
1 3 7 10
Tunnel 2 A B 4 8 D E
TA = 100°C 9 TE = 0°C

(a) Total thermal resistance of combination


12 æ L ö
is ç ÷
11 è KA ø
38 æ L ö
(b) Total thermal resistance of combination is ç ÷
15 è KA ø
(a) Time period of oscillation of a body in tunnel 1 is about 750 æ KA ö
(c) Heat flow rate is ç ÷
42 min 19 è L ø
(b) Time period of oscillation of a body in tunnel 2 is 225 æ KA ö
R (d) Heat flow rate is ç ÷
T=p 19 è L ø
g
(c) Time period of a body in tunnel 2 is nearly half that of a 7. A particle of 100 g mass is moving along X-axis
low orbiting body
under action of a force F = - 10 x(N). At t = 2 s,
(d) Time period in tunnel 1 and tunnel 2 are same
particle is crossing mean position and at t = 4 s, its
5. A cube of side a contains a charge of - e at its every
speed is 4 ms-1 . Then, choose the correct options.
corner and a charge of + 2e is placed at the centre
of cube. Now, choose the correct options. (a) Frequency (w) for particle is 10 rad s-1
(b) Frequency (w) for particle is 20 rad s-1
(a) Potential energy of charges, which are at a distance a 4
12 ke 2 (c) Displacement of particle is x = × sin 10° (t - 2 )
from each other is; U = , 10cos 20 °
a (d) Displacement of particle is
1
where k = x=
4
× cos 10° (t - 2 )
4pe0 10 cos 20 °

PREP CATALYSIS
Section 2 (Maximum Marks : 15)
l This section contains FIVE questions
l The answer to each question is a SINGLE DIGIT INTEGER ranging from 0 to 9, both inclusive.
l For each question, darken the bubble corresponding to the correct integer in the ORS.
l For each question, marks will be awarded in one of the following categories:
Full Marks : + 3 If only the bubble corresponding to all the correct answer is darkened.
Zero Marks : 0 In all other cases.

8. In a Young’s double slit experiment, we have sga 2 k


s as shown in the figure is . Find the value
following set up. n
Separation D of screen and slits = 1 m. of n.
Wavelength of light used is l = 600 nm. 11. A block of mass m is connected to a peg at point D
Distance of 10th bright band from central fringe is through a light and inextensible string of length
30 mm. 18 m. The length of section AB is 3 m. The point C
When a film of a transparent material is placed over of string is pulled down by applying a force F of
one of the slits, centre of fringe pattern displaced by 128 N. The block A remains stationary. If the mass
30 mm. If refractive index of transparent material is 16
m of block A is kg, find the value of n.
m, then find the value of . n
13
.
B 9m
9. It is found that, when 0.5 m was cut-off from the D
muzzle of a gun firing block of 50 kg, the velocity
A
of the block was changed from 700 m/s to 600 m/s.
th
The force exerted on the block by the powder of
m oo
m
Sm
C
gas at the muzzle when expanded in the bore was 45° F
6.5 ´ 10n N. Find the value of n.
12. Out of given following transitions; number of
10. If X-axis is frequently possible transitions is ……… .
which is Y
O E(eV)
paralledl to y2=kx
ground taken as (a, b) – 0.85eV 4s 4p 4d 4f
reference line
and Y-axis in – 1.5eV 3s 3p 3d
vertically upward
direction. The – 3.4eV 2s 2p
potential energy X
of a parabolic plate y 2 = kx of surface mass density – 13.6eV 1s

PREP CATALYSIS
JEE Advanced~Practice Set 12 267

Section 3 (Maximum Marks : 18)


l This section contains SIX questions of matching type.
l The section contains TWO tables (each having 3 columns and 4 rows)
l Based on each table, there are THREE questions.
l Each question has FOUR options [a], [b], [c] and [d]. ONLY ONE of these four options is correct.
l For each question, darken the bubble corresponding to the correct option in the ORS.
l For each question, Marks will be awarded in one of the following categories:
Full Marks : + 3 If only the bubble corresponding to the correct option isdarkened
Zero Marks : 0 If none of the bubbles is darkened
Negative Marks : -1 In all other cases

Direction (Q. Nos. 13-15) Consider a straight conductor, which is moved in region of earth’s magnetic field. In Column
I, II and III direction of motion of conductor and its effects are given. In Column I, direction of motion of conductor is
towards east or west. In Column II, direction of motion of conductor is from north to south or from south to north. In
Column II, orientation of conductor is given.
Let v = speed of conductor, B H and B V indicates horizontal and vertical component of earth’s field.

Column I Column II Column III


A. Conductor does not cuts any field I. Conductor moves in magnetic P. N

lines, so induced emf = 0 meridian, so induced emf = 0


E

Conductor

N
B. Conducor cuts horizontal II. Conductor cuts vertical component, Q.
component, so induced emf hence induced
is BH lv emf = Bv vl
E

Conductor
N
C. Conductor cuts both components of III. Conductor is moving along its length, R.
field, so induced emf is 2 Bv lv so induced emf is zero

Conductor
D. Conductor cuts field lines and IV. Conductor cuts both components of S. N
induced emf = BV vl field, so induced emf is 2 BH lv
45°

13. In this case, if conductor is moved vertically up or


down, then induced emf in conductor 15. In this case, when conductor is moved vertically up
is B H lv. or down, then induced emf is zero, because
conductor is moving along its length.
(a) A II P (b) D III Q
(c) C IV S (d) B I R (a) A II P (b) D III Q (c) C IV S (d) B I R
Direction (Q. Nos. 16-18) Consider 3 different bulbs,
14. In this case, if conductor is moved vertically up or
down, then induced emf is zero, because conductor
Bulb B1 (25 W, 220 V), Bulb B 2 (100 W, 220 V) and Bulb B 3
(1000 W, 220 V).
is moving in magnetic meridian.
(a) A II P (b) D III Q
In Column I, information about resistance of bulb’s
filament is given.
(c) C IV S (d) B I R
In Column II, information about thickness of filament is
given.

PREP CATALYSIS
268 JEE Advanced~Practice Set 12

In Column III, information about brightness of bulbs is 16. For bulb B1, best match is
given.
(a) A III R (b) B II Q
Column I Column II Column III (c) C IV S (d) D I P
A. Resistance I. Filament P. Bulb does not
of bulb is largest thickness is glows 17. For bulb B 2, best combination is
nearly zero (a) A III R (b) B II Q
B. Resistance II. Filament Q. Brightness of (c) C II S (d) D I P
of bulb is thickness is bulb is greatest
smallest moderate 18. For bulb B 3, best combination is
C. Resistance III. Filament R. Brightness of (a) A III R
of bulb is thickness is bulb is least (b) C II S
moderate smallest (c) D II Q
D. Resistance IV. Filament S. Brightness of (d) B IV Q
of bulb is zero thickness is bulb is
largest intermediate

CHEMISTRY
Section 1 (Maximum Marks : 28) OH NaNO2/HCl OH
Instructions : Same as given in Physics (c) NH2 OH
19. For the reduction of NO-3
ion in an aqueous
solution E° is +0 .96 V. Values of E° for some metal NaNO2/HCl
(d)
ions are given below. H2O
NH2 OH
V 2 + (aq) + 2 e- ¾® V; E° = -119
. V Optically active Optically inactive

Fe3 + (aq) + 3 e- ¾® Fe; E° = - 0 .04 V 22. In the brown ring test for NO-3 ion, complexes
Au3 + (aq) + 3 e- ¾® Au ; E° = 1.40 V [Fe(H 2O)5 NO]2+ is formed. In this complex
(a) NO transfers its electron to Fe2+ such that iron is Fe+ and
Hg2 + (aq) + 2 e- ¾® Hg ; E° = 0 .86 V NO is NO+
The pairs of metals that are oxidised by NO-3 in (b) three unpaired electrons are present so that its magnetic
moment is 3.87 BM.
aqueous solution are (c) the colour is due to charge transfer.
(a) V and Hg (d) two unpaired electrons are present so that its magnetic
(b) Hg and Fe moment is 1.414
(c) Fe and Au
23. Among the following, which alkyl bromide
(d) Fe and V
proceeds with retention of configuration in SN 2
20. Which of the following statements are correct reaction with CH3ONa(aq)?
regarding [Cu(NH3 )4 ]2+ ion? H H
H3C H 3C
(a) It involves sp3 -hybridisation with tetrahedral geometry
(a) (b)
(b) Cu is tetra coordinated and exist as square planar PhS Br PhS Br
complex.
(c) It involves dsp2 hybridisation
Br H 3C H
(d) It involves sp2d hybridisation (c) PhS (d)
21. Which of the following reactions is/are correct? H 3C H PhS Br
OH NaNO2/HCl 24. A power company burns approximately 474 tons of
(a) O
NH2 coal per day to produce electricity. If the sulphur
content of the coal is 1.30% by weight, then how
OH NaNO2/HCl OH many tons of SO2 are dumped into the atmosphere
+ –
(b) NH2 N2Cl each day?
(a) 0.19 (b) 3.08 (c) 6.16 (d) 12.3

PREP CATALYSIS
JEE Advanced~Practice Set 12 269

25. Which of the following is/are correct order?


Section 3 (Maximum Marks : 18)
(a) Increasing bond angle : SbH3 < AsH3 < PH3 < NH3
Instructions : Same as given in Physics
(b) Increasing boiling point : PH3 < NH3
(c) Increasing basic character : SbH3 > AsH3 > PH3 > NH3 Direction (Q. Nos. 31-33) Answer by appropriately
(d) Increasing acidic nature NF3 < NCl 3
matching the information given in the three columns of
following table. Column I, II and III contains crystal
system, intercept and crystal angle respectively.
Section 2 (Maximum Marks : 15) Column I
Instructions : Same as given in Physics Column II Column III
(Crystal
(Intercept) (Crystal angle)
26. Phosphorus has eight electrons in the penultimate system)
shell. It exists in X forms. When this X combines A. Cubic I. a¹ b¹c P. a ¹ b ¹ g ¹ 90 °
with oxygen, it readily forms Y, which is a waxy B. Tetragonal II. a= b¹c Q. a = g = 90°, b ¹ 90 °
solid having white colour. The number of P-O-P
C. Hexagonal III. a= b=c R. a = b = 90°, g = 120 °
bonds in the Y are
D. IV. S. a = b = g = 90 °
27. At 1000 K, pressure of CO 2 in equilibrium with
CaCO 3 and CaO is equal to 2.105 atm. The 31. Which of the following combination is correct for
equilibrium constant for the reaction, zinc blend?
(a) A II P (b) B III Q (c) C IV R (d) A III S
C(s) + CO2 (g) 4 2 CO(g)
is 1.9 at the same temperature when pressure are in 32. Which of the following combination is incorrect
atm. solid C, CaO and CaCO3 are mixed and for Mg and diamond?
allowed to come to equilibrium at 1000 K in a (a) C IV R (b) A III S
closed vessel. What is the pressure of CO(g) at (c) Both (a) and (b) (d) C II Q
equilibrium (in atm)? 33. Which of the following is correct for white tin?
28. Among the following, the number of reaction(s) (a) C III P (b) B II S (c) A I Q (d) B II R
that produce(s) alcohol is/are
Column I Column II Column III
(i) THF, D
(i) H ¾ C ºº N + C2H5 MgBr ¾¾®+ A. COONa
–+ I.
+ MgBr2
P. Kolbe’s
(ii) H3 O electrolytic
Electrolysis
→ reaction
(i) THF aq. sol.
(ii) CH 2 == O + PhMgBr ¾¾® – +
(ii) H3 O+ COONa

O B. Br Br II. Q. Freenkel
(i) THF, ∆ + Mg → I reaction
(iii) + MeMgBr
(ii) H3O+ + NaBr
Ph Ph
C. 1. Li, CuI
→ III. CHF2Cl R. Swart’s
Me Cl
2. CH I 3 reaction
(i) THF,
(iv) MeMgBr +
O (ii) H3O+ D. SbF3 IV. S. Finkelstein
CHCl 3 ¾¾® reaction
HF
29. IO 2 F2- has trigonal bipyramidal geometry. How E. ∆ V. Me T. Corey House
+ NaI →
Acetone
many lone pair(s) is/are lost to convert its geometry synthesis
Br
to sea-saw? Me

30. In zero-gravity environment, a sample of mercury 34. Which of the following is the correct option?
has a spherical shape. And such sample has a
(a) A V P (b) B III R (c) C IV S (d) D V S
radius of 1.00 cm. If 10 drops are formed x ´ 10 -4 J
work is performed. Surface tension of mercury is 35. Which of the following is correct for Swart’s
reaction?
414 erg/cm 2 . Assume that only work done is
related to change in surface energy. The value of x (a) D III R (b) B II R (c) A IV R (d) E V R
is ……… . 36. Corey house synthesis is represented by which
option?
(a) C II T (b) C IV T (c) B II T (d) E IV T

PREP CATALYSIS
270 JEE Advanced~Practice Set 12

MATHEMATICS
bc ca ab
(c) + + £ p
Section 1 (Maximum Marks : 28) a b c
Instructions : Same as given in Physics (d) None of the above
37. Probability of n heads in 2n tosses of a fair coin can 43. Which of the following function has point of
be given by extrema at x = 0 ?
n
2 å( C r )
n 2
(a) f(x ) = e - | x|
n 2r - 1 n+ r
n næ nC ö
(a) Õ (b) Õ (c) å ç n r ÷ (d) r =0
2n (b) f(x ) = sin| x |
r = 0è 2 ø
r =1 2r r = 1 2r
å
2n
Cr ìx 2 + 4x + 3 ; x < 0
r =0 (c) f(x ) = í
î -x ; x³0
38. a, b and c are unimodular and coplanar. A unit x<0
ì| x | ;
vector d is perpendicular to them. If (d) f(x ) = í
î { x } ; 0 £ x <1
1 1 1 $
(a ´ b) ´ (c ´ d) = $i - $j + k and the angle (where; {x} represents fractional part function)
6 3 3
between a and b is 30°, then c is equal to
1 $ 1 $ Section 2 (Maximum Marks : 15)
(a) (i - 2 $j + 2 k$ ) (b) (- i + 2 $j - 2 k$ )
3 3 Instructions : Same as given in Physics
1 1
(c) (2 $i + 2 $j - k$ ) (d) (- 2 $i - 2 $j + k$ ) 44. The infinite geometric series
3 3
¥
a
39. Let A(k) be the area bounded by the curves å (y 2 - 4 y + 5)r - 1
; where, y = x 2 - 6 x + 11; has a
r =0
y = x 2 - 3 and y = kx + 2.
finite sum, then x cannot be equal to
é 10 5 ö
(a) The range of A(k ) is ê , ¥÷
ø æ 5p ö
ë 3 45. The value of the expression E= log p ç tan ÷
é 20 5 ö sin è 12 ø
10
(b) The range of A(k ) is ê , ¥÷
ë 3 ø æ 401p ö
+ log æ p 2 p ö cot ç ÷ is
(c) If function k ® A(k ) is defined for k Î[- 2, ¥), then A(k ) is ç cos + cos ÷ è 12 ø
è 5 3 ø
many-one function
(d) The value of k for which area is minimum is 1 46. If the length of the projection of the line segment
40. The roots of the equation (a + b) x 2 - 15 with points (1, 0, - 1) and (- 1, 2 , 2) to the plane
2 éd ù
+ (a - b)x - 15
= 2a, where a 2 - b = 1 are x + 3 y - 5 z = 6 is d, then the value of ê ú, where [.]
ë2û
(a) ± 3 (b) ± 4 (c) ± 14 (d) ± 5
represents the greatest integer function, is ……… .
41. If (a,a 2) lies inside the triangle formed by the lines
47. Let f : R ® R, be a continuous function which
2 x + 3 y - 1 = 0 , x + 2 y - 3 = 0 and 5 x - 6 y - 1 = 0 , x
then satisfies f (x) = ò f (t)dt. Then, the value of f(ln 5) is
(a) 2 a + 3a2 - 1 > 0 (b) a + 2 a2 - 3 > 0 0
(c) a + 2 a - 3 < 0
2
(d) 6a2 - 5a + 1 > 0 ……… .

42. For positive real numbers a , b and c, such that xn xn + 2 xn + 4


æ1 1 öæ1 1ö
a + b + c = p, which one holds? 48. If y n yn + 2 yn + 4 = ç 2 - 2 ÷ ç 2 - 2 ÷
èy x ø èz y ø
(a) ( p - a)( p - b )( p - c ) £
8 3
p zn zn + 2 zn + 4
27
æ 1 1ö -1
ç 2 - 2 ÷, then is equal to ……… .
(b) ( p - a)( p - b )( p - c ) ³ 8abc èx z ø n

PREP CATALYSIS
JEE Advanced~Practice Set 12 271

51. Which of the following options is the only


Section 3 (Maximum Marks : 18) incorrect combination?
Instructions : Same as given in Physics
(a) A IV S (b) D III Q
Directions (Q. Nos. 49, 50 and 51) by appropriately (c) C I P (d) D I Q
matching the information given in three columns of the
following table. Directions (Q. Nos. 52, 53 and 54) by appropriately
Let f ( x ) = x 2 + bx + c , where D = b 2 - 4ac > 0 matching the information given in the three columns of
the following table.
Column I Contain the information about b and c.
Column I, II and III contain conics, equation of tangents to
Column II Contain the graph of y = f (| x | )
the conics at ( x 1 , y 1 ) and equation of normals to the conics
Column III Contain the information about the points of at ( x 1 , y 1 ) respectively.
non-differentiability of g ( x ) = | f (| x | )|
Column I Column II Column III Column I Column II Column III

A. b < 0, c > 0 I. Y P. 1 A. x + y =a
2 2 2
I. xx1 + yy1a = a
2 2
P. a2
y
x-
x1 y1

X′ X = a2 - 1
O
B. y 2 = 4ax II. xx1 - yy1a 2 = a 2 Q. xy1 = x1 y
Y′
C. x 2 + a 2 y 2= a 2 III. yy1 = 2 a ( x + x1 ) a2 y
B. c = 0, b < 0 II. Y′ Q. 2 R. x+
x1 y1
= a2 + 1
O X
X′
D. x 2 - a 2 y 2= a 2 IV xx1 + yy1 = a 2 y - y1
S. y1
. =- ( x - x1 )
Y
2a
C. c = 0, b > 0 III. Y′ R. 3
52. For a = 2, if a tangent has equation x + y =2 2 at
X′ X ( 2 , 2), then which of the following options is the
O
only correct combination for obtaining its
equation?
Y
(a) A IV Q (b) B III S
D. b = 0, c < 0 IV. Y′ S. 5 (c) C II P (d) D III R
53. The tangent to a suitable conic at ( 3 , 1 / 2) is
X′ X
3 x + 2 y = 4 , then which of the following option is
O
Y
the only correct combination?
(a) D II R (b) C I P
49. Which of the following options is the only correct (c) B IV P (d) C III R
combination? 1
54. For a = , the equation of normal passing through
(a) A IV S (b) B II P (c) C II R (d) D III P 2
(3, 2) is 2 x + y = 8 , then which of the following
50. Which of the following options is the only correct option is the only correct combination?
combination?
(a) A II Q (b) B I R
(a) A IV R (b) B II R (c) C II Q (d) D III R (c) A IV Q (d) B III S

PREP CATALYSIS
272 JEE Advanced~Practice Set 12

Paper 2
PHYSICS
Section 1 (Maximum Marks : 21)
l This section contains SEVEN questions.
l Each question has FOUR options (a), (b), (c) and (d). ONLY ONE of these four options is correct.
l For each question, darken the bubble corresponding to the correct option in the ORS.
l For each question, marks will be awarded in one of the following categories.
Full Marks : + 3 If only the bubble corresponding to the correct option is darkened.
Zero Marks : 0 If none of the bubbles is darkened.
Negative Marks : - 1 In all other cases

l=0
1. An isosceles prism of angle 120° value of the integral ò - E × dl (I = 0 being
has a refractive index 1.44. Two l=¥

parallel rays of monochromatic Rays centre of the ring) in volt is


light enter the prisms parallel to (a) + 2 (b) - 1 (c) -2 (d) zero
each other in air as shown in 120°
5. In an experiment of photoelectricity, light from
figure. The rays emerging from source is focussed over cathode of cell using a
the opposite face convex lens. Let current in this setting is I. If
(a) are parallel to each other central part of lens is covered with a black paper
(b) are diverging (as shown), then current through the photocell is
(c) make an angle 2 [sin-1 (0.72) - 30° ] with each other now
(d) make an angle 2 sin-1 (0.72) with each other
Lens
2. Time required to convert 1.5 mol of water at 100°C
completely into steam at 100°C (heat is supplied at d d/2 Black
rate of 17 Js -1 ) is paper

(a) around 1 h (b) around 4 h


1 I 3 2
(c) around 5 min (d) around min (a) I (b) (c) I (d) I
2 2 4 3
3. A proton is bombarded over a Li nucleus. Due to 6. A cube of side a rests over an inclined plane of
collision, two a-particles are produced. angle q.
α
θ
p Li ⇒
θ

Direction of motion of a-particles with the direction 45°


of bombarding proton is at angle q such that
1 The block will topple only when q is more than
cos q = . The correct relation of kinetic energy of
4 (a) 30° (b) 60° (c) 45° (d) 75°
proton (K p) and kinetic energy of a-particle (K a) is 7. Frequency of sound of a bell is 500 Hz. The velocity
1
(a) K p = 2 K a (b) K p = Ka of bell relative to still air is 60 ms-1 . An observer
2
(c) 2 K p = 3K a (d) K p = K a moves at 30 ms-1 along the same line as the source.
S O
4. A non-conducting ring of radius 0.5 m carries a
-10
total charge of 1.11 ´ 10 C distributed Source Observer

non-uniformly on its circumference producing an Sound frequency recorded by observer is


electric field E everywhere in space. The (a) 500 Hz (b) 553 Hz (c) 462 Hz (d) 660 Hz

PREP CATALYSIS
JEE Advanced~Practice Set 12 273

Section 2 (Maximum Marks : 28)


l This section contains SEVEN questions.
l Each question has FOUR options (a), (b) (c) and (d). ONE OR MORE THAN ONE of these four option(s) is (are)
correct.
l For each question, darken the bubble(s) corresponding to all the correct option(s) in the ORS.
l For each questions, marks will be awarded in one of the following categories :
Full Marks : + 4 If only the bubble(s) corresponding to all the correct option(s) is (are)
darkened.
Partial Marks : + 1 For darkening a bubble corresponding to each correct option, provided NO
incorrect option is darkened.
Zero Marks : 0 If none of the bubbles is darkened.
Negative Marks : - 2 In all other cases.
For example, if (a), (c) and (d) are all the correct options for a question, darkening all these three will get in + 4 marks;
darkening only (a) and (d) will get + 2 marks and darkening (a) and (b) will result in - 2 marks, as a wrong option is
also darkened.

8. A rod of length 1 m, whose linear mass density 11. In the given circuit,
increases uniformly from 1 kg/m to 2 kg/m is being 90V
g 8Ω
pulled along its length by a constant force F = 15 N c
a
on a rough horizontal surface. The coefficient of f e
kinetic friction between the rod and the horizontal R 5Ω 20W
surface is m k = 0.5. Then,
(a) the friction on the rod is 7.5 N b
(b) the acceleration of the rod is 5 m/s 2 h 13Ω d 7Ω
(c) the velocity of the rod at t = 2 is 20 m/s If power dissipated in 5W resistor is 20 W and value
(d) the distance travelled by the rod before coming in rest is
of resistance R is unknown, then
20 m
(a) Vab = 10 V (b) Vab = 9 V
9. A 600 pF parallel plate capacitor A is charged to (c) Vgc = 44 V (d) Vgc = 40 V
200 V. After sometime, it is disconnected from 12. A parallel plate capacitor is partially filled with a
supply and connected with another capacitor B of
dielectric of dielectric constant K = 5. Then
600 pF. Plates of B are then pulled apart to make
separation twice. (a) electric field inside the dielectric is 20% less than the
electric field due to free charges in air gap
Now, choose the correct options.
(b) electric field inside the dielectric is 10% less than the
(a) Energy gain in the process of pulling plates apart is electric field in air gap
33.33%
(c) bound charge density on the surface of dielectric is 20%
(b) Energy gain in the process of pulling plates apart is - less than the free charge density on plates of capacitor
33.33%
(d) bound charge density is 10% less than the free charge
(c) Ratio of final energies stored in A and B is 2 : 1 density on the plates of capacitor
(d) Ratio of final charges on plates of A and B is 2 : 1
13. A beam of ultraviolet light of all wavelengths
10. An aluminium bush is to be fitted over a brass rod. passes through the hydrogen gas at room
At 20°C inner diameter of aluminium ring is temperature in x-direction. Assume that all
5.00 cm and brass rod has a diameter of 5.05 cm. photons emitted due to electronic transitions of the
(Take, a Al = 24 ´ 10 -6 ° C-1 and hydrogen gas are confined to move in y-direction.
a brass = 19 ´ 10 -6 ° C-1)
Emitted
To make the assembly, which of the following can photons
be performed?
(a) Aluminium ring is heated above 437°C
(b) Brass rod is cooled UV Hydrogen Semi-reflecting
(c) Both aluminium ring and brass rod can be heated gas coated mirror
(aAl > a brass )
(d) Both aluminium ring and brass rod can be cooled using Transmitted UV
Reflecting
liquid nitrogen mirror

PREP CATALYSIS
274 JEE Advanced~Practice Set 12

Now, choose the correct option(s). (a) Its atomic number is 3


(a) Some of the incident wavelengths are absent in x-direction (b) An electron of 90 V excite it when atom is at room
(b) Only wavelengths are present in y-direction which are temperature
absent in x-direction (c) An electron of 91.8 eV kinetic energy can be brought
(c) In y-direction, some visible light is present almost to rest by this atom
(d) In y-direction, some infrared light is present (d) An electron of kinetic energy of 2.6 eV may emerge,
when electron of kinetic energy of 125 eV collides with
14. A particular hydrogen like atom has its ground this atom
state binding energy of 122.4 eV. Then, choose the
correct option(s).

Section 3 (Maximum Marks : 12)


l This section contains TWO (02) paragraphs. Based on each paragraph, there are TWO(02) questions.
l Each question has FOUR options. ONLY ONE of these four options corresponds to the correct answer.
l Four each question, choose the option corresponding to the correct answer.
l Answer to each question will be evaluated according to the following marking scheme:
Full Marks : + 3 If ONLY the correct option is chosen.
Zero Marks : 0 If none of the options is chosen (i.e. the question is unanswered).
Negative Marks : - 1 In all other cases.

Passage X Passage A
A
The mass of nucleus X is less that the sum of the masses
Z A new roller coaster ride has a giant compressed spring
of ( A - Z ) number of neutrons and Z number of protons in OFF, which the roller coaster car repeatedly bounces,
the nucleus. The energy equivalent to the corresponding gaining energy and eventually being pushed over a hill
mass difference is known as the binding energy of the (see diagram). The spring is always recompressed a
nucleus. A heavy nucleus of mass M can break into two distance of 2 m from its uncompressed position before
light nuclei of masses m1 and m 2 only, if (m1 + m 2 ) < M. each bounce.
Also, two light nuclei of masses m 3 and m 4 can undergo At the start, the car is up against the compressed spring,
complete fusion and form a heavy nucleus of mass M ¢ which is then released, pushing the car up the hill. It then
only, if (m 3 + m 4 ) > M ¢. The masses of some neutral rolls back down into the recompressed spring, which is
atoms are given in the table below then released, ejecting the car with more energy. This
1
1H 1.007825u 2
1H 2.014102u repeats until the car goes over the 10 m high hill. The
6 7
loaded car has a mass of 1000 kg.
3 Li 6.01513u 3Li 7.016004u
Uncompressed
152 206
64 Gd 151.919803u 82 Pb 205.974455u position
3 4 x
1H 3.016050u 2 He 4.002603u
70 82
30 Zn 69.925325u 34 Se 81.916709u 10 m

15. The correct statement is


(a) the nucleus 63 Li can emit an alpha particle Assume the spring obeys Hooke’s law, i.e. the force F it
210
exerts on the car is proportional to its compression
(b) the nucleus 84 Po can emit a proton x , F = - kx , where the spring constant k = 5 kN/m. The
(c) deuteron and alpha particle can undergo complete potential energy E stored in the spring is E = 0.5 kx 2 .
fusion 17. The energy gained by car in each bounce is
82
(d) the nuclei 70
30 Znand 84 Secan undergo complete fusion (a) 5 kJ (b) 98 kJ
(c) 10 kJ (d) 108 kJ
16. The kinetic energy (in keV) of the alpha particle,
210 18. How many bounces are needed, including the start
when the nucleus 84 Po at rest undergoes alpha
off, for the car to go over hill?
decay is
(a) 1 (b) 5
(a) 5316 (b) 5422
(c) 11 (d) 10
(c) 5707 (d) 5818

PREP CATALYSIS
JEE Advanced~Practice Set 12 275

CHEMISTRY
Section 1 (Maximum Marks : 21) HOOC HOOC
Instructions : Same as given in Physics (a) O (b)
O
19. Two students use the same stock solution of zinc O
HO
sulphate (ZnSO4) and a solution of copper sulphate CH3 CH3
(CuSO4). The emf of one cell is 0.03 V higher than
O O O
the other. The concentration of copper sulphate in
HOOC
the cell with higher emf value is 0.5 M. The O
(c) H3C CH3
concentration of CuSO4 in the other cell (d)
æ RT ö
ç2.303 = 0 .06 ÷ is HO HO
è F ø
(a) 0.01 M (b) 0.09 M (c) 0.05 M (d) 0.02 M 23. Ti 3+ ,V 3+ , Fe3+ and Co 2+ affords a large number of
tetrahedral complexes, while Cr3+ never.
20. Consider the following molecules.
The correct reason for this is
4
3
3 N(b) (a) Cr 3+ imparts high crystal field splitting with different
ligands
2
1 2
N (b) in Cr 3+ system, crystal field stabilisation energy plays a
N 1 N (a)
H deciding role for the formation of tetrahedral or
H
octahedral complexes
Pyridine Pyrrole Imidazole (c) the ionic radius of Cr 3+ is the largest among the other
(I) (II) (III)
M 3+ ions mentioned in the problem
Which one of the following statement(s) is/are (d) electronegativity of Cr 3+ is the largest among these
incorrect? trivalent 3d-metal ions. Hence, chromium prefers to be
(a) (I) and (III) are bronsted bases whereas (II) is not associated with as many ligands as its ionic radius
permits
(b) In (III), N a is more basic than N b
(c) When (III) is protonated in the presence of a strong acid, 24. Given, H 2S(g) ¾® HS(g) + H(g);
protonation occurs at C-2.
DH° = 376.6 kJ mol -1
(d) All the nitrogen present in (I), (II), (III) are sp2 hybridised
H 2(g) + S(g) ¾® H 2S(g); DH° = - 20 .1 kJ mol-1
21. A certain reaction proceeds in a sequence of three
elementary steps with the rate constants K a , K b H 2(g) ¾® 2H(g); DH° = 436 .0 kJ mol -1
and K c . If the observed rate constant (K obs) of the
1/ 3
S(g) ¾® S(g); DH° = 277 .0 kJ mol-1
éK ù
reaction is expressed as K obs = ê c ú ´ K a ; the Thus, D fH° (HS) is
ëK b û (a) 138.5 kJ mol -1
observed activation energy of the reaction is
1/ 3
(b) -238.5 kJ mol -1
Ec + Ea éE ù (c) -297.0 kJ mol -1
(a) (b) Ea ê c ú
3 ë Eb û (d) +277.0 kJ mol -1
1 é Ec ù 1
(c) ê ú + Ea (d) Ea + [Ec - Eb ] 25. Red precipitate is obtained when ethanolic
3 ë Eb û 3 solution of dimethylglyoxime (DMG) is added to
ammoniacal Ni (II).
22. The major product formed in the reaction. Which of the following statements is incorrect?
O O (a) Red complex has a square planar geometry
HOOC (b) Complex has symmetrical H-bonding
H 3C O CH3
H+
(c) Red complex has a tetrahedral geometry
HO (d) Dimethylglyoxime functions as bidentate ligand

PREP CATALYSIS
276 JEE Advanced~Practice Set 12

Kp
Section 2 (Maximum Marks : 28) (d) For the equilibrium, A(g ) - 2 B(g ), X = Kp + 4p
Instructions : Same as given in Physics
(X = Degree of dissociation)
26. The magnetic moment of [Mn(CN)6]3- is 2.8 BM
and the magnetic moment of [MnBr4 ]2- is 5.9 BM. 31. The major product of the following reaction is/are
3- 2- CH3
The geometries of [Mn(CN)6 ] and [MnBr4 ]
respectively, H3 C
Anhyd
(a) both are octahedral + 3C
H AlCl3
(b) trigonal planar and tetrahedral H3 C Cl
(c) tetrahedral and octahedral CH3 CH3
(d) octahedral and tetrahedral
CH3
27. Which of the following statement(s) is/are true CH3
(a) (b)
about the separate solution of 0.500 M C2H 5OH(aq), CH3
0.100 M Mg 3 (PO 4)2(aq), 0.250 M KBr (aq) and 0.125
M Na 3PO 4(aq) at 25°C CH3
[Assuming all salts to be strong electrolytes]? CH3
(a) 0.100 M Mg3 (PO4 )2 (aq ) has the highest osmotic CH3 CH3
pressure
CH3 H3 C
(b) 0.125 M Na 3PO 4 (aq ) has the highest osmotic pressure
(c) 0.500 M C 2H5OH(aq ) has the lowest osmotic pressure CH3
(d) They all have the same osmotic pressure (c) (d)
CH3 CH3
28. Which one of the following compounds will H3 C
produce hexanedial upon oxidation with H 3C
(CH 3COO)4Pb ?
32. Which of the following is/are correct statement(s)?
(a) H H (b) OH H (a) The ratio of wavelengths of mth and nth lines in Lyman
series of H-like species is
HO OH H OH lm ïì(m + 1)2 ïüïì (n + 1)2 - 1 üï
=í ýí ý
ln ïî (n + 1)2 ïþïî(m + 1)2 - 1ïþ
H H H OH
(c) (d) (b) Kinetic energies of emitted electrons during
HO OH HO H photoelectric effect is lower than the energy of incident
radiation.
29. In the commercial electrochemical process for
(c) Balmer lines are observed in visible region of
aluminium extraction, the electrolyte used as electromagnetic spectrum for H-like species.
(a) Al(OH)3 in NaOH solution (d) The scattering of a-particles involves a perfectly elastic
(b) a aqueous solution of Al 2 (SO 4 )3 collision.
(c) a molten mixture of Al 2O 3 and Na 3 AlF6
(d) a molten mixture of AlO(OH) and Al(OH)3
Section 3 (Maximum Marks : 12)
30. Select the correct relationships for the given Instructions : Same as given in Physics
equilibrium
Passage X
(a) N2 (g ) + O 2 (g )- 2NO(g ); K = K eq 1
1 1 A soluble compound of a poisonous element M, when
NO(g ) - N (g ) + O (g ); K
2 2 eq = K2, heated with Zn/H 2 SO 4 , gives a colourless and extremely
2 2
thus K1K 22 = 1 poisonous gaseous compound N, which on passing
through a heated tube gives a silvery mirror of element M.
(b) Free energy changes is DG1°
33. The molecular formula of M is
for 2NO2 (g ) - N O (g ) and DG°
2 4 2
(a) AsOH (b) As2N3
1
for N2O4 (g )
2
- NO (g ), then 2 DG ° + DG ° = 0
2 2 1 (c) AsH3 (d) As2O3

æ1 1ö D G ° D G ° 34. N in the above paragraph is


(c) van’t-Hoff equation is, D r H° ç - ÷ = r 2 - r 1
è T2 T1 ø T2 T1 (a) Mn (b) As (c) Mo (d) Rb

PREP CATALYSIS
JEE Advanced~Practice Set 12 277

Passage A 35. The molecular formula of the hydrocarbon P is


15 mL of gaseous hydrocarbon ‘P’ requires 357 mL of (a) C3H7 (b) C3H8 (c) C4H10 (d) C2H6
air for complete combustion and gaseous products
obtained occupy 327 mL volume. Assume all volumes 36. Which isomer of R on reaction with diethylmalonate
measured at STP. (DEM) will give cyclobutane derivative?
Cl
Cl + hv Cl + hv
P →
2
Mono
Q →
2
Mono R (a) Cl Cl (b) Cl
chlorination chlorination
Cl
Cl2 + hv (c) (d) Cl
Di chlorination Cl

MATHEMATICS
42. Sum of infinite terms of series,
Section 1 (Maximum Marks : 21)
cosec-1 10 + cosec -1 50 + cosec -1 170
Instructions : Same as given in Physics
+ cosec-1 442 + … is
1 × 3 × 5¼(2 n - 1)
37. If k n = , then p p
2 × 4 × 6 ¼2 n (a)
4
(b)
2
k 2 n + 1 + k 2 n × k1 + k 2 n - 1k 2 + … + k n + 1 × k n is (c) 0 (d) None of these
1 1
(a) (b)
2 4 nx n + 1 - (n + 1)x n + 1
43. lim , where n = 100 is equal to
(c) 1 (d) None of these x ®1 (ex - e)sin px
5050 100 5050 4950
38. Number of straight lines, which satisfy the (a) (b) (c) - (d) -
2 pe pe pe pe
dy æ dy ö
differential equation + x ç ÷ - y = 0 is
dx è dx ø
(a) 1 (b) 2
Section 2 (Maximum Marks : 28)
Instructions : Same as given in Physics
(c) 3 (d) 4
44. The number of words formed with or without
39. If the equation 2x + 4 y = 2 y + 4 x is solved for y in meaning with each of 2 vowels and 3 consonants
terms of x, where x < 0, then sum of the solution is taken from the letters of the word ‘EVOLUTION’ is
(a) x log 2 (1 - 2 x ) (b) x + log 2 (1 - 2 x ) written in the form 2 a × 3 b × 5 c × 13 d , then
(c) log 2 (1 - 2 ) x
(d) x log 2 (2 x + 1) (a) a + b = 5 (b) a + c = 5
(c) c + d = 2 (d) a + b + c = 6
x2 y2
40. If a tangent of slope 2 to the ellipse, = 1 is + ì x 2 (sin ([ x]) + { x}); 0 £ x < 2
a b2 2 45. If f (x) = í
normal to the circle x 2 + y 2 + 4 x + 1 = 0 , then the î sin x + | x - 3 |; 2£x<4

maximum value of ab is where, [.] and {.} represent the greatest integer and
the fractional part function, respectively
(a) 1 (b) 2
(a) f(x ) is differentiable at x = 1
(c) 3 (d) 4
(b) f(x ) is continuous but non-differentiable at x = 1
41. If y = f (x) and x = g (y) are inverse of each other, (c) f(x ) is non-differentiable at x = 2
(d) f(x ) is discontinuous at x = 2
then g ¢¢(y) is
f ¢ ¢( x ) f ¢ ¢( x ) 46. If the circle x 2 + y 2 = a 2 intersect the hyperbola
(a) - (b)
[f ¢(x )]3 [f ¢(x )]3 xy = c 2 in four points P(x1 , y1), Q(x 2 , y 2), R(x 3 , y 3)
f ¢ ¢( x ) and S(x 4 , y 4), then
(c) - (d) None of these (a) x1 + x 2 + x 3 + x 4 = 0 (b) y1 + y2 + y3 + y4 = 0
[f ¢(x )]2
(c) y1 y2 y3 y4 = c 4 (d) x1x 2 x 3 x 4 = c 4

PREP CATALYSIS
278 JEE Advanced~Practice Set 12

47. The function f (x) = x1/ 3(x - 1)


(a) has 2 inflection points Section 3 (Maximum Marks : 12)
1
(b) is strictly increasing for x > and strictly decreasing for Instructions : Same as given in Physics
4
1 Paragraph X
x<
4 A line L 1 passing through a point with position vector
-1
(c) is concave down in æç , 0ö÷ p = $i + 2$j + 3k
$ and parallel to a = $i + 2$j + 3k
$ . Another line
è2 ø
L 2 passing through a point with position vector
(d) area enclosed by the curve lying in the fourth quadrant
9 q = 2i$ + 3$j + k
$ and parallel to b = 3i$ + $j + 2k.
$
is
28
51. Equation of plane equidistant from line L1 and L 2
48. If in a triangle sin 2 B sin 2 C + 2 sin 2 C sin 2 A is
+ 2 sin 2 A sin 2 B = sin 4 A + sin 4 B + sin 4 C, then its (a) r × ($i - 7 $j - 5k$ ) = 3 (b) r × ($i + 7 $j + 5k$ ) = 3
angle A is equal to (c) r × ($i - 7 $j - 5k$ ) = 9 (d) r × ($i + 7 $j - 5k$ ) = 9
p p
(a) (b)
12 6 52. Equation of a line passing through the point
5p 7p (2 , - 3 , 2) and equally inclined to the line L1 and L 2
(c) (d) may be equal to
6 6
x -2 y- 3 z-2 x -2
(a) = = (b) = y+ 3= z-2
49. Let A, B, C be n ´ n real matrices and product is 2 -1 1 -2
pair wise commutative, also ABC = O n . If x -2 y+ 3 z- 5 x + 2 y+ 3 z-2
(c) = = (d) = =
l = det(A 3 + B 3 + C3) × det (A + B + C), then -4 3 2 4 3 -5
(a) l > 0 Paragraph A
(b) l < 0 A(z 1 ), B(z 2 ), C (z 3 ) are the vertices of a DABC inscribed in
(c) l = 0 the circle | z | = 2. Internal angle bisector of the angle A
(d) l Î(- ¥, ¥) - {0} meets the circumcircle again at D(z 4 ) .
p
l+ 53. Complex number representing point D is
50. The value of ò 2 (cos 4 x + sin 4 x) dx is
l 1 1 z2 + z3
2 (a) z4 = + (b)
p
(a) l æç ö÷
z2 z3 z1
è2 ø z2 z3
(c) (d) z4 = z2 z3
(b) independent of l z1
3p
(c)
8
54. arg [ z 4 / (z 2 - z 3)] is equal to
3p p p
æ
(d) ç ö÷ l2 (a)
4
(b)
3
è 8ø
p 2p
(c) (d)
2 3

PREP CATALYSIS
JEE Advanced~Practice Set 12 279

Answers
Paper 1
1. c, d 2. a, c, d 3. a, d 4. a,b,c,d 5. a,b,c,d 6. b, c 7. a, d 8. 1 9. 6 10. 4
11. 2 12. 2 13. a 14. b 15. d 16. a 17. c 18. d 19. a, b, d 20. b
21. a, d 22. a, b, c 23. a, b, c 24. d 25. a, b, d 26. 6 27. 2 28. 3 29. 1 30. 6
31. d 32. d 33. b 34. a 35. a 36. b 37. a, c, d 38. a, b 39. b, c 40. b, c
41. a, c, d 42. a, b 43. a, b, d 44. 3 45. 0 46. 3 47. 0 48. 0.25 49. a 50. b
51. d 52. a 53. b 54. d

Paper 2
1. c 2. a 3. d 4. a 5. c 6. d 7. b 8. a. b 9. a, c, d 10. a
11. a, c 12. a, c 13. a, c, d 14. a, c, d 15. a 16. a 17. c 18. d 19. c 20. b
21. d 22. a 23. b 24. a 25. c 26. d 27. a, c 28. a 29. c 30. a,b,c,d
31. d 32. a, b, d 33. c, b 34. b 35. b 36. a 37. a 38. b 39. b 40. d
41. a 42. a 43. c 44. a,b,c,d 45. a, c, d 46. a,b,c,d 47. a,b,c,d 48. b, c 49. a, c 50. b, c
51. d 52. b 53. d 54. c

SCORE SHEET - Paper 1


Section No. of Marks from Marks from Marks Obtained
Correct Questions Correct Questions (A) Incorrect Questions (B) (A-B)
................ ............................... ............................... ............................... ...............................
................ ............................... ............................... ............................... ...............................
................ ............................... ............................... ............................... ...............................
Percentage Marks = Marks Obtain/Total Marks x 100

SCORE SHEET - Paper 2


Section No. of Marks from Marks from Marks Obtained
Correct Questions Correct Questions (A) Incorrect Questions (B) (A-B)
................ ............................... ............................... ............................... ...............................
................ ............................... ............................... ............................... ...............................
................ ............................... ............................... ............................... ...............................
Percentage Marks = Marks Obtain/Total Marks x 100
Note To expect your success marks in the test should be between 65%-70%.

PREP CATALYSIS
PREP CATALYSIS
JEE Advanced

PRACTICE SET 13 (With Solutions)

Duration : 3 Hours Max. Marks . 360

Paper 1
PHYSICS
Section 1 (Maximum Marks : 24)
l This section contains SIX (06) questions.
l Each question has FOUR options for correct answer(s). ONE OR MORE THAN ONE of these four option(s) is (are)
correct options(s).
l For each question, choose the correct options(s) to answer the question.
l
Answer to each question will be evaluated according to the following marking scheme:
Full Marks : + 4 If only (all) the correct option(s) is (are) chosen.
Partial Marks : + 3 If all the four options are correct but ONLY three options are chosen.
Partial Marks : + 2 If three or more options are correct but ONLY two options are chosen, both of which are
correct options.
Partial Marks : + 1 If two or more options are correct but ONLY one option is chosen and it is a correct option.
Zero Marks : 0 If none of the options is chosen (i.e. the question is unanswered).
Negative Marks : - 2 In all other cases.
l
For example: If first, third and fourth are the ONLY three correct options for a question with second option being an
incorrect option; selecting only all the three correct options will result in + 4 marks. Selecting only two of the three
correct options (e.g. the first and fourth options), without selecting any incorrect option (second option in this case), will
result in +2 marks. Selecting only one of the three correct options (either first or third or fourth option), without selecting
any incorrect option (second option in this case), will result in +1 marks. Selecting any incorrect option(s) (second
option in this case), with or without selection of any correct option(s) will result in -2 marks.

1. A wheel of radius R rolls with constant velocity v0 (b) Velocity component parallel to X-axis is
along a horizontal plane as shown in the figure. v X = R w (1 - cos wt )
(c) Velocity component parallel to Y-axis is vY
Y = R w sin wt
(d) Acceleration component parallel to X-axis is
P aX = R w2 sin wt
φ
O X 2. Deuterons in a cyclotron describe a circle of 0.4 m
just before emerging. Frequency of alternating
Now, choose the correct otptions. potential between the dees is 107 Hz. Peak value
(a) Position of a point on its edge, (initially, at O at t = 0) of electric potential between dees is 2 ´ 104 V.
is given by x = R (wt - sin wt ) and y = R (1 - cos wt ), Now, choose the correct options.
v
where w = 0 (a) Magnetic field strength is 1.31 T
R (b) Minimum number of complete turns is 200

PREP CATALYSIS
JEE Advanced~Practice Set 13 281

(c) Minimum number of complete turns is 164 5. The solar constant for the earth is s. The
(d) Energy of emerging deuterons is 6.59 MeV surface temperature of the sun is T K. The
3. Let air resistance is proportional to square of sun subtends an angle q at the earth. The
velocity of a moving spherical body, correct option(s) is/are
i.e. Fresistance = - cv 2. (a) s µ Ts2 (b) s µ Ts4
Then, choose the correct options. (c) s µ q2 (d) s µ q4
(a) In case of horizontal motion, velocity at time t is
v0 6. A solid block of mass M is placed at rest on a
v (t ) = , where v 0 = initial velocity, smooth horizontal surface. The block is then
(1 + t / t)
bombared normally by constant stream of
t = instantaneous time and t = m / cv 0
particles, each of mass m ( < < M ) moving with
(b) In case of downward vertical motion, terminal
constant speed, then
velocity is v = mg / c
(c) If body is moving slowly, then Fresistance µ v (a) for elastic collision of each particle, the momentum
transferred to block is 2 mu
(d) If a body is falling in a viscous fluid from sufficient
height, then it cannot attains a speed greater than (b) for elastic collision of each particle, the momentum
its terminal speed transferred to block is 2 m (u - v), where v is velocity
of block
4. An electron in hydrogen atom makes transition (c) if in each dt time dn particles collides with block,
from n1 to n 2 state. The time period of dv m
then n = (u - vn ), where vn is velocity of block
revolution of electron in initial state is 8 times dn M
than in final state, then possible value(s) of after n collision
n1to n 2 is/are æ -
2mn ö
(d) after n collisions, vn = u ç1 - e M ÷
(a) n1 = 4 to n2 = 2 (b) n1 = 8 to n2 = 2 ç ÷
è ø
(c) n1 = 8 to n2 = 1 (d) n1 = 6 to n2 = 3

Section 2 (Maximum Marks : 24)


l This section contains EIGHT (08) questions. The answer to each question is a NUMERICAL VALUE.
l Four each question, enter the correct numerical value (in decimal notation, truncated/rounded-off to the second
decimal place; e.g. 6.25, 7.00.- 0.33, -.30, 30.27, -127.30) using the mouse and the on-screen virtual numeric
keypad in the place designated to enter the answer.
l
Answer to each question will be evaluated according to the following marking scheme:
Full Marks : + 3 If ONLY the correct numerical value is entered as answer.
Zero Marks : 0 In all other cases.

7. A rigid wire of mass 1 kg in the form of an arc


of a circle subtending an angle of 120° at its V0
centre O and having two weights of 9 kg and
3 3/ 2p kg at its ends, rests with its convexity
L
downward upon a horizontal plane as shown in
figure. The inclination of radius is q to vertical
at the end to which 9 kg is attached. Find the N × v02 sinq
value of 2p tan q . If L = , then find the value of N.
g
O
9. The linear charge density of the circumference of
θ 120° a ring of radius R is l = l 0 sin q , where q is
3√3/2π kg defined as shown in the figure. Find the
9 kg electric dipole moment (in C-m) of the ring, if
l 0 = 1 / p C/ m and R = 1 m.
Y
8. A particle falls from a height over an inclined
plane with velocity v0, whose angle of inclination
is q. Particle rebounds elastically and hits the θ X
plane second time at distance L as shown in
the figure.

PREP CATALYSIS
282 JEE Advanced~Practice Set 13

10. In given arrangement, there are two thin lens time period of SHM of pendulum is 3384 ´ k s.
are arranged as shown in the figure. The Find the value of k.
separation between them is 2f , where f is (Take, p = 31
. 4, g = 10 m / s2 and 2 = 1.41)
focal length for each lens, x-coordinate of final
Nf 13. Find the power factor for the circuit as shown
image for a = 30° is . Find the value of N . in the figure.
3
Y 20 Ω

L = 5/π H
α=30°
X XC = 20 Ω
C = 20/π µF

XL = 30 Ω
2f

11. A ball is thrown horizontally from the top of a V = 220 V, f = 50 Hz


tower of unknown height. Ball strikes a
vertical wall, whose plane is normal to the 14. A stream of photons with energy 4.4 eV strike
plane of motion of ball. Collision is elastic and a metal surface with work function 3.5 eV. The
ball falls on ground exactly at the mid-point electrons are ejected at an angle 45° with the
between the tower and the wall. Ball strikes normal to the surface. Uniform magnetic and
the ground at an angle of 30° with horizontal. electric field exist in the space outside, the
Find the height of the tower (in m). surface of metal as shown in figure. Determine
the value of pitch (in m) of helix in the third
revolution for electrons ejected with maximum
kinetic energy. Round off the answer to nearest
H integer. Intensity of electric field is 10-2 N /C
and magnetic field is p ´ 10-6 T. (Given, mass
30° of electron = 9 ´ 10-31kg, charge of electron
8√3 m = 1.6 ´ 10-19 C).
12. There is an isolated planet having mass 2M
and radius 2R, where M and R are the mass
and radius of the earth, respectively. A simple B v E
pendulum having mass m and length 2R is 45°
made to small oscillations on the planet. The
Metal sheet

PREP CATALYSIS
JEE Advanced~Practice Set 13 283

Section 3 (Maximum Marks : 12)


l This section contains TWO (02) paragraphs. Based on each paragraph, there are TWO(02) questions.
l Each question has FOUR options. ONLY ONE of these four options corresponds to the correct answer.
l Four each question, choose the option corresponding to the correct answer.
l Answer to each question will be evaluated according to the following marking scheme:
Full Marks : + 3 If ONLY the correct option is chosen.
Zero Marks : 0 If none of the options is chosen (i.e. the question is unanswered).
Negative Marks : - 1 In all other cases.

Paragraph X 16. The maximum distance moved upward


Consider motion of a cylinder of mass m , having the inclined plane is
radius r, moving upward on an rough inclined plane w20 r 2 é m cos q - sin q ù k
(a) ê ú
with angle q. Initially angular velocity is w0 , initial 4g êë (3 m cos q - sin q)2 úû
velocity of CM is zero and plane with m > tan q. w20 r 2 é m cos q - sin q ù m
(b) ê ú
4g sin a ë (3 m cos q - sin q) û
w20 r 2 é (m cos q - sin q) ù
(c) ê ú
4g sin q êë (3 m cos q - sin q)2 úû

(d) None of the above


ω0
Paragraph A
θ
A vertical spring mass system is left from the
unstretched position, answer the following questions
15. The distance moved before sliding stop is on the basis of above situation.
w20 r 2 é m cos q - sin q ù 17. The maximum displacement of the block from
(a) s =
g êë 3m cos q - sin q úû equilibrium position is
w2 r 2 é m cos q - sin q ù (a) mg / k b) 2 mg / k
(b) s = 0 ê
2g ë 3m cos q - sin q úû (c) mg / 2k (d) mg / 4k
2 w20 r 2 é m cos q - sin q ù 18. The work done by the gravitational force in
(c) s =
g êë 3m cos q - sin q úû maximum stretch is
w2 r 2 é (m cos q - sin q) ù m 2g 2 2m 2g 2 m 2g 2 m 2g 2
(d) s = 0 ê ú (a) (b) (c) (d)
2g ë (3m cos q - sin q)2 û k k 2k 4k

CHEMISTRY
Section 1 (Maximum Marks : 24) (c) Chemical reactivity increases with increase in
atomic number down the group in both the alkali
Instructions: Same as given in Physics. metals and halogens
(d) In alkali metals the reactivity increases but in the
19. Following statements regarding the periodic halogens it decreases with increase in atomic
trends of chemical reactivity of the alkali number down the group
metals and the halogens are given. Which of
these statements give the correct picture? 20. In the form of dichromate, Cr (VI) is a strong
(a) The reactivity decreases in the alkali metals but oxidising agent in acidic medium but Mo (VI)
increases in the halogens with increase in atomic in MoO3 and W (VI) in WO3 are not because
number down the group (a) Cr (VI) is more stable than Mo (VI) and W (VI).
(b) In both the alkali metals and the halogens, the (b) Mo (VI) and W (VI) are more stable than Cr (VI)
chemical reactivity decreases with increase in (c) Higher oxidation states of heavier members of
atomic number down the group group-6 of transition series are more stable

PREP CATALYSIS
284 JEE Advanced~Practice Set 13

(d) Lower oxidation states of heavier members of NO2


group-6 of transition series are more stable
21. Which of the following statement is/are correct (i) Sn/HCl Steam
(c)
(ii) NaNO2/HCl
for B2H 6 (boron hydride)?
(a) The two bridged hydrogen atoms and the two boron
atoms lie in one plane NO2
(b) Out of six B ¾H bonds, four B ¾H bonds can be
described in terms of 3-centre-2 electron bonds OCOCH3
(c) Out of six B ¾H bonds, two bonds can be
described in terms of 3-centre-2-electron bonds LiAlH4 HI (1mol)
(d)
(d) The four terminal B ¾H bonds are two centre —two H+
electron regular bonds
22. In the following reaction sequence, the correct OCH3
structure(s) of X and Y is/are
O 24. Given the following reactions and associated
OH equilibrium constants, select the correct
expression for the third equilibrium constant
H +/ D
X
(i) O3
Y
NaOH in terms of the first two.
(i) Zn/CH3COOH
(A) 2CO( g) + O2( g) - 2CO (g), K 2 C1
O 1
COOH (B) H 2( g) + O2( g) - H O(g), K 2 C2
(a) X = ; Y= 2
(C) CO( g) + H O( g) - CO ( g) + H ( g), K
2 2 2 C3
O KC1 (KC1 )1/ 2
H (a) KC 3 = (b) KC 3 =
; Y=
KC 2 KC 2
(b) X =
(c) KC 3 = KC1 ´ (KC 2 ) 1/ 2
(d) KC 3 = (KC1 )1/ 2 ´ (KC 2 )1/ 2
O
O
H Section 2 (Maximum Marks : 24)
(c) X = ; Y=
Instructions: Same as given in Physics.
O
25. What is the difference in the values of m and n
O in the anionic species [V(CO)m ]n - , if it is
(d) X = ; Y= following Sidgwick EAN rule and having
COOH octahedral shape?
26. The freezing point of the solution is lowered by
23. The reaction(s) leading to the formation of 0.873°C. On dissolving 10 g of NH 4Cl (molar
quinol is/are mass 53.5) in 1000 g of water. The degree of
H3C dissociation is 0.88. The value of degree of
CH hydrolysis of the salt will be ……… .
H3C
(Given : Cryoscopic constant for water = 1.86 K
(a)
(i) O2/D, dil. H2SO4 NaOH (fuse)
molality -1)
(ii) Conc. H2SO4 H+
27. Pb(NO3 )2 and KI react in aqueous solution to
H3C form a yellow precipitate of PbI2. In one series
CH of experiments the masses of two reactants
H3C were varied, but the total mass of the two was
(i) O2/D KMnO4
held constant at 5.0 g. The maximum mass of
(b) PbI2 that can be produced in the above
dil. H2SO4
experiment is ……… .

PREP CATALYSIS
JEE Advanced~Practice Set 13 285

28. In the following cyclic process, the heat absorbed controlled ozonolysis followed by the treatment with
by the system is x p Joule. The value of x is …… . (CH3 ) 2 S gives Y(C8H 8O3 ) and C2H 4O 2 . ‘Y’ can also
be obtained by reaction between ortho methoxy
16 phenol with CHCl3 in KOH solution followed by
acid hydrolysis.
V(L)
33. What is the correct structure of X?
OCH3
10
(a) HO
p(kPa)

29. A complex of Zn has the formula CH CH CH2OH


[Zn(NH3 )2x ]Cl2 and on reaction with HCl OCH3
forms NH +4 ions as:
(b) HO
[Zn(NH3 )x ]Cl2 + HCl ¾® Zn2+ + NH +4 + Cl-
1.02 g complex were treated with HCl and at CH CH CH2OH
the end point, solution was treated with excess OCH3
of Ag NO3 solution giving 4.305 g AgCl
( M = 143.5) precipitate. The volume of 1.0 M (c) HO CH CH CH2
HCl used up will be ……… .
OH
(Given: Molar mass of Zn is 65). OCH3

30. Egg protein from the albumin is precipitated


when an egg is cooked in the boiling water. (d) HO CH CH CH2OH
The activation energy for this first order
reaction is 120 kJ mol -1. The time required to 34. What would be the major product if ‘X’ is
prepare a 3.00 min egg at an altitude at which treated with cold concentrated H 2SO4?
water boils at 77°C is ……… . CH3O CH3O
31. A particular water sample is saturated in CaF2 (a) (b)
and has a total of Ca 2+ content of 115 pm (that HO HO
is, 115 g Ca 2+ per 106 g of water sample). The OCH3 OCH3
amount of F - ion content of the water in ppm HO HO
will be ……… .
(c) (d)
[Given : K sp(CaF2 ) = 5.3 ´ 10-9 ]

32. The six-legged water strider supports itself on Paragraph A


the surface of a pond on four of its legs. Each of
these legs causes a depression to be formed in Aniline reacts with ( P) to form acetanilide that
the pond surface. Assume that each depression further reacts with ClSO3 H to give [Q] that gives [R]
can be approximated as a hemisphere of radius in presence of ammonia. [R] on hydrolysis produces
1.2 ´ 10-4 m and q is 0°. The force that one of
[S] that is antibacterial. Acetanilide gives (T) in
the insect’s leg exerts on the pond will be a
´ 10-5 N. The value of a is ……… . presence of HNO3 /H 2SO 4 at 20°C followed by
hydrolysis and heat. The road map of the given
transformation is as follows :
Section 3 (Maximum Marks : 12) NH2 NHCOCH3
Instructions: Same as given in Physics.
[P] ClSO3H NH3
[Q] [R]
Paragraph X
An organic compound X(C10H12O3 ) is not soluble in (i) HNO3/H2SO4, 20°C H+/H2O
(ii) H+/H2O4, Heat
water or NaHCO 3 . A solution of Br2 in CCl 4 is [ T] [S]
decolourised by ‘X’ forming C10H12O3 Br2 . ‘X’ on Antibacterial

PREP CATALYSIS
286 JEE Advanced~Practice Set 13

35. The compound (T ) is 36. The compound S is


ρ NH2 NH2 NH2 NH2
NO2 NHCOCH3 NH2 NH3

(c) (d) (a) (b) (c) (d)


(a) (b)

NO2 NO2 CH3 OH SO2NH2 SO3


NO2 NO2

MATHEMATICS
Section 1 (Maximum Marks : 24) (c) The distance of the point (1, 1, 1) from the plane
passing through the point (-1, - 2, - 1) and whose
Instructions: Same as given in Physics.
normal is perpendicular to both the lines L1 and L2 is
13
37. For a non-zero complex number z, such that
|z - 2 + 2i| = 1, let arg ( z ) denotes the principal 5 3
(d) The acute angle between L1 and L2 cos-1 æç ö÷
11
argument with - p < arg ( z ) < p. Then, which of
è 14 ø
the following statement(s) is(are) false?
(a) The least modulus of z is 2 2 - 1 40. Let f : R ® R, g : R ® R and h : R ® R be
p differentiable functions such that
(b) The least arg (z) is
4 f ( x ) = x 2 + 3x + 2, g( f ( x )) = x and h( g( g( x )) = x
p for all x Î R. Then, which of the following
(c) When modulus of z is least, then arg (z) is -
4 is(are) TRUE?
p 1
(d) The greatest arg (z) is (a) g¢ (2) = (b) h¢ (1) = 666
4 15
38. Let equation of a parabola is y = x 2 + x + a. (c) h(0) = 16 (d) h (g (3)) = 36
When a is a parameter which changes at a 41. Let a , b, c are the sides opposite to angle
da
constant rate such that = 5. If a = 0, when, A, B, C respectively. In a DABC and the side of
dt a DABC satisfy the equation 2a 2 + 4b2 + c2
t = 0. Let A( t ) be area bounded by
y = x 2 + x + a, X-axis, Y -axis and x = a at the = 4ab + 2ac. Then, which of the following
statement(s) is(are) TRUE?
time t. Then, which of the following is(are)
(a) The triangle is isosceles
TRUE?
1450 (b) cosB is equal to 7/8
(a) Maximum value of A(t ), when t Î(0, 2) is
3 (c) The triangle is obtuse
(b) Maximum value of A¢ (t ), when t Î(0, 2) is 650 (d) cos A is equal to 1/4
1450
(c) A(2) is equal to 42. Consider the function f : R ® R defined by
3
(d) Maximum value of A(t ), when t Î(0, 2) is 650
x 2 - ax + 1
f(x) = , a Î( 0, 2) and
x 2 + ax + 1
x-1 y+ 2 z +1
39. Let L1 : = = and e x f ¢ ( t)
3 2 1 g( x ) = ò dt. Then, which of the following
0 1 + t2
x-2 y+2 z-3
L2 : = = be two lines. Then,
2 1 3 statements is (are) true?
which of the following statement(s) is(are) (a) f (x) is decreasing on (-1, 1) and has a local
TRUE? minimum at x = 1
11 (b) f (x) is increasing on (-1, 1) and has a local
(a) The shortest distance between L1 and L2 is
5 3 maximum at x = 1
(b) The unit vector perpendicular to both L1 and L2 is (c) g ¢ (x) is positive on (-¥, 0) and negative on (0, ¥)
5 $i + 7 $j - k$ (d) g ¢ (x) changes sign on both (-¥, 0) and (0, ¥)
5 3

PREP CATALYSIS
JEE Advanced~Practice Set 13 287

Section 2 (Maximum Marks : 24)


Section 3 (Maximum Marks : 12)
Instructions: Same as given in Physics.
Instructions: Same as given in Physics.
43. Let f ( x ) = ( x 2 + 3x + 2)cos( px ) . Then, the sum of
Paragraph X
all positive integers n for which
½n ½ Let A be the set of all 3 ´ 3 symmetric matrices all of
½ å log10 f ( k½
) = 1 is …… . whose enteries are either 0 or 1. Five of these entries
½k = 1 ½
are 1 and four of them are 0.
44. If S n is the sum of infinite geometric series, 51. The number of matrices B in A for which the
whose first term is n and common ratio is
1 é x ù é 1ù
system of linear equation B ê y ú = ê 0ú has a
. Then, the value of
n +1 ê ú ê ú
S1S n + S 2S n - 2 + ¼ + S n S1 êë z úû êë 0úû
lim is equal to …… .
n ®¥ S12 + S 22 + S32 + ¼ S n2 unique solution is
(a) less than 4 (b) at least 4 but less than 7
45. Number of solutions of the equation|sin x cos x| (c) at least 7 but less than 10 (d) at least 10
+ 2 + tan2 x + cot2 x = 3, x Î [0, 6p ] are ……… .
52. The number of matrices B in A for which the
b
46. The value of integral ò (x - a ) ( b - x ) dx is
3 4 é x ù é 1ù
a
system of linear equation B ê y ú = ê 0ú is
( b - a )m ê ú ê ú
. Then, the value of m + n is equal to êë z úû êë 0úû
n
… inconsistent is
(a) 0 (b) more than 2
47. The adjacent side vectors OA and OB of a (c) 2 (d) 1
rectangle OACB are a and b respectively,
where O is the origin. If 16|a ´ b| = 3(|a| +|b|) Paragraph A
and q be the acute angle between the diagonals 4x 2 + 2y 2 - 6xy
æq ö The limiting value of expression is A
OC and AB, then the value of cotç ÷ is …… 6x 2 + 2y - 8xy
è 2ø
as point ( x, y) on curve x 2 + y 2 = 1 approaches the
48. Nishkarsh and Dakshesh investigate two æ 1 1 ö
positive integers A and B such that : Nishkarsh position ç , ÷, where A is such that (5A , 0) is a
è 2 2ø
picks a positive factor ‘a’ of A, Dakshesh picks
a positive factor ‘b’ of B and they write down point as focus of parabola S having axis parallel to
the product ab on a sheet of paper. They repeat X-axis, vertex at origin.
the above procedure for all possible ordered
53. The two common tangents can be drawn to
pairs ( a , b). At the end, they calculate that the
both circle and parabola from external point
sum of all numbers on the paper is 2340. If
whose coordinates are
both A and B are divisible by 6 and have only 2
æ -4 ö æ -4 ö
and 3 as their prime factors. Then, the least (a) ç , 0÷ (b) ç , 0÷
possible value of A + B is ……… . è 15 - 1 ø è 17 + 1 ø
7 æ -4 ö æ -4 ö
49. Let f ( x ) = 3x 2 - 7x + c where, x > , then the (c) ç , 0÷ (d) ç , 0÷
6 è 17 - 1 ø è 15 + 1 ø
value of [c] such that f ( x ) touches f -1( x ) is
54. Locus of mid-points of chord of parabola, which
………. (where, [×] denotes the greatest integer
subtend a right angle at vertex of parabola is
function)
(a) y 2 - 4 x + 32 = 0
50. Two circles having radii r1 and r2 passing (b) y 2 + 4 x - 32 = 0
through vertex A of a DABC. One of the circle (c) y 2 - 32 x + 4 = 0
touches the side BC at B and the other circle (d) y 2 + 32 x - 4 = 0
touches the BC at C. If a = 5 and A = 30°, then
the value of r1r2 is equal to ……

PREP CATALYSIS
Paper 2
PHYSICS
Section 1 (Maximum Marks : 24)
l This section contains SIX (06) questions.
l Each question has FOUR options for correct answer(s). ONE OR MORE THAN ONE of these four option(s) is (are)
correct options(s).
l For each question, choose the correct options(s) to answer the question.
l Answer to each question will be evaluated according to the following marking scheme:
Full Marks : + 4 If only (all) the correct option(s) is (are) chosen.
Partial Marks : + 3 If all the four options are correct but ONLY three options are chosen.
Partial Marks : + 2 If three or more options are correct but ONLY two options are chosen, both of which are
correct options.
Partial Marks : + 1 If two or more options are correct but ONLY one option is chosen and it is a correct option.
Zero Marks : 0 If none of the options is chosen (i.e. the question is unanswered).
Negative Marks : - 2 In all other cases.
l For example: If first, third and fourth are the ONLY three correct options for a question with second option being an
incorrect option; selecting only all the three correct options will result in + 4marks. Selecting only two of the three correct
options (e.g. the first and fourth options), without selecting any incorrect option (second option in this case), will result in
+2 marks. Selecting only one of the three correct options (either first or third or fourth option), without selecting any
incorrect option (second option in this case), will result in +1marks. Selecting any incorrect option(s) (second option in
this case), with or without selection of any correct option(s) will result in -2 marks.

1. Electric field between the plates of a velocity 3. Ball A of mass m moving with speed v colliding
selector is 1.20 ´ 105 Vm -1. Magnetic field in head on with stationary ball B of same mass. If
region is of 0.60 T. The charged particles which e is coefficient of restitution, then which of the
passes through the region of crossed fields are following statements are correct?
1+ e ö
found to move in circular path of radius (a) Ratio of velocities A and B after collision is æç ÷
7.28 ´ 10-2 m (in magnetic field of 0.60 T). è1- e ø
1- e ö
Then, correct options are (b) Ratio of final and initial velocities of A is æç ÷
è 2 ø
(a) velocity of charged particles is 4 ´ 106 ms-1
(b) velocity of charged particles is 3.5 ´ 106 ms-1 (c) Ratio of velocities of balls A and B after collision is
æ1- e ö
(c) mass of charged particles is 4 ´ 10-26 kg ç ÷
è1 + e ø
(d) mass of charged particles is 3.5 ´ 10-26 kg
1+ e ö
(d) Ratio of final and initial velocities of ball B is æç ÷
2. A disc is given with an initial angular velocity è 3 ø
w0 and placed on rough horizontal surface as
shown in the figure. The quantities which will 4. Three processes form a p
thermodynamics cycle on 1
not depend on coefficient of friction is/are
the p-V diagram as shown
2
in the figure. Process 1® 2
is isothermal, process 2 ®
3 is isochoric and process 3
ω0
3 ® 1 is adiabatic. During V
the complete cycle, the
(a) time until pure rolling begins total amount of work done is 10 J. During
(b) displacement of disc until pure rolling begins process 3 ® 1, 20 J of work is done on the
2 w0R system. The amount of heat added to the
(c) velocity of rolling body is
3 system during process 1 ® 2 is
(d) work done by force of friction (a) 0 J (b) 10 J
(c) 20 J (d) 30 J

PREP CATALYSIS
JEE Advanced~Practice Set 13 289

5. Assume that the nuclear binding energy per (b) Fusion of a nucleus not lying in the range of
nucleon (B/A) versus mass number (A) is as 51< A< 100 will release energy
shown in the figure. Use this plot to choose the (c) Fission of a nucleus lying in the mass range of
correct choice (s) given below 100 < A< 200 will release energy, when broken
into two equal fragments
B/A
(d) Fission of a nucleus lying in the mass range of
8 200 < A< 260 will release, when broken into two
6 equal fragments
4 6. A particle is projected with velocity 20 m/s, so
2 that it just clears two walls of equal height
0 A
10 m, which are at a distance 20 m from each
100 200 other. The time of passing between the walls is
(a) 2 s (b) 2 10s
(a) Fusion of two nuclei with mass numbers lying in the
(c) 10 2 s (d) 1/2 s
range of 1 < A < 50 will release energy

Section 2 (Maximum Marks : 24)


l This section contains EIGHT (08) questions. The answer to each question is a NUMERICAL VALUE.
l Four each question, enter the correct numerical value (in decimal notation, truncated/rounded-off to the second
decimal place; e.g. 6.25, 7.00.- 0.33, -.30, 30.27, -127.30) using the mouse and the on-screen virtual numeric
keypad in the place designated to enter the answer.
l Answer to each question will be evaluated according to the following marking scheme:
Full Marks : + 3 If ONLY the correct numerical value is entered as answer.
Zero Marks : 0 In all other cases.

7. A ball is projected from a point ( P ) in a 9. Three rods AB, BC and BD of same length
horizontal plane, so as to strike a vertical wall l = 1 m and cross-section area, A = 10 cm 2 are
at right angle (at point A). After rebounding arranged as shown in the figure. The end D is
from the wall, it strikes the horizontal plane immersed in ice whose mass is 440 g. Heat is
once at ( B) and returns to the point of supplied at constant rate of 200 cal/s from end
projection just before second collision at the A. Find the time in which whole ice will melt.
horizontal surface. Find coefficient of Latent heat of fusion of ice is 80 cal/g and
restitution, assuming it remains same for both thermal conductivity of rods, K = 100 cal/m-s°C
collisions. A K, l B 2K / l C

A 100°C

K/2 l
D
Ice

10. A rod AB rests with the end A on rough


θ horizontal ground and the end B against a
P B smooth vertical wall. The rod is uniform and
weight of w, if the rod is in equilibrium in the
8. Two narrow bores of radius 3.0 mm and position as shown in the figure. Find the
6.0 mm are joined together to form a U -tube normal reaction at B.
open at both ends. If the U -tube contains Y
water, then what is the difference in its levels B
in the two limbs of the tube in (mm). Surface
tension of water is 7.3 ´ 10-2 N/m. (Take, the
angle of contact to be zero and density of water
to be 103 kg/m3 , g = 9.8 m / s2). A
X
O

PREP CATALYSIS
290 JEE Advanced~Practice Set 13

11. An aeroplane is flying in a horizontal direction made of heat insulator and the piston is
with a velocity 600 km/h at a height of 1960 m. insulated by having a pile of sand on it. By
When it is vertically above the point A on the what factor does the pressure of the gas
ground, a body is dropped from it. The body increase, if the gas is compressed to half its
strikes the ground at point B. Calculate the original volume?
distance AB in km.
14. In changing the state of a gas adiabatically
12. A particle moves in a circle of radius 2.0 cm at from an equilibrium state A to another
a speed given by v = 4t, where v is in cm/s and t equilibrium state B, an amount of work equal
is in seconds. Find the total acceleration at to 22.3 J is done on the system. If the gas is
t = 1 s ( 5 = 2.3). taken from state A to B via a process in which
the net heat absorbed by the system is 9.35 cal,
13. A cylinder with a movable piston contains 3 how much is the net work done by the system
moles of hydrogen at standard temperature in the later case? (Take, 1 cal = 419
. J)
and pressure. The walls of the cylinder are

Section 3 (Maximum Marks : 12)


l This section contains FOUR (04) questions.
l Each question has TWO (02) matching lists : Column-I and Column-II.
l FOUR option are given representing matching of element from Column-I and Column-II. ONLY ONE of these four
options corresponds to a correct matching.
l For each question, choose the option corresponding to the correct matching.
l Answer to each question will be evaluated according to the following marking scheme:
Full Marks : + 3 If ONLY the correct option is chosen.
Zero Marks : 0 If none of the options is chosen (i.e. the question is unanswered).
Negative Marks : - 1 In all other cases.

15. There is a equibiconvex lens, mode of refractive


index 3/2 and radius of curvature 60 cm. A Q
B a
Column I Column II b C
A. Its focal length is 1. Infinite
B. When placed in a liquid 2. Less than 60 cm Column I Column II
5
of m = , its focal length is b -r
2 A. For r < a potential is 1.
proportional to rb
C. When placed in a liquid 3. More than 60 cm
5 For a £ r £ b ab
of m = , its focal length is B. 2.
4 b -a
D. When placed in a liquid 4. 60 cm C. Potential difference between 3. a+b
3
of m = , its focal length is surface A and B is ab
2 proportional to
Codes D. Capacitance of the system 4. b -a
A B C D A B C D is proportional to ab
(a) 1 2 3 4 (b) 4 2 3 1
(c) 4 3 1 2 (d) 2 3 4 1 Codes
A B C D
16. Two conducting spheres A and B of radius a (a) 4 1 3 2
and b, is placed concentrically as shown in the (b) 4 1 2 3
figure. Sphere B is earthed, C is the common (c) 3 4 1 2
centre. Sphere A is given charge Q. For a given (d) 3 1 2 4
distance r.

PREP CATALYSIS
JEE Advanced~Practice Set 13 291

17. A stationary wave is given by 18. Consider a source of frequency f also let both
y = ( 0.06) . sin ( 2px ) × cos ( 5pt ) source and observer have same frequency.
Match Column I and Column II Column I Column II
Column I Column II A. Observer moving towards 1. Greater than f
source and source is
A. Amplitude of constituent wave 1. 0.06 moving away from observer
B. Observer and source both 2. Less than f
B. Position of mode 2. 0.5
moving towards each other
C. Position of antinode 3. 0.25 C. Observer and source both 3. Equal to f
moving away from each
D. Amplitude at x = 3 m 4. 0.03
4 other
D. Source is moving towards 4. None
Codes receeding observer
A B C D
Codes
(a) 1 2 3 4
A B C D
(b) 4 2 3 1
(a) 3 1 2 4 (b) 4 2 3 1
(c) 4 3 1 2
(c) 4 3 1 2 (d) 2 3 4 1
(d) 2 3 4 1

CHEMISTRY
Section 1 (Maximum Marks : 24) (c) The white fumes are due to aluminium chloride
(d) ‘A’ acts as a catalyst in Friedel-Craft’s reaction
Instructions: Same as given in Physics.

19. A metal complex having the composition 21. 1-bromobutane reacts with Mg/Et2O followed
Cr(NH3 )4Cl2Br has been isolated in two forms by pentanal in presence of H3O+ gives
A and B. The form A reacts with AgNO3 to give 1-butylpentanol ( A). ‘A’ undergoes the following
a white precipitate readily soluble in dilute transformations that are as follows:
aqueous ammonia whereas B gives a pale
H SO (Conc.) O
yellow precipitate soluble in concentrated B ¾¾¾¾®
2 4
Heat
C ¾¾¾®3
Zn-H O
Butanal + Pentanal
2
ammonia. The formula of complex A is
(a) [Cr(NH3 )4 Br]Cl 2 (b) [Cr(NH3 )4 ClBr]Cl (i) B2H6
F
(c) [Cr(NH3 )4 Cl 2 ]Br (d) [Cr(NH3 )4 ]Cl 2Br Br2 NaNH2 (ii) H2O2/NaOH
C D E
(i) NaNH2 (i) Butanone
20. Consider the following sequence of reactions: G
(ii) H3O +
Colourless
B in
mixture A+B C; A The product (G) is/are
excess
of salt A and B
White
ppts. (a)
OH
Atmospheric
Evolution of fumes
O2
A and form dimer
(b)
NH4Cl/NH4OH OH
White ppt.
(soluble in B)
(c)
The true statement(s) about the above reaction OH
sequence is
(a) ‘B’ turns red litmus blue (d)
OH
(b) In excess of B, ZnO dissolves to form salt

PREP CATALYSIS
292 JEE Advanced~Practice Set 13

22. Which of the following statement(s) is/are [Given that, N A = 6 ´ 1023 ]


correct about the maltose?
120°
MeOH/ HCl Ac2O/ NaOAc H3 O+
Maltose ¾¾¾®( B) ¾¾¾¾®(C ) ¾¾®( D )
(A ) b

Product C is
CH2OAc CH2OAc
O H H O 90°
(a) OAc OAc a
O (a) 8 (b) 6 (c) 4 (d) 2
AcO OMe
OAc OAc Section 2 (Maximum Marks : 24)
CH2OAc CH2OAc Instructions: Same as given in Physics.
O H O OMe
OAc OAc 25. Among the following compounds, the total
(b) O number of amphoteric species are ………
AcO H BeO, MgO, Mg(HCO3 )2, NaHCO3 , Ca(HCO3 )2,
OAc OAc CaO, ZnO, Al2O3 , MgH 2PO4, Mg2P2O7
CH2OH CH2OH 26. The Schrodinger wave equation for hydrogen
atom is
O O OMe 3/ 2
OH OH OH 1 æ1ö é r0 ù - r/ a0
(c) H
+ y 2s = ç ÷ ê2 - a úe
HO HO
4 2p è a0 ø ë 0û

OH OH Where, a0 is Bohr radius. If the radial node is 2s


at r0 and r0 = n a0 , Then the value of n is ……….
CH2OAc CH2OAc
27. 10% sites of catalyst have been absorbed by H 2
O H O OMe molecule. On heating, H 2 gas is evolved from
OAc
(d) O sites and collected at 0.06 atm and 300 K in
AcO H small vessels of 4.92 cm3 . Number of sites
OAc OAc OAc available is 12 ´ 1015 per cm 2 and surface area
is 1000 cm 2. The number of surface sites
23. The Edison storage cell is represented as occupied per molecule of H 2 will be ……… .
Fe(s)/ FeO(s)/KOH( aq )/Ni2CO3 ( s)/ NiO( s) (Given, N A = 6 ´ 1023 )
The half-cell reactions are: 28. For the given compound ‘X’, the total number
Ni2O3 ( s) + H 2O( l ) + 2e- 2NiO( s) + 2OH - ;
- of optically active stereoisomers is ………
E° = + 0.4 V OH H
FeO( s) + H 2O( l ) + 2e- Fe ( s) + 2OH - ;
- CH3
C
E° = - 0.87 V C
Choose the correct statement(s) regarding
C Br OH
above cell.
O
(a) The emf of cell is 1.27 V. H 3C CH3
(b) emf is independent of concentration of KOH.
(c) The maximum amount of electrical energy that can 29. An hourly requirement of an astronaut can be
be obtained from one mole of Ni 2O3 is -245.11 kJ. satisfied by the energy released when 34.2 g of
(d) The maximum amount of electrical energy that can sucrose (C12H 22O11 ) are burnt in his body. How
be obtained from one mole of Ni 2O3 is 345.11 kJ. many gram of oxygen would be needed in a space
capsule to meet his requirement for one day?
24. X 2Y molecule (molar mass 259.8 g/mol)
30. Consider the reaction,
crystalises in a hexagonal lattice as shown in 2+
XY 2 + ( aq ) -X ( aq ) + 2Y - ( aq )
given figure. The lattice constants were
a = 5 Å, b = 8 Å. If the density of crystal is The K sp of sparingly soluble salt XY 2 is
5 g/cm3 , then how many molecules are 1.26 ´ 10-4 mol3 /L3 at 25°C. The vapour pressure
contained in the given unit cell? of its solution in water is 28.48 mm of Hg at 33°C.

PREP CATALYSIS
JEE Advanced~Practice Set 13 293

Assuming 100% ionisation, if the vapour List I List II


pressure of H 2O at 33°C is 28.54 mm of Hg, A. CHCl 3 + HNO3 ® An 1. Gammaxene
then the enthalpy change of the reaction will insecticide and tear gas
be ……… in kJ. B. Benzene + Cl ¾® hn 2. Borodine-Hunsdiecker
2
31. 3 g of activated charcoal was added to 50 mL of reaction
0.06 M acetic acid solution in a flask. After an Product
hour, 10 mL of residual CH3COOH required C. C H Cl + Chloral ¾¾®
Conc. 3. Chloropicrin
14 mL of 0.03 M NaOH. If Freundlich 6 5 H 2 SO 4
adsorption isotherm is followed x/ m = K (C )1/ 2, Product
then the value of K is ……… D. Br 4. Compounds containing
Silver acetate ¾¾®
2
[x is the amount of adsorbate in gram absorbed CCl 4 oxygen
per gram of adsorbent and C is the equilibrium CH3Br
concentration in millimoles]. E. Cl 2 5. Dichlorodiphenyl
C6H5 COMe ¾¾®
32. The solid X reacts with oxygen to form a hn trichloroethane
gaseous compound XO( g) by the following PhCOCH2 Cl
reversible reaction : 6. Used as a lachrymator
2X ( s) + O2( g) 2XO( g)
- and also used to disperse
At 300 K, D rG° = 10 kJ mol -1 the mob by police
and D r H ° = 20 kJ mol -1 The correct option is
A container was filled with a big pile of solid X (a) A ® 3, 4; B ® 1; C ® 5; D ® 2; E ® 6
and some O2 and heated upto a temperature of (b) A ® 1, 2; B ® 3; C ® 4; D ® 5, 6; E ® 2
600 K at a constant pressure. If on reaching (c) A ® 1; B ® 2, 3; C ® 3, 4; D ® 4, 5; E ® 6
the equilibrium state, the partial pressure of (d) A ® 3, 4; B ® 1; C ® 2; D ® 5; E ® 6
O2 in the system found to be 2 bar. Then, the 35. List-I contains reactions and List-II contains
total pressure in the container will be …… . major products.
List I List II
Section 3 (Maximum Marks : 12) P. Br (i) H3O+
1.
Diethyl 2NaOEt
?
+
Instructions: Same as given in Physics. malonate Br
–2EtOH (ii) D,–CO2
(iii) Sodalime O
33. Match each set of hybrid orbitals from List I Me
with complexes given in List II. Q. 2.
Ethyl Br (i) H3O
+
2NaOEt
List I List II aceto + –2EtOH
?
(ii) Sodalime
acetate Br
P. dsp 2 1. [Ni(CN)4 ]2- , square planar
R. 3.
Q. sp 3d 2. PF5 , trigonal bipyramidal Br

R. sp 3d 2 3. [Pt(Cl)4 ]2- , square planar +2Na


2 3
S. d sp 4. BrF5 , square pyramidal
Br
5. PCl 5 , trigonal bipyramidal
S. +
4.
6. [Co(NH3 )6 ]3+ , octahedral hν
?
NH2NH2
O + CH2N2 σOH

The correct option is


(a) P ® 1, 3; Q ® 2, 5; R ® 4; S ® 6 T. O
(b) P ® 1, 5; Q ® 2, 3; R ® 4; S ® 6
(c) P ® 1, 3; Q ® 2, 4; R ® 5; S ® 6 C O Dry Zn-Hg/HCl
(d) P ® 1, 6; Q ® 1, 3; R ® 2; S ® 4 C O
Ba distillation
?

34. Match the reactions given in List I and


O
characteristic given in List II.

PREP CATALYSIS
294 JEE Advanced~Practice Set 13

The correct option is Column I Column II


(a) P ® 2; Q ® 1; R ® 3; S ® 4; T ® 1 A. Arrhenius constant (min ) -1
i. 1.0
(b) P ® 3; Q ® 1; R ® 3; S ® 2; T ® 4
B. Order of the reaction ii. 2.0
(c) P ® 3; Q ® 1; R ® 2; S ® 4; T ® 1
C. At 500 K, rate of reaction at iii. 105
(d) P ® 2; Q ® 3; R ® 1; S ® 4; T ® 3
[A]0 = 0.2 M
36. A ® Product, - d [ A]/ dt = k[ A]n . Rate constant D. Ea (in kcal mol -1) iv. 0.9212
( k) of a reaction is given by v. 9.2
æ 2 ´ 103 K ö
log10 k (min-1 ) = 5 - ç ÷ Codes
è T ø A B C D
(a) iii i ii iv
Match the given property in Column I with its (b) iv iii v ii
related value in the Column II and select (c) iii ii i v
answer from codes given below : (d) iii i ii v

MATHEMATICS
Section 1 (Maximum Marks : 24) 40. For z as real or complex, if (1 + z 2 + z 4 )8
= C0 + C1z 2 + C2z 4 … + C16z32, then which of the
Instructions: Same as given in Physics.
following is (are) TRUE?
37. In an experimental performance of a single (a) C0 - C1 + C2 - C3 + ¼ + C16 = 1
throw of a pair of unbiased normal dice, let (b) C0 + C3 + C6 + C9 + C12 + C15 = 3 7
three events E1, E2 and E3 are defined as E1 =
(c) C2 + C5 + C8 + C11 + C14 = 3 6
getting prime numbered face on each dice E2 =
(d) C1 + C4 + C7 + C10 + C13 + C16 = 3 7
getting the same number on each dice E3 =
getting total on two dice equal to 4 41. Let f ( x ) be twice differentiable function such
Which of the following is/are true? that f ¢ ¢ ( x ) > 0 in [0, 2]. Then, which of the
(a) The probabilities P (E1), P (E 2 ), P (E 3 ) are in AP following statement(s) is(are) TRUE?
(b) The events, E1 and E 2 are independent (a) f (0) + f (2) = 2f (c), for atleast one c, c Î(0, 2)
æE ö 2 (b) f (0) + f (2) < 2f (1)
(c) P ç 3 ÷ =
è E1 ø 9 (c) f (0) + f (2) > 2f (1)
17
(d) 2f (0) + f (2) > 3f æç ö÷
(d) P (E1 + E 2 ) + P (E 2 - E 3 ) = 2
36 è3ø
38. The equation sin x = [1 + sin x ] + [1 - cos x ] has, 42. A rod of length 2 units whose one end is
where [×] is the greatest integer function. (1, 0, -1) and other end touches the plane
p p p x - 2 y + 2z + 4 = 0, then
(a) No solution in éê - , ùú (b) No solution in éê , p ùú
ë 2 2û ë2 û (a) The rod sweeps the figure whose volume is p cu
3p ù
(c) No solution in éê p, (d) No solution for x ÎR units
ë 2 ûú (b) The area of the region for which the rod traces on
the plane, is 2p sq units
39. Let AC be a diameter of circle, AB is a tangent,
(c) The length of projection of the rod on the plane is
BC meets the circle again at D. AC = 1,
3 units
AB = a, CD = b. Then, which of the following is
(are) TRUE? (d) The centre of the region for which the rod traces or
2 2 -5 ö
(a) ab > 1 (b) ab < 1 the plane, is æç , , ÷
1 a 1 a è3 3 3 ø
(c) a 2 + > (d) a 2 + <
2 b 2 b

PREP CATALYSIS
JEE Advanced~Practice Set 13 295

S. The value of SG is 4. 6log 2


Section 2 (Maximum Marks : 24) 5. (0, ¥)
Instructions: Same as given in Physics. 6. 135
43. Consider the sequence of number 4, 7, 1, 8, 9, The correct option is
7, 6 … for n > 2 the nth term of the sequence is (a) P ® 3; Q ® 4; R ® 2; S ® 1
the unit digit of the sum of the two previous (b) P ® 1; Q ® 2; R ® 4; S ® 3
terms. Let S n denote the sum of first n terms (c) P ® 6; Q ® 1; R ® 5; S ® 2
of this sequence. The smallest value of n for (d) P ® 5; Q ® 4; R ® 2; S ® 1
which S n > 10085
2018p 52. Consider the equation 2 +|x 2 + 4x + 3| = m,
44. The value of the integral ò |sin 2018x|dx is m ÎR
0
equal to ……
List-I List-II
45. Tangents are drawn from the point (a , b ) to the
P. The least value of m for which 1. {3}
hyperbola 3x 2 - 2 y 2 = 6 and are inclined at
given equation is meaningful.
angles q and f to the X-axis. If tan q tan f = 2,
then value of 2a 2 - b 2 is Q. Set of all real value of m, so that 2. (2, 3)
given equation have two
46. If|a| = 2,|b| = 5 and a × b = 0 and solution is
a ´ (a ´ (a ´ (a ´ (a ´ (a ´ b ))))) = lb, then the R. Set of all real value of m, so that 3. 5
value of|l| is equal to …… given equation have three
47. A function y = f ( x ) satisfies the differential solutions is
equation f ( x ) sin 2x - cos x + (1 + sin2 x ) f ¢ ( x ) = 0 S. Set of all real value of m, so that 4. {4, 5}
æpö given equation have four
with f( 0) = 0, then the value of f ç ÷ is equal to
è 6ø distinct solutions is
…… 5. {2 } È (3, ¥)

48. If|z1| = 2,|z 2| = 3,|z3| = 4 and 6. 2


|2z1 + 3z 2 + 4z3| = 9, then value of The correct option is
|8z 2z3 + 27z3 z1 + 64z1z 2|1/ 3 is … (a) P ® 2; Q ® 1; R ® 4; S ® 6
49. A mapping is selected at random from all (b) P ® 6; Q ® 5; R ® 1; S ® 2
mapping f : A ® A, where set A = { 1, 2, 3, ¼ , n }. (c) P ® 4; Q ® 3; R ® 2; S ® 4
If the probability that mapping is injective is (d) P ® 3; Q ® 4; R ® 5; S ® 6
3
, then the value of n is …… 53. Consider the two curves C1 : y = 1 + cos x and
32 æ pö
C2 : y = 1 + cos( x - a ) for a Î ç 0, ÷ , where
50. If f ( x + y + 1) = ( f ( x ) + f ( y )2 , " x , y Î R and è 2ø
f( 0) = 1, then f( 2) is equal to …… x Î [0, p ]
List-I List-II
Section 3 (Maximum Marks : 12) P. If A1 is the area bounded 1. p/3
Instructions: Same as given in Physics. by curves C1, C2 and
x = 0, then A1 is equal to
51. Suppose f : R ® R + be a differentiable function
Q. If A2 is the area bounded 2. æ 3ö
and satisfies 3f ( x + y ) = f ( x ) × f ( y ) for all 1+ ç ÷
x , y Î R with f(1) = 6. If
by curve C2 and the lines è 2 ø
y = 1and x = p, then A2 is
æ æ 1ö ö 3
S = lim n ç f ç1 + ÷ - f (1)÷ and G = ò f ( x )dx. equal to
n ®¥ è è nø ø 0
R. If A1 = A2 , then the value 3. 1 - sina
List-I List-II of a is
a
P. The range of f (x) is 1. 126 S. For the value of a 4. 2 sinæç ö÷ - sin a
21 obtained above, the total è2ø
Q. The value of S is 2.
log 2 area bounded by C1, C2
and x = p is
R. The value of G 3. 10log 2

PREP CATALYSIS
296 JEE Advanced~Practice Set 13

The correct option is List-I List-II


(a) P ® 3; Q ® 4; R ® 2; S ® 1 3
P. The value of 1.
(b) P ® 1; Q ® 2; R ® 3; S ® 4
cos A × cosB × cosC 2
(c) P ® 4; Q ® 3; R ® 1; S ® 2
9
(d) P ® 1; Q ® 2; R ® 4; S ® 3 Q. The value of 2.
cos2 A + cos2 B 8R 3
54. Let ABC be an acute angled triangle with + cos2 C is
orthocentre H . D , E and F are the feet of
R. The value of R is 3. 3
perpendicular from A, B and C respectively on
opposite sides. Also, let R be the circumradius 8R 3
of DABC. Given, AH × BH × CH = 3 and S. The value of 4. 7
( AH )2 + ( BH )2 + (CH )2 = 7 HD × HE × HF is 4R 2

The correct option is


(a) P ® 3; Q ® 4; R ® 1; S ® 2
(b) P ® 2; Q ® 3; R ® 4; S ® 1
(c) P ® 4; Q ® 1; R ® 3; S ® 2
(d) P ® 1; Q ® 2; R ® 4; S ® 3

Answers
Paper 1
1. a,b,c,d 2. a,c,d 3. a,b,c,d 4. a, d 5. b, c 6. d 7. 1.00 8. 4.00 9. 1.00 10. 4.00
11. 6.00 12. 2.00 13. 0.00 14. 5.00 15. d 16. c 17. a 18. b 19. d 20. b,c
21. a, c, d 22. b 23. a, c, d 24. b 25. 5 26. 0.72 27. 4 28. 9 29. 20 30. 38.15
31. 25.84 32. 5.43 33. d 34. a 35. c 36. c 37. b,d 38. a,b,c 39. a,c,d 40. b,c
41. a,b,d 42. a 43. 21 44. 0.5 45. 0 46. 288 47. 3 48. 42 49. 5 50. 25
51. b 52. b 53. c 54. a

Paper 2
1. a, d 2. d 3. b,c 4. d 5. b,d 6. a 7. 0.50 8. 2.48 9. 5.55 10. 0.86
11. 3.33 12. 9.20 13. 2.64 14. 16.88 15. b 16. a 17. b 18. a 19. b 20. a, b, d
21. c 22. a 23. a, b, c 24. d 25. 7 26. 2 27. 6 28. 4 29. 921 30. 59.88
31. 0.01 32. 3.41 33. a 34. a 35. b 36. d 37. a,d 38. a,b,c,d 39. b,c 40. a, b, d
41. c, d 42. a,c,d 43. 2018 44. 4036 45. 14 46. 64 47. 0.4 48. 6 49. 4 50. 9
51. d 52. b 53. c 54. a

SCORE SHEET - Paper 1


Section No. of Marks from Marks from Marks Obtained
Correct Questions Correct Questions (A) Incorrect Questions (B) (A-B)
................ ............................... ............................... ............................... ...............................
................ ............................... ............................... ............................... ...............................
................ ............................... ............................... ............................... ...............................
Percentage Marks = Marks Obtain/Total Marks x 100

SCORE SHEET - Paper 2


Section No. of Marks from Marks from Marks Obtained
Correct Questions Correct Questions (A) Incorrect Questions (B) (A-B)
................ ............................... ............................... ............................... ...............................
................ ............................... ............................... ............................... ...............................
................ ............................... ............................... ............................... ...............................
Percentage Marks = Marks Obtain/Total Marks x 100
Note To expect your success marks in the test should be between 65%-70%.

PREP CATALYSIS
PREP CATALYSIS
JEE Advanced

PRACTICE SET 14 (With Solutions)

Duration : 3 Hours Max. Marks . 360

Paper 1
PHYSICS

Section 1 (Maximum Marks : 24)


l
This section contains SIX (06) questions.
l
Each question has FOUR options for correct answer(s). ONE OR MORE THAN ONE of these four option(s) is (are)
correct options(s).
l
For each question, choose the correct options(s) to answer the question.
l
Answer to each question will be evaluated according to the following marking scheme:
Full Marks : + 4 If only (all) the correct option(s) is (are) chosen.
Partial Marks : + 3 If all the four options are correct but ONLY three options are chosen.
Partial Marks : + 2 If three or more options are correct but ONLY two options are chosen, both of which are
correct options.
Partial Marks : + 1 If two or more options are correct but ONLY one option is chosen and it is a correct option.
Zero Marks : 0 If none of the options is chosen (i.e. the question is unanswered).
Negative Marks : - 2 In all other cases.
l
For example: If first, third and fourth are the ONLY three correct options for a question with second option being an
incorrect option; selecting only all the three correct options will result in + 4marks. Selecting only two of the three correct
options (e.g. the first and fourth options), without selecting any incorrect option (second option in this case), will result in
+2 marks. Selecting only one of the three correct options (either first or third or fourth option), without selecting any
incorrect option (second option in this case), will result in +1marks. Selecting any incorrect option(s) (second option in
this case), with or without selection of any correct option(s) will result in -2 marks.

1. In a resonance-column experiment to measure the 2. A player throws a ball horizontally with a spin as
velocity of sound, the first resonance is obtained at shown in the figure.
a length l1 and the second resonance at a length l 2 . A Top view
Then, which of the following is incorrect?
Player

of ball
(a) l2 > 3l1 (b) l2 = 3l1
(c) l2 < 3l1 B
(d) May be any of the above, depending on the frequency Now, choose the correct options.
of the tuning fork used

PREP CATALYSIS
298 JEE Advanced~Practice Set 14

(a) Due to spinning of ball pressure will decrease in region 5. A man can swim with a velocity v relative to water.
A He has to cross a river of width d flowing with a
(b) Due to spinning of ball pressure will decrease in region velocity u(u > v). The distance through, which he is
B carried downstream by the river is x. Which of the
(c) Ball will curve towards right of player following statement are correct?
(d) Ball will curve towards left of player du
(a) If he crosses, the river in minimum time, x =
3. A system is taken from A to B along path ACB. v
du
p (b) x cannot be less than
v
C B (c) For x to be minimum, he has to swim in a direction
p
making an angle of + sin-1 æç ö÷ with the direction of
v
2 èu ø
the flow of water
A D (d) x will be maximum, if he swims in a direction making an
V p
angle of - sin-1 æç ö÷ with the direction of the flow of
O v
2 èu ø
Work done in this process is 10 J. water
When the system returned from state B to state A 6. In a region a uniform magnetic field of 1.50 k$ T
along the curved path, the work done on the exists. Consider wire segments each carrying a
system is 20 J. Also, U A = 20 J and U D = 60 J. Now, current of 20 A, placed in a cube of side 1 m as
choose the correct options. shown in the figure.
(a) Heat absorbed in process AD = 50 J
Z
(b) Heat absorbed in process AB = 70 J
(c) Heat absorbed by system in process ADB = 60 J
(d) In the process BA (curved path), 70 J of heat is 3
absorbed by the gas B
i i
4. A thin-walled, spherical conducting shell S of Y 4
radius R is given charge Q. The same amount of
charge is also placed at its centre C. Which of the 2
i
following statements are correct?
Q X
(a) On the outer surface of S, the charge density is i
2 pR 2 1
(b) The electric field is zero at all points inside S Now, choose the correct options.
(c) At a point just outside S, the electric field is double, of (a) Force on segment 1 is - 0.3 $j (N)
the field at an inside point (b) Force on segment 2 is 0 (N)
(d) At any point inside S, the electric field is inversely (c) Force on segment 3 is 0.296 $i(N)
proportional to the square of its distance from C (d) force on segment 4 is ($i - $j ) 0.296 (N)

Section 2 (Maximum Marks : 24)


l
This section contains EIGHT (08) questions. The answer to each question is a NUMERICAL VALUE.
l
Four each question, enter the correct numerical value (in decimal notation, truncated/rounded-off to the second
decimal place; e.g. 6.25, 7.00.- 0.33, -.30, 30.27, -127.30) using the mouse and the on-screen virtual numeric
keypad in the place designated to enter the answer.
l
Answer to each question will be evaluated according to the following marking scheme:
Full Marks : + 3 If ONLY the correct numerical value is entered as answer.
Zero Marks : 0 In all other cases.

7. A U-tube is partially filled with a liquid. The 8. Two particles A and B are in XOY plane initially, at
horizontal part of the tube is 2 m in diameter. The (3m, 5m) and (-5m, -3m) velocities. They started
tube is accelerated horizontally with a constant m
to move with velocities, v A = 3 $i + 5 $j and
acceleration of 5 m/s 2 . What is the difference in s
the heights of the liquid in the two arms of the m
v B = ai$ + 7 $j at t = 0 s, respectively. If particles
U-tube? s
collides, then find the value of a.

PREP CATALYSIS
JEE Advanced~Practice Set 14 299

9. Light of wavelength 12. If n identical water droplets falling under gravity


Screen
l = 500 nm falls on with terminal velocity v coalesce to form a single
two narrow slits α C drop, which has the terminal velocity 4 v. Find the
d Centre
placed at a distance line value of number n.
-4
d = 50 ´ 10 cm α Transparent
slab 13. A satellite is moving with an angular speed equal
apart at an angle D
to the angular speed of earth’s rotation. If the
a = 30 ° relative to satellite is directly above the north pole at some
slits as shown in the figure. instant, then find the time it takes to come over
On the lower slit, a transparent slab of thickness the equatorial plane.
0.1 mm and refractive index 3/2 is placed. [Given, mass of the earth = 6 ´ 10 24 kg]
Interference pattern is observed on a screen placed
at distance D = 2 m from the slits. How many 14. A rectangular rigid fixed block has a long
fringes pass over C (centre of screen), if we remove horizontal edge. A solid homogeneous cylinder of
transparent slab? radius R is placed horizontally at rest with its
10. In the arrangement length parallel to the edge,
shown, the rod is freely such that the axis of the
pivoted at point O and is v cylinder and the edge of the
in contact with the block are in the same vertical R
O θ
equilateral triangular plane as shown in the figure.
2a/√3 There is sufficient friction
block, which can moves on
the horizontal frictionless ground. As the block is present at the edge, so that a
given a speed v forward, the rod rotates about point very small displacement causes,
O. Find the angular velocity of rod in rad/s at the the cylinder to roll of the edge without slipping.
instant, when q = 30 °. [Take, v = 20 m/s, a = 1 m] Determine the ratio of the translational to
11. A boy rolls a hoop over a horizontal path with a rotational kinetic energies of the cylinder, when its
speed of 7.2 km/h. Over, what distance can the hoop centre of mass is in horizontal line with the edge.
roll uphill at an incline of angle q at the expense of
1
its kinetic energy? [Given, sinq = ]
10

Section 3 (Maximum Marks : 12)


l
This section contains TWO (02) paragraphs. Based on each paragraph, there are TWO(02) questions.
l
Each question has FOUR options. ONLY ONE of these four options corresponds to the correct answer.
l
Four each question, choose the option corresponding to the correct answer.
l
Answer to each question will be evaluated according to the following marking scheme:
Full Marks : + 3 If ONLY the correct option is chosen.
Zero Marks : 0 If none of the options is chosen (i.e. the question is unanswered).
Negative Marks : - 1 In all other cases.

Paragraph X The gas is driven by a pressure difference Dp = p 1 - p 2 .


Figure shows a self-excited magneto-hydrodynamic Metal electrode of length l are set in the top and bottom
(MHD) generator in which an electrically walls of the duct, while a magnetic field exists
conducting gas of electrical conductivity s moves with perpendicular to the insulated side walls of the duct.
speed v along a duct with a square cross-section of An emf is therefore set up between the electrodes,
dimension a. which drives a current through a long solenoid of n
turns per unit length and negligible resistance, which in
duct turn provides the magnetic field.
15. The resistance R in the path provided by the gas
υ
between the electrodes is
p1 p2
l 1 l 1 1
(a) s (b) (c) (d)
electrode a sa sl s
solenoid

PREP CATALYSIS
300 JEE Advanced~Practice Set 14

16. The current generated is given by Mass of electron (me ) being negligible in comparison to
(a) i = aDPslv (b) a(DPslv )1/ 2 that of nucleus, mass comes to be almost m and taking this
approximation R H is calculated to be 109677 cm -1 .
(c) (aDPslv )1/ 2 (d) DP (aslv )1/ 2
In an atom or ion the energy of electron is quantized and
Paragraph A lowest possible energy state is known as ground state.
Spectral lines in the atomic spectra result due to The energy state just above the ground state is called 1st
downward transition of electron from some excited state excited state and so on. The amount of energy required to
(principal quantum number (n + x ) ) to less excited state or remove, the most loosely bound electron in an atom or ion
ground state (principle quantum number n). The wave in its gaseous state is called I p . All the other terms in
numbers of spectral lines in the spectra of atom or ion above equation have their usual meanings.
containing only one electron fit into the empirical
equation. 17. The wave number of the spectral line of H-atom
corresponding to the longest wavelength transition
1 é 1 1 ù
n = = RZ 2 ê 2 - ú
of the Lyman series is
l êë n (n + x ) 2 úû (a) R H (b)
3
RH
4
where, n = 1, 2, 3 and 4 for the lines of Lyman, Balmer, 5 R
Paschen and Brackett series, respectively and it remain (c) RH (d) H
36 2
the same for all the lines of a given series, whereas
x = 1, 2, 3 ¼¼ for successive lines 18. Suppose electron in H-atom is replaced by negative
2p 2 mK 2 e 4 meson (p) having the same charge as that of
RH = = Rydberg constant of hydrogen electron, but almost 200 times as heavier as
h 3C electron. The wave number of the second line of
m = reduce mass of atom or ion Balmer series of this mesonic H-atom ignoring
mass of electron ´ mass of nucleus nuclear motion will be
=
mass of electron + mass of nucleus (a) 4.113 ´ 108 m -1 . ´ 108 m -1
(b) 4113
. ´ 108 m -1
(c) 4113 (d) 4113
. ´ 106 m -1

CHEMISTRY
Section 1 (Maximum Marks : 24) (d) In 3d series, E° value is positive for Cu2+ /Cu while for
Instructions : Same as given in Physics others E° value is negative

19. Calcium cyanamide is obtained when X combines 21. Which of the following statement is/are correct
with calcium carbide at about 1000°C. At room regarding lanthanides?
temperature ‘X’ is inert but combine with (a) Cerium shows +4 and +3 oxidation states
atmospheric oxygen at about 3000°C to form its (b) Element with atomic numbers 66 and 99 belong to
oxide which is also a product of the reaction of f-block
copper and dilute nitric acid. The formula of oxide (c) Lanthanides belong to 3rd group and 7th period
of X is (d) Monazite is the mineral that constitutes the major
source of the lanthanides
(a) NO (b) N2O3
(c) N2O5 (d) NO2 22. In the following sequence, the correct structure(s)
20. Which of the following statement is/are correct for of X is/are
transition elements? +
(i) H /∆ O
2+ (ii) O3, Zn /CH3COOH
(a) In any transition series, magnetic moment of M ions X
(iii) NaOH
first increases and then decreases
(b) Metals of 4d and 5d series have greater enthalpies of OH
atomisation than 3d series O
OH OH
(c) In any series, metals with highest oxidation numbers lie CH3
towards the end of the series (c) (d)
(a) (b)

PREP CATALYSIS
JEE Advanced~Practice Set 14 301

23. The reaction(s) leading to the formation of Initially NaOH and HCl are at 22.50°C and final
2-cyclopentylbutan-2-ol is/are temperature of the mixed solution is 24.90°C. The
O
heat of neutralisation is ………
. g mL -1 and
MgBr +
H3O [Given : Density of solution = 100
(a) +
specific heat = 4 .184 J/g °C]
O +
H3O
(b) 27. The minimum concentration of NH 3 required to
+ H3C CH2 CHMgBr
prevent AgCl(s) from precipitating from 1.00 L of a
O CH3 solution containing 0.10 mol AgNO 3 and 0.010 mol
+
NaCl is ……… .
(c) CH3 + CH3 CH2 MgBr H3O
. ´ 10 -10 , K f[Ag(NH 3)+2 ]
[Given, K sp (AgCl) = 18
O
= 16
. ´ 10 7 ]
+
(d) + CH3 MgBr H3O
28. Surface area of one gram of activated charcoal is
10 3 m 2 . If there is complete coverage by
24. Consider the given plots for four gases at 300 K. monolayer, then the volume of NH 3 at STP that
would be absorbed on the surface of 38 g of the
1.01
charcoal is ……… cm 3 .
A
1.0 [Given : Diameter of NH 3 molecule = 0 .42 nm]

0.99 29. PCl 5 vapour decomposes on heating according to


B the reaction :
N

0.98 PCl 5(g) - PCl (g) + Cl (g)


3 2
Vm = Molar volume C
0.97 The density of a sample of a partially dissociated
Z = Compressibility
PCl5 at 1.0 atm and 500 K was found to be 4.8 g/L.
factor
0.96 D The value of DG° for the reaction at 500 K will be
0 0.02 0.04 0.06 0.08 0.1
……… .
1 30. The isomerisation of cyclobutene to 1, 3 -butadiene
Vm follows a first order kinetics and the rate constant
If the slope of line B is - 0.10 mol -1 L, then the is 2 ´ 10 -4 s-1 at 150°C in a 530 mL flask. If an
relationship between van der Waals’ constants ‘a’ isomerisation reaction is carried out at 150°C with
and ‘b’ is ……… . an initial pressure of 55 mm of Hg, the
concentration of cyclobutene after 30 mins is
[Ignore the higher terms of virial equation.] x ´ 10 -3 M. The value of x is ……… .
(a) a = æç b +
1ö 1
÷ atm L2 mol -2
è 10 ø 24.6 31. Elevation in boiling point, DTb is also given as
DTb = K x ,B.P, xsolvent , where K x ,B.P is the mole
(b) b = æç a - ö÷ ´ 24.6 L mol -1
1
è 10 ø fraction boiling point elevation constant. The
boiling point of a solution of 1.00 g napthalene
(c) a = æç b +
1ö -2
÷ ´ 24.6 atm L mol
2
(C10H18) dissolved in 100.00 g toluene (C7H 8) is
è 10 ø ……… .
(d) b = æç a +
1ö -1
÷ ´ 24.6 L mol [Given : The normal boiling point of toluene is
è 10 ø
110.7°C and K x , B.P = 36 .1 K]

Section 2 (Maximum Marks : 24) 32. Two flasks A and B of uniform size are joined by a
tube of 40 cm long as shown. A contains NH 3 and
Instructions : Same as given in Physics
B contains HCl. The distance from side A where
25. The number of oxide ores among rutile, beryl, white fumes first appear when stopcocks are
tinstone, pyrolusite, chromite, corundum, openend and closed simultaneously is ……….
chalcosite, gypsum, cryolite, feldspar and pitch A B
blende are O
NH3 HCl
26. A quantity of 100 mL of 0.5 M HCl is mixed with 40 cm
100.0 mL of 0.5 M NaOH in a constant pressure
calorimeter that has a heat capacity of 335 J/°C.

PREP CATALYSIS
302 JEE Advanced~Practice Set 14

Paragraph A
Section 3 (Maximum Marks : 12) A hydrocarbon A (C10 H18 ) is capable of showing both
Instructions : Same as given in Physics enantiomerism as well as diastereomerism. Treatment of
A either with HgSO 4 /H 2 SO 4 or B 2 H 6 /H 2 O 2 , NaOH
Paragraph X results in the same carbonyl compound B. Also,
A chiral organic compound ‘A’ (C 9 H12 O 2 ) does not Na O3
A ¾¾¾® B ¾¾® X (C 5 H10 O) C can also be
give positive Tollen’s test or iodoform test. However, ‘A C2H5 OH(Dry) Zn-H2O
’on treatment with 1.0 equivalent of CH 3 MgBr obtained as one of the product in the following reaction.
followed by acid hydrolysis gives B(C10 H16 O 2 ) that CO2 (CO) 8
But-l-ene + CO + H 2 ¾¾¾®C + other product.
gives positive iodoform test. B on refluxing with aqueous
Na 2 CO 3 solution gives the following compound as major 35. Which of the following is the most likely structure
product. of B ?
CH3 O CH3
Na2CO3 | || |
B Heat (a) CH3 ¾ C ¾ C ¾ CH2 ¾ C ¾ CH3
O | |
CH3 CH3
33. What is the most probable structure of A? CH3 CH3
| |
(a) (b) (b) CH3 ¾ CH2 ¾ CH ¾ C ¾ CH2 ¾ CH ¾ CH2 CH3
||
O O O O O
O O
||
(c) (d) (c) CH3 CHCH2 C(CH2 )2 CH ¾ CH3
O O O | |
CH3 CH3
34. B on treatment with hydrazine (N 2H 4) gives a cyclic O
hydrazone. What is the structure of hydrazone?
(d)

(a) (b) CH3


N
N N N 36. Consider the following reaction,
Et 2O H+
(c) N (d) C + CH 3 MgBr ¾® ¾® Alcohols
N H 2O
N N How many different alcohols are expected?
(a) 1 (b) 2 (c) 3 (d) 4

MATHEMATICS
Section 1 (Maximum Marks : 24) 38. Let f:(-¥ , ¥) ® (0 , ¥) be a differential function and
Instructions : Same as given in Physics satisfies 3f (x + y) = f (x) × f (y) for all x , y Î R with
37. For an non-zero complex number z1 and z 2 such 1 = 6 . Then, which of the following is (are)
f()
TRUE?
that z1 = a + ib and z 2 = c + id, where a , b,c ,d Î R
æ ö
(a) The value of lim x ç f æç1 + ö÷ - f(1)÷ is 6 ln2
1
and z1 = z 2 = 1 and Im(z1z2 ) = 0. If w1 = a + ic and
x®¥ è è xø ø
w 2 = b + id, where Im( z ) and Re(z) denotes the
imaginary and real part of z respectively. Then, (b) The curve y = f( x) passes through the point (2,12 )
which of the following is(are) TRUE? (c) The area bounded by the curve y = f( x), the X-axis and
(a) Im(W1 W2 ) = 0 (b) Im(W1 W2 ) = 0 the Y-axis is ln 8 sq units
æW ö æW ö (d) The area bounded by the curve y = f( x), the X-axis and
(c) Im ç 1 ÷ = 0 (d) Re ç 1 ÷ = 0 the Y-axis is 3log 2 e sq units
è W2 ø è W2 ø

PREP CATALYSIS
JEE Advanced~Practice Set 14 303

39. Consider the line L1:x = y = z and the line 45. The value of the integral
L 2 : 2 x + y + z - 1 = 0 = 3 x + y + 2 z - 2. Then, which 2018 dx
of the following statement(s) is(are) TRUE?
ò-20181 + sin2019 x + 1 + sin 4038 x
equals to ……

(a) Plane containing the line L2 and parallel to line L1 is


x- z-1= 0 46. Let the product of all the divisors of 1440 be P. If P is
(b) The shortest distance between the two lines is 1 / 2 divisible by 24 x , then the maximum value of x is ……
, ,-1)
(c) The direction ratio of line L2 is (11 r r r
47. If a , b,c are unit vectors, then the maximum value
(d) The perpendicular distance of origin from plane r r2 r r2 r r2
containing line L2 and parallel to line L1 is
1 of a - 2 b + b - 2c + c - 2a is equal to ……
2

40. A differentiable function satisfies 48. Let A be the set of positive integers that have no
x prime factors other than 2, 3 or 5. The infinite sum
f (x) = ò0 [f (t)cos t - cos(t - x)]dx. Then, which of 1 1 1 1 1 1 1 1 1
+ + + + + + + +
following statements is(are) TRUE? 1 2 3 4 5 6 8 9 10
(a) Minimum value of f(x ) is (1- e ) 1 1 1 1 1
+ + + + + …… of the reciprocal of the
(b) Maximum value of f(x ) is (1 - e -1 ) 12 15 16 18 20
(c) f ¢(0) + f ¢¢(0) = -2 m
elements of A can be expressed as , where m and
(d) f ¢(p ) + f ¢¢(p / 2 ) = e - 1 n
n are relatively positive integers, then the value of
41. An altitude BD and a bisector BE are drawn in the m + n is
D ABC from the vertex B. It is known that the
length of side AC = 1 and the magnitude of the 49. The ratio of the maximum value to minimum value
angle BEC, ABD, ABE, BAC form an arithmetic of 2 cos 2 x + cos x + 1 is m:n , where m and n are
progression. Then, which of the following relatively positive integers, then the value of m + n is
statements is (are) TRUE?
(a) The area of circle circumscribing DABC is p / 4 sq units
50. If the area bounded by the graph of y = xe- ax (a > 0)
3 1
(b) The area of D ABC is sq units and the abscissa axis is , then the value of ‘a’ is
4 9
(c) Let B¢ be the image of point B with respect to side BC of equal to
3
DABC, then the length of BB¢ is equal to units
2 Section 3 (Maximum Marks : 12)
3
(d) The area of DABC is sq units Instructions : Same as given in Physics
8
42. Let f (x) = x + (1 - x)x 2 + (1 - x)(1 - x)2 x 3 Paragraph X
+(1 - x)(1 - x 2)(1 - x 3)x 4 +¼+ (1 - x)(1 - x 2)... The function of satisfies f ( x ) + f (2x + y ) +
5xy = f (3x - y ) + 2x 2 + 1 for all real numbers x , y. Let a
(1 - x n -1)x n , n ³ 4. Then, which of the following
is(are) TRUE? chord to parabola x 2 = 4y, normals to parabola at ends of
n n which satisfy the relation, m1m 2 = -2, where m1 , m 2
(a) f(x ) = - Õ (1 - x r ) (b) f(x ) = 1 - Õ (1 - x r )
represents slope of normals, passes through a fixed point ‘
r =1 r =1
æ n rx r -1 ö æ n rx r -1 ö
p’ on axis of parabola. Let y = g ( x ) represent line passing
(c) f ¢(x ) = (1 - f(x ))çç å ÷ (d) f ¢(x ) = f(x )çç å
r ÷
÷
r ÷
through point P.
è r =1 (1 - x )ø è r =1 (1 - x )ø
51. Let y = g (x) intersects y = f (x) at two distinct points
A, B, then the slope of g (x) if length of segment AB
Section 2 (Maximum Marks : 24) is 4 units, is
Instructions : Same as given in Physics (a) ±1 (b) ±2 (c) ±3 (d) ±4
43. If log y x + log x y = 2 and x 2 + y = 12, then the value
52. The minimum area bounded by y = g (x) and
of xy is equal to …… y = f (x) is
¥ æ n2 + n + 4 ö 1 1
44. If å 2cos -1 ç ÷ = kp, then the value of k is
(a)
3
(b)
2
n =0 è 2 ø 2 5
(c) (d)
……… . 3 6

PREP CATALYSIS
304 JEE Advanced~Practice Set 14

Paragaph A 53. If a student attempts only two questions. One from


In an examination in an objective test there are two Section-1 and one from Section-2, then the
sections containing 10 questions each. In section-1 each probability that he is awarded marks in both
objective question have 5 choices and only one choice is question is
correct . Section-2 has 4 choices for each question and 1 1 1 1
(a) (b) (c) (d)
more than one choice may be correct. Marks of section-2 25 75 9 80
is awarded only if student gives all correct choices. There 54. The probability of getting marks less than 40 for a
is no negative making for any section. For each question students is
in section-1 and section-2, 1 and 3 marks will be awarded 10 10 10 10
(a) æç ö÷ (b) æç ö÷ (c) 1 - æç ö÷ (d) 1 - æç ö÷
1 74 74 1
respectively. è 75 ø è 75 ø è 75 ø è 75 ø

Paper 2
PHYSICS
Section 1 (Maximum Marks : 24)
l This section contains SIX (06) questions.
l Each question has FOUR options for correct answer(s). ONE OR MORE THAN ONE of these four option(s) is (are)
correct options(s).
l For each question, choose the correct options(s) to answer the question.
l Answer to each question will be evaluated according to the following marking scheme:
Full Marks : + 4 If only (all) the correct option(s) is (are) chosen.
Partial Marks : + 3 If all the four options are correct but ONLY three options are chosen.
Partial Marks : + 2 If three or more options are correct but ONLY two options are chosen, both of which are
correct options.
Partial Marks : + 1 If two or more options are correct but ONLY one option is chosen and it is a correct option.
Zero Marks : 0 If none of the options is chosen (i.e. the question is unanswered).
Negative Marks : - 2 In all other cases.
l
For example: If first, third and fourth are the ONLY three correct options for a question with second option being an
incorrect option; selecting only all the three correct options will result in + 4marks. Selecting only two of the three correct
options (e.g. the first and fourth options), without selecting any incorrect option (second option in this case), will result in
+2 marks. Selecting only one of the three correct options (either first or third or fourth option), without selecting any
incorrect option (second option in this case), will result in +1marks. Selecting any incorrect option(s) (second option in
this case), with or without selection of any correct option(s) will result in -2 marks.

Now, choose the correct options.


1. A thin convex lens made up of refractive index m 0 (a) Torque acting on the dipole is zero
is immersed in liquid m1 , then which statement is
(b) Force acting on the dipole due to the electric field
wrong? produced by Q is zero
(a) Lens will be converging, if m1 > m 0
(c) Potential energy of the dipole due to the point chargeQ is
(b) Lens will be convex only, if m1 < m 0 Qp
(c) Lens will become convex only, if m1 > m 0
4pe0 r 2
(d) Lens will be diverging, if m1 > m 0
(d) Force acting on the dipole due to the point charge Q is
2. A dipole of dipole moment p is Qp
placed at a distance r from a 2 pe0 r 3
point charge Q (as shown in p
figure). Q r

PREP CATALYSIS
JEE Advanced~Practice Set 14 305

3. A particle of mass m is doing horizontal circular 5. Block A of mass 1 kg is placed on a smooth


motion with the help of string (conical pendulum) horizontal surface. A smooth block B of mass 1 kg
as shown in the figure. If speed of the particle is starts to move on the inclined surface of block A.
constant, then Then, choose the correct options.
Z (vertical)

B
O 37° 53°
Y
θ g
X A

(a) Acceleration of A is 6g/17


(b) Acceleration of B with respect to A is 15g/17
(a) the angular momentum of the particle about O is
(c) Acceleration of B is 15g/17
changing
(d) Acceleration of A is 12g/17
(b) magnitude of angular momentum about O remains
constant. 6. A thin rigid uniform circular disc rolls without
(c) Z-component of the angular momentum remains slipping on a horizontal rigid surface (or the
conserved ground). At a certain instant, its position w.r.t
(d) Z-component of torque is always zero ground frame is as shown in the figure.
4. The network shown in the figure is part of a Y′ Translatory
complete circuit. Let at a certain instant, the Y B CM frame
electric current i is 10 A. v
A X´
10mH 10mH O´ C
A B ω
i 20V X
Ground D
3
(a) If electric current is increasing at a rate of 10 A / s, then frame
VA - VB is zero (a) Sector ABC has greater KE than sector ADC w.r.t
(b) If electric current is increasing at a rate of 103 A / s, then ground frame
time rate of increase of magnetic energy is one of the (b) Sector BO ¢C has greater KE than sector CO ¢D w.r.t
inductors is 10 J/s ground frame
(c) If electric current is increasing at a rate of 103 A/s, then (c) Sector BO’C has the same KE as sector AO ¢B, w.r.t
time rate of increase of the total magnetic energy stored ground frame
in both, the inductors is 200 J/s (d) All the sectors AO¢B, BO ¢C, CO ¢D and AO ¢D have same
(d) None of the above KE w.r.t. the centre of mass frame

Section 2 (Maximum Marks : 24)


l
This section contains EIGHT (08) questions. The answer to each question is a NUMERICAL VALUE.
l
Four each question, enter the correct numerical value (in decimal notation, truncated/rounded-off to the second
decimal place; e.g. 6.25, 7.00.- 0.33, -.30, 30.27, -127.30) using the mouse and the on-screen virtual numeric
keypad in the place designated to enter the answer.
l
Answer to each question will be evaluated according to the following marking scheme:
Full Marks : + 3 If ONLY the correct numerical value is entered as answer.
Zero Marks : 0 In all other cases.

7. Two bodies were thrown simultaneously from 8. A disc (mass = 50 g) slides with zero initial velocity
same point one straight up and other at 60° up down an inclined plane (q = 30 °). After moving 50
with horizontal. cm along the plane, the disc stops. Find work done
Initial velocity of each of bodies is 25 ms - 1 . by the friction.
Neglecting air resistance distance between bodies (Take, m = 0.15)
after, t = 1.75 is ......... .

PREP CATALYSIS
306 JEE Advanced~Practice Set 14

9. A glass capillary of length 110 mm and inside C1 C1


A
diameter 20 mm is submerged vertically into water.
The upper end of the capillary is sealed. Outside C2
pressure is atmospheric. To what length the C2
capillary to be submerged to make water levels
inside and outside same? B
10. A thin half ring (R = 20 cm) is uniformly charged Find the value of VAB .
(q = 0 .7 nC). Electric field strength at centre is 12. Calculate the period of oscillation of a hydrometer,
æ kV ö which is slightly pushed down vertically.
........... ç ÷.
èmø Mass of hydrometer is 50 g, radius of tube is
3.2 mm and density of liquid is 1g/cm 3 .
11. Given in the circuit shown, battery emf r = 110 V,
C2 13. Activity of a radioactive sample decreases 2.5 times
capacitance ratio = = 2.0 . after 7.0 days. Half life of sample in days is ........... .
C1
14. Velocity of photo electrons liberated by
electromagnetic radiation of wavelength l = 18 nm
from stationary. Helium (He+) ions in the ground
state is v ´10 6 ms- 1 . Find the value of v.

Section 3 (Maximum Marks : 12)


l This section contains FOUR (04) questions.
l Each question has TWO (02) matching lists : Column-I and Column-II.
l FOUR option are given representing matching of element from Column-I and Column-II. ONLY ONE of these four
options corresponds to a correct matching.
l For each question, choose the option corresponding to the correct matching.
l
Answer to each question will be evaluated according to the following marking scheme:
Full Marks : + 3 If ONLY the correct option is chosen.
Zero Marks : 0 If none of the options is chosen (i.e. the question is unanswered).
Negative Marks : - 1 In all other cases.

15. A bead of mass m slides on a smooth rigid wire in Codes


a vertical plane. As the bead passes through P, its A B C D
centre of rotation changes from A to B. Now match (a) 3 1 4 2
entries of Column I with their values in Column II. (b) 4 2 3 1
(c) 4 3 1 2
B (d) 2 3 4 1

P R 16. Considering an inertial frame, match entries of


m Column I with Column II.
R
A Column I Column II

A. Fnet 1. dU
Column I Column II -
dx
A. Change in normal reaction 1. 2v2 B. Fconservative 2. dk
R dt
B. Change in magnitude 2. 0 C. Total power 3. ma
of acceleration
D. Fnet × v 4. dp
C. Change in angular velocity. 3. 2 mv 2 dt
R
where, F = force, v = velocity, K = kinetic energy,
2v
D. Change in magnitude 4. p = momentum, m = mass, a = acceleration, U =
of velocity. R
potential energy and t = time.

PREP CATALYSIS
JEE Advanced~Practice Set 14 307

Codes Codes
A B C D A B C D A B C D
(a) 1 2 3 4 (b) 4 2 3 1 (a) 1 2 3 4
(c) 4 3 1 2 (d) 3,4 1 2 2 (b) 3 4 1 2
(c) 4 3 1 2
17. Match entries of Column I and Column II. (d) 2 3 4 1
Column I Column II 18. A man is crossing a river, If v mw = velocity of man
A. A ball of mass m strikes a stationary 1. m relative to water, v w = velocity of water w.r.t
ball of mass M centrally. The fraction of M ground, v m = velocity fo man relative to ground,
total mechanical energy, which cannot match Column I and Column II.
be lost during collision is
B. A bullet of mass m fired horizontally can 2. 4Mm Column I Column II
penetrate a block of mass M kept fixed, ( M + m)2 A. Minimum distance, 1. æv ö
through a distance x. If the block is free sin- 1 ç mw ÷
when v mw > v w è vw ø
to move, the distance of penetration of

the bullet is x¢. Then, is B. Minimum time for 2. v m ^ v w
x v mw ³ v w
C. A man of mass m jumps out of a boat 3. m
C. Minimum distance for 3. v mw ^ v w
of mass M horizontally with a speed u M + m v mw > v w
relative to the boat. In consequence,
D. Minimum time for 4. æ v ö
the boat recoils with a velocity v. The sin- 1 ç w ÷
v v mw < v w è v mw ø
value of is
u -v
D. A ball of mass m strikes a stationary 4. M Codes
trolley car of mass M resting over a M+ m A B C D
smooth rail. If the collision is head-on (a) 1 2 3 4
and elastic, then the fraction of (b) 4 2 3 1
mechanical energy lost by the ball is (c) 2,4 3 1 3
(d) 2 3 4 1
M
m

CHEMISTRY
X turns FeCl 3 yellow coloured solution to green.
Section 1 (Maximum Marks : 24) X gives white ppt. with NaOH soluble in excess of
Instructions : Same as given in Physics NaOH.
19. An aqueous pink solution of cobalt (II) chloride X gives yellow dirty ppt. on passing H 2S gas, soluble
changes to deep blue colour on addition of excess in yellow ammonium sulphide.
of HCl. This is because X gives white precipitate with HNO 3 /AgNO 3 .
(a) [Co(H2O)6 ]2+ is transformed into [CoCl 6 ]4- Substance ‘X’ can be
(b) [Co(H2O)6 ]2+ is transformed into [CoCl 4 ]2- (a) SnCl 4 (b) SnCl 2
(c) tetrahedral complexes have smaller crystal field splitting (c) CuCl 2 (d) AsCl 3
than octahedral complexes 21. o-methylnitrobenzene reacts with Sn/HCl to form ‘
(d) tetrahedral complexes have larger crystal field splitting A’. ‘A’ undergoes the following transformations
than octahedral complexes
(i) (CH3 CO) 2 O (i) NaNO2 , HCl (i) Sn/ HCl, OH -
20. Consider the following statements about salt ‘X’. A ¾¾¾¾¾®B ¾¾¾¾¾® C ¾¾¾¾¾® D
(ii) HNO3 +H 2SO4 (ii) CuCl, D (ii) HNO2
X reduces HgCl 2 solution to white ppt. changing to (iii) OH - (iii) H3 PO2
grey. (iv) Cl 2

PREP CATALYSIS
308 JEE Advanced~Practice Set 14

The major product(s) of the following sequence


is/are Section 2 (Maximum Marks : 24)
Me Me Instructions : Same as given in Physics
Cl NH2 Cl 25. The total number of unpaired electrons in the
given complexes are ……… .
(a) (b)
[Ti(H 2O)6 ]3+ , [V(H 2O)6 ]3+ , [Cr(H 2O)6 ]3+ ,
NO2 Cl
[Mn(CN)6 ]3- , [Fe(CN)6 ]4- , [FeCl 4 ]2-
Me Me
Cl NH2 26. A conductivity cell has electrodes 0.656 cm apart
(c) (d) and each has a area of 0 .8 cm 2 . Given, equivalent
Cl conductance of saturated solution of AgCl = 130
NO2 Cl
S cm 2 equiv -1 , solubility product of AgCl
22. Consider the following sequence of . ´ 10 -10 . It (AgCl) offers resistance of n ´ 10 5
= 16
transformations. ohm. The value of n is ……… .
Ba(OH) 2
DNA ¾¾® (P) 27. In the cubic crystal of CsCl (d = 3.97 g cm -3) the
MgO/ D HCl eight corners are occupied by Cl - ions with a Cs + at
DNA ¾® PO34- + (Q) ¾®(R) + (S) + sugar the centre and vice-versa. The radius ratio of the two
In the scheme shown above (P), (Q), (R) and (S) ions is ……… .
respectively are : 28. For the given compound X, the total number of
(a) (P) = purine base, (Q ) = pyrimidine base, chiral centres is ……… .
(R ) = nucleotide, (S ) = nucleoside Cl
(b) (P) and (Q ) are nucleotides, (R ) = pyrimidine bases,
(S ) = purine bases N
N H
(c) (P) = nucleoside, (Q ) = nucleotide, (R ) and (S ) = purine
bases
(d) (P) = nucleotide, (Q ) = nucleoside, (R ) = pyrimidine base,
(S ) = purine base
29. Aspirin C9H 8O 4, is prepared by heating salicylic
23. Consider the following sucessive decays: acid, C7H 6O 3 , with acetic anhydride, C4H 6O 3 . The
l1 l2 other product is acetic acid, C2H 4O 2 .
A ¾® B ¾® C : l1 and l 2 are the respective decay
constants. C7H 6O 3 + C4H 6O 4 ¾® C9H 8O 4 + C2H 4O 2
If initially only A is present and at some point an What is the percentage yield of aspirin when 2.00 g
equilibrium is established between A and B, which of salicylic acid is heated with 4.00 g of acetic
of the following will be true at equilibrium? anhydride? (Given : actual yield of aspirin is 2.1 g)
dNA dNB 30. To an aqueous solution of weak acid (HA) 20 mL
(a) - =
dt dt NaOH is added and connected to a reference
æ NA ö æ l 2 ö
(b) ç ÷ = ç ÷ where, NA and NB are the number of Ag|AgCl|Cl- (0.1 M) electrode. Emf of the cell was
è NB ø è l1 ø found to be 0.47 V. To the test solution, 30 mL of
atoms of A and B. NaOH from same stock was added further and new
(c) If l 2 >> l1, equilibrium would be established quickly. emf was found to be 0.50 V. The acid dissociation
(d) NB increases initially, attains a maximum value and then constant of the weak acid is ……… ´ 10 -4 .
decreases
[Given : E° (AgCl|Ag,Cl - ) = 0.23 V]
24. Empirical formula of a chloride of titanium is TiCl x 31. Dichlorodifluoromethane, CCl 2F2, one of the
and its 0.01 mol on treatment with excess of chlorofluorocarbon refrigerants responsible for
AgNO 3 gives 5.74 g of AgCl (molar mass of destroying part of Earth’s ozone layer, has vapour
AgCl = 143.5 g). Which of the following statements pressure = 40 .0 mm Hg at -816 . °C and vapour
regarding TiCl x is/are) true? (Molar mass of pressure = 400 mm Hg at -43.9 °C. The normal
Ti = 48) boiling point of CCl 2F2 is ……… .
(a) Its empirical formula is TiCl 3 .
32. In a certain polluted atmosphere containing O 3 at
(b) Its empirical formula is TiCl 4 .
a steady state concentration of 2.0 ´ 10 -8 M, the
(c) TiCl x has 25.26% Ti by weight.
hourly production of O 3 by all sources was
(d) TiCl x has 31% Ti by weight.
estimated as 7 .2 ´ 10 -15 M.

PREP CATALYSIS
JEE Advanced~Practice Set 14 309

If the only mechanism for destruction of O3 in the S. CH2Br 4. Heat


second order reaction is 2O 3 ¾® 3O 2. The rate H2C
constant for the destruction reaction, defined by C2H5O2C
the rate for -D[O 3 ] / Dt is x ´ 10 -3 m -1 s -1. The +
value of x is ……… .
O

Section 3 (Maximum Marks : 12) (i) K


(ii) KOH
(iii) HCl
Instructions : Same as given in Physics (iv) Na/moist ether

33. Match each set of molecules from List I with their


shapes given in List II. 5. HF
List I List II 6. Zn-Hg/HCl
P. IBr2¨ 1. T-shape 7. Pd-C
Q. SF4 2. Linear
8. Se
R. ClF4 3. Square planar
S. BrF5 4. Sea-saw 9. P2O 5
T. XeF4 5. Square pyramid
The correct option is
The correct option is (a) P ® 8; Q ® 1, 2, 3, 4; R ® 5, 6, 7; S ® 8, 9
(a) P ® 5; Q ® 4; R ® 3; S ® 2; T ® 1 (b) P ® 1, 2, 3; Q ® 4, 5; R ® 6, 7; S ® 8, 9
(b) P ® 1; Q ® 2; R ® 3; S ® 4; T ® 5
(c) P ® 1, 4, 3; Q ® 1, 5; R ® 2, 5; S ® 2, 9
(c) P ® 2; Q ® 4; R ® 1; S ® 5; T ® 3
(d) P ® 4; Q ® 3; R ® 2; S ® 1; T ® 5 (d) P ® 1, 8; Q ® 6, 2; R ® 3, 4; S ® 5, 7

34. The desired product ‘X’ can be prepared by reacting 35. List I contain reactions and List II contains
the major product of the reactions in List I with reagents.
one or more appropriate reagents in List II. List I List II
P. 1. CH2N2 ,
(X) Ag 2O/Pt, H2O
Diethyladipate

O
¨
Q. OEt Pentan-3-one 2. OH, H2O/H+
List I List II
Me
P. CH2CH2MgBr 1. H+
O

R. RCOCl ¾® RCH 2COOH 3. NaH/EtOH,


H3O+, D
O
H2SO4 S. BrCH 2COOEt ¾® Me2 C ¾ CH2 COOEt 4. EtONa/EtOH,
+ | H3O+, D
OH

Q. CHO 2. NaNO 2 /H2SO 4


T. 5. Zn,
O Me2 C == O,
NO2
H2O
CH2CO2Na Cl COOH

(CH3CO)2O 6. Carbene
+
Match each reaction is List I with one or more
R. 3. Cu, H2 SO 4 products in List II and choose the correct option
(a) P ® 1; Q ® 4; R ® 3; S ® 2, 5; T ® 6
(b) P ® 3; Q ® 4; R ® 1, 6; S ® 5; T ® 2
(i) AlCl3 (c) P ® 1; Q ® 2; R ® 3; S ® 4; T ® 5, 6
CH2CO (ii) Zn–Hg
+ O (d) P ® 1; Q ® 2; R ® 3, 4; S ® 5; T ® 6
HCl
CH2CO

PREP CATALYSIS
310 JEE Advanced~Practice Set 14

36. Match each set of electronic transitions from list I D. n= 5®n=1 4. Spectral lines in
with the spectral lines given in the list II. (H-atom) infrared region.
5. Spectral lines in UV
List I List II
region.
A. n=6®n=3 1. 10 lines in the
(H-atom) spectrum. The correct option is
A B C D
B. n=7 ®n= 3 2. Spectral lines in
(H-atom) visible region. (a) 3, 4 1, 4 2, 3, 4 1, 2, 4, 5
(b) 3, 4 1, 4 2, 3, 5 1, 2
C. n= 5®n=2 3. 6 lines in spectrum. (c) 3, 5 1, 4 2, 5 1, 2
(H-atom)
(d) 3, 5 1, 4 5, 3 4, 5

MATHEMATICS
¥
1 1
Section 1 (Maximum Marks : 24) (c) The value of å 8D is
81
n =1 n+1
Instructions : Same as given in Physics ¥
1 1
(d) The value of å is
æ pö 8Dn+1 72
37. Let f (x) = çcos x - cos ÷ n =1
è 8ø
ìï x
2n
; x Î[2 n ,2 n + 1)
æ 3p ö æ cos 5 p ö æ cos 7 x ö 40. Let f (x) = í , n ÎZ. Then,
çcos x - cos ÷ çcos x - ÷ çcos x - ÷ 2 n +1
; x Î[2 n +1,2 n + 2)
è 8 øè 8 øè 8 ø ïî x

Then, which of the following statements is(are) which of the following statements is(are) TRUE?
TRUE? (a) The function f(x ) will be even
1 (b) The number of integral points, where f( x) is continuous
(a) The maximum value of f(x ), "x Î R is are two
4
(b) Number of principle solution of f(x ) = 0 is 8 (c) The function f( x) is increasing in (1, ¥)
1 3 53
(c) f(0) = (d) The value of ò f( x) dx is
8 0 6
1
(d) The maximum value of f(x ), "x Î R is 41. Normals are drawn from points (4, 1) to the
8
parabola y 2 = 4 x. The tangents at the feet of normals
38. Consider two circles S1 and S2 externally touching to the parabola y 2 = 4 x from a DABC. Then, which
having centres at points A and B whose radii are 1 of the following statements is(are) TRUE?
and 2 respectively. A tangent to circle S1 from point
5
B intersects the circle S1 at point C. D is chosen on (a) The area of DABC is sq units
2
circle S2 , so that AC is parallel to BD and the two
(b) The distance of focus of parabola y2 = 4 x from
segments BC and AD do not intersect. Segment AD
10
intersect the circle S1 at E. The line through B and E circumcentre of DABC is units
intersects the circle S1 at another point F. Then, 2
which of the following statements is (are) TRUE? (c) The distance of focus of parabola y2 = 4 x from centroid
5
(a) The length of segment BC is 2 2 units of DABC is units
3
(b) The length of segment EF is 2 2 units
(d) The distance of focus of parabola y2 = 4 x from
(c) The length of segment DE is 2 units
(d) The area of DABD is 2 2 sq units orthocentre of DABC is 5 units

39. Let M n be the n ´ n matrix with entries as follows 42. Let f (x)= ax 2 + bx + c , a >0 and f (2 - x) = f (2 + x),
for 1£ i £ n , m i , i = 10 for 1 £ i £ n -1, "x ÎR and f (x)=0 are non-real complex. Then,
m i +1 = m i ,i +1 = 3, all other entries in M n are zero. which of the following statements is(are) false?
px
Let D n be the determinant of matrix M n , then (a) f(x ) = sin must have two solutions
which of the following is(are) TRUE? 4
(b) If log f( 2 ) f(3) is not defined, then f(x )³1, "x ÎR
(a) The value of Dn = 10Dn -1 - 9Dn - 2 for n ³ 3
(c) 4a - 2 b + c < 0
(b) The value of Dn = 9Dn -1 + 1 for n ³ 2 (d) All, a, b,c are positive

PREP CATALYSIS
JEE Advanced~Practice Set 14 311

log (p+ e)[ax 4 + (7 a - 2 b)x 3 + (12a -


Section 2 (Maximum Marks : 24) 14 b - c)x 2 - (24 b + 7 c)x +1 -12c ]
Instructions : Same as given in Physics f1 (x) =
a - sgn(1 + ac + b2)
43. Let P(x) be a non-zero polynomial such that
(x -1) P(x +1) = (x + 2)P(x) for every real x, and f2 (x) = -2 + 2 log 2
cos(tan -1
æ7 ö m
(P(2))2 = P(3) Then, P ç ÷ = , where m and n are 7
è2ø n (sin(p(cos(p(x + ))))))
2
relatively prime positive integer. Then, m + n is
equal to …… List I List II
¥ p Domain of f1( x ) 1 [-3,-2 ]
æxö
44. Let f (x) = Õcos çè 2k ÷ø. For x ÎR, let [ x] be denotes q Range of f2( x ) in the 2 [-4,-2 ]
k =1
p/ 4 domain of f1( x ) is
the greatest integer function. If ò xf (x)dx = L,
0 r Range of f2( x ) is 3 ( -¥, ¥)
then the value [L] is ……
r r r r r s Domain of f2( x ) is 4 ( -¥,-4] È [-3, ¥)
45. If b and c are orthogonal unit vector and b ´ c = a,
r r r r r r r The correct option is
then the value of [a + b + c a + b b + c] is ……
(a) P®4, Q ®1, R®3, S®2
46. If ( 3 +1)6 + ( 3 -1)6 = 416, if xyz = [( 3 +1)6], x, y, z Î (b) P®4, Q ®1, R®2, S®3
N, (where [×] denotes the greatest integer (c) P®1, Q ®4, R®2, S®3
function), then the number of ordered triplets (d) P®4, Q ®2, R®3, S®1
(x , y , z) is …… .
52. Consider the letters of the word MATHEMATICS,
47. Let f (x) be a real valued function not identically set of repeating letters = {M,A,T}, set of the
zero, such that f (x + y n ) = f (x) + f (y)n ; "x , y ÎR non-repeating letters = {H,E,I,C,S}
(where, n is odd natural number greater than one), List I List II
then the value of f (5) + f ¢ (5) is …… .
P The number of words taking all 1 28(7 !)
48. Let N be the number of ordered pairs of non-empty letters of the given word such that
sets A and B that have the following properties. atleast one repeating letter is at odd
position is
(1) A ÈB = {1,2 ,3 ,4 ,5 ,6 ,7 ,8 ,9 ,10 ,1112
, }
Q The number of words formed taking 2 (11)!
(2) A ÇB = f all letters of the given word in which (2 !)3
(3) The number of elements of A is not an element not two vowels are together is
of A R The number of words formed taking 3 210(7 !)
(4) The number of elements of B is not an element of all letters of the given word such that
B, then the value of N is equal to …… . in each word both M’s are together
and both T’s are together but both
dy A’s are not together is
49. If = xy + x 3 y 3 and y(0) =1, then the value of
dx S The number of words formed taking 4 840(7 !)
2
æ æ1öö all letters of the given word such that
3 ç y ç ÷ ÷ is ……… . relative order of vowels and
è è2øø consonants does not change is
50. If y = mx + c is a tangent to hyperbola 5 ( 4 !)(7 !)
2 2 (2 !)3
x y
- = 1, then the least value of ( 6 !)(7 !)
l 2
(l + l2 + l)2
3 6
(2 !)3
16 m 2 is equal to ……… .
The correct options are
Section 3 (Maximum Marks : 12) (a) P®4, Q ®2, R®4, S®1
Instructions : Same as given in Physics (b) P®4, Q ®2, R®1, S®5
(c) P®2, Q ®4, R®4, S®1
51. Consider ax 4 + (7a - 2b)x 3 + (12a - 14 b - c)x 2
(d) P®2, Q ®3, R®1, S®5
- (24 b + 7 c)x +1 -12c = 0 has no real roots and

PREP CATALYSIS
312 JEE Advanced~Practice Set 14

53. The circle x 2 + y 2 = 25 intersects X-axis at points A cx + ay + bz =0


and B. The line x =11 intersect X-axis, at point C. and match the following.
Point P moves along the line x =11 above the X-axis List I List II
and AP intersect the circle at Q.
P a + b + c ¹ 0 and 1 The equations
List I List II a2 + b 2 + c2 = represent planes
ab + bc + ca meeting only at a
P The Coordinates of point P if the 1 (11, 0)
single point
DAQB has the maximum area is
Q a + b + c = 0 and 2 The equations
Q The Coordinate of point P if Q is the 2 (11, 8) a2 + b 2 + c 2 + represents the line
middle point of AP is ab + bc + ca x=y=z
R The Coordinate of P if the area 3 (11, 12) R a + b + c ¹ 0 and 3 The equations
th
ofDAQB is æç ö÷ of the area of
1 a2 + b 2 + c2 ¹ represents identical
è 4ø ab + bc + ca planes
DAPC is S a + b + c = 0 and 4 The equation
S The Coordinate of P if (AP - BP) is 4 (11, 16) a2 + b 2 + c2 = represents the whole
ab + bc + ca of the three
The correct options are dimensional space
(a) P®4, Q ®3, R®2, S®1 (b) P®3, Q ®2, R®1, S®4
(c) P®4, Q ®2, R®3, S®1 (d) P®4, Q ®3, R®1, S®4 The correct option is
(a) P®1, Q ®2, R®3, S®4
54. Consider the following linear equation. (b) P®3, Q ®2, R®1, S®4
(c) P®3, Q ®2, R®4, S®1
ax + by + cz =0
(d) P®2, Q ®4, R®3, S®1
bx + cy + az =0

Answers
Paper 1
1. a, b, d 2. b, c 3. a, b, c 4. a, c, d 5. a, c 6. a,b,c,d 7. 2.00 8. 6.00 9. 150.00 10. 3.00
11. 6.00 12. 7.00 13. 4.00 14. 5.00 15. c 16. b 17. b 18. d 19. a 20. a, b, d
21. a, c, d 22. 6 23. 0.79 24. 23.39 25. 110.73 26. c 27. b 28. a, b, d 29. c 30. c
31. 56.24 32. 10.31 33. 1.46 34. 23.78 35. b 36. d 37. a, b, c 38. a, b, d 39. b, d 40. a, b, c
41. a, c, d 42. b, c 43. a 44. 1 45. 2018 46. 30 47. 21 48. 19 49. 39 50. 3
51. 9 52. c 53. b 54. d

Paper 2
1. a, c 2. a, c, d 3. a,b,c,d 4. a, c 5. a, b 6. a, b, c 7. 61.25 8. 0.05 9. 1.40 10. 0.10
11. 10.00 12. 2.50 13. 5.30 14. 2.30 15. a 16. d 17. b 18. c 19. a, c 20. c
21. a,b,c,d 22. 12 23. 0.73 24. 80.76 25. –30.6 26. c 27. b 28. a, b, d 29. c 30. c
31. 56.24 32. 10.31 33. 1.46 34. 23.78 35. b 36. d 37. b, c, d 38. a, c, d 39. a, b, d 40. b, c, d
41. a,b,c,d 42. a, c, d 43. 109 44. 0 45. 1 46. 9 47. 6 48. 772 49. 4 50. a
51. b 52. d 53. c 54. b

SCORE SHEET - Paper 1


Section No. of Marks from Marks from Marks Obtained
Correct Questions Correct Questions (A) Incorrect Questions (B) (A-B)
................ ............................... ............................... ............................... ...............................
................ ............................... ............................... ............................... ...............................
................ ............................... ............................... ............................... ...............................
Percentage Marks = Marks Obtain/Total Marks x 100

SCORE SHEET - Paper 2


Section No. of Marks from Marks from Marks Obtained
Correct Questions Correct Questions (A) Incorrect Questions (B) (A-B)
................ ............................... ............................... ............................... ...............................
................ ............................... ............................... ............................... ...............................
................ ............................... ............................... ............................... ...............................
Percentage Marks = Marks Obtain/Total Marks x 100
Note To expect your success marks in the test should be between 65%-70%.
PREP CATALYSIS
JEE Advanced

PRACTICE SET 15 (With Solutions)

Duration : 3 Hours Max. Marks . 360

Paper 1
PHYSICS
Section 1 (Maximum Marks : 28)
l
This section contains SEVEN questions.
l
Each question has FOUR options (a), (b), (c) and (d). ONE OR MORE THAN ONE of these four options is (are) correct.
l
For each question, darken the bubble(s) corresponding to all the correct option(s) in the ORS.
l
For each question, marks will be awarded in one of the following categories:
Full Marks : + 4 If only the bubble(s) corresponding to all the correct option(s)
is (are) darkened.
Partial Marks : + 1 For darkening a bubble corresponding to each correct option, providedNO
incorrect option is darkened
Zero Marks : 0 If none of the bubbles is darkened.
Negative Marks : - 2 In all other cases.
l
For example, if [a], [c] and [d] are all the correct options for a question, darkening all these three will get +4 marks;
darkening only [a] and [d] will get +2 marks; and darkening [a] and [b] will get-2 marks, as a wrong option is also
darkened

1. A uniform with edge ‘a’ rests on horizontal plane (d) Shortest distance between line of normal force exerted
whose friction coefficient equal to m. The cube is by plane and gravitation force is x = a /2
set in motion with an initial velocity v. It travels 2. When one of the surface of a lens is silvered, it
some distance over the plane and comes stand still. behaves like a mirror. Now, consider a lens with
(a) Angular momentum of cube is not conserved relative to radius of curved surface =R and refractive index of
the axis lying in the plane at right angles to the cube’s medium of lens = m.
motion direction Now, choose the correct options.
(b) Angular momentum is always zero about the axis
(a) When plane surface of a plano-convex lens is silvered,
passing through COM of cube at right angles to the R
cube’s motion focal length of mirror formed is f =
2 ( m - 1)
(c) Net torque is zero about the axis passing through COM
of cube at right angles to the cube’s motion (b) If spherical surface of plano-convex lens is silvered, it
behaves as a convex mirror

PREP CATALYSIS
314 JEE Advanced~Practice Set 15

(c) If plane surface of a plano-concave lens is silvered, it 5. Choose the correct options.
R
behaves as a convex mirror of focal length (a) de-Broglie wavelength association with the orbital
2 ( m - 1) electron in the nth orbit of H-atom is 3.3 n (Å)
(d) If spherical surface of plano-concave lens is silvered, it (b) If a material particle (mass m0 ) is moving with velocity(v )
R
behaves as a convex mirror of focal length f = comparable to the velocity of light (c), then de-Broglie
2m wavelength associated with
3. A particle is moving along a parabola y = x 2, so v2
h 1-
c2
that its velocity v x = 3 ms - 1 remains constant with particle is l =
m0 v
time.
(c) de-Broglie wavelength is independent of charge on the
Now, choose the correct options. particle
2
(a) At x = m, velocity of particle is 5 ms - 1 (d) de-Broglie wavelength of an electron in n = 4 orbit of the
3 Bohr modal of H-atom is nearly 1.33 nm
2
(b) At x = m, acceleration of the particle is 18 ms - 2
3 6. The mathematical statement, v = vc + v, where vc
2 is the velocity of centre of mass, v¢ is the velocity of
(c) At x = m, velocity of the particle is 18 ms - 1
3 the point with respect to the centre of mass and v
2
(d) At x = m, acceleration of the particle is 5ms - 2 is the total velocity of the point with respect to
3 ground
4. The vessel shown in the figure has two sections of (a) is true for a rolling sphere.
areas of cross-section A1 and A 2 . A liquid of (b) is true for a block moving on frictionless horizontal
density r fills both the sections, up to a height h in surface.
each. Neglect atmospheric pressure (c) is true for a rolling cylinder.
(d) None of the above
A1
h 7. A particle of mass m and charge q is projected
X vertically upwards. A uniform electric field E is
A
applied vertically downwards. If the total potential
h energy is U (gravitational plus electrostatic) and
height is h (<< radius of earth) (Assume, U to be
ρ zero on earth’s surface), then
(a) the pressure at the base of vessel is 2hrg (a) work done by the gravity mgh
(b) the force exerted by the liquid on the base of vessel is (b) work done by electrostatic force qEh
2 hrgA2 (c) graph between potential energy versus height (h) is
(c) the weight of liquid is <2 hrgA2 straight line
(d) The walls of the vessel at the level X exert downward (d) graph of potential energy versus height (h) is parabola
force hrg ( A2 - A1 ) on the liquid

Section 2 (Maximum Marks : 15)


l
This section contains FIVE questions
l
The answer to each question is a SINGLE DIGIT INTEGER ranging from 0 to 9, both inclusive.
l
For each question, darken the bubble corresponding to the correct integer in the ORS.
l
For each question, marks will be awarded in one of the following categories:
Full Marks : + 3 If only the bubble corresponding to all the correct answer is darkened.
Zero Marks : 0 In all other cases.

8. The displacement y of a particle executing a certain 9. 600 J of heat is added to a monoatomic gas in a
ö2æ1
process in which the gas performs a work of 150 J.
periodic motion is given by y = 4 cos ç t ÷ sin (1000 The molar heat capacity for the process is ……… R.
è2 ø
t). This expression may be considered to be the 10. First excited state of hydrogen atom is 10.2 eV
superposition of n independent harmonic motions. above its ground state. Let the temperature at
Then, n is equal to which hydrogen atom will be excited to this state
is T ´ 10 4 K. Then, value of T is near to integer.
(Take, k B = 1.38 ×10 -23 J/K)

PREP CATALYSIS
JEE Advanced~Practice Set 15 315

11. Two containers A and B are connected by a 22


radiation loss. (Take, R = 8 .3 J/mol-K and p = ).
conducting solid cylindrical rod of length 7
242 A B
cm and radius 8 .3 cm. Thermal conductivity
7 O2 He
of the rod is 693 W/mol-K. The container A
contains two mole of oxygen gas and the container 12. The equation of a transverse wave is given by
B contains four mole of helium gas. At time, t = 0 y = 10 -2 sin p (30 t - x 3 - y), where x , y and y are
temperature difference of the containers is 50°C. in metre and t in second. If phase difference
After, what time (in second) temperature between two points A (2 3 m , 2 m) and
difference between them will be 25°C. Transfer of
B(3 3 m , 3 m) be np. Then, find the value of n.
heat takes place through the rod only. Neglect

Section 3 (Maximum Marks : 18)


l This section contains SIX questions of matching type.
l The section contains TWO tables (each having 3 columns and 4 rows)
l Based on each table, there are THREE questions.
l Each question has FOUR options [a], [b], [c] and [d]. ONLY ONE of these four options is correct.
l For each question, darken the bubble corresponding to the correct option in the ORS.
l For each question, Marks will be awarded in one of the following categories:
Full Marks : + 3 If only the bubble corresponding to the correct option isdarkened
Zero Marks : 0 If none of the bubbles is darkened
Negative Marks : -1 In all other cases

k2 Direction (Q. Nos. 16 to 18) Consider an ideal gas.


Direction (Q. Nos. 13 to 15) In Column I, value of 2
are
R Column I Column II Column III
given in Column II, rotational kinetic energy is given in A. p I. p P. V
Column III, total kinetic energy is given.
v is velocity of the rolling body, k is radius of gyration and
R is radius of cross-section of rolling body of mass (m ) .
V T T
Column I Column II Column III
A. k2 I. 1 P. 7 B. p II. p Q.
=1 mv 2 mv 2 V

R 2 3 10
B. k2 1 II. 1 Q. 3
= mv 2 mv 2
R 2
2 5 4
T
1 V T
C. k2 2 III. mv 2 R. mv 2
=
R2 5 4
C. p III. p R. V
D. k2 2 IV. 1 S. 5
= mv 2 mv 2
R2 3 2 6

13. Object is either a hollow cylinder or a circular loop. T


V T
(a) C — I — P (b) B — III — Q
(c) A — IV — R (d) D — II — S D. p IV. p S. p

14. Rotational kinetic energy is nearly 33.3% of total


energy.
(a) B — III — Q (b) A — II — P T
V T
(c) C — I — R (d) D — IV — S
15. Translational energy of body is nearly 72% of its’
16. In this process, pressure remains constant.
total energy.
(a) B — III — Q (b) C — IV — S
(a) A — III — S (b) C — II — P
(c) A — I — R (d) D — II — P
(c) B — I — Q (d) D — IV — R

PREP CATALYSIS
316 JEE Advanced~Practice Set 15

17. In this process, temperature is always constant 18. No heat exchange from surroundings.
(a) B — III — Q (b) A — I — R (a) B — III — Q (b) A — I — R
(c) C — IV — S (d) D — II — P (c) C — IV — S (d) D — II — P

CHEMISTRY
23. After completion of the reactions (I and II), the
Section 1 (Maximum Marks : 28) organic compound(s) in the reaction mixtures is/are
Instructions : Same as given in Physics
O
Br2(1.0 mol)
19. Two moles of a monoatomic ideal gas (V =15R
. )
Reaction I Aq. NaOH
initially at 400K in an isolated, 1.0 L, piston is H3 C CH3
allowed to expand against a constant pressure of a O
Br2(1.0 mol)
1.0 atm till the final volume reaches to 10 L. Which
Reaction II CH3COOH
of the following conclusions regarding the above H3 C CH3
change(s) is/are true? O O O
(a) The final temperature of the gas is 363K
(b) If the same process were carried out to the same final H3 C CH2Br H3C CBr2 Br2C CBr2
volume but under reversible conditions, final
(P) (Q) (R)
temperature would have been less than 363K
(c) In the above process, the initial and final temperatures and O O
r -1
æT ö æV ö –+
volumes are related as ç 1 ÷ = ç 2 ÷ BrH2C CH2Br H3C ONa CHBr3
è T2 ø è V1 ø
(S) (T ) (U)
(d) Entropy change of a system (DS system) is zero.
(a) Reaction I : P and reaction II : P
20. BeCl2 exist in vapour as well as in solid phase, (b) Reaction I : U, acetone and reaction II : Q, acetone
choose the correct option for solid phase BeCl2 (c) Reaction I : T, U, acetone and reaction II : P
(a) BeCl 2 has zero dipole moment (d) Reaction I : R, acetone and reaction II : S, acetone
(b) lone pair present on Cl atoms are used for bonding with 24. A solution of non-volatile solute in ethanol
vacant orbitals of the Be atom (boiling point of ethanol = 78.4°C) has a vapour
(c) it exists in dimer form of BeCl 2 pressure of 730 mm of Hg at 78.4°C. The
(d) it is functional as a Lewis acid temperature to which the above solution must be
21. Which of the following reagents can be used for heated to have the vapour pressure of 760 mm of
the following conversion? Hg is …… .
O [Given : K b of ethanol = 1.22 K kg/mol]
(a) 79.5°C (b) 81°C (c) 88°C (d) 100°C
C CH CH3
25. Which of the following is the correct statement?
(a) (i) O3 /Red P (ii) AlCl 3 (iii) MeCOOH (a) Fluorine is the strongest oxidising agent
(b) (i) H2 SO4 + HgSO4 (ii) H2O, heat (b) If negative valencies of a species increase after reaction
(c) (i) O3 /Zn¾ AcOH (ii) H2 SO4 + HgSO4 it is known as reduction process
(iii) H2O, heat (c) Conversion of glucose into CO2 is an exothermic
-
(d) (i) CH3 COOH (ii) H2O2 + OH / H2O process
(d) Hypo prefix in hypophosphorus acid indicates it will act
22. ‘X’ reacts with aqueous NaOH solution to form ‘Y’ as an oxidising agent
and H 2. Aqueous solution of ‘Y’ is heated to
323-333K and on passing CO2 into it, Na 2CO3 and Section 2 (Maximum Marks : 15)
Z were formed. When Z is heated to 1200°C, Al2O3 Instructions : Same as given in Physics
is formed. X , Y and Z respectively are
26. Consider the following transition metal complex
(a) Al, AlCl 3 , NaAlO2
compound. [Fe(CN)6 ]3 - , [Fe(CN)6 ]4- ,
(b) Zn, Na 2 ZnO2 , Al(OH)3
(c) Al, Al(OH)3 ,AlCl 3 [Co(NH3 )6 ]3 + , NiCl3 (PMe3 )2, [Co(CN)6 ]3 -
(d) Al,NaAlO2 ,Al(OH)3 How many of them show diamagnetic behaviour?

PREP CATALYSIS
JEE Advanced~Practice Set 15 317

27. When 1 mole of Cr2O3 reacts with 2 moles of Al 32. Which of the following combination represents
metal, 1 mole of Al2O3 and 2 moles of Cr metal are orthorhombic structure?
produced. Find the number of grams of reactants (a) C I Q (b) A I R (c) B II Q (d) D III P
in nearest integer that would be left unreacted
33. Which of the following does not represented by
when 4.98 g of Al is made to react with 20.0 g of
KCl, BaSO 4 , CaCO 3 ?
Cr2O3 .
(a) A II R (b) C I Q (c) D II P (d) B III S
28. The total number of cyclic isomers possible for a Match the following.
hydrocarbon with the molecular formula C4H 6 is.
(Directions (Q.Nos 34-36)
29. Total number of salts that are isostructural …… .
Column I Column II Column III
FeSO4 ×7 H 2O, CuSO4 ×5H 2O, MnSO4 ×4 H 2O, +
A Asn I P 5.7
ZnSO4 ×7 H 2O O NH3

30. 19K (t1 / 2 =13
40
. ´10 9 year) consists of 0.012% potassium OCCH2 CHCO2–
B Asp II + Q 5.4
is nature. The human body contains 0.35% OH NH3
potassium by weight. The total radioactivity
resulting from, 19K 40 decay in a 75 kg human body CH3 CH CHCO2–
+
is 4 .81 ´10 x d/pm. The value x is. C Tyr III
O NH3
R 5.6

H2NCCH2 CH CO2
Section 3 (Maximum Marks : 18) D Thr IV + S 2.8
Instructions : Same as given in Physics NH3

(Direction (Q.Nos 31-33) By appropriately matching HO CH2 CHCO2–


the information given in the three columns of following
table. Column I, II and III contains intercept, crystal
34. Which of the following combination is correct for
angles and examples respectively.
tyrosine amino acid?
Column I Column II Column III (a) D IV R (b) C II P
(c) D II R (d) D I Q
A. a= b=c I a = b = g = 90° P. Diamond
B. a¹ b=c II a = b = g = 90° Q. Rhombic sulphur 35. Which of the following combination is correct for
acidic amino acid?
C. a ¹ b ¹ c III a = b = g ¹ 90° R. HgS
(a) C IV R (b) A I S
D. a = b = c IV a = g = 90° b ¹ 90° S. Monoclinic (c) B I S (d) B I Q
sulphur
36. Which of the following combination is incorrect
31. Which of the following combination is correct for for neutral amino acid?
NaNO3 ?
(a) B I S (b) C II R
(a) D II Q (b) A I R (c) A I S (d) B I P
(c) A III R (d) None of these

MATHEMATICS
Section 1 (Maximum Marks : 28) 38. If one root of the equation ea x 2 - e2a x + ea - 1 = 0
Instructions : Same as given in Physics lies between 1 and 2, then which of the statements
37. If the normal y = mx - 2am - am 3 to the parabola is/are correct?
(a) 2e 2 a - 5e a + 1 < 0
y 2 = 4 ax subtends a right angle at the vertex, then
(b) e 2 a - 5aa + 1 < 0
m equals
æ 5 - 17 ö æ 5 + 17 ö
(a) - 2 (b) 3 (c) logç ÷ < a < logç ÷
(c) 2 (d) - 3 è 4 ø è 4 ø
(d) (e a - 1)2 > 0

PREP CATALYSIS
318 JEE Advanced~Practice Set 15

39. If a function f (x) satisfying the differential (a) g (x ) is decreasing in (- ¥, 3)


dy sin x (b) g (x ) is increasing in (3, ¥)
equation - y cot x + 2 = 0, is such that y ® 0 (c) g (x ) is decreasing in (3, ¥)
dx x (d) g (x ) is increasing in (- ¥, 3)
as x ® ¥, then
p/2
(a) ò f(x ) dx > 1 (b) f(x ) is an even function Section 2 (Maximum Marks : 15)
0
p/2 p Instructions : Same as given in Physics
(c) lim f(x ) = 1] (d) ò f(x ) dx <
x® 0 0 2 44. In a multiple choice question, there are five
40. If a = tan(a + b) - tan(a - b), b = tan(a + b) alternative answers, of which one or more than
one option is correct. A candidate will get marks
tan(a - b), then
on the question, if he ticks all the correct answers.
2 sin 2b
(a) a = If he decides to tick answers at random, then
cos 2 a + cos 2b the chances should be allowed so that the
p 1+ a + b
(b) tanæç + 2b ö÷ =
probability of his getting marks on the question
è4 ø 1- a + b 1
exceeds , is … .
sin 2 a 8
(c) a =
cos 2 a - cos 2 b
45. In a plane, there are two families of lines y = x + r,
sin2 a + cos 2 b - 1
(d) b = y = - x + r, where rÎ{0 , 1, 2 , 3 , 4 }. The number of the
cos 2 a + cos 2 b - 1 squares of the diagonal of length 2 formed by these
lines is
41. Let a, b and c be three non-coplanar vectors and d
be a non-zero vector, which is perpendicular to é pù
46. If f (x) = sin x, "x Î ê0 , ú, f (x) + f (p - x) = 2,
(a + b + c). If d = (a ´ b)sin x +(b ´ c)cos y + 2(c ´ a), ë 2û
then
æp ù
d × (a + c) "x Î ç , p ú and f (x) = f (2p - x), "x Î (p , 2 p), then the
(a) =2 è2 û
[a bc]
d × (a + c) area enclosed by y = f (x) and the X-axis is
(b) = -2
[a bc] 47. If the chord, x cos a + y sin a = r of the hyperbola
p2 x 2 y2
(c) minimum value of x + y is2 2
4 - =1, subtends a right angle at the centre,
16 18
5p 2
(d) minimum value of x 2 + y2 is and the diameter of circle, concentric with the
4
hyperbola, to which the given chord is a tangent is
42. If the focus, centre and eccentricity of an ellipse d, then the value of d is
are respectively (3, 4), (2, 3) and 1/2, then its 48. Let OA = a, OB =10a + 2b and OC = b, where O A
equation is given by
and C are non-collinear points. Let p denotes the
(x + y - 5)2 (x - y + 1)2
(a) + =1 area of quadrilateral OACB, and let q denote the
16 12 area of parallelogram with OA and OC as adjacent
(x - y + 1)2 (x + y - 5)2 sides. If p = kq, then k is equal to
(b) + =1
16 12
(c) (x - 3)2 + ( y - 4)2 + (x - 2 )2 + ( y - 3)2 = 4 2
Section 3 (Maximum Marks : 18)
(d) (x - 3)2 + ( y - 4)2 + (x - 1)2 + ( y - 2 )2 = 4 2 Instructions : Same as given in Physics
43. In a question, a student was given to find the Direction (Q.Nos. 49, 50 and 51) by appropriately
derivative of the product of two functions f and g. matching the information given in the three columns of
The student by mistake thought (fg)¢ = f ¢ g ¢ for his the following table.
question f (x) = x 3 and he got the correct answer. If Column I, II and III contains f ( x ), f ¢ ( x ) and ò f ( x ) dx
g(4) = 1, then which of the following is/are correct respectively,
possibilities (where, g (x) > 0)?

PREP CATALYSIS
JEE Advanced~Practice Set 15 319

Column-I Column-II Column-III The expansion of (1 + x )n = n C 0 + n C 1 x + n C 2 x 2 +¼+


A 1 I x 2(2 a 2 - x 2 ) P 1 x
sec -1 + C
n
C r x r +¼+ n C n x n where, n c r is binomial coefficient of
(a + x )
2 2 3/ 2
( a 2 - x 2 )3/ 2 a a r
x .
B x2 II -(2 x 2 - a 2 ) Q ésin-1 x - ù Column I Contains some series of binomial coefficient
x ( x 2 - a 2 )3/ 2
2 a2 ê a ú Column II Contains rth terms of the series of binomial
a -x 2 2
ê ú
2 ê x a2 - x2 ú coefficient
ëa 2
û Column III Contains sum of series of binomial
+C coefficient
C 1 III -3 x R -x
+C Column-I Column-II Column-III
( x 2 - a 2 )3/ 2 ( a 2 + x 2 )5/ 2 a2 x2 - a2
-3 x I C 0 + 2C1 + 3C 2 +¼ i n
C r -1 P 2n -1
D 1 IV S x
+C 3r
r n+ 1
x x2 - a2 ( x 2 - a 2 )5/ 2 a2 x2 + a2
II C1 C 3 C 5
+ + +¼ ii r( r + 1)nC r Q n( n + 3)2 n - 2
x
49. For a =1, if ò f (x)dx = + C, then which of the
2 4 6
x 2 +1 III 1 ´ 2C1 + 2 ´ 3C 2 + 3 iii r nC r -1 R 2 2n+ 2 - 1
following combination is correctly matched? ´ 4C 3 +¼ n+ 1
(a) A III S (b) B I Q IV 3C + 3 2 C1 + 3 3 C 2 ¼ iv n
C 2r -1 S n × 2 n-1
0
(c) A IV S (d) B III S 2 3 2r
-2(x 2 - 2) 31
50. For a =2, if f ¢ (x) = , then which of the 52. For n =5, the sum of the binomial coefficient is ,
x 2 (x 2 - 4)3/ 2 6
following is correctly matched? then which of the following option is the only
(a) C I R (b) D II P correct combination.
(c) B III Q (d) D II R (a) (II) (iv) P (b) (IV) (i) P
x 2 (c) (IV) (i) R (d) (III) (ii) Q
51. If f (x)= , which of the following is correctly
1- x 2 53. For n =6, the 4th term of series of binomial
matched? coefficient is 300, then which of the following
option is the only correct combination?
(a) A III R
(b) B II Q (a) (I) (iii) S (b) (III) (ii) S
(c) D IV P (c) (I) (ii) Q (d) (III) (ii) Q
(d) B I Q
54. The ratio of the sum of first 4 terms to the sum of
Direction (Q.Nos. 52, 53 and 54) by appropriately next four terms is 1 : 31, then which of the
matching the information given in the three columns of following option is the only correct combination?
the following table. (a) (I) (iii) (S) (b) (II) (iv) P
(c) (III) (ii) Q (d) (iv) (i) R

PREP CATALYSIS
Paper 2
PHYSICS
Section 1 (Maximum Marks : 21)
l This section contains SEVEN questions.
l Each question has FOUR options (a), (b), (c) and (d). ONLY ONE of these four options is correct.
l For each question, darken the bubble corresponding to the correct option in the ORS.
l For each question, marks will be awarded in one of the following categories.
Full Marks : + 3 If only the bubble corresponding to the correct option is darkened.
Zero Marks : 0 If none of the bubbles is darkened.
Negative Marks : - 1 In all other cases

1. An isoceles glass prism stands with its (horizontal) 3. A ball of mass m and radius r falls through a
base in water as shown in the figure. An incident viscous fluid of coefficient of viscosity h.
ray of light, above and parallel to the liquid surface Rate of dissipation of its energy after it reaches
and perpendicular to the prism’s axis is internally terminal speed is
reflected at the glass-liquid interface and m2 g 2 2 mg 2 2 m2 g 2 mg 2
(a) (b) (c) (d)
subsequently re-emerges into the air. Refractive 6phR 3phR 3phR 6phR
index of glass is m g and liquid is m l , then which
relation holds good 4. A parallel plate capacitor is partially filled with a
dielectric (k = 5), then correct options are
(a) electric field inside dielectric is 20% less than field in air
gap
(b) electric field in dielectric is 10% less than the field in air
θ θ gap
(c) bound charge density on surface of dielectric is 30%
Liquid
less than the charge density of capacitor plates
(a) m g2 - m l2 ³ cos 2
q ( m g2 + 1 - 2 ml ) (d) bound charge density is 20% more on the dielectric
(b) m g = m l cos q (c) m 2g = m 2l cos q surface

(d) m 2g - m 2l < cos 2 q(m 2g + 1 - 2m l ) 5. A particle slides back and fourth between two
inclined frictionless planes joined smoothly at the
2. A disk is released from a height h over the ramp, bottom. Initial height of particle is h.
which is just sufficient to successfully complete
the loop (neglect friction).
h
θ θ
Correct option is
h (a) motion is oscillatory but not simple harmonic with time
1 h
period, T =
2p g
C 1 2h
(b) motion is simple harmonic with time period,T=
Then, 2p g
(a) velocity at C is 2gR 2h
(c) motion is simple harmonic with time period, T = 4
(b) velocity at C is 3gR g

5 (d) motion is oscillatory but not simple harmonic with time


(c) velocity at C is gR
2 2h
period, T = 4
(d) velocity of C is 5gR g

PREP CATALYSIS
JEE Advanced~Practice Set 15 321

6. A neutron having kinetic energy 15.5 eV collides 7. A racing car driver drives his car on a flat circular
with a hydrogen atom at rest. Assuming the track of radius 25/3 m and coefficient of friction
neutron does not leave the line of motion, the 0.5. The magnitude of its tangential acceleration at
possible kinetic energy of H-atom after collision is an instant, when car starts slipping at a speed of
(Take, m H = m N) 5 m/s is
(a) 0 (b) 15.5 eV (a) 2 m/s 2 (b) 3 m/s 2
(c) 10.2 eV (d) 13.6 eV (c) 4 m/s 2
(d) 1 m/s 2

Section 2 (Maximum Marks : 28)


l This section contains SEVEN questions.
l Each question has FOUR options (a), (b) (c) and (d). ONE OR MORE THAN ONE of these four option(s) is (are)
correct.
l For each question, darken the bubble(s) corresponding to all the correct option(s) in the ORS.
l For each questions, marks will be awarded in one of the following categories :
Full Marks : + 4 If only the bubble(s) corresponding to all the correct option(s) is (are)
darkened.
Partial Marks : + 1 For darkening a bubble corresponding to each correct option, provided NO
incorrect option is darkened.
Zero Marks : 0 If none of the bubbles is darkened.
Negative Marks : - 2 In all other cases.
For example, if (a), (c) and (d) are all the correct options for a question, darkening all these three will get in + 4 marks ;
darkening only (a) and (d) will get + 2 marks and darkening (a) and (b) will result in - 2 marks, as a wrong option is
also darkened.

8. Average lifetime of an excited state of hydrogen is ratio of wavelength, momentum and energy of
-8 photons emitted in these two cases be a , b and c,
of the order of 10 s.
Option out correct ones. respectively. Then,
1 27 5 5
(a) When electron is in n= 2 state, number of rounds an (a) c = (b) a = (c) b = (d) c =
electron can make is about 82 . ´ 106 , before transition a 5 27 27
to n = 1state 11. A point object is placed at 30 cm from a convex
(b) Number of rounds an electron can make in n = 15 state æ 3ö
. ´ 104 , before transition to n= 1state.
is about 194 glass lens çm g = ÷ of focal length 20 cm. The final
è 2ø
(c) Electron does not revolve in an excited state, it only
image of object will be formed at infinity, if
oscillates about nucleus
(d) Speed of electrons in an excited state can reach upto (a) another concave lens of focal length 60 cm is placed in
velocity of light contact with the previous lens
(b) another convex lens of focal length 60 cm is placed at
9. A projectile (mass m) is fired from origin with the distance of 30 cm from the first lens
initial speed v 0 at an angle a with horizontal. (c) the whole system is immersed in a liquid of refractive
Now, choose the correct options. index 4/3
(a) Its x-component of angular momentum is zero. (d) the whole system is immersed in a liquid of refractive
(b) Its y-component of angular momentum is index 9/8
1 æ mg ö 2
- ç ÷ × t × cos a× $j 12. Two circular coils A and B with their centres lying
2 è v0 ø on the same number of turns and carry equal
(c) It’s z-component of angular momentum is currents in the same sense. They are separated by a
æ mg cos a ö 2 $ distance have different diameters but subtends
-ç ÷×t ×k
è 2v0 ø same angle at a point P lying on their common
(d) Rate of change of angular momentum of projectile is
axis. The coil B lies exactly midway between coil
dL æ mg cos a ö $ A and the point P. The magnetic field at point P
=- ç ÷t ×k
dt è v0 ø due to coils A and B is B1 and B 2 , respectively
(a) B1 > B2 (b) B1 < B2
10. An electron in hydrogen atom first jumps from B B 1
(c) 1 = 2 (d) 1 =
second excited state of first excited state and then B2 B2 2
from first excited state to ground state. Let the

PREP CATALYSIS
322 JEE Advanced~Practice Set 15

13. One mole of an ideal gas at initial pressure p1 and


R
initial volume V1 expends to volume V2 according
to pV n = constant.
Now, choose the correct options. S
(a) When n = 0, work done is W = p1(V2 - V1 ) E1 E2
æV ö
(b) When n = 1, work done is W = p1v1 log e ç 2 ÷ (a) the amount of heat produced in the resistor
è V1 ø
is 63000 J.
2
(c) When n = , work done is W = 3 p1V1 × (V2 - V11/ 3 )
1/ 3
(b) the amount of heat produced in the resistor
3
2 is 7000 J.
(d) When n = , work done is W = 0
3 (c) if total amount of heat produced is used to heat
3 kg of water at20°C, the final temperature will be25°C.
14. In the circuit shown in the figure R = 50 W,
(d) the value of direct current that will produce same
E1 = 25 3 V and E2 = 25 6 sin (wt) V, where
amount of heat in same time through same resistor will
w = 100 ps -1 . The switch is closed at t = 0 and be 15. A.
remains closed for 14 min, then it is opened

Section 3 (Maximum Marks : 12)


l
This section contains TWO paragraphs.
l Based on each paragraph, there are TWO questions.
l Each question has FOUR options (a), (b) (c) and (d). ONLY ONE of these four options is correct.
l
For each question, darken the bubble corresponding to all the correct option in the ORS.
l For each questions, marks will be awarded in one of the following categories.
Full Marks : + 3 If only the bubble corresponding to all the correct answer is darkened.
Zero Marks : 0 In all other cases.

Passage X
A muon is an unstable elementary particle whose mass is A B
207 me and whose charge is either +e or –e. A negative
micron (m - ) can be captured by a nucleus to form a Opaque
muonic atom. A proton captures a micron. Bohr radius of disc
the hydrogen atom is 5.29 ´ 10-11 micron and ionisation
energy of the hydrogen atom is 13.6 eV. C D
1
15. Find the kinetic energy of the atom in first excited r = (1 - kt ) m, where k = s -1
state. 4
(a) 2 .53 ´ 103 eV (b) 5.06 ´ 103 eV 17. The time, when the source is just visible from the
. ´ 10 eV
(c) 126 2
(d) 6.32 ´ 102 eV face ABCD is
4 8
16. Find the ionisation energy of the atom. (a) 1 s (b) s (c) s (d) None of these
3 5
(a) 2 .53 ´ 10 eV
3
(b) 5.06 ´ 10 eV
3
18. The source starts moving with the velocity v
. ´ 102 eV
(c) 126 (d) 6.32 ´ 102 eV
towards the centre of disc at time t = 1 second. If
Passage A the source is just visible during t < 4 s , then
A solid glass cube of edge length 2m and index of 5 1
(a) v = m/s (b) v = m/s
refraction 5/3 has a point source at its’ centre. An opaque 8 3
disc of radius 1m covers the face ABCD as shown in figure. 5
(c) v = m/s (d) None of these
The radius of disc starts contracting for 0 £ t £ 4 s as given 4

PREP CATALYSIS
CHEMISTRY
Section 1 (Maximum Marks : 21) (c) A= B=
Instructions : Same as given in Physics CH3 I HO OH
C= D=
19. The value of log 10 K for a reaction A =B is HOOC COOH HOOC COOH
[Given : D r H 298K = -54 .07 kJmol-1 ]
(d) A= B=
D r S°298 K = +10 Jmol-1 and R = 8 .31J K -1 mol-1 ; I I HO OH
2.303 ´ 8 .314 ´ 298 = 5705] C= D=
(a) 5 (b) 10 HOOC COOH HOOC COOH
(c) 95 (d) 100
23. Select the incorrect statement among the
20. Which of the following is the most acidic following.
compound?
(a) The first ionisation potential of Al is less than the first
COOH COOH ionisation potential of Mg
HO OH OH (b) The second ionisation potential of Mg is greater than the
(a) (b) second ionisation potential of Na
(c) The first ionisation potential of Na is less than the first
ionisation potential of Mg
COOH COOH
(d) The third ionisation potential of Mg is greater than third
ionisation potential of Na
(c) (d)
24. Two flasks A and B are joined by a stop clock of
OH negligible volume. Assuming that all the
OH operations were carried out at a uniform constant
21. In the reaction, 2Al(aq ) + 6 HCl(aq) temperature, volume per cent of helium in the
mixture is
¾® 2 Al3 + (aq) + 6 Cl- (aq)
+3H 2(g) 450 mL A B 250 mL
(a) 6 LHCl(aq ) is consumed for every 3LH2 (g ) produced.
(a) 22.7% (b) 77.3% (c) 40.2% (d) 59.8%
(b) 33.6 LH2 (g ) is produced regardless of temperature and
pressure of every mole Al that reacts. 25. Among the following observations, the correct
(c) 67.2 LH2 (g) at STP is produced for every mole Al that statement that differentiate SO2- 2-
3 and SO4 is
reacts.
(d) 11.2 L H2 (g) at STP is produced for every mole HCl(aq ) (a) Both form precipitate with BaCl 2 , SO2-
3 dissolves in HCl
consumed. but SO2-4 does not
(b) SO2- 2-
3 forms precipitate with BaCl 2 , SO4 does not
22. Complete the following reactions.
(c) SO2- 2-
4 forms precipitate with BaCl 2 , SO3 does not
Excess (i) Mg + ether
of HI
A
(ii) CO2/H3O+
B (d) Both form precipitate with BaCl 2 , SO2-
4 dissolves in HCl

O OHσ
but SO2-3 does not
(O)
C D
KMnO4/H
+
Section 2 (Maximum Marks : 28)
(a) A= B= Instructions : Same as given in Physics
CH3 I HOOC COOH
26. The correct statement(s) regarding, (i) HCIO, (ii)
D=HOOC CH CH COOH HClO2 (iii) HClO3 and (iv) HClO4 is/are
C=
HO OH (a) the number of Cl == O bonds in (ii) and (iii) together is two
(b) A= B= (b) the number of lone pairs of electrons on Cl in (ii) and (iii)
I I HOOC COOH together is three
C= D= (c) the hybridisation of Cl in (iv) is sp3
HO OH HOOC COOH (d) amongst (i) to (iv), the strongest acid is (i)

PREP CATALYSIS
324 JEE Advanced~Practice Set 15

27. Consider the following structure. (a) The de-Broglie wavelength in the third Bohr orbit of
H-atom = 6x
(b) The fourth Bohr’s radius of He+ ion = 8x
(c) The de-Broglie wavelength in third Bohr’s orbit of
Li 2+ = 2 x
(d) The second Bohr’s radius of Be2+ = x

31. The major product obtained in the following


reaction is
O
Anion = K2CO3
Cation = OH PhCH2Br
Which of the following statements is/are correct OH
about tetrahedral voids in the given unit cell? O O
(a) Number of tetrahedral voids unit cell is 2.
(b) Number of tetrahedral voids per unit cell is 8. (a) OCH2Ph (b) OH
(c) Number of tetrahedral voids is twice the number of OH OCH2Ph
atoms in the given unit cell.
(d) Number of tetrahedral voids is equal to the number of O O
atoms in the fcc unit cell.
(c) OCH2Ph (d) OH
28. In the following reaction, the product(s) formed is/are OCH2Ph HO CH Ph
2
OH
. ´10 -10 . Which of the following
32. K sp of CaF2 is 17
CHCl3
–OH combinations will bring out precipitation of CaF2?
(a) 10 mL10-4 MCa 2+ + 10 mL 2 ´ 10-3 M F -
CH3 (b) 10 mL10-5 M Ca 2+ + 10 mL 2 ´ 10-2 MF -
OH O (c) Equal volumes of 10-2 M Ca 2+ and 10-3 M F -
OHC CHO (d) 10 mL 103 MCa 2 + + 10 mL 10-5 M F -

H3C CHCl2 Section 3 (Maximum Marks : 12)


CH3 Instructions : Same as given in Physics
(P) (Q)
Paragraph X
OH OH A, B and C are three complexes of chromium (III) with
CHO empirical formula H12 O 6 Cl 3 Cr. All the three complexes
have water and chloride ion as ligands. Complex A does
not react with conc. H 2 SO 4 , whereas complexes B and C
lose 6.75% and 13.5% of their original mass, respectively,
H 3C CHCl2
CH3 on treatment with conc. H 2 SO 4 .
(R) (S) 33. Compound B is
(a) P is major product (b) Q is minor product (a) [Cr(H2O)6 ]Cl 3 (b) [Cr(H2O)5 Cl]Cl 2 ×H2O
(c) R is minor product (d) S is major product (c) [Cr(H2O)3 Cl 2 ]Cl × 2H2O (d) [Cr(H2O)4 Cl 2 ]Cl × 2H2O

29. Select the correct statement. 34. Compound C is


(a) SbCl 3 is hydrolysed to orange coloured SbOCl soluble (a) [Cr(H2O)2 Cl 2 ]Cl × H2O (b) [Cr(H2O)2 Cl 2 ]Cl × 2H2O
in dil. HCl (c) [Cr(H2O)6 ]Cl 3 (d) Cr(H2O)2 Cl] × Cl 2
(b) BiCl 3 is hydrolysed to white coloured BiOCl soluble in
dil. HCl Paragraph A
(c) NCl 5 and PCl 5 are stable compounds An optically active compound H[C 5 H 6 O] on treatment
(d) PCl 5 can change to PCl -6 and PCl +4 with no change in with H 2 in the presence of Lindlar’s catalyst gave a
hybridisation of P compound I (C 5 H 8 O). Upon hydrogenation with H 2 and
Pd/C, compound H gave J (C 5 H12 O). Both I and J were
30. If the radius of first Bohr’s orbit of H-atom is x, found to be optically inactive.
which of the following is the correct conclusion?

PREP CATALYSIS
JEE Advanced~Practice Set 15 325

35. The structural formula of compound I is 36. The structural formula of J is


C2H5 H CH2CH2CH3
CH
(a) H3C C OH (b) H C CH3
(a) H C C2H5 (b) H C CH2
CH3 OH
OH CH2
C2H5 H
C2H5 CH
(c) H C C2H5 (d) H5C2 C OH
(c) H C CH3 (d) H C CH CH2
OH CH3
OH OH

MATHEMATICS
Section 1 (Maximum Marks : 21) 42. If a and b are two vectors, such that| a | = 1,| b| = 4
Instructions : Same as given in Physics and a × b = 2. If c = (2a ´ b) - 3 b, then the angle
x between b and c is
éæ e ö æ1 x öù
37. lim ê ç ÷ç - ÷ ú is equal to p p 3p 5p
x ® ¥ è1 - e ø è e 1 + x øû (a) (b) (c) (d)
ë 3 6 4 6
æ1 - e ö
ç ÷ 43. Least natural number a for which x + ax -2 > 2,
(a) e( 1 - e ) (b) e è e ø
æ e
ç
ö
÷ æ1 + e ö "x Î (0 , ¥) is
ç ÷
(c) e è 1 - e ø (d) e è e ø (a) 1 (b) 2
(c) 5 (d) None of these
p 3p 7p 9p
38. The value of cos cos cos cos
20 20 20 20
p 2p 4p 8p Section 2 (Maximum Marks : 28)
+cos cos cos cos is Instructions : Same as given in Physics
15 15 15 15
(a) 0 (b) 2 é3 - 1 - 2ù
(c) 4 (d) -2 44. Let P = ê2 0 a ú, where a ÎR. Suppose
ê ú
39. Let bi > 1 for i = 1, 2, ..., 101. Suppose log e b1 , êë3 - 5 0 úû
log e b2 , ..., log e b101 are in AP with the common Q = [q ij ] is a matrix such that PQ = kI, where k Î R,
difference log e 2 . Suppose a 1 , a 2 , ..., a 101 are in AP, k ¹ 0 and I is the identity matrix of order 3. If
such that a 1 = b1 and a 51 = b51 . If k k2
t = b1 + b2 + ... + b51 and s = a 1 + a 2 + ... + a 51 , then q 23 = - and det(Q) = , then
8 2
(a) s > t and a101 > b101 (a) a = 0, k = 8 (b) 4a - k + 8 = 0
(b) s > t and a101 < b101
(c) s < t and a101 > b101 (c) det(P adj(Q )) = 2 9 (d) det(Q adj(P)) = 213
(d) s < t and a101 < b101 ìx + a; x ³ 0
45. If f (x) = í ,
æ æ p ö ö log 10 6 -1 î2 - x; x < 0
40. If log 10 çsin ç x + ÷ ÷ = , then the value of
è è 4 øø 2
ì { x}; x <0
log 10 (sin x) + log 10 (cos x) is g (x) = í and f (g (x)) is continuous at
îsin x + b; x ³ 0
(a) – 1 (b) – 2
(c) 2 (d) 1 x = 0, then which of the following is/are true
(where, { x} represents the fractional part
41. Let a i denotes the number of non-negative integral function)?
solution of the equation x1 + x 2 + x 3 +¼+ x11 - i = i. If (a) If b = 1, then a can taken any real value
‘i’ varies from 1 to 9, then a 1 + a 2 +¼+a 9 is equal to (b) If b < - 1, then a + b = 1
(a) 1020 (b) 1022 (c) No values of a and b are possible
(c) 1023 (d) 1024 (d) There exist finite ordered pairs (a, b )

PREP CATALYSIS
326 JEE Advanced~Practice Set 15

46. If (sin - 1 x + sin - 1 w)(sin - 1 y + sin - 1 z) = p 2, then (c) The volume of the figure bounded by the points of S and
N1 N2 20p
x y the planes is cu units
D= (where, N1 , N 2 , N 3 , N 4 Î N) 6
zN3 w N4 (d) The area of the curved surface formed by the points of S
(a) has maximum value of 2 10p
is cu units
(b) has minimum value of 0 3
(c) 16 different D are possible
(d) has minimum value of - 2 Section 3 (Maximum Marks : 12)
æ 1ö Instructions : Same as given in Physics
- çt + ÷
x è tø dt
47. Let f : (0 , ¥) ® R be given by f (x) = ò1/ x e t
,
Paragraph X
then Football teams T 1 and T 2 have to play two games against
(a) f(x ) is monotonically increasing on (1, ¥) each other. It is assumed that the outcomes of the two
(b) f(x ) is monotonically decreasing on [0, 1) games are independent. The probabilities of T 1 winning,
1 1 1
(c) f(x ) + f æç ö÷ = 0, for all x Î(0, ¥)
1 drawing and losing a game against T 2 are , and ,
èx ø 2 6 3
(d) f(2 x ) is an odd function of x on R respectively. Each team gets 3 points for a win, 1 point for
a draw and 0 point for a loss in a game. Let X andY denote
48. A curve y = f (x) passes through (1, 1) and tangent the total points scored by teams T 1 and T 2 , respectively
at P(x , y) cuts the x-axis and y-axis at A and B after two games.
respectively, such that BP :AP = 3 :1, then 51. P(X > Y) is equal to
(a) equation of curve is xy¢-3 y = 0 1 5 1 7
, ) is x + 3 y = 4
(b) normal at (11 (a) (b) (c) (d)
4 12 2 12
(c) curve passes through æç2, ö÷
1
è 8ø 52. P(X = Y) is equal to
(d) equation of curve is xy¢+3 y = 0 11 1 13 1
(a) (b) (c) (d)
49. For z as real or complex, if (1 + z 2 + z 4)8 36 3 36 2
= C0 + C1 z 2 + C2 z 4 + K + C16 z 32 , then Paragraph A
(a) C 0 - C1 + C 2 - C 3 + K + C16 = 1 Let F1 ( x 1 , 0) and F 2 ( x 2 , 0), for x 1 < 0 and x 2 > 0 be the foci
x2 y2
(b) C 0 + C 3 + C 6 + C 9 + C12 + C15 = 37 of the ellipse + = 1 . Suppose a parabola having
9 8
(c) C 2 + C 5 + C 8 + C11 + C14 = 36 vertex at the origin and focus at F 2 intersects the ellipse at
(d) C1 + C 4 + C 7 + C10 + C13 + C16 = 37 point M in the first quadrant and at point N in the fourth
quadrant.
50. Consider a set of point S in the space which is at a
x y -1 z + 2 53. The orthocentre of DF1MN is
distance of 2 units from the line = =
(a) æç - , 0ö÷ (b) æç , 0ö÷ (c) æç , 0ö÷ (d) æç , 6 ö÷
9 2 9 2
1 -1 2
è 10 ø è3 ø è 10 ø è3 ø
between the planes x - y + 2 z + 3 = 0 and
x - y + 2 z - 2 = 0. Then, 54. If the tangents to the ellipse at M and N meet at R
(a) The volume of the figure bounded by the points of S and and the normal to the parabola at M meets the
10 X-axis at Q, then the ratio of area of DMQR to area
the planes is p cu units
3 3 of the quadrilateral MF1NF2 is
(b) The area of the curved surface formed by the points of S (a) 3 : 4 (b) 4 : 5
20p (c) 5 : 8 (d) 2 : 3
is sq units
6

PREP CATALYSIS
JEE Advanced~Practice Set 15 327

Answers
Paper 1
1. a, b, d 2. a, c, d 3. a, b 4. a,b,c,d 5. a,b,c,d 6. a, b, c 7. a, b, c 8. 3 9. 2 10. 8
11. 3 12. 4 13. c 14. a 15. b 16. a 17. b 18. d 19. a, b 20. b, c
21. b 22. d 23. c 24. a 25. a, b, c 26. 3 27. 6 28. 5 29. 2 30. 5
31. a 32. b 33. d 34. b 35. c 36. a 37. a, c 38. a, c, d 39. a,b,c,d 40. a,b,d
41. b,d 42. b,d 43. c,d 44. 4 45. 9 46. 6.28 47. 24 48. 6 49. a 50. b
51. d 52. a 53. d 54. a

Paper 2
1. a 2. d 3. a 4. a 5. d 6. b 7. c 8. a, b 9. a, c, d 10. a,b,c,d
11. a, d 12. b, d 13. a, b, c 14. a, c, d 15. d 16. a 17. a 18. b 19. b 20. a
21. d 22. b 23. b 24. b 25. a 26. b, c 27. b, c 28. b, d 29. a, b 30. a, c
31. b 32. b, c 33. b 34. b 35. d 36. c 37. c 38. a 39. b 40. a
41. b 42. d 43. b 44. b, c 45. a, b 46. a, c, d 47. a, c, d 48. c, d 49. a, b, d 50. b, c
51. b 52. c 53. a 54. c

SCORE SHEET - Paper 1


Section No. of Marks from Marks from Marks Obtained
Correct Questions Correct Questions (A) Incorrect Questions (B) (A-B)
................ ............................... ............................... ............................... ...............................
................ ............................... ............................... ............................... ...............................
................ ............................... ............................... ............................... ...............................
Percentage Marks = Marks Obtain/Total Marks x 100

SCORE SHEET - Paper 2


Section No. of Marks from Marks from Marks Obtained
Correct Questions Correct Questions (A) Incorrect Questions (B) (A-B)
................ ............................... ............................... ............................... ...............................
................ ............................... ............................... ............................... ...............................
................ ............................... ............................... ............................... ...............................
Percentage Marks = Marks Obtain/Total Marks x 100
Note To expect your success marks in the test should be between 65%-70%.

PREP CATALYSIS
PRACTICE SET - 1
Paper 1
b c From Bernoulli’s principle, 1 1 1
1. (a, c) As, U = a − + 3 ⇒ − =
x x ρ v − x 9
∆p = ( v 22 − v12 ) = ρ vavg ( v 2 − v1 )
For stable equilibrium, U must be 2 1 1 1
or = − …(i)
minimum, v + v1 v 9 x
where, vavg = 2
dU d 2U 2 For object O 2, image is virtual and it
so = 0 and 2 > 0
dx dx = 960 km/h also formed at I1
Now,
dU
= 0⇒ x =
2c
or x = ∞ = 267 ms −1 1

1
=
1
…(ii)
dx b v 2 − v1 ∆p v (24 − x ) − 9
So, = 2
d 2U 2c vavg ρvavg As, source O 2 is on left side of lens, so
and > 0, for x =
dx 2 b focal length of lens is negative,
6.5 × 10 3
2c = ≈ 8% From Eqs. (i) and (ii), we have
So, for stable equilibrium, x = . . × (267 )2
12
b 1 1 1 1
− =− +
−dU 6 9 x 9 24 − x
Also, F = kx and F = 4. (a, c) I = = 1 A,
dx 6 ⇒ x 2 − 24 x + 108 = 0
d  dU  d 2U b4 I = nevd A ⇒ x = 6 cm or x = 18 cm.
So, k = −   =− = .
dx  dx  dx 2
8c 3 ⇒ vd =
I Now, substituting for x( x = 18 cm) in
neA Eq. (i), we get
2. (a, c, d) 1 1 1 1
= = −
(a) Distance from earth’s centre . × 10 −19 × 10 −6
10 29 × 16 v 9 x
= 6400 + 640 = 7.04 × 10 6 m 1 1 1 1 1
1 = − ⇒ =
⇒ vd = × 10 −4 ms −1
mv 2 GMm v 9 18 v 18
Now, = .
16
r r2 ⇒ v = 18 cm
KE of all electrons,
GM 1 6. (b, c) Using relative velocity formula,
⇒ v = = 56.8 × 10 6 = mvd2 nAl
r 2 we have
= 7.56 × 10 3 ms −1 1 1 v G = v A + ω × rGA
= × 91 . × 10 −31 × × 10 −8
2 2.56 and v B = v A + ω × rAB
(c) Total energy after 1500 rounds
× 10 29 × 10 −6 × 10 −1 = v A + ω k$ × L ( − cos θ $i + sinθ$j )
= Initial energy − Energy loss
GMm = 2 × 10 −17
J ⇒ − v = 2 $i + 3$j
=− − 142
. × 10 5 × 1500 B
2r Ohmic loss = I R = 6 Js 2 −1 − ω L ⋅ (cos θ$j + sinθ $i )
= ( − 6253
. − 0213
. ) × 10 9
2 × 10 −17  1  1 3 $
= −6.466 × 10 9 J Time to loss KE, t = = 2 $i + 3$j − ω    $j + i
6 2 2 2 
GMm
So, r = − 20 Y vA
− 6.466 × 10 9 = × 10 −18 A
6
= 6808 km
= 3.4 × 10 −18 s
∴ Its height above the earth’s
surface = 6808 − 6400 5. (b, c, d) Images of O1 and O 2 G
= 408 km
(d) Average retarding force, B
dU vB
|F| = X
dr I2
Energy loss
⇒F = O1 O2 I1 Comparing both sides, we have
Distance ω
L 0 = 3 − or ω = 4 3 rads −1
. × 10 5
142 4
=
2 × 314
. × 6.924 × 10 3 Subtituting values in Eq. (i), we get
may coincides, if L is placed as shown 1
= 3.3 × 10 −3N in ray diagram. If O1L = x, then v G = v A + ω k$ × (cos θ $i − sinθ$j )
2
3. (b, d) Pressure difference holds, the LO 2 = 24 − x. ωL
= vA + (cos θ$j + sinθ $i )
weight of air craft, Now for object O1, a real image is 2
mg 3.3 × 10 5 × 9.8 formed at I1, 1 3 $
∆p = = = (2 $i + 3$j ) + 3  $j + i
A 500 1 1 1
− = 2 2 
= 6.5 × 10 3 Pa v u f
= 3.5 $i + 2.6 $j
332 JEE Advanced~Practice Set 1

7. (2.95) 10. (35.60) 1 q2


So, u = ε 0E 2 =
Work done, W =  ∫ F . dr  × 10 −2 8 π ε0L2r 2
B dB 2
Induced emf, e = NA =2V 2
 A  dt Energy in cylindrical volume of radius r
=  ∫ Fxd x + Fyd y + Fzd z  × 10 −2
B e
Current in coil, i = = 125
. A is
 A  R
U = ∫ x ⋅ dV
B Power dissipated, P = i 2R = 2.5 W
= ∫A (2 xy + z2 ) dx + x 2dy + 2 xzdz where, dV = differentiable volume
B Total heat obtained in 12000 cycles, = 2 πrL1dr
= ∫A (2 xydx + x 2dy) + ( z2dx + 2 xzdz)
= H = P × t = 12000 J r q 2.2 πrL
B −2
Hence, U = ∫a 8 π 2ε L2r12 . dr
= ∫A d ( x y) + d ( z x ) × 10 This heat is used in raising
2 2
0
temperature of coil and water.
= (x y + z
2 2
x )|BA × 10 −2 q2  r
If θ = final temperature, then = loge  
4 πε0L  a
= ( x 2 y + z2 x )|(( 50,, 12,, 27)) × 10 −2 H = mw s w (θ − 30° ) + mc sc (θ − 30° )
For r = b, U = U b
(where, w = water and c = coil)
= ( 5 2 × 2 + 7 2 × 5 − 0) × 10 −2 q2  b
−2 ⇒ 12000 = 0.5 × 4200 (θ − 30° ) and U b = loge  
= ( 50 + 245) × 10 J = 2.95 J 4 πε0L1  a
+ ( 0.06) ( 500) (θ − 30° )
Ur 1
8. (2.88) ⇒ θ = 35.60°C For = , we have
At x,t , Ub 2
11. (0.63)
y = 0.012 sin ( 015
. x ) ⋅ sin ( 4.8 × 10 4 t )  r 1  b
Let, L = length of edge of cube. Light log   = log  
 a 2  a
∂y leaving the bubble at angle θ is totally
Particle velocity is given by .
∂t reflected, when θ = θc . ⇒
r
=
b
∂y a a
∴ vparticle =
∂t L θc or r = ab = 6 ≈ 2.45
= 0.012 sin ( 015
. x ) ⋅ cos ( 4.8 × 10 t ) 4 2
13. (2.50)
⋅ ( 4.8 × 10 4 ) Heat rejected by water
This is maximum, when
= mw s w (Ti − Tf )
cos( 4.8 × 10 4 t ) = 1
Heat absorbed by ice,
So, vparticle (maximum at x = 3.5 cm ) So, each face of cube must be = mi s i ( 0 − Ti ) + mi s w (Tf − 0) + mi L
0
 180  covered (to prevent bubble from Equating both and subtituting values,
= 0.012 × sin  015
. × 3.5 × 
 . 
314 being visible) is a circle of radius r that we get
is associated with the angle θc .
× 4.8 × 10 4 Tf = 2.5°C
1
Now, sinθc = 14. (N = 210
= 0.012 × 4.8 × 10 × sin 30° 4
n . )
1 1 1 1 Assuming ideal behaviour,
= 576 × = 288 cm - s −1 = 2.88 ms −1 and r = tanθc = ×
2 2 2 n2 − 1 Number of molecules per unit volume
9. (a = 4.91) 10
of gas,
⇒ r= = 4.47 mm N p
Net force on system is . )2 − 1
2 (15 n= =
V k BT
Fnet = mg − Fm
There are six forces of cube, so six . × 10 5
101
vB2L2 discs of radius r are required. =
= mg − . × 10 −23 × 293
138
R Fraction of surface area that must be
vB2L2 covered is = 2.5 × 10 25 molecules per m3
⇒ anet = g −
mR 6 πr 2
πr 2
So, mean free path is
f = =
When velocity is terminal, 6L2 L2 1
l=
anet = 0 ⇒ v = vT π ( 4.47 )2 2 π d 2n
2 2
= 2
vT B L 10 1
⇒ 0=g − =
mR = 0.63 2 × π × (2 × 10 −10 )2 × 2.5 × 10 25
mgR
⇒ vT = 12. (2.45)
B2L2 = 2.25 × 10 −7 m
Energy density at any point in
Now, when velocity is half of terminal capacitor is So, collision frequency is
velocity, 1 q v
u = ε0E 2 and E = f =
v mgR 2 πε0Lr l
v = T = 2
2 2 B2L2 =
473
where, L = length of cylindrical
Then acceleration, capacitor 2.25 × 10 −7
mgR B2L2 g and r = distance of point from centre = 2.10 × 10 9/s
a=g − 2 2
× =
2B L mR 2 or axis. So, N = 2.10
= 4.91ms −2
JEE Advanced~Practice Set 1 333

On adding K 3[Fe(CN)6 ] to FeCl 2 solution, Fe II [Fe III (CN)6 ]−


15. (a) Error in D 2 = 2 
0.001  l

 = 57
. %
 0.035  and Fe III [Fe II (CN) ]¨ are formed due to side redox reaction.
6
 0.001
Error in L = 2  Fe III [Fe II (CN)6 ]− is diamagnetic while Fe II [Fe III (CN)6 ]− is
 = 17
l
2
. %
 0120
.  paramagnetic *[Fe II (CN)6 ]−
1
D2 x −6 = − 1− 3 ⇒ x =− 4 + 6 ⇒ x = 2
Error in = ( 57
. 2 + 17
. 2 )2 % = 6%
L2 3d 6 4s 0 4p 0
1
Fe2+ =
 D2  2
So, error in  1 + 2  = 025
. %
 L  [FeII(CN)6]– =
1 Diamagnetic
So, error in measurement of B = ( 025
. 2+ . 2 )2
10 = 10
. % 6
3d 4s 0 4p 0
1 III – 3+
16. (d) Fractional error in V = = 5% For [Fe (CN)6], Fe =
20
01. 4s 0 4p 0
Fractional error in r 2 = 2 × = 5.9% II
[Fe (CN)6] = –
3.4
Fractional error in B = 2 × 1% = 2%
2 Paramagnetic

1
So, fractional error in e/m is = ( 5 2 + 5.9 2 +
21. (b)
2 2 )2 % = 8% l
KClO 3, KNO 3, sulphur and antimony contains the head of
17. (c) In region r1 < r < r2, we first find electric field using Gauss’ match stick. The sides of match box contains red
law, we get phosphorus and sand powder.
l
Ammonia has the highest proton affinity.
Q r 3 − r13
E= NH 3 + H + → NH 4+
4 πε0 r ( r2 − r13 )
2 3
l
Molecular nitrogen is less reactive than that of oxygen
Potential at distance r from centre is given by because nitrogen has high dissociation energy in
r Q  1  r  r3  comparison to oxygen. So, its reactivity is less. Also bond
V = V( r = r2 ) − ∫ E ⋅ dr = Vr = r2 −   ∫r  r − 12  dr length of nitrogen is shorter than oxygen because of the
r2 4 πε0  r23 − r13  2  r 
presence of triple bond between nitrogen atoms. So, both
Q 1  3r 2 r 2 r13  are true but not correct explanation.
= . 3 . 2 − −  Thomas slag or phosphatic slag is a mixture of calcium
4 πε0 ( r2 − r1 )  2
l
3
2 r
phosphate and calcium silicate [Ca 3(PO 4 )2 ⋅ CaSiO 3 ]. It is
3Q used as manure.
Now, using ρ = ; r1 = r and r2 = 3r
4 π( r23 − r13 ) 22. (a)
Also r = 2 r, we get from above expression Cl Cl Cl
ρ  3r22 r 2 r13  ρ  3 × 9r 2 4r 2 r 3 
V(at r = 2r ) =  − − =  − −  Sn/HCl CH3COCl
3 ∈0  2 2 r  3 ∈0  2 2 r –HCl

ρr 2  27  ρr 2
=  − 2 − 1 = 3.5 V N NH2 NHCOCH3 CH3 CH CH2
3 ∈0  2  ∈0 H+
O O
18. (d) Inside the shell, when r < r1 electric field is zero. Cl Cl
So, potential everywhere is same as a point on the inside CH3
H3O+
surface of shell ( r = r1 ).
CH
So, from result of previous part with r = r1, we get
Q 3( r22 − r12 ) ρ NH2 NHCOCH3CH3
V = . = . ( r22 − r12 )
4 πε0 2( r23 − r13 ) 2 ε0
23. (a, b, d) Only reaction (c) will not give 1, 3-dichlorobenzene
With r2 = 3r and r1 = r, above result is reduced to
NO2 NO2
ρ 4ρr 2
V = ( 9r 2 − r 2 ) =
2 ε0 ε0 Nitration Cl2
(a)
Conc. HNO3+ AlCl3
19. (b, c) Conc. H2SO4 Cl
In the aqueous solution of Pd(NH 3 )2Cl 2 , the atoms chlorine
SnCl2
are in coordination sphere and the van’t Hoff factor of the
compound are unite. –
+
Cl N2Cl NH2
20. (a, b, d)
On adding FeCl 3 solution to K 4 [Fe(CN)6 ] solution, two CuCl NaNO2
products are formed i.e. Fe III [Fe II (CN )6 ]¨ and Fe II [Fe III (CN )6 ]¨ .
Cl Cl Cl
These are result of side redox reaction.
1,3-dichloro benzine
334 JEE Advanced~Practice Set 1

CH3 CH3 COOH 4d8 5s0 5p0


Cl Cl 2+
Pd =
Cl2 KMnO4
(b)
AlCl3 NaOH,∆ 4d8
[PdBr4]2=
Cl Cl
Diamagnetic
CaO,NaOH,∆ due to the presence
of unpaired electrons
Cl l
In [Ni(CO)4 ], Ni is in zero oxidation state.
3d8 4s2 4p0
Ni0(3d84s2)=
Cl
3d8 4s0 4p0
1,3-dichloro benzene
[Ni(CO)4]=
CH3
Diamagnetic
due to the presence
(c) KCl,∆ of paired electrons
No substitution
Cl
26. (1) By Freundlich adsorption isotherm for adsorption of
NO2 NH2 solution on a solid surface.
1 1
x x
Nitration Zn/ HCl NaNO2 +HCl = K (C )n ⇒ = 0.50(C )3
(d) m m
∆ O°C
NO2 NH2 Let, solute absorbed = x g
N2+ Cl

Cl Then, equilibrium concentration = (2 − x )g
1
x
CuCl
∴ = 0.50 (2 − x )3 ⇒ x = 1 (2 − x )3 ⇒ x 3 = (2 − x )
2
N2+ Cl

Cl ∴ x3 + x − 2 = 0
1,3-dichloro
benzene Thus, x = 1 g

∆H °
27. (0.05) Since, K f of Ag(S 2O 3 )32 − is very high hence, it is
24. (b, c) log K = log A = assumed that all the Ag + from AgBr is complexed, Thus, the
2.303 RT
∆H °
molar solubility of AgBr in S 2O 32 − can be calculated as

K = Ae RT
AgBr( s ) - Ag + ( aq ) + Br − ( aq ), K1 = Ksp
∴ log K = 10 = OP Ag + + 2S 2O 23 − - Ag(S 2O 3 )32 − ( aq ), K 2 = K f
∴ A = 1010
AgBr( s ) + 2S 2O 23 − - Ag(S 2O 3 )32 − + Br − , K = Ksp ⋅ K f
∆H °
Slope = Initial conc. 0.1 0 0
2.303 R
Concentration at 0.1− 2 x x x
∆H ° = Slope × 2.303 R equilibrium
∴∆H° = 2.303 × 8.314 J mol − 1 K − 1 × 0.5 = 9.574 J mol − 1 ∴K = K sp ⋅ K f = 5.4 × 10 − 13 × 2.9 × 1013 = 15.66
25. (4) Diamagnetic molecules are [Zn(OH)4 ]2 − , K 4 [Fe(CN)6 ], [Ag(S 2O 3 )23 − ] [Br − ] x× x
2− Also, K = = 15.66 =
[Pd Br4 ] , [Ni(CO)4 ]. [S 2O 23 − ]2 (0.1 − 2 x )2
l
In [Zn(OH)4 ]2 − , Zn2 + = 3d 10 4s 0
x 2 = ( 0.1 − 2 x )2 15.66 ⇒ x = 0.05 M
3d10 4s0 4p0
Zn2+= Hence, solubility of AgBr(s) in 0.1 M S 2O 23 − = 0.05 M

28. (0.23) Given, K p (1000 K) = 0.48 ⇒ K p (700 K) = 0.83


[Zn(OH)4 ]2 − is diamagnetic due to the paired electrons.
As we know that,
l
In K 4 [Fe(CN)6 ], Fe is in + 2 oxidation state.
K (1000 K) ∆H °  3   0.48 
3d6 4s0 4p0 ln p =   = ln   ...(i)
K p (700 K) R  7000   0.83 
2+
Fe = K (1000 K) ∆H  15  0.48
ln p =   = ln ...(ii)
3d6 4s0 4p0 K p ( 850 K) R  85000  K p ( 850)
[Fe(CN)6]4–= 0.48
ln
K p ( 850 K) 35
Diamagnetic d2sp3 hybridised = ⇒ In 0.48 − ln K p ( 850 K) = 0.749
due to the presence  0.48  85
ln  
of paired electrons  0.83 
l
In [Pd Br4 ]2 − , Pd is in + 2 oxidation state with electronic ⇒ K p(850 K) = 0.23 ⇒ 0.23 =
p(CO 2 )
⇒ p(CO 2 ) = 0.23
configuration 4d 8 5s 0 1.21 − p(CO 2 )
JEE Advanced~Practice Set 1 335

29. (0.01) Total vapour pressure is due to hydrogen formed and ∴K = antilog of 0.6990= 5.0 bar dm 3
aqueous tension of H 2O. 1 p 0.2 bar
= = = 0.04 dm− 3
pTotal = pH 2O + pH 2 V K 5 bar dm3
∴pH 2 = p Total – pH 2O Mass
∴Density = = 2 × 0.04 = 0.08 g dm− 3
753.5 Volume
∴pH 2 = (780 − 26.5) mm Hg = atm = 0.99 atm
760 33. (c) OH OH OH
By ideal gas equation CHO
n Mass of H 2( w ) CHCl3/KOH
pH 2 = RT = × RT Reimer-
+
V Molar mass × V of H 2( M.M) Tiemann
C
pH 2 × N × M × V reaction
CHO
Thus, w = Steam
RT volatile
B
0.99 × 2 × 0.128
w= = 0.01 g Compound ‘C’ undergoes intramolecular H-bond.
0.0821 × 295
O H
30. (373.65) Mass of NaCl = 3.5 g C O
H
3.5
∴Number of moles of NaCl =
58.5
So, carbonly group in this case does not react with Schiff’s
Number of ions furnished by one molecule of NaCl is 2.
reagent.
So, actual number of moles of particles furnished by sodium
3.5 34. (b) OH OH OH
chloride = 2 ×
58.5 CHO CH2OH COOH
LiAlH4 [O]
Similarily, actual number of moles of particles furnished by
0.13
magnesium chloride = 3 × (D) (CH3CO)2O/ (E)
95
CH3COOH
 3.5 0.13 
Total number of moles of particles =  2 × + 3× 
 58.5 95  OCOCH3
= 0.1238 COOH
Mass of water = (100 − 3.5 − 0.13) = 96.37 g
0.1238
∴Molality = = 1.2846 F
96.37 (Pain reliever)
(Acetyl salicylic
∆Tb = K b × m acid aspirin)
∴ ∆Tb = 1.2846 × 0.51 = 0.655 K
35. (b)
Thus, ∆Tb = Tb − Tb° O Cl
Cl Me Me
Tb = 373.655 K AlCl3
PCl5 Cl
+ Me
Friedel-Craft
31. (0.72) Cell reaction is O acylation (A ) (B)
Zn + 2H + - Zn2 + + H 2; E° = + 0.76 V

x2 NH2NH2+OH
Also, HIO 3 - H + + IO −3 ⇒ 0.2 = 2NaNH2

0.1 − x x x 0.1 − x Me

or, x 2 + 0.2 x − 0.02 = 0


− 0.2 + 0.04 + 0.08
x= = 0.073 M
2
(C)
0.059 [Zn2 + ]
Nernst equation is given as, E = 0.76 − log 36. (c)
2 [H + ]2
0.059 0.1 H H
= 0.76 − log = 0.722 V Me
2 (0.073)2 H2, Ni2B Me
Syn–add.,H2
K (C)
32. (0.08) p = for Boyle’s law under isothermal condition. (D)
V
Li, Liq NH3 + H2O
Taking log on both sides, we get
1
log p = log + log K H Me
V
H
Thus, log K = 0.6990 bar dm 3
Slope = tan 45° = 1 (E)
336 JEE Advanced~Practice Set 1

37. (b,c) Area of quadrilateral ABCD is BD = 6 Required area


1
maximum when area of ∆ACD is and DC = 8 1
 x4 
maximum = ∫ ( x 3 + 3 x 2 )dx =  + x3 
B OD 4 2
⇒ Distance of D from AC is maximum
∴ tan = = = 0  4 0
2 BD 6 3
i.e. cos θ − sin θ is maximum 5
C OD 4 1 = sq units.
 π and tan = = = 4
= 2 cos  θ +  is maximum 2 DC 8 2
 4 42. (b, c) We have,
 π A B C
7π tan −  = tan + 
⇒ θ= 2 2 2 2 α, β are the roots of ax 2 − bx + c = 0
4 and γ , δ are the roots of
B C
and area =
6
× 2 2 = 12 sq unit tan + tan px 2 − qx + r = 0
2 = 2 2
B C Since, α, β, γ , δ are in AP, then they
[Q ABCD is a rectangle] 1 − tan tan
2 2 are α, α + d , α + 2d , α + 3d
38. (a,b,c,d) 2 1 b q
+ ∴α + α + d = , α + α + 5d =
(a) We know that, the plane A 7
⇒ cot = 3 2 = a p
ax + by + cz + d = 0 contains the 2 1− 2 × 1 4 q b
x−α y−β z−γ 3 2 ∴ 4d = −
line = = if p a
l m n A 4
⇒ tan = aq − bp
aα + bβ + cγ + d = 0 2 7 ∴ d =
4ap
and al + bm + cn = 0 B C s−a
⇒ tan tan =
2 2 s Now, if α, β, γ, δ are in GP, then they
Now, since
s−a 1 are α, αr, αr 2, αr 3
( − 1) − 2( 3) + 7( − 2 ) + 21 = 0 ⇒ = ⇒ 2 s = 3a
and ( −3) (1) + 2( −2 ) + 1 (7 ) = 0 s 3 c r
α ⋅ αr = , αr 2 ⋅ αr 3 =
⇒ 2 s = 42 ⇒ s = 21 a p
The line given in (a) lies on the given
plane. ∴ ∆ = rs ⇒ ∆ = 4 × 21 = 84 sq cm r/ p
∴ r4 =
(b) Since, 0 − 2 (7 ) + 7 ( − 1) + 21 = 0 A B C c/a
Q tan , tan , tan all less than 1 1/ 4
∴The point (0, 7, −1) lies on the plane. 2 2 2 ar  ar 
⇒ r4 = ⇒r =  
∴∆ is acute angled. cp  cp 
(c) Direction ratios of the normal to the
given plane are (1, − 2, 7 ) which are 41. (b, c) We have, 43. (0) Given a, b, c, d are the vertices of a
same as those of the given in (c). So x
−t cyclic quadrilateral ABCD and
f( x ) = 3 x + ∫e f( x − t ) dt
2
the plane is perpendicular to the line. A ≠ 90°.
0
(d) The direction ratios of the normal to a a
D(d) C(c)
the planes given in (d) are same as Apply the rule ∫ f( x ) dx = ∫ f( a − x ) dx
those of the given plane. So, the plane 0 0
in (d) is parallel to the given plane. x
1 f( x ) = 3 x 2 + e − x ∫ e f(t )dt
t

2 1 − x2 0
39. (a, b, c) We have, f( x) = x e
x
⇒ e x f( x ) = 3 x 2e x + ∫ e f(t ) dt A(a) B(b)
1 t
 2 x3  1 − x 2
⇒ f ′( x) =  2 x + e 0
 (1 − x 2 )2  Now, as ABCD is a cyclic quadrilateral
Differentiate with respect to x by
1 ∴ A + C = 180°
 Leibnitz rule, we get
x2  1 − x 2 B + D = 180°
⇒ f ′( x) = 2 x  1 + e e ( f ′ ( x ) + f( x )) = e ( 3 x + 6 x ) + e f( x )
x x 2 x and
 (1 − x 2 )2  ⇒ A = 180° − C
⇒ f ′( x) = 3 x 2 + 6 x
Clearly, f ′ ( x ) > 0 for x ∈ ( 0, 1) and ⇒ tan A = tan(180° − C )
f ′( x ) < 0 for x ∈ ( −1, 0) . So, f( x ) is On integrating f( x ) = x 3 + 3 x 2 + C
⇒ tan A + tan C = 0
decreasing in ( −1, 0) and increasing in But from the given result f( 0) = 0
sin A sin C
1
⇒C =0 ⇒ + =0
1 − x2 cos A cos C
(0, 1) lim x e 2
= ∞ and f( 0) = 0
x → ±1 ∴f( x ) = x 3 + 3 x 2
| AB × AD | | CB × CD |
⇒ + =0
⇒ f( x ) = 1 has exactly one solution in f ′ ( x ) = 3 x 2 + 6 x = 0 ⇒ 3 x( x + 2 ) = 0 AB ⋅ AD CB ⋅ CD
each of the intervals ( − 1, 0) and (0, 1) |(b − a ) × (d − a )|
⇒ x = 0, − 2 ⇒
40. (a,b,c) Here, r = 4 cm f ′ ′( x) = 6 x + 6 (b − a ) ⋅ (d − a )
A |(b − c ) × (d − c )|
But f ′ ′ ( − 2 ) = − ve and f ′′ ( 0) = 6 + =0
Q x = − 2 gives maximum of f( x ) (b − c ) ⋅ (d − c )
∴ f( − 2 ) = − 8 + 12 = 4 | b × d + a × b + d × a|

A turning point is ( − 2, 4) (b − a )⋅ (d − a )
O
f( x ) = x 3 + 3 x 2 | b × d + d × c + c × b|
+ =0
4 (b − c )⋅ (d − c )
B 6 D 8 C
JEE Advanced~Practice Set 1 337

44. (4) Given f( x) = f( y) f( x − y) QTr + 1 is independent of x Eq. (iii) divides Eq. (iv), we get
9 − 3r α − β 3 −2
Replacing x by x + y, we get ∴ = 0⇒r = 3 tan   =
6  2  3+2
f( x + y) = f( y) f( x + y − y)
∴ T4 = 9C 3 a 6 b 3
⇒ f( x + y) = f( x ) f( y) ( 3 − 2 )2
9× 8×7 6 3 =
⇒ f( x ) = e kx
⇒ f ′ ( x ) = ke kx
= a b = 84 a 6 b 3 −1
3×2 ×1
4
= −7 + 4 3
But f ′ ( 0) =∫ {2 x} dx = 2 a2 + b
Now, ≥ ( a 2b )1/ 2 ∴ a = − 7, b = 4
0
2 | a| + | b| = |− 7| + |4| = 7 + 4 = 11
⇒ f ′ ( 0) = k = 2 ⇒ f ′ ( x ) = 2e 2x 1 ≥ ( a 2b )1/ 2
50. (0) Let us first draw the graph of given
⇒ f ′ ( − 3) = 2e − 6 ⇒ a 2b ≤ 1 ⇒ a 6b 3 ≤ 1 equation of lines, as shown in figure.
⇒ | α + β | = | 2 − 6| = 4 ⇒ 84 a b ≤ 84
6 3 Now, the equation of angle bisector of
45. (1024) We have, ∠A is
∴ Maximum value of T4 is 84.
( x − 16) f(2 x ) = 16( x − 1) f( x ) Y
48. (153) Without any restriction
Here f( x ) is divisible by ( x − 16)
x1 + x2 + x3 = 35
[Q f( x ) is polynomial function]
Number of non-negative integral
⇒ f(2 x ) is divisible by ( x − 8)
solution is
⇒ f( x ) is divisible by ( x − 8) 35 + 3 − 1
C3 − 1 = 37
C 2 = 666 X′
A 3x+ X
⇒ f(2 x ) is divisible by ( x − 4) 4y=
0
Now allotting one party 18 seats and 4x B(3,–9/4)
⇒ f( x ) is divisible by ( x − 2 ) +3
distributing 17 seats among 3 parties. y=
0
i.e. f( x ) = ( x − 2 )( x − 4)( x − 8)( x − 16) φ( x ) The number of ways is the C(3,–4)
Putting in the given equation, we get non-negative integral solution of Y′ x=3
φ(2 x ) = φ( x ) = c = 1 y1 + y2 + y3 = 17
⇒ f( x ) = ( x − 2 )( x − 4) ( x − 8) ( x − 16) = 17 + 3 − 1
C3 − 1 = 19
C 2 = 171 3x + 4y 4x + 3y

⇒ f( 0) = 2 × 4 × 8 × 16 = 1024 ∴ Number of ways in which any one 3 + 4
2 2
42 + 32
n
party gets at least 18 seats ⇒ 3 x + 4 y = ± ( 4 x + 3 y)
1 − Π (cos rx )1/ r
46. (6) Let L = lim r=2 = 171 × 3 = 513 ⇒ x=± y
x→ 0 x2 ∴ Number of ways in which no party Clearly, equation of internal bisector is
By using L, Hospital’s rule gets absolute majority = 666 − 513 x=− y
d n = 153
Π (cos rx )1/ r Now, as ( h, k ) lies on the line x = − y,
r=2
lim − dx 49. (11) We have, therefore we get
x→ 0 2x
1 sinα + cos β =
1
… (i) h = − k ⇒h + k = 0
n −1
Π (cos rx )r sin( r x) 2
51. (a) Let the common tangent to the
r=2
= lim 2 a
x→ 0 2x cos α + sinβ = …(ii) parabolas be y = mx + solving it
1 n
3 m
n
with x 2 = 4ay, we get
Π
r=2
(cos rx )r ∑ tan rx Subtract Eq. (ii) from Eq. (i), we get
= lim
r=2
(sinα − sinβ ) − (cos α − cos β )  a
x 2 = 4a  mx + 
x→ 0 2x  m
1 2
n = −
n
1 ∑ tan rx 2 3 ⇒ x 2 − ( 4 am) x −
4a 2
=0
r=2
⇒ lim Π (cos rx )r × lim α +β α − β m
x→ 0 r = 2 x→ 0 2x ⇒ 2 cos sin  
2  2  D = 0 gives m = − 1
1 n
= ∑r α +β α −β 3 −2 So the common tangent is
2 r=2 + 2 sin sin = x+ y+ a=0
2 2 6
1  n( n + 1)  1 2
=  − 1 = ( n + n − 2 ) α − β 52. (c) Solving y 2 = 4ax and x 2 = 4ay,
2  2  4 ⇒ 2 sin  
 2  we get A = ( 4a, 4a )
Since, L = 10
n2 + n − 2  α +β α + β 3 −2 So the point dividing OA internally in
∴ = 10 ⇒ n = 6  cos + sin =
4  2 2  6 (1 + 3 ) : (7 − 3 ) is
47. (84) Let Tr + 1 be the general term in …(iii)  (1 + 3 ) 4a 
the expansion of ( ax1/ 6 + bx −1/ 3 )9  , 
Add Eqs. (i) and (ii), we get
S = (1 + 3 ) + (7 − 3 ) 
∴ Tr = 9C r ( ax1/ 6 )9 − r ( bx −1/ 3 )r  (1 + 3 ) 4a 
+1
α − β  α+β α + β  (1 + 3 ) + (7 − 3 )
9− r 2 cos    cos + sin   
9− r

r
 2   2 2 
= Cra
9 r
b x 6 3
3+2  (1 + 3 ) 1 + 3 
9 − 3r = … (iv) S = a, a
= 9C r a 9 − r b r x 6 6  2 2 
338 JEE Advanced~Practice Set 1

So the equation of the parabola with (i) Number of numbers having 3 zeroes 54. (c) (i) Two of the numbers are zero
focus S and directrix x + y + a = 0 is =9 = 9 × 3 = 27
2 2
 (1 + 3 )   (1 + 3 )  (ii) Number of numbers exactly three (ii) Numbers of number with zero
x − a +  y − a
digits are same with zero excluded excluded
 2   2 
2 = 9C 2 × 2 × 4 = 288 = 9C 2 × 4C 2 = 216
 x + y + a
=  ∴ Required probability
 2 ∴ Required probability
288 + 9 216 + 27
53. (b) Total number of 4-digits number = =
9000 9000
using 0, 1, 2, 3, 4, 5, 6, 7, 8, 9 is
297 33 243 27
9 × 10 3 = 9000 = = = =
9000 1000 9000 1000

Paper 2
1. (a,d) Momentum conservation gives, Also, Z1 + Z 2 = 2 × 0.4 = 0.8 m Given, volume at C = V
(with respect to ground) From above equations, we get and volume at D = 32 V
60 (1 − v ) = 100 v Zmin = 0275
. m For adiabatic expansion CD,
3 Z1 = 0.325 m TC VCγ − 1 = TD VDγ − 1
⇒ v = ms −1 Also,
8 and Z 2 = 0.475 m γ −1 γ −1
TC V   32 V 
So, speed of A with respect to plank, Z1 0.325 ⇒ =  D = 
3 −11 ⇒ = = 0.684 TD  VC   V 
= − 1− = ms −1 Z 2 0.475
8 8 7
−1
 −tR 
= 32 5
Speed of B with respect to ground, 4. (c) i = i 0  1 − e L 
  2
3
= ms −1.  
TC = ( 32 )5 = 4
8  −2 × 10 
10  1 − e 20  Now, efficiency of Carnot’s cycle,
2. (b,c) For hanging block, i =
10   T 1
1
s = ut + at 2   η = 1− D = 1−
2 TC 4
i = (1 − e −1 )
1 3
⇒ 2 = × a × 22 = = 075
.
5. (b) e AB =   area of ∆AOB
2 dB
4
⇒ a = 1 ms −2  dt 
8. (173.20)
So, velocity of hanging block 1  3 
= ( 4) × 4 × × 2 × 2 Fundamental frequency initially,
= velocity of 10 kg block after 2 s. 2  2 
1 T 1 Vg (2000)
v = u + at n1 = =
Total emf induced in the loop 2l µ 2l µ
v = 0 + 1× 2
 1 3 
or v = 2 ms −1 = 3 × 4 × × 4 × × 2 × 2 Fundamental frequency finally,
 2 2 
From work-energy theorem,  1000 
Vg  2000 − 
= 2 × 24 3 = 48 3 V 1  2 
KE of 10 kg block = Work done by n2 =
gravity on hanging block, 6. (a,b,c,d) According to question, 2l µ
1
⇒ × 10 × 2 2 = m × 10 × 2 v +u =D (Here, we applied loss of weight due
2 and v − u = X to upthrust)
⇒ m = 1 kg D+ x D− x 1 Vg × 1500
⇒ v = ,u = =
or weight of hanging block 2 2 2l µ
= 1 × 10 = 10 kg D2 − X 2
and f = ⇒
n1
=
2000
=
4
3. (a,c) Zmin = minimum level of 4D n2 1500 3
rotational profile of liquid. D+ X D− X
m1 = , m2 = n1 × 3
r12ω 2 D− X D+ X ⇒ n2 = = 100 3 Hz
Then, Z1 = Zmin + 2
2g 7. (0.75)
p T1 = 1732
. Hz
. )2(10)2
( 01
= Zmin + B C 9. (r = 0.10)
2 × 10
Adiabatic
r 2ω 2 expansion µ 0Ir 2
and Z 2 = Zmin + 2 We have, B =
2g A D
2( r + x 2 )3/ 2
2

. )2 × (10)2
( 02 V µ 0Ir 2
= Zmin + and so, B =
2 × 10 T2 2( r + 02
2
. 2 )3/ 2
JEE Advanced~Practice Set 1 339

µ 0Ir 2 14. (45.73)


Also, 8B =
2( r + 0.05 )
2 2 3/ 2
In the given situation,
M M′
⇒ 16 ( r 2 + 0.05 2 )3/ 2 = 2( r 2 + 02
. 2 )3/ 2
⇒ 2 2/ 38 2/ 3 ( r 2 + 0.05 2 ) = 2 2/ 3( r 2 + 02
. 2)
R v
⇒ 4r 2 + 4 × 0.05 2 = r 2 + 02
. 2 R\2
0.04 − 0.01
⇒ r2 = ⇒ r = 010
. m
3 By energy conservation law, we have
10. (N = 100
. ) R 1M 2 1 2
MgR − M ′g = v + Iω
Energy of emitted photons, 2 2 4 2
E1 = 5 eV = 8 × 10 −19 J 7 3Mv 2
⇒ MgR =
8 16
Energy emitted by source per 14 × R × g 14 × 1 × 9.8
second = power = 32. × 10 −3 W ⇒ v2 = = = 4573
. units
3 3
Number of photons emitted, 15. (c) (A-pqst, B-q, C-s, D-s)
P
n1 = = 4 × 1015 photons/s A time varying magnetic field will produce a non-conservative
E1 electric field. Due to this electric field, a current starts flowing in
Photons incident per unit area at a distance of 0.8 m, the resistive loop and heat is produced.
n1 4 × 1015 16. (a) (A-pt, B-qst, C-prst, D-q)
n2 = = = 5 × 1014 (per s per m2)
4 πr 2
4 π × ( 0.8)2 Constant velocity means, net acceleration or net force = 0
Number of photons incident on sphere = n3 = n2 A ∴Net force exerted by X and Y = Mg in upward direction
(opposite to its weight Mg). Since, Y is moving with constant
= π ( 8 × 10 −3 )2 × 5 × 1014 = 1011s −1
velocity, some friction is there between X and Y.
So, number of photoelectrons emitted, Therefore, some work is done against friction and mechanical
1 energy of ( X + Y ) is continuously decreasing.
n4 = 6 × n3 = 1 × 10 5 s −1
10 Potential energy of Y is decreasing, but same volume of X
11. (1.20) rises up. Hence, potential energy of X is increasing. Some part
In case of single slit diffraction, a sinθ = nλ of mechanical energy of X + Y is lost in the form of heat in
doing work against viscous forces. Net force on Y in this case
For first minima on one side,
is downwards before Y attains, its terminal velocity.
n=1
After attaining the terminal velocity, net force on Y becomes
∴ asinθ = λ zero. Hence, force on Y from X is Mg upwards. Rest of the
λ 6 × 10 −5 logics are same as discussed in part (s).
or θ≈ = = 3 × 10 −3 rad
a 0.02 17. (d) (A-prs, B-rs, C-pqt, D-rs)
Total angular width of central maxima = 2 × 3 × 10 −3
18. (c) (A-qr, B-p, C-qr, D-q)
= 6 × 10 −3 rad 19. (a, b, c, d)
Linear width = d × θ = 2 × 200 × 6 × 10 −3 = 12
. cm (a) In [MnBr4 ]2 , Mn is present in + 2 oxidation state as:
12. (39.20) x + 4( − 1) = − 2
At distance y above mean position, velocity of block is x= −2 + 4
v =ω A 2 − y2 x=2
As block is detached, its downward acceleration is g. Mn2 + [Z = 25] : [Ar] 3d 5 : 5 unpaired electrons.
So, total height attained by block above mean position is 3d
v2 ω 2 ( A 2 − y2 ) [MnBr4]2–=
h= y+ = y+
2g 2g sp3 hybridisation
dh g
For h to be maximum, = 0 ⇒ y = 2 = 3920 . m In [MnBr4 ]2 − , five unpaired electrons are present, thus it has
dy ω
magnetic moment 5.9 BM.
13. (N = 124
. )
(b) Magnetic moment (µ) = n( n + 2 ) BM
Using, g ′ = g − Rω 2 cos 2 φ [Cr(H 2O)6 ]2 + = Cr 2 + = 3d 4 = 4 unpaired electrons.
We have,
[Fe(H 2O)6 ]2 + = Fe 2 + = 3d 6 = 4 unpaired electrons.
g ′ = 0 = g − Rω 2 cos 2 0°
So, [Cr(H 2O)6 ]2 + and [Fe(H 2O)6 ]2 + has same magnetic
g 9.8 −3 −1
⇒ ω= = = 124
. × 10 rads moment.
R 6400 × 10 3
(c) The IUPAC name of [CoCl(NH 3 )3 (H 2O)2 ] Cl 2 is
chlorodiaquatriammine cobalt (III) chloride.
340 JEE Advanced~Practice Set 1

(d) Benzoyl acetonato beryllium exhibit optical isomerism as follows: 23. (b) At equilibrium A2( g ) + B2( g ) - + 2 AB( g )
4 − x 8− x 2x
Me Me Me Me
2
C O O C C O O C 4x
At equilibrium KC =
4C Be CH HC Be CH ( 4 − x) ( 8 − x)

C O O C C O O C 4 x2
⇒ = 4 ⇒ x 2 = [32 − 2 x + x 2 ]
Ph Ph ( 4 − x) ( 8 − x)
Ph Ph
32 8
x= = = 2.66 mol
20. (a, b) The organic mixture contains (NH 4 )3PO 4 and 12 3
(NH 4 )3 AsO 4 . On reaction with conc. HNO 3 and ammonium 2 × 2.66
[ AB] = ⇒ [ AB] = 0.886 = 0.89 L
molybdate [(NH 4 )2MoO 4 ], yellow precipitate are obtained. The 6
reactions are as follows:
24. (d) Since reactants are combining in 1 : 1 molar ratio,
AsO 34 − + 12(NH 4 )2 M0O 4 + 21 HNO 3
maximum mass of products, would be formed if taken in
→ (NH 4 )3 AsO 4 ⋅ 12MoO 3 ↓ yellow ppt of equimolar amounts. In experiment, increasing mass of
ammonium arseno molybdate Pb(NO 3 )2 (starting from zero), increases mass of products
+ 21 NH 4NO 3 + 12 H 2O first and then will start decreasing.
(NH 4 )3PO 4 + 12 MoO 3 + 6H 2O → 25. (7) CH 2Cl 2, H 2O, CHCl 3, o-cresol, SCl 2, IBr and HCHO (in all
(NH 4 )3PO 4 ⋅ 12MoO 3 ⋅ 6 H 2O ↓ seven) have dipole moment.
Ammonium phosphate molybdate (yellow ppt)
21. (c)
OH OC2H5 OC2H5 OC2H5
26. (159.41) Given, r(Li) = 1.52 × 10− 8 cm
Ac2O+ 4r
C2H5I Sn+HCl AcOH Q a=
3
[H] O
4 × 1.52 × 10 − 8
NO2 NO2 NO2 NHCMe ∴ a= = 3.5 × 10 − 8 cm
p-ethoxy 1.732
acetarnilide Z ×M
ρ=
(A) (B) (C) (D) Na × ( a )3
2 ×7
(b) OH I ∴ ρ= = 0.542 g/cm 3
Excess
6.023 × 10 23 × ( 3.5)3 × 10 − 24
HI HI 100
(B) + C2H5I + C2H5I ⇒ Volume of 100 g Li = = 184.50 cm 3
0.542
NO2 NO2 2 × 39 × 3 3
Density of K =
(G) H 6.023 × 10 23 × 64r 3

(a) 2 × 39 × 3 3
OSO2Ph F = g cm3 = 0.864 g/cm 3
6.023 × 10 23 × 64 × (2.3)3 × 10 − 24
Ph SO2Cl NaF/DMSO
(A) ⇒ Mass of K in 184.50 cm 3 = 184.50 × 0.864 = 159.41 g

NO2 NO2 27. (1.96) For the reaction,


(E) (F ) LiCl ⋅ 3NH 3( s ) - LiCl ⋅ NH 3( s ) + 2NH 3( g )
K p = 18 (atm)2
22. (c) Following reaction takes place with ( − )-gulose.
CHO CHO On reversing the equation, we have
LiCl ⋅ NH 3( s ) + 2 NH 3( g ) - LiCl ⋅ 3 NH 3( s )
H OH H OH
Initial moles 0.2 x 0
H OH Ruff HO H
HO H degradation At equilibrium 0 ( x – 0.4) 0.2
H OH
1
HO OH K p′ =
CH2OH 18(atm)2
CH2OH (+)-xylose Suppose the reaction gas for completion if x moles of NH 3
Ruff degradation
(–)-gulose added.
CHO 1
Thus, at equilibrium = K p′ =
COOH ( p′NH 3 )2
HO H
HO H HNO3
p′NH 3 = 5 atm
H OH
H OH pV 5×8
CH2OH Now, moles of NH 3 at equilibrium = =
COOH RT 0.0821 × 313
(–)-threose
(–)-tartaric ( x − 0.4) = 1.56
acid
x = 1.96 moles
JEE Advanced~Practice Set 1 341
Z ×M
28. (9) The total number of chiral centres are 9. 32. (8.66) Density of solid =
H NA × a 3
where, Z = number of atoms
Me
Me M = mass of atom
H a = side of crystal
NA = Avagadro’s number
HO H H
Then, determine the total density by adding density of P andQ.
H H Let the volume of unit cell = V
29. (3.03) Dish of 20 cm diameter (10 cm = 01 4 × MP 8 × MQ
. m radius) has area dP = ⇒ dQ =
NA V NA V
= πr 2 = π × 0.01 m2
dP MP 60
0.21 × 10 − 18 m2 is the area of 1 molecule = = = 0.3
dQ 2 MQ 2 × 100
π × 0.01
∴ π × 0.01 m2 is the area of molecules dP 2
0.21 × 10 − 18 dQ = = = 6.66 g/cm
0.3 0.3
1.4 × 10 − 4 Total density = d P + d Q = 2 + 6.66 = 8.66 g/cm 3
1.4 × 10 − 4 g stearic acid = N0 molecules.
284
33. (a) I → (q, t), II → (p, r), III → (q, r, t) IV → (q, s)
1.4 × 10 − 4 N0 π × 0.01
= In K 2CrO 4 , Cr has + 6 oxidation state and can easily be
284 0.21 × 10 − 18 manufactured from chromite ore.
N0 = 3.03 × 10 23
l
KMnO 4 is used as Baeyer’s reagent and also act as
oxidising agent in acidic medium.
pV In K 2Cr2O 7, transition element is in + 6 oxidation state. It act
30. (1620.55) Number of moles of H 2 present in 1 litre = l

RT as oxidising agent in acidic medium. It is manufactured


2 ×8 from chromite ore.
= = 0.654 mol
0.0821 × 298 l
In K 2 MnO 4 , Mn is in + 6 oxidation state and manufactured
from pyrolusite.
Thus, energy required to break H H bonds in 0.654 moles of
H2 34. (a)
= 0.654 × 512 = 334.85 kJ O
Now, energy required to excite one H-atom from n = 1 to n = 2 KMnO4
(I) CH3CH2OH [O] CH3CHO + CH3C C C2H5+2NH3
is
1 1
∆E = 13.6  2 − 2 
n
 1 n2 H 3C
N
1 1  + 3H2O
⇒ ∆E = 13.6 −
 1 4  N
H5C2 CH3
H
∆E = 10.2 eV
or ∆E = 10.2 × 1.69 × 10 − 19 J/atom O

Since, one atom absorbs one quantum of energy, Thus, (II) CH3CNH2+NH3 CH3 C NH2+CH3 C CH Br
energy required to excite 0.654 × 2 × 6.023 × 10 23 atoms of H.
O NH C2H5
= 0.654 × 2 × 6.023 × 10 23 × 10.2 × 1.6 × 10 − 19
= 128.570 × 10 4 J = 1285.70 kJ
Thus, total energy needed = 334.85 + 1285.70 kJ = 1620.55 kJ H 3C –HBr +
N H3COC NH2
K –H2O
31. (3.42 × 1020)Q KC = 1 = 1010
K2 CH C
H5C2 N CH3
∴ K1 = K 2 × 1010 H H5C2 N CH3
H Br–
54

Ea −
−3
8.314 × 10 × 298 O O
Also, K 2 = 10 20 e RT = 10 20 e
= 10 20 × 3.42 × 10 − 10 s − 1 = 3.42 × 1010 s − 1 (III) CH3CCl+H3C H3 C – + (CH3CO)2O+H3CCHNH2
ONa
∴K1 = K 2 × 1010 = 3.42 × 10 20 s − 1 C O
Thus, K1 + K 2 = 3.42 × 10 20 + 3.42 × 1010 H5C2
= 3.42 × 1010[1010 + 1] = 3.42 × 10 20 s − 1
AcONH4
342 JEE Advanced~Practice Set 1

H3C H 3C ∴Ionic mass of M 2 + = 50


CH NH –H2O C NH
Hence, Q → (i)
H5C2 C C OCH3
H5C2
C C O l
X − has protons = 17

OH NH2 CH3 Electrons = 17 + 1 = 18


 18 × 111.1
H 3C H 3C Neutrons =  18 +  = 18 + 2 = 20
NH  100 
–H2O N
OH
C Ionic mass of X = P + N = 37
H5C2 N N
CH3 H5C2 CH3 Hence, R → (v)
H H l
N + P = 56
35. (c) Electrons in M = P = Z

CHO Electrons in M 3 + = P – 3
(I) CHO
30.4
Neutrons in M 3 + = ( P − 3) + ( P − 3) ×
100
CHO CHO
N = ( P − 3) + 0.304P − 0.912 + P = 56
P = 26
Thus, S → (ii)
OHC 37. (a,b,d) SinceSP andS ′ P diameter of circleC1 andC 2 respectively.
OHC Clearly,

(II)
T

CHO CHO

P A
B C2 C1 x2 + y2 =1
a2 b2
Me Br Me
S
EtBr NBS
(III) S′
AlCl3 Allylic
x2+y2=a2
Me C
Alc.KOH
π
∴ ∠S ′ QP = ∠SQP =
2
O ⇒ S ′, Q, S are collinear
T is radical centre of C, C1 and C 2
MeCOCl Me Equation of circle C 2
(IV)
AlCl3
NaBH4 C 2 : ( x − a cos θ) ( x + ae ) + ( y − b sin θ) y = 0
C : x 2 + y2 = a 2
OH
Equation of tangent to C 2 and C is
H2SO4 Me a(cos θ − e ) x + b sin θy + a 2(e cos θ − 1) = 0
–H2O
Put x = a cos θ
a 2(cos 2 θ − e cos θ) + b sin θy + a 2(e cos θ − 1) = 0
36. (a) P → (iii); Q → (i); R → (v); S → (ii)
l
N + P = 81 a 2 sin θ
⇒ b sin θy = a 2 sin2 θ ⇒ y =
N = 31.7% more than protons b
 a2 
Then N = ( P + 0.317 P ) ⇒ T =  a cos θ, sin θ
∴( P + 0.317 P + P ) = 81  b 
2.317 P = 81 Equation of chord of contact of T w.r.t
P = 35 ya
x 2 + y 2 = a 2 is x cos θ + sin θ = a
Thus, P → (iii) b
M 2 + = 6 + 16 = 22 electrons x y
l
⇒ cos θ + sin θ = 1 which is same as equation of tangent
a b
M 2 + = 24 protons
to ellipse at P.
Z ( M 2 + ) = 24 ∴ SA and S ′ B both are perpendicular to AB
M = 24 + 2 = 26 ⇒ S ′ B is parallel to SA
JEE Advanced~Practice Set 1 343

 z1  π 15 35
38. (a, c, d) We have, =5+ =
41. (b, c, d) (a) If arg   = , then z1 4 4
x x2 0  z2  2 35 5
= a⇒a = 7
f( x, y) = 1 2 x + y ( x + y)2 and z2 subtend right-angle at 4 4
0 1 2 x + 3y circumcentre origin Area of square = a 2 = (7 )2 = 49
∴ The chord joining z1 and z2 will The coordinate of
Apply C 2 → C 2 − xC1, we get subtend an angle θ at ‘z’ such that  a   7 
π F =  − , a =  − , 7 
x 0 0   3   3 
θ = , if |z| = 1
f( x, y) = 1 x + y ( x + y)2  4 7
 π Hence abscissa of F is − .
0 1 2 x + 3y θ < , if |z| > 1 3
 4
Expand along R1, we get θ > π , if |z| < 1 43. (7) We have, a = x$i + y$j + zk$
 4
f( x, y) = x [( x + y) (2 x + 3 y) − ( x + y)2 ] b = $i − 2 $j + 3k$
f( x, y) = x ( x + y) [2 x + 3 y − x − y] (b) | z1 z2 + z2 z3 + z3 z1|
c = 2 $i + 3$j − k$
1 1 1
f( x, y) = x( x + y) ( x + 2 y) = | z1|| z2|| z3| + + and d = $i − $j + k$
z1 z2 z3
Clearly x, ( x + y) and ( x + 2 y) is a a makes equal angle θ with b and c
factor of f( x, y) = | z1 + z2 + z3| = | z1 + z2 + z3|
∴ a = α( b + c) + β( b × c)
 ( z1 + z2 )( z2 + z3 )( z3 + z1 )
39. (a, d) We have, 3 x − 4 x + 2
3< 0 (c)   Here,
 z1 z2 z3 
⇒ 3x − 3x − x +
2
3< 0 b + c = $i − 2 $j + 3k$ + 2 $i + 3$j − k$
 ( z + z2 )( z2 + z3 )( z3 + z1 )
⇒ ( 3 x − 1) ( x − 3 ) < 0 = 1  = 3$i + $j + 2 k$
 z1. z2. z3 
 1  and
⇒ x∈ , 3  ( z + z2 )( z2 + z3 )( z3 + z1 ) $i $j
 3  ⇒  1 − k$
 z1 z2 z3  b × c = 1 −2 3 = − 7 $i + 7 $j + 7 k$
Since tan A, tan B satisfies this
 ( z1 + z2 )( z2 + z3 )( z3 + z1 ) 2 3 −1
inequalities.   =0
 1   z1 z2 z3 
∴ tan A, tan B ∈  , 3 ∴ a = α ( 3$i + $j + 2 k$ )
 3  Hence,
 ( z + z2 )( z2 + z3 )( z3 + z1 ) + β ′ ( − $i + $j + k$ )
⇒ 30° < A, B < 60° Im  1  =0
 z1 z2 z3  where β ′ = 7β
⇒ 60° < C < 120°
Q a is perpendicular to d.
a + b −c
2 2 2 (d) The triangle formed by joining z1, z3
cosC = and z2 is isosceles and right angle at ∴ a ⋅ d = 0 ⇒ ( α( 3$i + $j + 2 k$ )
2 ab
1 1 z3 + β ′ ( − $i + $j + k$ )) ⋅ ( $i − $j + k$ ) = 0
Since, − < cosC < [Q  z − z1 
2 2 Hence, Re  3  =0 α ( 3 − 1 + 2 ) + β ′ ( − 1 − 1 + 1) = 0
C ∈ ( 60° , 120° )]  z3 − z2  4α − β ′ = 0 ⇒ 4α = β ′
1 a2 + b 2 − c 2 1 λ 15 / 4 ∴ a = α( 3$i + $j + 2 k$ ) +4α ( − $i + $j + k$ )
∴ − < < 42. (a, d) ∆ODE ~ ∆CDA ⇒ =
2 2 ab 2 a 5 a = α ( 3$i + $j + 2 k$ − 4$i + 4$j + 4k$ )
⇒ − ab < a 2 + b 2 − c 2 < ab 3  3 
⇒ λ = a ⇒ E =  0, a  = α ( − $i + 5$j + 6k$ )
4  4 
⇒ a + b + ab > c
2 2 2
a ⋅ b = | a|| b| cos θ
and a 2 + b 2 − ab < c 2 =|α( − $i + 5$j + 6k$ ) |
F B (0, a) C (a, a)
40. (c, d) We have, f( x) = 2 x
4
− 4x2 | ( $i − 2 $j + 3k$ )|cos θ

Let f( x ) = y E(0, λ) = α (1)2 + ( 5)2+ ( 6)2 .


4 2
− 4x
∴ y = 2x ⇒ log 2 y = x 4 − 4 x 2 15 5 (1)2+ (2 )2+ ( 3)2 cos θ
4 D
⇒ x − 4 x + 4 = log 2 y + 4
4 2
7 = ( 1 + 25 + 36 )( 1 + 4 + 9 ) cos θ
7
⇒ ( x − 2 ) = log 2 y + 4
2 2
O A (a, 0) cos 2 θ = = 124 cos 2 θ = 7
124
⇒ x2 = 2 + log 2 y + 4 44. (149) We have,
x2 + x + 1
⇒ x= 2 + log 2 y + 4 = a0 + a1 x + a2 x 2 + ...
Similarly ∆BFE ~ ∆CFA 1− x
⇒ f −1( x ) = 2 + log 2 x + 4 BF BE a/4 ⇒ (1 + x + x 2 )(1 − x ) (1 − x )−2
⇒ = =
sin x + 4 CF AC a = a0 + a1 x + a2 x 2 + ...
Now, g ( x) =
sin x − 2 1
⇒ BF = ( a + BF ) ⇒ BF =
a
⇒ (1 − x ) (1 − x )−2 = a0 + a1 x + a2 x 2 ...
3

sin x − 2 + 6 4 3
g ( x) =  a  ⇒ (1 − x 3 )(1 + 2 x + 3 x 2 + 4 x 3 + ... )
sin x − 2 ⇒ F =  − , a
 3  = a0 + a1 x + a2 x 2 + ...
6
g ( x) = 1 + 2
sin x − 2 3  5 ⇒1 + 2 x + 3 x 2 + 3 x 3 + 3 x 4 + ...
AE = a 2 +  a = a
∴ Range of g ( x ) = [− 5, − 2 ] 4  4 = a0 + a1 x + a2 x 2 + ...
344 JEE Advanced~Practice Set 1

On equating the coefficient of dx dt AC 2 1


⇒ = ( ∆ABC ) =
1
⋅ AC ⋅ BC = = .
x, x 2, x 3, x 4 ... respectively, x t +2 2
2 2 2
we get 1 8 8 So = 25 sq unit.
when x = ,t = − and x = 3, t =
a0 = 1, a1 = 2 , a2 = a3 = a4 = 3 3 3 3
50 8/ 3 51. (b) P. Since in a 3 × 3 determinant,
∑ ar = a1 + a2 + a3 ... a50 ∴I= ∫
cosec t
dt there are 9 elements and each
− 8/ 3 t + 2
r =1 2
element can take two values 1 or − 1.
= 2 + 3 + 3 + ... 49 times ∴ Total number of such determinants,
I = 0 [Q the integral is an odd function]
= 2 + 3 × 49 = 2 + 147 = 149 i.e. n( M ) = 2 9
49. (3) Let the position vector of P,Q, R are
45. (26) Let x = y = 1, then we get Q. The first row can either be filled in
p, q, r, respectively.
3
3f(1) = 2 + ( f(1))2 area of ∆PQR C 3 ways (− 1 at all the places) or 3C1
given =4
⇒ ( f(1))2 − 3f(1) + 2 = 0 ⇒ f(1) = 1, 2 area of ∆OQR ways (− 1 at one place)
But it is given that, f(1) ≠ 1 1 ∴First row can be filled in ( 3C 3 + 3C1 )
|(p − q ) × (r − q )|
∴ f(1) = 2 2 =4 ways. Corresponding to each such
1 1 case the second row can also be filled
Now, put y = , then we get |q × r|
x 2 in ( 3C 3 + 3C1 ) ways. But third row
 1  1 P(p) elements have got to be fixed
f( x ) + f   + f(1) = 2 + f( x ) ⋅ f  
 x  x automatically.
 1  1 ⇒ Number of minus special
⇒ f( x ) + f   = f( x ) ⋅ f  
 x  x determinants
⇒ f( x ) = ± x n + 1 O
= ( 3C 3 + 3C1 ) ( 3C 3 + 3C1 )
Q f( 4) = 17 = 4 × 4 = 16 = 2 4 ways
∴ ± ( 4) + 1 = 17 ⇒ n = 2
n Q(q) R(r) R and S. Clearly on expanding,
Thus, f( x ) = x + 1
2 Also p + λq + µr = 0 determinant = Sum of six terms each
⇒ p = − λq − µr = 0 of which is 1 or − 1.
Hence, f( 5) = 5 2 + 1 = 26
⇒λ + µ + 1= 4 ⇒λ + µ = 3 ∴ Minimum value can’t be less than
3 ! ( 3n − 3)! ( n!)3 − 6 and maximum value can’t be
46. (11) Pn = 50. (25) We have,| z2 + iz1| = | z1| + | z2| greater than 6.
(( n − 1)!)3 ( 3n)!
A(z2) But − 6 is possible only in case only
lim Pn
n→ ∞ when
6( 3n − 3)! ( n)3 (( n − 1)!)3
= lim a1 b 2 c 3 = − 1
(( n − 1)!)3 3n ( 3n − 1) ( 3n − 2 )
n→ ∞
( 3n − 3)! a2 b 3 c1 = − 1
6n 3 2 A(z1) a3 b1 c 2 = − 1
⇒ lim = C(z3)
n → ∞ 3n ( 3n − 1) ( 3n − 2 ) 9 a1 b 3 c 2 = 1
O
a3 b 2 c1 = 1
∴ m + n = 2 + 9 = 11
a2 b1 c 3 = 1
47. (4) We know that,
These equations can’t exist
∆ = s( s − a )( s − b )( s − c ),
a+ b+c simultaneously because if we multiply
where s = all of them we get 1 = − 1, which is a
2
Since, AM ≥ GM contradiction.
s + ( s − a) + ( s − b ) + ( s − c ) ⇒ arg( iz1 ) = arg ( z2 ) Similarly maximum value can’t be 6.
∴ ≥ ⇒ arg( i ) + arg( z1 ) = arg( z2 )
4 Now, we can observed that there is a
( s( s − a )( s − b )( s − c ))1/ 4 π determinant whose value is − 4 and a
⇒ = arg( z2 ) − arg( z1 ) … (i)
4s − ( a + b + c ) s 2 determinant whose value is 4.
⇒ ≥ ∆1/ 2 ⇒ ≥ ∆1/ 2
4 2 z − iz1 −1 1 1
Let z3 = 2 ⇒ (1 − i ) z3 = z2 − iz1
s2 s2 1− i Take ∆1 = 1 1 −1 = − 4
⇒ ≥∆⇒ ≥4
4 ∆ z2 − z3 1 −1 1
⇒ z3 − z2 = i ( z3 − z1 ) ⇒ =i
3 z1 − z3
48. (0) Let I = ∫ cosec  x −  dx
1 1 1 1 −1
x  x π
1/ 3 ⇒ ∠ACB = and| z2 − z3| = | z1 − z3| and ∆ 2 = −1 1 1 =4
1  1 2
put x − = t ⇒  1 + 2  dx = dt 1 −1 1
 x  π
x ⇒ ∠ACB = and AC = BC
2 ∴ Minimum value is − 4 and maximum
 x 2 + 1 dx  1  dx
  = dt ⇒  x +  = dt AB = AC + BC
2 2 2 value is 4.
 x  x  x x Q
52. (a) Clearly, AH = 2 R cos A
2 ⇒ 100 = 2 AC 2 [Q AB = 10]
 1 dx BH = 2 R cos B
⇒ x−  + 2 = dt ⇒ 50 = AC 2 ⇒ AC = BC = 5 2
 x x CH = 2 R cos C
Hence, area
HD = 2 R cos Bcos C
JEE Advanced~Practice Set 1 345

HE = 2 R cos A cos C Y 2 x2 
1

=  x 3/ 2 − 
HF = 2 R cos A cos B
2 √3 3 2 0
A y2=x
x2+y2=12 2 1  1
= − = sq unit
 3 2  6

F E X′ X 54. (b) P. We have,


–2 √3 3
H  x 2, x≥ 0
f1( x ) = x 2sgn( x ) = 
− x , x< 0
2

B
D
C –2 √3  2 x, x > 0
⇒ f1 ′ ( x ) = 
Now, as AH ⋅ BH ⋅ CH = 3
Y′ x=3  − 2 x, x < 0
3 Now, area bounded by the curves S1 ⇒ f1 ′ ( 0 + ) = 0 = f1 ′ ( 0 − )
∴ cos A ⋅ cos B ⋅ cos C = 3
… (i)
8R and S 2 is ⇒ f1 ′ ( 0) = 0
Also, we have AH 2 + BH 2 + CH 2 = 7 3 2 3   2, x> 0
= 2  ∫ x dx + ∫ 12 − x 2 dx  Now, f1 ′ ′ ( x ) = 
∴4R 2 (cos 2 A + cos 2 B + cos 2 C ) = 7  0 3   − 2, x< 0

{ ⇒ f1 ′ ′ ( 0 ) = 2 and f1 ′ ′ ( 0 − ) = − 2
7  2 3/ 2 3 1 +
⇒ cos 2 A + cos 2 B + cos 2 C = =2 [ x ]0 + x (2 3 ) − x 2
2

4R 2  3 2 
So first derivative exists at x = 0 but
… (ii) 2 3 second derivative does not exist there.
 x   
We know, +12 sin−1    Q. We have,
2 3   3 
cos A + cos B + cos C = 1 − 2
2 2 2   x −1/ 3|sin x|, x≠0
f2( x ) = 
2  0, x=0
cos A cos Bcos C =2 [ 3 3 − 0]
 3

7
= 1− 2 ⋅
3 Clearly, lim f2( x ) = lim x −1/ 3 − sin x
x → 0− x → 0−
4R 2 8R 3 1  −1  3 

+ 12 sin−1(1) −  3 3 + 12 sin    sin x
[using Eqs. (i) and (ii)] 2    2    = − lim . lim x 2/ 3
x → 0− x x → 0−
⇒ 4R − 7 R − 3 = 0
3
 3 3 π
= 2 2 3 + 3 π − − 6⋅  =0
⇒ ( R + 1) (2 R + 1) (2 R − 3) = 0 2 3
 Similarly, lim f2( x ) = 0
3
⇒ R =  3  x → 0+
2 = 2 + π ∴f2( x ) is continuous at x = 0
[Q R can’t be negative]  2  x −1/ 3 sin x
Now, f2 ′ ( 0 + ) = lim , which
Now, HD ⋅ HE ⋅ HF = (2 R cos Bcos C ) = 2π + 3 x→ 0 +
x
(2 R cos A ⋅ cos C ) R. Required area does not exist.
(2 R cos A cos B) = Area of circle − Area of square So, f2( x ) is continuous at x = 0 but not
= 8R cos A cos Bcos C
3 2 2 2
= 12 π − 24 differentiable there.
2 R. We have,
 3  Y  0, − 1< x ≤ 0
= 8R 3   [using Eq. (i)]
 8R 3  f3 ( x ) = x 3[− x ] = 
 − x 3
, 0< x< 1
9
=  0, − 1< x < 0
8R 3 ⇒ f3′( x ) = 
− 3 x , 0 < x < 1
2
2 √3 √ 24
53. (c) P. We have,
( x − y) f( x + y) − ( x + y) f( x − y)
X′
O 2 √3
X ⇒ f3 ′ ( 0 + ) = 0 = f3 ′ ( 0 − )
= 4 xy ( x − y )2 2
∴ f3 ′ ( 0) = 0
= ( x − y ) [( x + y) − ( x − y) ]
2 2 2 2  0, − 1< x < 0
Now, f3 ′ ′ ( x ) = 
 − 6 x, 0 < x < 1
= ( x − y) ( x + y)3 − ( x + y) ( x − y)3 Y′ ⇒ f3′′ ( 0 + ) = 0 = f3′′ ( 0 − )
1
On comparing, we get ∴ f3′′ ( 0) = 0
S. Required area = ∫ ( x − x ) dx
f( x + y) = ( x + y)3 0
Thus, second derivative exists at
Y x = 0.
and f( x − y) = ( x − y)3 2x dt
y2=x S. We have,f4 ( x ) = ∫ , f4 ( 0) = 0
Thus, f( x ) = x 3 and f( y) = y 3 x t
(1, 1)
1
Now, ∫ f( x ) dx =
1 3 1 41 1 = [log|t|]2x x
0 ∫ 0
x dx =
4
[ x ]0 =
4 X′
1
X = log|2 x| − log| x|
Q. Clearly, S1 ≡ y 2 − x = 0 … (i) O 2x
= log
and S 2 ≡ x 2 + y 2 = 12 … (ii) x
On solving Eqs. (i) and (ii), we get = log2
x = 3 and y = ± 3 Y′ ∴ It is discontinuous at x = 0.
346 JEE Advanced~Practice Set 2

PRACTICE SET - 2
Paper 1
1. (a, b) 16 5. (a,b)
and that from GH is Q.
Velocity of particle is 17 T = 27 ° C = 300 K
∆p 17 p0
v =
ds
= 3 t 2 − 12 t + 9 Now, Q = = Let m = mass of neon gas in the
dt Xnet 288 X mixture, then
= 3(t −1) (t − 3) If E = pressure at E, then mass of argon = 28 − m
2 p0 − pE = ( 8 X ) Q = ( 8 X ) 
Now, (t −1) (t − 3) is positive when 17 p0  Number of gram moles of neon

(t − 1) and (t −3) both are positive or  288 X  m
= = n1
both are negative, i.e. particle moves 17 55 20
in positive direction when t < 1 or t > 3. ⇒ pE = 2 p0 − p0 = p0
36 36 Number of gram moles of argon
In internal 1< t < 3, particle moves 28 − m
3. (a, c, d) = = n2
backwards.
Given, E 3 − E 2 = 47.22 40
 13.6 Z 2   − 13.6 Z 2  Total pressure of mixture, p = Partial
⇒ − −  = 47.22 pressure of neon + Partial pressure of
 g   4 
t=0 t=1 argon
13.6 Z 2   = 47.22
s=0 s=4 5 n RT n RT
⇒ ∴ p = p1 + p2 = 1 + 2
 36  V V
So, distance travelled by particle in
47.22 × 36 RT
first 5 s = 4 + 4 + 20 = 28 m ⇒ Z2 = ≈ 25 ⇒ Z = 5 = ( n1 + n2 )
13.6 × 5 V
Also, displacement in first 5 s = 20 m.
In ground state, total energy is Substituting the given values, we have
2. (a, b, c)
13.6Z 2  m 28 − m 
From Poiseuille’s equation, we have E1 = − = − 13.6 × 5 2 1 × 10 5 =  + 
n2  20 40 
p − p2 ∆p
Q= 1 = = − 340 eV  8.314 × 300 
8nL X  
So, KE = − E1 = 340 eV  0.02 
πR 4
8nL L and PE = 2 E1 = 2( − 340) = − 680 eV ⇒ m = 4.074 g
where, X = ⇒X ∝ 4
πR 4 R 4. (a, b, c, d) So, mass of neon = 4.074 g
This equation is comparable with Inside a conducting shell, electric field Mass of argon = 28 − m = 23.926 g
∆V
electrical current, I = is always zero. Therefore, option (a) is 6. (a, b, c, d)
R correct. Potential energy of particle is
So, to solve these types of problems, When the two are connected, their U = mVx = 0.01 × 0.08 x 2
we make use of electrical analogy. potentials become the same,
Hence, Q ≡ I, ∆V ≡ ∆p and R ≡ X = 8 × 10 −4 x 2
∴ VA = VB
We can draw equivalent electrical QA QB  1 Q 1 2
= Also, U = kx
circuit as,
or V =  2
RA RB  4 πε0 R 
16X 2 ×U
Since, R A > R B ⇒ Q A > Q B ⇒ k= = 16 × 10 −4 Nm−1
Q 8X 8X x2
2p0 X p0 ∴Option (b) is correct. Angular frequency of particle is
Potential is also equal to
Equivalent resistance values are σR k 16 × 10 −4
V = , VA = VB ω0 = =
L ε0 m 0.01
X ∝ 4
R
∴ σ A R A = σ BR B = 0.4 rads −1
⇒ X AB : XCD : X EF : XGH σA R Total energy of oscillator is
1 1 1 or = B or σ A < σ B
σ B RA 1
E = ma 2ω 20 = 8 × 10 −4
= 2 4 : 2 : 2 : 1 2
4 4
14 ∴Option (c) is correct.
 1  1  1 2 × 8 × 10 −4 2 × 8 × 10−4
      Electric field on surface, ⇒ a2 = =
2 2 2
σ mω 20 0.01 × ( 0.4)2
= 8 : 8 : 16 : 1 E= or E ∝ σ
ε0 = 1 ⇒ a = 1m
As, current is distributed in inverse the
Since, σ A < σ B So, x = a sin (ω 0 t + φ)
ratio of resistances.
Q ∴ EA < EB = sin ( 0.4t + φ)
So, volume flow rate through EF is
17 ∴ Option (d) is correct. or x = a cos (ω 0 t + φ)
= cos ( 0.4t + φ)
JEE Advanced~Practice Set 2 347

7. (0.80) Angular displacement, ′ =0


Along Y-axis, v BY kx = 0.1 N − 0.011 N = 0.089 N …(ii)
θ = 2 πn Now, from Eqs. (i) and (ii), we get From Eqs. (i) and (ii), we get
Speed of particle, v = Rω = at mu cos θ 0.089
v B′ = v BX
′ = ⋅ (1 + e ) x= = 0.89
at dθ (m + M ) 0.1
⇒ ω= =
R dt For the given condition, = 89 × 10 −2 cm
2πn t at
∴ n=2
So, ∫0 dθ = ∫0 R dt ′ = v A′ cos θ and v AX
v AY ′ = v A′ sin θ

v AX
∴ tan θ = …(iii) 12. (46.00)
at 2 ′
v AY
⇒ 2 πn = V2 =
V1
and T1V1γ − 1 = T2V2γ − 1
2R 10
Now, from Eq. (ii), we get
4 πnR γ −1
⇒ t2 = mu cos θ V 
a ′ = eu cos θ −
v AX ⇒ T2 = T1  1 
m+ M  V2 
Now, radial acceleration,
emu cos θ
v 2 a 2t 2 − = 293 × (10)0.4 = 736 K
aR = = (m + M )
R R Work done,
u cos θ
a 2 ⋅ 4 πnR ∴ ′ =
v AX [eM − m] nR(T1 − T2 )
= = 4 πna m+ M W =
Ra γ −1
Q v AY′ = u sin θ
and tangential acceleration, 5 × 8.3 × (293 − 736)
So, from Eq. (iii), we get =
dv 0.4
at = =a v′ u cos θ [eM − m]
dt tan θ = AX = = − 46 kJ

v AY u sin θ ( m + M )
So, total acceleration, and ∆U = ∆Q − ∆W
eM − m
atotal = aR2 + at2 ∴ tan2 θ = = 0 − ( − 46) = 46 kJ
m+ M
= a + ( 4 πna )
2 2 13. (0.11)
Substituting given values, we get
At a point distant x above the bottom,
= a 1 + ( 4 πn)2 e =1
mass of string below x length is
with a = 0.5 ms − 2 and n = 01
. , we get 10. (2.00)
−2 Dimension of B = [M0L0T 0 ]
atotal = 0.8 ms
Dimension of e = [AT]
8. (4.00) According to law of
Dimension of ε0 = [A 2M−1L−3T 4 ] l –x
conservation of energy,
1  K2 Dimension of h = [ML2T −1 ]
mgh = mv 2  1 + 2 
2  R  Dimension of c = [LT −1 ]
1 3 Equating, x
or mg × 3a = mv 2 ×
2 2 n
[AT]
[M0L0T 0 ] = Mx
or v = 2 ga [A 2M−1L−3T 4 ][ML2T −1 ][LT −1 ] µ′ =
−1 l
= 2 10 × 0 . 4 = 4 ms ⇒ [M L T ] = [A n− 2L0T n− 2 ]
0 0 0
Tension at this point is
9. (1.00) Velocity of A will not change ∴ n − 2 = 0 or n = 2 Mgx
T =
along Y-axis. 11. (2.00) Let the spring constant be k. l
′ = u sin θ
So, v AY When the piece is hanging in the air, Speed of wave at point x is
Now, along X-axis, the equilibrium condition gives, T Mgx / l
v ′ + v BX
′ k(1cm) − (0.01 kg) (10 m /s 2 ) v = = = gx
e = AX µ M/l
u cos θ or k(1cm) = 0.1 N …(i) Time taken by wave to travel from top l
⇒ v AX ′ = eu cos θ
′ + v BX …(i) The volume of the copper piece is to x is
0.01 kg 1 l
v′AX v′AY y V = = × 10 −5 m2 l dx  x
9000 kg / m3 9 t1 = ∫x = 2 
gx  g  x
v′BY This is also the volume of water 1
1/ 2
v displaced when the piece is immersed  l  x2
A θ in water. The force of buoyancy (U ) is =2   − 2 
g g
U = weight of the liquid displaced
and time taken by a body to fall freely
B 1
U = × 10 −5 m3 × (1000 kg / m3 ) by distance l − x is
v′BX 9
× (10 m /s 2 ) 2( l − x )
t2 =
U = 0.011 N g
Now, applying momentum If the elongation of the spring is x As t 1 = t 2, we get
conservation along X-axis, when the piece is immersed in water, 2( l − x ) l x
=2 −2
mu cosθ = Mv BX
′ − mv AX
′ …(ii) then the equilibrium condition of the g g g
piece gives
348 JEE Advanced~Practice Set 2

⇒ 2( l − x ) = 2 l − 2 x Then, = v0 −
2 1
v0 = v0
⇒ l − x = 2 l + 2 x − 4 lx New 3 3
⇒ 4 lx = l + 3 x Old orbit 17. (a) Current in capacitor leads voltage
orbit π
⇒ 16lx = l + 9 x + 6lx
2 2
by and current amplitude in lower
2
⇒ 9 x 2 − 10lx + l 2 = 0 Planet V V
branch is = .
l r0 Z XC
⇒ x = l or x =
g π π
r1 200 2 sin  ω t + + 
l  4 2
Clearly, x = ∴ Icapacitor =
g 10
Star  3π 
with l = 1m = 20 2 sin  ω t + 
1  4 
x= = 011
. m r1 1+ e
g ⇒ =ρ=
r0 1− e 18. (a) In upper branch,
14. (0.01) R1 R1
ρ−1 cos θ1 = =
⇒ e = Z1 R 2 + X L2
Before explosion, let mass of star is M ρ+ 1
and mass of planet is m. Then, orbital 3 3
1 = =
velocity of planet is Now, ρ = gives,
2µ − 1 32 + 42 5
GM
v 02 = , where r0 = orbital radius.
r0 1− µ ∴ φ1 = 53°, current lags behind
e =
µ voltage.
Energy of planet immediately after
explosion in this orbit For µ = 0.99,
= KE + PE 1 − 0.99
eccentricity, e = = 0.01
GM  − GM ′ m  0.99 90°
= +   V
2 r0  r0  15. (d)
53°
 where, M ′ = 0.99 16. (a) Friction = µ k mg = ma
 
 M  ⇒ a = µ kg
Energy of planet after a long time In lower branch,
(when its orbital radius is r1) π
GMm  GM ′m  ω0 θ=− = − 90°,
= + −  v0 2
2 r12  r1  current leads the voltage.
2
1 1 v r  So, angle between current is
∴ KE = mv12 = m  0 0  f
2 2  r1  = 53° − ( − 90° ) = 143°
As, m1v1r1 = m0v 0r0 Torque by friction, 19. (a,b)
τ = − Iα = µ k mgR NiCl 2 + 6NH 3( l ) → [Ni(NH 3 )6 ]Cl 2
From energy conservation
(N)
GMm GM ′ m Angular speed,
= − The metal ‘M’ is Ni (Nickel).
2 r0 r0 ω = ω 0 + αt
µ mgR NiCl2 + 2CH3 C NOH
GMmr0 GM ′ m ⇒ ω = ω0 − k ⋅t ...(i) + 2NH4OH
= −
r12 r1 I CH3 C NOH
2µ k g (dmg)
M′ r ⇒ ω = ω0 − ⋅t
Let = µ and 1 − ρ R OH O
M r0
 mR 2 
1 2µ Q I = 2  CH3 C N N C CH3
Then, 1 − 2µ = −
ρ2 ρ   N
⇒ ρ2 (1 − 2µ ) = 1 − 2µρ This is a linear function of time. CH3 C N N C CH3
When pure rolling starts, v = Rω
⇒ (ρ − 1) [ρ(1 − 2µ) + 1] = 0 O OH
v 2µ k g
1 ⇒ = ω0 − ⋅t ...(ii) [O]
So, ρ = 1 and ρ = R R +2 NH4Cl + 2H2O
2µ − 1
µ k gt 2µ k g
⇒ = ω0 − ⋅t 2+
where, ρ = 1 ⇒ r1 = r0, this is condition R R Ni has 8 electrons in its outermost
before explosion Rω 0 shell. with electronic configuration
1 ⇒ t = 3d 8 4s 0
Hence, ρ = 3µ k g
2µ − 1
At this instant, rolling starts. Ni2+ =
Now, if e = eccentricity of orbit, Now, by using v = u + at , we have
then r0 = a (1 − e )  Rω 0 
v = v 0 + (− µ kg )  It is sp3d 2 hybridised forming an outer
and r1 = a (1 + e ) 
 3µ k g  orbital complex.
where, a = semi-major axis of ellipse.
JEE Advanced~Practice Set 2 349

20. (a,b) 23. (a,b,c,d) All given reactions produce 24. (b,c,d)
— The bonds present in interhalogens the desired product The explanations of given statements
(a) O
are essentially covalent because of are as follows:
the small electronegativity difference. OHσ
O C NH2 (a) X reaches end point earlier than Y,
–H+
The melting and boiling points hence X is at lower concentration.
increases as the difference in Thus, option (a) is incorrect.
O
electronegativity increases. (b) Greater change in pH of X at end
σ
— Hydrolysis of interhalogens give O C NH point indicates that X is stronger acid
halide and oxohalide ions. Oxohalide –Br Br–Br than Y.
ion is always formed from the larger O Thus, option (b) is correct.
halogen present. σ
(c) Volume of base required to
— ICl and IBr can be used as O C N H
Slow (rds) –Br – neutralise Y is twice to that required
non-aqueous ionising solvents. O for X.
3ICl - [I 2Cl]+ + [ICl 2 ]− Rearrangement
O C N 1
[X] = = 0.01M,
— ClF3 is a T- shaped molecule with a 100
bond angle of 87° 0′ and bond 2
H2O/ OH
– [Y ] = = 0.02 M
lengths 1.716 Å and 1.621 Å. O N C O 100
21. (a,c,d) Thus, option (c) is correct.
(a) CO 2 is an acidic oxide, H 2O is O NH2 + CO2
(d) Weaker the acid (Y ), stronger its
neutral, CaO is strongly basic and conjugate base. Thus, option (d) is
CuO is weakly basic. Therefore, CO 2 correct.
This reaction is called Hofmann-
has more acidic strength than H 2O. Bromamide reaction. 25. (4) [Ni(CN)4 ]2− , [Pt(Cl)4 ]2−
(b) SiO 2 is an amphoteric oxide. It react O Hybridisation - dsp2
(b)
with acid as well as base.
R′ CON3

R′ C N Shape of molecule = Square planar.
(c) PbI 4 is least stable and has –N2
Slow — PF5 , PCl 5
doubtful existence. Due to inert pair (rds) R′ N C O
effect, the stable oxidation state of Alkyl isocyanate Hybridisation = sp3d
lead is +2. RNH2 Shape of molecule = Trigonal
(d) Pb shows +2 as stable oxidation bipyramidal.
This is curtius rearrangement.
state. Hence, Pb 4 + act as a strong — BrF5 Hybridisation is sp3d 2 and shape
oxidising agent, itself reduced to (c)
of molecule is square pyramidal.
Pb 2 + very easily. R
O SF6, [CrF6 ]3− , [Co(NH 3 )6 ]3 +
22. (b)
O Cl O C N OH Hybridisation - sp3d 2, d 2sp3
S H Shape of molecules = Octahedral
H OH
O O O — CH 4 , NH 4+
σ σ
OH OH Hybridisation : sp3
(S)-(+) sec butano
– H 2O
R′ C N OH R′ C N
Shape of molecules - Tetrahedral.
Pyridine

Acyl nitrile
26. (8) At constant volume and
N temperature for a gas p ∝ W .
R′ N C O Slow 1
H2 O Alkyl isocyanate Given, for N 2, p1 = 3 atm, p2 = atm,
2
R′NH2+CO2
O O w1 = 14 kg
S This is Lossen rearrangement t = 30 mins.
H O reaction. p1 w1 3 14
Q = ⇒ =
(S) (d) R p2 w2 1 w2
O 2
N3H,H+ 14
O COOH ⇒ w2 = kg N 2
– 6
O O
∴Mass of N 2 diffused

R CN3 14 70
DMF SN2 –N2 = 14 − = kg
reaction O 6 6
O O Similarly, for H 2
O R C N
Slow 1
S p1 = 3 atm, p2 = atm w1 = 1 kg
σ 2
O O Acyl nitrile
1
+ H2O ∴ w2 = kg
RNH2 R N C O 6
(R) Alkyl isocyanate
1 5
(Inversion ) ∴Mass of H 2 diffused = 1 − = kg
6 6
350 JEE Advanced~Practice Set 2

According to Graham’s law of ⇒ ∆T =


6236
= 100 K . × 10 −11)
(α × 025
Now, Kb = = 10 −12
diffusion, 7.5 × 8.314 (1 − α )
rN 2 ( MH 2 ) w t ⇒ T = 400 K
= or H 2 × N 2 =
2 ⇒ 2.5 × 10 −12α = 10 −12 (1 − α )
rH 2 ( MN 2 ) wN 2 t H 2 28 Therefore,
1
T2 α = = 0 .2857 = 28.57%
5 In ∆S = nC pln 3.5
T1
6 × 30 = 1 32. (0.5) In the beginning of 4 mins of the
70 t H 14 500
2 = 3 × 2.5R × 2.3 log reaction,
6 400
d [ B] d [C ]
⇒ t = 8 mins ⇒ ∆S = 13.90 J = K1[ A ] , = K 2[ A ]
dt dt
27. (13) As shown in the given figure, 30. (74.93) N2 O 4 - 2NO 2 B 1
Given, =
single unit cell consists of three Initial moles 1 0 C 01.
cubes. When, the reaction is complete, the
Moles at equil. 1 − α 2α
Number ofNi 2 + ions in each cube is 8. 1− α 2α system reaches thermal equilibrium.
∴Number of Ni 2 + ions = 8 × 3 (in three Mole fraction then,
1+ α 1+ α
1 [ B] K [C ] K
cubes) × ( per corner share ) 4α 2 = 1 and = 2
8 ⇒ Kp = p [ A ] K −1 [ A ] K −2
1 −α2 1
= 3 per unit cell.
Number of Ca 2 + ions = 2 per unit cell Also, pM = ρRT ∴
[ B] K 1 / K −1
= = 0.01 = 100
[C ] K 2 / K −2 01. 200
Number of Zr 4 + ions = 1 per unit cell At 288 K,
ρRT 3.62 × 0.082 × 288 0.0005
Number of O 2− ions = 10 (at edge M = = = 85.48
p 1 = 0.5
center of of cube 1) + 8 ( at edge
92 33. (c)
center of cube 2) + 10 ( at edge center ⇒ 85.48 = ⇒ α = 0.076
of cube 3) 1+ α OH OH OH
1 4( 0.076) 2 CHO
= 28 (edge center) × ( per edge center) K p (288 K ) = = 23 × 10 −3 CHCl3
4 1 − ( 0.076)2 +
KOH
= 7 per unit cell At 348 K, Reimer-
Phenol Tiemann reaction (C)
Therefore, formula of the compound . × 0.082 × 348
184
4+ 2+ 2+ 2− M = = 52.5 Steam volatile CHO
can be Zr(1) Ca (2) Ni(3) O(7) as total 1 (B)
positive charge (14) is equal to the 92
⇒ = 52.5 or α = 075
. ‘C’ undergoes intramolecular
total negative charge (14). Thus, the 1+ α
value of a + b + c + d = 13. hydrogen bonding.
. )2
4 ( 075 O H
28. (0.08) For half-cell; K p ( 348 K ) = = 075
.
1 − ( 075
. )2 C O
Ca 2+ + 2e − - Ca Intramolecular H-bond
0.059 1  K ( 348 K ) ∆H
=
E =E° − log Now, log  p H
2 [Ca 2+ ]  K p (288 K )  2.3 × 8.314
= E° +
0.059
log[Ca 2+ ]  348 − 288  So, carbonyl group in this case does
 
2  348 × 288  not react with Schiff’s reagent.
Let the concentration of Ca 2 + in first 51
. ∆H  60  34. (b)
analysis be x M. log =  
23 × 10 −3 19122
.  348 × 288  OH OH
0.059
⇒ − 0.053 = E ° + log x …(i) ∆H = 74.93 kJ CHO CH2OH
2
LiAlH4
When 1.0 mL 0.05 Ca 2 + is added to 31. (28.57) The dissociation of thymol
50 mL, it would add 10 −3 M to x. blue occurs as follow :
0.059 ThOH Th+ + OH −
-
⇒ −0.0422 = E ° + log( x + 10 −3 ) (D)
2 Initial conc. C O O
…(ii) At equil. C (1 − α ) Cα Cα OH
On solving Eq. (i) and (ii) gives [Th+ ][OH − ] COOH
Kb = …(i)
x = 7.55 × 10 −3 [ThOH] [O]

⇒ Original concentration of Ca 2 + in ⇒ K b = [OH − ]


sea water = 10 x = 0.076 M ≈ 0.08 M [Q At 50% dissociation, (E)
29. (13.90) Since, the expansion occurs [Th + ] = [ThOH ] ]
OCOCH3
against a constant external pressure, ⇒ K b = [OH − ] = 10 −12
COOH
pressure of the gas will remain
[Q[H + ] [OH − ] = 10 −14 ] (CH3CO)2O/CH3COOH
constant in the given condition and it
will be equal to the external pressure. Given, [H+ ] = 4 × 10 −3 M
⇒ q p = nC p ∆T 10 −14 F
∴ [OH − ] = . × 10 −11
= 025
or 6236 J = 3 × 2.5 R ∆ T 4 × 10 −3
(Pain reliever, aspirin)
JEE Advanced~Practice Set 2 351

35. (b) P gives iodoform test so, it must If θ is the acute angle between P1 and g ′ ( x ) = a( x a − 1 + x a ) e ax
be methylketone. From the given P2, then f ′( x)
structures (P) can be either (a) or (b). n1 ⋅ n 2 Now, L = lim
cos θ = X→∞ g ′( x)
Here (a) and (b) on reaction with | n1 || n 2 |
MeMgBr followed by cyclisation would xe 2x (1 + 3 x 2 )1/ 2
∴ L = lim
give products (Q) and (R) ( $i + $j ) ⋅ ( $i + 2 $j − k$ ) X→∞ ae ax ( x a − 1 + x a )
=
respectively. It is clear from the 2 6 1/ 2
x 2e 2x  2 + 3
structures of (Q) and (R) that (S) can 1
3 3
be either (b) or (c). On proceeding = = x 
2 6 2 ⇒ L = lim
reverse from (b) or (c), products (R) a ax  1
ax e  + 1
X→∞
and (Q) can be determined. 3 π x 
CHO ∴ θ = cos −1 =
σOH/∆ 2 6 1/ 2
( xe x )2  2 + 3
CH3 1
As, L is contained in P2, θ = 0°
O
Intramolecular
x 
aldol
O 38. (a,d) We have, ⇒ L = lim
x a 1
a ( xe )  + 1
Me Me X→∞
(R) x x
12 x 
f( x ) =   +   − 1
(b) 5
 13   13  The value of L is non-zero finite
O O
x x number.
f ′ ( x ) =   log +   log
σ CH3
5 5 12 12
OH/∆
 13  13  13  13 ( 3)1/ 2 3
Intramolecular CHO ∴ a = 2 ⇒L = =
aldol f ′ ( x ) < 0 ∀ x ∈ R ∴ f( x ) is strictly 2 2
(c) (R′) decreasing for all x ∈ R 41. (a, b, c) We have, f( x) = ax 2 + bx + c
f( x ) = 0
Product S is (b) x x and f(2 + x ) = f(2 − x )
 5
⇒   +   =1  12 
Now, proceed reverse from (R) to get
 13   13  and D> 0
back structure (Q).
∴Vertex of the parabola is  2, −
5 12 D
CHO Let sinθ = , cos θ .
O O3/Red Me  4a 
(Q) 13 13
⇒ sinx θ + cos x θ = 1, ⇒ x = 2 Since, a > 0
(R) ∴Vertex of graph.
∴ f( x ) = 0 has exactly one solution.
The correct structure of P is y = f( x ) lie in IVth quadrant.
39. (c, d) We have the sides of right angle Y
Me
T1 are 20, x and hypotenuse y
O Me ∴ y 2 = x 2 + 400 …(i)
Another triangle T2 sides are 30, x − 5 X′ X
36. (a) and hypotenuse y + 5
Me MeMgBr
∴ ( y + 5)2 = ( x − 5)2 + 900 …(ii)
Y′ (2, –D/4a)
+
H 3O
O Me Solving Eqs. (i) and (ii), we get
(P) (Q) x = 21, y = 29 Clearly f( x ) is decreasing in ( − ∞, 2 ).
CHO ∴ Sides of triangle T1 are 20, 21 ∴ f( 0) > f(1) > f(2 )
O3 O
and 29. and atleast one root must be positive.
1 −D
Area of ∆T1 = × 20 × 21 = 210 = ∆ 1 Minimum value of f( x ) at 2 which is
2 4a
(R)
Radius of incircle i.e. −D
and is negative.
37. (a, c) Given, plane P1 contains the line ∆ 210 4a
P1 = 1 = =6
s1 35 42. (b, c) We have,
r = $i + $j + k$ + λ( $i − $j − k$ ),
Sides of triangle T2 are 30, 16 and 34. x 2 + y 2 − 8 x − 16 y + 60 = 0 …(i)
∴It contains the point $i + $j + k$ and is
Radius of incircle, i.e. Equation of chord of contact from
normal to vector ( $i + $j ).
∆ 240 ( − 2, 0) is
P2 = 2 = =6
Hence, equation of plane P1 is s2 40 − 2 x − 4( x − 2 ) − 8 y + 60 = 0
(r − ( $i + $j + k$ )) ⋅ ( $i + $j ) = 0 ∆ 1 210 ⇒ 3 x + 4 y − 34 = 0 … (ii)
Now, =
or x+ y=2 ∆ 2 240 Solving Eqs. (i) and (ii), we get
Plane P2 contains the line ⇒ 8∆ 1 = 7 ∆ 2  34 − 3 x 
2
x2 +  
r = $i + $j + k$ + λ( $i − $j − k$ ) and the P1 6  4 
and = ⇒ P1 = P2
point $j. P2 6  34 − 3 x 
− 8 x − 16  + 60 = 0
∴The equation of plane is 40. (b, c) We have,  4 
x− 0 y−1 z− 0 x
⇒ 16 x 2 + 1156 − 204 x + 9 x 2 − 128 x
f( x ) = ∫0te (1 + 3 t 2 )1/ 2dt
2t
1− 0 1− 1 1− 0 = 0
− 2176 + 192 x + 960 = 0
1 −1 −1 ⇒ f ′ ( x ) = xe 2x (1 + 3 x 2 )1/ 2
⇒ 5 x 2 − 28 x − 12 = 0
or x + 2y − z = 2 and g ( x ) = x ae ax
352 JEE Advanced~Practice Set 2

2 Now, c = a + 2b − 3(a × b ) 2 π
⇒ ( x − 6) ( 5 x + 2 ) = 0 ⇒ x = 6, − = −2 +
5 ∴ a ⋅ c = |a|2 + 2(a ⋅ b ) − 3 a ⋅ (a × b ) 9| x | 2
⇒ Points are (6, 4),  − ,
2 44  ⇒a ⋅c =1 + 1 = 2 ⇒ x 2 + 5| x | + 3 = 9| x |
.
 5 5 5
Similarly, b ⋅ c = ⇒ x 2 − 4| x | + 3 = 0
43. (510) Given, man has 3 friends. 2
⇒ | x | = 1, 3 ⇒ x = ± 1, ± 3
Now, three cases arise : λ = |(a × b ) × c|
⇒ λ = |(a ⋅ c )b − (b ⋅ c )a| Product of all value of x = 9
He can invites 2 friends three times each.
Let us select first those 2 friends in 5 5
2 49. (33) Total number of cases
3 ⇒ λ = 2b − a ⇒ λ2 = 2b − a = 100
C1 = 100
C 2 ways. 2 2
Now, these two friends each three 25 100
⇒ λ2 = 4|b|2 + |a|2 − 10 a ⋅ b Now, consider x + > 50
6! 4 x
times can be invited on 6 days in
3! 3! 25 ⇒ x 2 + 100 > 50 x
ways. ⇒ λ =4+2
− 5 ⇒ 4λ2 = 21
4 ⇒ x 2 − 50 x > − 100
Thus, total number of ways, 2 friends x
can be invited three times each 47. (1) y = 3
⇒ x − 50 x + 625 > 525
2
x
6! x+ ⇒ ( x − 25)2 > 525
= 3C 2 3
x
3! 3! x+
He can invites all three friends two
3
x ⇒ ( x − 25) − ( 525 )2 > 0
2
x+
times each .... infinity ⇒ ( x − 25 − 525 ) ( x − 25 + 525 ) > 0
6! ⇒ x < 25 − 525 or x > 25 + 525
Number of ways = =
x
2 !2 !2 ! 1 x
x+ ⋅ Since, x is a positive integer and
He can invites one friend three times, x 2/ 3 3
x 525 = 22.91,
one two times and the last only one x+ 3
x we must have x ≤ 2 or x ≥ 48.
time. Number of ways = 3C1 x+
.... infinity Thus, the favourable number of cases
Hence, total number of ways
x x 5/ 3 is 2 + 53 = 55
6! 6! 6! × 6 = ⇒ y=
= 3C 2 × + + = 510 1 x 5/ 3
+ y Hence, the required probability is
3 ! 3 ! 2 !2 !2 ! 3 !2 ! x+ .y 55 11
x 2/ 3 =
[ x ], x ≤ 2
44. (0.25) We have, f( x) = 
100 20
⇒ y 2 + ( x 5/ 3 ) y − x 5/ 3 = 0
 0, x > 2 ∴ m + n = 11 + 20 = 33
2 − ( x 5/ 3 ) ± x10/ 3 + 4 ⋅ x 5/ 3
2 xf( x ) ⇒ y= 50. (1.25) Given, f( x) = ax 2 + bx + c … (i)
I= ∫− 1 2 + f( x + 1)
dx
2
∴ f ( 0) = c … (ii)
0 x×0 x×0
1 x 5/ 3 + x10/ 3 + 4 x 5/ 3 f(1) = a + b + c
⇒ I= ∫− 1 2 + 0
dx + ∫0 2 + 1 dx =−
2
[Q y > 0] … (iii)
and f( − 1) = a − b + c … (iv)
2 x×1 2 x×0 4 x 5/ 3
+ ∫1 2 + 0
dx + ∫ 22 + 0
dx ⇒y=
1–x + + + + + +
2(x 5/ 3
+ x10/ 3 + 4 x 5/ 3 )
1
2
1  x2  1 1 1 [by rationalising]
⇒ I= = 1 −  =
2  2 1 2 2 4 ⇒ y=
2
0
1 + 4 x − 5/ 3 )
x
(1 +
45. (4) We have,
2
λ = log 5(log 5 3) Now, lim y = lim
⇒ 5 λ = log 5 3
x→ ∞ x→ ∞
(1 + 1 + 4 x − 5/ 3 ) x+1 –1

1 2
+ 5− λ −λ λ
= =1
Now, 3k = 3k ⋅ 35 = 3k ⋅ 3 5 1+ 1
1
48. (9) Given, equation can be written as Solving Eqs. (ii), (iii), (iv), we get
= 3 k ⋅ 3log 5 3 = 3 k 3log 3 5 f( − 1) + f(1) − 2 f( 0)
π  2 x + 10 | x | + 4  2 a= ,
= 3k ⋅ 5 − cos −1 cos  2  2
2  x + 5| x | + 3  f(1) − f( −1)
Given, 3 k ⋅ 5 = 405 b= , c = f ( 0)
  2 − 18 | x |   π 2
= cot cot − 1   +
⇒ 3 k = 81 ⇒ 3 k = 3 4 ⇒ k = 4  9 | x |  2 Substituting the values of a, b and c in
 
46. (21) We have, Eq. (i), we get
π 2( x + 5 | x | + 3) − 2
2
⇒ − f( − 1) + f(1) − 2 f( 0) 2
|a| = |b| = 1 2 x 2 + 5| x | + 3 f( x ) =   x
and |a + b| = 3  2 
2 − 18 | x | π
= + f(1) − f( − 1)
⇒ |a + b|2 = | 3|2 9| x | 2 +   x + f ( 0)
 2 
⇒ |a|2 + |b|2 + 2a ⋅ b = 3 π 2
⇒ −2 + 2 ⇒ 2 f( x ) = ( x 2 − x ) f( − 1) + ( x 2 + x )
1 2 x + 5| x | + 3
2 (a ⋅ b ) = 3 − 2 ⇒ a ⋅ b =
2 f(1) + 2(1 − x 2 )f( 0)
JEE Advanced~Practice Set 2 353

∴|2 f( x )| = |( x 2 − x )f( − 1) + ( x 2 + x ) 1 − ( − 1)n ⇒ 4( x 2 − 2 x ) − 9( y 2 − 4 y) = − 4


=
f(1) + 2(1 − x 2 )f( 0)| 2 ⇒ 4( x − 1)2 − 9 ( y − 2 )2 = − 36
Now,| 1 + α k | ⇒ 9( y − 2 )2 − 4( x − 1)2 = 36
≤ | x 2 − x|| f( − 1)| + | x 2 + x || f ( − 1)|
2 kπ 2 kπ kπ
+ |2 (1 − x 2 )|| f( 0)| = 1 + cos + i sin = 2 cos ( y − 2 )2 ( x − 1)2
n n n ⇒ − =1
4 9
≤ | x 2 − x| + | x 2 + x| + 2|1 − x 2| For n = 45,
Eccentricity of the given conic is,
[Q| f( − 1)| ≤ 1, | f( 0) ≤ 1| and| f(1)| ≤ 1] kπ
| 1 + α k| = 2 cos = 2| cos( 4k )| a2 + b 2
= | x|| x − 1| + | x|| x + 1| 45 e =
+2 |1 − x|| 2 + x| a2
44 1 − ( − 1)45
Since, Π (1 + α k ) = =1 4+ 9 13
Q x ∈ [− 1,1], k=1 2 ⇒ e = =
∴ 0 ≤ 1 + x ≤ 2, 0 ≤ 1 − x ≤ 2 and also 4 2
∴2 (cos 4° cos 8° cos 12 °
44

0 ≤ 1 − x2 ≤ 4 54. (d) Shifting the origin to ( h, k ) i.e. to


Kcos 176° ) = 1
Then, 2| f( x )| ≤ − | x| ( x − 1) + | x| (1, − 2 ) the equation of the hyperbola is
⇒ cos 4° cos 8° cos 12 ° K cos 88°
( x + 1) + 2(1 − x 2 ) Y2 X2
= 22
1 − =1 … (i)
4 9
= 2| x| + 2 − 2 x 2 2
n−1 The equation to the chord AB is,
or| f( x )| ≤ | x| + 1 − x 2
∑ | z1 + ω
k
52. (a) We have, z2|2 lX + mY = 1 …(ii)
= − ( x − | x| − 1) = − (| x| − | x| − 1)
2 2 k=0
Homogenising Eq. (i) with the help of
n−1
 
∑ ( z1 + ω Eq. (ii), we get
2
= k
z2 ) ( z1 + ω z2 )
k
= −  | x| −  − − 1 
1 1
 2  4 k=0 Y2 X2
  n−1
− = ( lX + mY )2
4 9
= ∑ ( z1 z1 + (ωω ) z2 z2
2 k
5 
− | x| −  ≤
1 5
⇒ X 2  + l 2  + Y 2  − + m2 
= k=0
1 1
4  2 4 9   4 
+ z1 z2ω k + z1 z2ω k )
5 + 2 lm XY = 0
Hence,| f( x )| ≤ . n−1
4 = ∑ (| z1 | + | z2 |2 )
2
But, given ∠ACB = 90°
51. (b) Given, 1, α1, α 2, α 3, … , α n − 1 are n k=0
∴Coefficient of X 2 + Y 2 = 0
distinct, nth roots of unity. (1 − ω n ) 1 − ωn 1 1
+ ( z2 z1 ) + z1 z2   Hence, + l 2 − + m2 = 0
∴( z − 1) ( z − α1 )( z − α 2 ) ... ( z − α n − 1 ) 1− ω  1− ω  9 4
= zn − 1 = n[| z1 |2 + | z2 |2 ] 5
l + m2 =
2

Putting z = − 1, we get 36
53. (b) We have,
(1 + α1 )(1 + α 2 ) K (1 + α n − 1 ) 4 x 2 − 9 y 2 − 8 x − 36 y + 4 = 0
⇒ 4 x 2 − 8 x − 9 y 2 − 36 y = − 4

Paper 2
1. (c, d) So, energy levels of atoms given are Iav
For a perfect absorber, p =
F⋅L .................................. n = ∞, 0 eV c
As, T =
 πd 2  .................................. n = 4, − 7.65 eV So, force = 3.33 × 10 −10 N
l 
 4  .................................. n = 3, − 13.6 eV 4. (a, b, c, d)
50 × 11
. ×4 .................................. n = 2, − 30.6 eV Current distribution in given network is
⇒Y =
−3 22 91.8 eV
× ( 5 × 10 −4 )2
a
(1.25 × 10 ) × .................................. n =1, − 122.4 eV 2kW
7
So, 90 eV is not sufficient to excite this
= 2.24 × 1011 Nm−2 1mA 1-a C b a +b B
atom, also atom can absorb 91.8 eV 1kW 1kW 1kW
∆Y ∆L ∆l 2 ∆d A D 1mA
and = + + energy.
Y L l d
3. (a, c) 2kW
+ 2 
01
. 0.001 0.001 1–a–b
= +  = 0.0489 For a perfect reflector radiation
110 0125 .  0.05  Now, we can apply KVL by considering
2I
pressure, p = av
Also, maximum error occurs due to c voltages across each resistor.
incorrectness in diameter. Force due to incidence of radiation, Voltage drop in upper 2k Ω resistor
2. (a, c, d) 2 × 100 × 10 −3 × 1 = 2a
pA =
BE = 13.6 Z 2 = 122.4 ⇒ Z 2 = 9 3 × 10 8 Voltage drop in resistor of branch
AC = 1 − a
So, z=3 = 6.67 × 10 −10 N
354 JEE Advanced~Practice Set 2

Voltage drop in resistor of branch Force on loop = Force on its upper arm Then, by conservation of energy,
CD = b πB2vd 2a GMhf
hf2 = hf1 − 2 1
F = BId =
Voltage drop from A to D 16ρ c R
= 1 − a + b = 2a ∆f f2 − f1 GM
If loop reaches its terminal speed, ⇒ = = 2
Voltage drop from C to B then magnetic force is equals to f1 f1 c R
= 2 (1 − a − b ) = b + a + b weight of loop.
So, red shift, ∆f = f ⋅
GM
On solving, we get, 2
πB2vd 2a c 2R
So, ρm 4aπ   g =
d
2
a = and b =
1 2 16 ρc 6.67 × 10 −11 × 2 × 10 30
= 500 nm ×
5 5 9 × 1016 × 7 × 10 8
2 Here, ρm = density of copper and
∴Current in 2k Ω resistors, a = mA ρc = resistivity of copper. ≈ 500 × 10 −9
× 2 × 10 −6m
5
Current in left 1 k Ω resistor, 16 ρcρmg ≈ 1 × 10 −12m = 1 pm
⇒ v =
3 B2
1 − a = mA ∴ N = 1.00
5 7. (2.45)
11. (3.75)
Current in middle 1 k Ω resistor
For lift OFF, mg = v  −
dm 
 Free body diagram of the block is
1  dt 
b = mA shown in figure.
5 dm mg ( 450 + 50) × 9.8
⇒ = = F
Current in right side 1 k Ω resistor, dt v 2 × 10 3
3
a + b = mA = 2.45 kg s −1
5
8. (28.54) a
5. (a, b, c)
When α is a constant, Momentum conservation gives,
dL = α L ∆T 0.012 × 70 = 0.412 × v
w+T
dL ⇒ v = 2.04 ms −1
⇒ = α ∆T In the figure,
L Heat produced = Loss of KE
F = upthrust force
⇒ [log L ]LL2 = [αT ]TT2 1 1
1 1 = 0.012 × (70)2 − ( 0.412 ) × (2.04)2 = Vρ ω ( g + a )
L2 2 2
⇒ log = α (T2 − T1 ) = 28.54 J  mass of block 
L1 =  ρω ( g + a )
 density of block 
9. (8.00)
⇒ L2 = L1 e α (T2 − T1 )
= 
1 
The acceleration is given by  (1000)(10 + 1) = 1375
. N
When α is a function of temperature,  800 
k
L2 T2 a = − ω2 x = − x
∫L
1
dL = ∫T 1
α dT m ∴ w = mg = 10 N
x Equation of motion of the block is
T2 or | amax| = | xmax|
L2 T2 ∫T1 α dT
F − T − w = ma
⇒ log
L1
= ∫T 1
α dT ⇒ L2 = L1e m
i.e. Acceleration will be maximum, . − T − 10 = 1 × 1
1375
6. (b, c, d) when x is maximum. ∴ T = 2.75 N
× × × × i.e. xmax = A = 0.02 m When the string is cut, T = 0
1200 F −w
∴ a= × 0.02 = 8.0 ms −2 a=
3 m
× × × ×
10. (1.00) When photon is away from the . − 10
1375
=
sun’s surface, its frequency increases 1
× × × ×
B=1.2T
from f1 to f2. = 375
. m / s2

B=0 Sun 12. (0.04)


f1 f2 For a bubble,
a
p hf 4T
Mass of photon = = 2 pin = p0 +
emf induced in coil has a magnitude, c c r
E = Bav
PE of photon as it is emitted
Resistance of loop is − GMm r2
ρ( 4a ) 16 ρa (on surface of sun) =
R = = R
2
πd 2
π  
d GMhf1 p2
=− p
2 C 2R
Current in loop is Total energy of photon at surface p1
GMhf
E πBvd 2 = hf1 + G ⋅ PE = hf1 − 2 1
I= = c R r1
R 16 ρc
When photon is away and free from
This current flows clockwise following gravitational pull of sun, its total
Lenz’s law. energy is hf2.
JEE Advanced~Practice Set 2 355
As, r2 < r1 Momentum is conserved, so (R) ln fission, parent nucleus breaks
∴ p2 > p1 4mp − u = 4mp v α + mp v p into all most two equal fragments.
i.e. Pressure inside the smaller bubble (As, mα = 4mp ) (S) ln proton emission, both mass
will be more. The excess pressure, number and atomic number
Also, coefficient of restitution, e = 1
 r − r2  decreases by 1.
p = p2 − p1 = 4T  1  …(i) ∴Velocity of approach = Velocity of
 r1r2  separation 19. (a) The trans effect causes the
introduction of the ligand trans to
This excess pressure acts from ⇒u − 0 = vp − vα
concave to convex side, the interface chloride rather than trans to ammonia
Eliminating u from these equations, in both the cases.
will be concave towards smaller
3 2−
bubble and convex towards larger we get v α = v p  Cl 
8
bubble. Let R be the radius of    +NH 3
interface, then λα h m v Cl Pt Cl  →
Now, = × p p
λp mα v α h    −Cl −
4T
p= …(ii)  Cl 
R 1 8 2
= × = = 0.67  
From Eqs. (i) and (ii), we get 4 3 3 3+
 Cl 
rr
R = 12 =
( 0.004) ( 0.002 ) 15. (d) P—2, Q—3, R—4, S—1    +NH 3
r1 − r2 ( 0.004 − 0.002 ) Cl Pt NH 3  →
For e → i , 45° > θC   −Cl −

= 0.004 m ⇒ sin 45° > sinθc  Cl 
 
So, 10R = 0.04 m ⇒ µ 1 > 2µ 2 2+
 Cl 
13. (450.00) For e → f ,   
Angle of refraction is lesser than angle Cl  Pt NH 3 
600
D
of incidence.   
 NH 3( X ) 
So, µ 2 > µ 1 and µ 2 > µ 3.  
2+
For e → g , µ 1 = µ 2  NH 3 
p(Nm–2)

For e → h, µ 2 < µ 1 < 2µ 2    +Cl −


H N  Pt NH  →
F and µ2 > µ3  3 3
 −NH 3
300 E 
16. (b) [k]= [ML2T−2K −1 ], [η] = [ML−1T−1 ]  
NH
 3

[h] = [ML2T −1 ], [w] = [MLT −3K −1 ] +
G H
V(m3)  NH 3 
0 2.0 5.0 17. (a) In F → G, work done in isothermal    −Cl −
H 3N  Pt Cl  →
W1 = Work done during the process process is
   −NH 3
from D to E (expansion) V   32 V0   NH 3 
nRT ln  f  = 32 p0V0 ln    
= Area of DEHGD  Vi   V0  2+
 NH 3 
W1 =  × 3 × 300 + 3 × 300 J
1 = 32 p0V0 ln 2 5 = 160 p0V0 ln2
∴   
 2  p Cl  Pt Cl 
= 1350 J …(i) 32p0
F   
32T0  
W 2 = Work done during the process NH 3
 
from E to F (compression) (Y )
Iso
Isochoric

the
Ad

= Area EHGF = − 300 × 3 J rm 20. (b,c) The poisonous element ‘M’ may
iab

al
= − 900 J …(ii) be As (Arsenic). On the basis of given
atic

∴Total work done, information.


T0
p0 G Zn
W = 1350 − 900 = 450 J H AsCl 3 + 6H → AsH 3 + 3 HCl
E Isobaric H 2SO 4 (N )
14. (0.67) V
V0 ∆
Given, situation is 32V0 2AsH 3 → 2As + 3H 2 ↑
(M)
1
a 1H In G → E, ∆W = p0∆V = p0( 31V0 )
21. (c)
u = 31 p0V0 O O
Before collision In G → H, work done is less than
σ
31 p0V0, i.e. 24 p0V0 PhCHO + OH
PhCH H+

ln F → H, work done is 36 p0V0 H


va OH
18. (c) (P) In α-decay, mass number
decreases by 4 and atomic number O Ring O
decreases by 2. expansion +
+
(Q) lnβ + -decay, mass number Ph CH Ph
1
vp remains unchanged, while atomic
1H number decreases by 1.
After collision
356 JEE Advanced~Practice Set 2

O O 26. (0.39) Let the mass of Na 2 CO 3 and 28. (8) There are eight chiral centres in the
HOOC K 2CO 3 be x and y g respectively. given structure
[O] –H2O *
Ph ∴ x + y = 122
. … (i)
Ph KMnO4/H
+

For neutralisation reaction of 100 mL *


(B) (A) *
O solution meq. of Na 2 CO 3 + meq. of
O
K 2 CO 3 = meq. of HCl * * * *
CHO x y
O3/Red × 1000 + × 1000 *
2 106 138
CHO HO
2 2
(C) 40 × 01. × 100
= ∗ = Chiral centres
Glyoxal
20 29. (0.48) First, we need to find the
O ⇒ 63( x ) + 53( y) = 7314
. …(ii)
O O OHC molecular formula of A. In anhydrous
On solving Eq. (i) and (ii), we get residue,
+ OHC C C x = 0.53 g, y = 0.69 g C H O
.
018 0.015 0.48
Now, Eq. of BaCO 3 ( ppt.) n:
(D) 12 1 16
2, 3, 7- trioxooctan-1,8-dial
= meq. of Na 2CO 3 + meq. of K 2CO 3 Simple ratio : 1 1 2
( in 20 mL)
Now in hydrated (A), CHO 2 H 2O
= meq. of HCl for 20 mL mixture
22. (b) Anomers differ in configuration of 107
. 0.43
40 × 01
. × 20 n :
anomeric carbon at C 1 position. = =4 45 18
20
23. (a,c,d) When temperature of a Mass of BaCO 3 Simple ratio : 1 1
system drops during a chemical ∴ × 1000 = 4
197 / 2 Emperical formula of A = CHO 2 ⋅ H 2O
reaction, then that process is called
endothermic process, i.e. heat taking Thus, mass of BaCO 3 = 0.394 g As, 1.0 mol of B(CHO 2 ) produces
or heat consuming process. 2 mol of CO 2.
27. (8.23) At anode
∆H = heat of system before the Molecular formula of B = C 2H 2O 4
reaction (larger value) − heat of Co(CN)46 − −→ Co(CN)36− + e − ,
Molecular formula of
system after reaction (smaller value) E °OP = + 0.83 V
= positive. A = C 2H 2O 4 ⋅ H 2O
Adiabatic
At cathode
When A(C 2H 2O 4 2H 2O) reacts with
∆V = O rev. exp
Co 3+ + e − → Co 2+ ,
24. (c) A → B → oxygen.
300K, 2atm 5atm, 750K
E ° RP = + 182
. V C 2H 2O 4 ⋅ 2H 2O + O 2 → 2CO 2 + 3H 2O
Iso, rev. comp
C → A Overall cell reaction 126 g 54g
300 K 300 K,
Co(CN)46 − + Co 3+ - Co(CN)36− 54
2atm 1.12g × 112
. = 0.48 g
C V = 15
. R, C p = 2.5R + Co 2+ 126
5 −2
⇒ γ = and 1 − γ = Nernst equation for this cell is given 30. (5.70)
3 3
as: pH of an acidic buffer is given by
γ −5
⇒ = Ecell = E°cell [Conjugate base]
1−γ 2 pH = − log K a +
Between B and C : −0.0591 [Co 2 + ][Co(CN)36− ] [Acid]
log
γ 1 [Co 3 + ][Co(CN )46 − ] For X buffer solution
pT = constant [Base]
1− γ or Ecell = E°cell H = − log 10 −5 + log
−5 −5 0.0591 [Co 3 + ][Co(CN)46 − ][CN − ]6 [0.5]
⇒ 5(750) 2 = pC ( 300) 2 + log
1 [Co 2 + ][Co(CN)36− ][CN − ]6 ∴[Conjugate base] = 0.05 M
−5
For Y buffer solution
pC = 5
750  2 …(i)
⇒  = 0.5 atm [Base]
 300  Now, 6 CN + Co − 2+
- Co(CN)46 − 6 = − log 10 −5 + log
[5]
Therefore, pC < pA and hence graph [Co(CN)+6 ]
(C) applies most appropriately. ∴ K f2 = = 1019 …(ii) ∴ [Conjugate Base] = 5 M
[Co 2+ ] + [CN − ]6
25. Al, U, S reacts with ClF to produce Cl 2, On mixing the two buffers in equal
Also, 6CN − + Co 3+ - Co(CN)6−3 proportions, the new concentration of
reaction involved are as follows
6ClF + 2Al → 2AlF3 + 3Cl 2 [Co(CN)36− ] NaA in mixed buffer will be
K f2 = …(iii) 0.05 × V + 5 × V 0.05
6ClF + U → UF6 + 3Cl 2 [Co 30 + ][CN − ]6 = M
2V 2
6ClF + S → SF3 + 3Cl 2 On solving Eqs. (i), (ii) and (iii), we have
K New concentration of HA in the mixed
SF4 , Co , SO 2 does not evolve Cl 2 on 0.0591 0.5 × V + 0.5 × V
Ecell = E°cell + log f1 buffer = = 0.5 M
reaction with ClF. Reaction involved 1 K f2 2V
are as follows 1019
ClF + SF4 → SF5Cl or 0 = ( 0.83 + 182. ) + 0.0591 log ∴ pH
K f2 5.05
ClF + Co → CoFCl = − log 10 −5 + log ≈ 570
.
K f2 = 823
. × 10 63 2 × 0.5
ClF + SO 2 → ClSO 2F
JEE Advanced~Practice Set 2 357

31. (0.85) Let the resultant temperature of — H O N == o, σ and π- bonds are C − 2 carbocation is more stable and
mixture is T K. present. In this molecule, oxidation the group X migrates to C − 2 to give
64 number of N = + 3. products.
∴ × 816. × ( 333 − T )
32 It can be increased to +5 and IV. C − 2 carbocation is more stable
128 decreased to −3. It also act as a ligand. than PhMeC + . So, the group X
= × 816
. × (T − 293)
32 34. (a) P →1 ; Q → 2,4 ; R → 3 migrates from C − 1 to form product.
⇒ T = 306.3 K NaCl 36. (a) P → (ii), q → (iii), s → (i), r → (v)
P. C 15H10O 5 + 10%H 2SO 4 →
∆ For P. (i.e. 300 mL of 0.1 M
∆S due to cooling of added CH 3OH (Corncobs)
CH 3COONa + 100 mL of 0.1 M HCl )
T
= nC p ln 2 CH 3COONa + HCl
T1 O Initially millimoles 30 mmol 10 mmol
CHO Final millimoles 20 mmol 0
306.3
= 2 × 816
. × 2.303 × log 200°C Pd/C → CH 3COOH + NaCl
333 0 0
0
= − 13.64 J/K 10 mmol
Since, it is a mixture of weak acid and
∆S due to heat ions of CH 3OH in + CO
a salt of weak acid strong base. Thus,
the vessel O it is an acidic buffer.
306.3
= 4 × 816
. × 2.303 × log
∴ pH = pK a + log 
Q. Salt 
293 CHO  Acid 
= + 14.49 J/K Ribonucleotide
Ribose-5 (CHOH)3 [20 / 400]
⇒ ∆S total = 14.49 − 13.64 J/K reductase ∴ pH = 474 . + log
phosphate [10 / 400]
= 0.85 J/K
CH2OH pH = 474
. + 0.3 = 5.03
32. (0.03) For the reaction, Ribose
For q 0.1M CH 3COOH (pK a = 4 ⋅ 74) +
A n + → A( n + 4 ) + Acid O2
0.1 M NH 4OH (pK b = 4 ⋅ 74)
Intial millimole a 0 weak acid
O CHO O COOH It will form a salt of
( at t = 0) Furfural weak base
At, t = t ( a − x) x 200°C 1
+CO2 QpH = (pK w + pK a + pK b )
According to the given conditions, 2
O
A n + + 2e − → A( n − 2) + As pK a = pK b
(n + 4 ) − ( n − 1) +
A + 5e → A R. HO (CH 2 )4 OH →
PCC
∴ pH = 7 at 298 K.
Let the normality of reducing agent be N. H 2SO 4 For r 0.1 M CH 3COOH (pK a = 4 ⋅ 74) +
OHC (CH 2 )2  CHO → 0.1M CH 3COONa
∴At, t = 0; a × 2 = N × 30
Since it is a mixture of weak acid and
2 a = 30 N ⇒ a = 15 N
salt of weak acid/strong base, thus it
At t = t , ( a − x ) ⋅ 2 + x ⋅ 5 = 42 × N O is an acidic buffer.
3 x = 12N Furan
Henderson equation for acidic buffer
⇒ x =4N 35. (c) is given as
For first order reaction,
pH = pK a + log 
I → p, r, II → p,r III → q,s, IV → q,s Salt 
K =
2.303
log
a
In pinacol-pinacolone rearrangement  Acid 
t a−x reaction OH group is lost as H 2O 0 ⋅ 1
∴pH = 4 ⋅ 74 + log  = 4 ⋅ 74
∴ K =
2.303
log
15N from the C-atom that form a stable  0 ⋅ 1
10 11N carbocation and then groups from
Hence, it is an acidic buffer at
∴ K = 0.0310 min−1 other C-atom migrate to stable
maximum buffer capacity as pH = pK a
carbocation (Ph > H > R).
33. (a) HO  N For s 0.1 M CH 3COOH + 0 ⋅ 1 MHCl
I. Carbocation formed at C − 1 is more
 Since, it is a mixture of weak acid and
stable than 2º benzyl carbocation
N OH strong acid.
PhMeC + . So, Ph group from C − 2
σ and π-bonds are present, cis and Thus, pH of 0.1 MHCl will
migrates to give product.
trans isomer are present due to N== N predominate and pH < 7 at 298 K.
bond. II. Presence of electron donating
• group results in more stable 37. (c, d) (a) ( NT MN )T = NT M T N = NT MN
— 2 NO 2 - N 2O 4 if M is symmetric and is − NT MN if M
carbocation. So, C − 1 carbocation is
σ and π-bonds are present due to more stable due to + R and − I effects is skew symmetric.
dipole-dipole repulsion when (N  N) of OMe group at para. Hence, (Ph)
∴ (a) is TRUE.
bond is formed (N  N) bond length group migrates from C − 2 to give
product. (b) ( MN − NM )T = NT M T − M T NT
is increased.
O O III. OMe group is more electron = NM − MN
N O N releasing than the Me group because = − ( MN − NM )
O O + R effect is more powerful them + I So, ( MN − NM ) is skew symmetric.
σ and π-bonds are present. and hyperconjugative effect. Thus,
∴(b) is TRUE.
358 JEE Advanced~Practice Set 2

(c) ( MN )T = NT M T = NM ≠ MN if M B + C   1
sin   ⇒ min  lim f( x ), lim f( x ) = ≠ f(1)
and N are symmetric. So, MN is not AP AQ  2   x → 1− x → 1+  2
Now, + =
symmetric. BI CI A
sin Point of discontinuity at 1 and 2
∴ (c) is FALSE 2
A max (1, 2 ) = 2 = f(1)
(d) (adj M) (adj N) = adj ( NM ) ≠ adj MN = cot
2 42. (a, b, c, d) The vertices of the ∆ ABC,
∴(d) is FALSE A  1 − 2M 
= 3 , 2  and C  0, 
Q cot 1
38. (a,b,c) A( 0, 2 ), B 
2  l   M
(a) Let f( x ) = e x cos x − 1 and let a and b ∴ ∠A = 60° Y
are any two roots of f( x ). 40. (a, b, c, d) The equation of any line
Then, f( a ) = f( b ) = 0 making an acute angle θ with positive C (0, 1/m)
Now, according to Rolle’s theorem direction of X-axis and passing
there exists c ∈ ( a, b ) such that through the point P( 3, 4) is
y=2
f ′ (c ) = 0 x−3 y−4
= =r …(i) A (0, 2) 1 –2m) ,2
⇒ ec (cos c − sin c ) = 0 cos θ sinθ B
l
where,| r | is the distance of any point
⇒ tan c = 1 X′ X
( x, y) from P. O
Thus, tan x = 1, has a root between
Therefore, A( r cos θ + 3, r sinθ + 4) is lx+my=1
two roots of f( x ) = 0 Y′
a general point on line (i).
(b) Let f( x ) = e sin x − 1, then
x
If A is R, then rcosθ + 3 = 6 Let ( h, k ) be the circumcentre of
f ′ ( x ) = e x (sin x + cos x ) 3 ∆ABC
or r= = 3 sec θ
f ′( x) = 0 cos θ 1 − 2m 1 + 2m
Now, h= ,k=
⇒ tan x = − 1 Since, θ is an acute angle, cosθ > 0. 2l 2m
Thus, tan x = − 1, has a root between Therefore, PR = r = 3sec θ … (ii) Solving these equation, we get
two roots of f( x ) = 0. If A is S, rsinθ + 4 = 8, then 1 k−2
m= ,l=
−x r = 4 cosec θ 2k − 2 2 h ( k − 1)
(c) Let f( x ) = e − cos x
∴ PS = 4 cosec θ Since ( l, m) lies on y 2 = 4ax
⇒ f ′ ( x ) = − e − x + sin x
3 4 ∴ m2 = 4al
Now, f ′ ( x ) = 0 Now, PR + PS = +
cos θ sinθ 2
⇒ e − x = sin x 2( 3 sinθ + 4 cos θ)  1   k −2 
= ⇒   = 4a  
⇒ e x sin x = 1 sin2θ  2k − 2   2 h ( k − 1)
9 16 ⇒ h = 8a ( k 2 − 3k + 2 )
Thus, e x sin x = 1, has a root between and + = cos 2 θ + sin2 θ = 1
( PR )2 ( PS )2
two roots of f( x ) = 0. Locus of ( h, k ) is x = 8a( y 2 − 3 y + 2 )
41. (a, b, d) We have, f :  0,  →  , 3
5 1
39. (a, b, d) In ∆ APB, we have  
2
 y − 3  = 1 ( x + 2 a ) which
 2 2  ⇒  
B  2 8a
AP = AB sin [ x] + 1 [ x] + 1
2 f( x ) = ⇒ f( x ) = represent the equation of parabola
A { x} + 1 x − [ x] + 1
vertex is  − 2 a,  .
3
 1 , 0≤ x< 1  2
x + 1
 2 Q Length of smallest focal chord
f( x ) =  , 1≤ x < 2 = length of latusrectum =
1
Q P  x 8a
C/2
 3 5
B/2 l , 2 ≤ x< From the equation of curve C it is
 x − 1 2
B C clear that it is symmetric about the line
3
A Y y= .
sin 2
BI 2
In ∆AIB, we have = 3
AB cos C 43. (2018) We have,
2 C(α ) is coefficient of x 2018 of (1 + x )α
2
Q ∠AIB = π −  A + B   ∴ C (α ) = αC 2018
  
  2   − y −1
Hence, C ( − y − 1) = C 2018
B C 1
sin cos ⇒ C ( − y − 1)
AP 2 2 1/2
⇒ = ( − y − 1) ( − y − 2 ) ( − y − 3)
BI A X
sin ... ( − y − 2018)
2 1 2 5/2 =
C B 2018 !
sin cos Clearly, f( x ) is discontinuous and
AQ 2 2 ⇒ C ( − y − 1)
Similarly, = bijective function
CI sin
A ( − 1)2018( y + 1)( y + 2 )( y + 3)
1 K ( y + 2018)
2 lim f( x ) = ⇒ lim f( x ) = 2 =
x → 1− 2 x → 1+ 2018 !
JEE Advanced~Practice Set 2 359

⇒ C ( − y − 1) Putting x = y = 0, we get f( 0) = 0 α − x β− y
∴ =
( y + 1)( y + 2 )( y + 3) K ( y + 2018) Putting y = − x, we get 1 −1
=
(2018)! f ( x ) + f ( − x ) = f ( 0) = 0 −2 ( x − y − 2 )
=
1
⇒ f( − x ) = − f( x ) 1+ 1
Now ∫ C ( − y − 1) … (ii)
0
f ( x + h) − f ( x ) α − x β− y
 1 + 1 + 1 + Clearly, f ′ ( x ) = lim ⇒ = =− x+ y+2
  h→ 0 h 1 −1
y+1 y+2 y + 3  dy f ( x + h) + f ( − x ) ⇒ α = y + 2, β = x − 2
 ... +
1
 = lim [using Eq. (ii)]
h→ 0 h
 y + 2018  ∴ ( x, y) = (β + 2, α − 2 )
1 ( y + 1)( y + 2 ) K ( y + 2018)  x+ h− x  ( x − 4)2 ( y − 3)2
=∫ f  ∴ + =1
0 2018 !  1 − ( x + h) ( − x )  16 9
= lim
 1 1 1  h→ 0 h (β + 2 − 4)2 (α − 2 − 3)2
 + + ... +  dy ⇒ + =1
y+1 y+2 y + 2018  [using Eq. (i)] 16 9
1 1  h  (β − 2 )2 (α − 5)2
=
2018 ! ∫ 0
[( y + 2 )( y + 3) f 
 1 + x( x + h)
 ⇒
16
+
9
=1
= lim
... ( y + 2018) + ( y + 1) h→ 0 h ⇒ 16α 2 + 9β 2 − 160α − 36β + 290 = 0
( y + 3) ... ( y + 2018)  h  Equation of reflection of ellipse is
f  
+ .... + ( y + 1)( y + 2 )....  1 + x( x + h) 16 x 2 + 9 y 2 − 160 x − 36 y + 292 = 0
= lim
( y + 2017 )] dy h→ 0 h
1 1 × (1 + x ( x + h)) ∴ λ = − 160, µ = 290
1 + x ( x + h)
=
2018 ! ∫ 0
d [( y + 1)( y + 2 )( y + 3)
∴ λ + u = − 160 + 290 = 132
 h 
f   1 − 3
K ( y + 2018)]dy  1 + x( x + h) 49. (12) We have, A =  
1 = lim −
 1 1
= [( y + 1)( y + 2 )( y + 3) h→ 0 h
2018 ! n
1 + x( x + h)
A + A + ...  − 
1 2 1 3 1
... ( y + 2018)]10 and A −
1 3 9  3
1 ⋅ lim
= [(2 ⋅ 3 ⋅ 4 ... 2019) h → 0 1 + x ( x + h)
3 1 9
2018 ! A n − 1 + ... ∞ =
1 b 1
=2 × 13 
− (1 ⋅ 2 ⋅ 3. ... 2018)]
1 + x2 1 2 1 3
=
1
(2019 ! − 2018 !] Let B = A − A + A + ... ∞ … (i)
2 3 9
2018 ! ⇒ f ′( x) =
= 2019 − 1 = 2018 1 + x2 A
Premultiplied by − , we get
2 3
44. (1) If| z| = | z − 1|, then| z|2 = | z − 1|2 ∴ f ′ (1) = =1
2 AB A2 1 1 4
− =− + A3 − A + ...
⇒ zz = ( z − 1) ( z − 1) 3 3 9 27
47. (5) We have,
⇒ z+ z=1 AB
r = (a × b ) sin x + (b × c ) ⇒− =B− A [from Eq. (i)]
and if| z| = | z + 1|, then| z|2 = | z + 1|2 3
cos y + 2 (c × a )
AB
⇒ zz = ( z + 1) ( z + 1) ⇒ r ⋅ a = [b c a ]cos y ⇒ A=B+
3
⇒ z+ z= −1 ⇒ r ⋅ b = 2 [c a b ]
A = B  I + 
A
Thus, | z + z| = 1 ⇒
⇒ r ⋅ c = (a b c ) sin x  3
Given, r ⋅ (a + b + c ) = 0
45. (3) We have, ⇒ B = 3 ( 3I + A )−1 A
sec 2 ( a + 2 ) x + a 2 − 1 = 0 ∴ [a bc ] [cos y + 2 + sin x ] = 0 −1
  3 0  1 −3 
⇒ sec 2( a + 2 ) x − 1 + a 2 = 0 Since, a , b, c are non-zero and ⇒ B=3 + 
non-coplanar.   0 3  −1 1  
⇒ tan2( a + 2 ) x + a 2 = 0
∴ sin x + cos y + 2 = 0  1 −3
⇒ tan2( a + 2 ) x + a 2 = 0, if and only if  −1 1 
⇒ sin x + cos y = − 2  
tan2( a + 2 ) x = 0 and a 2 = 0 It is possible when sin x = cos y = − 1 −1
 4 −3  1 −3
a=0 π ⇒ B=3
Now,
∴ x=− ,y= π  −1 4   −1 1 
   
∴ tan2(2 x ) = 0 ⇒ tan2 x = 0 2
3  4 3  1 −3
π π ∴ Minimum value of ⇒ B=
⇒ x = 0, − , 13  1 4  −1 1 
2 2 4 4  π2 
( x 2 + y2 ) = 2  + π 2 3  1 −9
∴ Number of ordered pairs are π 2
π  4  ⇒ B=
π π 13  −3 1 
( 0, 0),  0, −  and  0,  . 4 5π 2
 2  2 = × =5 3  1 −9 3  1 0
π2 4 Now, =
46. (1) We have, 13  −3 1  13  b 1
48. (132) Let image of ( x, y) on ellipse
 x + y Equating, we get a = − 9, b = − 3
f( x ) + f( y) = f   … (i) about the line
 1 − xy  ∴ | a + b| = |− 9 − 3 | = 12
x − y − 2 = 0 is (α, β)
360 JEE Advanced~Practice Set 2

50. (3) We have, Hence, g ( x ) is non-differentiable at Q. We know, G1 ⋅ G 2 .... G m = (G )m


 1 1
x = − 1, − and Q G1G 2, .... G m = 2 28
 − 2, − 2 ≤ x<− 1 2 2
 1 ∴ ( 1 × 256 )m = 2 28
f( x ) =  − 1, − 1≤ x ≤ − 51. (c) The given digits are 8, 7, 6, 4, 2, x
 2 ⇒ (16)m = 2 28
and y. We know that a number is
2 x 2 − 1, − 1 < x ≤ 2
 2 divisible by 3, if sum of its digits is ⇒ 2 4 m = 2 28 ⇒ m = 7
divisible by 3.
 1

Here, m = 7, therefore r = 
 2, −2 ≤ x< −1 ∴ 27 + x + y, will be divisible by 3. 256  8
 =2
 1 Now, as the digits are distinct,  1 
⇒ | f( x )| =  1, − 1≤ x ≤ −
 2 therefore x and y can takes values and so, G1 + G 2 + ... + G 7
|2 x 2 − 1|, 1 from 0, 1, 3, 5, 9.
− < x≤2 = 2 + 2 2 + ... + 2 7
 2 Possible pairs are (0, 3), (0, 9), (1, 5),
 2, − 2 ≤ x<− 1 (2 7 − 1)
(3, 0), (3, 9), (5, 1), (9, 0), (9, 3). =2 = 2 8 − 2 = 254
 1 ∴ Maximum value of x + y is 2 −1
 1, − 1≤ x ≤ −
 2 9 + 3 = 12 S. Clearly, common difference (d ) of
= 1 1
1 − 2 x 2, − < x≤ Minimum value of x + y is 3 + 0 = 3 A1, A 2,..., A n
 2 2
 2 Maximum value of x − y is 9 − 0 = 9 b − a 971 − ( − 1) 972
1 = = = =4
2 x − 1, < x≤2 n+1 242 + 1 243
 2 Minimum value of xy is 0
∴ Common difference of
 52. (b) We have, y = e − x cos x
 − 2, − 2 ≤ | x| < − 1 A1, A 3, A 5, ... A n − 1 is 8
 1 Clearly,
54. (c) P. f( x) = 2 cos 2 x + sin x − 8
and f(| x|) =  − 1, − 1 ≤ | x| ≤ −
 2 y1 = − e − x cos x − e − x sin x
f( x ) = 2 − 2 sin2 x + sin x − 8
2| x|2 − 1, − 1 < | x| ≤ 2 π
 2 = − 2e −x
cos  x −  f( x ) = − 2 sin2 x + sin x − 6
 4
= 2 | x |2 − 1,| x | ≤ 2 2
π
⇒ y2 = − 2  − e − x cos  x −  f( x ) = − 2  sin x −  −
1 47
= 2 x − 1, − 2 ≤ x ≤ 2
2
  4  4 8
Now, g ( x ) = | f( x )| + f(| x |) π 47
− e − x sin x −   Maximum value of f( x ) is − at
2 x 2 + 1, − 2 ≤ x < − 1  4   8
 1 sin x =
1
2
− 1≤ x ≤ − π
 2x ,
2 = + ( 2 )2e − x cos  x −  4
  2
= 1 1 Minimum value of f( x ) is − 9 at
0, − < x≤
 3π 
 2
2 2 ⇒ y3 = − ( 2 )3e − x cos  x −  sin x = − 1
1  4 
 4 x − 2, < x≤2 ∴Integer in its range is
 2 ⇒ y4 = + ( 2 )4 e − x cos( x − π ) { − 9, − 8, − 7, − 6} = 4
Clearly, g ( − 1− ) = lim (2 x 2 + 1) = 3,
x → −1− = − 4 e − x cos x Q. f( x ) = sin2 x + 3 cos 2 x + 5

∴ g ( − 1+ ) = lim 2 x 2 = 2 , ⇒ y4 + 4 y = 0 … (i) f( x ) = 2 cos 2 x + 6


+
x → −1
∴ k4 = 4 Maximum and minimum value of f( x )
 1−  1 Differentiating Eq. (i) 4 times, we get is 8 and 6, respectively.
⇒ g  −  = lim 2x = ,2

 2  1− 2 y8 + 4 y4 = 0 The integers in range is {6, 7, 8} = 3


x→ −
2
⇒ y8 + 4 ( − 4 y) = 0 [using Eq. (i)] R. f( x ) = 4 sin x cos x − sin2 x + 3 cos 2 x
 1+
⇒ g  −  = lim 0 = 0, ⇒ y8 − 16 y = 0 … (ii) = 2 sin2 x + cos 2 x − sin2 x + 2 cos 2 x
 2  1+
x→ − ∴ k 8 = − 16 = 2 sin2 x + cos 2 x + 1 + cos 2 x
2

1  − Now, differentiating Eq. (i) 8 times, = 1 + 2 (sin2 x + cos 2 x )


⇒ g   =0 we get Maximum value of f( x ) is
 2 y12 + 4 y8 = 0 1 + 2 2 = 1 + 2.28 = 3.28
 1+  ⇒ y12 + 4(16) y = 0 Minimum value of f( x ) is
and g   = lim 4 x2 − 2 = 0
 2 1 + ⇒ y12 + 64 y = 0 [using Eq. (ii)) 1 − 2 2 = 1 − 2.28
x→
2 ∴ k12 = 64 = − 1.28
∴ g ( x ) is discontinuous at x = − 1, −
1 Similarly, we get y16 − 256 y = 0 The integer in its range is
2 ∴ k16 = − 256 { − 1, 0, 1, 2, 3} = 5
1
and continuous at x = 53. (c) P. We know, S. f ( x ) = cos(sin x ) + sin(cos x )
2
π
 1 −
A1 + A 2 + A 3 + ... + A n = nA = 2 sin  + sin x 
Now, g ′   = 0 Q A1 + A 2 + ... + A n = 970 × 121 4 
 2 
 − 1 + 971 Maximum value of f( x ) = 141
.
∴ n  = 970 × 121
 1 +  1   2  Minimum value of f( x ) = − 0.301
and g ′   = 8 .
 2   2 ⇒ n = 242 The integer in its range is { 0, 1} = 2
PRACTICE SET - 3
Paper 1
1. (a, b, d) Given situation is 5. (a, b) When piston is released, it stops Fundamental frequency of vibration
when pressure in both chambers A (ω / 2 π )
= = 20 Hz
C Object P and B is same. 5
P′ O
Let final pressure is p′ and volumes 7. (a, d) (a) At equilibrium, velocity of
f/3 F
Image are VA and VB, then particle is ‘u 0, so at time t , speed of
A B particle is u = u 0 cosωt
For point P, p, 5 V When u = 0.5 u 0, 0.5 u 0 = u 0 cos ωt
8 p, V
u = −  2 f −  = −
f 5f

Initially
π 2π π T
 3 3 ⇒ ωt = ⇒ t = ⇒t =
3 T 3 6
1 1 1 1 3 1
By + = , we get − = T T 2π m
v u f v 5f −f (b) t = t AB + t BA = + =
− 5f 5f 6 6 3 k
⇒v = or OP ′ = .
2 2 A B T T T 7π m
(c) t = + + =
p′, VA p′, VB 6 6 4 6 k
Length of image = OP ′ − OC

Finally
5f f T T T T 5π m
= − 2f = (d) t = + + + =
2 2 6 6 4 4 3 k
− f /2
∴Magnification, m = = − 1.5, 8. (7) Let’s divide one complete
f/3
oscillation into 8 equal parts,
so image is inverted. For adiabatic processes in A and B 6/8 7/8 1 1/8 2/8
2. (a, d) Number of α-particles emitted, part, we have p( 5V )γ = p′ VAγ
238 − 206 8 p( V )γ = p′⋅VBγ
Nα = =8 and
5/8 4/8 3/8
4
Dividing Eq. (i) by Eq. (ii), we get
Number of β-particles emitted, γ Now, when displacement of particle is
Nβ = 82 + 8 × 2 − 92 = 6 5 γ  VA  V 5 A A T
=  ⇒ A = 1 , = A sinωt ⇒t = 0
3. (b, d) Centripetal acceleration, 8  VB  VB 2 2 12

ac = k 2rt 2 where, T0 = time period of oscillation
VA 5 VA 5 T
2 ⇒ = 2
or = and for x = A, A = A sinωt , t = .
v VB V 4
⇒ = k 2rt 2 ⇒ v = krt (2 3 )3
B 4
r
Hence, when particle covers   th
3
dv Also, VA + VB = 6V
Tangential acceleration, at = = kr  8
dt 10
Hence, VA = V oscillation from an extreme position in
Total acceleration is a = ac + at , so 3
total force on the particle is time T,
10 Extreme Mean Extreme
5V − V
F = Fc2 + Ft 2 = m2k 4 r 2t 4 + m2k 2r 2 V − VA final 3 position position position
Now, A initial =
VA initial 5V
= mkr 1 + k 2t 4
5V 1 V − VB initial
Only tangential force does work on the = = and B final T0 T0 T0
15V 3 VB initial
particle. 12 12 6
8
∴Power = Ft ⋅ v = ( mkr ) ⋅ ( krt ) = mk 2r 2t V −V T0 T T
5 ∴Time taken, T = + 0 + 0
= 3 =
4. (b, c, d) As sphere falls, viscous force V 3 12 12 6
increases with its speed and when 4T0 T0
6. (b, c) From given equation, = =
terminal speed is reached, Fnet = 0. 2π
12 3
k= = 62. 8 m− 1
⇒ anet = 0 and when particle covers   th of
5
λ
 8
Also, at terminal speed, weight 2π
⇒ λ= = 01 . m oscillation from mean position, then
= upthrust + viscous force 62.8
time taken is
4
mg = πr 3ρg + 6 πηrv 5λ
So, length of string, l = = 0.25 m T T T T T
3 2 T ′ = 0 + 0 + 0 + 0 + 0 = 7T0 /12
4 12 6 6 12 12
50 × 10 × 9. 8 = π × (10 − 3 )3
−3
Extreme Mean Extreme
3
P
× 1260 × 9. 8 + 6 π( 0. 8) (10 − 3 ) vT
l
⇒ vT = 32 . 5 ms − 1
Mid-point of string is antinode, therefore 7 7
So, sphere’s maximum speed is ∴T ′ = × 3T = T
32.5 ms − 1. its displacement is 0.01 m. 12 4
362 JEE Advanced~Practice Set 3

9. (1) For 1 mole of ideal gas, pV = RT Hence, frequency observed is From free body diagram of first block,
increased due to motion of source
RT  V 3
m1
⇒ p= = p0  1 − α 3  and observer also.
V  V0  F f21
Observed frequency is
p0  αV 
4
 v + vO  6 − f21 = m1a
⇒ T = V − V 3  … (i) f = f0  S 
R  0   vS − vS  ⇒ f21 = 5N
From free body diagram of second
Temperature maxima occurs, when  340 + 20 
= 400 ×   = 450 Hz block,
dT p  4αV 3   340 − 20 
= 0 or 0 1 − =0 f21 f32
dV R  V03  m2
∴f / 90 = 5
V03
⇒ V03 = 4αV 3 or V 3 = … (ii) 12. ( 9) As light ray crosses
5 − f32 = m2a

Also, given that, ⇒ f32 = 5 − 2 = 3 N
3 p0V0 15. (c)
Tmax = … (iii)
4 R F = 13 N m3 m2
m1
So, from Eqs. (i), (ii) and (iii), we get
3 p0V0 p f
= 0
4 R R Friction force, f = µ( m1 + m2 + m3 )g
 
 V0 αV04  = 02
. × 6 × 10 = 12 N
 1/ 3 1/ 3 − 
4 α 4 ∴Fnet = 13 − 12 = 1N
  lens two times, focal length F of given
 4 3 .α 4 / 3 ⋅ V03  1
system is given by asystem = = 0167
. ms − 2
3 1 1 6
⇒ = 1 − 4 1 2 1
4 = − From free body diagram of first block,
4 3 α 1/ 3 4 3 . α 1/ 3 F fl fm
F f13
⇒ = 1 1  1 − 
3 1 1

1  2( n − 1) 1 m3
= 2  −
4  4 F  R   R
4 3α 3 −  f
 2
1
1 1 R 40 13 − 3 × 0.2 − f13 = 0;∴f13 = 12.4 N
⇒α 3 = ⇒α = ⇒ F = =
41/ 3 4 2(2 n − 1)  3×2  In last case, Fnet = 12 − 12 = 0
2 − 1
So, 4α = 1  2  So, anet = 0
10. (5) For refraction at first surface, = 10 cm F m3
f23
 3 − 1 ∴F − 1 = 9 cm
3/2 1  
= = 2  13. (a) F − f23 = 0 ⇒ f23 = F = 12 N
v1 ( − 20)  5  F m3 m2
  m1
16. (a)
6 17. (b)
a= = 1 ms − 2
6 18. (d) We use, number of decayed
N0
O P1 P2 I2 I1 From free body diagram of first block atoms is N =
5 cm (from left), 2n
t
R1=5cm F f13 ∴ n=
R2=5cm m3 T 1/ 2
So, v1 = 30 cm Number of undecayed atoms,
F − f13 = m3a N ′ = N0 − N
Now, refraction occurs at second
surface and for second surface, ⇒ f13 = 6 − 3 = 3N Activity, A = A 0e − λt
image of first acts like object for From free body diagram of second
block, 19. (b) Trioxalatochromate (III) ion and
second.
1 3/2 1− 3/2 trans-diammine dichlorobisethylene
∴ − = f13
m1
f21 diammine chromium (I) are optically
v 25 −5
active.
1 3 1 50
⇒ − = or v = = 6.25 cm 3 − f21 = 1 ⇒ f21 = 2N (a) Trioxalatochromate (III) ion
v 50 10 8 [Cr(C2O 4 ) 3 ] 3−
4 14. (b)
∴ d =5 3– 3–
5 F m2 m3 C2O4
C2O4
m1
11. (5) As both O and S are oscillating
simultaneously, they are moving C2O4 Cr Cr C2O4
F
towards each other at maximum asystem =
m1 + m2 + m3
speeds when they are crossing mean C2O4 C2O4
6
positions. So, their speeds are = = 1 ms − 2
1+ 2 + 3 Non-superimposable mirror images
v = Aω = 0. 5 × 40 = 20 ms − 1
JEE Advanced~Practice Set 3 363

(b) trans-dichlorobisethylenediammine (C) 2Al 2O 3 + 3C → 4Al + 3CO 2 y=


1
= 0.5
platinum (II) chloride The equation(C) represents 2
Cl Cl
electrolysis of alumina which is done  x − 1
 
by using electrolytic cell that contain  2 
k eq = … (ii)
en Pt en en Pt en steel cathode and graphite anode. (2) (3)2
23. (a) On equating Eq. (i) and (ii), we have
Br2
Cl Cl (Major
Superimposable mirror images
(a) UV x = 4M
Br product)
(+) 3
– 27. (8) ∆E = × 0.85 eV
(c) trans-diamminedichlorobisethylene 4
diamminechromium (I) H⊕ H 2O
Photon will be in Brackett series
Cl Cl (b) H 2O ⊕ OH (Q 0.31 ≤ E ≤ 0.85) for Brackett
NH2 H2N

0.85  1 −  = 13.6  2 − 2 
en Cr Cr en O OH 1 1 1
 4 4 n 
NH2 H2N C—H C—CN
 4 
2
0.85  1 −  =  1 −   
Cl Cl (c) + HCN 1 13.6
Non-superimposable mirror images H  4 16   n  

20. (b) 4 1
CH3 O ⇒ = ⇒ n=8
n 2
CH3—CH—CH2—C—CH3 C—NH2 NH2 Hence, n = 8
Br2 + KOH
Methane (d)
CH3 CH3 unit 28. (3) The transformation of KMnO 4 to
Achiral MnO 2 occurs in acidic medium as
Derived, name of iso-octane is follows:
iso-butyltrimethyl methane. 24. (a,b,d)
Crystal structure of diamond and 4KMnO 4 + 4H + → 4MnO 2 + 3O 2
21. (a,b,d) 3 A (g) → 2 B( g ) + C ( g )
l

corrundum is same. + 2H2O


t = 0; p0 0 0 l
Carbogen is a mixture of 5-10% Oxidation state of Mn in KMnO 4
2x 2x CO 2 and O 2 which is used in
t = 20; p0 − x artificial respiration of pneumonia x − 8 + 1 = 0, x = 7
3 3
patients. Oxidation state of Mn in MnO 2
2 p0 2 p0
t =∞ l
PbO is known as litharge which has x − 4 = 0, x = + 4
3 3 yellow orange colour.
4P0 l
SnCl 2 is a powerful reducing agent Change in oxidation number
⇒ =4
3 and is used to reduce nitrobenzene =7− 4= 3
into aniline.
∴ p0 = 3 atm 29. (4) Zieglar-Natta catalyst is a mixture
x NO2 NH2 of (C2H5 ) 3 Al + TiCl 4 . Therefore,
p0 + = 3.5 oxidation state of Ti is +4.
3
SnCl2
x = 1.5 30. (2) pH at the 2nd and point of titration
is given as
⇒ t 50% = 20 min is the half-life.
pk 2 + pk 3
t 75% = 2 × 20 = 40 min
Nitrobenzene Aniline pH = = 10.5
2
t 87.5% = 3 × t 50% = 3 × 20 = 60 min 25. (a, c)
Q °
E cell = E cell + 0.059 pH
2
l
NaBH 4 reduces only the (  CHO)
t 99% = × t 99.9% group to (  CH 2OH) group and ∴ Ecell = 13805
. + 0.059 × 10.5 = 2 V
3 does not reduce (C == C) bond.
31. (b) IF7 has a pentagonal bipyramidal
=
2
× 10 × t 50% =
400 l
H 2/Pt reduces both (  CHO) group
3 3 to (  CH 2OH) group and the structure.
(C == C) to (C  C) bond. ∴It has sp3d 3 hybridisation with 72°
22. (a,c,d) (A) Formation of slag When l
H 2, Pd/C reduces only (C == C) to and 90° as a bond angle.
ore is treated with silica it form slag of (C  C ) bond.
iron silicates 32. (a) Only square planar geometry has
26. (4) A( g ) + 2 B( g ) s C(g ) 2 lone pair.
FeO + SiO 2 → Fe ⋅ SiO 3
Slag At V 3M 4M xM ∴The correct combination for square
planar is (a).
(B) Reduction of Zn ZnO on reaction  3 + y  4 + 2 y  x − y
At 2 V     
ZnO with coke, converts ZnO into Zn 2  2  2  33. (d) CCl 4 has a tetrahedral geometry
and CO. This is due to reduction of x x with sp3 hybridisation and bond angle
ZnO into Zn. k eq = = … (i) 109°28'. So, none of these option is
3 × 42 48
ZnO + C → Zn + CO correct.
Given, 2y + 2= 3
364 JEE Advanced~Practice Set 3

34. (c) The reaction of α-haloketone with 38. (a, b, d) We have, ∴ N divides PM in the ratio 2:1
alkoxide ions to give rearranged esters 3π
Hence, the centroid of ∆PQR lies
is called Favorskii rearrangement.
f( x ) = ∫0 cos zcos( x − z)dx … (i)

on | z| = 1
e.g. O = ∫0 cos(3 π − z)cos( x − (3 π − z) dz As PQR is an equilateral triangle, so
3π orthocentre, circumcentre and
O H C = ∫ ( − cos z) (cos( 3 π − ( x + z)) dz
0 centroid will coincide
Cl 3π
OCH3
NaOCH3 = ∫0 cos zcos( x + z) dz … (ii) Now,
z1 + z2 + z3
=1
3
Adding Eqs. (i) and (ii), we get

[Q centroid lies on| z| = 1]
35. (a) The aldehyde and ketone 2 f( x ) = ∫0 cos z ⋅ [cos( x − z) ⇒ | z1 + z2 + z3|2 = 9
containing α-hydrogen undergo + cos( x + z)]dz ⇒ ( z1 + z2 + z3 ) ( z1 + z2 + z3 ) = 9…(i)
condensation with formaldehyde and 3π
ammonia to produce amino
= ∫0 cos z ⋅ 2⋅.cos x ⋅ cos z ⋅ dz Also, | z1| = 2 ⇒| z1|2 = 4
methylated compound.
⇒ f( x ) = cos x ∫

cos 2 z dz ⇒ z1 z1 = 4
0
e.g. O CH3 Similarly, z2 z2 = 1 and z3 z3 = 1 …(ii)
cos x 3π

H C CH + CH2 + NHCH3
=
2 ∫0 (1 + cos 2 z) dz From Eqs. (i) and (ii), we get
 4 1 1  4 1 1
cos x  sin2 z 

 + +   + +  =9
H O H = z+  z1 z2 z3   z1 z2 z3 
2  2  0
cos x Note that
O CH3
= [( 3 π + 0) − )] z  2π
2 arg  2  = ∠QOR = 120° =
H C CH CH2NHCH3
3π  z3  3
= cos x,
36. (b) The reaction in which 1, 2-diol 2 40. (a,c) Let ( h, k ) be the point of
rearranges to give ketone in the which is continuous and differentiable. intersection of two tangents, then
presence of sulphuric acid, is known 3π equation of AB is
max. f( x ) =
as pinacol-pinacolone rearrangement. 2 xh yk
+ =1 …(i)
37. (b,c,d) Let S be the sample space, 3π 4 1
and min f( x ) = − .
then n (S ) = 5 and given A i = the 2
event that the i th digit from left on the Also, f( x ) satisfies all the conditions of A (h, k)
number of ticket drawn is 1. Rolle’s theorem in [0, 4 π ], there exist P
∴ A1 = {112, 121} c ∈ ( 0, 4 π ) such that f ′ (c ) = 0.
A 2 = {112, 211} 39. (a,b,c,d) Clearly, OQ = 1 and OP = 2 O
A 3 = {121, 211, 221} Y B
2 2 3
and P( A1 ) = , P( A 2 ) = , P( A 3 ) = (z2) Q P (z1)
5 5 5 θ
N
Note that A1 ∩ A 2 ∩ A 3 = φ, M x2
A1 ∩ A 2 = {112}, A1 ∩ A 3 = {121}, X Equation of ellipse is + y2 = 1
O R 4
A 2 ∩ A 3 = {211} (z3)
Joint equation of OA and OB is
∴ P ( A1 ∩ A 2 ∩ A 3 ) = 0 (obtained by homogenizing equation
1 of ellipse using Eq. (i))
P ( A1 ∩ A 2 ) =
5 2
x2 y 2  xh yk 
P ( A1 ∩ A 3 ) =
1 OQ 1 + = + 
∴ sin θ = = ⇒ θ = 30° 4 1  4 1
5 OP 2
1  h2 − 4 
P ( A2 ∩ A3 ) = ∴ ∠QPR = 60°
5 ⇒ x2  
Also P( A1 ∪ A 2 ∪ A 3 ) Also, PQ = PR (length of tangents  16 
drawn from external point to a circle 2 hk
= P( A1 ) + P( A 2 ) + P( A 3 ) + y 2 ( k 2 − 1) + xy = 0 …(ii)
are equal) 4
− P( A1 ∩ A 2 ) − P( A1 ∩ A 3 ) ∴∆PQR is equilateral triangle Given equation of OA and OB is
− P( A 2 ∩ A 3 ) + P( A1 ∩ A 2 ∩ A 3 ) Also, OM ⊥ QR ⇒ ∠OMQ = 90° x 2 + 4 y 2 + αxy = 0 …(iii)
2 2 3 1 1 1 4
= + + − − − + 0 = and ∠MOQ = 60° Since Eqs. (ii) and (iii) represent same
5 5 5 5 5 5 5
3 [Q ∠OQP = 90° and ∠OPQ = 30°] line
Clearly, ∑ P( Ai ∩ Aj)

MO
= cos 60° =
1 h2 − 4 k 2 − 1 hk
i, j = 1
OQ 2 ∴ = = …(iv)
i<j 16 4 2α
= P( A1 ∩ A 2 ) + P( A 2 ∩ A 3 ) 1
MO = ⇒ h2 − 4 = 4 ( k 2 − 1)
2
+ P( A1 ∩ A 3 )
⇒ NM =
1 ⇒ h 2 − 4k 2 = 0
1 1 1 3
= + + = 2
5 5 5 5 Locus is ( x − 2 y) ( x + 2 y) = 0
and PN = 1
JEE Advanced~Practice Set 3 365

π
= 2 sin  +
x 2 − 9 x + 20 ⇒ g IV( x ) = − g IV( − x ) B C
41. (a,b,c) lim − 
x → 5− x − [ x] 2 2 2
⇒ g ( 0) = − g ( 0) ⇒ g ( 0) = 0
IV IV IV

= 2 cos  − 
B C
( x − 5)( x − 4)
= lim  a − x 2 2
x → 5− x − [ x] Since, log   is an odd function.
 a + x Thus,
( 5 − h − 5) ( 5 − h − 4) B − C
= 2 cos 
= lim A A
therefore, f( x ) is an even function. AA1 cos  cos
h→ 0 5 − h − [ 5 − h] 2  2  2
43. (a,c) Let the point of intersection of E1
− h(1 − h) − h(1 − h)
= lim = lim and E 2 be ( h, k ) B − C B + C
h→ 0 5 − h − 4 h → 0 (1 − h) = 2 cos   sin  
h2 k2  2   2 
= lim ( − h) = 0 ∴ + k = 1 and h +
2 2
=1
h→ 0 a2 a2 = sinC + sin B
x 2 − 9 x + 20 B
Again, lim h2 k2 Similarly, BB1 cos = sin A + sinC
x→ 5 +
x − [ x] ⇒ 2
= 1 – k 2 and 1 – h2 = 2
a a2
( x − 5) ( x − 4) C
and CC1 cos = sin A + sin B
= lim h 2
x → 5+ x − [ x] ⇒ = a2 2
1 – k2 So,
( 5 + h − 5) ( 5 + h − 4) A B C
= lim k2 AA1 cos + BB1 cos + CC1 cos
h→ 0 5 + h − [ 5 + h] and a 2 =
1 – h2 2 2 2
h(1 + h) h(1 + h) sin A + sin B + sinC
= lim = lim h 2
k2
h→ 0 5 + h − 5 h→ 0 h ∴ 2
= =2
1– k 1 – h2
= lim (1 + h) = 1 1
h→ 0
Hence, the limit at x = 5 does not
⇒ h2(1 – h2 ) = k 2(1 – k 2 ) 46. (7) Let I = ∫0(1 − x 4 )7dx. Then,
⇒h – h = k – k
2 4 2 4 1
I= ∫0(1 − I x
4 7
exist. ) .1 dx
⇒ h2 – k 2 – h4 + k 4 = 0 II
42. (a,c) We have,
⇒ h – k – (h − k ) = 0
2 2 4 4 = [(1 − x 4 )7 ⋅ x − ∫ 7 (1 − x 4 )6
ax
a− x eloge x2 ( − 4 x 3 ) ⋅ x dx ]10
⇒ ( h2 – k 2 ) – ( h2 – k 2 )( h2 + k 2 ) = 0
a 3x
g(x) = a − 3x eloge x4 1
a 5x
⇒ ( h2 – k 2 )[1 – ( h2 + k 2 )] = 0 = [(1 − x 4 )7 ⋅ x ]10 + 28∫ x 4 (1 − x 4 )6dx
a − 5x e loge 1 0
⇒ h = k or h + k = 1
2 2 2 2 1
= 0 − 28 ∫ (1 − x 4 − 1) (1 − x 4 )6 dx
a− x ax x2 ⇒ x 2 = y 2 or x 2 + y 2 = 1 1
0
1
= a − 3x a 3x x4 ⇒ x = ± y or x 2 + y 2 = 1
= − 28 ∫0(1 − x 4 )7 + 28∫ (1 − x 4 )6dx
0
a − 5x
1
a 5x 1 = − 28I + 28 ∫ (1 − x 4 )6 dx
44. (20) If three numbers form a GP, then 0
[Q eloge f( x )
= f( x )] their exponents must be in AP. 1

x −x 2 We know, if a, b, c are in GP, then


⇒ 29I = 28 ∫0(1 − x 4 )6 dx
a a x 1
∫0(1 − x ) dx = 7
4 7
b 2 = ac, 29
g ( − x ) = a 3x a − 3x x4 ⇒ 1
a 5x a − 5x 1 Since, exponent of b is even, 4∫ (1 − x 4 )6dx
0
exponent of a and c must be either
−x x 2 47. (8) Let the coordinates of B and C be
a a x both odd or both even.
= − a − 3x a 3x x4 ( at 12, 2 at 1 ) and ( at 22, 2 at 2 ) and let (α, β )
Now, two odd exponents or two even
a − 5x
a 5x
1 exponents (from 1, 2, 3… ,10) can be be the circumcentre of ∆ABC,then
selected in 5C 2 + 5C 2 = 10 + 10 = 20 Y
2
C(at2 ,2at2)
[interchanging C1 and C 2]
ways y 2=yax
= − g ( x)
∴ N = 20
g ( − x) + g ( x) = 0
45. (2) Let us join A, B as shown in the
∴ g ( x ) is an odd function.
figure. Then,
[f( x ) is odd if f( − x ) = − f( x )]
AA1 = 2 sin  B + 
A A X
Hence, the graph is symmetrical  2
about origin. A
Q g ( − x) = − g ( x)
C1 B1
∴Differentiating it w.r.t. x, we get 2
C(at1 ,2at1)
− g ′ ( − x ) = − g ′( x )
⇒ g ′ ( − x) = g ′ ( x) at 12 + at 22 2 at 1 + 2 at 2
α = ,β =
Again differentiating w.r.t. x 2 2
− g ′′( − x ) = g ′′( x ) [Q The circumcentre of a right angled
B C triangle is the mid-point of its
Again differentiating two more time
hypotenuse.]
w.r.t. x, we get A′
2α β
− g IV( − x ) = g IV( x ) ⇒ t 12 + t 22 = and t 1 + t 2 = … (i)
= 2 sin  B + ( π − ( B + C )
 1
a a
⇒ g IV( x ) is an odd function.  2 
366 JEE Advanced~Practice Set 3

Also, AB ⊥ AC Solutions (Q. Nos. 49-51)  x, 0≤ x< 1


2 at 2 2 at 1 
∴ × = − 1 ⇒ t1t 2 = − 4 1 ⇒ y =  1 + x − 1, 1 ≤ x < 2
I. f( x ) = −3
at 22 at 12 x 2 + x − 2 , 2 ≤ x < 3
2

β
Now, consider (t 1 + t 2 )2 =  
Here, we follow certain steps to plot and so on.
 a First we plot, ∴ Graph for y = [ x ] + x − [ x ] is
1 Y
β2 and then successively
⇒ t 12 + t 22 + 2t 1t 2 = x 4 3
a2 1 1 1 3+
x–
, − 3, −3
2α β 2
x x x 3 x–
2
⇒ −8= 2 2+
a a
(i) Y 2 x–
1
⇒ 2αa − 8a 2 = β 2 1+
1
Thus, locus of (α, β ) is y 2 = 2 ax − 8a 2 y= 1 x
x X
–1 10 1 2 3
x+
Hence, λ = 8 X +
–1

48. (2) We know, (1 + x)100 = 100


C 0 x100
III. f( x ) = min{| x |,| x − 1|,| x + 1|}
+ 100
C1 x 99 + ... + 100
C100
First plot the graph for;
and (1 + x )100 = 100
C0 + 100
C1 x
1 1 y = | x |, y = | x − 1| and y = | x + 1|
(ii) → (i.e., taking images about
+ 100
C 2 x + ... +
2 100
C100 x100 x x by a dotted curve, as shown below
⇒ (1 + x )200 = (100C 0 x100 + 100
C1 x 99 X-axis for –ve values of y) and then darken those dotted lines for
Y which| x | < {| x − 1|,| x + 1|};
+ 100
C2x 98
+ .... + 100
C100 )
| x − 1| < {| x |,| x + 1|}
( 100
C0 + 100
C1 x + 100
C 2 x + ...2
y=|1| y=|1|
x x and| x + 1| < {| x |,| x − 1|}
+ 100
C100 x100 )

y= 1 |
X

|
Y

|x
⇒ 200
C102 = 100
C0⋅ 100
C2 + 100

|x
C1

|
–1
y=

|x
⋅ 100C 3 + 100
C 2 .100 C 4 + ....

y=
+ 100
C 98 ⋅ 100C100 )
1 1 1/2
Let a = 100
C0 ⋅ 100
C2 + 100
C 2 ⋅ 100C 4 (iii) → − 3 i.e. shifting 3 units 0
x x X
+ .... + 100
C 98 ⋅ 100C100 –3 –2 –1–1 1 1 2 3
below 2 2
and b = 100
C1 ⋅ 100C 3 + 100C 3 ⋅ 100C 5 Y As from the above graph;
+ .... + 100C 97 ⋅ 100C 99 min{| x |,| x − 1|,| x + 1|}
Then, we have  − ( x + 1), x≤ −1
a+ b= 200
C102 = 200  1
 ( x + 1), − 1 ≤ x ≤ − 2
C 98 … (i)
Clearly,  1
(1 + x )100 ⋅ (1 − x )100 = (100C 0 x100 X  − ( x ), − ≤ x≤ 0
1 0 1 = 2
– 1
+ 100
C1 x 99
+ ... + 100
C100 ). 3 3  x, 0≤ x≤
y=|1|–3 y=|1|–3  2
(100C 0 − 100
C1 x + 100
C 2 x 2 − .... x x  − ( x − 1), 1
≤ x≤1
+ 100
C100 x100 )  2
x=–3  ( x − 1), 1≤ x
⇒ (1 − x 2 )100 = (100C 0 x100 + 100
C1 x 99
2x
+ .... + 100
C100 ) . (iv)
1
− 3→
1
− 3 (i.e. taking images IV. f( x ) = . As we know 2 x is
x x 2[x ]
(100C 0 − 100
C1 x + 100
C 2 x 2 − .... exponential function and we want to
about X-axis for negative values of y)
transform it to 2 x − [ x ]. So, retain the
+ 100
C100 x100 ) Y graph for x ∈ [0, 1) and repeat for rest
⇒− 100
C 51 = 100
C 0 . 100C 2 − 100
C1 points.
y=||1|–3|
. 100C 3 + 100
C 2 . 100C 4 x Clearly, graph of y = 2 x is
− .... + 100
C 98 . 100
C100 X Y y = 2x
1 01
⇒− 100
C 49 = a − b … (ii) –
3 3
On adding Eqs. (i) and (ii), we get
II. f( x ) = [ x ] + x − [ x ] 2
2a = 200
C 98 − 100
C 49
Let y = [ x ] + x − [ x ]
1 200
a= [ C 98 − 100
C 49 ] ⇒ y = k + x − k, 1
2
k ≤ x < k + 1; k ∈ integer 0 1
X
Hence, λ = 2
JEE Advanced~Practice Set 3 367

Now, we need to retain graph ⇒ (2 x − 1) ( 3 x + 1) = 0 Meet X-axis at 30 x 2 − 35 x + 5 = 0


between x ∈ [0, 1), and repeat it for 1 1 1
⇒ x = or x = − ⇒ 6 x 2 − 7 x + 1 = 0 ⇒x = 1 or x =
other points. 2 3 6
Y 1 1 5
⇒ Intercept on X-axis = + = 1 5
So, intercept on X-axis = 1 − =
x 2 3 6 6 6
y= 2
2 [x] Similarly, to find intersection with 5 1
Similarly intercept on Y-axis = 1 − =
Y-axis put x = 0, then we get 6 6
X − 6 y2 + 5 y − 1 = 0 Now, consider the equation
–2 –1 0 1 2 3
⇒ 6 y2 − 5 y + 1 = 0 6 x 2 + 12 xy + 6 y 2 − 7 x − 7 y + 1 = 0.

49. (a) 50. (c) 51. (d) ⇒ (2 y − 1) ( 3 y − 1) = 0 Here, h2 = ( 6)2 = ( 6) ( 6), so the lines
1 1 L5, L6 are parallel.
⇒ y = or y =
Solution (52, 53 and 54) 2 3 Meet X-axis at 6 x 2 − 7 x + 1 = 0
Consider the equation 1 1 1 1
⇒ Intercept on Y-axis = − = ⇒ x = 1 or x =
6 x 2 + 5 xy − 6 y 2 − x + 5 y − 1 = 0 2 3 6 6
Here, coefficient of x 2 + coefficient of Consider the equation 1 5
So, intercept on X-axis = 1 − =
y2 = 0 30 x 2 + 36 xy + 6 y 2 − 35 x 6 6
∴The lines L1, L2 are perpendicular. − 11y + 5 = 0 5
and intercept on Y-axis is also =
6
To find intersection with X-axis put Clearly, the lines L3, L4 are neither
y = 0, then we get parallel nor perpendicular. Hence, 52 (b), 53 (c) and 54 (d).
6 x2 − x − 1 = 0 [Q a + b ≠ 0 and h2 ≠ ab]

Paper 2
1. (a) In polytropic process, 4 × 0.5 × 1 × 12.5 25 ms − 1
∴∆p = −2 2 ⇒ vsnow /car = = 31 ms − 1
c = molar specific heat (10 × 10 ) cos37 °
=
R

R . Nm−2
= 025 v
Now, tan 37º = snow /ground
γ −1 n−1 vcar /ground
3. (c) Volume specified,
Now, 4 3 4 ⇒ Vsnow /ground = 25 × tan 37º
V = πr = π( 0.84)3 = 2.483 m3
∆Q = heat absorbed = nC∆T 3 3 . ms − 1
= 1875
 R R  Volume maximum
= 1×  −  ∆T 5. (d) Initially, 2T = mg or
 γ − 1 n − 1 4 4
= π( rmax
3
) = π( 0.88)3 = 2.855 m3 mg
( n − γ ) R∆T 3 3 T = = 300 N
= = 011
. kJ 2
( n − 1) ( γ − 1) Volume minimum
Now, tension is increased by 15%,
4 4
2. (b) For a small thickness cylinder of = π( rmin
3
) = π( 0.80)3 = 2.145 m3 then
3 3
radius r and thickness dr of fluid inside ma = 2T ′ − mg
tube, if F = viscous friction force, then Percentage uncertainty is
2  × 300 − 60 × 10
115
∆V V − Vmin
R × 100 = max × 100  100 
V V or a =
60
= 28.59%
E = 1.50 ms − 2
4. (18.75) Velocity. of car with respect to
ground = 25 ms− 1 6. (a) Free body diagram is
dr
r
ca
w/
vsnow/ground

F v sno l–y
shear stress =
2 πRl
l
dv
But shear stress = η 37°
dr r=R m2
vcar/ground
2 v 
= η 0 
 R  Velocity of snow relative to car =
Velocity of snow relative to car + m1
So, F = 4 πv 0lη
Velocity of car relative to ground y l− y
So, pressure difference across ends m1 = M and m2 = M
v l l
F F 4 v 0l η Also, cos 37º = car /ground
∆p = = = vsnow /car where, M = mass of complete cable.
A πR 2 R2
368 JEE Advanced~Practice Set 3

So, acceleration of system is MR 2 $ Potential drop across inductor,


= ω(k ) = Anti-clockwise rotation
m − m2 2 dI
a= 1 ⋅g V2 = L
m1 + m2 and L = M(r × v ) = Mvd( − k$ ).
translation 0 0
dt
y − ( l − y) 2y − l = ( 0.4) ( 30) e − 5t = 12 e − 5 t
= g = g 10. (c, d) F = 2t $i + 2 $j
y + ( l − y) l 13. (a, b, c, d) AB is isobaric compression,
dv
dv dv dy dv ⇒ m = F = 2t $i + 2 $j
So, = ⋅ =v dt ∴∆W = negative,
dt dy dt dy
 2 y − l  g dy vdv
⇒ ∫ ∫
dv = (2t $i + 2 $j )dt The gas must be cooled to compress
the gas.
∴  =
 l  ⇒ v = t 2$i + 2t$j ∴∆Q and ∆U both are negative.
Let y0 = length initially hanging from ∫ dr = ∫ (t i + 2 j ))dt
2$ $
⇒ BC is isochoric process,
pulley, then 3 ∴∆V = 0 ⇒ ∆W = 0
t $
l 2 y  vf or r= i + t 2$j
∫y0  l − 1 gdy = ∫0 vdv 3
CA is isothermal expansion,
∴ ∆Q is positive.
Now, torque is
v f = 2 gy0  1 − 0 
y
⇒ t 3  14. (b, c, d) Electric field is perpendicular
 l  = r × F =  $i + t 2$j  × (2t $i + 2 $j ) to length of wire at every point, so b
3  and d are correct.
2
For y0 = l, we get
3  2t 3  4 q
⇒τ =  − 2t 3  k$ ⇒ τ = t 3( − k$ ) Flux, φ = enclosed
 3  3 ε
v f = 2 g  l  1 − 2 
2
  λl λ × 1 λ
3   3 Angular momentum of body is = = =
ε ε ε
=
2
gl =
2
10 = 2.10 ms − 1 L = r × mv = m(r × v )
15. (d) We have,
3 3 t 3 
=  $i + t 2$j  × (t 2$i + 2t$j ) [G ] = [M− 1L3T − 2 ]
7. (c) At some instant t , q 1 + q 2 = Q and 3 
q1 q 2 [L3T − 2 ]
= 2t 4 $ t4 ⇒ [M] =
C1 C 2 = ⋅ k − t 4k$ = ( − k$ ) [G]
3 3
Also, C1 =
ε0 A
and C 2 =
ε0 A and [c ] = [LT − 1 ]
11. (b,c) Height of liquid column,
(d 0 + vt ) (d 0 − vt ) ⇒ L = [cT]
2T cosθ
h=
Hence, from there, we get ρgr Also, [h] = [ML2T − 1 ]
 d − vt   1 1 − 5
q1 = Q  0  For water-glass, θ ≈ 0º
 2d 0  ⇒ [ T] =  h 2 G 2 c 2 
2T
∴ h=  
 d + vt  ρgr
and q 2 = Q  0  −2
 2d 0  ⇒ r = 2T / ρgh  1 1 − 5
3
[cT]  h 2 G 2 c 2 
Current in circuit = Reduction in Work done by forces of surface  
charge of first capacitor = Charge tension is ∴ M =
[G ]
increase in second capacitor. 4 πT 2
W = 2 πrTh =  3 3 − 15 
− dq 1 dq 2 ρg 3 −1 −1 5
∴ I= = c h 2 G 2 c 2  [h G c ]
dt dt  
Qv Q×1
Heat liberated is = 
= = = 0.50 A mgh [G ]
2d 0 2Q Q=W −
2  1 − 1 1
8. (b,d) Area under F - t graph indicates 4 πT 2 2 πT 2 2 πT 2 W = c 2 G 2 h 2 
= − = =  
momentum. ρg ρg ρg 2
Also, A = p = 2 Km 16. (d) Momentum, p = mv = [M L T− 1 ]
12. (a,b) Time constant of given L - R
A2  1 1 1 
⇒ K = circuit is
2m = c 2 G 2 h 2 c 
L 0.4
2 τ = = = 0.2 s  
  4 × 3  −  15
. × 2 R2 2
    = [ c 3/ 2 G − 1/ 2 h1/ 2]
   2 
= 
2 Steady state current in inductor,
17. (c) Fractional error in X is
2 ×2 E 12
i0 = = = 6A ∆X 2 ∆a 3∆b 2.5∆c 2 ∆d
R2 2 = + + +
= 5.0625 J X a b c d
9. (c,d) Angular momentum of disc, So, current in inductor, = 2 × 0 ⋅ 01 + 3 × 0. 02
L = Lrotation about COM i = i 0(1 − e − t / τ ) + 2.5 × 0.03 + 2 × 0.04
+ L translation about origin
= 6 (1 − e − t / 0. 2 ) = 0235
. ≈ 024
.
where, Lrotation = Iω
= 6 (1 − e − 5t ) 18. (d) As error occurs in first decimal,
MR 2
= ω( − k$ ) = clockwise rotation hence we round off to 2.8.
2
JEE Advanced~Practice Set 3 369

19. (a) A set of quantum numbers gives in Cause of colour in N2O 3 ρ


Br
1
(a) denotes 3 p which is valence shell Due to low difference between
+ Br+
electronic configuration of Al. occupied and unoccupied energy (b)
levels of electrons among molecules
(b) denotes 3 p3 which is valence shell H
of N2O 3 , it absorbs a part of visible
electronic configuration of P. light spectrum which causes colour in ρ
(c) denotes 3 p2 which is valence shell N2O 3 . (c) + H+
electronic configuration of Si. 3 Cl
1 × 3R + 2 × R
(d) denotes 3s1 which is valence shell 23. (d)QC V = 2 = 2R
electronic configuration of Na. 1+ 2 (d) C + Ag+
The elements of period 3 forms As we know that,
following respective oxides as: T1V 1r −1 = T2V 2r − 1 …(i) ρ
C
Element Oxides Cp
Q C p – C V = R and r =
Na Na 2O (Alkaline) CV 45° tilted to
each other
Mg MgO (Alkaline) ∴C p = 3R ⇒ r = 1.5
Al Al 2O3 (Amphoteric) Now, putting the given values in the
above equation, we have Option (d) gives a non-planar
Si SiO2 (Acidic)
320 × (1)0.5 = T2 × ( 4)0. 5 carbocation. The non-planar structure
P P2O5 ≡≡ P4O10 (Acidic) is due to steric hindrance of three
⇒ T2 = 160 K phenyl rings. All three phenyl ring is
S SO3 , SO2 (Acidic)
Now, ∆E = nC V (T2 − T1 ) tilted to plane by 45° like wings of a fan.
Cl Cl 2O7 (Acidic)
= 3 × 2 R(160 − 320) 26. (a) Molecular formula of
Out of above oxides Al 2O 3 is the only = − 960R coordination compound
one which produces amphoteric O O [Co(H2O)3F3] H 2O
solution after hydrolysis.   Homolysis
H 2O F
20. (d) Determination of molecular 24. (c) C6H5  COOC  C6H5 → Co
Benzoyl peroxide
structure Degree of unsaturation is 4 H 2O F
and it is an aromatic compound. So O
 • F
possible structure of compound is
Cl 2 C6H5  C O → 2 C6H•5
 CO2 CFT in coordination compound
6
 Cl Co3+ = [18Ar] 3d 4s 0 2
eg
(A)   3d
6
 Cl  C Br
 
Alkyl halide are extremely less reactive ↓ Cl
4
towards SN reaction due to resonance t 2g
+ C6H5 Br
• CCl
3
effect. CH3  CH2  CH == CH2 + • CCl 3 →
⊕ Number of electrons in e g set of
Cl Cl CH3  CH2  CH  CH2  CCl 3 orbitals = 2

Double bond Secondary radical ••
σ character 27. (b) H  O  Cl + H⊕ →
 Br ••

  H O  Cl → Cl ⊕ + H2O
 Cl  C  Cl 
C6H5 Cl + NaOH → No reaction  
↓ H
Cl
21. (a) Normality = Molarity × Valency factor Ph  C ≡≡ CH + Cl ⊕ → Ph  C⊕
60 CH3  CH2  CH  CH2  CCl 3 == CH  Cl
∴ x×6= × 0.1
1000 
Br → Ph  C == CH  Cl
Q Number of electrons transferred = 6
H 2O

25. (d) OH
6
Thus, x = = 10 −3 mol F
6 × 1000 ρ ¼¿ Ta utomerism
22. (a) Cause of colour in NO 2 SbF5
[SbF6]σ
Ph — C— CH2 — Cl
(a) 
Molecular orbital electronic
O
configuration of NO 2 confirms the Tropylium
presence of unpaired electron which carbocation 28. (b) Maximum number of nuclei will be
can easily undergo transition from present, when rate of decay = rate of
ground state energy level to excited formation. Thus,
state level by absorbing light of α
λN = α ⇒ N =
suitable wavelength. λ
370 JEE Advanced~Practice Set 3

29. (b,c) In the product, (3° alcohol) at 32. (c,d) (a) SO 2 and CO 2 cannot be 10
and nB =
least two alkyl groups must be same, distinguished by using lime water as 3
after the reaction. both produce the milky white mixture As ure know that, pV = nRT
We cannot synthesise such 3° alcohol of CaCO 3 and CaSO 3 .
∴ p × 8.21 =  +
2 10 
 RT
through the reaction in which all the Lime water
CO 2 and SO 2 → milky solution 3 3
alkyl groups are different. Thus, (b)
and (c) cannot be synthesised. (b) SO 2 and CO 2 on reaction with Thus, p = 4 atm

30. (a) NaCl (rock salt ) type structure is BaCl 2 produces the same white ppt. 36. (a) According to Graham’s law of
fcc type in the form of barium sulphate and diffusion
calcium sulphate in which both are rA p MB n MB
soluble in dil. HCl. = A = A
rB pB MA nB MA
SO 2 + BaCl 2 → BaSO 3 ↓
Q pV = nRT,
I.e. p ∝ n 
White ppt.

CO 2 + BaCl 2 → BaCO 3 ↓ rA ( 8 / 3) 1
White ppt. =
Z eff = 4 ( 4 Na + and 4 Cl − ) rB ( 4 / 3) 1
(c) SO 2 on reaction with H2O 2
Now, according to the conditions given rA 2
produces H2 SO 4 due to oxidation of ⇒ =
Number of B− ions removed SO 2 and H2 SO 4 on reaction with
rB 1
1 1 BaCl 2 produces BaSO 4 . 37. (c) Consider the given series
= 4 × + 2 × = 3/2
8 2 SO 2 + H2O 2 → H2 SO 4 100
∑ (− 1) ( 99C r +
r 99
Cr − 1)
Number of A + ions removed H2 SO 4 + BaCl 2 → BaSO 4 ↓ r=0

=2 × =
1 1 White ppt. 1 3r 7r 
4 2 CO 2 does not reacts with H2O 2 ,  2 r + 2 2r + 2 3r + … upto 10 terms 
 
But body centre A + ion is not removed. hence, SO 2 can be separated easily. 100
= ∑ (− 1)
r 100
Cr
∴ Number of B s ions left (d) SO 2 on reaction with acidified
r=0
dichromate turns orange colour of
3 5
=4− = = 2.5 dichromate to green while there is no 1 3r 7r 
2 2 effect of acidified K 2 Cr2O 7 on CO 2 .  2 r + 2 2r + 2 3r + … upto 10 terms 
 
∴ Number of A + ions left Acidified n +1
SO2 [Q nC r + nC r − 1 = Cr ]
1 7
= 4 − = = 3.5 K 2 Cr2O 7 (orange )  → Green
r
2 2 100 100
C r   +
CO2 1
Acidified K 2 Cr2O 7  → No change = ∑ (− 1) ∑ (− 1)
r 100 r
Thus, the formula of the unit cell (orange) in colour r=0
2 r=0
= A 3.5 B2.5 r 100 r
C r   + C r  
Hence, correct choice is (c) and (d). 3 7
∑ (− 1)
100 r 100
31. (c,d) 33. (a) O  4 r=0
 8
Dioxide
OH OH O O + … upto 10 terms
Cr Peroxide
r r
100
−1 100
−3
CO2, NaOH —COOH O O = ∑
100
C r   + ∑
100
C r  
r=0
2  r=0
 4
+
H 34. (c) The compound obtained during r
100
−7 
with Cl s and + ∑ 100C r 
Kolbe reaction
Phenol Salicylic acid reaction of Cr2O 2−
7  + … upto 10 terms
r=0
 8 
(A ) H2 SO 4 is CrO 2 Cl 2 .
100 100
O—C—CH3 s
7 + Cl + H 2 SO 4 →
Cr2O 2− =  1 −  +  1 − 
CrO 2 Cl 2 1 3
+ ... upto 10
Red vapour  2  4
O
(CH3CO)2O
—COOH terms
Let oxidation state of Cr in CrO 2 Cl 2 is x.
 n 
Q ∑ C r ( − x ) = (1 − x ) 
n r n
Acetylation Then, x + 2 ( − 2 ) + 2 ( − 1) = 0
Aspirin
x− 4−2 = 0  r=0 
100 100
(B)
=   +  
x−6=0 1 1
+ … upto 10 terms
OH 2  4
x=6
  100  
10

AlCl3 —COOH Hence, correct choice is (c).  1 −   1   


100
 2  
 
→ =  
35. (b) A B 1
H 2O  
2 
1

Fries rearrangement 8 4 1 − 100
At, t = 0,  2 
C—CH3 3 3  
4 8
O t = 5 min, 1 − 1 
2 1000  = 1  2 − 1
3 3 1  1000
(C) = 100    
2 1 1000 100

After two half times, nA = 2
 1 − 100 
2  2 1
3  2 
JEE Advanced~Practice Set 3 371

38. (c) Given equations, 1 ∴ Area of


Replacing x by , we get
x 3 3
3l + m + 5n = 0 …(i) ∆ABC = ( AB)2 = (2 (1 − cos θ))
1− 1 
6mn − 2 nl + 5ml = 0 …(ii)  1    4 4
2 f  − 1 − f  x = 1
x  1 3
( 3 l + m)   x = (1 − cos θ)
From Eq. (i), n = −  x  2
5
1 − x
2 f  Let G be the centroid of ∆ABC, then
1
Substituting in Eq. (ii), we get ⇒  − f ( x − 1) = …(ii)
 x  x 1
OG = |a + b + c|
6 l 2 + 9 lm − 6 m 2 = 0 On multiplying by 2 in Eq. (i) and then 3
2
adding Eqs. (i) and (ii), we get 1 a 2 + b 2 + c 2 + 2a ⋅ b
⇒ 6   + 9   − 6 = 0
l l =
3 + 2b.c + 2c.a
 m  m  1 − x 
2 2 f ( x − 1) − f  
  x   1
l1 1 l  = 3 + 6cosθ
∴ = and 2 = − 2 …(iii) 3
 1 − x 
m1 2 m2 + 2 f  − f ( x − 1)
  x   =
1
1 + 2 cosθ
Using Eq. (i), we get  
3
1 1
l1 l
= − 1 and 2 = − 2 …(iv) = 2 x + ⇒ 3f ( x − 1) = 2 x + Now, [a b c ] = Volume of
n1 n2 x x
parallelopiped = OG × 2 ar ( ∆ABC )
From Eqs. (iii) and (iv), we have Now, replacing x by x + 1, we get [to
1 3
generate f( x )] = 2⋅ 1 + 2 cos θ × (1 − cos θ)
l1 m1 n
= = 1 3 f [( x + 1) − 1] = 2 ( x + 1) +
1 3 2
1 2 −1 ( x + 1) = (1 − cos θ) 1 + 2 cos θ
+ l12 + m12 n12 1 1 1  cos 2α k + i ⋅ sin2 α k
= = ∴ f( x ) =  2 ( x + 1) +  43. (a) Given, ω k =
1+ 4 + 1 6 3 ( x + 1) zk
l2 m2 n 2 (1 + x )2 + 1 (cos α k + i sin α k )2 2
and = = 2 =
z
= 2k
2 −1 −1 =
3 (1 + x ) zk µ k zk
l22 + m22 + n22 1 2
= = x 1 1
4 + 1+ 1 6
41. (a) f( x) = ∫0 f(t )dt ⇒ f ′( x) =
f( x )
=
zk
zk zk zk
2
(µ k = zk zk ) =
1
zk
∴ If α is the angle between two lines, It means f ′ ( x ) ⋅ f( x ) = 1 1 1 1
then Now, ω1 + ω 2 + ω 3 = + +
dy
Now, ⋅ y = 1 ⇒ ∫ ydy = ∫ 1dx z1 z2 z3
 1   2   2   1  dx
cosα =     +   −   1 1 1
 6  6  6  6 (by separating the variables) = + + =0
 z1 z2 z3 
 − 1  − 1 1 y2
+     = = x+C ...(i)
 6  6 6  1 1 1 
Q z + z + z = 0
2
⇒ α = cos −1  1 1 1  
f (1) = ∫
1 2 3
  Now, dt = 2
 6 0 f (t ) ω1 + ω 2 + ω 3
∴ =0
39. (c) We know the prime digits are { f(1)} 2 3
∴ = 1+ C [from Eq. (i)]
2, 3, 5, 7 . If we fix 2 at the first place 2 Hence, centroid of ∆A1 A 2 A 3 is origin.
then rest of 2 n − 1 places can be filled ( 2 )2
in 4 2n − 1 ways. Sum of 2 consecutive ⇒ = 1+ C ⇒ C = 1− 1 = 0 44. (a, b) Given,x 3 − y 2 = 0 ...(i)
2
digits is prime when consecutive dy dy 3 x 2
So, { f( x )} 2 = 2 x ⇒ f( x ) = 2 x ⇒ 3 x2 − 2 y =0 ⇒ =
digits are (2, 3) or (2, 5), thus 2 will be dx dx 2y
fixed at all alternate places. So, f (200) = 2 ⋅ 200 = 400 = 20
i . e. ∴Slope of tangent at
42. (d) Let OA = a , OB = b and OC = c .  3 x2 
P =   = 
dy
2 2 2  = 3m
Then,|OA| = |OB| = |OC| = 1, which  dx  P  2 y  2 3
( 4 m , 8m )
For filling n places by 2, we have only implies ∆ABC is a equilateral triangle.
Now, the equation of tangent at P is
1 way and for filling rest of n places, O
by 3 or 5, y − 8m3 = 3m( x − 4m2 )

number of ways = 2 n ⇒ y = 3mx − 4m3 ...(ii)


∴Number of favourable ways = 2 n Since, it cuts the curve again at point
B Q.
2n
Required probability = On solving Eqs. (i) and (ii), we get
4 2n−1 G
2n × 4 4 A C x = 4 m 2, m 2
= =
2 4n 2 3n On putting x = m2 in Eq. (ii), we get
and
40. (a) Given functional equation is AB = OA 2 + OB2 − 2OA ⋅ OBcosθ y = 3m( m2 − 4m2 ) = − m3
1 − x
2 f( x − 1) − f   = x …(i) = 2 − 2 cosθ ∴ Q ≡ ( m2, − m3 )
 x 
= 2 1 − cosθ Now, the slope of tangent at Q
372 JEE Advanced~Practice Set 3

3( m4 ) ⇒ 3 p2 − 2 p − 1 = 0
=  
dy 3 Thus, f( x ) is a polynomial function and
= =− m
 dx  ( m2 , − m3 ) 2( − m3 ) 2 hence twice differentiable for all x ∈ R.
⇒ ( 3 p + 1) ( p − 1) = 0
1 Also, f ′ ( 3) = 3 2 − 1 = 8
−1 2 ⇒ p = 1, −
Slope of normal at Q = = 3
 − 3  m 3m 51. (a) We have,
  48. (a, b, c, d) We have,
 2  f( x ) = 2 x 3 − 3( a − 3) x 2 + 6ax + a + 2
y = a x + bx …(i)
Since, the tangent at P is normal at Q. f ′ ( x ) = 6 x 2 − 6( a − 3) x + 6a
Since it passes through (1, 2),

2
= 3m ⇒ 9m2 = 2 ⇒ m = ±
2 therefore we get = 6{ x 2 − ( a − 3) x + a}
3m 3 a+ b=2 …(ii) For exactly one point of local maxima
45. (a, b, d) Now, as the area of region bounded and one point of local minima of f( x ),
Given. by the curve (i), x = 4 and y = 0 is quadratic equation f ′( x ) has two
1 dx 1 8 sq units. distinct and real roots
In = ∫0 (1 + = ∫0(1 + x 2 )− n dx 4
So, D > 0 ⇒ ( a − 3)2 − 4a > 0
x 2 )n ∴ ∫ (a x + bx ) dx = 8
1 0 a 2 + 9 − 10a > 0
 x  1 2 − n−1 4
= 2 n
− ∫0(− n)(1 + x ) 2
⇒  a x 3/ 2 +
bx
=8
2
( a − 1)( a − 9) > 0
 (1 + x )  0 3 2  0 ⇒ a ∈ ( −∞,1) ∪ ( 9, ∞ )

⋅ 2 x ⋅ xdx 2a 52. (c) For local minima at some negative
⇒ + b=1
1 1 x2 3 real x, quadratic equation f ′( x ) has
= + 2 n∫ dx
2n 0 (1 + x 2 )n + 1 ⇒ 2 a + 3b = 3 … (iii) both roots negative. So
1 1 1 + x2 − 1 On solving Eqs. (ii) and (iii), we get D > 0 ⇒ a ∉ ( −∞, 1) ∪ ( 9, ∞ )
= + 2 n∫ dx
2 n 0 (1 + x )2 n +1 a = 3, b = − 1 B
and − < 0 ⇒ a − 3 < 0
49. (a, b) Equation of required plane is A
1
= + 2 nIn − 2 nIn + 1 C
2n lx + my + λz = 0 …(i) ⇒ a ∈ ( −∞, 3), and > 0
A
⇒ 2 nIn + 1 = 2 − n + (2 n − 1)In Now, as the angle between (i) and
lx + my = 0 is α. ⇒ a> 0
1
⇒ 2 I2 = + I1 l 2 + m2 ⇒ a ∈ ( 0, ∞ )
2 ∴ cosα =
l 2 + m2 l 2 + m2 + λ2 ⇒ a ∈ ( 0, 1)
1
⇒ 2 I2 = + (tan− 1 x )10
2 l 2 + m2 53. (c) As second row of all the options is
⇒ cos α = 2

1 π l 2 + m2 + λ2 same, we look at the elements of first


∴ I2 = + row.
4 8 ⇒ cos αλ = ( l + m )(1 − cos α )
2 2 2 2 2

4 I3 = 2 − 2 + 3 I2 a b c
Also, ⇒ cos 2 α λ2 = ( l 2 + m2 )sin2 α Let the left inverse be
d e f 
.
 
π
+  + 
1 3 1 ⇒ λ2 = ( l 2 + m2 ) tan2 α
⇒ I3 =  1 − 1
16 4  4 8 a b c   1 0
⇒ λ=± l 2 + m2 tanα Then, 1 1 = .
1 3π d e f     0 1
∴ I3 =
+   2 3  
4 32 Hence, equation of plane is  
π π π lx + my ± z l 2 + m2 tanα = 0 ⇒ a + b + 2c = 1, which does not
46. (a, b) For θ ∈  0,  ∪  , 
 4  4 2  50. (a, c, d) We have,
satisfy by the elements of first row of
1 matrix in option (c)
Let cos 4θ = f ( x + y) = f( x ) + f( y) + xy ( x + y)
3 Also, it is given that f ′ ( 0) = − 1 Thus, matrix in option (c) is not the left
1 + cos 4 θ 2 f ( h) inverse.
⇒ cos 2θ = ± =± ⇒ lim = −1 … (i) a b
2 3 h→ 0 h
54. (d) Let right inverse be c d .
2 cos 2 θ [Q f( 0) = 2 f( 0) ⇒ f( 0) = 0]  
∴ f   =
1 2
=
 3  2 − sec θ 2 cos 2 θ − 1
2 Now, consider e f 
f ( x + h) − f ( x )
1 3 3 f ′ ( x ) = lim a b
=1 + = 1− or 1 + h→ 0 1 − 1 2    1 0
cos 2θ
h Then, c d =
2 2 f( x ) + f( h) + xh( x + h) − f( x ) 2 − 1 1    0 1
= lim   e f  
 
47. (b, d) Here, a = (2 p) $i + $j h→ 0 h
f ( h) ⇒ a − c + 2e = 1,
Now, when a system is rotated, the = lim + lim x ( x + h)
h→ 0 h h→ 0 b − d + 2f = 0
new component of a are( p + 1) and 1.
= − 1 + x 2 (using Eq. (i) 2 a − c + e = 0,
Let b = ( p + 1) $i + $j . The,
⇒ f ′( x) = − 1 + x 2 and 2b − d +f = 1
⇒ |a| = |b| ⇒ |a|2 = |b|2
On integrating both sides, we get This system of equations has infinite
⇒ 4 p2 + 1 = ( p + 1)2 + 1 3 solutions.
x
⇒ 4 p2 = p2 + 2 p + 1 f( x ) = − x + +C
3
PRACTICE SET - 4
Paper 1
1. (c, d) The characteristic wavelength of So, only 5 kg block will slip due to r r ω2
⇒ =
X-rays is given by, applied force fo 30 N. R −h g
hc Also, acceleration of 5 kg block is ω2 = g / R − h
λ kα = or
102. ( z − 1)2 F
a = net ⇒ h = R − g / ω2
As, z1 = z2 = z3 m
30 − 25 For non zero h, R > g / ω 2
λ1 = λ 2 = λ 3 =
5 g 9.8
So, λ 1 = λ 2λ 3 ⇒ ωmin = = = 9.8 rads −1
5 R 01
.
2. (a, b, c, d) = = 1ms −2
5 7. (8.00) Maximum friction between A
As, y is a length, and B = µ mA g
5. (d) Momentum conservation,
[a 3 ] = [M0LT0 ] ⇒ [a ] = [M0L1/ 3T0 ] 2E or f1 = 025
. ( 3) (10) = 7.5 N
= mv (QE = mc 2 )
SI unit of sin−1  − 1 is radian or
x c Maximum friction between B and
b 
From energy conservation, C = µ ( mA + mB ) g
sin−1  − 1 is an angle.
x
1 or f2 = 025
. ( 3 + 4) (10) = 17.5 N
b  mv 2 = mgl (1 − cos θ)
2 Maximum friction betweenC and ground
 x − 1 must be dimensionless, so f3 = µ ( mA + mB + mC ) g
 
b  θ = 025
. ( 3 + 4 + 8) (10) = 37.5 N
dimensions of x and b must be same.
Block C and hence block B are
For an angle, dimensions are [M0L0T0 ]. moving in opposite directions with
3. (c) For da, temperature constant and constant velocities and block A is at
E/c h rest. Hence, net force on all three
pressure increase and volume
decreases from equation, pV = nRT . blocks should be zero. Free body
E/c diagrams have been shown below
So, V-T graph must be vertical.
For small angle, (only horizontal forces are shown)
For ab, temperature increases at f1
constant pressure, so volume also mv 2 f2
= mgl [2 sin2 θ /2 ]
nR 2 B C T
increases V = T and V -T graph T F
p  θ2  f3
mv 2 f2
straight line passing through origin. = mgl  
2 4 For equilibrium of B,
bc as ad will be vertical line, cd as ab
From both equations, θ =
2E T = f1 + f2 = 25 N
will be a straight line passing through
origin. mc gl For equilibrium of C,
V F = T + f2 + f3 = 80 N
c 6. (b, c, d)
FBD of particle in bowl is 8. (4.00) After elastic collision,
 m − 2 m 2(2 m)
d
b v ′A =   ( 9) + ( 0)
N  m + 2 m m+ 2m
a θ = − 3 ms −1
T r mg Now, from conservation of linear
1
For bc, T constant p ∝ p↓ V ↑ momentum after all collisions are
V complete,
For cd, p constant V ∝ T as T ↓ V ↑ m ( + 9 ms −1 ) = m ( −3 ms −1 ) + 3m ( vC )
r
4. (a,c) Limiting friction value for 5 kg From FBD, tanθ = or vC = 4 ms −1
R −h
block is
Also, N cosθ = mg …(i) 9. (5.00) Let v1 = velocity of block 2 kg
f s(max) = µ mg
just before collision,
= 0.5 × 5 × 10 = 25 N and N sin θ = mrω 2 …(ii)
v 2 = velocity of block 2 kg just after
Limiting friction value for 10 kg block is On dividing Eq. (ii) by Eq. (i), we get collision,
f s(max) = µ ( m1 + m2 )g rω 2 and v 3 = velocity of block M just after
tanθ =
= 0.5 × 15 × 10 = 75 N g collision.
374 JEE Advanced~Practice Set 4

Applying work-energy theorem, and L2 = 540 cal-g −1 13. (1.16) Image coincides with object
(change in kinetic energy = work done Solving these two equations, we get when after refraction from the concave
by all the forces) at different stages as surface fall normally on concave
λ = 9.
shown in figure. mirror, so formed by silvering the
11. (11.00) The given system behaves like convex surface.
M a mirror of power given by Image after refraction from concave
/s surface should form at centre of
m
10 6 m h 1 h1 = 6 sin θ curvature of concave mirror, i.e. at a
θ = 0.3 m
2 kg distance of 20 cm on same side of
O
combination.
θ
For Fig. (1),
10 cm
∆KE = W friction + Wgravity x
P = 2 PL + PM
 1 m { v 2 − (10)2}  = − 6 µmg
1   −2 
or − = 2 
 2  1
 + 
1

F  02
.   022
. 
cos θ − mgh1
1 Let distance of pin is ‘x1’ from lens,
or v12 − 100 = − 2 [6 µg cos θ + gh1 ] As, PL =
f(m) then
cos θ = 1 − sin2 θ = 1 − ( 0.05)2 −1 −2 µ 2 µ1 µ 2 − µ1
and PM = = − =
≈ 0.99 f(m) R(m) v µ R
∴ v12 = 100 − 2 [( 6) ( 025
. ) (10) ( 0.99) Solving this equation, we get 15
. 1 . −1
15
⇒ − =
F = −1.1 m = − 110 cm − 20 − x1 − 60
+ (10) ( 0.3) ]
i.e. the system behaves as a concave 120
⇒ v1 ≈ 8 m / s ⇒ x1 =
= 15 cm
mirror of focal length is 18.33 cm. 8
For Fig. (2),
Using the mirror formula, When concave side of lens is filled
∆KE = W friction + Wgravity
1 1 1 with water, then the rays are first
+ = , we have
v u f refracted from a plane surface.
1 1 1 Let x2 be the distance of pin, then the
v2 − =− or v = 11 cm
v 10 110 plane surface will form it’s image at a
/s 4
1m i.e. virtual image will be formed at a distance of x2 (happ = µh) from it.
3
distance of 11 cm.
µ 2 µ1 µ 2 − µ1
12. (1.65) Now, − =
v u R
Before entering the water, the velocity 4
1
m[(1)2 − ( v 22 )] = − 6 µ mg cos θ of ball is 2 gh. If after entering the . −
15
15
. 4/3 3
2 water this velocity does not change, Given, − =
− 20 − 4 x2 − 60
+ mgh1 then this value should be equal to the
3
or 1 − v 22 = 2 [−6 µ g cos θ + gh1 ] terminal velocity.
⇒ x2 = 13.84 cm
= 2 [− ( 6)( 0.25)(10) ( 0.99) + (10) ( 0.3)] Shift in pin, so that its image again
= − 237
. coincides is
∴ v 22 = 247
. or v 2 ≈ 5 m /s a ∆x = x1 − x2
= 15 − 13.84
10. (9.00)
= 116
. cm (down)
0°C 400°C 100°C
w+T 14. (2.06)
A P B If µ = mass per unit length of the wire,
Therefore, then
Heat will flow both sides from point P,
2 r 2(ρ − σ ) g M 2.10
2 gh = ⋅ µ = = = 0175
. kg - m−1
L1 1 = 
dm Temperature difference 
 9 η I 12.0
dt  Thermal resistance  1
 2 r 2(ρ − σ ) g 
2 Speed of transverse wave in the
400
= ...(i)  ⋅  wire is
( λx )/ kA 9 η
∴h=  
v =
T
dm2 400 − 100 2g µ
L1 = ...(ii)
dt (100 − λ ) x / kA 2 r 4 (ρ − σ )2 g T
= × or v2 =
In about two equations, 81 η2 µ
dm1 dm2
= (given) 2 ( 3 × 10 −4 )4 (10 4 − 10 3 )2 × 9.8 or T = µv 2 = 0175
. × ( 343)2
= ×
dt dt 81 ( 9.8 × 10 −6 )2 = 2.06 × 10 4 N
L1 = 80 cal-g −1
= 165
. × 10 m 3
JEE Advanced~Practice Set 4 375

15. (d) To reach at the opposite point B, A B C 20. (a, b, c, d)


v m sinθ = v r . From this expression, we In general, the ionisation energy
can find the value θ. But observe decreases on descending a group
I
carefully that the value of v m < v r . FB FA I and increases on crossing a period
So, the component of v m cannot be because removal of successive
equal to v r . Thus, this case is not electrons becomes more difficult and
first ionisation energy < second
possible. x ionisation energy < third ionisation
To reach at B the displacement along d+x d–x
energy.
X-axis should be zero.
By reversing the direction of I, the The ionisation energy values of noble
For that v m sinθ must be equal to v r , direction of FA and FB will be reversed, gases are the highest in their periods
v m sinθ = v r because a great deal of energy is
Fnet = FB − FA (Here, FB > FA )
12 required to remove an electron from a
sinθ = = 1.2 I ′ Lµ o I I ′ Lµ o I
10 = − stable fully-filled shell of electrons.
2 π(d − x ) 2 π(d + x )
But the value of θ cannot be more than 21. (a, b, c)
one. I ′ Lµ o I  1 1  Three centre
= − H H H
2 π  (d − x ) (d + x )
two electron
So, this case is not possible for the bond
given values of v r and v m. C
I ′ Lµ o I  d + x − d + x  H 3C CH3
=
2 π  d 2 − x 2 
16. (d) If the man could reach at point B, Al Al
then find the time taken to cross the
d I ′ Lµ 0 I  2 x  H 3C C CH3
river, t = . =  
v m cosθ 2 π  d 2 − x2  H H H
First find the value of θ by equating If x is small than d, then x 2 <<< d 2.
v m sinθ = v r , then put this value in So, for this condition (b) BCl 3 is stronger Lewis acid than
time t expression. AlCl 3 due to greater extend of
I ′ Lµ 0 I x
Fnet = − ⋅ 2 pπ - pπ back bonding in AlCl 3.
Time taken to cross the river π d
d (c) Three center four
= I ′ Lµ 0 I
v m cosθ Fnet =− x …(i) electron bond
πd 2
As, he wants to reach at B, then
Fnet ∝ − x Cl Cl Cl
v m sinθ = v r
Above expression satisfy, the
v 12 1 Al Al
sinθ = r = = condition for SHM.
v m 24 2 Cl Cl Cl
For SHM, F = −mω 2 x
θ = 30° (d) Three center two
[m → mass of wire B]
electron bond
On comparing Eqs. (i) and (ii), we get
d
So, t =
v n cosθ I ′ Lµ o I µ o LII ′ H H H
mω 2 = , ω=
10 π mπ B B
=
24 × cos 30° 2a mπ H H H
10 × 2 5 T = = 2π
= = s ω µ o LII ′
24 × 3 3 22. (b) O
19. (b) K 2Cr2O 7 + 6H 2SO 4 + 4KCl
17. (b) The magnitude of FA = FB C(CH2)2CH3
→ 6KHSO 4 + 3H 2O + 2CrO2 Cl2
µ I µ I
I ′ L  o  = I ′ L  o  (J)
 2 πd   2 πd  Chromyl
So, wire B is at equilibrium. chloride
Cl
When a mixture of a metal chloride O
If we displace it towards A, then
and potassium dichromate is heated
FA > FB with conc. H 2SO 4 , orange red vapour C(CH2)2CH3
So, the net force ( FA − FB ) is towards or chromyl chloride are evolved. Cl2
wire A, so the equilibrium is unstable. 4KOH + CrO2 Cl2 → K2 CrO 4 FeCl3

18. (b) As, the direction of current in B is (J) (Yellow Cl


solution)
reversed, then the equilibrium Cl
become stable. So, now when we (K )
+ 2KCl + 2H 2O (A)
displace the wire B with a small
amount x, the net force on it will be 2CrO24 − + 2H+ → Cr2O 72− + H 2O (CH2)3CH3
proportional to − x, means wire will (K) Na-Hg
execute SHM. If we could find time
period of SHM, then it will be equal to Cr2O 27− + H 2SO 4 + 3SO 2 → SO 24− HCl

the time taken by wire B to reach at + Cr2(SO 4 )3 + H 2O Cl


the same point again. (M) Cl
Green coloured
376 JEE Advanced~Practice Set 4

NO2
⇒ 0.8 × 2 V = 0.6 × 0.0821 × 300
(CH2)3CH3
[Initial condition]
HNO3 OH
H2 SO4 V = 9.236 L
(1) Sia2 BH, THF
Cl (2) NaOH O Substituting value of V is Eq. (ii), we
(3) pH 7-8, H2 O2 have
Cl
OH p2 × 9.236 = (0.3 − 0.04) × 0.0821 × 390
(C)
p2 = 0.90 atm
NO2
24. (a, c, d) Let the solution contains x 27. (0.06) Consider the cell
–+ and y moles of NaHCO 3 and
H 2C CH CH2ONa Pt H 2(1 atm)| HA 2 || HA1|H 2 (1 atm)Pt
Na 2 CO 3 respectively.
At L.H.S,
⇒ x = a and x + 2 y = b
− 0.0591
Cl Thus, for complete neutralisation E = E° log [H+ ]2
H /H + OP H /H +
O CH2CH CH2 1
of Na 2 CO 3, 2y mol HCl (i.e. 2 a
(D) volume) would be required. Q − log [H+ ] = pH

NO2 ∴Volume of HCl required for ∴E = E° + .059 (pH)2 … (i)


H /H + OP H /H +
complete neutralisation of
NaHCO 3 = x = b − 2 a At R.H.S.
H2/Pd/C
If methyl orange is added at 0.059
= E° log [H+ ]1
l
E +
phenophthalein end point, then H + /H RP H + /H
1
Cl x + y mol HCl = ( b − a ) mL HCl
∴E = E° –0.059 (pH)1 ...(ii)
would be required. H + /H RP H + /H
O (CH2)3CH3
l
If the mixture is titrated against For acid HA1,HA1 - H+ + A1−
(E)
NaOH, only NaHCO 3 will react
[H ] = Cα = KaC
+
23. (b, c) H2O and x mol NaOH, i.e. ( b −2 a ) mL
H+ OH NaOH would be required. 1 1
Ethene Ethanol
∴ pH1 = pK a1 − log C
25. (4) Fe 3+ = 3d 5 and CN − is strong 2 2
1 1
C5H5N.CrO3.HCl ligand, paramagnetic. Similarily, pH 2 = pK a2 − log C
O l
Co 3+ = 3 d 6 and NH 3 is strong 2 2
Acetaldehyde
ligand, diamagnetic. (Q C are same)
O
+ l
Co 3+ = 3 d 6 and C2O 4 is a Using Eq. (i) and (ii), we have
O strong ligand, diamagnetic.
°
Ecell = EOP ° + + 0.059
+ E RP
NaOH/H2O (aldol condensation) l
Ni 2+ = 3 d 8 and Ni octahedral H /H + H /H

OH complexes are always  1 pk − 1 pk 


O paramagnetic.  2 a2 2 a1 
+ l
Pt 2 + = 5 d 8 and CN − is square
O = 0 + 0.059 × [5 − 3] / 2
planar complex, diamagnetic.
OH = + 0.059V or 0.06 V
3-hydroxy-2- l
Zn2+ = 3 d 10 and it is
methylbutanal diamagnetic. 28. (0.21) 2HI - H2 + I2
OH 26. (0.90) Initial condition for the two  0.96 
bulbs are : Initial moles  128  0 0
O
+ + T1 = 27 ° C or 300 K, p1 = 0.8 atm, = 7.5 × 10 −3
O
VI = VII = V Moles at equilibrium
3-hydroxy OH
3-hydroxy-2-methyl n = 0.6 mol (i.e 0.3 mol in each bulb) x x
butanal
pentanal
= (7.5 × 10 −3 − x )
If on heating flask II to 117°C, 2 2
(c) n moles of gas are diffused from Q Meq. of I 2 formed at equilibrium =
Br
CH2 
 CH2
Br2 flask II to I. Then, Meq. of hypo used
CCl4 Br
Ethene 1, 2-dibromo ethane Number of moles in flask W 1
× 1000 = 157
. ×
+
KOH. NR4Cl
– I = 0.3 + n E 10
CH 

 CH
Hexane-2,6-diol Number of moles in flask  W  of I = 1.57 × 10 −3
Ethyne
II = 0.3 − n
or   2
O  E
NaNH2, NH3
Na + Now, ∴Moles of I 2 formed at equilibrium
Ethyne Ethynyl Propionaldehyde
For I, 1.57 × 10 −3
sodium =
(1) NH3, ether
p2 × V = ( 0.3 + n) × R × 300 … (i) 2
(Q pV = nRT )
(2) NH4Cl, H2O = 0785
. × 10 −3
For II,
x
p2 × V = ( 0.3 − n) × R × 390 … (ii) or = 0785
. × 10 −3
2
OH On solving Eq. (i) and (ii), we have
. × 10 −3
x = 157
Pent-1-yn-3-ol n = 0.04 mol
JEE Advanced~Practice Set 4 377

∴Degree of dissociation ( ∝) of HI 31. (7.52) We have


Moles of HI dissociated T W BC = − 1 × R × ln 1162
= ∆S = 2.303 m × C p × log 2 .
Initial moles T1 = − 981 R
x . × 10 −3
157 Entropy change for heating water from ∴ W = W AB + W BC
⇒ =
7.5 × 10 −3
7.5 ×10 −3 27 °C to 100° C. = − (150 + 981) R
1000 4180 × 18 = − 1131 J = − 1131
. kJ
= 0209
. ≈ 021
. ∆S = 2.303 × × log
18 1000 33. (d)
29. (232) 373
= 910.55 J HO CH CH CH2OH
For acetone + benzene mixture 300
∆T = 278.40 − 277.12 = 128
. K Entropy change for heating 1 kg H 2O (X)
k f × 1000 × w to 1 kg steam at 100°C. OCH3
∆T =
m×W ∆H v 23 × 10 5 O3
∆S = = = 6166.21 J HO CHO
1000 × k f × 14
. T 373 (CH3)2S
. =
128 … (i)
100 × 58 +OH C—CH2OH
Entropy change for heating 1 kg
OCH3 (C2H4O2)
For solute ( A ) + benzene mixture (Let steam form 373 to 473 K.
(Y )
m be the molar mass of A) 473 C dT
∆S = ∫ m p
1000 × k f × 28 373 T 34. (a) OH
(278.40 − 277.76) = 473 (1670 + 0.49T ) dT
100 × m = m∫ CH3O
373 T
1000 × k f × 2.8
0.64 = … (ii) 473 1670 dT 473
100 × m = m∫ + m∫ 0.49 dT
373 T 373

Using Eqs. (i) and (ii), we have = m × 1670 × 2.303 log [T ]473
373 CH CH2CH2OH
m = 232 + m × 0.49 [T ]473
373
OH
30. (495.75) For the reaction, 473
7 = 1 × 1670 × 2.303 log H3CO
CI 2 O 7( g ) → CI 2( g ) + O 2( g ) 373
2 –H+
+ 1 × 0.49 × 100
At t = 0 p0 0 0 = 39072
. + 49 = 44572
. J
ρ
At t = ∞ 0 p0 7 ∴ Total entropy change CH CH2 CH2
p0
2 = ( 910.55 + 616621
. + 44572
. )J OH
At t = 15 min p0 − p p 7 = 7522.49 J = 7.52 kJ H3CO
p
2 –H+
32. (−1.13) The process mentioned in the
question can be shown by T-V graph
9
Total p(t = ∞ ) = p0 = 900 as:
2
p0 = 200 mm of Hg
A 35. (c) Compound ‘p’ is an amide, which
7
At t = 15 min : p0 + p = 240 mm T on alkaline hydrolysis gives amine (R)
Ad

2
iab

and acidification of alkaline mixture


80
ati

p= mm of Hg gives precipitate S (C 8H 8O 2). Thus,


c

7 there are seven carbon in the R.


B C
1 p0 Since, R gives tribromide with
Q k = ln Isothermal
t p0 − p Br2 - H 2O, it must be meta-toluidine.
200 1400 Thus,
⇒ 15 K = In = In … (i) V
80 1320 O H
200 −
7 W AB = ∆C AB = nC V (TB − TA )
P=CH3 C N
Let p′ be the decrease in partial 3
= 1 × R ( 400 − 500) = − 150 R
pressure of Cl 2O 7 after 140 min. 2 36. (d)
200 p
⇒ 140 K = ln …(ii) W BC = − nRT ln B … (i) SOCl2
200 − p′ pC S Me COCl
From Eq. (i) and (ii), we have Applying adiabatic condition between
p′ = 84.5 mm A and B. H 2C COH
γ
⇒ Total 1− γ
AlCl3, C6H6
pT = constant O
7 7
p = p0 + p′ = 200 + × 84.50 −5 −5 O
2 2 ⇒ 20.30 ( 500) 2 = ( 400) 2 . pB
= 49575
. mm of Hg Me C
pB = 1162
. atm
Substituting the value of pB in Eq. (i),
378 JEE Advanced~Practice Set 4

37. (a, d) We have, dy 42. (a, b, c, d) We have, f( x) = f(1 − x)


⇒ sec 2 x + y sec x = sec x
z1 = 5 + 12 i , | z2| = 4 dx 1
dy Put x= + x
(a)| z1 + iz2| ≤ | z1| + | iz2| = 13 + 4 = 17 ⇒ + y cos x = cos x 2
dx
⇒ f  + x  = f  − x 
1 1
(b)| z1 + (1 + i ) z2| ≥ || z1| − |1 + i|| z2||
IF = e ∫ 2  2 
cos x dx
= esin x
≥ |13 − 4 2| = 13 − 4 2
Hence, f  x +  is an even function
1
∴Solution is yesin x = ∫ esin x cos xdx
4 4  2
⇒ z2 + ≤ | z2| + = 4 + 1= 5
z2 | z2| = esin x + C  1
or f  x +  sin x is an odd function.
4 4 Now, for x = 0 and y = 1, we have  2
⇒ z2 + ≥ | z2| − = 4 − 1=3 1
z2 | z2| 1= 1+ C ⇒ C = 0 Also, f ′ ( x ) = − f ′ (1 − x ) for x =
2
⇒ yesin x = esin x ⇒ y=1
We have, f ′   = 0
1
z1 13 ∴ f( x ) = 1 2
(c) Minimum value of =
4 5 which is continuous, periodic,
z2 + 1
Also ∫ (1 − t )esin πt = − ∫
0
f( y)esin πydy
z2 differential and even function. 1/ 2 1/ 2
∴Option (a, b, c) are correct answer. [Q put 1 − t = y]

f ′   = 0, f ′   = 0,
13
z1 40. (a, b, d) The given equation, 1 3
(d) Maximum value of = Since,
4 3 x− 5 y−7 z+2  4  4
z2 + = = …(i)
−1
f ′   = 0
z2 3 1 1
Also,
x+ 3 y−3 z−6 2
38. (a, b, d) We have, and = = …(ii)
−3 2 4 ⇒ f ′ ′ ( x ) = 0 at least twice in [0, 1] by
π
sin−1  x 2 − 6 x +
17  −1
 + cos k = Any point P on Eq. (i) is Rolle’s theorem.
 2  2
( 3r1 + 5, − r1 + 7, r1 − 2 )
π Y
⇒ sin−1  x 2 − 6 x +
17  −1
 = − cos k and any point Q on Eq. (ii) is
 2  2 1/4 1/2 3/4
( −3r2 − 3, 2 r2 + 3, 4r2 + 6) the X′ X
= sin−1 k, direction ratio of PQ
O

where, k ∈ [− 1,1] ( 3r1 + 3r2 + 8, − r1 − 2 r2 + 4r1


x 2 − 6 x + 17 / 2 = k − 4r2 − 87 ) …(iii)
1 Suppose the line with direction ratio’s
⇒ ( x − 3)2 − = k Y′
2 2, 7, –5 will be proportional to the
1 DR’s given by Eq. (iii) 43. (37) ∆ABC is a right angle triangle:
⇒ ( x − 3)2 = k +
2 3r1 + 3r2 + 8 − r1 − 2 r2 + 4 ∠B = 90°
∴ =
Y 2 7 ⇒ ∠PBA = 90° − α
r1 − 4r2 − 8
= …(iv) ∠APB = 180° − ( ∠PAB + ∠PBA )
−5
–1 ∠APB = 180° − (α + 90 − α )
Solving Eq. (iv), we get r1 = r2 = − 1 ∠APB = 90°
O 3
X′ X
–1/2
So, the point of intersection In ∆APB, AP = 3cosα
–1 P( 2, 8, − 3) and Q( 0, 1, 2 ) and
C
intercepted length,

Y′ PQ = (2 − 0)2 + ( 8 − 1)2 + ( −3 − 2 )2 α
= 78
(a) The largest value of k is 1 for two 5 4
The equation of PQ is
distinct solution. ρ
x−2 y−8 z+ 3 λ−A
1 = =
(b) For real roots k + > 0 2 7 −5
2 α
41. (a, b, c, d) (a) Graph of α
k ∈  − , 1
1
∴ y = | x 2 − 4| x | + 3| is shown above. A 3 B
 2 
Clearly, the above equation AP AC
(c) The equation has unique solution if In ∆APC, =
| x 2 − 4| x | + 3| = 0 sinα sin( π − A )
1 1
k + = 0⇒k = − 3 cos α 5
2 2 has four solutions ±1 and ±3. ⇒ =
sinα sin A
39. (a, b, c) Consider the given So, (a) is correct option.
3
differential equation, (c) also f( x ) = 0 and y = a intersect at ⇒ tanα = sin A
5
8 distinct point, if 0 < a < 1. 3 4 12
dy dy ⇒ tanα = × =
tan2 x = sec x (1 − y) − Thus, f( x ) = a has 8 real roots for 5 5 25
dx dx 0 < a < 1. Here, m = 12, n = 25
dy (d) is also the correct option as y ≥ 0.
⇒ (1 + tan x ) = sec x (1 − y)
2
∴ m + n = 12 + 25 = 37
dx
JEE Advanced~Practice Set 4 379

44. (361) Given, words ‘ABBCABBC’ ⇒ log P = [3 log 3 − 3] − [2 log 2 − 2 ]  3 2   3q 


1 − (q − 3)  p +  =0
Total number of ways that can be − [(2 log 2 − 2 ) − ( 0 − 1)]  2   2 
8! ⇒ log P = log 27 − 3 − log 4 + 2 − log 4
formed = = 420 q (q 2 − 3)
2 !2 ! 4 ! + 2 −1 ⇒ p= = p = f(q )
4
Let ABBC be a unit, then this unit and 27
⇒ log P = log 27 − log 16 = log ⇒
dp 1
= ( 3q 2 − 3)
other ABBC can be arranged in 16
5! 27 dq 4
= 60 ∴ P=
2! 16 f(q ) is decreasing when,
But this includes ABBC ABBC two times Here, a = 27, b = 16 3q 2 − 3 < 0 ⇒ q ∈ ( − 1, 1)
∴Actual number = 60 − 1 = 59 ⇒ a + b = 27 + 16 = 43 ∴ a = − 1, b = 1
∴ Required number of ways 46. (128) We have, x 2 = 1 + 6log 4 y ⇒ | a + b| = |−1 + 1| = 0
= 420 − 59 = 361 and y 2 = 2 x y + 2 2x +1 48. (6) Given,
3n
Cn a = sin10° , b = sin 50° and c = sin70°
45. (43) Given, yn = x = 1 + 3 log 2 y
2
… (i)
2n
Cn 8 abc = 8 sin10° sin 50° sin70°
+1
y 2 − 2 x y − 2 2x =0 = 8 sin10° sin( 60° − 10° ) sin ( 60° + 10° )
3n ! n! n!
⇒ yn = × y = 2x +1 8
n! (2 n)! (2 n)! = sin 30° = 1
4
3n( 3n − 1)... (2 n + 1) and y = − 2 x (no possible, y > 0)
= a + b sin10° + sin 50°
+1 ∴ =
2 n(2 n − 1)... ( n + 1) If y = 2x c sin70°
(2 n + 1)(2 n + 2 )(2 n + 3)...(2 n + n) ⇒ log 2 y = x + 1 2 sin 30° cos 20°
= = =1
( n + 1)( n + 2 )( n + 3)...( n + n) Putting the value of log 2 y = x + 1 in cos 20°
∴  +
 2 n + 1  2 n + 2   2 n + 3  Eq.(i), we get 1 1 1 1 1
−  = +
=      x 2 = 1 + 3 ( x + 1) a b c  sin10° sin 50°
 n+ 1  n+ 2   n+ 3 
1
 2n + n ⇒ x2 − 3 x − 4 = 0 −
...   sin70°
 n+1 ⇒ ( x − 4)( x + 1) = 0
sin 50° sin70° + sin10° sin70°
 ⇒ x1 = 4 − sin10° sin 50°
n   n  =
= 1 +  1 +  sin10° sin 50° sin70°
 n + 1  n + 2 and x2 = − 1
⇒ y1 = 2 5 = 32 2 sin 50° sin70° + 2 sin10° sin70°
 n  − 2 sin10° sin 50°
...  1 +  =
 n + n and y2 = 2 = 10
2 sin10° sin 50° sin70°
Now, let P = lim ( yn ) n ∴| x1 x2 y1 y2| = |4 × ( −1) ( 32 ) (1)| = 128 cos 20° − cos 120° + cos 60°
n→ ∞ − cos 80° − cos 40° + cos 60°
47. (0) We have, =
1/ n 2 sin10° sin 50° sin70°
 n   n  → → 1 3$
1 +  1 +  a = 3 $i − $j, b = $i + j 3
= lim   n + 1  n + 2  2 2 + cos 20°− (cos 80° + cos 40° )
n→ ∞   n  → → → = 2
....  1 +  x = a + (q 2 − 3) b 2 sin10° sin 50° sin70°
  n + n  


Taking log both sides, we get ⇒ x = ( 3 $i − $j ) + (q 2 − 3)
3 + 2 cos 20° − 2 (2 cos 60° cos 20° )
=
log P = lim
1 1$ 3 $ 4 sin10° sin 50° sin70°
n→ ∞ n
 i + j
2 2  3 + 2 cos 20° − 2 cos 20°
  n   n   =
log  1 +  + log  1 +  . → → → 4 sin10° sin 50° sin70°
+ + and y = − pa + q b
  n 1  n 2  3
 n  =
 .. log  1 +  ⇒

y = − p( 3 $i − $j ) + q 4 sin10° sin 50° sin70°
  
  n + n  1$ 3 $ =6 [Q 8 sin10° sin 50° sin70° = 1]
1 n  1   i − j
⇒ log P = lim Σ log  1 +  2 2   a + b 1 1 1
∴8 abc   + − 
n→ ∞ n r = 1  1 + r/n  c  a b c
→  q 2 − 3 $
 1 1  ⇒ x= 3+ i = 1× 1× 6 = 6
⇒ log P = ∫ log  1 +  dx  2 
0  1 + x 49. (1023) We have,
 3 2 
 x + 2
1 − 1 − (q − 3) $j A 2B = BA … (i)
= ∫ log   dx  2 
0  x + 1 Now, ( AB)2 = ABAB
 3q  $
y =  3 p +  $i +  p +
→ q
1 ⇒ j = AA 2BB [from Eq. (i)]
⇒ log P = ∫0log( x + 2 )dx1  2  2 
− ∫ log( x + 1) dx ⇒ ( AB)2 = A 3B2
→ →
0
⇒ x⋅y =0 ⇒ ( AB)3 = ( AB)2 AB = A 3B2 AB
⇒ log P = [( x + 2 )log( x + 2 ) − ( x + 2 )]10 = A 3BBAB = A 3BA 2BB
 q 2 − 3  q
− [( x + 1) log( x + 1) − ( x + 1)]10 ⇒ 3 +   − 3p +  +
 2   2 = A 3BAAB2
380 JEE Advanced~Practice Set 4

= A 3 A 2BAB2 51. (c) Given, P( Bi ) =


i x y 1
= A A A BB = A B
3 2 2 2 7 3 10 + λ 3 7 1 =0
1 2
⇒ ( AB) = ( AB)3( AB) = A 7B3 AB
4 ∴ P( B1 ) = , P( B2 ) = , 6 5 1
10 10
S : x + y + x(2 λ − 9) + y ( 3λ − 12 )
2 2
= A 7B2 A 2BB P( B3 ) =
3
, P( B4 ) =
4
10 10 + 53 − 27 λ = 0
= A 7BA 2BAB2
E  C : x 2 + y2 − 4 x − 6 y − 3 = 0
= A B15 4 P( E1 ) = P( B1 ) × P  1 
 B1  Equation of common chords is
n
Similarly, ( AB) = A 2
n −1 n
B E  E 
+ P( B2 ) × P  1  + P( B3 ) × P  1 
∴ ( AB)10 = A 2
10
− 1 10
B  2
B  B3  S −C = 0
E  ⇒ x (2 λ − 5) + y( 3λ − 6)
= A 1023
B 10
+ P( B4 ) × P  1 
 B4  + 56 − 27 λ = 0
Hence, K = 1023 1 1 2 1 or λ(2 x + 3 y − 27 )
= P( E1 ) = × + ×
50. (25) We have, 10 1 10 2 − ( 5 x + 6 y − 56) = 0
f( x ) = ax + cos 2 x + sin x + cos x +
3 1
× +
4
×
1
⇒ 2 x + 3 y − 27 = 0
f ′ ( x ) = a − 2 sin2 x + cos x − sin x 10 3 10 4
or 5 x + 6 y − 56 = 0
4 2
Let g ( x ) = −2 sin2 x + cos x − sin x = = Solving these equation, we get
10 5
= − 2 {(cos x − sin x )2 − 1} 23
P ( B3 ∩ E 2 ) x = 2, y =
+ cos x − sin x 52. (c) P( B3 / E 2 ) = 3
P( E 2 )
Put cos x − sin x = t
∴Concurrent points at  2, 
23
E   3
∴ t ∈ [− 2 , 2 ] P ( B3 ) × P  2 
∴ g ( x ) = − 2 (t − 1) + t
2  B3 
= 54. (c) Centre of C lies on common chord
 E2  E 
= − 2t 2 + 2 + t P( B2 ) × P   + P( B3 ) × P  2  of S and C
 B2   B3 
17 ⇒ (2, 3) lies on
⇒ − 2 − 2 ≤ g ( x) ≤ E  x (2 λ − 5) + y ( 3λ − 6)
8 + P( B4 ) × P  2 
 B4  + 56 − 27 λ = 0
Since, f( x ) is strictly increasing.
∴ f ′( x) ≥ 0
3 1 ∴ 2 (2 λ − 5) + 3 ( 3λ − 6)
×
17 = 10 3 =
1 + 56 − 27 λ = 0
Hence, a ≥ 2 1 3 1 4 1 3 ⇒ λ =2
8 × + × + ×
10 2 10 3 10 4 ∴ S = x 2 + y 2 + x (2(2 ) − 9)
a ∈  , ∞ 
 17
 53. (b) Equation of circle S passing + y ( 3(2 ) − 12 ) + 53 − 27(2 ) = 0
 8
through the points A and B are
m = 17, n = 8 S = x 2 + y2 − 5 x − 6 y − 1 = 0
( x − 3) ( x − 6) + ( y − 7 ) ( y − 5)
∴ m + n = 17 + 8 = 25

Paper 2
λD 2. (a, b) For slow compression, no net gain in
1. (a, d) Using, Y =
d Work done by force is equal to KE of block, so work done by extreme
We have, force is equal to potential energy
3λ1 ⋅ D potential energy stored by spring,
y1 = 3 β = 1 stored in spring.
d i.e. W = kx 2
For same deformation in springs,
3 × 6500 × 10 −10 × 1.2 2
= and magnitude of force is given by k k
2 × 10 −3 WA = A x2 ⇒ WB = B x2
| F| = k x 2 2
. × 10 −4 m = 117
= 117 . mm
Here, it is to be noted that at Q kA > kB
Let n1 bright fringe of λ1 meets with n2 equilibrium same force is applied, So, WA > WB
bright fringe of λ 2, then
n 4
if F1 = k A x1 For same force at equilibrium,
n1λ1 = n2λ 2 or 1 = and F 2 = k B x2 F = k A xA = k B xB
n2 5
As, F1 = F 2 ⇒ k A x1 = k B x2 kA > kB
So, 4th bright fringe of λ1 meets with
5th bright fringe of λ 2 and 8th bright kA So, xA < xB
fringe of λ1 meets with 10th bright F1 k A xA2 ( k A xA )2
WA = =
fringe of λ 2, etc. 2 2 kA
4λ1D 4( 6500 × 10 −6 ) (12
. ) k B xB2 ( k B xB )2
So, Ymin = = kB
WB = =
d 2 × 10 −3 F2 2 2 kB
= 15.6 × 10 −4 m = 156
. mm For k A > k B, W A < W B
JEE Advanced~Practice Set 4 381

3. (a,c,d) 6. (a,b,c,d) From the graph, we can see


– q2′ +q2′ 1
For the formation of standing wave, λ = = 0.5 yr −1
two waves of same frequency and 2
same amplitude, wave must travel in +q1 A B – q2 Now, applying the equation
opposite directions with same speed – q′1 +q′1 N = N0 e − λt = N0 e −0. 5 × 4.16
also resultant amplitude of two waves
N0
on superposition is given by Due to induction effect, the situation is = N0 e −2.08 = 0125
. N0 =
shown clearly in figure. Due to + q 1, 8
AR = A12 + A 22 + 2 A1 A 2 cos φ
net induced charges is q 1′ at end A i.e. nuclei decreases by a factor of 8.
As, both waves are travelling in same and q 1′ at end B while due to − q 2 Hence, the answer is 8.
direction, so resulting wave cannot be induced charges are q 2′ and q 2′ at
standing, because for standing wave, 10. (4.00)
ends A and B, respectively. Thus, the In adiabatic process,
two waves must be moving in
end A acquires negatively charged
opposite direction. TV γ − 1 = constant
and B acquires positive charge.
A 2R = A12 + A 22 + 2 A1 A 2 cos θ Electric force experienced by q 1 or ∴ Ti Vi0.4 = TfTf0. 4
π
= ( 3)2 + ( 3)2 + 2 ( 3) ( 3) cos   − q 2 has to be computed by using (as, γ = 14
. for diatomic gas)
 3 principle of superposition. 0. 4
A 2R = 27  V 
or Ti Vi0.4 = (α Ti )  i 
A R = 3 3 cm 7. (0.20)  32 
The equation is given by y = f ( x, t ) Least count of Vernier calipers
or α ( 32 )0.4 = 4
that means wave is moving in LC = 1 MSD − 1 VSD
x-direction and particles are oscillating Smallest division on main scale 11. (2.00)
=
in xy-plane. Number of divisions on vernier scale Voltage across the capacitors will
4. (a,c) increase from 0 to 10 V exponentially.
20 divisions of vernier scale The voltage at time t will be given by
= 16 divisions of main scale V = 10 (1 − e −t / τ C)

∴1 VSD =
16
mm = 0.8 mm Here, τc = Cnet Rnet
y 20
= (1 × 10 6 ) ( 4 × 10 −6 ) = 4 s
∴ LC = 1 MSD − 1VSD
v ∴ V = 10 (1 − e −t / 4 )
N = 1 mm − 0.8 mm
= 02
. mm Substituting V = 4 volt, we have
The impact force of the emerging
4 = 10 (1 − e −t / 4 )
liquid on the vessel + liquid content is
equal to 3
8. (10.00) or e −t / 4 = 0.6 =
vdm v  5
F = = va ρv = aρv 2 2 1  = 2
v
dt  2 l1  4l 2 Taking log both sides, we have
F = a ρ( 2 gy )2 = 2 aρgy t
T /µ 320 − = ln 3 − ln 5
∴ = 4
Now, consider the condition, to slide l1 4l 2
or t = 4 (ln 5 − ln 3) = 2 s.
the vessel f ≥ F
(where, µ = mass per unit length of 12. (7.00) Magnetic field at point P due to
fmin = F = f ( say) wire) wires BP and RQ is zero. Only wire QR
The force of friction = f = F = 2 aρgy 50 / µ 320
or = will produce magnetic field at P.
⇒ µN = 2 aρgy 0.5 4 × 0.8 r = 3 x cos 37 °
⇒ µ ( A ρgy) = 2 aρgy
= ( 3 x )   =
Solving, we get 4 12 x
2a µ = 0.02 kg/m = 20 g/m  5
⇒ µ = 5
A
∴Mass of string =  20  (0.5 m)
g Q
5. (c, d)  m
At constant temperature density = 10 g 53°
increases. From ideal gas equation, 3x
RT
9. (8.00) 5x
p=ρ , p∝ρ dN
M = |Activity of radioactive r l
dt
If ρ increases, then p also increases, substance| l 37°
53° 37°
so that it is isothermal compression, = λN = λN0e − λt P R
because if p increases volume has to 4x l
decrease. Taking log both sides,
µ0 I
pA VA = pB VB
dN Now, B = ⋅
ln = ln ( λN0 ) − λt 4 π 12 x / 5
dt
pB > pA (ρB > ρA )
dN [sin 37 ° + sin 53°]
∆W < 0 Hence, ln versus t graph is a
µ I
∆U = 0
dt = 7  0 
straight line with slope − λ.  48 πx 
(isothermal process)
382 JEE Advanced~Practice Set 4

13. (8.00) Work done is area under p-V graph. If 20. (a, c, d) In the given situation, first
Net pulling force volume increases work done by gas period would have 3 elements. H, He
Total mass is positive and if volume decreases and Li. Number of groups = 15
. times
. g − 0.36g
072 g work done by gas is negative. number.
= = Further ∆U is positive if product of
. + 0.36
072 3 21. (b) Compound H undergoes aldol
pV is increasing and ∆U is negative,
= at 2 =   (1)2 =
1 1 g g if product of pV is decreasing. condensation reaction followed by
2 2  3 6 If heat is taken by the gas Q is
dehydration of aldol that gives
α, β-unsaturated carbonyl compound,
positive and if heat is lost by the gas
not the β, γ -unsaturated carbonyl as
Q is negative.
major product.
16. (a) (A→ p t, B→ p s , C→ q s; D→ q r)
In organ pipes, longitudinal waves CHO
a T
are formed.
a As a result, B is
0.36 kg In string, transverse waves are .
T formed. Open end of pipe is
0.36 g
0.72 kg displacement anti-node and closed
end is displacement node. In case of In the second reaction, following steps
0.72 g string fixed end of a string is node. take place.
Further, least distance between a In the second reaction, following steps
g λ
T − 0.36 g = 0.36 a = 0.36 node and an anti-node is and take place.
3 4
∴ T = 0.48g λ (a) In first step, the more reactive
between two nodes is .
2 aldehyde group is acetylated and
Now, WT = TS cos 0°
then ketonic group is reduced with
(on 0.36 kg mass) 17. (d) (A → r s t, B → q r s t, C → q p,
D → q r s t) LiAlH 4 .
= ( 0.48 g )   (1)
 g
In circuit P, I cannot be zero in steady (b) In the final step, hemiacetal formation
 6
state. occur at aldehyde group and led to
= 0.08 ( g 2 ) OH
In circuit Q,
O
= 0.08 (10)2 = 8 J V1 = 0 and V2 = 2 I = V . the formation of
(CC)
14. (2.00) In circuit R, 22. (b)
From Gauss theorem, V1 = X 2, V = (2 πfL )I O O
∴ V = (2 π × 50 × 6 × 10 −3 ) I = 188
. I σ
q
E∝ 2 V2 = 2 I CH3 COC CH3 + CH3 COO
r
In circuit S, O O
(where, q = charge enclosed)
V1 = ( X L ) I = 188
. I σ
E 2 q 2 r12 V2 = Xc I = (1061) I CH2 C O C CH3 +CH3COOH
∴ = = 2
E1 q 1 r2 In circuit T, O
R V1 = IR = (1000)I O O
∫ ( 4 πr
2 a CH
)kr dr V2 = Xc I = (1061) I σ
2
0 (R / 2 ) + CH2 CH3
or 8= × 18. (c) ( A → p r, B → q s t, C → p r t, C O C
R/ 2 ( R )2 D → q s)
∫ ( 4 πr
2
)kr adr O

O O
0 (A) → since µ 1 < µ 2, the ray of light will
Solving this equation, we get bend towards normal after first C CH2 COC CH3
a=2 refraction.
(B) → since µ 1 > µ 2, the ray of light H
15. (a) (A → p r t, B → p r,
will bend away from the normal after
C → q s, D → r t) Hρ
first refraction.
Internal energy ∝ T ∝ pV (C) → since µ 2 = µ 3, means in OH
This is because second refraction there will be no
nf f C CH2 COOH
U = RT = ( pV ) change in the path of ray of light.
2 2 Hρ
(D) → since µ 2 > µ 3, ray of light will H
Here, n = number of moles and bend away from the normal after
CH CHCOOH
f = degree of freedom. second refraction.
If the product pV increases, then (Q) H2,Pd,/C, MeOH
19. (a, b, c) In Pt (NH 3 )2Cl 2, platinum is
internal energy will increase and if in + 2 oxidation state with d 2
product decreases the internal energy CH CHCH2OH
configuration. It has dsp2
will decrease.
configuration with square planar (i) SOCl2
Further, work is done on the gas, if
geometry. Number of unpaired
volume of gas decreases. For heat electrons are zero. So, it is CH CHCH2Cl
exchange, AlCl3
diamagnetic in nature.
Q = W + ∆U
JEE Advanced~Practice Set 4 383
+ – °2
CH CH CH2AlCl4 RTf 29. (200) In the presence of 0.5 mole of
For benzene, K f =
1000 l f (cal /g ) Na 2 CO 3 and 4-hydroxy phenol, the

8.314 × (2847 2
nucleoplile is

. )
∴ Kf = HO
.
11062 × 1000
1000 ×
23. (b) Using the formula, 78 (M. W = 109)
. Km−1
= 475
Enthalpy of solution = lattice energy + Now, let molar mass of leaving group
hydration enthalpy ∆Tf = K f × molality × [1 − (α / 2 )] … (ii) be X. Then.
171 − x + 109 = 200
or ∆ sol H s = ∆ lattice H + ∆ hyd H
s s Molality of acetic acid in benzene
W × 1000 W A × 1000 ⇒ x = 80 (Br)
∴ ∆ hyd H s = ∆ sol H
s
– ∆ lattice H s = A = Therefore, in substrate, molar mass of
M A × wB w
MA × B × MB remaining group is 171 − 80 = 91
. − 180 = − 179.0 kcal mol −1
= 10 MB
nA × 1000 0.04 × 1000 As given is question, ‘A’ has a phenyl
The ratio of hydration energies of Na + = =
nB × M B 0.96 × 78 ring (C 6H 5 = 77) remaining part
and Cl − is 6 : 5.
has molecular weight = 14 (CH 2 )
∴Hydration energy of = 0.534 M [Q xA = 0.04 mol ]
6 Hence, C 6H 5CH 2Br( A )
Na + = × ( − 179.0) Substitutiing value of m in Eq. (ii) we have
11 and
= − 97.63 ≈ − 98.0 kcal mol −1 2 = 475
. × 0.534 × (1 − α / 2 )
24. (a, b, c, d) The two end points at V1 [Q ∆Tf = Tf ° − Tf ) HO O CH2 (B)
and V2 indicates that weak acid being α = 0.42
titrated is dibasic. Assuming molarity = molality, then The molar mass of (A) is 200.
H 2 A + OH − → H 2O + HA − from Eq. (i), 30. (0.01) p(NO) = 2 p (Br2 ) (from
HA − + HO − → A 2− + H 2O 0.534 × 0.42
KC = = 117
. balanced equation)
l
If first end point is obtained on 2 × ( 0.534)2 × (1 − 0.42 )2
Because it is 34% dissociated.
adding V1 mL of base then, V2 mL
27. (− 19.67) The reaction quotient QC for p(NOBr ) = 0.66 p° (NOBr)
of same base would be required to
the reaction is p(NO ) = 0.34 p° (NOBr ) and
further reach the 2nd end point.
l
At half-way of the first end point, pH N2 O 4 - 2NO 2 p(Br2 ) = 017
. p° (NOBr )
= pK a1 [NO 2 ]2 0.5 × 0.5
QC = = = 0.5 p(NOBr) + p(NO) + p(Br2 ) = 025
. atm
l
Similarly pH corresponding to [N 2O 4 ] 0.5
mid-point between V2 and V1 0.66 p° + 0.34 p° + 017
. p° = 2.5 atm
corresponds to pK a . Since, Q p = QC ( RT )∆n . p° = 025
117 . atm
2
and ∆n = 2 − 1 = 1 0.25
25. (b) Following compounds gives O 2 on p° (NOBr) = = 0214
. atm
∴ Q p = 0.5 ( 0.082 × 291) =1193
. 1.17
decomposition:
Now, p(NOBr ) = ( 0.66) ( 0214
. atm) = 014
. atm

2KClO 3  → 2KCl + 3O 2 ∆n
K p = KC ( RT ) = 114
. ( 0.082 × 291)

NaNO 3  → NaNO 2 + O 2 p(NO ) = ( 0.34) ( 0214
. atm) = 0.073 atm
= 27.1

2KMnO 4 → K 2MnO 4 p(Br2 ) = 017
. ( 0214
. atm) = 0.036 atm
Substituting these values in the
+ MnO 2 + 3O 2 following equation, we get p(NO )2 p(Br2 )

Q Kp =
4K 2Cr2O 7  → 4K 2CrO 4 ∆G = ∆G ° + RT ln Q p p(NO )
+ 2Cr2O 3 + 3 O 2 = − RT ln K p + RT ln Q p = 0.0098 × 10 −3 ≈ 0.01 × 10 −3

2H 2O 2  → 2H 2O + O 2 = − 2.303 RT (log K p − log Q p )

31. (99.97) The cell reactions are:
2Pb 3O 4  → 6 PbO + O 2 ∆G = − ( 0.082 × 291 × 2.303) At anode H 2 → 2H+ + 2e −
Decomposition of other given
(log 27.2 − log 1193
. ) At cathode 2 Ag+ + 2e − → 2 Ag
compounds are as follows:
− 54.95 (1.4346 − 1.0766) ° °

NH 4NO 2  → N 2 + 2H 2O Thus, Ecell = EOP H 2 + E RP Ag

= − 19.67 L-atm
2NH 4NO 3  → 2N 2O + 4H 2O 0.059 [Ag+ ]2 × PH 2
28. (512) Total nine (9) stereocentres are + log
∆ [H+ ]2
(NH 4 )2Cr2O 7 → Cr2O 3 + H 2O + N 2 present in given compound (sucrose).
2
26. (1.17) For acetic acid, HO or
0.059
2CH 3COOH - (CH 3COOH)2 * O O * O 0.503 = 0 + 0.80 + log [Ag+ ]2
HO OH 2
Initial conc. C 0 * *
* or [Ag+ ] = 9225
. × 10 −6 M
After association C(1 − α ) Cα / 2 * * *
HO * OH ∴Mole of Ag+ in 350 mL
Total number of particles at OH OH 350
equilibrium = 1 − α + α / 2 OH = 9225
. × 10 −6 × × 108
1000
= 1− α /2
Cα / 2
Hence, total number of stereoisomers = 3.487 × 10 −4 g
∴ KC = … (i) = 2 9 = 512 .
(C (1 − α ))2
384 JEE Advanced~Practice Set 4

CH2Cl
∴% of Ag in 1.05 g alloy CH2
O
−4
3.487 × 10
= × 100 = 0.033% CH
105
. + +
∴% of lead (Pb) in alloy = 99.97% CH
ClCH2
32. (174 nm) For normal O-atoms CH2
O
 498 × 10 3 J   1 mol O 2  AlCl3
O
   
 mol O 2   6.02 × 10 molecules O 2 
23

. × 10 −19 J
CrO3
= 827
Extra energy for excited ‘O’-atom
 1.60 × 10 −19 J  –2H
(1.967eV)   O
 1 eV 
. × 10 −19 J
= 315 Q. Zn
Red hot
Total energy for excited ‘O’-atom Fe tube
3CH CH + 2CH2Br2
873 K
hc
= 1142
. × 10 −18 J ⇒ λ = 35. (b) P → 2; Q → 1; R → 4; S → 3
E OH
O
(6.63 × 10 −34 J.s) (3 × 10 8 m / s) H+
= AlCl3 LAH
1.142 × 10 −18 J
= 1.74 × 10 −7 m = 174 nm
33. (a) O3/Red CHO
–2H CHO
Chemical Structure Hybridi Shape
formulae -sation

OSF2 sp 3 Trigonal R. O O O
pyramidal CH O
S CHO
C OH
F F D + HCl
CHO
O
Protection
O 2SF2 F sp 3 Tetrahedral C OH of CHO group

S O COOH COOH
O F O
O
C O O O O
sp 3 d 2

H OH
AlCl3
O H
XeF4 Square
F F O+
planar H
Xe
C SOCl2 DIBAL-H
F F
ClO −4 O – sp 3 Tetrahedral O .

O
COCl CHO
Cl
O O C CHO
H2SO4
O

I −3 I sp 3 d Linear H3O+
COOH
:
: I O
:
I CHO
Zn
O
34. (b) P → 2; Q → 1, 2; R → 3, 2, 4; Me
S → 5, 6
O Ph
CH3
PCl5

Anhyd. Cl Cl
P. + CH3Cl Me
AlCl3 O
Ph
2NaNH2
CH2Cl CrO3
CH3COOH Ph C C Me
Cl2,hν
O
JEE Advanced~Practice Set 4 385

⇒ 2 cos θ − 2 cos θ sin φ Pre multiply by M


H 2C O = 2 sinθ cos φ − 1 ⇒ ( M 2 + MN 2 )( M − N 2 ) = 0 … (ii)
⇒ 2 cos θ + 1 = 2 sin (θ + φ)
Let M − N = U ⇒ from Eq.(ii) their
2
NaBH4
H 2C OH ⇒ tan(2 π − θ) > 0 ⇒
EtOH exist some non-zero
tanθ < 0
U ( M 2 + MN 2 )U = 0
3
36. (C) P → v; Q → i; R → iii; S → ii and − 1 < sinθ < − 40. (a, b, c)∴ f ′ ( x) = 0 has a root α 1,
2
l
(P) → v such that 0 < α1 < α 0
3π 5π 
According to Boyle’s ⇒ θ ∈  ,  [using Rolle’s theorem]
law, “for a fixed  2 3 
Now, f ′( x ) = 0 is a fourth degree
p (atm)

amount of gas, at 1
< sin(θ + φ) < 1 equation. As imaginary roots occur in
constant temperature, 2
the product of pairs, f ′( x ) = 0 will have another real
1/V (L–1) π 5π root α 2.
pressure and volume is ⇒ 2π + <θ + φ< + 2π
6 6 As f ( x ) = 0 is an equation of the degree
constant, i.e. pV = constant or
1 K π 5π
p ∝ ⇒ p = . Thus, the graph ⇒ 2 π + − θmax < φ < 2 π + − θmin five, it will have atleast 3 real roots.
V V 6 6 Thus, f ′( x ) will have at least two real
1 π 4π roots and f ′′( x )will have atleast one real
between pV / s gives < φ<
V 2 3 root.
a straight line.
38. (a, c, d) We have, 41. (a, b, d) We have,
(Q ) → (i)
p (atm)

A plot of pressure of x 2 + y 2 − 10 x − 6 y + 30 = 0 a2 + b 2 = 7
an ideal gas as a Centre O ( 5, 3) ⇒ a 2 + b 2 = ( a + b )2 − 2 ab = 7
function of its volume V (L )
Radius = ( 5)2 + ( 3)2 − 30 = 4=2 ⇒ a 3 + b 3 = ( a + b ) ( a 2 + b 2 − ab )
yields a plot whose form is that of
rectangular hyperbola.
l
(R) → (iii) ⇒ 10 = ( a + b ) (7 − ab )
On taking log of  ( a + b )2 − 7 
Log p

equation pV = K , ⇒ 10 = ( a + b )  7 − 
 2 
we get O (5, 3) A (7, 3)
 ( a + b )2 − 7 
Q ab =
Log V
log p + log V = log K 2 
 
⇒ log p = − log V + log K … (i)
 21 − ( a + b )2 
On comparing Eq. (i) with ⇒ 10 = ( a + b )  
B (5, 1) C (7, 1)  2 
y = mx + c, the graph which is
plotted is as follows: ⇒ 20 = ( a + b ) (21 − ( a + b ) )
2
(a) Area of quadrilateral
l
(S ) → (ii) ⇒ ( a + b )3 − 21 ( a + b ) + 20 = 0
= OA 2 = 2 2 = 4
K a+ b= x
On taking log of equation p = Let
V (b) Radical axis of family S = 0 is line
∴ x 3 − 21x + 20 = 0
⇒ log p = log   + log K
1 passing through AB equation of
V AB = x − y = 4 ⇒ ( x − 1)( x + 5) ( x − 4) = 0
and comparing it with equation (c) Smallest possible circle of family ⇒ x = 1, − 5, 4
y = mx + c, the graph obtained is S = 0 has AB as diameter given by Hence, a + b = 1, a + b = − 5 and
( x − 5) ( x − 7 ) + ( y − 3) ( y − 1) = 0 a + b =4
⇒ x 2 + y 2 − 12 x − 4 y + 38 = 0
Log p (atm)

(d) The coordinates of C are (7, 1).


42. (a, b, c, d) Given,
x2 y2
39. (a, b) (a) ( M − N )2 ( M + N 2 ) = 0 … (i) ellipse : + =1 … (i)
16 7
[∴MN = N M ]
2 2
Log 1 ∴ e1 = 1 −
7
=
3
V ⇒ | M − N || M + N 2| = 0
2
16 4
37. (a, c, d) We have, Case I If| M + N 2| = 0 Foci ( ae1 = 0) = ( ± 3, 0) Hyperbola
2 cos θ (1 − sin φ) = sin2 θ x2 y2
∴ | M + MN | = 0
2 2 = − =1 …(ii)
144 / 25 81 / 25
 tan θ + cot θ  cos φ − 1 Case II If| M + N 2| ≠ 0
  81 / 25 5
 2 2 e2 = 1 + =
⇒ M + N 2 is invertible. 144 / 25 4
sin θ ⋅ cos φ
2
⇒ 2 cos θ(1 − sin φ) = −1 From Eq.(i) Foci = ( ± ae 2, 0) = ( ± 3, 0)
θ θ
sin cos ( M − N 2 )( M + N 2 ) ( M + N 2 )−1 = 0 (a) Foci are same
2 2
2 sin2 θ cos φ ⇒ M − N 2 = 0 which is wrong. (b) Solving Eqs. (i) and (ii), we get
⇒ 2 cos θ (1 − sin φ) = −1
sinθ (b) ( M + N 2 )( M − N 2 ) = 0 x=±
16
5
386 JEE Advanced~Practice Set 4

π /2
and y = ±
3 7 Now, Let I = ∫0 (cos f( x ))2dx 48. (4) We have,
5 2  2 + 4 − x2 
π /2 π  y = ln   − 4 − x2
Square of ordinate
2
I= ∫0  cos f  − x   dx
 2   2 − 4 − x2 
 
3 7 63
y2 =   = Q af( x ) dx = x ) dx  Put x = 2 sinθ
a
 5  25  ∫0 ∫0 f ( a − 
 2 + 2 cos θ 
2 ∴ y = ln   − 2 cos θ
(c) Equation of asymptote π /2 π   2 − 2 cos θ 
y=± x
3 ⇒ I= ∫0  cos  − f( x )  dx
 2  θ
4 π /2
= ln  cot 2  − 2 cos θ
 2
∴ Point P =  ±
16

12 

⇒ I= ∫0 (sin f( x ))2dx
 5 5 π /2
⇒ y = 2 lncot θ / 2 − 2 cos θ

Sum of square of coordinate of


⇒ 2I = ∫0 (cos 2 f( x ) + sin2 f( x ))dx dy − cosec 2 θ /2
= + 2 sinθ
π /2 dθ cot θ /2
P=
256 144
+ = 16 ⇒ 2I = ∫0 dx
− 2 cos 2 θ
25 25 =
π π sinθ
(d) Auxiliary circle formed by ellipse is ⇒ 2I = ⇒ I=
2 4 dx
x 2 + y 2 = 16 = 2 cos θ
1 dθ
∴ k = = 025.
∴ P lies on auxiliary circle. 4 dy
1 ∴ = − cot θ
→ → → dx
43. (0) e − |ln { x}|
− { x}
|ln { x}|
= [sgn( x )] 46. (1) Given|a| = |b| = |c| = 1
Equation of tangent at P is
→ →→ →→ → → →
 − 1, x< 0 [b × c c × a a × b ] = (b × c ) ( y − 2 lncot θ / 2 + 2 cos θ)

[sgn( x )] =  0, x=0 → → → → = − cot θ( x − 2 sinθ)
 1, . [(c × a ) × (a × b )]
 x> 0 ∴ T = ( 0, 2 lncot θ /2 )
→ → → → → → → → → →
1
= (b × c ) . [(c × a ⋅ b ) a − [(c × a ) . a )b ] P = (2 sinθ, 2 lncot θ / 2 − 2 cos θ)
− |ln { x}| |ln { x}|
e − { x} = 0, ∀ x ∉ Z → → →→→ → PT 2 = (2 sinθ)2 + (2 cos θ)2
= (b × c ) . [a b c ] a
∴ No solution exists. = 4 (sin2 θ + cos 2 θ) = 4
→ → → →→→
44. (150) We have, = [a . (b × c )]2 = [a b c ]2
cos( x + α ) sin( x + α ) sin (β − γ )
49. (8.5) Line through point P ( − 2, 3, − 4)
→→→ → →→ →→ →
f ( x) = cos ( x + β ) sin( x + β) sin( γ − α ) ⇒ [a b c ] = [b × c c × a a × b ] and parallel to the given line
cos( x + γ) sin( x + γ ) sin( α − γ ) →→→ →→→ 2 x + 2 2 y + 3 3z + 4
⇒ [a b c ] = [a b c ] = = is
− sin( x + α ) sin( x + α ) sin (β − γ ) 3 4 5
→→→
f ′ ( x) = − sin ( x + β ) sin( x + β) sin( γ − α ) ⇒ [a b c ] = 1 x+2 y + 3/2
⇒ =
− sin ( x + γ) sin( x + γ ) sin( α − γ ) →→→
3 2
⇒ |a b c| = 1 z+ 4/3
= =λ
cos( x + α ) cos( x + α ) sin (β − γ ) → → → → 5/3
Projection of b + c on a × b is
+ cos ( x + β ) cos( x + β) sin( γ − α ) + Any point on this line is
→ → → → →→→
cos( x + γ) cos( x + γ ) sin( α − γ ) (b + c ) . (a × b )
Q  3 λ − 2, 2 λ − , λ − 
[a b c ] 3 5 4
= =
cos( x + α ) sin( x + α ) 0 → → → →  2 3 3
|a × b| |a × b|
cos ( x + β ) sin( x + β) 0
1 Direction ratio of PQ are
cos( x + γ) sin( x + γ ) 0 = =1  4λ − 9 5λ + 8 
1 3λ, ,
⇒ f ′( x) = 0 + 0 + 0 = 0  2 3 
47. (2020) Let z = x + iy
∴ f( x ) = C , f( x ) = 6 Now, PQ is parallel to the given plane
25 ∴ z = x − iy
∴ Σ 6 = 150 4 x + 12 y − 3 z + 1 = 0
r =1 Now, z = re θ , z = re − i θ
Hence, line is perpendicular to the
45. (0.25) We have, ∴ ( x + iy)2018 = ( x − iy) normal to the plane.
π π π 4λ − 9 
f  + x  + f  − x  = ⇒ ( re iθ )2018 = e − i θ Thus, 4( 3λ) + 12  
6  3  2  2 
⇒ r 2018 . e i 2018 θ = re − i θ
π 5λ + 8 
x is replaced by − x, we get 2017 − 3   =0
e i 2019 θ =  
3 1  3 

 π π π π π  r
f  + − x  + f  −
 + x  = ⇒ λ = 2 ⇒ Q  4, , 2 
5
6 3  3 3  2 ∴e i 2019 θ has 2019 roots  2 
π π
⇒ f  − x  + f( x ) = ∴Number of total ordered pair of ( x, y) 2
6 2 +  − 3 + 6 2
2  is 2019 + 1 = 2020 5
2 ⇒ PQ = d =
2 
π π
f  − x  =
[Q (0, 0) is also roots possible value]
⇒ − f( x ) 17
2  2 = = 8.5
2
JEE Advanced~Practice Set 4 387

50. (77)
1
ΣΣ i . j = 50
Cm Q. Centre C1 = (2 R 3, 2 r )
15 1 < i < j < 9 q q
[Q C r + pC r − 1 C1 + PC r − 2 C 2
p
= ( 3 3 r, 2 r )
p +q
1  ΣΣ i . j + 1 + 2 + 3 
2 2 2 + .... + q C r = Cr ] Centre C 2 = ( R 3, R )
= 1 < i < j < 9 
15  + ... + 9 2  P. Clearly, f( m) is maximum when  3 3 r 3r 
m = 25. = , 
 (1 + 2 + 3 K + 9) 2  2 2
∴ Maximum value of f( m) is 50C 25 .
1  − (12 + 2 2 + ... + 9 2 ) Distance between their centre
= 
15  2 Q. Clearly,
50 50 r 2 27 r 2
 Σ f ( m) = Σ 50
Cm = 50
C0 C1C 2 = + = 7 r
m= 0 m= 0 4 4
+ 12 + 2 2 + 3 2 + ... 9 2 ]
+ C1 + ... +
50 50
C 50 R. Length of common tangent
1
= [(1 + 2 + 3 + ... + 9)2
30 =2 50
= (C1C 2 )2 − ( R + r )2
+ (12
+ 2 + 3 + ... 9 )]
2 2 2
R. Clearly,
25r 2 3
50 50
= 7r2 − =
1   9 × 10   9 × 10 × 19  
2 r
Σ ( f( m))2 = Σ ( 50C m )2 = ( 50C 0 )2
=   +   m= 0 m= 0
4 2
30   2   6 
 S. The value of length CE = 2 rcot 30°
2
+ ( C1 ) + ... + ( 50C 50 )2 =
50 100
C 50
1 9 × 10  9 × 10 19  = 2 3r
= × × + [Q ( C 0 ) + ( C1 ) + ...+ ( C n ) =
n 2 n 2 n 2 2n
Cn]
30 2  2 3 
54. (c) We have, f( x + y) = e y f( x) + e x f( y)
3 308 S. Consider,
= × = 77 Differentiate w.r.t. x y as constant,
2 6 3f( 0) − 8f(1) + 13f(2 ) − 18f( 3)
we get
51. (a) Let f( x) = a0 + a1 x + a2 x 2 + .... + 253 f( 50)
50 f ′ ( x + y) = e y f ′ ( x ) + e x f( y)
+ a3 x 3 + a4 x 4 + a5 x 5 + a6 x 6 = Σ ( −1) ( 3 + 5m) f( m)
m
m= 0 Put x = 0 and y = x, we get
1
50
f ′ ( 0 + x ) = e x f ′ ( 0 ) + e 0f ( x )
Given, lim  1 +
f( x ) x = Σ ( − 1)m( 3 + 5m) 50C m
= e2 3  m= 0
x→ 0  x  ⇒ f ′ ( x ) = e x ⋅ (2 ) + f( x )
f( x )  50 
∴ lim =0 = 3 Σ ( −1)m 50C m  [Q f ′ ( 0) = 2]
x → 0 x3 m= 0 
⇒ f ′ ( x ) − f ( x ) = 2e x
or a0 = a1 = a2 = a3 = 0  50 
+ 5  Σ ( −1)m. m 50C m  dy
2
+ a5 x + a6 x ) m= 0  Let f( x ) = y, f ′ ( x ) =
∴ lim e( a4 = e 2 ⇒ a4 = 2 dx
x→ 0
 50 
= 3  Σ ( −1)m 50C m  dy
∴ f ( x ) = 2 x 4 + a5 x 5 + a6 x 6 m= 0  ∴ − y = 2e x
dx
 50 
IF = e∫
− dx
+ 5  Σ ( −1)m . 50 . 49
Cm − 1 = e −x
⇒ f ′ ( x ) = 8 x + 5a5 x + 6a6 x
3 4 5
m=1 
Solution of the given differential
⇒ f ′ ( x ) = x 3( 8 + 5a5 x + 6a6 x 2 )  50 
= 3  Σ ( −1)m 50C m  equation is
x = 1 and x = 2 are points of local m= 0  y ⋅ e − x = ∫ 2e x ⋅ e − xdx
maxima and local minima.  50 
− 250  Σ ( −1)m − 1 . 49
Cm − 1 ⇒ ye − x = 2 x + C
∴ f ′ (1) = 0 and f ′ (2 ) = 0 m=1 
∴ 8 + 5a5 + 6a6 = 0 ⇒ y = 2 xe x + ce x
= 3 (1 − 1)50 − 250 (1 − 1)49 = 0
and 4 + 5a5 + 12 a6 = 0 ∴ f( x ) = 2 xe x + ce x
12 2 53. (c) P. Perpendicular distance between
Solving, we get a5 = − , a6 = from AB and CD f( x ) = 2 xe x
5 3
3 [Q f( 0) = 0 ⇒ c = 0]
12 5 2 6 2 R = 3r ⇒ R = r
∴ f( x ) = 2 x −
4
x + x 2
5 3 P. Domain of f( x ) is ( − ∞, ∞ )

E 60º Q. Range of f( x ) is  − , ∞ 
52. (d) We have, D 60º C 2
 30   20 
m
 e 
f ( m) = Σ     r
i = 0  30 − i   m − i  1 1
R R. ∫ f( x )dx = ∫0 2 xe
x
dx
C1 0
m  30  20  2R = [2e x ( x − 1)]10 = 2
= Σ     3r
i = 0  i  m − i C2
60º S. lim f( x ) = lim 2 xe x = 0
[Q n
C r = nC n − r ] x→ 0 x→ 0
60º
= Cm +
20 30
C1 . 20
C m − 1 + ... + 30
Cm
A B
PRACTICE SET - 5
Paper 1
1. (b, c, d) length of prongs of tuning fork increases F = W − πr 2( l + x ) pg
Volume of liquid upto lower level of and thus frequency decreases. = W − πr 2lρg − πr 2ρxg …(ii)
meniscus = πr 2h
From Eqs. (i) and (ii), we get
r
r F = − πr 2 pgx = − kx
R = πr 2ρg
h
m
The time period, T = 2 π
k
m
= 2π
1 T πr 2ρg
Also, f =
2L µ
∴ Frequency of oscillation,
Volume of liquid above meniscus
1 T
= Volume of cylinder of radius r and = . 1 1 πr 2gρ
2L  πD 2ρ  ν= =
height r − Volume of hemisphere of   T 2π m
2 1  4 
radius r = πr 3 − πr 3 = πr 3 5. (c, d)
3 3
=
1
.
T If mg ≥ kx − mg
So, total volume of liquid in capillary LD πρ When spring is taut, the acceleration of
πr = πr 2  h + 
1 3 r
= π r 2h + 3. (a, d) each blocks are same.
3  3
Force on steel = Force on copper If a2 = 0, T = mg
For equilibrium, weight of liquid T + kx − mg kx
σ steel A steel + σ Cu A Cu = 0 ∴ a1 = =
= Upward force of surface tension m m
π 2 π
σ steel ⋅ d steel = − σ Cu ⋅ (d 22 − d 12 )
= πr 2  h +  = 2 πrT cos θ
r If mg < kx − mg , string slacks
4 4
 3 kx − mg
2
and a1 =
−  
d
T cos θ T cos θ 2
m
T σ Cu = 2
⋅ σ steel and a2 = g
d 2 −  
d
θ 6. (b)
2
T sin θ Suppose ground state energy (in eV) be
−  
1
T sin θ  4
E1.
⇒ σ cu = ⋅ σ steel So, E 2n − E1 = 204 eV
1 −  
1
E1
 4 − E1 = 204 eV …(i)
4n 2
1
2T cos θ r σ Cu = − σ steel and E 2n − E n = 40.8 eV
⇒h = − 3
rρg −3
E1  2  = 40.8 eV
3
Also, strain in copper tube …(ii)
r  4n 
If we neglect in comparison to h, then σ
3 = Cu + α Cu ∆T From Eqs. (i) and (ii), we get
YCu
hrρg n=2
T =
2 cos θ 4. (a,d)
From Eq. (ii), we get
Let length l of the cylinder is dipped in −4 2
r  h +  ρg
r
liquid, the weight of liquid displaced by E1 = (2 ) × 40.8 eV
 3 3
or T = the volume of .l-part of cylinder
2 cos θ E1 = −217.6 eV
π r 2l ρ g = W …(i ) E1
For water and glass, θ is very small z2 =
θ ≈ 0° where, ρ = density of the liquid when −13.6
cylinder is furture dipped through a ⇒ z=4
r  h +  ρg
r
 3 distance x, then volume of displaced Emin = E 2n − E 2n−1
∴ T = .
2 [liquid πr 2( l + x )] −7
= E
2. (b, c) Therefore, vertically upward force, (i.e. 144
−7
For observing resonance paper riders force of buoyancy) acting on mass ⇒ = × ( −217.6) eV
144
are used and vibrating tuning fork is equals πr 2( l + x )ρg .
Emin = 10.58 eV
touched to sonometer box (not with At situation of equilibrium, net force on
wire). When temperature increases, the mass
JEE Advanced~Practice Set 5 389

7. (0.00) 8. (4.00) (b) v 2 = u 2 − 2 gh = u 2 − 2 gL

h =  R +  (1 − cos θ)
d In 10 s number of nuclei has been (1 + sin θ) …(ii)
 2 reduced to half (25% to 12.5%). 1
h (c) QD = (Range)
Therefore, its half-life is 2
θ
t 1/ 2 = 10 s  L cos θ − L  = v sin 2 ( 90° − θ)
2
v
θ  
mg
From initial 100% to reduction till 6.25%,  8 2g
it takes four half lives.
v 2 sin 2 θ
t1/ 2 t1/ 2 t1/ 2 = …(iii)
100% → 50% → 25% → 12.5% 2g
t1/ 2 Eq. (iii) can be written as,
→ 6.25%
 cos θ − 1  =  v  sin θ cos θ
2

Velocity of ball at angle θ is ∴ t = 4 t 1/ 2 = 4 (10) = 40 s    


 8   gL 
v 2 = 2 gh t = 40 s
 v2 
= 2  R +  (1 − cos θ) g Substituting value of   = sin θ from
d 9. (1.00) Constant velocity means net
…(i)
 2 acceleration of the system is zero.  gL 
Let N be the normal reaction (away from Or Eq. (i), we get
centre) at angle θ. Then, Net pulling force on the system is zero.  cos θ − 1  = sin2 θ ⋅ cos θ
 
mv 2 While calculating the pulling force,  8
mg cos θ − N =
R + d  tension forces are not taken into
  consideration. Therefore, = (1 − cos 2 θ)cos θ
 2
(a) M1g = M 2 g sin 37 ° + µ M 2 g or cos θ − 1 / 8 = cos θ − cos 3 θ
Substituting value of v 2 from Eq. (i), we
cos 37 ° + µ M 3 g ∴ cos 3 θ = 1 / 8
get
or M1 = M 2 sin 37 ° + µM 2 cos 37 ° cos θ = 1 / 2
mg cos θ − N = 2 mg (1 − cos θ) or
+ µ M3 θ = 60°
∴ N = mg ( 3 cos θ − 2 ) or
Substituting the values, we get From Eq. (i),
(b) The ball will lose contact with the
M1 = ( 4)   + ( 0.25) ( 4)  
inner sphere when, N = 0 3 4
v 2 = gL sin θ = gL sin 60°
 5  5
or 3 cos θ − 2 = 0 or θ = cos −1  
2
3
 3 + (0.25) (4) = 4.2 kg or v2 = gL
2
After this, it makes contact with outer (b) Since, M 3 is moving with uniform
sphere and normal reaction starts acting velocity 11. (4.00) Let N0 be the initial number of
238
towards the centre. Thus, for T = µ M3 g nuclei of U.
n
θ ≤ cos −1   = (0.25) (4) (9.8) = 9.8 N
2
After time t, NU = N0  
1
 3 M3 2
NB = 0 Here, n = number of half-lives
T
and NA = mg ( 3 cos θ − 2 ) t 1.5 × 10 9 1
= = =
θ ≥ cos −1  
2
and for µM3g t 1/ 2 4.5 × 10 9 3
 3
1
NA = 0
NU = N0  
10. (3.00) Let the string slacks at point Q 1 3
and NB = mg (2 − 3 cos θ) as shown in figure. From P to Q path 2
The corresponding graphs are as is circular and beyond Q path is
 1/ 3 
and NPb = N0 − NU = N0 1 −   
follows parabolic. At point C, velocity of 1
NA particle becomes horizontal therefore, 2
QD = half the range of the projectile.  
1/ 3
v  1
C 90° – θ  
mg NU 2
D Q ∴ = 3
= 3.861
θ NPb
1 −  
1
mg 2
θ So, Z ≈4
cos θ
−1 2/3 +1 L L + L sin θ 12. (8.00)
NB L 8 Angular momentum is conserved.
5 mg I1 ω1 = I2 ω 2
u I
P ω 2 = 1 ω1
I2
L cos θ  1
MR 2 
2 mg  
Now, we have following equations = 2  × ω1
cos θ 1
 MR + 2 mr 
2 2
−1
mv 2
2/3 +1 (a) TQ = 0. Therefore, mg sin θ = 2 
L
∴ NA = 0 …(i) = 8.00 rads −1
390 JEE Advanced~Practice Set 5

13. (2.82) So, final p-V variation is In PCl 5, the five sp3d orbitals of
Between C and D block moves with a p phosphorus overlap with the singly
D
constant speed of 120 cms −1. occupied p-orbitals of chlorine atoms to
form five P Cl sigma bonds. Three P
So, time period of oscillation is C
A  Cl bond lie in one plane and make
T2 T
T = t CD + + t DC + 1 an angle of 120° with each other, these
2 2 B
bonds are termed as equatorial bonds.
60 m 60 m The remaining two P—Cl bonds one
= + 2π + + 2π V
120 k2 120 k1 lying above and the other lying below
the equatorial plane, make an angle of
= 2.82 s 16. (a) Here, AD, AB and CD are
90° with the plane. These bonds are
14. (2.95) isothermal and BC and DA are
called axial bonds.
isochoric, then now it is easier to draw
Let area of cross-section of tube is A,
T-p graph for the given situation. Solid PCl 5 exists as [PCl 4 ]+ and [PCl 6 ]− .
10 cm
AB and CD isothermal process, in AB 20. (a,b) NiCl 2 in presence of DMG gives
volume increases and in CD volume a complex [Ni(dmg)2 ].
10 cm 45 cm decreases. BC and DA that constant
Position I OH
volume process. So, p-T graph straight
CH3 C N
45+x line passes through origin.
T D 2 +Ni2+
C
CH3 C N
10 OH
Position II CH3 CH3
A
B O HO
45–x C N N C
p Ni +2Hρ
C N N C
p1V1 = p2V2, gives 17. (d) As, emergent rays are parallel to O
OH
incident ray, hence prism combination
p0( 45)( A ) = p1( 45 − x ) ( A ) CH3 CH3
is producing no deviation.
or 76 × 45 = p1 ( 45 − x ) ∴Deviation produced by prisms are The red coloured complex precipitated
Also, equal and opposite. by Ni 2 + and dmg from slightly
(µ 1 − 1) A1 = − (µ 2 − 1) A 2 ammoniacal solution is a square planar.
p0( 45)( A ) = p2 ( 45 + x ) ( A )
The formation of this complex is used
⇒ A1(µ 1 − 1) = A 2 (1 − µ 2 )
⇒ 76 × 45 = p2 ( 45 + x ) both for the detection and quantitative
18. (d) Given, that emergent beam is estimation of Ni. The dmg loses a proton
So,
white, so net angular dispersion is and forms a stable complex molecule.
 1 1 
p1 − p2 = 76 × 45  −  zero. The complex is stabilised because two
 45 − x 45 + x
(µ V − µ R ) A + (µ ′ V − µ ′ R ) A ′ = 0 five - membered chelate rings are formed
Also, (p1 − p2) A = 10 A′ (µ V − µ R ) and also by internal hydrogen bonding.
⇒ =−
So, equating, we have A (µ ′ V − µ ′ R ) 21. (a,c,d) White phosphorus on reaction
with thionyl chloride (SOCl 2 ) produces
x = 2.95 cm δ ′ (µ ′ − 1) − (µ V − µ R )
⇒ = phosphorus trichloride.
15. (b) Isothermal processes BC and DA δ (µ − 1) (µ ′ V − µ ′ R )
P4 ( s ) + 8 SOCl 2( l ) → 4 PCl 3( l )
are isochoric process. For AB, U δ ω′
constant and ρ increases. Constant U ⇒ =− + 4 SO 2( g ) + 2S 2Cl 2( g )
δ′ ω l
NO is paramagnetic in gaseous
means constant temperature.
⇒ ωδ = − ω ′ δ ′ state because in gaseous state, it
From ideal gas equation, or ω1δ1 = − ω 2 δ 2 has one unpaired electron.
ρRT
p= 19. (a,b,c) P atoms are sp3d hybridised.
l
In P4 , all phosphorus are
M sp3-hybridised and has 75%
The ground state and the excited
∴ If ρ increases, then p increases. p-character
state outer electronic configurations of
m
Q ρ= phosphorus ( Z = 15) are represented P sp3
V below:
If p increases, then V decreases. P P
3s 3p 3d
At constant temperature, so pV = K P(Ground state) P
isothermal process. P(excited state) l
SO 2 cannot be collected over water
For BC , U increases at constant density.
PCl5 because it reacts with water
Constant density, constant volume and
forming H 2SO 3.
if T increases (as U increases) pressure,
also increases. Similar for rest two
Cl Cl Cl Cl Cl SO 2 + H 2O → H 2SO 3
processes.
JEE Advanced~Practice Set 5 391
3
22. (c) On reversing the final product (i.e. O N 2Fe + O 2 → Fe 2O 3;
retrosynthesis ) gives. O O , O O, 2
Ozone Nitrogen ∆Hf° = − 209kcal mol −1 …(ii)
dioxide
On substracting 2 x Eq. (i) from Eq. (ii),
HO
S S we have
O O
(C) (B) H H , O O 1
Hydrogen 2FeO + O 2 → Fe 2O 3,
Sulphur 2
sulphide dioxide
∆Hf° = −209 − (2 × 85) = −39 kcalmol −1
O O 26. (0.49) AgI ( s ) - Ag + ( aq ) + I − ( aq ) ∴Now, let the initial total moles of FeO
(A) Ksp = [Ag ][I ] = 1.2 × 10 −17
+ −
…(i) and Fe 2 O 3 be x.
2x 1 x
23. (a,b,c) Ag + ( aq ) + 2CN − ( aq ) ∴ FeO = = x ; Fe 2 O 3 =
4 2 4
O OH - [Ag(CN)2 ]− ( aq )
Let moles of FeO converted into Fe 2 O 3
(a) (i) C2H5MgBr [Ag(CN)−2 ] be y.
Kf = + − 2
…(ii)
(ii) NH4Cl/H2O [Ag ][CN ] 1
∴ FeO left = x − y
From (i) and (ii), we have 2
x y
+ CH3MgBr [Ag(CN)−2 ][I − ] Total Fe 2O 3 formed = +
(b) Keq = Ksp × K f = 4 2
O [CN − ]2 1
NH4Cl
x−y
H2 O ⇒ 1.2 × 10 −17 × 7.1 × 1019 ∴ 2 =
1
OH Keq = 8.52 × 10 2 x
+
y 3
O 4 2
Now, FeO 1
(c)
CH3MgBr
AgI( s ) + 2CN − - [Ag(CN)2− ] + I − [Q Given, after oxidation = ]
Fe 2O 3 3
OCH3 Initial 1 0 0 10 3
⇒ x= y and y = x
moles 3 10
CH3MgBr
Moles 1 −2 x x x Q Heat released by 2 moles of
NH4Cl
OH
after FeO = − 39 kcal mol −1
O reaction
Let ‘x’ moles of AgI be dissolved in CN − ∴Heat released by 1 mole of
−39 × y −39 × 3 × x
O solution then, FeO = =
(d) AgI + 2CN − Ag(CN)−2 +I −
- 2 2 × 10
+ 2C2H5MgBr
1− 2x x x = − 5.85 x
CH3 NH2
x. x x2 Thus, thermal energy released per mole
Ethanamide
O Keq = 8.52 × 10 2 = = of initial mixture
(1 − 2 x )2 (1 − 2 x )2
(i) THF, D −5.85 x
CH3 C2H5+MgBr2 x = = − 5.85 kcal mol −1
(ii) H3Oρ
C = 29.2 ⇒ x = 0.49 mol x
Butan-2-one 1 −2 x
K1
NH2
27. (150) Cu(H 2O)2+ 29. (196.20) P - Q
+ Mg 6 + H2 O K2
OH [Cu(H 2O)5(OH)2+ ] + H 3O+
- Conc. at t = 0 4 0
24. (b) Mole fraction of liquid A in vapour C 0 0 Conc. at equil. 4− x x
phase, C − x x x dx
⇒ = K1[P ]0 − K 2[Q ]1
X A p° A −5 x2 (10 −5 )2 dt
i.e. YA = …(i) ∴ 10 = = ∴pH = 5]
X A ( p° A − p° B ) + p° B C − x C − 10 −5 [∴K1 = 4 × 10 −3 mol L−1 sec −1 (zero
On reversing Eq. (i) we have ⇒ C = 2 × 10 −5 moL/L order)
 p° − p° B  p° B  1  K 2 = 5 × 10 −3sec −1 ( first order)]
1 ∴Moles of CuSO 4 dissolved
= A  +   dx
YA  p° A  p° A  X A  = 2 × 10 −5 × 0.5 = 10 −5 At equilibrium =0
dt
1 p° B – p° A p° A  1  10 −5 × Na
or, = +   …(ii) ∴Number of unit cells = ∴ O = K1 − K 2[ x ] eq
XA p° B p° B  YA  4
K 4 × 10 −3
6.023 × 1018 ∴ [ x ] eq = 1 =
On comparing Eq. (ii) with ⇒ = 15. × 1018 K2 5 × 10 −3
4
y = mx + c, we get = 0.8 mol L−1 = KC
Number of unit cells along one edge of
p° A
slope = ; cube = 3 15
. × 1018 = 114
. × 10 6 P
- Q
p° B Initial moles 3.2 0.8
p° B − p° A If edge length is a then,
intercept = At eq. 3.2 0.8 + 0.9 = 17
.
p° B . × 10 6 = 17.1 mm = 150 Å
a × 114
At time t ( 3.2 + x )(17
. − x)
1 ∴ (1.7 − x ) = 1.2
25. (4) Among the given options, only 28. (−5.85) Fe + O 2 → FeO;
x = 0.5 Q [Q ] = [P ] eq 
O 3, NO 2, SO 2 and H 2S show bent 2 3
shape. ∆Hf° = − 85 kcal mol −1 …(i)  8 
392 JEE Advanced~Practice Set 5

dx K d2 3
⇒ −W = − ∫ dW =
2 ∫d1
Now, = K1 − K 2[ x ] = 0.8K 2 − K 2 x π d 35. (b)
dt Cl CN CN
d2
dx Kπ  d 4 
= K 2.dt = Br
( 0.8 − x ) 2  4  d
1 KCN Br2/Fe
Taking log on both sides, D
Kπ 4
− 2.303 log( 0.8 − x ) = K 2 t + c = (d 2 − d 14 )
8
at t = 0, x = 0
2π N N N
∴ C = − 2.303 log 0.8 = [(2.5)4 − ( 0.5)4 ] bar m3
8 O O O O O O
2.303 0.8
t = log (A)
= 30.62 bar m (CNσ is a better
3
5 × 10 −3 0.8 − x
nucleophile
2.303 0.8 = 3.06 × 10 J 6
as compared to Clσ)
t = log
5 × 10 −3 0.8 − 0.5 32. (0.36) The distance d hkl between Cl Cl CN
t = 196.20 sec planes with Miller indices hkl is Br Br
O2 given by Br2/Fe KCN
30. (5) 2Fe x (CO)y → Fe 2O 3 + 2 yCO 2 D
a2 Electrophilic
Pressure after oxidation is due to CO 2
2
d hkl = substitution

only because iron carbonyl and iron h + k +l


2 2 2 reaction
N N N
oxide are in the solid state. ∴For d 112 plane O O O O O O
Molar mass of (B)
a 2 = d 112
2
( h2 + k 2 + l 2 ) (Br+ shows
Fe x (CO)y = 56 x + 28 y = 28(2 x + y) electrophilic substitution
a 2 = ( 0.3080)2 (12 + 12 + 2 2 )
Moles of Fe x (CO)y =
0142
. o to Clσ and m to  NO2)
28[2 x + y] ∴ a = 0754
. nm
Moles of CO 2 formed By Bragg’s equation, Aand B are identical molecules.
 44.9  atm(150 nλ = 2 a sinθ 36. (b)
  . L)
1 × 0.542
pV  760  ∴ sinθ = = 0.36 Cl Cl Cl
= =
RT 0.0821 L atm mol −1 K −1 × 298K 2 × 0754
.
Sn/HCl CH3COCl

= 3.622 × 10 −3 mol 33. (c) –HCl

Based on above reaction H+ N NH2 NHCOCH3


Moles of Fe x (CO)y 2 O OH O
= O
CH2/H+
Moles of CO 2 2y CH3 CH

0142
.
(A) Cl CN
28 (2 x + y) 1 (B)
=
3.622 × 10 −3 y 2–
H3O+
Cr2O7 CH3
 2 x + y 0142
.
 = = 14
. CH3
 y  28 × 3.622 × 10 −3 O NH2 NHCOCH3
x x y (C) (2° carbocation is
2 + 1 = 14
. ⇒ = 02
. = =5 more stable)
y y x
C2H5MgBr
OH a b c
31. (3.06) p ∝ d H3O+
37. (a, b, c, d) Given, b c a =0
⇒ p = Kd ⇒ 1 = K.0.5 c a b
p d
⇒ K = 2 bar m−1 ⇒ 2 = 2 (Achiral)
p1 d 1 ⇒ a 3 + b 3 + c 3 − 3abc = 0
p  ‘A’ + HI + C2H5I ⇒ (a + b + c ) (a2 + b 2 + c 2
⇒ d 2 = d1  2  I
 p1  − ab − bc − ca) = 0
= 0.5 × 5 = 2.5 m 1
(D) ⇒ ( a + b + c ) [( a − b )2
Also, − dW = pdV 2
3 34. (c) + ( b − c )2 + (c − a )2 ] = 0
4 d3 π
v = π   = π
4 d
= d3
3  2  3 8 6 + C2H5OK
–+
⇒ ( a − b )2 + ( b − c )2 + (c − a )2 = 0
I
π 2 [Q a + b + c ≠ 0]
⇒ dV = 3d
6 ⇒ a= b=c
π (D)
⇒ d = d2 Hence, OA = OB = OC , where O is the
2 origin and A, B, C are the points
C2H5OH
π K representing z1, z2 and z3 respectively.
⇒ −dW = Kd . d 2 = πd 3 Ph CH CH CH3
2 2 Show cis-trans Therefore, O is the circumcentre of
isomerism ∆ABC.
JEE Advanced~Practice Set 5 393

Now, their exists some α > 1, such that Hence, (a) is TRUE.
1
− log x ≤ 2 2 − 10 for all x ∈ (α, ∞ ) Similarly, ( f(c ))2 = ( g (c ))2
B (z2) A (z1)
θ x
1 ( f(c ) − g (c )) ( f(c ) + g (c )) = 0
as − log x is strictly decreasing Hence, (d) is also TRUE.
O x
function. b and c are FALSE.

1 If f( x ) = g ( x ) = λ ≠ 0, then
∴ g ( x ) ≥ − log x
x λ2 + λ = λ2 + 3λ which is not possible
39. (a, b, d) We have, and λ2 + 3λ = λ2 + λ which is not
C (z3) ( x − a ) cos θ + y sinθ = a possible.
= ( x − a ) cos φ + y sin φ 41. (b, c, d) Now,| AM| = Projection of b on a
z 
Now, arg  3  = ∠BOC = 2 ∠BAC  1 − tan2 θ / 2  a
 z2  ⇒ ( x − a)   = b⋅a = b
 1 + tan θ / 2 
2
| a|
 z − z1  2 tanθ / 2 D C
= 2 arg  3  + y =a
 z2 − z1  1 + tan2 θ / 2
2
 z − z1  ⇒ x tan2 θ /2 − 2 y tanθ /2 + 2 a − x = 0 b
= arg  3 
 z2 − z1  Similarly,
( z + z2 + z3 ) x tan2 φ / 2 − 2 y tan φ /2 + 2 a − x = 0 A
M a
B
Also, centroid is 1 . Since,
3 tanθ / 2 and tan φ /2 are the roots of
HG : GO = 2 : 1, then orthocentre is equations b⋅a 
⇒ AM =  a
z1 + z2 + z3. xz2 − 2 yz + 2 a − x = 0  | a| 
θ φ (b ⋅ a ) a (a ⋅ b )
When triangle is equilateral centroid ∴ tan + tan = 2 y / x = = a
coincide with circumcentre. 2 2 |a| | a| |a|2
2a − x In ∆ADM,
∴ z1 + z2 + z3 = 0 tanθ / 2 tan φ / 2 =
x
Area of equilateral triangle AD + DM = AM
⇒ (tanθ / 2 + tan φ / 2 )2 → → →
3 DM = AM − AD
= × (side )2 φ
4 = (tanθ/ 2 − tan φ/ 2 ) + 4 tanθ /2 tan
2
→ → →
2 → (a ⋅ b ) a →
Since, radius is| z1| 2 ⇒ DM = −b
4(2 a − x )
⇒   = (2 b )2 +
2y →2
|a|
∴ Side = 3 | z1|  x x
→→ → → →
Then, area Q tanθ / 2 − tan φ = 2 b  (a ⋅ b ) a − |a|2 b
=
 3 3 3  2  →
|a|2
=  ( 3 | z1|) =
2
| z1|2
 4  4 ⇒ y = 2 ax − (1 − b ) x
2 2 2
→ → → → →
1
= {(a ⋅ b ) a − |a|2 b}
38. (b, c) We have, y 1 → 2
Also, tanθ / 2 = + b = ( y + bx ) |a|
x x x
f( x ) = log x + ∫0 1 + sint dt
y 1 ⇒
→
MD =
1 → →
{|a|2 b − (a ⋅ b )a }
→ → →
tan φ /2 = − b = ( y − bx )
1 → 2
⇒ f ′( x) = + 1 + sin x x x |a|
x 40. (a, d) Let f( x) and g ( x) assume their → → →
f ′( x ) is not differentiable at sin x = − 1 or a × (a × b ) 1 → → →
maximum at x1 and x2 respectively, Now, = [(a ⋅ b )a
π where x1 < x2 →2 →2
x = 2 nπ − |a| |a| → → →
2 ∴ f( x1 ) = g ( x2 ) = λ − (a ⋅ a ) b ]
log x ∈ (1, ∞ ) f( x ) > 0 f ′ ( x ) > 0 →
Now, let h( x ) = f( x ) − g ( x )
= DM
Consider f( x ) − f ′ ( x ) ∴ h( x1 ) = f( x1 ) − g ( x1 ) = λ − g ( x1 )
⇒ λ − g ( x1 ) > 0
42. (a,d) The equation of tangent is
x 1
= log x + ∫0 1 + sint dt − − 1 + sin x
x and h( x2 ) = f( x2 ) − g ( x2 ) = f( x2 ) − λ ty = x + t 2
⇒ f ( x2 ) − λ < 0 ⇒ x − ty + t 2 = 0
=  ∫ 1 + sint dt − 1 + sin x  ...(i)
x
 0  If x1 > x2, then h( x1 ) < 0 and h( x2 ) > 0
1 Equation of the normal at the point
+ log x − So, by intermediate value theorem
x ( 5 cos θ, 2 sin θ) is
h(c ) = 0
Consider, ( 5 sec θ) x − (2 cosec θ) y = 5 − 4
From ( f(c ))2 + 3f(c ) = ( g (c ))2 + 3g (c )
x ⇒ ( 5 sec θ) x − (2 cosecθ) y = 1 …(ii)
g ( x) = ∫0 1 + sint dt − 1 + sin x ( f(c ) − g (c )) ( f(c ) + g (c ) + 3) = 0
Eqs. (i) and (ii) represent same line
It can be proved that, 5 sec θ − 2 cosec θ
So, their exist ‘c’ such that ⇒ = = − 1/t 2
g ( x ) ≥ 2 2 − 10, ∀ x ∈ ( 0, ∞ ) 1 −t
f(c ) − g (c ) = 0
394 JEE Advanced~Practice Set 5

2 Number of ways of selecting antiparallel 47. (4) Given, x + y + z = a …(i)


⇒ t = cot θ
5 pair = 4
Taking dot product with a
and t = − 1/2 sin θ Number of ways of selecting the third a ⋅ x + a ⋅ y + a ⋅ z = |a|2
vector = 6
2 1 3 7
⇒ cot θ = − sinθ Total = 24 ⇒ + + a ⋅z = 4
5 2 2 4
Number of non-coplanar selection 3
⇒ a ⋅z =
⇒ 4 cos θ = − 5 sin2 θ 4
= 8C 3 − 24 = 32
Again, taking dot product of Eq. (i) with
⇒ 4 cos θ + 5 (1 − cos θ) = 0 2
Hence, λ = 32 x, y and z, we have
⇒ 5 cos 2 θ − 4 cos θ − 5 = 0 45. (2018) Let y = lim x⋅ x + x⋅ y + x⋅ z = x⋅a
n→ ∞ 3
⇒ ( 5 cos θ + 1) (cos θ − 5 ) = 0 ⇒ 1 + x⋅ y + x⋅ z =
 11( k − 9)  2
n +9 n−1
⇒ cos θ = − 1 / 5 (Q cosθ ≠ 5)  Σ 2 n − Σ
58  ⇒ x⋅ y + x⋅ z =
1
…(ii)
 k = 10 log e n/11 k = 0 π ( n − k ) ( n + k )  2
⇒ θ = cos −1 ( −1/ 5 )
 
2
3
⇒ x⋅ y + y ⋅ z = …(iii)
− 1 11( k − 9)
Putting cos θ =   in n +9
2 n
4
 5 ⇒ y = lim Σ 1
n → ∞ k = 10 n ⇒ x⋅ z + y ⋅ z = − …(iv)
log 2 e 4
t = − 1/2 sin θ, we get 11 ∴ a ⋅ x = − 3, a ⋅ y = 7 , a ⋅ z = 3 
−1 1 1 n−1
58  4 
t = 1 − = − 1/ 5 − lim Σ 2 4
2 5 n→ ∞ n k=0 2
π 1 −  
k From Eqs. (ii), (iii) and (iv), we have
Hence, θ = cos −1  1   n 3
−  x ⋅ y = , y ⋅ z = 0,
 5 11k 4
11 n 1
⇒ y = lim Σ 2 n loge 2 ⇒ z⋅ x = −
and t = − 1/ 5. n→ ∞ n k=1 4
1 n−1 Now, ( x × y ) × z = c
43. (75) Let log10 x = a ⇒ x = 10a − lim Σ
58
n→ ∞ n k=0 2 ⇒ (x ⋅ z) y − (y ⋅ z) x = c
log10 y = b ⇒ y = 10 b
π 1 −  
k
1
 n ⇒ − y =c
log10 z = c ⇒ z = 10c 4
11 x 1 58 ⇒ y = − 4 c and y = − λc ⇒ λ = 4
Given, xyz = 10 81 ⇒y= ∫0 2 loge 2 dx − ∫0 dx
π 1 − x2
∴ 10 a ⋅ 10 b ⋅ 10c = 10 81 48. (80) We have, y = x 2 and y = 2 x − 17
58
10 a + b +c
= 10 81 ⇒ y= −
[2 x ]11
0 [sin−1 x ]10 Let any point on the parabola is, t , t 2
π
⇒ a + b + c = 81 and distance from parabola to line is a
58
⇒ y = 2 11 − 2 0 − (sin−1 1 − sin−1 0)
Also, (log10 x )(log10 yz) + (log10 y) π 2t − t 2 − 17
∴ a=
(log10 z) = 468 58  π  1+ 4
⇒ y = 2048 − 1 −  
⇒ log10 x(log10 y + log10 z) π 2
⇒ t − 2t + 17 −
2
5a=0
+ (log10 y)(log10 z) = 468 ⇒ y = 2047 − 29 ⇒ y = 2018
Y
⇒ a ( b + c ) + bc = 468  ( −1)n − 1 x 2n 

y=x2
46. (3) We have, sin−1  Σ 
⇒ ab + bc + ac = 468 n=1 3n − 1 
⇒ ( a + b + c )2 = a 2 + b 2 + c 2  ∞ ( − 1)n − 1 x 4 n  π C
+ cos −1  Σ n−1
 =
+ 2 ( ab + bc + ca ) n=1 3  2 D B
X′ X
⇒ ( 81)2 = a 2 + b 2 + c 2 + 2 ( 468) ∞ x 2n ∞ x4 n
O
n−1 n−1 A
∴ Σ ( −1) = Σ ( − 1)
⇒ a 2 + b 2 + c 2 = 6561 − 936 n=1 3n − 1 n=1 3n − 1 y=2x–17
4 6
⇒ a + b + c = 5625
2 2 2
x x
⇒ x2 − + − ....
3 9
Now, (log10 x )2 + (log10 y)2 + (log10 z)2 Y′
x8 x12
= 5625 = 75 = x4 − + −......
3 9 If C (t 1, t 12 ) and D(t 2, t 22 ), then t 1, t 2 are the
44. (32) Let (1, 1, 1), (− 1, 1, 1), (1, − 1, 1), x 2
x 4 roots of equation and CD = a
⇒ =
→ → → → x2 x4 ∴ t1 + t 2 = 2
(− 1, − 1, 1) be vectors a , b, c , d. Rest 1+ 1+
3 3 t 1 t 2 = 17 − 5 a
→ → → →
of the vectors are − a , b, − c , − d and 3 3 x2 ⇒ a 2 = CD 2 = (t 1 − t 2 )2 + (t 12 − t 22 )2
⇒ = or x = 0
let us find the number of ways of 3+ x 2
3+ x 4
⇒ a 2 = (t 1 + t 2 )2 − 4t 1t 2
selecting coplanar vectors.
⇒ 3 + x = 3 x + x or x = 0
4 2 4
+ (t 1 + t 2 )2 (t 1 − t 2 )2
Observe that out of any three coplanar
vectors, two will be collinear ⇒ x = 1 ⇒ x = ± 1, 0
2
⇒ a = 4 − 4(17 −
2
5a )
(antiparallel) ∴ The number of solution is 3. + 4 ( 4 − 4 (17 − 5a ))
JEE Advanced~Practice Set 5 395

⇒ a 2 = 20 − 20 (17 ) + 20 5 a Multiply by y, z, x to R1, R 2 and R 3 52. (c) Let A denote the event of a family
⇒ a 2 − 20 5 a + 320 = 0 respectively and then take common including at least one boy.
y, z, x from c1, c 2 and c 3 respectively,
⇒ a = 16 5 or a = 4 5 ∞
then Then, P( A ) = 2α Σ pk(2 − p)−( k + 1)
Minimum area when, a = 4 5 k=1
y +13
y 3
y 3
∴ A = a 2 = ( 4 5 )2 = 80 ∞ 
k
z3 z3 + 1 z3 = 11 2α P 
H x y = Σ  
49. (6) Given, r = also, = x3 x3 x3 + 1 2 − p = 1 2 − P
k
3 r H
3x y Apply, C1 → C1 − C 2 and  p 
⇒ = ⇒ y = 3x  
H H C2 → C2 − C3 2α  2 − p 
=
1 0 y3 2 − p 1 − p 
 
r −1 1 z3 = 11  2 − p
0 −1 x + 1 3
αp
=
(2 − p) (1 − p)
⇒ 1 ( x + 1 + z ) + y (1) = 11
3 3 3

x ⇒ x 3 + y 3 + z3 = 10 53. (c) Let f( x) = tan− 1 x


H
So, solution are (1, 1, 2), (1, 2, 1) or Then, for some α ∈ ( x, y)
(2, 1, 1). f( y) − f( x )
y f ′(α ) = (LMVT)
51. (b) If a family of n children contains y− x
exactly k boys, then by binomial
distribution, its probability is 1 tan− 1 y − tan− 1 x
⇒ =
k n− k 1+ α 2
y− x
C k    
n 1 1
dv
= πx 2 2 |tan−1 y − tan−1 x |
dt ⇒ ≤1
d 1 2 
Hence, by total probability law, the | y − x|
⇒  πx y  = πx probability of a family of n children
dt  3 
1
having exactly k boys is given by
d 1 Now, ≤1
3 k n− k
1 + α2
⇒  π 3 x  = πx
α p C k    
n n 1 1
dt  3 
2 2
dx ⇒ |tan− 1 y − tan− 1 x| ≤ | y − x|
⇒ 3 πx 2 = πx ∴ Required probability is
dt k n− k ⇒ |tan− 1 x − tan− 1 y| ≤ | x − y|

= Σ αp C k    
⇒ 3 x dx = dt 1 1 n n
r t n= k 2 2 54. (b) Let f( x ) = sin x and g ( x ) = cos x
⇒ 3∫ x dx = ∫0 dt π
Also, g ′( x ) = − sin x ≠ 0 for x ∈  0, 
0 ∞ k n− k
= α   pk Σ nC k  
1 1
3r 2 3 H2 pn − k  2
⇒ =t⇒ =t 2 n= k 2
2 2 9
C1  
1 + k +1 P Then, by Cauchy’s theorem
H2 36 1 k 
k
2
 f(β ) − f(α ) f ′(θ)
⇒ t = =t = ⇒t =6 
= α  p 
6 6  =
2  P
2
g (β ) − g (α ) g ′(θ)
+ k +2
C 2   .....
50. (3) We have,  2  sin β − sin α cos θ
y3 + 1 y2 z y2 x k − ( k + 1) ⇒ =
cos β − cos α − sin θ
= α   pk  1 − 
1 P
yz2 z3 + 1 z2 x = 11 2  2
= − cot θ
yx 2
x z2
x +13
= 2αpk(2 − p)− ( k + 1)
= 2αpk(2 − p)− k − 1

Paper 2
1. (a, c) For horizontal motion of elevator, spring
For upward moving elevator, kx2 will be inclined at some angle as given
in figure.
kx1 − mg = ma …(i) ma
kx1
θ
ma mg kx3 cos θ

Dividing Eq. (i) by Eq. (ii), we get θ kx3


mg g+ a x 4 2 kx3 sin θ
= 1= m
g −a x2 3 2
For downward moving elevator,
g ma
mg − kx2 = ma …(ii) ∴ a=
7 mg
396 JEE Advanced~Practice Set 5

kx3 cos θ = mg …(iii) To emit photoelectrons from all the three From equation of continuity,
kx3 sin θ = ma …(iv) metals, λmax should corresponds to λmax A
Av1 = av 2 = v 2 ⇒ v 2 = 3v1
Squaring and adding Eqs. (iii) and (iv), for Ni (as, it has highest W 0 ) 3
we get ⇒ λmax (to start ejection from Ni) 9 v12 − v12 = 2 gh ⇒ 8 v12 = 2 gh
( kx3 )2 = m2 ( g 2 + a 2 ) 12400 12400 gh gh
= = Å = 2101.7 Å ⇒ v1 = =
m W 0 (eV) 5.9 4 2
x3 = g 2 + a2 …(v)
k If wavelength of the radiation is less than Acceleration of top layer,
dv 1 1 dh
From Eqs. (i) and (v), we get 2000 Å, then photoelectrons from all the a= 1 = g × ×
dt 2 2 h dt
x3 g 2 + a2 metal surface will be emitted.
= 1 g −1 g gh
x1 g + a 4. (a) As, the peg is moving with = × ( − v1 ) = ×
constant speed in the horizontal 4 h 4 h 2
 g 2 + a2  g
x3 =   direction, then distance covered will = − m /s 2
 g + a  be calculated by the multiplication of 8
 
speed and the duration (t). 7. (6.00)
Hence, x3 = 5 mm From figure, Ltotal ( I + IB ) w
= A
2. (a,b,c,d) OP = 2 r sin θ = vt LB IB . w
At final equilibrium, force must be equal vt
sin θ = mA rA2 11
from two side, so 2r = + 1= + 1= 6
mBrB2 2.2
dθ v
cos θ =
dt 2r 8. (4.20)
dθ v When an excited atom emits a photon of
pf1 A pf2 A =
dt 2 r 1 − sin2 θ momentum p, conservation of
momentum requires that atom recoil
v v
pf1 A = pf2 A ⇒ pf1 = pf2 = = with same momentum p.
2 2
v t 4r − v 2t 2
2
For emitted photon,
In final condition, final temperature will 2r 1 − 2
4r hc
also be same, because separator is ∆E = = pc
conducting. 2 × 10 λ
5. (a) Time of flight, T = s
5 3 h
∴ = p
10 λ
pf A pf A 90° Here, ∆E = E 5 − E1
y 13.6   − 13.6 
V1, T V2, T 5m =  −  − 
/s 2  25   1 
Conservation of moles in I and II x 24
°

5√3 m/s2 = 13.6 × . × 10 −19 J


× 16
30

components gives
g = 10m/s2 30° 25
pV p (V )
= f 1 …(i) Momentum of photon,
RT RT
Range along incline, . × 10 −19
13.6 × 24 × 16
2 p (2 V ) pf V2 p=
= …(ii) 1 25 × 3 × 10 8
R = u xT + ax × T 2
RT RT 2
2 = 6.96 × 10 −27 kg ms −1
V1 =
pV 1  2 × 10 
= 0×T + × 5×  
pf 2  5 3  So, recoil speed of H-atom is
=
40 p 7 × 10 −27
V2 =
4 pV
…(iii) m ≈ = 4.20 ms −1
pf 3 . × 10 −27
m 167
6. (a, b)
V1 + V2 = 3 V 9. (0.95)
Applying Bernoulli’s equation between P
pV 4 pV and Q points,
+ =3V 1  Mg   ∆l 
pf pf p U =     ( πr l ) = 0.95 J
2
2  πr 2   l 
pV 5p v1
× 5 = 3 V ⇒ pf = 10. (75.00)
pf 3
h
Work done at constant pressure is
3V 12 V
So, V1 = , V2 =
∆W =  1 −  × ∆Q total
5 5 Q 1
PE = 0  r
3. (b, d)
25 =  1 −  ∆Q total
v2 1
Ephoton =
12400
=
12400
= 31
. eV ⇒
 r
λ (in Å) 4000 p0 + ρgh +
1 2 1
ρv1 = p0 + 0 + ρv 22
2 2
As W 0 for Zn, Fe and Ni > 3.1 eV, there
2 2 But, r = 1+ = 1+
ρ f 6
will be no photoelectric emission from ρgh = ( v 22 − v12 )
4
any surface. 2 or r=
v 22 − v12 = 2 gh …(i) 3
JEE Advanced~Practice Set 5 397
1 1

So, 25 =  1 −  × ∆Q total
3  1  1  3 q 2  3 18. (c) A—p, B—q r, C—p, D—q r
 4 or a= 2 +  ×  
 4 πε0  2   r  19. (a,b,c)
or ∆Q total = 100 J In the complex [Fe(H 2O)5NO]SO 4 , Fe
∴ N=3
So, net heat absorbed by the gas is in +1 oxidation state because NO is
= 100 − 25 = 75 J 14. (1.10) in +1 oxidation state. Also, NO is a
Intensity corresponding to each strong ligand and the complex has
11. (0.60)
3.6 × 10 −3 3 d 7 configuration at Fe(I) as:
v wavelength =
= 2 or|u | = 2 | u | 3
u
= 1.2 × 10 −3 Wm−2
When u = − x, then v = + 2 x
Energy incident per second in given 3d7
and| x | + | v | = 18
. m
area = 1.2 × 10 −3 × 10 −4
⇒ 3 x = 18. m or x = 0.60 m
= 1.2 × 10 −7 Js −1
12. (5.00)
Let n1 = number of photons incident of
At some point P, relative velocity of cat,
wavelength 4144 Å, then Fe (I) possess three
θ
1.2 × 10 −7 unpaired electrons.
l n1 =
θ  12375  × 1.6 × 10 −19 20. (b) The transition metal is Cu2 + . The
 
 4144  compound is CuSO 4 .5H 2O. It
P
= 2.5 × 1011 dissolves in water to give blue
coloured solution due to presence of
Let n2 = number of photons of Cu2 + . On passing H 2S( g ) in acid
wavelength 4972 Å.
medium of salt solution black
Then, n2 = 3.0 × 1011
precipitate of CuS is obtained which is
approaching to rat is So, total electrons liberated in 2s not soluble in aqueous KOH solution.
dl = 2 ( n1 + n2 ) = 2 × (2.5 × 3.0) × 1011 H+
− = vc − v r cosθ = 5 − 3 cosθ
dt = 11
. × 10 12 CuSO 4 +H 2S → CuS ↓ + H 2SO 4
0 t0 t0 Black
⇒ ∫ − dl = ∫0 5 dt − 3 ∫0 cosθ dθ 15. (c) (i) U =
1
k BT
16 On adding KI solution to aqueous
t0
2
16 = 5 t 0 − 3 ∫0 cos θ dt
⇒ [k B ] =
[U ]
= [ML2T −2 K −1 ]
solution of CuSO 4 yellow solution of CuI 2
is formed in the beginning which
[T ]
If cat catches rat, horizontal distance decompose into white ppt. of CuI.
travelled by both must be same. ⇒A→ s
CuSO 4 + 2KI → CuI 2 +K 2SO 4
(ii) F = ηA  
t0 dv
v rt 0 = ∫0 vc cosθdt
 dx  2CuI 2 → 2CuI ↓ +I 2
White
t0 −2
⇒ 3 t 0 = 5∫ cos θdt F [MLT ] 21. (b) Benzylchloride reacts with KCN in
0 ⇒ [ n] = =
 dv  [L2 ] [T −1 ] presence of DMF to give benzyl cyanide.
t0 3 ( A)  
⇒ ∫ cos θ dt = t 0  dx  Sequence of reactions are as follows:
0 5
t0 = [ML−1T −1 ] Cl CN
Equating ∫ cos θdt , we get
0 ⇒B→ q
3 E
16 = 5 t 0 − 3 × t 0 ⇒ t 0 = 5 s. (iii) E = hf ⇒ h =
5 f KCN
DMF
2 −2
13. (3.00) [ML T ]
F1 F ⇒ [ h] = = [ML2T −1 ]
[T −1 ]
A B 45° NC Ph
⇒ C→ p
dQ kA∆θ [ML2T −3 L ]
F2 (iv) = ⇒ [k ] = –+
H
dt l [L2 K ] C2H5ONa/C2H5OH
−3 −1 C6H5CHO/D
D C 45° = [MLT K ]
⇒D→ r (B)
F1 F1
16. (a) A-p, B-s, C-r, D-q COOH +
Let F1 = electrostatic force H3O,D
17. (b) (A) PE is minimum at mean Ph
and F2 = force of surface tension. position (p, s).
From diagram, (B) For a = 0 ⇒ s = vt : (q)
for equilibrium of line BC, or s = s 0 + vt : (r) SOCl2/CH3NH2

2 F1 cos 45° = F2 For a = constant (C)


1 NH CH3
1 q2  1  s = xt + at 2 option(s) O
⇒ ⋅ 2 2 +  = ra 2
4 πε0 a  2 Ph
v 2 sin2θ
1  1  q2 (C) R = ⇒ option(s)
⇒ a =3
2 +  g
4 πε0  2 r (D) T 2 ∝ l option (q)
398 JEE Advanced~Practice Set 5

22. (a) The open chain structure of or K p =


x
×
18 90  K1C  K1C
27. (5) ×  lim  =
mycarose is 2 + x2 − 3 x 0.082 × 298 100  c→∞ 1 + αC  1 + αC
CHO  
[Q For an ideal gas pV = nRT
and n = 1 mol] 90  K1  K1C
CH2 ⇒ ×  lim  =
⇒ 2 + x 2 − 3 x = 0.37 x 100  c→∞ 1 + α  1 + `αC
H3C OH  C 
HIO3
CH3CHO x 2 − 3.37 x + 2 = 0 90 K1 K1C
H OH ⇒ × =
Acetaldehyde +3.37 ± ( 3.37 )2 − 8 100 α 1 + αC
H OH x= = 077
. , 2.60 0.9 C
2 ⇒ =
CH3 α 1 + αC
∴ x = 077
.
αC
+ HCOOH+CH3COCH2CHO
Thus, ⇒ 0.9 =
Formic 1 + αC
0.082 × 298
acid p( A ) = (2 − 077
. )× ⇒ 01. αC = 0.9
The cyclic structure of mycarose is 18
9 9
= 167
. atm ⇒ αC = 9 ⇒ C = =
CH3 α 9 × 10 5
HO
25. (4) PCl 5 produces POCl 3 with the [∴Given, α = 9 × 10 5]
O following reagents.
OH OH ⇒ C = 10 −5 M
H3C PCl 5 + SO 2 → POCl 3 + SOCl 2
∴ |log C| = − 5
PCl 5 + H 2O → POCl 3 + 2HCl
H 28. (6) There are six chiral centres in
2PCl 5 + H 2SO 4 → SO 2Cl 2 + 2POCl 3
23. (a) 2B - B2 testosterone. These are shown as
+ 2HCl
Initial number of particles 1 0 follows:
α 6PCl 5 + P4O10 → 10 POCl 3
After association 1− α OH
2 26. (258.05) Following reactions occur at
Total number of particles anode and cathode during first phase O
α α of electrolysis.
= 1− α + = 1− O OH
* *
2 2 At anode 2H 2 O → 4H+ + O 2 + 4 e − H
∴ ∆Tb = i × K b × m
α At cathode Cu2+ → Cu + 2e − *
∴ ∆Tb = K b × m ×  1 −  * *
*
 2 Equivalent of O 2 = Equivalent of H H
 n 0.8 × 2
Q Molality ( m) =
n
= Cu = . × 10 −3
= 2520 O
 w(kg) 1  63.5
29. (6.24) The balanced reaction is
When Cu2 + ions are discharged
 ∆Tb 
2 Al(OH)3 + 3HCl → AlCl 3 + 3H 2O
⇒ α = 1 − …(i) completely , i.e. during second phase of
 K b × m  electrolysis, following redox reactions Now, moles of HCl produced = 0.08 × 3
[A ] mα occurs. Moles of Al(OH)3 needed
Now, KC = 22 , KC = 2 = 0.08 × 78 = 624
. g
[ A] m (1 − α )2 × 2 At anode

…(ii) 2H 2 O → 4H + O 2 + 4 e
+
30. (46.60) The gaseous mixture contains
[ Molality = molarity ( dilute solution)] At cathode
80% O 2 and 20% gas.
From Eq. (i) and (ii), we have − − ∴Average molar mass of mixture ( M m )
2H 2 O + 2e → H 2 + 2OH
m ∆Tb  32 × 80 + 20 × M
1− ×2 Now, equivalent of H 2 = equivalent of O 2 Mm = …(i)
2  K b × m  i ×t . × 8 × 60
100
KC = 2 = =
14
Now, for diffusion of gaseous mixture
  ∆Tb   96500 96500
m2 1 −  1 −  × 2 and pure O 2.
  K b × m  Q w = i  rO 2 Mm V t Mm
 E 96500  = or O 2 × m =
K b [K b × m − ∆Tb ] rm MO 2 Vm t O 2 32
⇒ KC =
[2 ∆Tb − K b × m]2 = 6.96 × 10 −3
1 234 Mm
∴Total equivalent of or × =
24. (c) A( g ) + B( g ) - C (g ) 224 1 32
. + 6.96) × 10 −3 = 32.16 × 10 −3
O 2 = (2520
Inital moles 2 1 0 ∴ M m = 34.92
Moles at equil. 2 − x 1 − x x Thus, 32.16 × 10 −3 equivalent of O 2 at Putting the above value in eq (i), molar
Q Kp =
pC NTP will occupy = 18010
. mL mass of gas ( M ) = 46.6 g mol −1
pA × pB [Q 4 Eq. of O 2 at NTP ocucpy = 22.4 L]
31. (7.50)
 x  Similarly, 2KCN + AgNO 3 → [Ag (CN )2 ]−1
 
 3 − x p 6.96 × 10 −3 equivalent of H 2 at NTP will 0.1 0.03 0

 2 − x ⋅  1− x  occupy = 77.95 mL 0.1 – 0.06 0 0.03
    + KNO 3 + K +
 3 − x p  3 − x p [∴2 Eq. of H 2 at NTP occupy = 22.4 L]
x( 3 − x ) ∴Total volume of gases occupied 0 0
=
(2 − x ) (1 − x ) p = 258.05 mL 0.03 0.03
JEE Advanced~Practice Set 5 399

∴ [Ag(CN)2 ]− = 0.03 M 34. (a) 35. (c) I → s; II → s ; III → q ; IV → p



Ag(CN)2 - +
Ag + 2CN P. Urea is prepared by allowing liquid I → Propargyl halogenation
carbon dioxide and liquid ammonia to
0.03 0 0.04 II → BH 3 cannot cause hydroboration
interact, and heating the ammonium
( 0.03 − a ) a 0.04 + a of terminal alkynes
carbamate so formed to 130 - 150° C
Since, KC is too small and dissociation under about 35 atm pressure. III → Kolbe‘s electrolysis
of [Ag(CN)2 ]− is very less. IV → Nucleophilic addition
2NH 3 +CO 2 → NH 2COONH 4
∴0.04 + a ≈ 0.04 and 0.03 − a ≈ 0.03 → CO(NH 2 )2 +H 2O 36. (c) (i) → (s) Temperature coefficient of
[Ag(CN)2 ]− = 0.03, [Ag+ ] = a, When urea react with ethyl acrylate at emf
[CN]− = 0.04
 dEcell  = ET°2 − ET°1
210°C, it lead to the formation of
[Ag+ ][CN − ]2 dihydrouracil. Finally, it gives uracil in  
Now, KC =  dT  T2 − T1
presence of Br2, AcOH and C 5H 5N.
[Ag(CN)2 ]− . − 023
021 .
NH2 EtO2C = = − 1 × 10 −3 VK −1
−19 a × ( 0.04)2 + 308 − 288
⇒ 4 × 10 = OC CH
0.03 NH2 H 2C . mVK −1
= − 10
a = 7.50 × 10 −18 M (ii) → (t)
O
Thus, the value of x is 7.50. ∆G ° = ∆H ° − T∆S °
32. (106) Moles of He 210°C HN Br2 By Gibb’s Helmholtz equation,
d∆G ° 
∆G ° = ∆H ° + T 
pV 560 15
. AcOH
= × = 0.027 mol
RT 760 0.082 × 500 O N  dT  p
H
Moles of He + ions = 0.9 × 0.027 d ( ∆G ° ) 
∆S ° = − 
O O 
= 0.02433 Br Boil in  dT  p
HN
He + in 3rd level
C5H5N
HN dE °cell 
= nF  
= 0.85 × 0.0243 ≡ 20.655 × 10 −3  dT 
O N N
O
He + in 2nd level H H = − 1 × 96500 × 1 × 10 −3
= 0.0243 × 01. = 2.43 × 10 −3
S = − 96.5 J K −1
⇒ E (liberated)
∆ (iii) → (r)
20.655 × 10 −3 × 6.023 × 10 23  Q. NH 4NCS → C
=  170°C ∆G ° = ∆H ° − T∆S °
−18  1
 ×2.18 × 10 × 4 1 − 9 
NH2 NH2
 − nFE °cell = ∆H ° − T∆S °
  Thiourea
S At 288 K , E°cell = 023
. V
2.43 × 10 −3 × 6.023 × 10 23  Et O2C
+  −18  1  − 1 × 96500 × ( 023
. ) = ∆H ° − 288( −96.5)
 ×2.18 × 10 × 4 1 − 4  J
C + CH
  + – ∴ ∆H ° = − 50 kJ
NH2 NH2 NaOH C
= ( 96427.75 + 957187
. )J Sodioformyl-
At 308K, E°Cell = 021
. V
Thiourea
= 105.99 × 10 3 J ≈ 106 kJ acetic ester − 1 × 96500 × 021
. = ∆H ° − 308( −96.5)
O
33. (a) A → s ; B → p ; C → q ; D → q,r ; ∴ ∆H ° = − 50 kJ/moL
(iv) → (p)
E →t HN aq. ClCH2COOH
boil At 298 K,
A. Silver is extracted by amalgamation
process. S N O −1 × 96500 E °cell = − 50,000
H − 298 × ( −96.5)
Ag + Hg → Ag(Hg)
Amalgam HN 50000 − 298 × 96.5
+HSCH2COOH E°cell = = 022
. V
Distillation
→ Ag( s ) + Hg( ν) ↑ 96500
O N
H AgCl( s ) + e − → Ag + Cl − ; E° = 022
. V
B. Calcium is extracted by electrolysis
of fused CaCl 2. R. Br C H COOH Ag 2O
Ag → Ag + + e − ; E° = − 0.80V
 → AgCl( s ) → Ag +
+ Cl ; −
E° = − 0.58V
C. Zinc is extracted by carbon reduction
CH2 COOH −
method. K = Ksp = [Ag ][Cl ] +

ZnO + C → Zn + CO Succinic acid HO  C HCOOH


 0.0591
E° = log Ksp
D. Iron is extracted by both carbon CH2 COOH 1
reduction method and CO reduction 0.58
methods. HO CHCOOH H2SO4
Malic Acid log Ksp = − = − 9.81
0.0591
Fe 2 O 3 + 3C → 2Fe + 3CO CH2COOH
O 37. (a,b,c) Consider
Fe 2O 3 + 3CO → 2Fe + 3CO 2
COOH
E. Copper is extracted by self reduction HN f ′ ′ ′ ( x ) = g ′ ′ ′ ( x ), ∀ x ∈ [0, 3]
NH2CONH2
methods. CO2+H2O+ CH2
On integrating, we get
Cu2S + O 2 → Cu2O +SO 2 O N
CHO f ′ ′ ( x ) = g ′ ′ ( x ) + c1
Cu2O + Cu2S → Cu +SO 2 H
400 JEE Advanced~Practice Set 5

1 41. (b) We have, z = (1 − t ) z1 + t z2


For x = , we have Y
2 (1 − t ) z1 + t z2
f ′ ′   = g ′ ′   + c1 z=
1 1
2 2 (1 − t ) + t
P (ae, b/a2) ∴ z divides the line segment joining z1
⇒ 0 = 0 + c1 ⇒ f ′ ′ ( x ) = g ′ ′ ( x ) …(i)
and z2 in the ratio 1 − t : t internally as
On Integrating Eq. (i), w.r.t. x, we have X′ X
O 0 < t < 1.
⇒ f ′( x ) = g ′( x ) + c 2
∴ z, z1, z2 are collinear.
For x = 1, we have Q ⇒ arg ( z2 − z1 ) = arg ( z − z1 )
f ′ (1) = g ′ (1) + c 2
and | z − z1| + | z − z2| = | z1 − z2|
⇒ 4 = 2 + c2 ⇒ c2 = 2 Y′ z − z1 z − z1
⇒ f ′ ( x) = g ′ ( x) + 2 …(ii) ⇒ =
9 z2 − z1 z2 − z1
On integrating Eq. (ii), we get Put y=
x z − z1 z − z1
f( x ) = g ( x ) + 2 x + c 3 ⇒ =0
∴ x−
9e
= ae 3 z2 − z1 z2 − z1
For x = 3, we have x
But ( z − z1 ) ≠ ( z − z2 )
f ( 3) = g ( 3) + 6 + c 3 ⇒ x 2 − ae 3 x − 9e = 0
⇒ 12 = 3 + 6 + c 3 Hence, (b) is FALSE.
x1 x2 = − 9e
⇒ c 3 = 3 ⇒ f( x ) = g ( x ) + 2 x + 3 42. (a, b, d) We have,
9e 3e
(a) Clearly, f(2 ) = g (2 ) + 4 + 3 If x1 = 6, then x2 = − =− 4a 2 − 5b 2 + 6a + 1 = 0
6 2
⇒ f(2 ) − g (2 ) = 7 9 −6 ⇒ 9a 2 + 6a + 1 = 5( a 2 + b 2 )
y2 = =
(b) | f( x ) − g ( x )| < 5 x2 e 3a + 1
⇒ = 5
⇒ |2 x + 3| < 5 ⇒ Slope of OP × slope of OQ a2 + b 2
⇒ − 5< 2x + 3< 5  y1   y2  9 9 81
⇒ − 8< 2x< 2 ⇒− 4< x< 1     = 2 × 2 = >0 Perpendicular distance from (3, 0) to
 x1   x2  x1 x2 ( x1 x2 )2 line ax + by + 1 = 0 is 5
⇒ x ∈ ( − 4, 1)
40. (a, b) Let ∠A = 30°, ∠B = 45° ∴Centre of circle = ( 3, 0)
(c) | f( x ) − g ( x )| > 5
∴ ∠C = 180° − ( 30° + 45° ) = 105° Radius = 5
⇒ x ∈ ( − ∞, − 4) ∪ (1, ∞ )
Equation of circle = ( x − 3)2 + y 2 = 5
A
38. (c,d) Apply concept of divisibility of which is passes through (1, 1).
given expression by polynomial.
30º 43. (2019) We have,
We know, 1/ n
x 2 + x + 1 = ( x − ω) ( x − ω 2 ) yn = n2 ∫ (2018 sin x
√3+1 b − 1/ n
+ 2019cos x )| x | dx
Since, f( x ) is divisible by x + x + 1
2

= n2  ∫ (2018 sin x )| x| dx
1/ n

∴ f (ω ) = 0 and f (ω ) = 0 2  −1/ n
45º 105º
1/ n 
Here, P(ω 3 ) + ωQ (ω 3 ) = 0 B a C + ∫−1/ n(2019cos x)| x| dx
⇒ P (1) + ω Q (1) = 0 ...(i) AB = 3 + 1 1/ n
= n2 ∫ (2019 cos x )| x| dx
By using sine rule, − 1/ n
P (ω ) + ω Q (ω ) = 0
6 2 6
1/ n
sin A sin B sinC
= = = 2 × 2019n2 ∫ x cos xdx
⇒ P (1) + ω Q (1) = 0
2
...(ii) a b c
0

Q 2018 sin x | x| dx = 0, 
1/ n

 ∫−1/ n it is an odd function 


sin 30° sin 45° sin105°
Solving Eqs. (i) and (ii), we obtain ∴ = =
a b 3+1
P(1) = 0 = Q (1) = 2 × 2019 n2 [ x sin x + cos x ]10/ n
∴ P( x ) and Q ( x ) are both divisible by ( 3 + 1) sin 30° ( 3 + 1) (2 2 )
⇒a= =
= 2 × 2019 n2  sin + cos − 1
1 1 1
x − 1, sin105° 2( 3 + 1)
 n n n 
∴ P ( x 3 ) and Q ( x 3 ) are both divisible by ⇒a= 2
⇒ k = lim yn = lim 2 × 2019 n2
x3 − 1 n→ ∞ n→ ∞
( 3 + 1) sin 45° ( 3 + 1) (2 2 )  1 sin 1 + cos 1 − 1
Here, f( x ) = P ( x ) + x Q ( x ) is divisible
3 3
b= =
sin105° 2 ( 3 + 1)  n n n 
by x − 1
∴Hence, c and d are correct options. ⇒b=2 Put
1
=t
1 n
39. (b, c, d) Area of ∆ABC = bc sin 30°
t sint + cos t − 1
 b2 
2 ∴ k = lim 2 × 2019  
Equation of normal at P  ae,  is 1 1 3+1 t →0  t2 
 a = ×2 × 3 + 1× =
2 2 2  (1 − cos t )
⇒ k = 2 × 2019 lim 1 −
a 
t →0  t2 
⇒ x − ay = a 2e 2 Ratio of greatest side to smaller side
e 3+1
⇒ k = 2 × 2019  1 −  = 2019
c 1
⇒ x − ey = ae 3 i.e. =
a 2  2
JEE Advanced~Practice Set 5 401

44. (11) A plane containing the line of  3 4  3 −4 49. (51) We have,
Now, BC =
2 3  −2 3  dy
x = t − t3⇒ = 1 − 3t 2
intersection of the given planes is  
dt
x − y − z − 4 + λ ( x + y + 2 z − 4) = 0 ( 9 − 8) − 12 + 12   1 0 dy
= and y = t − t4 ⇒ = 1 − 4t 3
( 6 − 6) − 8 + 9   0 1
⇒ ( λ + 1) x + ( λ − 1) y + (2 λ − 1) z     dt
− 4 ( λ + 1) = 0… (i) dy dx
2 1  1 0 2 1 So, x − y
Now, A ⋅ [BC ] = ⋅ = dt dt
Normal vector of above plane is  4 1  0 1  4 1
      = (t − t 3 ) (1 − 4 t 3 ) − (t − t 4 ) (1 − 3 t 2 )
( λ + 1)$i + ( λ − 1)$j + (2 λ − 1)k$ 2 1  1 0 2 1
A ⋅ ( BC )2 = = ⇒ (t − 4t 4 − t 3 + 4t 6 )
 4 1  0 1  4 1
Now, the vector along the line of      − (t − 3t 3 − t 4 + 3 t 6 )
intersection of the planes Now, ⇒ t − 3t6 4
+ 2t 3
2 x + 3y + z − 1 = 0 and  A ( BC )2 
tr ⋅ ( A ) + tr ⋅ 
ABC  ∴ Required area
x + 3y + 2 z − 2 = 0 is  + tr ⋅   + ...
 2   4  1 1
$i $j = ∫ (t 6 − 3 t 4 + 2 t 3 ) dt
k$ 2 −1
→ A A A
n = 2 3 1 = 3 ( $i − $j + k$ ) = trA + tr + tr + tr + ... 1
2 4 8 1  t 7 3t 5 2t 4 
1 3 2 = 7 − 5 + 4 
trA = sum of digits of diagonals 2   −1
→ 3 3 3 1  1 3 2   1 3 2  
As n is parallel to the plane (i), we have ⇒ 3 + + + + .... = − + − − + + 
2 4 8 2   7 5 4   7 5 4  
(( λ + 1)$i + ( λ − 1)$j + (2 λ − 1)k$ )
3 1 2 6 
⇒ S∞ = =6 = − =−
16
. ( 3( $i − $j + k$ )) = 0 1
1− 2  7 5  35
⇒ ( λ + 1) − ( λ − 1) + (2 λ − 1) = 0 2
16
Area = [∴ m + n = 16 + 35 = 51]
⇒ λ=−
1 47. (9) Given, sinθ + cos θ = 1 35
2
AM ≥ GM 50. (1) We have,
Hence, the required plane is sinθ + cos θ 1008
∴ ≥ sinθ cos θ 2018 !
x − 3y − 4z − 4 = 0 2 ∑ − 2 n)!(2 n − 2 )!
n = 1(2018
∴ A = − 3, B = − 4, C = − 4 1
⇒ ≥ sinθ cos θ  + ... 
1 1
∴ | A + B + C| = |− 3 − 4 − 4| = 11 2 +
1 = 2018 ! (2016)! 0 ! 2014 !2 ! 
45. (1) Given, cos y  + e − x   
dy ⇒ sinθ cos θ ≤ 1
 +
 dx  4 2 !2014 ! 
Now, let sinθ = x and cosθ = y
+ sin y  e − x −  = ee  2016 ! + 2016 ! + ...
dy −x

 dx  Let (1 + cosec θ) (1 + sec θ) ≥ a 2018 !  2016 ! 0 ! 2014 !2 ! 


=
1  1 2016 !  2016 ! 

dy
(cos y − sin y) ∴  1 +   1 +  ≥ a  + 
dx  x  y  2 !2014! 
2018 !  C 0 + 2016C 2 + ... 2016C 2014 
2016
+ e − x (cos y + sin y) = ee
−x
( x + 1) ( y + 1)
⇒ ≥a =  
xy 2016 !  + 2016C
2016 − 
2016
Put sin y + cos y = v  C 2016
dy dv ⇒ xy + x + y + 1 ≥ axy
⇒ (cos y − sin y) = 2018 ! 2016−1
⇒ x + y + 1 ≥ ( a − 1) xy = [2 − 1]
dx dx 2016 !
dv −x ⇒ 1 + 1 ≥ ( a − 1) xy
∴ + ve − x = ee [Q nC 0 + nC 2 + nC 4 + ... = 2 n−1 ]
dx [Q x + y = 1]
e − x dx −x 2 = 2018 × 2017[2 2015 − 1]
IF = e ∫ = e −e ⇒ ( a − 1) xy ≤ 2 ⇒ xy ≤
a−1 = 2 1 × 1009 × 2017(2 2015 − 1)
Solution of the given differential
2 1 Q xy ≤ 1  Here, 1009 × 2017 × (2 2015 − 1) is odd
equation is But, =
−x −x −x a−1 4  4  number = a × 2 b
v ⋅ e −e = ∫ ee . e −e dx = x + c When, a = 1009 × 2017(2 2015 − 1)
a=9
− e −x and b = 1
⇒ (sin y + cos y) e = x+c ∴ Minimum value of
π (1 + cosec θ) (1 + sec θ) is 9. 51. (b) P. f( x) = [ x] { x} − | x|
Put x = 0, y = − , we get c = 0
y 48. (342) The number of triangle with Let f( x ) is continuous at x = a, where
−x
∴ (sin y + cos y)e − e = x vertices on sides AB, BC , CD is a is any integer
3
C1 × 4C1 × 5C1 ∴ f ( a + ) = f ( a −1 )
Put x = t and y = 0, we get
−t −t Similarly, for other cases, the total LHL at x = a
e −e = t ⇒ tee =1
number of triangle is lim f( x ) = lim f ( a − h)
2 1  3 4 3
C1 × 4C1 × 5C1 + 3C1 × 4C1 × 6C1 x → a− x→ a− h
46. (6) Here, A =  
, B=
2 3 = lim [a − h] { a − h} − | a − h|
 4 1   h→ 0
+ 3C1 × 5C1 × 6C1 + 4C1 × 5C1 × 6C1
 3 − 4 = lim ( a − 1) (1 − h) − | a − h|
C = h→ 0
and
− 2 = 60 + 72 + 90 + 120 = 342
 3  = a − 1− a = − 1
402 JEE Advanced~Practice Set 5

RHL at x = a Equation of line passing through ⇒ P( n) (2 n + 3) = ( n2 + 3n + 2 ) q ( n)


lim f( x ) = lim f ( a + h)
Q  ,  and perpendicular to
1 11 ∴P( n) = n2 + 3n + 2 and q ( n) = 2 n + 3
x → a+ x→ a + h
3 3
= lim [a + h] { a + h} − | a + h| m
P( n) − 2 m
n 2 + 3n + 2 − 2
h→ 0
y = 2 x + 3 is 2 y + x = 23 / 3 … (iv) P. ∑ = ∑
= lim ah − | a + h| = − a n=1 n n=1 n
h→ 0 Solving Eqs. (i) and (iv), we get
f( x ) is continous at x = a iff a = 1
m
m( m + 1)
4 3 4 S  ,  i.e. focus of parabola.
17 26 = ∑ ( n + 3) =
2
+ 3m
 9 9 n=1
Q. ∫ [ x ]{ x} − | x|)dx =
3 2 3 m( m + 7 )
P. Foot of directrix is =
3 2
4 x 2   17    5  26 
=  2   −
3 3
∫22 ( x − 2 ) − x)dx = 
3 2
− 4 x
2  2
, 2   −
2   2

9 Q. ∑
m
q ( n) − 3
= ∑
m
2n + 3 − 3
n=1 2 n=1 2
=  , 
4  3 10 19
= −  = 2
m
m( m + 1)
3  2
 9 9 = ∑ n=
2
n=1
R. [ x ] { x} = − 1, x ≤ 0 Equation of directrix is m
P( n) + q 2( n) − 11 m
1 1 x+ y=
29
⇒ 9 x + 9 y − 29 = 0 R. ∑ = ∑
x= − 3+ ,−2 + n=1 n n=1
3 2 9
[ x ]{ x} = 2 x – 1, x > 0 ∴a = b = 9 and a + b = 9 + 9 = 18 ( n2 + 3n + 2 ) + (2 n + 3)2 − 1
1 Q. Length of latusrectum = 4 OS n
x=
m( m + 1)
= ∑ ( 5n + 15) = 5  + 3 m
m
2 2 2
 
= 4 
17 3 
−  + 
∴Solution of f( x ) = g ( x ) is 26 5   2
−  n=1
–8 –3 1  9 2  9 2 5m( m + 7 )
, , =
3 2 2 (7 )2 + (7 )2 2
=4 m
q 2( n) − p( n) − 7 m
Hence, there are three solutions.
( 9 × 2 )2 S. ∑ =∑
S. L = lim f( x ) = lim [ x ]{ x} − | x| n=1 n n=1
x → 4+ x → 4+ 7 2 14 2
=4× = (2 n + 3)2 − ( n2 + 3n + 2 ) − 7
= 4( 0) − |4| = − 4 9×2 9 n
⇒ | L| = |− 4| = 4 m( m + 1)
= ∑ ( 3n + 9) = 3  + 3 m
m
∴ a = 14, b = 9
52. (c) Equation of axis of parabola ⇒ a + b = 14 + 9 = 23  2 
n=1
y= x+1 … (i) R. Extremities of latusrectum 3m( m + 7 )
=
Equation of tangent to parabola at vertex   17 7 2  3π   2
±
y+ x=4 … (ii)   9 9 
 cos ,
4   54. (d) Given, f( x) =
lnx
− ax + x 2, a ≥ 0
Vertex of parabola is intersection point   8
  26 ± 7 2  sin 3π    
of axis of parabola and equation of   9   
9   4   f ′( x) =
1
− a + 2x
tangent at vertex.   8x
  17 7  26 7  
Solving Eqs. (i) and (ii), we get =  ±  −   ,  ±  f ′ ( x ) = 1 − 8ax + 16 x 2
  9  9   9 9 
Vertex O  , 
3 5
For maxima or minima, D > 0
2 2
=  ,
10 33   24 19 
 or  ,  ∴ 64a 2 − 64 > 0, a 2 > 1
 9 9  9 9
y=2x+3 a>1
Here,
10 33 24 19 8a ± 64a − 64
2

y=x+1 a1 = , b1 = , a2 = , b2 = x=
9 9 9 9 32
[a1 + b1 + a2 + b2 ] a± a2 − 1
S
10 + 33 + 24 + 19   86  x=
Q (1/3, 11/3) = = =9 4
 9   9 
P. f( x ) gives a local maximum at
O S. Equation of parabola is a− a2 − 1
(3 a > 1, x =
/2 2
,5  y − x − 1 14 2 y + x − 4 4
/2 y+x=4   =
)  2  9 2 Q. f( x ) gives a local maximum at
⇒ 9 ( y − x − 1)2 = 28 ( y + x − 4) a+ a2 − 1
a > 1, x =
Here, a = 9, b = 28 4
Equation of another tangent of parabola R. f( x ) given a point of inflection when,
y = 2x + 3 … (iii) ∴ a + b = 9 + 28 = 37 f ′( x ) and f ′ ′ ( x ) = 0
Solving Eqs. (ii) and (iii), we get 53. (a) Given, ⇒ 32 x − 8a = 0
P( n) f( n + 2 ) = P( n) f( n) + q ( n) 1
Q  , 
1 11 a = 1 and x =
⇒ P( n) [f( n + 2 ) − f( n)] = q ( n) 4
3 3
S. f( x ) is strictly increasing.
Equation of line perpendicular to  1 1 
⇒ P( n)  + = q ( n) ∴ D< 0
2 
tangent lies on tangent at vertex is
 n + 1 n +
passing through focus. a < 1, a∈ [0, 1)
2
PRACTICE SET - 6
Paper 1
1. (a, d) = 15 : 160 : 60 : 96 : 12 As, 95th circular scale division
Let η ∝ mad b v c So, let us write, R A = 15 R, coincides with reference line when
R B = 160 R, etc and draw a simple jaws are fully closed, error is negative
η = kmad b v c zero error.
electrical circuit as shown in figure.
Equating dimensions, we get 1
HB 160 R Error = − (100 − 95) × (mm)
[ ML−1T−1] = [Ma Lb +c T −c ] 100
15 R 60 R 12 R = − 0.05 mm
⇒ a = 1, b = − 2 and c =1
H A HC H E Diameter of wire,
mv 96 R
So, η=k 2
d HD D = MSR + CSR × LC − Error
kmv = 2 mm + 45 × 0.01 mm
Now, d = 2
H = Heat current = Rate of heat flow
η − ( − 0.05 mm)
HA = HE = H (let)
At same temperature mean KE is a = 2 + 0.45 + 0.05 = 2.50 mm
In parallel current distributes in inverse
constant, 5. (a, b, c)
the ratio of resistance.
1
⇒ mv 2 = constant 1 1 1 Natural oscillating frequency of water
2 ∴ HB : HC : HD = : :
R B RC R D molecules is around 2450 MHz and
1
− microwaves of this frequency causes
⇒ ηd 2m 2 = a constant =
1
:
1 1
: vibration, rotation and torsion of water
1/ 2 1/ 4 160 60 96 molecules resulting in conversion of
d CH 4  η He   m CH 4 
So, =     = 9 : 24 : 15 energy of microwave photons
d He  η CH 4   m He   9  3  E = hc = hf 
∴ HB =   H= H,  
1/ 2 1/ 4
 9 + 24 + 15  16  λ 
⇒ d CH 4 = 2.1 × 10 −10 
2.0   16 
  
 11
.   4  24  1 ≈ 6.6 × 10 −34 × 2450 × 106
HC =   H = H,
= 4 × 10 −10 m  9 + 24 + 15  2 ≈ 1 × 10 −5 eV into heat energy.

2. (c, d)  15  5 6. (a,c)
and HD =   H= H
For total internal reflection to take  9 + 24 + 15  16 The intensity of light is
δ
place,
HC = HB + HD I(θ) = I0 cos 2  
angle of incidence, i > critical angle, θc 2
Temperature difference (let us call it T)
2π 2π
or sin i > sinθc
= (Heat current ) × (Thermal resistance) where, δ = ( ∆x ) =   (d sinθ)
or sin 45° > 1 / n λ  λ 
TA = HA R A = ( H ) (15R ) = 15 HR
1
>
1 (a) For θ = 30°,
TB = HBR B =  H  (160 R ) = 30 HR
or 3
2 n  16  c 3 × 10 8
λ= = = 300 m
or n> 2 ν 10 6
TC = HC RC =  H  ( 60 R ) = 30HR
1
or n > 1. 414 2  and d = 150 m
2π  π
δ =   1
Therefore, possible values of n can be
TD = HD R D = 
5 
H  ( 96 R ) = 30 HR  (150)   =
1.5 or 1.6.  300  2 2
 16 
3. (a, c, d) δ π
TE = HE R E = ( H ) (12 R ) = 12 HR ∴ =
l 2 4
Thermal resistance, R = Here, TE is minimum. Therefore,
π
∴ I(θ) = I0 cos 2  
KA
option (c) is also correct.
L 1  4
∴ RA = = ∴ Correct options are (a), (c) and (d).
(2 K ) ( 4Lw ) 8Kw I0
4. (a, c) = [option (a)]
(Here, w = width) 2
1
4L 4 Least count of instrument = mm (b) For θ = 90°,
RB = = 100
2π 
δ = 
3K ( Lw ) 3Kw
= 0.01 mm  (150) (1) = π
4L 1  300 
RC = =
( 4K ) (2 Lw ) 2 Kw δ π
0 or = and I(θ) = 0
4L 4 2 2
RD = = 95
( 5K ) ( Lw ) 5Kw Zero of circular (c) For θ = 0°,
90 scale is above δ
RE =
L
=
1 δ = 0 or =0
main scale zero 2
( 6K ) ( Lw ) 6Kw
∴ I(θ) = I0 [option (c)]
R A : R B : RC : R D : R E
404 JEE Advanced~Practice Set 6

7. (a, d) directions and the total extension x is FBD of M


As, a ⋅ v = |a || v | cosθ given by R = N cosθ + Mg
a ⋅v kx kx µR = N sinθ
⇒ cos θ = m1 m2
|a || v | µ ⋅ ( N cos θ + Mg ) = Nsinθ
 a ⋅v  x1 x2 µMg = N[sinθ − µ cos θ]
⇒ θ = cos −1  
|a || v |  x = x1 + x2 µMg = mg cos θ[sinθ − µ cos θ]
−1 4 and m1 x1 = m2 x2 N
= cos
9 2
d x1
∴ m1 = − k( x1 + x2 )
4 dt 2
or cosθ =
9 a
d 2 x2
and m2 = − k( x1 + x2 )
Now, tangential component of dt 2 mg sin θ mg cos θ
acceleration is
After suitably manipulating the µM
4 4 m=
at = | a | cos θ = 3 × = ms −2 equations, we get cos θ[sinθ − µ cos θ]
9 3
d 2 x2 − k( m1 + m2 )
and normal component of =− ⋅x m = 4 kg
dt 2 m1m2
acceleration is| a | sinθ R
1 k( m1 + m2 )
∴ an = 3  1 −
16  65 i.e. the frequency =
 = ms −2 2π m1m2
 81 3 N sinθ
1 300(2 + 3) N cosθ
8. (4) =
GM 2π 2 ×3 µR
g = … (i) Mg
R2 = 2.5 Hz
G2 m ∴ Number of complete oscillations in 12. (1) N = mg
g′ = … (ii)
4R 2 60 µN = mrω 2
1 min = = 24
g 2.5
g′ = … (iii) µmg = m ⋅ 2 sinθω 2
2 So, number of complete oscillations in
mg 1 24 µg
F = − mg ′ sin ∝, F = − ⋅ sin2θ min = = 6. ω=
2 4 4 2 sinθ
As, θ is much small. 10. (1) From CLM, . × 10
01
ω=
mg M
v0 =
11 v
Mv ⇒ v = 0 2 × sin 30°
ma = − ⋅ 2θ … (i)
2 10 10 11
ω = 1 rad/s
x For block B to leave ground,
a = − g ⋅ θ, a = − g ⋅ N
2R Mg
K ( x0 + x ) = 2 Mg (where, x0 = )
g 2R K
∴ ω= f, T = 2π , mrω2
2R g
µN
2 × 6400 × 10 3 M
T = 2 × 314
.
10 10 A mg
= 2 × 3 × 800 × 141
. M K
T = 6768 s Solutions (For Q.Nos. 13-15)
B
∴ T = 1692 × 4(s ) 2M For an object rolling down an inclined
plane.
Mg N
∴ x= … (ii)
K f
2R θ
mg sinα From COE,
θ
Mgx −  K ( x0 + x )2 − Kx02 
11 1 1
10  2 2  sin
g
α
m mg cos θ
2R θ 1 11
= 0 − ⋅ Mv 2
mg′ 2 10
88M θ
mg cosα On solving, v 0 = g
K We have,
On putting the value, we get N = Mg cosθ
v 0 = 1 ms − 1 F = MaCM = Mg sinθ − f
9. (6) The total momentum of the system
11. (4) FBD of m, Ia
in the horizontal direction is f = CM
conserved. We draw the FBD. N = mg cosθ … (i) R2
Assuming the displacement of the mg sinθ = ma g sinθ
∴We have, aCM =
block to be x1 and x2 in opposite I
a = g sinθ 1+
MR 2
JEE Advanced~Practice Set 6 405

f
Also, µ min = In Ni(dmg) 2 complex, nickel is in +2 B
N oxidation state electronic B
configuration of Ni = 3d 8 4s 2 B B
And, we have B B
MR 2 3d8
Idisc = , Iring = MR 2 2+
Ni = ×× ×× ××××
2 B B
B B
2 B
and Isphere = MR 2 dsp2 hybridised
5 B
13. (c) 14. (a) 15. (b) 22. (c,d) (i) Aq.KOH always give Structure of Baron (B12 )
Solutions (For Q. Nos. 16-18) substitution reaction. 25. (a,c,d) I. For SN 2, 1° RX > 2 ° RX
nf2( nf+ 1) 2 (ii) Alc. KOH does not II. For SN 1, rate ∝ stability of
We have, λmax = dehydrohalogenate vinyl halide
(2 nf + 1)R carbocation, so
formed because OH − is weaker base I
nf2 than NH−2 . Cl >
and λmin =
R (Form 2° (Form 1°
Cl carbocation) carbocation)
Also, Lyman series is in ultraviolet 
region. CH3  C  CH3 +NaNH2 / KNH2 III. For SN 2, rate ∝ stability of
16. (a) 17. (b) 18. (d)  intermediate, so
Cl Cl
19. (c) Using the notation t 2xg e gy and −NH3  Alc. CH2—Br > Br
equation LFSE = (0.4 x − 0.6 y)∆ 0 , we → CH3  C  == CH2 →
Vinyl chloride KOH
can assign the following values.
 NaNH 2 /KNH 2 No reaction IV. For E2, rate ∝ stability of alkane
A. In d 3, x = 3, y = 0 therefore ↓ which is formed in transition state
LFSE = 1.2 ∆ 0 CH 3 C ≡≡ CH 3° > 2 ° RX.
Propylene
B. In high-spin, d 5, x = 3, y = 2 26. (4) Since, the molecule has six pairs, it
therefore LFSE = 0 23. (d) For BCC lattice, should have octahedral geometry. But
C. In d 9, x = 6, y = 3, therefore d 200 = a / 2 the structure is planar, therefore, it
LFSE = 0.6∆ 0 So, for second-order reflection, should have two lp and four bp.
a Therefore, value of X in ML x is 4.
20. (c) 2 λ = 2 × sinθ1
CHO COOH
2 27. (1) Only (ii) contain both centre of
2λ symmetry and plane of symmetry.
H OH H OH or sinθ1 =
a H H
HO H HNO3 HO H 2 × 0.50
H OH H OH
i.e. sin θ1 =
4.287 Cl Cl
H OH H OH and θ1 = 13° 49′
Cl
CH2OH COOH a Cl
Now, d 110 =
(Glucose) (B) Aldaric acid (Glucaric 2 H i
acid) optically active (PoS) (Centre of H (Plane of
a symmetry)
CHO COOH So, 2 λ = 2 × sinθ 2 symmetry)
2
H OH H OH
28. (6) Protein, cellulose, terylene, nylon
2λ 2 × 0.50
HO H HNO3 HO H
(Plane of ⇒ sinθ 2 = sinθ 2 = 6, bakelite, malamine, formaldehyde
HO H HO H symmetry) a 4.287 resin etc.
H OH H OH ⇒ θ 2 = 9° 49′, All condensation polymers are step
a
CH2OH COOH Now, d 111 = , growth polymers.
Galactose (A) (Optically inactive) 2 3
29. (1) Since, the inversion of sucrose
a
21. (a,c) So, 2λ = 2 × sinθ 3 proceeds with constant t 1/ 2.
2 3
N The order w.r.t. sucrose is 1.
OH 2 3λ − d (sucrose)
Ni2+ + 2 ⇒ sinθ 3 =
OH a = k (H + )b (sucrose)1
dt
N 2 × 3 × 0.50
sinθ 3 = K (H + ) = k
b
i.e., Let
σ H 4.287
− d (sucrose)
O O and θ 3 = 23° 83′ ⇒ = k (sucrose)1
ρ dt
N N 24. (a,c,d) I. Boron has two isotopes
0.6932
= k (H + )
10
B and 11B. Hence, k=
b

Ni t 1/ 2
II. Boron has very low abundance in
N N earth crust. 0.6932
= t 1/ 2 (H + ) = constant
b
ρ
III. Kernite [Na 2B 4O 5 (OH) 4 ] is an ore of k
O Oσ
boron. 500 × (10 −5 ) = 50 (10 −4 )b
b
H ⇒
IV. Crystalline boron has B12 unit.
Ni(dmg H)2 ⇒ b=1
Hence, atomicity is equal to 12.
406 JEE Advanced~Practice Set 6

30. (4) 37. (a, b, c, d) We have, z = 5 − y − x where x1 = x − 1, x2 = y − 1,


C F x3 = z − 1, x4 = w − 1 and
⇒ xy + y( 5 − y − x ) + ( 5 − y − x ) x = 3
x1, x2, x3, x4 ≥ 0
⇒ xy + 5 y − y 2 − xy + 5 x − yx − x 2 = 3 Thus, we can say that, the number of
F solutions is equal to the number of
⇒ y 2 + y( x − 5) + x 2 − 5 x + 3 = 0
B Sb F ways in which 15 identical things can
F ⇒ ( x − 5)2 − 4 ( x 2 − 5 x + 3) ≥ 0 be distributed among 4 persons,
⇒ x 2 − 10 x + 25 − 4 x 2 + 20 x − 12 ≥ 0 Also, number of solution = coefficient
A F of x15 in (1 + x + ....... + x15 )4 or
⇒ 3 x − 10 x − 13 ≤ 0
2
3 planes are passed through SbF Coefficient of x19 in
bond dividing other 2 Sb  F bonds ⇒ 3 x − 13 x + 3 x − 13 ≤ 0
2
( x + x 2 + ... + x19 )4
and incorporating 2 Sb  F bonds
⇒ ( 3 x − 13) ( x + 1) ≤ 0 Hence, option (a) and (d) are correct.
(axial)
13 41. (a, c, d) Let the lines represented by
1 plane incorporating all 3 Sb  F ⇒ − 1≤ x ≤
bonds reflecting 2 axial Sb  F bond.
3 the given equation be y = x tanα and
Such that upper Sb  F bond can be 13 y = x tanβ.
Similarly, −1 ≤ y ≤
seen as mirror image of lower Sb  F 3 2 tanθ
Then, tanα + tanβ =
bond. 13 sin2 θ
and −1 ≤ z ≤
3 2 4
31. (a) Grignard reagent on reaction with = =
alcohol leads to the formation of Thus, option (a), (b) and (c) are correct. sinθ cos θ 2 sinθ cos θ
alcohol. 4
Now, required probability = 4cosec 2θ … (i)
CH 3  C  H + C 2H 5MgCl → 13 sin2θ
13/ 3
 ( Grignard reagent ) ∫
= 13/ 3 =
0
dx
3 =
13
and tanα ⋅ tanβ =
tan2 θ + cos 2 θ
O 13
(Aldehyde) ∫−1 dx 3 + 1 16 sin2 θ
C 2 H5 = sec θ + cot 2 θ … (ii)
2
H
 | Hence, option (d) is also correct.
H 3O + Now, consider
CH 3 C H → CH 3  C  C 2 H 5 38. (a,b,c) Since, angle between a and b (tanα − tanβ )2 = (tanα + tanβ) 2
  is acute, therefore − 4 tanα tanβ
O −Mg + Cl OH − 3 x + x 2 + 2 > 0, 4
(Alcohol) = − 4 (sec θ + cot 2 θ)
2

i.e. x ∈ ( − ∞, 1) ∪ (2, ∞ ) . sin2 θ cos 2 θ


32. (b) CH 3CH == CH 2 + H 2 [using Eqs. (i) and (ii)]
Also, as the angle between a and c is
(alkene)
obtuse, there fore  1 − (sin2 θ + cos 4 θ)
RhCl (Ph P) = 4 
 
3 3
→ CH 3CH 2CH 3 3 x 2 + 11x + x 3 − 9 x 2 − 6 < 0  sin2 θ cos 2 θ 
Wilkinson catalyst

This is an example of homogenous i.e., x 3 − 6 x 2 + 11x − 6 < 0  cos 2 θ − cos 4 θ 


=4 
 sin θ cos θ 
2 2
catalytic hydrogenation. i.e., ( x − 1) ( x − 2 ) ( x − 3) < 0
∴The correct combinations is A I V Q. ∴ x ∈ ( − ∞, 1) ∪ (2, 3)  cos 2 θ (1 − cos 2 θ) 
=4 
33. (d) As LDA is a strong base, it is used
 sin θ cos θ 
Hence, option (a), (b) and (c) are 2 2

to remove a less acidic proton. correct.


 cos 2 θ ⋅ sin2 θ 
O O =4 2 
=4
39. (a, c) Let A( at 12, 2 at 1 ) and B( at 22, 2 at 2 ).
 cos θ ⋅ sin θ 
2
H (i-C3H7)2 N–Li+ – 2
Then, we have t 2 = − t 1 − . ⇒ tanα − tanβ = ± 2 …(iii)
H H t1
(LDA)
Let us consider,
(Ketone) For AB to be shortest t 1 = ± 2 tanα − tanβ = 2 … (iv)
⇒ t2 = m 2 2
∴The correct combination is (C) (I) (P). Then, on solving Eqs. (i) and (iv),
⇒ t 1t 2 = − 4 we get
34. (a) Since, hydrogen atom has 1s-orbital
∴ It’s quantum number will be n = 1, ⇒ ∠AOB is right angle 2 tanα = 4 cosec 2θ + 2
l = 0 and the correct radial probability ∴Mid-point of AB is circumcentre. and 2 tanβ = 4 cosec 2θ − 2
curve is R. tanα 4 cosec 2θ + 2
Hence, the circumcentre is ( 5a, 2 a ) Thus, =
35. (b) Since, 3 p has 1 radial node and tanβ 4 cosec 2θ − 2
or ( 5a, − 2 a ).
1 angular node. Its quantum number 2 + sin2θ
40. (a, d) We have, =
are n = 3, l = 1 and it’s graphical 2 − sin2θ
representation is (P). x + y + z + w = 19,
where x, y, z, w ≥ 1
42. (a, b, d) Given, f(2 − x) = f (2 + x)
36. (d) Since, 3d has the highest energy,
This can be rewriten as … (i)
so it’s quantum will be n = 3, l = 2 and
is represented by graph (Q). x1 + x2 + x3 + x4 = 15, and f ( 4 − x ) = f ( 4 + x ) … (ii)
JEE Advanced~Practice Set 6 407

Consider, 44. (1) Given equation of planes are, 46. (14) Let
f ( 4 + x ) = f ( 4 − x ) = f (2 + (2 − x )) x − cy − bz = 0 …(i) g ( x ) = 3 x 4 − 8 x 3 − 6 x 2 + 24 x.
= f (2 − (2 − x )) [using Eq. (i)] cx − y + az = 0 …(ii)
Then, g ′ ( x ) = 12 x 3 − 24 x 2 − 12 x
= f( x ) and bx + ay − z = 0 …(iii)
Now, equation of plane passing = 12 x ( x 2 − 2 x − 1)
Thus, 4 is a period of f( x ).
through the line of intersection of = 12 x ( x 2 − 2 x + 1 − 2 )
Now, consider planes (i) and (ii) may be takes as
50 48 50
= 12 x (( x − 1)2 − 2 )
∫0 f( x ) dx = ∫0 f( x) dx + ∫48 f( x) dx ( x − cy − bz) + λ(cx − y + az) = 0
48 2 i.e., (1 + λc ) x + ( − c − λ ) For x ∈ [1, 2 ), g ( x ) is decreasing,
= ∫ f( x ) dx + ∫ f( x ) dx
0 0
y + ( − b + a λ ) z = 0 …(iv) ∴min of g (t ) in 1 ≤ t ≤ x will be g ( x )
[putting x = 48 + t in second integral] 1
4 2 Clearly, the planes (iii) and (iv) are Now, let h( x ) = 3 x + sin2 πx + 2 , then
= 12 ∫0 f( x) dx + ∫0 f( x) dx same
4
π
h′ ( x ) = 3 + sin(2 πx ) > 0, ∀ x ∈ R
= 12  ∫ f( x ) dx + x ) dx  + 5 1 + cλ (c + λ ) − b + aλ
2 2
 0 ∫0 f ( 4 −  ∴
b
=−
a
=
−1
4
∴ h( x ) is increasing, ∀ x ∈ R
Q 2f( x ) dx = 5, given
 ∫0  By eliminating λ, we get So, maximum of h( x ) in 2 ≤ t ≤ x will
a 2 + b 2 + c 2 + 2 abc = 1 be h( x ).
= 12  ∫ f( x ) dx + x ) dx  + 5
2 2
 0 ∫0 f ( 4 +  π ⇒
2
45. (3) Since, in ∆ABC , ∠C = ,
 3 x 4 − 8 x 3 − 6 x 2 + 24 x, 1 ≤ x < 2
= 24 ∫0 f( x) dx + 5 = 125
therefore
2 
f( x ) =  1
3 x + sin2 πx + 2, 2 ≤ x ≤ 4
50 48 
Also, ∫ f( x ) dx = ∫−4 f( x) dx a2 + b 2 = c 2 … (i) 4
0
From the graph, it is clear, that
[putting x = 4 + t ] Clearly,| x| ≤ 1, also greatest value of f( x ) is 14.
2 2 2 2
and a x b x
52 52 2 + = x2 ≤ 1
∫2 f( x ) dx = ∫0 f( x ) dx − ∫0 f( x ) dx = 13 c2 c2 Y
4 2 [using Eq. (i)]
∫0 f( x) dx − ∫0 f( x) dx = 13(10) − 5 ∴sin−1 x = sin−1  ax  + sin−1  bx  14
y=f(x)
   
= 130 − 5 = 125 c c  13

43. (b, c) Equation of chord joining, ⇒ sin−1 x = sin−1


( a sec θ, b tanθ) and ( a sec φ, b tan φ)  ax b 2 x2 bx a2 x2  6
 1− + 1− 2 
θ − φ  θ + φ
is cos  c c 
x y 2
 − sin    c c
a  2  b  2 
ax b 2 x2 bx a2 x2 X
θ + φ
= cos  ⇒x= 1− + 2 1− 1 2 4
 2
 2  c c c c y=f(x)
ax
Since, it passes through ( ae, 0), ⇒ x= 2 c −b x
2 2 2

therefore we get c bx 47. (8) We have, [| x|] + [| y|] = 1


+ 2 c 2 − a2 x2
θ − φ  θ + φ
e cos   = cos   c This is possible, when [| x|] = 0 and
 2   2  ⇒ x=0 [| y|] = 1 or
θ − φ
cos   or c 2 = a c 2 − b 2 x 2 + b c 2 − a 2 x 2 [| x|] = 1 and [| y|] = 0 [Q [| x|] and
 2  1
⇒ = [| y|] are integers]
 θ + φ e c = a (c − b x ) + b (c − a x )
4 2 2 2 2 2 2 2 2
cos   Case I When [| x|] = 0 and [| y|] = 1
 2  +2 ab c 2 − a 2 x 2 c 2 − a 2 x 2
θ − φ  θ + φ Then, 0 ≤ | x| < 1 and 1 ≤ | y| < 2
cos   − cos   ⇒ c 4 = ( a 2 + b 2 )c 2 − 2 a 2b 2 x 2
 2   2  1− e ⇒ | x | < 1 and 1 ≤ | y| < 2
⇒ =
 θ − φ  θ + φ 1+ e + 2 ab (c 2 − b 2 x 2 )(c 2 − a 2 x 2 )
cos   + cos   ⇒ x ∈ ( − 1, 1) andy ∈ ( − 2, − 1] ∪ [1, 2 )
 2   2  ⇒ c 4 = c 4 − 2 a 2b 2 x 2
[applying dividendo and Y
componendo] +2 ab (c 2 − b 2 x 2 )(c 2 − a 2 x 2 ) x=–1 x=1
y=2
2
θ φ
2 sin sin [using Eq. (i)]
⇒ 2 2 = 1− e 1 y=1
θ φ 1+ e ⇒
2 cos cos
2 2 abx 2 = c 4 − c 2( a 2 + b 2 ) x 2 + a 2b 2 x 4
X′ X
θ φ 1− e –2 –1 O 1 2
⇒ tan tan =
2 2 1+ e ⇒ a 2b 2 x 4 = c 4 − c 4 x 2 + a 2b 2 x 4 –1 y=–1
Since, the chord may also pass ⇒ x = 1 ⇒x = ± 1
2
through ( − ae, 0). –2 y=–2
∴ In a similar manner, we get Thus, the different values of x are 0, 1
Y′
θ φ 1+ e and − 1 .
tan tan =
2 2 1− e
408 JEE Advanced~Practice Set 6

Case II When [| x|] = 1 and [| y|] = 0 n( n + 1)  2 n + 1 + 3  n

Then, 1 ≤ | x | < 2 and 0 ≤ | y| < 1


=
2  3  = Σ [k ((k
k=1
2
+ 3k + 2 ) ( k − 3) + 7 k + 6)]
n( n + 1) ( n + 2 )
= = B (2, n) n
⇒x ∈ ( − 2, − 1] ∪ [1, 2 ) and y ∈ ( − 1, 1)
Thus, we have the following graph.
3 = Σ [k ( k
k=1
3
− 3k 2 + 3k 2 − 9k
Also,
n n + 2 k − 6 + 7 k + 6)]
Y
Σ k(k + 1) (k + 2 ) = Σ k (k 2
+ 3k + 2 ) n n

Σ k(k ) = Σ k
x=–1 x=1
y=2 k=1 k=1
2 = 3 4
= S 4 ( n)
n n n k=1 k=1

1 y=1 = Σk
k=1
3
+ 3 Σk
k=1
2
+2 Σk
k=1
Now, consider
 k + 1  k + 1
X′ X n2( n + 1)2 n( n + 1) (2 n + 1)   S k ( n) +   S k − 1( n) + ....
–2 –1 O 1 2 = + 3.  1   2 
4 6
–1 y=–1
n( n + 1)  k + 1  k + 1
+ 2. +   S1( n) +   S 0( n)
2  k   k + 1
–2 y=–2 n n
n( n + 1) 2 n + 1
Y′
= n ( n + 1) 
 4
+
2
+ 1

 k + 1
= 
 1  r =1
k +
rk + 
 2
Σ 1
Σ
 r = 1r

k−1

Hence, area of required region  n + n + 4n + 2 + 4 


2 n n
= 4 (2 − 1) {1 − ( − 1)} = 8 sq units. = n( n + 1) 
 4


 k + 1
+ .... +  
 k  r =1
r+  Σ
 k + 1

 k + 1 r = 1
r0 Σ
48. (8) Equation of tangent of P( x1, y1 ) of n ( n + 1) ( n + 5n + 6)
2
n
y = f( x )
=
4 = Σ   k +1 1 r
r =1
k  k + 1 k − 1
+ 
 2 
r
dy n ( n + 1) ( n + 2 ) ( n + 3) 
y − y1 = ( x − x1 ) = = B ( 3, n)
dx 4  k + 1  k + 1 
+ ... +  r +  
This tangent cuts the X-axis so Similarly,  k   k + 1 
n n

Σ [(r + 1)
dx
x2 − x1 = − y
dy
B ( 4, n) = ∑ k(k + 1) (k + 2 ) (k + 3)
= k +1
− rk +1
] = ( n + 1)k + 1 − 1
k =1 r =1
n
Q x1, x2, x3, ..., xn are in AP and
common difference is log 2 e
Clearly, S1( n) = 1 + 2 + ... + n = Σk
k=1
Hence, 49. (d); 50. (b) and 51. (d)
Solution (52, 53 and 54)
dx dy dx n( n + 1) Using standard result, we have the following
∴ log 2 e = − y ⇒ =− = = B(1, n)
dy y log 2 e 2 correct combination.
n n n n (I) (iii) (Q); (II) (i) (S); (III) (ii) (P); (IV) (iv) (R)
On integrating both sides, we get
loge y = − x loge 2 + C
S 2( n) = Σk =Σk + Σk− Σk
k=1
2
k=1
2
k=1 k=1
52. (b) Clearly, either option (b) or (c) or (d)
− x loge 2 n n is correct. But a = 2 and b = 3 gives
⇒ y = Ae
= Σ k(k + 1) − Σ k = B(2, n) − B(1, n),
k=1 k=1
area 32 sq units, when area =
16
( ab ) .
⇒ y = f( x ) is passes through ( 0, 2 ) 3
n
⇒ A=2
Hence, equation is
B( 3, n) − 3B (2, n) + B(1, n) = Σ k(k + 1)
k=1
53. (b) Clearly, either option (b) or (d) is
correct.
n n
loge 2− x
Σ k(k + 1) + Σ k
− x loge 2 Consider,
y = 2e ⇒ y = 2e
(k + 2 ) − 3 ( 4a 2/ 3 b1/ 3, 4a1/ 3 b 2/ 3 ) = ( a, a )
−x k=1 k=1
⇒ y = 2 ⋅2 ⇒ y = 21 − x
n ⇒ a 2/ 3b1/ 3 = a1/ 3b 2/ 3
When, x= −2 =
k=1
Σ [k(k + 1)k + 2 ) − 3k (k + 1) + k ] ⇒ a1/ 3 = b1/ 3 ⇒ a = b
then y = 21 + 2 = 2 3 = 8 n

Q k=8 = Σ [k ( k
k=1
2
+ 3k + 2 − 3k − 3 + 1)]
In this case,
( 4a 2/ 3 b1/ 3, 4a1/ 3b 2/ 3 ) = ( 4a, 4a )
Solution (49, 50 and 51) n n 9
Clearly, B(1, n) =
n( n + 1)
,
= Σ [k(k )] = Σ k
k=1
2
k=1
3
= S 3( n)
So, take a = b =
4
, which gives area

2 = 27 sq units
n ( n + 1) ( n + 2 ) and B( 4, n) − 6B ( 3, n) + 7 B (2, n) − B(1, n)
B(2, n) = ; n n
So, option (b) is correct.
Σ k(k + 1) (k + 2 ) (k + 3) − 6 Σ k
3
n ( n + 1) ( n + 2 ) ( n + 3) = 54. (d) Clearly, either option (a), (c) or (d)
B( 3, n) = k=1 k=1
4 is correct.
n n
and
B( 4, n) =
n ( n + 1) ( n + 2 ) ( n + 3) ( n + 4) ( k + 1) ( k + 2 ) + 7 Σ k(k + 1) − Σ k
k=1 k=1
(I) The curves x 2 = 4ay and y 2 = 4bx
intersect X-axis only at (0, 0) only.
n

n
5
n n = Σ [k ((k + 1)(k + 2 )(k + 3) (II) The curve y 2 = 4a ( x + a ) intersect
X-axis at ( − a, 0) and the curve
Σ k(k + 1) = Σ k Σk
k=1
Now, consider 2
+ − 6( k + 1) ( k + 2 ) + 7 ( k + 1) − 1)] y 2 = 4b( x − b ) intersect X-axis at
k=1 k=1 k=1
( b, 0).
n( n + 1) (2 n + 1) n( n + 1) n
=
6
+
2 = Σ [k ([(k + 1) (k + 2 ) {k + 3 − 6}]
k=1
(III) The curves y 2 = 4ax and y = bx
n( n + 1)  2 n + 1  intersect X-axis only at (0, 0).
= + 1 + 7 k + 7 − 1)]
2  3  So, option (d) is correct.
JEE Advanced~Practice Set 6 409

Paper 2
1. (c) From energy conservation, This is because ray of light is travelling V2 L2 2 1 V
∴ = = = or 1 = 4
1 2 1 from right to left. V1 L1 8 4 V2
kx = ( 4k ) y 2
2 2 5. (a) If we take a small strip of dr at Power given to the two coils is same,
y 1 distance r from centre, then number of i.e.
∴ =
x 2 turns in this strip would be i V 1
V1i 1 = V2i 2 or 1 = 2 =
2. (d)  N  i2 V1 4
dN =   dr
Q  b − a 1 2
Energy stored, W = Li
Magnetic field due to this element at 2
2
the centre of the coil will be W 2  L2   i 2 
=     =   ( 4)2
F=12N 1
µ (dN )I ∴
µ 0NI dr W1  L1   i 1   4
15° dB = 0 =
2 r ( b − a) r
W1 1
45° or =
P µ 0NI
ln  
r=b b
∴B = ∫r = a dB =
2( b − a )  a 
W2 4
Resolution of F along given directions 9. (a, c, d)
using Lami’s theorem, we get 1
dy If both E and B are zero, then Fe and
F sin β 6. (a) As, y = 2 x 3/ 2, = 3x2
P= dx Fm both are zero. Hence, velocity may
sin(α + β ) remain constant. Therefore, option (a)
and ds = (dx )2+ (dy)2
12 sin 15° 12 sin ( 45 − 30)° is correct.
= =
sin 60° sin 60° y If E = 0, B ≠ 0 but velocity is parallel or
 1  3 anti-parallel to magnetic field, then
1  1 
12    −   also Fe and Fm both are zero. Hence,
 2  2  2 2 option (b) is also correct.
= ds dy
3 /2 E
If = v , then Fe and Fm are equal.
= 2 (3 2 − 6 ) N dx B
x
F sin α 12 sin 45° 10. (b,c)
Q= = = 4 6N
sin (α + β ) sin 60° Motion of pendulum is part of a
So, ds = (1 + 9 x ) ⋅ dx
3. (c) After critical angle reflection will be 7
circular motion. In circular motion, it is
100% and transmission is 0%. ⇒ s= ∫0 1 + 9 x ⋅ dx better to resolve the forces in two
Options (b) and (c) satisfy this perpendicular directions. First along
2 1022
condition. But option (c) is the correct = [(1 + 9 x )3/ 2 ]70 = m radius (towards centre) and second
27 27
option. Because in option (b), along tangential. Along radius net
transmission is given 100% at θ = 0°, 7. (b) Range up the incline is mv 2
force should be equal to and
which is not true. u2 R
R = sin(2θ − α ) − sinα
4. (b) Object is placed at distance 2 f g cos 2 θ along tangent it should be equal to
MAT , where AT is the tangential
from the lens. So, first image I1 will be
acceleration in the figure.
formed at distance 2 f on other side.
This image I1 will behave like a virtual mv 2
II T − Mg cosθ =
object for mirror. The second image I2 II θ 4 L
θ
will be formed at distance 20 cm in 4
α and Mg sinθ = MaT
front of the mirror or at distance 10 cm or aT = g sinθ
to the left hand side of the lens.
Range is same for both, O
10 cm
∴ sin(2θ − α ) = sin(2θ ′ − α )
I3 θ
π
O I1 ⇒ θ′ = − (θ − α ) T
I2 2 v
6 cm
T sin(θ − α )
So, =
30 cm 10 cm 20 cm T ′ sin(θ ′ − α ) θ
sin(θ − α ) Mg sinθ Mg cosθ
Now, applying lens formula = Mg
cos θ
1 1 1
− = 8. (a,c,d) ∴ Correct options are (b) and (c).
v u f
1 1 1 From Faraday’s law, the induced 11. (a,b,d)
∴ − = or v = 6 cm voltage
v + 10 + 15 Magnetic force does not do work.
V ∝ L, if rate of change of current is From work-energy theorem,
Therefore, the final image is at
constant  V = − L  W Fe = ∆KE
di
distance 16 cm from the mirror. But
 dt 
this image will be real.
410 JEE Advanced~Practice Set 6

1
or (qE ) (2 a ) = m[4v 2 − v 2 ] 15. (c) The mass penetrate plate, the pα = pX + pp ⇒ pα − pp = pX
2 force exerted by mass is opposed by ⇒ pα2 + pp2 − 2 pα pp cos θ = pX2
3  mv 2  plate materials.
or E=    m E − mα E α − mp E p 
4  qa  The work done by all the forces is ⇒ cosθ = −  x x 
equals to change in kinetic energy.  2 mα mp E α E p 
∴Correct option is (a).
By work energy theorem, we can say ≈ 0.4429
At P, rate of work done by electric field that 1
= Fe ⋅ v = (qE ) ( v ) cos 0° As, cos 60° = , so above value of θ
mu 2 mu 2 2
 3 mv 2  Favd = − 0, ⇒ Fav=
3  mv 3  2 2d is just above 60°.
= q ⋅  v =  
 4 qa  4 a 
16. (c) In collision, there is no external In fact, θ = 63° 42 ′ (from tables).
Therefore, option (b) is also correct. force acting on the system, so
Rate of work done at Q of electric field conservation of momentum should be
19. (a)
introduced.
= Fe ⋅ v = (qE ) (2 v ) cos 90° = 0 OH OH
Secondly, net retarding force by the
and of magnetic field is always zero. Br Br
plate on the mass is responsible for Br2/H2O
Therefore, option (d) is also correct.
stopping the mass.
Note that, F = q E $i .
e CH3 CH3
Let say this time distance is d 1, but
12. (b,d) A rotating / revolving frame is plate is free to move that means (A) Br
accelerating and hence non-inertial. penetration stops when they will (B)
Therefore, correct options are (b) aquire same speed. OCH3
OCH3
and (d).
u Br2/H2O
Br Br
13. (a,c) ⇒ v
mv P 2 Km m
r= = =
Bq Bq Bq (C) Br
Conservation of momentum, Isomer of A (D)
mu = ( m + M ) v
20. (b) (i) This is a free radical addition
mu
∴ v= reaction which typically give mainly
m+ M 1, 4-adduct.
From work-energy theorem,
m + BrCCl3
i.e. r ∝ , if K and B are same. mu 2 1
q Favg d 1 = ∆K = − ( M + m) v 2 Br
2 2
CCl3
mu 2 1 m2u 2 A
1 4 16 = − ( M + m)
i.e. rH + : rHe + : rO 2 + = : : 2 2 ( M + m)2 • •
1 1 2 (ii) R  CH == CH2 + C Cl 3 Cl →
Md
= 1: 2 : 2 Solve for d 1, d 1 = R  CH (Cl)  CH2  CCl 3
M + m
Therefore, He + and O 2 + will be
B
(Less steric hinderance)
deflected equally, but H + having the 17. (a) Conservation of energy gives,
or R  CH  CH2
least radius will be deflected most. ( E α + mαc 2 ) + (O + mNc 2 ) | 
14. (a, d) = ( E p + M pc 2 ) + ( E X + M Xc 2 ) CCl 3 Cl
Precision is given by least count of Product C can be obtained by the
and reaction energy is
measuring instrument. combination of two intermediate free
Q = KE f − KE i = ( E p + E X ) − E α radicals.
Accuracy is given by error in
= ( mα + mN − mp − mX ) c 2 • •
measurement. For 20 oscillations, R CHCH2 CCl 3 + R CH(Cl) CH2 →
precision is 0.1 s. − 1.26
⇒ = (4.00388 + 14.00752
931.5 R  CH  CH2  CH  R
So, for 1 oscillation precision is − 100814
. − MX ) 
01
. 
= 0.005 s ⇒ mX ≈ 17 CH2 CCl 3 Cl
20
So, X is 17
9 O. Two radicals combine
Also, average time for 20 oscillations (C )
t + t2 + t3 18. (d) Conservation of momentum gives,
is t avg = 1 p 21. (a,c,d) N 2O 4 - 2NO 2
3
39.6 + 39.9 + 39.5 Initialy a 0
= = 39.6 s
3 Equilibrium a(1 − α ) 2α
∴Average time period α θ Vapour density =
46
: 30.67
t N 1+ α
39.6
= Tavg = avg = = 1995
. s
20 20 46
So, (1 + α ) = = 15
.
∴Maximum observed error 30.67
= 1995
. − 1980
. = 0.015 s x or α = 50%
JEE Advanced~Practice Set 6 411

. × 15
15 . × 0.082 × 300 two SCN ligands and two phenyl 32. (a,b,c) The true statements are:
Total pressure =
82. rings. The molecular structure of (a) All real gases are less compressible
= 675
. atm compound will be [Pd (C 6H 5 )2(SCN)2]. than ideal gas at high pressure.
4α 2 Isomerism in coordination (b) H2 and He are more compressible
So, K p = = 9 atm compound This coordination than ideal gases for all values of
1 − α2
compound will show linkage pressure.
138 (c) Except H2 and He the
Density of mixture = g/L isomerism due to presence of
compressible factor  Z = < 1
82
. ambidentate ligand. pV
= 16.83 g/L  nRT 
27. (c) The compounds which are
22. (c) Maximum temperature is attained optically inactive still they have chiral for all gases at low pressure.
between B and C. carbon are called meso compounds. (d) The compressibility factor for real
p− 4 V −1 gas is dependent on temperature.
= Z = pV/nRT
1− 4 2 − 1
The value of Z first decreases then
Me Me
⇒ p = 7 − 3V increases.
RT
For 1 mole, =7 − 3V 33. (c)
V H H
⇒ RT = 7 V − 3V 2 (position of symmetry)
RdT 7
= 7 − 6 V = 0⇒V = Number of chiral carbons = 2
dV 6
7 7 49 Due to presence of symmetry,
RT = (7 − 3 × ) × ⇒ T = the compound is optically inactive O3, Zn + H2O
6 6 12 R
n
28. (b) Nt = N0  
1
23. (c) The weakest acid is the Fe2 + …(i) O
2
complex on account of its relatively
large ionic radius and low charge. The Let the volume of final solution be
increase of charge to +3 increases V mL, so activity after 4 half-lives 2
the acidic strength. The greater acidity x
= ×V …(ii)
of Al 3 + can be explained by its 20
Hence, option (c) is correct.
smaller radius. The anomalous ion in Initial activity = x × 200 …(iii)
the series is the Hg 2 + complex. This 34. (d) B gives iodoform test, it must
Putting values in Eq.(i), we get contain MeCO-group so, that the
complex reflects the failure of an ionic n
 x. V  = ( x × 200)  1  structure of B may be PhCOMe,
model, because in the complex there    
is a large transfer of positive charge to  20  2 structure of A is
Ph Ph
oxygen as a result of covalent V = 250 mL (final volume)
bonding. C == O + O == C →
So, added volume
W 2.52 1.2 Me Me
24. (a) H 2 S 2 O8 = − = 250 − 200 = 50 mL
194 22.4 22.4 Ph Ph
29. (d)
2 2 4 C == C
CH3
5.04 − 4.8 194 O O Me Me
WH 2 S 2 O8 = × = 1039
. g CH3
22.4 2 O3 CHO Ph Me
+ 2CH3—C—CH
25. (d) When zinc blende (ZnS) is heated Zn+H2O or C == C
in air it produces zinc oxide which on CHO
Me Ph
reaction with dil sulphuric acid CH3 O
CH3 Ph Me
produces zinc sulphate. It is also CHO Syn
O3 O C == C + H 2 →
known as white vitriol and it has 7 2 + Add n
moles of water of crystallisation Zn+H2O CHO Me Ph

ZnS + O 2 → ZnO Racemic mixture
30. (c) Except halogens all deactivating s
35. (c) a. EtO is a strong base and so a
ZnO + dil. H2 SO 4 → ZnSO 4 groups are meta directing. poor leaving group.
ZnSO 4 ⋅7H2O is white vitriol. Halogens are ortho and para directing. 0° C
(b) ArNH2 + NaNO 2 + HCl  at
 →
ZnSO 4 can also be prepared by But they deactivate ring. ⊕ s
reaction of ZnCO 3 with dil. H2 SO 4 .
31. (b, d) The reactions involved during Ar N ≡≡ N  Cl.

ZnCO 3 + H2 SO 4 → ZnSO 4 development of photographic plate are: The benzene diazonium ion (Ar N2 ) is
(dil)
2AgBr + C6H4 (OH) 2 → stabilised by resonance. So, it is a
Uses of ZnSO4 A mixture of ZnSO 4 2Ag + 2HBr + C6H4O 2 r
and BaS is known as lithophone and poor leaving group. But in RN2 it is not
Quinone (developing)
is used as white pigment. stabilised, so it is a good leaving
AgBr + 2Na 2 S 2O 3 →
group.
26. (b) Pd C 14H10 S 2 N2 is a coordination Na 3 [Ag(S 2O 3 ) 2 ] + NaBr
(c)
compound that contain SCN and Soluble (fixing the image) R OH + TsCl → R  O  Ts + HCl.
phenyl ligand means there must be
412 JEE Advanced~Practice Set 6

( a + b + c )  + +  ≥ 9
s 39. (b) Point of intersection of line and 1 1 1
O Ts is a good leaving group because
a b c
plane is  , 0 . Since, the
1 1
it is a conjugate base (weak base) of ,
 2 2 
≥ 4R  − 3 = 6R
a strong acid (TsOH) bc ca ab 9
∴ + +
curve y = Pi( x ) intersect the line on r1 r2 r3  2 
O
XY-plane, Therefore it pass through
42. (c) Total number of points = 3 p
the point  , 0 .
1 1
Me S—OH ,
 2 2  Total number of ∆s (if point are not
O 1 a b collinear) = 3pC 3
⇒ = i + i + ci
(d) RNH2 + NaNO 2 + HCl → 2 2 2 Number of ∆s that cannot be formed
ROH + N2 + NaCl ai [Q if 3 points are selected from a
⇒ b i = 1 and + ci = 0
2 single line = 3 ⋅ ( p C 3 )]
36. (d) Acidity order
HCN > MeOH > H2O > CH4 ⇒ ai is even ∴Maximum number of triangles
Q | Pi′′( x )| ≤ 9, i.e.,| 2 ai | ≤ a = 3pC 3 − 3 ( pC 3 )
−s − s
Basic order CN < MeO < OH <CH3 ∴ ai ∈ {− 4, − 3, − 2, − 1, 0, 1, 2, 3, 4} ( 3 p) ( 3 p − 1) ( 3 p − 2 )
=
Weaker the base, better is the leaving Clearly, acceptable values of ai are 3 ⋅2 ⋅ 1
group. − 4, − 2, 0, 2, 4. p ( p − 1) ( p − 2 )
−3
Fugacity order Now, a1 = − 4 ⇒ c1 = 2 3⋅2 ⋅1
⇒ P1( x ) = − 4 x 2 + x + 2
s s s s
CN > MeO < OH < CH3 3p
= [( 3 p − 1)( 3 p − 2 ) − ( p − 1)( p − 2 )]
a2 = − 2 ⇒ c 2 = 1 6
37. (a) Let n1 and n 2 be the vectors p
⇒ P2( x ) = − 2 x 2 + x + 1 = [( 9 p2 − 9 p + 2 ) − ( p2 − 3 p + 2 )]
normal to the planes determined by 2
$i , $i + $j and $i − $j, $i + k$ , respectively. a3 = 0 ⇒ c 3 = 0 p
= [8 p2 − 6 p] = p2 [4 p − 3]
Then, n = $i × ( $i + $j ) ⇒ P3( x ) = x 2
1
a4 = 2 ⇒ c 4 = − 1 43. (d) For fog ( x) to be exists range of
and n 2 = ( $i − $j ) × ( $i + k$ )
P4 ( x )= 2 x 2 + x − 1 g ≤ domain of f( x ).
⇒ n1 = k$ and n 2 = − $j + k$ − $i Here, domain of f( x ) is given by
a5 = 4 ⇒ c 5 = − 2
Since, a is parallel to the line of ⇒ P5( x )= 4 x 2 + x − 2 3| x| − x − 2, ≥ 0 ⇒ 3 | x| − x ≥ 2
intersection of the planes determined
n
Now, when x ≥ 0, then x ≥ 1
by $i , $i + $j and $i − $j, $i + $j . 1
∑ ∫−1(− 1) Pi( x) dx
i 1
Hence, and when, x < 0, then x ≤ −
i =1 2
∴ a || (n1 × n 2 )
$i $j k$ 1 1
= ∫−1(− P1( x) + P2( x) − P3( x) ∴sin x ≥ 1 and sin x ≤ −
2
⇒ a = λ (n1 × n 2 ) = λ 0 0 1
+ P4 ( x ) − P5( x )) dx [Q fog ( x ) = f( g ( x )) = f(sin x )]
−1 −1 1 1
= ∫− 1 (4 x 2 − x − 2 x 2 + x + 1 − x for f{ g ( x )} to be exists.
= λ ( − 1) [− $i + $j ] i.e. sin x = 1 and − 1 ≤ sin x ≤ −
1
+ 2 x 2 + x − 1 − 4 x 2 − x + 2 ) dx
2
= λ ( $i − $j ) 1
= ∫− 1( − x ) dx = 0
⇒ x = ( 4m + 1) and
π
Let θ be the angle between a and 2
$i − 2 $j + 2k$ . 40. (b) Let α is the real root. Then, we get 7π 11π
α 2 − ( 3 + i )α + m + 2 i = 0 2 nπ + ≤ x ≤ 2 nπ +
Then, 6 6
⇒ (α 2 − 3α + m) + i(2 − α) = 0 = 0 + 0i Thus,domain of fog ( x ) is
λ ( $i − $j ) ⋅ ( $i − 2 $j + 2k$ )
cos θ = ⇒ α =2 ( 4m + 1) π , m ∈ I 
λ2 + λ2 1+ 4 + 4  
Now, let α ′ be the non-real root, then  2 
λ (1 + 2 ) 1 α + α′ = 3 + i ⇒  7π 11π 
= = U 2 nπ + , 2 nπ +
λ 2⋅3 2 α′ = 3 + i − 2 = 1 + i n ∈I 
 6 6 
π bc ca ab 44. (a, b, c) Here, L.H.S. ≥ 4
⇒ θ= 41. (a) Clearly, + +
4 r1 r2 r3 ∴ tan 2 x = 0, cos y = ± 1
38. (a) Here, n(S ) = 64
C3 bc( s − a ) ca( s − b ) ab( s − c ) and sin 3 z = − 1
= + +
Now, let ‘E’ be the event of selecting 3 ∆ ∆ ∆ ⇒ 2 x = nπ; y = nπ
1 3π
squares which form the letter ‘L’. = [s( ab + bc + ca ) − 3abc ] and 3 z = 2 nπ +
Clearly, number of ways of selecting ∆ 2

abc  s( ab + bc + ca ) ⇒ x= ; y = nπ
− 3
squares consisting of 4 unit squares
=
= 7C1 × 7C1 = 49 ∆  abc  2
2 nπ π
 a + b + c  1  and z= + , n ∈ I.
= 4R   +  − 3
Q Each square with 4 unit square forms 1 1
  + 3 2
4 L shapes consisting of 3 unit squares  2  a b c  45. (a, b, c) Consider, ∫ sec 2 x dx
∴n( E ) = 49 × 4 = 196 ...(i)
1 2 sec 2 x (sec 2 x + tan 2 x )
196
Hence, P( E ) = 64 Now, applying AM ≥ GM we can say =
2 ∫ (sec 2 x + tan 2 x )
dx
C3
JEE Advanced~Practice Set 6 413

=
1
log| sec 2 x + tan 2 x | + c (nC 0 + nC1 (1 + x)1 + nC 2 (1 + x )2 + … ⇒ A, D are on one side of the plane
2 +1 and B, C are on the other side,
+ nC r (1 + x )r + nC r + 1 (1 + x )
r

1 1 + sin 2 x therefore the line segments


= log +c + … + nC n (1 + x )n )] AB, AC , BD, CD intersected by the
2 cos 2 x
[Q first r terms have no term of x r plane.
π
1 + cos  − 2 x  therefore adding them does not affect 50. (b, c) We have, Ax 2 − |G| x − H = 0
1 2 
= log +c the coefficient]
2  π  Let α, β be the roots of the given
sin  − 2 x  n
2  (3 + 2 )
n n
Σ [coefficient of x
r =1
r
in
equation. Then,
|G| −H
π α +β= and αβ =
= log cot  − x  + c
1
4  {1 + (1 + x )} n ] A A
2
n As, A>G> H> 0
⇒ f( x ) =
1
2
| log x | ( 3n + 2 n ) Σ [coefficient of x
r =1
r
in ∴α + β = positive fraction and
αβ = negative fraction.
π
g ( x ) = cot  − x  n

Σ
and Also, D = G 2 + 4 AH > 0 ⇒ roots are
4  (2 + x )n ] = ( 3 n + 2 n ) Cr 2n − r
n
r =1 real and unequal.
Clearly, dom f( x ) = ( − ∞, ∞ ) − { 0} n
Now, αβ = negative fraction
Σ
r
and range g ( x ) = ( − ∞, ∞ ) C r  
1
= ( 3n + 2 n ) 2 n n
∴At least one root is a fraction and
Also, f ′ ( x ) =
1
,∀ x∈R+
r =1 2
2x one of the two roots is positive while
π the other is negative.
and g ′ ( x ) = cosec 2  − x   n 
= ( 3 n + 2 n ) 2 n   1 +  − 1
4  1 Since, α + β = positive
  2   ∴ The positive root must be greater in
46. (a, b, c) We have,
= ( 3n + 2 n ) [3n − 2 n ] magnitude.
2 − x , x <2
 2 So, the equation has a negative
f( x ) =  x = ( 3 n )2 − (2 n )2 = 9 n − 4 n
x−2 fraction root.
 2 , x≥2 n n
 x Clearly, Σ Σ (3
r =1 m= r
n m
+ 2 n ) nC m C r is
51. (a) We have,
f( x + y) =
f( x ) + f( y)
, ∀ x, y
 x + 4, x<2 1 + f( x ) ⋅ f( y)
 x3 independent of r.
 Putting x = y = 0, we get
⇒ f ′ ( x ) = undefined, x=2 48. (a,c) Consider the given differential
 4− x 2 f ′ ( 0)
, x>2 equation f ( 0) =
 x3 1 + ( f( 0))2
 2
x   + ( y − 2 x )
dy dy
− 2y = 0
Clearly, f ′ ( x ) = 0 at x = 4. So, x = 4  dx  dx ⇒ f( 0) + ( f( 0))3 = 2 f( 0)
 ⇒ ( f( 0))3 = f( 0)
 dy  − 2 x dy 
is a critical point. 2

Q f ′( x ) is not defined at x = 2, ⇒x  


  dx  dx  ⇒ f( 0)(( f( 0)) − 1) = 0
2

therefore x = 2 is also a critical point.
⇒ f( 0) = 0 or f( 0) = ± 1
+  y − 2 y = 0
Also, x = 0 is a critical point as f( x ) is dy
 dx  But − 1 < f( x ) < 1, ∀ x ∈ R
discontinuous at x = 0. ∴ f( 0) = 0
dy  dy
− 2  + y  − 2  = 0
dy
47. (b,d) Consider the given expression, ⇒ x  f ( x + h) − f ( x )
dx  dx   dx  52. (b) Now, f ′ ( x) = lim
n n h→ 0 h
Σ Σ (3 n
+ 2 n ) nC m C r
n
⇒ 
dy
− 2   x
dy
+ y = 0 f ( x ) + f ( h)
− f( x )
r =1 m= r  dx   dx  1 + f ( x ) ⋅ f ( h)
n = lim
= ( 3n + 2 n ) Σ
r =1

dy
dx
= 2 or x
dy
dx
=− y
h→ 0 h
f( x ) + f( h) − f( x ) − ( f( x ))2 ⋅ f( h)
= lim
n The solution of differential equation h→ 0 h−0
Σ
m= r
n
C m mC r dy
= 2 is y = 2 x + c

1
dx
n 1 + f ( x ) ⋅ f ( h)
= ( 3 n + 2 n ) Σ ( nC r rC r + nC r dy dy dx
+1 and x =− y ⇒ + =0 f( h) (1 − ( f( x ))2 ) 1
r =1 dx y x = lim ⋅
r +1
C r +…+ nC n nC r )
h→ 0 h− 0 1 + f ( x ) ⋅ f ( h)
⇒ log ( xy) = log c ⇒ xy = c
n f( h) − f( 0) 1 − ( f( x ))2
= ( 3n + 2 n ) Σ
r =1
[coefficient of x r in 49. (b, c) For A (1, 1, 1),2 x − y − 3 z − 5 = lim
h→ 0 h−0

1 + f ( x ) f ( h)
= 2 − 1− 3 − 5< 0 = f ′ ( 0) (1 − ( f( x ))2 ) = 1 − f 2( x )
(nC r (1 + x) r + nCr + 1
For B (2, 1,− 3),
(1 + x) r +1
+ K+ nC n (1 + x )n )] [Q f ′ ( 0) = 1]
2 x − y − 3z − 5 = 4 − 1 + 9 − 5 > 0
n ⇒ f ′ ( x ) = 1 − f 2( x ) … (i)
For C (1, − 2, − 2 ),
= ( 3n + 2 n ) Σ
r =1
[coefficient of x r in 2 x − y − 3z − 5 = 2 + 2 + 6 − 5 > 0 ⇒
df( x )
= dx
For D ( − 3, 1, 2 ), 2 x − y − 3 z − 5 1 − f 2( x )
(nC 0 + nC1 (1 + x)1 + nC 2 (1 + x )2 + …
= − 6 − 1− 6 − 5< 0
414 JEE Advanced~Practice Set 6

On integrating, we get − 2 lim


x 1 2
x → ∞ e 2x +1 = ( x − x ),
=e 2
1  1 + f( x )
 1 − f( x )  = x + c
1
ln − 2 lim
d∆ 1 d 2∆
2   =e x → ∞ 2e 2 x
= e0 = 1 = (2 x − 1) and 2 = 1 > 0
dx 2 dx
1 + f( x ) Hence, 51 (a) and 52 (b).
⇒ = ke 2x d∆ 1
1 − f( x ) Putting =0 ⇒x=
53. (a) Since, the points A, B and C lie on dx 2
Now, f( 0) = 0 ⇒ k = 1 the curve y = ax 2 + bx + c, therefore
1
e 2x
−1 1− e −2x
we have c = 5 , a + b + c = 3 ∴∆ is minimum when x =
∴ f( x ) = = 2
e 2x + 1 1 + e −2x and 16a + 4b + c = 9
Thus, the coordinates of D are
Clearly, f( x ) is differentiable for all Solving above equations, we get  1 , 15  .
x ∈ R.  
a = 1, b = − 3, c = 5 2 4 
x
 1 − e −2x  ∴ y = x 2− 3 x + 5 is the curve.
Now, lim [f( x )] = lim  x
 54. (d) We have, y = x 2 − 3 x + 5
x→ ∞ x → ∞  1 + e −2x 
Now, let D = ( x, y) = ( x, x − 3 x + 5).
2
dy
The lowest point exist when =0
[1∞ form] Clearly, D will be nearest from the line dx
 1 − e −2 x
lim 

− 1 x
AB when, area of ∆ABD is minimum. 3
x → ∞  1 + e −2 x 

∴ x=
=e Here, area of ∆ABD 2
− 2e − 2 x
0 5 1
Hence, the point is  ,  .
3 11
lim .x 1
x → ∞ 1 + e −2 x =∆ = 1 3 1 2 4 
=e 2
x x2 − 3 x + 5 1
PRACTICE SET - 7
Paper 1
1. (a, b, c, d) 1 1 ⇒ i 1 = 448 × 10 −5 A
e 4
⇒ TB =  A TA4  =  × ( 5802 )4 
F 1 4

 B 
e  81  ∴ i 1R 3 = 112
. V
R h Current without voltmeter is
⇒ TB = 1934 E
i′ =
And from Wien's law, R1 + R 2 + R 3
F = MaCOM and τ = Iα λ A × TA = λ B × TB
Potential drop without voltmeter is
And Fh = Iα λ A TB
⇒ = 3 × 250
λ B TA i ′ R3 = = 115
. V
⇒ MaCOM h = Iα 250 + 300 + 100
⇒ MαRh = Iα λ B − λ A TA − TB
⇒ = So, error in measurement
I λB TA
⇒ h= . − 112
115 .
MR 1 5802 − 1934 3868 = × 100 = 3%
⇒ = = 115
.
I λB 5802 5802
So at h = , the point in contact 7. (2.00)
MR
⇒ λ B = 1. 5 µm
does not have any tendency to slip Acceleration of A down the plane,
and so friction does not comes into 4. (b,d) aA = g sin 45° − µ A g cos 45°
play. In such case, body rolls over As face ABCD has positive charge on it
= (10) 
1   1 
surface wheather surface is smooth or and the gas consists of ionised  − (0.2)(10)  
 2  2
rough. hydrogen, therefore isotropy is lost. The
usual expression for pressure on the = 4 2 m/s 2
2. (b,d)
basis of kinetic theory will not be valid as Similarly, acceleration of B down the
Force increases linearly therefore,
A ions would also experience forces, other plane,
force acting on the particle at x = than the forces due to collisions with the
2 aB = g sin 45° − µ B g cos 45°
walls of the container.
will be –2F. Potential energy U ∝ x 2,
= (10) 
1   1 
5. (b,d)  − ( 0.3)(10)  
A  2  2
i.e. potential energy at x = will
2 In Davisson-Germer experiment, given
= 3.5 2 m/s 2
become 4U. sample of Ni is first annealed to
remove any oxide layer formed over The front face of A and B will come in
Speed of particle is given by
surface. Due to this sample becomes a line, when
v = ω A 2 − x2 a single Ni crystal. As an electron sA = sB + 2
i.e. v ∝ A 2 − x2 enters metal surface, this process is 1 1
or aA t 2 = aB t 2 + 2
reverse of photoemission, so kinetic 2 2
A 15
at x=− , A 2 − x2 = A energy of electron increases by an 1 1
4 16
amount equals to work function of × 4 2 × t 2 = × 3.5 2 × t 2 + 2
A 3 2 2
and at x = , A 2 − x 2 = A sample.
2 4 Solving this equation, we get
6. (a, c)
t =2 s
4
i,e, A 2 − x 2 has become times.
5 + 8. (9.00)
3V – R2=300Ω
A Given, m1 = 20 kg, m2 = 5 kg,
Therefore, velocity at x = may be i
2 R1=100Ω i –i 1 M = 50kg, µ = 0.3
±
4
v or kinetic energy will become
4 i1 and g = 10 m/s 2.
5 5 A R3=250Ω V RV=5kΩ (a) Free body diagram of mass M is
times or 0. 8 times. given as,
3. (a,b) T
According to Stefan's law, f1 N1
E = eAσT 4 By Kirchhoff’s loop rule, we get
T T
⇒ E A = e A AσTA4 E − iR 2 − i 1R 3 − iR1 = 0 Mg
F
and E B = e B AσTB4 and i i R 3 − ( i − i 1 ) R V = 0
From above equations, we get T
Q EA = EB
∴ e ATA4 = e BTB4 i 1[( R1 + R V ) R 2 − R V R 3
− ( R 3 + R V ) R1 ] N
E=
RV
416 JEE Advanced~Practice Set 7

N dN t  3  3
(b) The maximum value of f1 is or ∫N α − λN
= ∫ 0 dt µmg cos θ =   (2 )(10)  
( f1 )max = (0.3) (20) (10) = 60 N 0
 2   2 
The maximum value of f2 is Solving this equation, we get = 2121
. N = F2 (say)
( f2 )max = (0 . 3) (5) (10) = 15 N 1
N = [α − (α − λ N0 )e − λt ] …(i) (a) Force required to move the block
Forces on m1 and m2 in horizontal λ
down to the plane with constant
direction are as follows (i) Substituting α = 2 λN0 and velocity.
ln(2 )
T T t = t 1/ 2 = in Eq. (i), we get F2
m1 m2 λ F
f1 f2
3
N = N0 v
Now, there are only two possibilities 2
1. Either both m1 and m2 will remain (ii) Substituting, α = 2 λN0 and t → ∞
stationary (w.r.t. ground) in Eq. (i), we get F1
2. Both m1 and m2 will move (w.r.t. α 30°
N = = 2 N0 or N = 2 N0
ground) λ
F1 will be acting downwards, while F2
First case is possible, when N
∴ =2 upwards.
T ≤ ( f1 )max or T ≤ 60 N N0
Since F2 > F1, force required,
and T ≤ ( f2 ) max 10. (0.00) F = F2 − F1 = 11.21 N
or T ≤ 15 N Acceleration of block A, (b) Force required to move the block
These conditions will be satisfied, Maximum friction force that can be up the plane with constant velocity.
when T ≤ 15 N obtained at A is F1 and F2 both will be acting
Say T = 14 N, then f1 = f2 = 14N ( fmax )A = µ A ( mg cos 45° ) downwards.
Therefore, the condition f1 = 2 f2 will 2 2 mg ∴ F = F1 + F2 = 31.21 N
= ( mg / 2 ) =
not be satisfied. Thus, m1 and m2 both 3 3
cannot remain stationary. v F
In the second case, when m1 and m2 2m
both move, mg sin 45° m B
mg A 2 mg sin 45°
f2 = ( f2 ) max = 15 N =
√2 45°
2mg
45° = =F1
∴ f1 = 2 f2 = 30 N = F2 √2 +
F2
F1 30°
Now, since f1 < ( f1 )max , there is no Similarly,
relative motion between m1 and M, i.e. ( fmax )B = µ B(2 mg cos 45° )
all the masses move with same ( Fup − Fdown )
1 So, =2
acceleration, say a = (2 mg / 2 ) 10
3
∴ f2 = 15 N and f1 = 30 N 12. (3.10)
2 mg
Free body diagrams and equations of = m
3 ρ=
motion are as follows π r 2l
Therefore, maximum value of friction
a a ∆ρ
that can be obtained on the system is × 100
m1 m2 ρ
T T ( fmax ) = ( fmax )A + ( fmax )B
∆m 2 ∆r ∆l 
f1 = 30 N f2 = 15 N 2 2 mg =  + +  × 100
= …(i)  m r l 
f1 = 30 N a 3
0.003 2 × 0.005 0.006 
Net, pulling force on the system is =  + +  × 100
M F
2 mg mg mg  0.3 0.5 6 
F = F1 − F2 = − = …(ii)
2 2 2 = ( 0.01 + 0.02 + 0.001) × 100
For m1, 30 − T = 20 a …(i)
From Eqs. (i) and (ii), we can see that = 0.031 × 100 = 310
. %
For m2, T − 15 = 5 a …(ii)
For M, F − 30 = 50 a …(iii) net pulling force < fmax . 13. (5.00)
Therefore, the system will not move or Displacement of boy with respect to
Solving these three equations, we get
ground =  115
. + at 2 
the acceleration of block A will be zero. 1
F = 60 N  2 
3 11. (2.00)
T = 18 N and a =
Also,  115
. + at 2  = ( x cos 60° ) × t
m/s 2 1
mg sin θ = (2 )(10)   = 10 N = F1
1
5  
2 2
T (say)
So, =9 ⇒ a = 5 ms −2
2
9. (2.00) 14. (14.11)
Let at time t, number of radioactive 3 (3m) v ′ = 2 mv
kg µ =√
nuclei are N. 2 2 2
⇒ v′ = v
Net rate of formation of nuclei of A 3
dN dN 2
= α − λN or = dt 30° ∴ v′ = . ms −1
× 30 = 1411
dt α − λN 3
JEE Advanced~Practice Set 7 417
¨ +
15. (c) Suppose, the block B go downwards, 22. (a) (CH 3CH 2 CHCH 3 )MgBr attack to
so A and C will go upwards.
1
2
Ray less substituted carbon of epoxide
As, the length of string is fixed, so ring from the side opposite to  CH 3
Ray X A + XC
= XB group.
2 23. (a, b, c) Reaction (a, b, c) all give
Differentiating the above expression N-ethylcyclopentylamine as major
twice, aA + aC = 2 aB product.
h
t 18. (d) The relation between the C2H5
acceleration of the three blocks will NH
not depend on the masses of the
l blocks, as they are connected H2/Pt
together and length of the string is + NH2
In ray 1, there is no phase shift due to
fixed, so the relation between their
reflection at a soft boundary. C2H5
accelerations are independent of their
In ray 2, there is a phase shift of half masses.
cycle due to reflection from a hard NH
19. (b) In complexes A and B, one halide
boundary. − −
(Cl or Br ) is outside the coordination NH2 + H
H2/Pt
Now, as the thickness t of the air sphere and complexes are :
wedge at each point is proportional to
[Cr(NH 3 )4 Br2 ]Cl and [Cr(NH 3 )4 BrCl]Br
the distance from line of contact, we
have, ‘A’ gives white precipitate of AgCl with
CH3C Cl
t h excess of AgNO 3 which dissolve in NH2 NH C CH3
= Pyridine
excess ammonia. Therefore, A must
x l (i) Li AlH4 ,
be [Cr(NH 3 )4 Br2 ]Cl.
and for destructive interference, diethylether
B gives a pale yellow precipitate with NHC2H5
(ii) H2O
2t = nλ 0, where n = 0, 1, 2......... excess of AgNO 3, which dissolve in
Combining both equations, we get concentrated ammonia solution.
2 hx Therefore, precipitate is AgBr and
24. (a, b, c)
= nλ 0
l complex B is [Cr(NH 3 )4 ClBr]Br. (a) ∆S ( x → y) taking place at constant
lλ In both A and B, hybridisation of temperature.
⇒ x = n. 0 V
2h chromium is d 2sp3 and magnetic ∴∆S = 2.303nR log 2
. × 500 × 10 −9
01 moment (µ = n( n + 2 BM) is 0. V1
= n. p1
2 × 0.02 × 10 −3 20. (c) In the given complex, = 2.303nR log
p2
= n (125
. mm) NiCl 2{P(C 2H 5 )2(C 6H 5 )} 2 nickel is in +2
= 2.303 × 5 × 8.314 log  
oxidation state and the ground state 10
So, successive dark fringes are
electronic configuration of Ni 2 + ion in  1
spaced 1.25 mm apart.
free gaseous state is . JK −1
= 9574
16. (a) 3d8
If we subtitute x = 0, which is the 2+ Thus, (a) is the correct option.
Ni = ××××××××
location of contact line of slides, we get (In paramagnetic) 3
sp hybridised
(b)Qq = 0,
n=0 ∴ ∆S ( x → z) =
q
=0
Hence, at line of contact a dark fringe For the given, four coordinated T
appears. complex to be paramagnetic, it must Thus, (b) is the correct option.
possess unpaired electrons in the
17. (b) Observe carefully that, if blocks A valence shell. To satisfy this condition, (c) x → z takes place adiabatically,
and C moves by distance X A and XC four lone pairs from the four ligands hence temperature falls to 118.6 K.
respectively, then distance moved by occupies the four sp3 hybrid orbitals 5
X + XC ∆S x→ z = 0, C p = R;
B is A . as: 2
2
Ni2+= For monoatomic gas
(In diamagnetic) 3d8 dsp2 ∆S z→ y = 2.303nC p log
T2
T1
The above electronic arrangement
298
gives dsp2 hybridisation and therefore, = 2.303 × 5 × 2.5 × 8.314 log
118.6
have square planar geometry.
. J K −1
= 9576
21. (a, b, c) Li + ion being the smallest in
size and has the highest charge/size Thus, ∆S x→ y = ∆S x→ z + ∆S z→ y
XC
XA
C ratio amongst the alkali metal ions, . J K −1
= 9576
ac
A B aB get much more hydrated (i.e. holds
aA more water molecules in its hydration ∴(c) is the correct option
XB
sphere) than Na + ion and the latter (d) ∆S x→ z → y ≠ 0, thus (d) is the
gets more hydrated than K + ion and incorrect option.
so on.
418 JEE Advanced~Practice Set 7

25. (5) II group radicals Pb 2 + , Cu2 + , Cd 2 + , ⇒ Volume of one unit cell 4 [(100)3 − ( 40)3 ]
2+ 2+ = π
Hg , Sn are precipitated with H 2S . × 01
(162 .) 3
3 25
=
in acidic medium. IV group cations 2.81 × 1019 4
π × ( 40)3
Zn2+ , Mn2 + , Co 2 + , Ni 2 + are = 3
precipitated with H 2S in alkaline = 1513
. × 10 −22 cm3
n(N 2 )
medium. ⇒ Edge-length of unit cell
∴ n(N 2 ) = 17
. mol
26. (36.84) The process can be = (1513
. × 10 −22 )1/ 3
Similarly, applying gas law on II to
described diagramatically as: = 5.32 × 10 −8 cm = 532 pm N 2( g ),
Rev, isothermal
Initial → A Also, in bcc, we have
(3.0 mol,1.0 bar, compression 3 mol, 20 bar, 4 3
300K) 300 K 3 πr
4r = 3a ⇒ r = × 532 4 [(100)3 − ( r )3 ] 3
Irreversible, adiabatic
4 π =
→ B 3 37.5 17
.
expansion 3 mol, 2 bar, T = 230.09 pm 3
⇒ 
VB 100  392
→ C 28. (3.20)  = ⇒ r = 3513
. cm
 r  17
(3 mol, 4 VB,
400) C 6H 5COOAg( s ) - C 6H 5COO −
+ Ag+ , K1 = Ksp 30. (1480.20)
400 1
∆S m = C p ln + R ln …(i) − + Number of atoms present in 1 g
300 pC C 6H 5COO + H - C 6H 5COOH;
1 H = 6.023 × 10 23
Now, between A and B : Applying the K2 =
irreversible and adiabatic conditions. Ka ∴Number of atoms present in 4.8 g H
C 6H5COOAg( s )+H + - C 6H5COOH+ Ag+ = 6.023 × 10 23 × 4.8
θ = 0 = nC V ∆T + pext ∆V ; Also
pext = p2 Ksp = 28.91 × 10 23 atoms
K3 =
T T  Now, number of atoms in III energy
⇒ p2  1 − 2  nR = nC V (T2 − T1 ) Ka
 p1 p2  28.91 × 10 23 × 18
Let x is the solubility of C 6H 5COOAg in level =
 ( p / p )R + C V  100
⇒ T2 =  2 1 a buffer of pH = 319
.
T1 = 520.38 × 10 21 atoms
 Cp  ∴ [H + ] = 10 −3.19 = 6.46 × 10 −4 M
. R + 3.5 R
01 [C 6H 5COOH]Ag+ ]
Number of atoms in II energy level
= × 300 = 240 K3 =
4.5 R Q 28.91 × 10 23 × 10
[H+ ] =
K 100
x⋅ x x2 Ksp
Applying ideal gas equation between = +
= = = 2891
. × 10 21 atoms
[H ] [H + ] Ka
B and C, 
pV   pV 
 =  ∴
Now, total energy evolved when all the
 T  B  T C atoms return to ground state will be
−13
2.5 × 10

2 VB p 4V
= C B ⇒ pC = bar
5 x= × ( 6.46 × 10 −4 ) E 31 = ( E 3 − E1 ) × 520.38 × 10 21
240 400 6 6.46 × 10 −5
× 1602
. × 10 −19
5 . × 10 −6 M
x = 16
Now, substituting pC = in Eq. (i)
6
−13.6
=  + 13.6 × 833.6 × 10 2
gives In aqueous solution, solubility
9 4 6 S = 2.5 × 10 −13 = 5 × 10 −7 M  9 
∆S m = R ln + R ln
2 3 5 x 16. × 10 −6 = 10.078 × 10 5 J
−1 −1 ∴ = = 3.20
= 12.28 J K mol 5 5 × 10 −7 E 21 = ( E 2 − E1 ) × 2891
. × 10 21
⇒ ∆S = n∆S m = 3 × 12.28 × 1602
. × 10 −19
−1
29. (35.13)
= 36.84 J K
=  − + 13.6 × 46314
13.6
. × 10 2
50g H2(g) P 75g H2(g) P′  4 
27. (230.09) +25g
Moles of metal 40cm (H2)g r cm
= 4724
. × 10 5 J
N2 N2
pV 12.5 0.5 Thus,
= = ×
RT 760 0.082 × 1075 P P E total = (10.078 + 4724
. ) × 10 2 kJ
10 m
cm 0
6

0
c

= 9.33 × 10 −5 = 148020
. kJ
I II
⇒ Total number of atoms 31. (2.54) Before electrolysis, the mass of
= 9.33 × 10 −5 × 6.023 × 10 23 The pressure inside and outside the water present is
balloon is same in both the cases as Q Mass of solution = 1000 × 1.261
= 5.62 × 10 atoms19
illustrated in above diagram.
= 1261 g
Q In bcc, there are 2 atoms per unit Applying the gas law on I, i.e. to N 2
Q d = Mass 
cell. and H 2 gases, we have
 V 
⇒ Number of unit cells in given V(H 2 ) V(N 2 )
= . × 1261
382
1
sample = × 5.62 × 1019 nH 2 n(N 2 ) Mass of H 2 SO 4 =
2 100
[Q p and T are constant]
= 2.81 × 1019 = 481702
. g
JEE Advanced~Practice Set 7 419

∴Mass of water = (1261 − 481702


. )g 33. (c) 36. (b) Q is the secondary alcohol that
= 779298
. g can give the alkene shown in previous
Now, let ‘a’ g H 2 O is formed during the question.
CH3 OH CH3
reaction. O + H+ +
CH3—C—–CH—CH3 –H2O
CH3—C— CH—CH3
Q Ratio of number of moles of H 2 O
CH3 CH3
and number of moles of H 2 SO 4 = 1 : 1 2º alcohol (Q) 2º carbocation
Me–
(A) Shift
∴Mass of H 2 SO 4 used C8H4O3 CH3 CH3
98a Al Cl3 –H+ +
= = 5.44a g CH3—C==C—CH3 CH3—C— CH—CH3
18 CH3 CH3
∴Mass of H 2 SO 4 left HOOC
= ( 481702
. − 5.44a ) g …(i) (B) 37. (a,c,d) We have,
tan2{ x}
Now, lim f( x ) = lim
34. (c) x→ 0 +
x→ 0 +
( x 2 − [ x ]2 )
mass of new solution = mass of (H 2 O
2
formed + old solution − H 2 SO 4 used) tan x
= lim =1
∴Mass of new solution x → 0+ x2
= a + 1261 − 5.44a …(ii) [Q x → 0 + , [ x ] = 0 ⇒ { x} = x]
HOOC
Thus, from Eq. (i) and (ii), we have Also, lim f( x ) = lim { x} cot{ x}
PCl5
percentage by mass of new solution x → 0− x → 0−
Cl
481702
. − 5.44a C
= = cot 1
1261 − 4.44a
[Q x → 0 − , [ x ] = − 1
22
= H2, Pd/BaSO4 ⇒ { x} = x + 1{ x} → 1]
100
∴ a = 4577
. g Also,
2
 
∴Moles of H 2 O =
4577
.
= 2.54 mol N N2H4 cot − 1  lim f( x ) = cot − 1 (cot 1) = 1
18  x → 0− 
N
Q Moles of H 2 O = Eq. of H 2O OHC   π
Also, tan− 1  lim f( x ) = tan− 1 1 =
Now, 1 mole of H 2 O formed by (C)  x → 0+  4
passage of 1 F.
35. (d) To deduce the structure of 38. (b, c, d), If P is ( x, y, z), then from
2.54 moles of H 2O formed by = 2.54 F compound P, we need to study all the figure
32. (84.42) A( s ) - X( g ) + H 2 S( g ) reactions. As the final ozonolysis x = r cos φ sinθ
product indicates that the alkene
K A = p × pH 2 S y = r sin φ sinθ
involved in the reaction is 2,3-dimethyl
Q px = pH 2S = 34 mm [Q but-2-ene. z = r cosθ
px = pH 2 S]
Z
H3C CH3
∴ K A = ( 34 × 34) = 1156 mm2 C==C
O3 2 C
H3C CH3 Zn–H2O H 3C CH3
Similarily,
Also, P(C 6H10 ) has two degree of P(x, y, z)
B( s ) - Y ( g ) + H 2 S( g )
K B = py × pH 2 S unsaturation and θ
oxymercuration-demercuration Y
Q py + pH 2 S = 50 mm
hydration indicates that it is an alkyne.
∴ py = pH 2 S = 25 mm [Q py = pH 2 S] φ
As alkyne, on hydration, gives a
⇒ K B = 625 mm2 carbonyl compound which on reduction
with NaBH 4 gives a 2° alcohol. X
In the mixture let, px = x, py = y, then M
total pH 2 S = ( x + y) O
|| 1 = rcos φ sinθ
∴K A = px ⋅ pH 2S = x( x + y) = 1156 C ≡≡ C +H 2O → C CH 2 
2 = rsin φ sinθ
…(i) OH
| 3 = rcosθ
K B = py ⋅ pH 2 S = y( x + y) = 625 …(ii) (i)NaBH 4
→  C CH 2  ⇒ 12 + 2 2 + 3 2 = r 2
x KA
∴ = = 185
. ⇒ x = 185 . y (ii)H + |
y KB ⇒ r=± 14
2 ° alcohol
1
On putting value of x in Eq. (i), we have CH3 ⇒ sinθ cos φ =
14
185 . y + y) = 1156
. y(185 HgSO4
H3C—C—C≡≡ C—H H SO 2
⇒ y = 14.81 mm and x = 27.40 mm 2 4 ⇒ sinθ sin φ =
14
∴Total pressure = px + py + pH 2 S CH3
3
= 2( x + y) CH3 ⇒ cosθ =
14
= 2(14.81 + 27.40) CH3—C— C—CH3
(i) NaBH4
(Q) sinθ cos φ 1
(ii) H+ ⇒ = ⇒ tan φ = 2
= 84.42 mm sinθ sin φ 2
CH3
420 JEE Advanced~Practice Set 7

1 − cos 2 θ 14 − 9 1 1 1 1 1 3 1
Also, tanθ = = ⇒ P( X ) = × × + × × A −2 = A −1 × A −1 =
cos θ2
9 2 4 4 2 4 4 ( ad − bc )2
1 3 1 1 1 1 − b  d − b
+ × × × × × d
5 5  −c
= = 2 4 4 2 4 4
 a   −c a 
9 3 1+ 3 + 3 + 1 8 1
⇒ P( X ) = = = 1
39. (a, b, c) We have, 32 32 4 ⇒ A −2 =
( ad − bc )2
π π  P( X ∩ X11 )
f( x ) = sin  sin sin x  
1 / 32 1
(a) P( X11 / X ) = = =
 6 2  P( X ) 1/ 4 8  d 2 + bc − bd − ba 
  … (ii)
We know that, P ( X ∩ X 2 ) 5 / 32 5  − cd − ac a 2 + bc 
(b) P( X / X 2 ) = = =
sin x ∈ [− 1,1] P( X 2 ) 1/ 4 8 a+d
⇒ tr( A −1 ) = , tr( A −2 )
π π π P ( X ∩ X1 ) 7 / 32 ad − bc
∴ sin x ∈  − ,  (c) P( X / X1 ) = =
2  2 2  P( X1 ) 1/ 2 a 2 + 2 bc + d 2
=
π = 7 /16 ( ad − bc )2
⇒ sin  sin x  ∈ [− 1, 1]
2 
(d) P (exactly two engines of the ship are Now, (tr( A −1 ))2 − tr( A −2 )
π π π π
⇒ sin  sin x  ∈  − ,  functioning /X) =
7 / 32
=7/8 ( a + d )2 a 2 + 2 bc + d 2
6 2   6 6  1/ 4 = −
( ad − bc )2 ( ad − bc )2
π π 
⇒ sin  sin  sin x  ∈  − , 
1 1 41. (a, b, c, d) We have,
6 2   2 2  1 1
cosα = x+  a 2 + 2 ad + d 2 − a 2 − 2 bc − d 2
2 x =
Hence, range of f( x ) is  − , .
1 1
 2 2  ( ad − bc )2
1 1 2 ( ad − bc )
and cosβ =  y +  , xy > 0 = =
2
π 2  y
We have, g ( x ) = sin x ( ad − bc )2 ad − bc
2 1
π π π  ∴ x+ ≥ 2 or ≤ − 2 2
fog ( x ) = sin  sin sin  sin x  x =
| A|
[Q| A| = ad − bc]
 6 2 2  1
and y + ≥ 2 or ≤ − 2 2
∴Range of fog ( x ) is  − , .
1 1 y = [Q| A| = 2018]
 2 2  2018
⇒ cos α = 1, − 1 or cos β = 1, − 1 1
f( x ) ∴ cos α cos β = 1 =
Now, lim 1009
x → 0 g ( x)
⇒ α + β is an even multiple of π ∴ m + n = 1 + 1009 = 1010
π π 
sin  sin  sin x  (cos α + cos β )2 = (1 + 1)2 = 4
44. (4) Given, sin−1(|log 26(cos x) − 1|)
6 2 
⇒ lim ⇒ π
x→ 0 π + cos −1 (|3 log 26(cos x ) − 7|) =
sin x sin(α + β + γ ) = sin(2 nπ + γ) = sin γ 2
2
Also, sinα = sinβ = 0 ∴|log 26(cos x ) − 1| = |3 log 26(cosx ) − 7|
π π 
sin  sin  sin x  ∴
2 6 2  [Q if sin−1 x + cos −1 y = π / 2,
= lim sin(α + β + γ ) = sinα + sin β + sin γ
x→ 0 π π  π  then, x = y]
sin  sin x 
6 2  42. (b, c) We have, Let log 26(cos x ) = m
π π f( x ) is continuous function in [0, 2]
sin  sin x  f( 0) = f(2 ) and f( x ) = f( x + 1)
∴ | m − 1| = |3m − 7|
6 2  ⇒ m − 1 = ± ( 3m − 7 )
× ∴f( x ) is a polynomial function.
sin x ⇒ m − 1 = 3m − 7
× x By Rolle’s theorem,
x or m − 1 = − 3m + 7
π f( x )has at least one root lie between [0, 2].
sin  sin x  π sin x Let g ( x ) = f( x ) − f( x + 1)
⇒ m = 3 or m = 2
2 π 2  2
= lim × × × ∴log 26(cos x ) = 3 or log 26(cos x ) = 2
x→ 0 π 6 π x g ( 0) = f( 0) − f(1) … (i)
sin x ⇒ log 6(cos x ) = ± 3
2 and g (1) = f(1) − f(2 ) … (ii)
1 π π or log 6(cos x ) = ± 2
= × = Adding Eqs. (i) and (ii), we get
3 2 6 g ( 0) + g (1) = f( 0) − f(2 ) ⇒ cos x = 6 − 3
or cos x = 6 − 2

Range of ⇒ g ( 0) + g (1) = 0 [Q f( 0) = f(2 )] ∴ Total number of solution is 4 in


 π 1 π 1  x ∈ [0, 4 π ].
gof( x ) ∈  − sin  , sin   ⇒ g ( 0) = − g (1)
 2 2 2 2 45. (80) First, we select 4 pairs in 5C 4
⇒ g( 0) and g(1) are of opposite sign.
⇒ gof( x ) ≠ 1 ⇒ f( x ) = f( x + 1) has at least one root ways = 5 ways
1 1 1 in [0, 1]. Now, we select exactly one shoe from
40. (c, d) P( X1 ) = , P( X 2 ) = , P( X 3 ) =
2 4 4 a b each pair in 2C1 ways.
43. (1010) Let A =  
P( X ) = P( X1 ∩ X 2 ∩ X 3 ) + P( X1 ∩ X 2 ∴ Required number of ways,
c d 
∩ X 13 ) + P( X1 ∩ X 12 ∩ X 3 ) + P( X11 ∩ X 2 = 5 × 2C1 × 2C1 × 2C1 × 2C1
1  d − b
∴ A −1 = … (i)
∩ X3) ad − bc  −c a  = 5 × 16 = 80
JEE Advanced~Practice Set 7 421

46. (6) Given function is, Then, the position vector of point D Putting in Eq. (iii), we get
 n + 3, if n is odd will be 2 / 5 b [19 − 3a ]2 = 20b
f ( n) =  n The equation of AD is,
, if n is even And, b = 2 a − 1
 2
[from Eq. (i)]
r = a + l  b − a 
2
Since, k is given to be an odd integer, 5  361 + 9a − 114a = 20 (2 a − 1)
2

hence
The equation of BE is, 9 a 2 − 114 a − 40 a + 361 + 20 = 0
f( k ) = k + 3
r = b + µ  a − b 
1
Now, k is odd ⇒ k + 3 is even
4  9a 2 − 154a + 381 = 0
[Since, odd + odd = even]
k+ 3 For intersection, we have 19 − c
⇒ f { f( k )} = f ( k + 3) = a=
a + λ  ⋅ b − a  = b + µ  a − b 
2 2 1 3
Hence, f[f{ f( k )}] = 27 5  4 
and then, 3c 2 + 40c − 700 = 0
µ
a  1 − λ −  + b  λ − 1 + µ  = 0
⇒ f{ f ( k + 3)} = 27 2
70
 4 5  So, after solving c = 10, −
k + 3
⇒ f   = 27
3
 2  From figure, a and b are non-collinear
c is number between 1 to 20,
vector
Now, there are two possible cases, µ So, c = 10
∴ 1− λ − = 0
k+3 k+ 3 4 c2
Case I Case II 2 b=
2 2 and λ + µ − 1= 0 20
5
is odd is even 10 2
5 2 ⇒ =5
k +3 k +3 ⇒ λ = ,µ = 20
If is odd, then If is an 6 3
2 2 1+ b
1 1 and a=
k + 3 even number, Hence, H= a+ b 2
f   = 27 then 6 3
 2  1+ 5
k + 3 Similarly, we are able to determine the ⇒ =3
k +3 
⇒ + 3 = 27
f   = 27 2
 2  position vector of the point F as,
2 a+ 2b Now, a + b + c = 3 + 5 + 10 = 18
 k + 3
⇒ k = 45   3
 2  51. (b) Area of region bounded by arc AP,
k + 3 45 + 3 ⇒ = 27 AF 2
⇒ = 2 ∴ = ⇒ k=2 the straight line PD and AD, i,e, ‘R’
2 2 FB 1
⇒k = 105 R = Area of sector APBA + area of
= 24 1
∆BPD
(which is not an odd ⇒
k +3 49. (2) Let I = ∫− 1[ x[1 + sin π x] + 1] dx
number) 2 A
0
Hence, this case is =
105 + 3
= 54
⇒ ∫− 1[ x[1 + sin π x] + 1]dx
not possible. 2
which is an even 1 a
number. + ∫0[ x[1 + sin πx] + 1]dx
Hence, the possible value of k is 105 ⇒ [1 + sin πx ] = 0 If − 1 < x < 0 E
∴ Sum of digits of k is, 1 + 0 + 5 = 6 and [1 + sin πx ] = 1 H F
47. (7) Given equation is, If 0 < x < 1
[tan x ]2 + tan x − a = 0 0 1 C b D B
I= ∫−1dx + ∫0[ x + 1] dx
2
Q [tan x ] and a are integral in nature. 1 2 1
1 Area ( R ) = (1) ( π − θ) + (1)(2 )sinθ
1 + 1 ∫ dx ⇒ 1 + [ x ]10 2 2
⇒ tan x is integer. 0
π θ
⇒ Equation reduces to 1 + (1 − 0) = 2 = − + sinθ
2 2
tan2 x + tan x − a = 0 50. (18) It is given that a, b, c are π
⇒ tan x (tan x + 1) = a = + sinθ − θ / 2
numbers between 1 to 20, such that 2
∴ a is product of consecutive integer. their sum = 18 dR 1
Here, a = 12, 20, 30, 42, 56, 72, 90 ⇒ = cos θ −
(i) 1, a, b are in AP dθ 2
∴ a has 7 values. (ii) b, c, 20 are in GP R is maximum, for θ = π / 3
48. (2) We taken C as origin, Now, a + b + c = 18 …(i) π 3 2π + 3 3
CA = a, CB = b ∴Rmax = + =
2a = 1 + b …(ii) 3 2 6
A
c 2 = 20b …(iii) 52. (c) L = 3 + ( π − θ)
From Eqs. (i) and (ii), we get + (2 − cos θ)2 + sin2 θ
a 2a − b = 1 L = 3 + ( π − θ) + 5 − 4 cos θ
E a + b + c = 18 dL 4 sinθ
H F = − 1+
3a + c = 19 …(iv) dθ 2 5 − 4 cos θ
C B
b D
Now, c = 19 − 3a
422 JEE Advanced~Practice Set 7

For maxima or minima


dL
=0 Lmax = 4 + π where, arg( z) < 0
dθ Lmin = 6 = − arg( z) − arg( − z)
4 sinθ ⇒ Lmax − Lmin = 4 + π − 6 = π − 2 = − {arg( z) + arg( − z)}
∴ − 1+ =0
2 5 − 4 cos θ ∴ 3 λ1 = − π
53. (b) For any complex number
⇒ 4 sin2 θ = 5 − 4 cos θ z = x + iy, x, y ∈ R the arg ∴ For arg z < 0, we have

⇒ 4 cos θ − 4 cos θ + 1 = 0
2
z = tan−1   always give the principal
y arg ( z) + arg( − z) = π
 x and 2 λ 2 = arg( − z) − arg( z)
1
⇒ cosθ = ⇒ θ = π/3 value.
2 = arg( − z) + arg( z)
We know that, arg ( z) = − arg ( z) 2λ2 = − π
L( 0) = 3 + π + 1 = 4 + π
arg( − z) − arg( z) = arg( − z) + arg( z) ∴ 3λ1 − 2 λ 2 = 0 ...
L( π ) = 3 + 5+ 4=6 choice (b)
= − π as arg( z) > 0 5π
2π 4 where as, 2 λ1 − 3λ 2 = ≠0
L( π / 3 ) = 3 + + 5−
3 2 6

54. (b) arg( z) + arg( z) = 0 for arg z < 0 λ1 − λ 2 =/ π / 6 ≠ 5 π / 6
=3 3+
3 3λ1 = arg( z) − arg( − z), and λ 2 − λ1 =/ − π / 6

Paper 2
∆g ∆l 2 ∆t 
1. (b, c) = × 100 =  +  × 100
For first lens,
Net force on α-particle is g  l t  f1 = − 20 cm, u1 = − ∞
Fnet = q α [E + v × B ] So, error is inversely proportional to l 1 1 1
= + ⇒ v1 = − 20 cm
So, acceleration of α-particle is and t . Hence, observation is least v1 f1 x1
q accurate for C and most accurate For second lens, f2 = + 30 cm
a net = α (E + v × B ) for A.
mα x2 = − (20 + 8) = − 28 cm
qα 4. (b, c, d) 1 1 1
= . { 4$i − $j − 2k$ + So, = +
mα Let a parallel beam is incident on v 2 f2 u 2
convex lens.
(2 $i + 3$j − k$ ) × (2 $i + 4$j + k$ )} Gives, v 2 = − 420 cm
q So, beam appears to diverge from a
= α (13$i − 5$j ) point 416 cm from the centre of lens

system.
$
As, a ⋅ k = 0 ⇒ a is perpendicular to So, we do not have a simple equation
Z-axis. true for all x (and v).
h h 8cm
Also, λ = = The motion of effective focal length.
p m(u + at ) For first lens, f1 = + 30 cm ∴Does not seem to be meaningful for
So, λ decreases with time. x1 = − ∞ this system.
Trajectory of particle is helical with 1 1 1 5. (a, b, c)
Now, = +
increasing radius. v1 f1 u1 dU
Force, F( x ) = −
2. (a,d) Gives v1 = 30 cm. dx
In the given graph, the region AB For second lens,  −c 2cx 2 
represents no change in temperature =− 2 + 2 
x + a ( x + a 2 )2 
2
f2 = − 20 cm,
with time. It means ice and water are
in thermal equilibrium. The region BC x2 = 30 − 8 = + 22 cm c 2cx 2
1 −1 1 = −
shows the change in temperature with ∴ = + x + a
2 2
( x + a2 )
2
time. The region CD represents a v 2 20 22
constant temperature (100°C) with For stable equilibrium, F( x ) = 0
⇒ v 2 = − 220 cm
time. It means, water and steam are in c 2cx 2
So, the parallel beam appears to ⇒ =
thermal equilibrium at boiling point. x + a
2 2
( x + a 2 )2
2
diverge from a point 216 cm.
3. (a,d) ⇒ 2 x2 = x2 + a2
From centre of lens system, when
l
We know that, T = 2 π parallel beam is incident over concave ⇒ x = ± a.
g lens, then d 2U
t l Force constant, k =
⇒ = 2π dx 2 x=a
n g
 6 cx 8 cx  3
4 π 2n 2l = 2 − 2
⇒ g = 2 3
t 2  ( x + a 2 2
) ( x + a ) x = a
So, percentage error in g c
8cm =
2 a3
JEE Advanced~Practice Set 7 423

Also, angular frequency of small Energy ratio is 16. (a) A → s, B → r, C → q, D → p.


oscillations, 2
Eresultant
=   = ( 2 + 1)2
A 17. (a) When ∆Q = 0, then
k c  a
ω= = . Ering ∆S = 0
m 2 a 3m
= (3 + 2 2 ) For all other processes,
PE is negative, when x > 0.
6. (b,c)
∴ Eresultant = 5.82 J
∆S =
∫ dQ
10. (1.50) T
The given equation is
2 πx  Thermal stress = Yα∆θ So, A → r, B → q C → p, D → s
y ( x, t ) = 0.06 sin   cos(120 πt ) As, thermal stresses are equal
 3  18. (d) Conduction in solids at low
(b) As terms involving x and t are Y1α1∆θ 1 = Y2α 2∆θ 2 temperature is due to random erratic
independent of each other, the given Y1 α 2 3 lattice vibration (phenon vibration). At
⇒ = = .
equation represents a stationary Y2 α 1 2 high temperature, electrons also
wave. carries energy from one place to
11. (76.00) another. Conduction in gases is due
(c) Compare the given equation with
the standard form of equation of q 2B 2r 2 to molecular collisions. Conduction in
E= = 76 MeV
stationary wave, 2m liquids due to molecular collisions and
y ( x, t ) = 2 r sin kx cos ωt 12. (0.75) electrons drift.
2π 2π
k= = We know that, 19. (b) Ammonia forms the complex ion
λ 3 [Cu(NH 3 )4 ]2+ with copper ions in the
Z = R +
2
Xc2
∴ λ = 3m alkaline solutions but not in acidic
ω = 120 π = R 2 + (2 πνC )−2 solutions. The reason is that in acidic
ω 120 π solutions, protons co-ordinate with
∴ v = = = 60 Hz = (200)2+ (212 )2
2π 2π ammonia molecules forming NH 4+ ions
and v = νλ = 60 × 3 = 180 m/s = 2915
. Ω and NH 3 molecules are not available.
Hence, the given stationary wave is So, current in circuit, 20. (a, b, c)
the result of superposition of two V
Irms = rms According to the question,
waves of wavelength 3 m and Z NaOH, ∆ H 2O
frequency 60 Hz each, travelling with a 220 ( A ) →( B) ← Mg 3N 2
= = 075
. A
velocity of 180 m/s in opposite 2915
. A has NH 4+ ion and B is NH 3.
directions.
13. (6.53)
7. (3.00) Mg 3N 2 + 6H 2O → 3Mg(OH)2 + 2NH 3
Potential energy = Kinetic energy +
2/ 3 NH+4 + NaOH → Na + + NH 3 + H 2O
T  Rotational energy
T ∝ r 3/ 2 ⇒ r2 =  2  . r1
 T1  1 1 Resultant solution after NH 3 has
mgh = mv 2 + Iω 2 escaped completely contains Na +
2/ 3 2 2
=  
8 and anion of ( A ) which also gives ring
× 10 4 = 4 × 10 4 km 1 1  mr 2  v 2
 1 = mv 2 +   test of NO −3 . Thus, ( A ) is NH 4NO 3.
2 π × r1 2 2  2  r2
Now, v1 = = 2 π × 10 4 km/h Explanation
T1 ∆
∴ v2 =
4
gh NH 4NO 3 + NaOH → NH 3 ↑
2 πr2
and v 2 = = π × 10 4 km/h 3 (A) ( B)
T2
Using v = u + 2 gh, we get
2 2 + NaNO 3 + H 2O
So, ω r = angular speed of S 2 relative 2 NaNO 3 + H 2SO 4 → NaHSO 4 + HNO 3
to S1 a= g
3
v 2 − v1
= = 3.0 × 10 −4 rad/s = 6.53 ms −2 3H 2SO 4 + 6FeSO 4 + 2HNO 3
r2 − r1 → 3Fe 2(SO 4 )3 + 2NO + 4H 2O
14. (9.60)
8. (0.62) [Fe(H 2O)6 ]SO 4 + NO
Let, y = length of oil column, then
We know that, 10 × 13
. × g = y × 0.8 × g → [Fe(H 2O)5NO]SO 4 + H 2O
Brown coloured ring (C )
( 4 πε0 ) mR 3 + (10 − y) × 13.6 × g
T = 2π ∆
NH 4NO 3 → N 2O + 2H 2O
Qq ⇒ 13 = 0.8 y + 136 − 13.6 y (A) (D ) (E)
−4 123
9 × 10 ×1
3
⇒ y= = 9.6 cm N 2O + H 2O → H 2N 2O 2( F )
= 2π 12.8
9 × 10 9 × 10 −5 × 10 −6 (Nitrous acid)

15. (d) A → s, B → q, C → r, D → p. It exist in following two forms.


= 0.62 s
X L = ω L, ⇒ X L ∝ ω OH
9. (5.82) N OH N
1 1
ynet = y1 + y2 + y3 XC = ⇒ XC ∝ ,
Cω ω N N
= a [sinωt + sin(ωt + 45° ) HO OH
R is independent of ω, Z first
+ sin(ωt + 90° )] trans cis
decreases, then increases after ω r .
= ( 2 + 1) a sin (ωt + 45° )
424 JEE Advanced~Practice Set 7

21. (d) 25. (6) SO 3, XeO 3, H 3PO 4 , ClO ¨4 , SO 2−


4 ,
= 6.023 × 10 23 × 10 3
K2Cr2O7
(CH3)3 C CH2 CH2 OH XeOF2 have dπ - p π bonding. This = 6.023 × 10 26
H2SO4
H2O, ∆ type of bonding is important in the Surface sites adsorbed by N 2(20%)
compounds containing third (or 20
SOCl2
= 6.02 × 10 26 ×
(CH3)3 C CH2 COOH higher) period elements (Si, P, S, Cl, 100
(A) etc.) These elements have vacant = 12.04 × 10 24
(CH3)2NH
3d-orbitals. These vacant d-orbital ∴Surface sites adsorbed by one N 2
(CH3)3 C CH2 COCl –HCl form (d - p) π bonding when Si, P, S 12.04 × 10 24
(B) etc, are bonded with N, O, F which molecule = = 2.02 × 10 8
O 6.02 × 1016
have lone pair electrons in their
p-orbitals. 28. (4) Total number of chiral centres
(CH3)3CCH2—C —N(CH3)2 present in the given molecules are 2
(C) 26. (11.36) At 15 min, let α be the degree
(two).
of dissociation of N 2O 5.
(i) LiAlH4,
1 * OH
CH3 ether N 2O 5 → 2NO 2 + O2
(CH3)3C CH2 CH2N (ii) H2O 2 *
(D) CH3 N
OH
22. (a) Reactant is β-D glucopyranosyl, so Initially 1 0 0
the product formed will also be OH
D-glucitol. In this reaction NaBH 4 At 15 min 1− α 2α α
reduces  CHO group into CH 2OH. 2

23. (d) AB - A + + B− , K1 Hence, the total number of


Now, stereoisomers = 2 2 = 4
…(i)
AB + B− - AB2− , K 2 …(ii) rate of effusion of NO 2 2α 32
= 29. (143) Mass percent of
rate of effusion of O 2 α 46 12 100
+
2 AB A +
- AB2− 2 C = × 0138
. × = 25
44 015
.
K = K1 × K 2  mole fraction of NO 2 
Also, subtracting equation (ii) from (i), =  1
H = × 0.0566 ×
100
= 42
.
 mole fraction of O 2  outside 9 015
.
we have
K 32 0.66 14 100
AB2− - A + + 2 B− ; K = 1 ⇒4 = N= × 0238
. × = 9.8
K2 46 xO 2 17 2
35.5 100
K1 [ A + ][B− ]2 Cl = × 0251
. × = 49.67
⇒ = xO 2 =
0.66 46
≈ 02. 143.5 0125
.
K2 [ AB2− ] 4 × 32 O = 1133.
[A + ] K1 1 ⇒ Mole fraction of N 2O 5 in mixture
⇒ = × Emperical formula determination
[ AB2− ] K 2 [B− ]2 collected outside after 15 min = 014
.
rN 2O 5 1 − α 46 014
.
24. (a, b, c, d) ∴ = = Element C H N Cl O
rNO 2 2α 108 0.66
(a) At a particular pressure (separately m% 25 4. 9.8 49.6 11.3
1− α
for (O 2 and N 2) ideal behaviour is ⇒ = 0.33 ⇒ α = 0.60 2 7 3
reached. 2α
n = (m / M) 2.0 4. 0.7 1.4 0.7
Thus, z=1 Now, applying first order kinetics, 8 2
1
(b) For H 2, attractive force is zero. k × 15 = ln Simple 2.9 6 1 2 1
1 − 0.6
Hence, p( V − b ) = RT ratio 7
1 5
pV = RT + pb ⇒ k= ln
15 2
PV pb Emperical formula = C 3H 6NCl 2O
z= = 1+ ⇒ k = 0.061
RT RT Molecular mass
In 2
∴ z> 1 ∴ t 1/ 2
= = 1136
. min = 12 × 3 + 6 × 1 + 14 + 2
0.061
(c) For N 2 (in low pressure range) p is −3 × 35.5 + 16
27. (2.02) V = 2.46 cm = 2.46 × 10 L
3
low, V is high = 143
pV
Thus, (V − b ) ≈ V n=
RT 30. (0.90) Given, monoatomic gas is
 P + a  V = RT 0.001 × 2.46 × 10 −3 being expanded adiabatically,
  ∴ n(N 2 ) =
 V2 p1 = 10
. k Pa = 1000 Pa, V1 = 1 dm3
0.0821 × 298
a
pV = RT − . × 10 −7 mol
= 10 = 0.001 dm3, n = 0.4, p2 = 01
. kPa
V
pV  a  Moleculesd (N 2 ) = n × NO For monoatomic gas,
z= = 1 −  . × 10 −7 × 6.02 × 10 23
= 10 5 3
RT  RTV  γ = and C V = R
= 6.02 × 1016 molecules 3 2
∴ z< 1
pV 1000 × 0.001
(d) This statement is also correct (same
Total surface sites on surface of ∴T1 = 1 1 = = 0.300 K
10 3 cm2. nR 0.4 × 8.314
as in (c)).
JEE Advanced~Practice Set 7 425

. × (1)5/ 3
p1( V1 )5/ 3 10 ∴ Keq = 178
. × 1017 35. (a) Borodine-Hunsdiecker reaction
⇒ ( V2 )5/ 3 = =
p2 01. 2+ − + CCl 4
Let x be the concentration of Ni that (A) PhCOO Ag + Br2 → PhBr
[Q p1V1γ = p2V2γ ] have been reduced to nickel at ∆
+ CO 2 + AgBr
equilibrium.
∴ V2 = 3.98 dm3 Birnbaum-Simonini reaction
2+ 2+
5 
Zn( s ) + Ni ( aq ) - Zn ( aq )
− 1 (1 − x ) (B)
1   3
x
T2 = 0.300

Also,  − +
 3.98  + Ni( s ) C 2H 5COOAg + I 2 → C 2H 5COOC 2H 5
 γ −1 [Zn2 + ] x
V   Keq = = = 178
. × 1017 + 2AgI
Q T2 = T1  1   [Ni 2 + ] 1− x (C)
  V2  

∴ x = 10
. M
∴ T2 = 0.300( 0.398) = 012
. K + CH2I2
So, (1 − x ) = [Ni 2 + ] = 5.62 × 10 −18 M
now, W = − nC V (T2 − T1 ) Zn/Cu
3 33. (c) A → p, s; B → p, q, r, t;
∴W = − 0.4 × × 8.314( 012. − 0.300)
2 C → p, q, t; D → p
Bischler-Napieralski reaction
= 0.90 J 3XeF4 + 6H 2O → XeO 3 + 2Xe
31. (6.98) 3 POCl3
+ 12HF + O2
Since, density of H 2O (at 23° C) is 2 –HOPOCl2
NH
1 g cm−3 (concentration of H 2O) Cl 2 + H 2O → HCl + HOCl
1000 1 C
= 1000 g L−1 = mol L−1 → HCl + O2 NH
18 2 Me
10 6 VCl 5 + H 2O → VOCl 3 + 2HCl Me
= mol m−3 Bucherer reaction
18 34. (b) P → 1; Q → 2; R → 3; S → 3, 4, 5
Now, NH2
COO–Na+
λ 0(H 2O) = λ 0(H + ) + λ 0(OH − ) aq. NaHSO3
= ( 3.4982 × 10 −2 + 198
. × 10 −2 ) P
CaO
+ NaOH ∆ OH
S m2 mol −1
= 5.4782 × 10 −2 ∆
2 −1
S m mol –H2

K . × 10 −6 S m−1
57 36. (b) (i) → (t), (ii) → (q), (iii) → (p),
λC = = +–
N2 Cl
C 10 6 NH2 (iv) → (r)
mol m−3
18 Before current is passed mixture of
= 1026
. × 10 −10 S m2 mol −1 Q
(i) NaNO2+HCl H 2PO −4 and HPO 2−
4 is a buffer of
(ii) KI, ∆
equimolar value.
λC 1026
. × 10 −10 S m2 mol −1
α = = Aniline H 2 PO −4 4 H + HPO 24−
λ 0 5.4782 × 10 −2 S m2 mol −1
I Thus, by Hasselbalch Henderson
= 1875
. × 10 −9
equation
1000 KI Cu
∴[H + ] = [OH − ] = Cα = ∆

[HPO 24− ]
18 pH = pK a + log
[H 2PO 4 ]
× 1875
. × 10 −9 Iodobenzene
NH2
= 2.15 + log  
1
= 1042
. × 10 −7 M (i) Conc. H2SO4
+Conc. HNO3
 1
∴ pH = − log [H + ] ` R
(ii) Sn/HCl
⇒ pK a = 2.15
= − log [1042
. × 10 −7 ] = 6.98
Benzene Aniline (ii) → (q)
32. (5.62) The reaction can be On electrolysis,
NaNO2
considered as: — Anodic reaction
Zn( s ) + Ni 2+ ( aq ) - Zn2+ ( aq ) 2H 2O( l ) → 4H + + O 2 + 4e −
NO2
+ Ni( s ) Cathodic reaction
Zn, HCl H H HCl
The cell involving the reaction would S —N N—
4H 2O( l ) + 4e − → 4OH − + 2H 2
be:
Zn( s )| Zn2+ ( aq )|| Ni 2+ ( aq )|Ni( s ) ∴H + formed = zit
H2N— — —NH2 1 × 201 × 60
E°cell = − 024
. + 075. = 0.51 Volt = 125
. ×
96500
nFE °
log Keq = NaNO2 = 0156
. g L−1 = 0156
. M at anode
2.303RT HCl,

2 × 96500 × 0.51 H3PO2 Thus, OH − formed = 0156
. M at
= = 17.25
2.303 × 8.314 × 298 cathode
426 JEE Advanced~Practice Set 7

(iii) → (p) ∴ r1 + r2 > d > r1 − r2 Since, C is the excentre of ∆ABP,


At cathodic side [OH − ] is formed. Hence, the circle intersect at two We have,
H 2PO −4 + OH 4 −
HPO 42− + H 2O distinct point. ∠APQ = ∠QPC
There are two common tangents. Also, R is the excentre of ∆APC.
Initial conc. 1.0 M 0 1.0 M Also, So, ∠APR = ∠RPS
2 g 1g 2 + 2 f1f2 = 2(1)( 3) + 2(2 ) ( − 4) So, ∠RPQ = 90°
After 1 − 0.156 0 (1.0 + 0.156)
= 0.844 M = 1156
. M = − 10  a2 + b 2 − c 2 
Electrolysis
and c1 + c 2 = − 20 + 10 = − 10 cosC =  
 2 ab 
[HPO 24− ] Thus, the two circles are orthogonal.
∴ pH = pK a + log 64 + 25 − 49 1
[H 2PO 4− ] Length of common tangent cos C = =
2( 8) ( 5) 2
[1156
. ] = d 2 − ( r1 − 2)2
⇒ 2.15 + log = 2.29
[0.844] In ∆PQC, we have
1/ 4
40 − ( 5 − 15 ) = 5  
12 CQ 2 + PC 2 − PQ 2
(iv) → (r) In anodic side = 2
cosC =
 5 2 CQ ⋅ PC
HPO 24− +H +
- H 2PO −4
Length of common chord 1 CQ 2 + PC 2 − PQ 2
2 r1 r2 3 ⇒ =
= =5 2 2 CQ ⋅ PC
Initial conc. 1.0 M 0 1.0 M 2
r12 + r22 ⇒ PQ 2 = CQ 2 + PC 2 − CQ ⋅ PC
After 0.844 0 1156
. 39. (a,b, d) (a)
M M ⇒ PQ = CQ 2 + PC 2 − CQ ⋅ PC
Electrolysis 6 = 0(2 ) + 6(1) = 12
( ) + 4(1) = 2(2 ) + 2(1)
100 64 80
= 3(2 ) + 0(1) ⇒ PQ = + −
9 9 9
[log HPO 24− ] Total number of ways
∴pH = pK a + log  ab ab 
[H 2PO −4 ] 5! 4! 3! Q PC = b + c and CQ = a + c 
=1+ + +  
[0.844] 4 ! 2 !2 ! 3 !
⇒ pH = 2.15 + log = 2.01
= 1 + 5 + 6 + 1 = 13 84
[1156
. ] ⇒ PQ =
∴ f( 6) = 13 3
37. (a, c), We have,
sin x cos x sin x (b) f( f( 6)) = f(13) 41. (b, d) Equation of chord AB is,
f( x ) = cos x sin x cos x , 13 = 0(2 ) + 13(1) = 12
( ) + 111
( )
= 2(2 ) + 9(1) = 3(2 ) + 7(1) A (at12, 2at1)
cos x sin x sin x
= 4(2 ) + 5(1) = 5(2 ) + 3(1)
f( x ) = sin3 x + cos 3 x − cos x sin x
= 6(2 ) + 11
( )
(sin x + cos x ) Total number of ways
= (sin x + cos x )3 − 4 sin x cos x C (c, 0)=(–at1t2, 0)
12 ! 11! 10 ! 9!
=1+ + + +
(sin x + cos x ) 11! 2 ! 9 ! 3 !7 ! 4 ! 5 !
B (at22, 2at2)
= (sin x + cos x ) (1 − sin2 x ) 8! 7!
+ +
Now, f( x ) = 0 5 ! 3 ! 6 !1!
∴ (sin x + cos x ) (1 − sin2 x ) = 0 = 1 + 12 + 55 + 120 + 126 + 56 + 7 2 a(t 2 − t 1 )
= 377 y − 2 at 1 = ( x − at 12 )
tan x = − 1 or sin2 x = 1 a(t 22 − t 12 )
⇒ f( x ) = 0 has 2 real solution in [0, π]. ∴ f( f( 6)) = f(13) = 377
2
f(1) = 1, f(2 ) = 2 ⇒ y − 2 at 1 = ( x − at 12 )
Also, f( x ) = (sin x + cos x )(1 − sin2 x ) t 2 + t1
f( x ) f( 3) = 3, f( 4) = 5
⇒ = sin x + cos x This chord cut axis of parabola C (c, 0)
1 − sin2 x By taking higher value of n in f( x ), we
2
f( x ) always get more value of f( x ). Hence, ∴ 0 − 2 at 1 = (c − at 12 )
∴ Range of is [− 2 , 2 ]. t 2 + t1
f( x ) is one-one. Clearly, f( x ) is into.
1 − sin2 x
40. (c, d) Given, ⇒ c = − at 1t 2
38. (a, b, c, d) We have, ⇒
c1 ≡ x 2 + y 2 + 2 x + 4 y − 20 = 0 AC 2 ( at 12 + at 1t 2 )2 + 4a 2t 12 t 12
A 60º = =
c 2 ≡ x + y + 6 x − 8 y + 10 = 0
2 2 CB 2
( at 2 + at 1t 2 ) +
2
4a 2t 22 t 22

Centre of c1 ≡ ( − 1, − 2 ) 60º 60º CB t


⇒ =± 2
AC t1
and radius r1 = 1 + 4 + 20 = 5
R Q CB t
Centre of c 2 ≡ ( − 3, 4) ⇒ + 1= ± 2 + 1
AC t1
and radius r2 = 9 + 16 − 10 = 15
AB t 2 + t 1
Distance between the centre, ⇒ = =3
AC t1
d = ( − 3 + 1) + ( 4 + 2 )
2 2
B P C
AB t 1 − t 2
= 4 + 36 = 40 ∠BAC = 120° or = =3
AC t1
JEE Advanced~Practice Set 7 427

⇒ t 2 + t 1 = 3 t 1 or t 1 − t 2 = 3 t 1 Now, or λ + 3µ = 5 and 6λ − 5µ = 7
⇒ 2t 1 = t 2 or 2t 1 + t 2 = 0 x x x 15 x + 10 x + 6 x 31 ⇒ λ = 2, µ = 1
+ + = = x
2 3 5 30 30
⇒ 6t 12 − 2t 22 − t 1t 2 = 0 Hence, the points C and D are
…(ii) (1, 10, 10) and (7, 4, 7)
⇒ 6t 12 + 3t 1t 2 − 4t 1t 2 − 2t 22 = 0
Also, LHS of given Eq. (i) is integer ⇒ CD = (7 − 1)2 + ( 4 − 10)2 + (7 − 10)2
⇒ ( 3 t 1 − 2t 2 ) (2t 1 + t 2 ) = 0 ∴RHS is an integer
= 36 + 36 + 9
42. (a, b, c), We have, As evident from Eq. (ii) sum of x
f( x + y) = f( x ) + f( y) + 3 xy ( x + y) … (i) x x x
+ + is equal to
31
x. 47. (0) We have, f( x) = ∫− 1 4 + t 2 dt
On differentiating w.r.t. x y as constant 2 3 5 30 1

f ′ ( x + y) = f ′ ( x ) + 6 xy + 3 y 2
x x x
∴ , , are all integers and
31
x is
and g ( x) = ∫x 4 + t 2 dt
2 3 5 30 Let h( x ) = f( x ) ⋅ g ( x )
Put x=0 Also an integer
f ′ ( y) = f ′ ( 0) + 3 y 2 ∴h′ ( x ) = ( f( x ) ⋅ g ( x ))′ = f( x )g ′ ( x )
∴ x = multiple of 2, 3, 5 and 30
+ f ′ ( x )g ( x )
⇒ f ′ ( y) = 3 y 2 − 4 [Q f ′ ( 0) = 4] Thus, x = 30 × 1, 30 × 2, … , 30 × 33 x
⇒ h′ ( x ) = − ∫ 4 + t 2 dt 4 + x2
⇒ f ′( x) = 3 x 2 − 4 45. (8) tan4 x + cot 4 x + 2 = 4 sin2 y −1
1
On integrating, we get ⇒ (tan2 x − cot 2 x )2 + 4 = 4 sin2 y + 4 + x2 ∫ 4 + t 2dt
x
f( x ) = x − 4 x + c
3
Now, LHS ≥ 4 and RHS ≤ 4 − x

Now, put x = y = 0 in Eq. (i), we get ⇒ h′ ( x ) = 4 + x2  ∫− 1 4 + t 2 dt

⇒ LHS = RHS = 4 1
 + ∫x 4 + t dt 2
f( 0) = 0 ⇒ tan2 x − cot 2 x = 0  
∴ f ( 0) = 0 + c ⇒ c = 0  1 4 + t 2 dt 
⇒ h ′ ( 0 ) = 4 + 0  ∫0 0
and sin y = 1 and sin y = ± 1
2
…(i)

∴ f( x ) = x 3 − 4 x ⇒ tan x = 1 ⇒ tan x = ± 1
4
...(ii) − ∫ 4 + t 2 dt 
 −1 
f( x ) = x( x + 2 ) ( x − 2 ) But ( x, y) lies on circle x 2 + y 2 = 9  1 4 + t 2 dt 
f( x ) = 0 Q −3 ≤ x ≤ 3, −3 ≤ y ≤ 3 ⇒ h ′ ( 0 ) = 2  ∫0 0 
⇒ x( x + 2 ) ( x − 2 ) = 0  + ∫ 4 + y 2 dy
Y  1 
x = 0, − 2, 2 (0, 3) where, t = − y ⇒ h′ ( 0) = 0
Hence, three roots f( x ) is passing a b c
through (2, 0) and ( − 1, 3) 48. (12) Let the matrix be  p q r .
X  
f( x ) = x 3 − 4 x is defined (–3, 0) (0, 0) (3, 0)  x y z
x 3 − 4 x ≥ 0 or x ∈ [− 2, 0] ∪ [2, ∞ ) We have five entries as 1 and
(0, –3)
remaining four entries are 0. Since,
∴ The domain of f( x ) is
Values of x and y satisfying Eqs. (i) matrix is symmetric, we must have
[− 2, 0] ∪ [2, ∞ ). even number of zero for i ≠ j
n−1 and (ii) are
43. (11) Given 2 students gave wrong
π (i) If two zeroes are the entries in the
answer to atleast 1 questions. x=± and ± 3 π / 4, diagonal, then
4
2 n − 1 students gave wrong answer to 3
C 2 × 3C1 = 3 × 3 = 9
π 3π
atleast 1 questions. y= ± ,±
2 2 (ii) If all the entries in the principle
2 n − 2 students gave wrong answer to
Hence, number of points P ( x, y) are 8. diagonal is 1, then 3C1 = 3
atleast 2 questions.
2 n − 1 − 2 n − 2 students gave wrong 46. (9) Equation of line L1 are ∴ Total number of matrices
answer to exactly one equation. x−7 y− 6 z−2 = 9 + 3 = 12
= = =λ → → →
(exactly 1 wrong) + (exactly 2 wrong) −3 2 4 49. (4) We have,|a| = |b| = |c| = 1
So, that any point on it is → → → → → → 1
+ …+ (exactly n wrong) = 2047 a ⋅ b = b ⋅ c = c ⋅a =
n −1 n− 2 n− 2 n− 3 (7 − 3λ, 6 + 2 λ, 2 + 4λ ). 2
1 (2 −2 ) + 2 [2 −2 ]
Let this point be C. → → → → → → →
+…+ ( n − 1) [2 − 2 ] + n (1) = 2047
1 0 Also, (a × b ) + (b × c ) = pa + q b + r c
Equation of line L2 are
→ → → → → → → →
1 ⋅ 2 n − 1 + 1.2 n − 2 + 1.2 n − 3 x−5 y−3 z−4 ⇒ a ⋅ (b × c ) = p + q (a ⋅ b ) + r (a ⋅ c )
= = =µ
+ … + 1.2 ° 2 1 3 →→→ q r
⇒ [a b c ] = p + + … (i)
= 2047 So, that any point on it is 2 2
2n − 1 ( 5 + 2µ , 3 + µ, 4 + 3µ). Similarly, taking dot product with
⇒ = 2047 ⇒ 2 n − 1 = 2047 →
2 −1 Let this point be D. vector b, we get
Now, C and D lie on line L3 with →→→ p r
⇒ 2 n = 2048 [a b c ] = +q + …(ii)
direction ratio (2, − 2, − 1). 2 2
2 n = 2 11 ⇒ n = 11 2 − 3λ − 2µ 3 + 2λ − µ →
Hence, = and taking dot product with vector c ,
44. (33) Given equation is, 2 −2
we get
 x  +  x  +  x  = 31 x − 2 + 4λ − 3µ
…(i) = →→→ p q
 2   3   5  30 −1 [a b c ] = + + r … (iii)
2 2
428 JEE Advanced~Practice Set 7

+ 4 − 
Solving Eqs. (i), (ii) and (iii), we get 3 A B
Similarly, rb = r cot 2 , rc = r cot 2
p= r = −q  4 2 2
2
p2 + 2q 2 + r 2 q 2 + 2q 2 + q 2 = 1− +
3 27 27
− =− <0
1
rarb rc = r 3  cot + cot + cot 
A B C
∴ = =4
q2 q2 2 16 16 2  2 2 2
∴ f( x ) changes its sign in  − , −  .
3 1 ⇒ ra rb + rb rc + rc ra
50. (28) Let at any moment concentration
 4 2
= r  cot cot + cot cot
of solution is x gm/L. C A A B
So, f( x ) = 0 has its rootsS is  − , − 
3 1  2 2 2 2
Let change is dx in dt time
 4 2
cot 
2x B C
∴ dx = 4dt − dt 3 1 + cot
50 S. We have,|S| = t as − <S< − , 2 2
4 2 A B C
⇒ 25 dx = (100 − x ) dt = 4R sin sin sin
1 3 2 2 2
dx we get < 1 <
⇒ 25 = dt 1 4  cot C cot A + cot A cot B
100 − x 
Here,  2 2 2 2
80 dx t
25∫ ∫0 dt cot 
⇒ =
1/ 2 t 3/ 4 B C
40 100 − x ∫0 f( x ) dx < ∫0 f( x) dx < ∫0 f( x ) dx
2 2
+ cot

⇒ − 25 [log(100 − x )]80 … (A)  A B C A B


40 = [t ] 0
t
= 4R  cos sin cos + cos cos
Y  2 2 2 2 2
60
⇒ =t
sin + sin cos cos 
25 log C A B C
20
2 2 2 2
⇒ t = 25 loge 3 f(x)=0 = r2 + r3 + r1
Q a = 25, b = 3 ⇒ a + b = 28
sin2 x + 2 tan x 54. (d) P. Putting x = 2 , we get
51. (b) Let I = ∫ dx 6m
(cos 6 x + 6 cos 2 x + 4)
∑ C r 2 r / 2 = (1 + 2 )6m
6m

2  cos x +
1  X r= 0
 sin x O 1 t 3
 cos x  = ( 3 + 2 2 )3m
=∫ dx 2 4
(cos x + 6 cos x + 4)
6 2 3m
1
Q. ∑ ( − 1) C 2r = r 6m
Now, f( x ) = 1 + 2 x + 3 x 2 + 4 x 3 2
1 6 4 r= 0
= log 1 + + +C t

∫0 f( x) dx = [ x +
6m
12 cos 4 x cos 6 x x 2 + x 3 + x 4 ] t0 6m  1 1 
( 2 )  + i 
Here, a1 = 12, a2 = 1, a3 = 4, a4 = 6   2 2
From (A), we get
52. (a) f ′ ( x) = 2 + 6 x + 12 x 2 1 1 1 1 t 6m

 −1 
+ + + < ∫0 f( x) dx + ( 2 )6m 
1
−i
1 
 
f ′ ′ ( x ) = 6 + 24 x = 24  x −   
2 4 8 16  2 2 
  4  3
+
9
<
+
27
+
81
1 3mπ
⇒ f ′ ′ ( x ) < 0 if x < − 4 16 64 256 = 2 3m ⋅ cos
4 15 t 525 2
16 ∫0
1 ⇒ < f( x ) dx <
and f ′ ′ ( x ) > 0 if x > − 256  0, if m is odd
4 3 15 t 525 = m

4 16 ∫0
1 ∴ < < f( x ) dx < <3 ( − 1) 2 2 3m if m iseven
P.∴ f ′′( x ) is increasing if x > − , i.e. 256
4
 − 1, t  C R. Putting x = 1, we get
  53. (c) DC = r cot 6m
 4  2 Σ 6m
C r = (1 + 1)6m = 64 m
1 C C C r=0
Q. and f ′′( x ) is decreasing if x < − , i.e. ra = O1D = r cot cot = r cot 2
4 2 2 2 3m
 − t, − 1 A S. Σ ( − 3)r − 1 6mC 2r − 1
  r =1
 4
1
R. f ′ ( x ) = 2 + 6 x + 12 x 2 for f ′ ( x ) = 0 = { 3i 6m
C1 + ( 3i )3 6m
C3 +
3!
discriminant = 6 2 − 4 ⋅ 12 ⋅ 2 < 0 6m
( 3i )5 C1 + ... + ( 3 i )6m − 1 6m
C 6m − 1}
So, f ′ ( x ) = 0 has imaginary roots,
therefore f( x ) = 0 must have a pair of ⇒ (1 + 3i) 6m
= 6m
C0 + 3i 6m
C1
imaginary roots. f( x ) = 0 being a cubic + ( 3i ) 2 6m
C2 + ( 3 i ) 3 6m
C 3 + ...
O
equation, f( x ) = 0 has only one real
roots which is S. C/2 ⇒ (1 − 3 i )6m = 6m
C0 − 3i 6m
C1
Now, B C + ( 3 i )2 6m
C 2 − ( 3 i )3 6mC 3 + ...
2 D
f  −  = 1 + 2  −  + 3  − 
1 1 1
 2  2  2 ∴ (1 + 3 i )6m − (1 − 3 i )6m
C/2

3
= 2 [ 3i 6m
C1 + ( 3 i )3 C 3 + ... ]
6m
+ 4 − 
1
 2
3 1 O1 ∴ Given expression,
= 1− 1+ − > 0 1
4 2 [(1 + 3 i )6m − (1 − 3 i )]
⇒  −  = 1 + 2  −  + 3  − 
3 3 3 2 3i
 4  4  4 =0
PRACTICE SET - 8
Paper 1
1. (a,b,c,d) For sudden compression, 2 ( f1 − f2 )
= ...(iv)
As A is suspended vertically from B, p1V1γ = p2V2γ MR
hence horizontal motion will not effect 2( f − f2 )
By ideal gas equation, this can be a2 = Rα = 1 ...(v)
the vertical motion and hence A will M
written as
have same horizontal velocity as that γ −1 γ −1 Solving Eqs. (i) and (v), we get
of B. T1V1 = T2V2
8F0
At any height h considered, the length a1 =
300 × ( 3)3 /2 − 1 = T × ( 3 /2 )3 /2 − 1 3M + 8m
of the string will remain same and
T = 300 2 K 4F0
y+ x 2 + h2 = L = constant ⇒ a2 =
At higher temperature C p and C V 3M + 8m
Motion B and A are related to each varies with temperature. 3MF0 MF0
other, so it is easy to say that A will f1 = ⇒ f2 =
So, γ also changes with temperature. 3M + 8 m 3M + 8 m
have horizontal velocity and that will
be equal to velocity of B. For adiabatic chamber ∆Q = 0, so
∆U + ∆W = 0 4. (a,d)
According to the string constraint, Magnitude of induced electric field
total length of string must be constant. So, work done and internal energy are
inter convertable. due to change in magnetic flux is
y+ x 2 + h2 = L given by
3. (b,c) dφ dB
On differentiating w.r.t. t , we get
− dy For the no slipping, there will not be ∫ E ⋅ dl = dt = S ⋅ dt
=0  = vA 
dy x dx
+ any relative motion.
 dt 
or E ⋅ l = πR 2 (2 B0 t ) Q = 2 B0 t 
dt x + h
2 2 dt dB
So, v = ωR  dt 
x
vA = vB Also, τnet = Iα
x 2 + h2 Here, E = induced electric field due to
where, τnet is net torque, change in magnetic flux.
From figure, we can say that
I is moment of inertia and α is angular or E (2 πR ) = 2 πR 2B0 t

tan θ = sec 2 θ   =
x 1 dx
acceleration.
h  dt  h dt or E = B0Rt
The forces on different surfaces are
dθ vB v cos 2 θ shown below. Hence, F = QE = B0QRt
⇒ = = B
dt h sec θ
2
h m a1 This force is tangential to ring. Ring
F0
f1 starts rotating when torque of the force
2. (a,b,c,d) is greater than the torque due to
f1
C pmix maximum friction, τ F ≥ τ fmax
γ mix =
C Vmix M a2 So, τ F = τ f`max or F ⋅ R = (µmg ) R
n 1 C p1 + n2 C p 2 α or F = µmg or B0QRt = µmg
C pmix = f2
n 1 + n2 It is given that ring starts rotating after
Here, in above diagram, all the forces 2 s. So, putting τ = 2, we get
n 1C V1 + n2C V2
C Vmix = are shown. 2 B0RQ
n 1 + n2 µ =
where, mg
C p(He) =
5R
, C p (H ) =
7R f1 = friction between plank and
After two seconds, τ z ≥ τ f`max
2 2
2 cylinder,
C p`mix f2 = friction between cylinder and Therefore, net torque is
∴γ mix = ground, τ = τ F − τ f`max = B0QR 2t − µmgR
C V`mix
n1 C p1 + n2C p 2 a1 = acceleration of plank and 2 B0RQ
C p mix = a2 = acceleration of CM of cylinder. Substituting µ = , we get
n1 + n2 mg
As, there is no slipping anywhere
2 ×
5R
+2 ×
7R or τ = B0QR 2 ( t − 2 )
2 2 = 12 R = 3R a1 = 2 a2 ...(i)
= dω 
2 +2 4
F −f
a1 = 0 1 ...(ii) or I   = B0QR ( t − 2 )
2

m  dt 
n1C V1 + n2C V2
∴ C Vmix = f1 + f2 dω 
n1`+ n2 a2 = ..(iii) or mR 2   = B0QR ( t − 2 )
2
M  dt 
3R 5R τ
2 × +2 × ∴ τnet = Iα ⇒ α = net ω B0Q 4
= 2 2 = 2R I
or ∫0 dω =
m ∫2
( t − 2 )dt
2 +2 ( f1 − f2 )R ( f1 − f2 )R
⇒ α= = 2 B0Q
3R 3 1 or ω=
⇒ γ mix = = I MR 2 m
2R 2 2
430 JEE Advanced~Practice Set 8

When magnetic field is switched OFF Case II If initial aM > initial am, this is 24M π
I= × ×
only retarding torque is present due to possible, if µ 1 < µ 2. 5 πR 3 2
friction. So, angular retardation will be Case III If initial aM < initial am, (it will R
 4 2 R 2 x3 x5 
τf
α = max =
µmgR µg
=
be happen when µ 1 > µ 2 ), then R x − 3 + 5 
normal reaction will come into play.   R/ 2
I mR 2 R
Therefore, applying ω 2 = ω 20 − 2 αθ θ 24M π 53R 5 53
si n = × × I= MR 2
mg 5 πR 3
2 2 480 200
µg 
0 =  0  − 2 
2B Q
or θ 53 53mR 2
 m   R  N I= MR 2 =
fk2 200 40 x
2 B02Q 2R N
or θ= θ f k1 ⇒x=5
µm2g sin
g 8. (4.00)
2 B0RQ M θ T
Substituting µ = , we get
mg Fixed f
mg sin θ
BQ 7. (5.00) For the moment of inertia of the
θ= 0
m disc of mass dm at distance x about θ
5. (b, c, d) the given axis, we will use
1 We have to write the equation
n1 n n − n1 dI = dmy 2, dm = ρdV
Using + 2 = 2 2 mg sinθ − T − f = 0
u v R
where, y will be radius. where, f is frictional force.
We have for first surface,
FBD of the system is shown in figure.
1 1  n2 − n1 n1 
=  −  x2 + y2 = R2 A The equation of motion for the 14 kg
v1 n2  R u1  mass is as follows, where summations
1 . − 10
 15 . 1 −1 are parallel and perpendicular to the
=  − = R y
15
.  10 15  90 plane.
O x a
or v1 = − 90 cm x C B
So, it’s distance from centre dx 3/8
= 90 + 10 = 100 cm
B T
a RB
I1 O I2 Density of spherical segment is T 7g
M M M
ρ= = R = R 0.25 RA
V A
And for second surface,
∫ πy dx ∫ π(R − x )dx
2 2 2
RA
R = − 10 cm, R R/ 2 14 g
u = 90 cm + 20 cm = 110 cm 2 45°
M 24 M
n1 = 15
. and n2 = 10 ⇒ ρ= =
1
.
1  n2 − n1 n1 
5 πR 3 5 πR 3 ∑ Fparallel = 14g sin 45°
Hence, =  −  24 − 0.25 R A − T = 14a
v 2 n2  R u
Consider a small disc of radius y and 137.2 sin 45° − 0.25 R A − T = 14a …(i)
110
⇒ v2 =
4
= 27.5 cm thickness dx as shown in figure. ∑ Fperpendicular = R A − 14g cos 45° = 0
Mass of small disc, dm = ρ ⋅ πy dx 2 R A = 14g cos 45° = 97.11 …(ii)
So, distance of second image from The equation of motion for the 7 kg
Moment of inertia of disc about O,
centre = 27.5 + 1.0 = 28.5 cm mass is as follows:
X-axis,
6. (d) The normal reaction between 1
dI = dm ⋅ y 2dl ∑ Fparallel = T + 7 g sin 45°
two will be zero when initial 2 3
− RB = 7 a
acceleration of M is equal to or greater 1 ρπ 4 8
dI = ρπy 2dx × y 2 = y dx
than the initial acceleration of m. 2 2 3
θ T + 68.6 sin 45° − RB = 7 a …(iii)
Total moment of inertia of segment of 8
sin fk2
mg R 2 sphere is
R ∑ Fperpendicular = R B − 7 g cos 45° = 0
ρπ
R1 mg
Fixed I = ∫ dI =
2 R∫ y 4dx R B = 68.6 cos 45° = 48.5 …(iv)
θ f k1 Solving Eqs. (i), (ii), (iii) and (iv), we
sin
Mg θ
mg R
2
get
ρπ
I= ∫ (R − x 2 )2dx
2
FBD T =4N
2 R
2
9. (1.00) According to given condition,
⇒ Initial aM = g (sin θ − µ 1 cos θ) i ×4 AD 40 2
ρπ
R = = =
Initial am = g (sin θ − µ 2 cos θ) = ∫ (R − 2 R 2 x 2 + x 4 )dx
4
i × R 0 DC 60 3
Case I If µ 1 = µ 2 , then initial 2 R
4 2 R × 12
aM = initial am
2 ⇒ = R0 = 6 =6
R0 3 R + 12
So, N = 0.
JEE Advanced~Practice Set 8 431

⇒ R = 12 Shear stress, τ = µ
du So, x = 15 + 21 = 36 cm
Now, R = R 0 (1 + α ∆T ) dy x 36
Hence, = = 4 cm
9 9
12 = 6 (1 + α 100) ω
⇒ α = 1 × 10 −2 15. (d)
× × × × × × × × A ×
So, x=1 Liquid h
× × × × × × × ×
10. (5.00) Here, we have to use r R 2R v
the concept of flux as well as current. dr × × × × × × × ×
We will also use the relation, Assuming, the gap h to be small, so
that the velocity distribution may be × × × × × × × × B×
dV = –E ⋅ dr
assumed linear. 16. (a) Here, the loop has given a push
Electric field is along the direction of v ωr
τ =µ × =µ and left, it means that its speed will
maximum change in potential. The
h h keep on decreasing due to magnetic
equation used for electric flux is
Viscous force, dF = τ × Area force [I (l × B )]. But observed carefully
φ = ∫E ⋅d S that magnetic retarding force will also
= τ × 2 πrdr
where, dS is the surface area. decrease with time.
Torque dT on the element,
Consider the diagram shown below. If we just pushed the loop and left it,
dT = dF × r = τ 2 πr 2dr then due to magnetic force [− I l × B ],
µω r 2 πµω r 3dr the speed of the loop will start
or dT = × 2 πr 2dr =
h h decreasing.
x
Total torque, − vB 2 l 2
Here, F=
d /2 2 πµω r dr µπd ω
3 4
R
R T = ∫0 h
=
4h [where, R = resistance of the loop]
Sphere Thus, x = 4 dv
∝ −v
We have considered a surface of 13. (6.00) Case I If rolling body is a ring dt
f s = 0, there is no need of friction. dv
thickness dx at distance x. = − kv [where, k = constant]
Surface area of the considered F dt
surface = 4 πx 2 dv
∫ v = ∫ − kd t
Now, we know that, m
r ⇒ log v = − kt
dV
E= ⇒ dV = Edx v = e − kt
dx
ρdx ⇒ So, graph is
Resistance, R = Case II Other than ring f s ≠ 0
4 πx 2 v0
dV Edx E ⋅ 4 πx 2
Current, i = = =
R ρdx ρ m
r v
4 πx 2
φ 10
⇒ i = = = 5A
ρ 2 fs O t
11. (5.00) For Helium ion (He ), + ⇒ F + fs = m × a …(i)
17. (c) Radius of wire
−13.6 Z 2 ⇒ (F − fs ) r = Iα …(ii)
En = eV = MSR + VSR × LC
n2 For pure rolling, a = Rα …(iii)
= 0 + 50 × 
. 
01
By solving above three equations, we get  cm
So, E1 = 54.4 eV,  100 
3
E 2 = −13.6 eV, f = F = 50 × 10 −3 cm = 5 × 10 −4 m
7
E 3 = − 6.04 eV, Young’s modulus of wire,
Put F = 14 N
E 4 = −3.4 eV and MgL
3
f = × 14 = 6 N Y =
E 5 = − 2.2 eV. 7 π r 2l
Now, using ∆E =
hc 1240eV - nm
≈ 50 × 11.
14. (4.00) The condition is possible only =
λ λ(nm) π × ( 5 × 10 −4 )2 × (1.25 × 10 −3 )
when the foci of lens and mirror
For λ = 110 nm and 30 nm, we get coincides. Then, only the final image = 0.56 × 1011 Nm−2
∆E = 114
. eV and 40.8 eV. will be at 15 cm left to lens.
wL
These corresponds to transitions, 18. (c) As, Y =
l( πr 2 )
5 → 2 and 2 → 1. So, He + ion must F1 and F2
∆Y ∆L ∆l 2 ∆r
be in 5th state (or 4th excited state) ⇒ = + +
I Y L l r
initially.
01. 0.001 2 × 0.001
12. (4.00) Consider an element of disc at = + +
a radius r and having a width dr. 110 0125 . 0.05
Linear velocity at this radius = ωr. 15 cm = 0.0489
432 JEE Advanced~Practice Set 8

∆Y As we know that,
∴% error = × 100 CH CH2
Dehydrobromination
Y  p + a  ( V − b ) = RT
= 0.0489 × 100  
Br Br NaNH2
 V2
= 4.89 ≈ 5% 1, 2-dibromo-1-cyclohexyl (–2HBr)
∴ p( V − b ) = RT
ethane (A)
19. (b) Compound ( X ) LiAlH
 4 → Y (a
pV = RT + pb
hydride) + other compound. Hydride
C CH ( pV2 ) = RT + p2 b …(i)
Y contains 21.72% hydrogen.
∆ ( pV1 ) = RT + p1b …(ii)
Y + O 2  → B 2O 3 + H 2O Cyclohexyl ethyne
(B) On subtracting Eq.(ii) from (i), we have
Therefore, ‘Y’ is a hydride of boron
and it is obtained by reduction of ‘X’ ( pV2 ) − ( pV1 ) = b( p2 − p1 )
O
with LiAlH 4 . So, X is either BCl 3 or BF3. Substituting the given values,
(B) HgSO4/H2SO4
4BCl 3 + LiAlH 4 → B 2H 6 C CH3 (2.6 − 2.0)
⇒ b= = 0.2 mol −1dm3
(X ) (Y )
(5 − 2 )
Cyclohexylmethyl
+ 3AlCl 3 + 3LiCl ketone (C) VC = 3b = 0.6 dm3 mol −1
144244 3
Other products
Now, pC VC = 3RTC
Molar mass of B 2H 6 = 2 × 11 + 6 = 28 NaOD/D2O
O 3RTC 3 × 0.083 × 100
% of H in B 2H 6 ∴ pC = =
VC 0.6
6
= × 100 C CD3
28 = 415
. bar
= 21.42 ≈ 21.5 Cyclohexyl –1, 1, 1- trideutero 25. (6) SiO 44 − anion is present in
B 2H 6 + 3O 2 → B 2O 3 + 3H 2O + Heat methylketone (X) orthosilicate, single chain silicate,
(Y )
23. (b, c, d) double chain silicate and sheet like
20. (b, c) In the aqueous solution of silicate. Double chain structures are
NO2 NO2
Pd(NH 3 )2Cl 2, the atoms of chlorine are also known as amphiboles. Micas are
in coordination sphere and the van’t the example of sheet like silicates,
Hoff factor of the compound are unite. (b) Cl2 while pyrosilicate has Si 2O 76− and
AlCl3
21. (a, c, d) Si 6O12−
18 anions (Si - O ) is present in
Cl
(a) Thermal stability of sulphates and pyroxene, zeolite.
NO2 NO2
carbonates increases as one moves 26. (−1.27) From the combustion of
from Be to Ba. benzoic acid, the heat capacity of
NaOH (aq) HCl
BeSO 4 < MgSO 4 < CaSO 4 calorimeter and its content C can be
–HCl
< SrSO 4 < BaSO 4
–+ determined as:
ONa OH
3251
BeCO 3 <MgCO 3 <CaCO 3 NO2 NO2 × 0.825 × 10 3 J = 194
. ×C
122
<SrCO 3 <BaCO 3
∴ C = 11332.07 JK −1
Thus, given statement is correct. HNO3/H2SO4
∆ (c) Heat produced in combustion of
(b) 2Mg(NO 3 )2  → 2MgO
0727
. g D-ribose = 0.910 × 11332.07
+ 4NO 2 + O 2 NO2
∆ = 10312.18 J
3LiNO 3  → 2Li 2O + 4NO 2 + O 2 NO2 NO2
⇒ Internal energy of combustion of
Both give same products. Thus,
10312.18
statement (b) is incorrect. NH4HS HNO2 D-ribose = − × 150
(Selective H 2O 0727
.
(c) Ionic size of group 2 cations is
smaller than that of group 1. Thus,
reduction)
NH2 OH = −2.13 × 10 6 J mol −1
polarising power of M 2+ (Group 2) NO2 NO2 The combustion reaction is
< M + (Group 1). Hence, lattice C 5H10O 5( s ) + 5 O 2( g )
energy of group 2 salts is higher than
SO3 → 5 CO 2( g ) + 5 H 2O( l )
that of group 1 salts. (d)
H2SO4 Q ∆ng = 0, ∆H = ∆E
(d) Being smaller in ionic size of group 2
SO3H ⇒ −2.13 × 10 6 = − 5 ( 394 + 286) × 1000
than group 1, hence group 2 cations
NO2 NO2
are more hydrated than group 1 − ∆Hf° (D-ribose)
cations, hence hydration energies of
⇒ ∆Hf° (D-ribose) = − 1.27 × 10 6 J
group 2 are higher than group 1. NaOH(s) HCl
Thus, statement is correct. Fusion
–Na2SO3 –+ 27. (0.05) m mol of HCl dropped
ONa OH 1
22. (b, c, d) = + 15
. = 0.25
6
Br2/CCl4 24. (a) Since, intermolecular forces are
weak, ⇒ mmol of Al reacted
1
a = × mmol of HCl
Vinylcyclohexane Thus, =0 3
V2
JEE Advanced~Practice Set 8 433
0.25 ∴K p (1400 K ) Thus. heat released = −2.3542 × 5640
Mass of Al reacted = × 10 −3 × 27
3 −2 2 −2
(2.54 × 10 ) × (1.27 × 10 ) = −13277.69 kJ
= 2.25 × 10 −3g =
10 6 32. (64.93) Pressure =
Force
= hρ g
Volume of Al removed
= 8.2 × 10 −12 Area
2.25 × 10 −3 −4 h = height = 77.8mm = 0.0778m
= g = 83.3 × 10 cm 3
At 1000K; ∆G ° = − RT ln K
2.7 gcm−3 ρ = density = 1 × 10 3 k gm−3
= −8.314 × 1000 ln ( 3.44 × 10 −20 )
Also, V = πr × thickness
2
g = 9.81 ms −2
−4 = 372.60 kJ
8.33 × 10 cm 3
⇒ r2 = K (1400) ∆H  1400 − 1000  ∴ π (osmotic pressure)
. × 01
314 . cm Also, ln =   = 0.0778 m × 1 × 10 3 k gm−3 × 9.81 ms −2
K (1000) R  1400 × 1000 
= 2.65 × 10 −3cm2
= 56130
. kJ = 763.2 kg m−1s −2
∴ r = 0.051 cm
∆H °− ∆G ° = 763.2 N m−2
28. (0.048) TF = T+ + F − ∴∆S ° =
T n w
Initial C 0 0
. − 372.60) × 1000
( 56130 Also , π = RT = RT …(i)
= V M ⋅ w× V
At equi C(1− α ) Cα Cα 1000
Q pT (like pH) = 1.8 Substituting the given values in the
. J K −1
= 18870
∴[T + ] = 0.016 = Cα Eq.(i), we have
30. (9) Total surface area of six faces M⋅w
Now, ∆Tf = K f × (1 + α ) × molality
= 6 (1 × 1) = 6 cm2
∆Tf = K f × (1 + α ) × C 20 × 10 −3 kg × 8.314Jmol −1k −1 × 298K
=
[Q For dilute solution, If each side is divided into two equal 763.2 Nm−2 × 1 × 10 −33 m
1
molality = molarity] halves, then length = cm.
2 = 64.93 Kg mol −1
∆Tf = Tf° − Tf = 0 − ( −0.383) = 0.383
Cubes = 8 with six faces 33. (d)
⇒ 0.383 = 186
. (C + 0.016) 2
C CH
Surface area of one split = 8 × 6 
C = 0.21 − 0.016 = 0194
. M 1 NaNH2
2 NH3(l)
Also, 0.914 mole of TF contains
0.914 mole of T (including T + ) per litre Surface area due to n splits (A)
2n
Thus, moles of TF in 500mL
= ( 8 × 6) ×   = 2 n × 6
1 σ
0194
. × 500 C C
= = 0.097 2 CH3Br
1000
2 n × 6 = 768 cm2
Given, t 1/ 2 for T = 117
. years
N0 = 0.097 2 n = 128 = 2 n

Amount left in 23.4 yrs =


0.097
= 0.049 ∴ n= 9 C C CH3
2 KMnO4
31. (–13277.69) O 2 retained in the body H+
Since, one tritium atom emits one = 20 − 15 = 5%
β-particle. B
O 2 cosumed per day by volume
∴ Number of β-emitted particles 20 breath COOH
= 480 mL = × 24
= [0.097 − 0.049] × 6.023 × 10 23 min 5
Sodalime
× 60 min ×
= 2.89 × 10 22 100
= 691200 mL = 691.2 L C
Thus, total charge carried by
β-emitted particles in ∴ The number of moles of O 2 under Cl
2.89 × 10 22 × 1602
. × 10 −19 standard conditions will be
Faraday = Cl2 (Single
96500 pV hν isomer)
n=
= 0.048 F RT
29. (188.70) For the reaction, 1 × 691.2 L 34. (c)
= C C CH3
2CO 2( g ) = 2CO( g ) + O 2( g ) 0.0821 L atm mol −1K −1 × 298 k
Na/NH3(l)
∴At 1000K : %(O 2) = 2.05 × 10 −5 = 28.25 mol
∴ . × 10 −5
− 41
%(CO) ~ Thermochemical equation (B)
representing combustion of sucrose is
% (CO 2 )≈ 100 H CH3
C 12 H 22 O11 ( s ) + 12 O 2( g )
. × 10 −5 )2(2.05 × 10 −5 )
( 41 C C
⇒ K p (1000) = → 12CO 2( g ) + 11H 2O( s )
10 6 H
−20 ∆ C H° = −5640 kJ mol −1
= 3.44 × 10
At 1400K : %(O 2 ) = 1.27 × 10 −2 Number of moles of C 12 H 22 O11( s )
combusted by 28.25 moles of O 2 gas Br2, CCl4
−2
% (CO ) = 2.54 × 10 28.25
= = 2.3542 mol Racemic dibromide
% (CO 2 ) ≈ 100 12
434 JEE Advanced~Practice Set 8

35. (b) Me a 2k 2 + c 2k 2 = 2 b 2k 2
Me Me COOH ⇒ a 2 + c 2 = 2 b 2 ⇒ a 2, b 2, c 2 are in AP.
Sulphonation
Cl 39. (a,b) Area of shaded region
Cl2/Fe
PhMe + Y
(A )
SO3H
(B) 1
Cl (G)
2-methyl
Conc. H2SO4 (C)
Desulphonation 5-sulphonic acid
–SO3H, X
H3O+ COOH O 1
Carbonation

COOH 1
Me Me = ∫0 ( x − x 2 ) dx
[O]
Cl 1
2 x3 
Cl2/Fe =  x 3/ 2 −
 3 3  0
SO3H 2 1 1
(H) = − = sq unit …(i)
SO3H SO3H 3 3 3
4-sulphonic
phthalic acid ∴Total area of square
Me ( 0 ≤ x ≤ 1, 0 ≤ y ≤ 1) = 1 sq unit
 1  Now, A ∪ B = Entire region enclosed
x + 1 , 0≤ x< 1 
Cl2/Fe in 0 ≤ x ≤ 1, 0 ≤ y ≤ 1
 
 2  ⇒ A and B are exhaustive events.
37. (a,b,d) f( x) =  , 1≤ x < 2 
 x 
1

COOH Me  3 
Now, P( A ) = ∫0 x dx
, 2 ≤ x < 5 /2
(D)  x−1  2 3/ 2 1
Cl   = [ x ]0 = 2 / 3
Y 3
1
3 also P( B) = ∫0 x dy
1 2 3/ 2 1 2
COOH 2 = ∫0 y dy =
3
[ y ]0 =
3
(E)
∴P( A ) P( B) =    
1 2 2
3-chloro-4-methyl
benzoic acid 1/2  3  3
4
Me 1 2 5/2 3 X = ≠ P( A ∩ B)
9
From the graph, f( x ) is discontinuous 1
[Q from Eq. (i) P( A ∩ B) = ]
Nitration Br2/Fe and bijective function. It is also not 3
A differentiable.
∴ A and B are not independent events.
Hence, options a and b are correct.
z − 1− i
NO2 1 40. (a, b, d) Since arg   is the
lim f( x ) = , lim f( x ) = 2  z 
(I) x →1− 2 x → 1+
angle subtended by the chord joining
f(1) = 2 the points O and 1 + i at the
Me COOH
∴ min ( lim f( x ), lim f( x )) ≠ f(1) circumcentre of the circle| z − 1| = 1
x →1− x → 1+ z − 1− i π
so arg 
Br Br
 =−
[O] Maximum points of discontinuity = 2  z  4
38. (a,b) We have, P (z)
cos ( A − C ) cos B + cos 2 B = 0
NO2 NO2
cos( A − C )cos [180° − ( A + C )] + 1
(J) (K)
2-bromo- − 2 sin2 B = 0 X′ θ A X
4-nitrobenzoic O 1 2
acid ⇒ − cos ( A − C ) cos ( A + C ) + 1
− 2 sin2 B = 0
36. (c) ⇒ −(cos 2 A − sin2 C ) + 1 − 2 sin2 B = 0
Me
COOH ⇒ −(1 − sin2 A ) + sin2 C + 1 − 2 sin2 B = 0
z − 2 π
Carbonation ⇒ sin2 A + sin2 C = 2 sin2 B arg   =±
B  z  2
sin A sin B sinC z−2
We know, = = = k (say) ∴ is purely imaginary.
(F ) a b c z
∴ sin A = ak, sin B = bk, sin C = ck
JEE Advanced~Practice Set 8 435

π 1 ⇒ sin x = sin2 − 2 < −1 not possible


We have, ∠OPA = ⇒ x2 +
x2 π
∴ No solution exist in  0,  .
2
 z −2 z−2  2
∴ f( x ) + f   = f  x −
π AP 1 1
∴ arg   = ⇒ = i  +2
 z − 0 2 z OP  x  x So, number of solution = 0.
Now, in ∆OAP,  1
3

 log 6 3 
46. (1403) We have,
AP 43. (409) Let N =  ( 81)log 5 9
+ ( 3)  A2 = 5 A − 7 I
tanθ =
OP  
  ⇒ A 4 = ( 5 A − 7 I )2
z−2
∴ = i tanθ  2 
z  ( 7 )log 25 7 − (125)log 25 6  ⇒ A 4 = 25 A 2 − 70 A + 49I
 
41. (a, b, c) Let r = x $i + y$j   ⇒ A 4 = 25 ( 5 A − 7 I ) − 70 A + 49I
∴ r . (10 $j − 8 $i − r ) = 40 ⇒ N = (( 3 4 )log 9 5 + 3log 3 ( 6 ) )
3 ⇒ A 4 = 55 A − 126I
6
⇒ ( xi + yj ) . (10$j − 8$i − x $i − y$j = 40 (7 log 7 25 − 53log 5 2 ) ⇒ A6 = A4 ⋅ A2
⇒ x 2 + y 2 + 8 x − 10 y + 40 = 0, ⇒ N = ( 3log 3 25 + 3log 3 6 6 ) (25 − 6 6 ) ⇒ A 6 = ( 55 A − 126I ) ( 5 A − 7 I )
which is circle centre C ( − 4, 5). ⇒ N = (25 + 6 6 ) (25 − 6 6 ) ⇒ A 6 = 275 ( 5 A − 7 I ) − 385 A
Radius = 1 ⇒ N = 625 − 216 = 409 − 630 A + 882 I
λ = max {|r + 2 $i − 3 $j|2 } ⇒ A 6 = 1375 A − 1015 A − 1925I
44. (1) Given, the angle between u and $i
λ = max {( x + 2 )2 + ( y − 3)2} is 60° + 882 I
µ = min {|r + (2 $i − 3 $j )|2 } |u| ⇒ A 6 = 360 A − 1043I
∴ u ⋅ $i = |u|| $i| cos 60° =
µ = min {( x + 2 )2 + ( y − 3)2} 2
∴ a = 360, b = −1043
Let P be ( − 2, 3), then We are also given that
⇒ a − b = 360 + 1043 = 1403
CP = 2 , r = 1 |u − $i| is the geometric mean of|u| and 47. (308) Given 10 and 8 boys.
∴ λ = (2 2 + 1)2 |u − 2 $i|. Case I If Ravi is include
⇒ λ =2 2 + 1 ∴ |u − $i|2 = |u ||u − 2 $i|
7
C 5 × 9C 8 = 189
µ = (2 2 − 1) 2
Case II If Ravi is not include
Squaring both sides, we have
λ + µ = (2 2 + 1)2 + (2 2 − 1)2 = 18 7
C 6 × ( 8C 7 + 9C 8 ) = 119
[|u|2 + | $i|2 − 2u ⋅ $i ]2 = |u|2 [|u|2
Slope of AB =  
dy Total number of ways
= −2
 dx  2, 2 + 4 | $i|2 − 4 u ⋅ $i ]
= 189 + 119 = 308
2
Equation of AB, 2 x + y = 6  2|u | 
⇒ |u |2 +1 − 48. (0.6) We know that, SP ⋅ S ′ P ′ = b 2
 2 
OA = 2 $i + 2 $j , OB = 3$i 10 10
∴ ∑ (SPi )(S ′ Pi ′ ) = ∑ b
2
= |u|2 |u |2 +4 − 4
|u | 
AB = $i − 2 $j
 2  i =1 i =1

AB ⋅ OB = ( $i − 2 $j ) ( 3$i ) = 3 ⇒ 10b 2 = 2560 ⇒ b 2 = 256


⇒ |u| +1 + |u| + 2 |u| − 2 |u| − 2 |u|
4 2 2 3

42. (c, d) Given relation is Given a = 20


f( x + y) − f( x − y) = 4 xy = |u|4 + 4 |u|2 − 2 |u|3
∴ a 2 = 400
= ( x + y)2 − ( x − y)2 ⇒ |u|2 +2|u| − 1 = 0
b2 256 3
⇒ f( x + y) − ( x + y)2 = f( x − y) ⇒ e = 1− = 1− = = 0.6
−2 ± 2 2 a2 400 5
⇒ |u| =
− ( x − y)2 2 π
49. (6) Let tan = x⇒ x = 2 − 1
⇒ f( x + y) − ( x + y) = f( x − y)
2
⇒ |u| = 2 − 1 8
−( x − y)2 = k (say) ∴ ( 2 + 1)|u| = ( 2 + 1) ( 2 − 1) Q tan π = 2 = 1
= 2 − 1= 1  8 
⇒ f( x + y) = k + ( x + y)2
π ⇒ ( x + 1)2 = ( 2 )2
⇒ f( x ) = k + x 2 45. (0) If 0 < x < ,
2 ⇒ x + 2x + 1= 2
2
−π /2
f( 0) = 0 x
Now,
∫−2|cos x|dx = ∫−2 |cos x|dx
⇒ x2 − 1 = − 2 x
⇒ 0=k+ 0 x
⇒ k=0 +∫ |cos x|dx ⇒ ( x − 1)2 = 4 x 2
2
−π /2
∴ f( x ) = x 2 −π /2
⇒∫
x
⇒ x4 − 2 x2 + 1 = 4 x2
−2
cos xdx + ∫– π / 2 cos xdx
f( x ) + f   = x 2 + 2 ⇒ x4 − 6 x2 + 1 = 0
1 1
also x
 x x = ∫ |cos x|dx = 0
−2 ∴ a=6
−π /2
2
⇒ |− sin x| +|− sin x|x =0
f  x −  + 2 =  x −
1 1 π
and  +2 −2 − 50. (5) We have,
 x  x 2
f ′ ′ ( x ) − 5f ′ ( x ) + 6f ( x ) ≥ 0
1 ⇒ 1 − sin2 + sin x + 1 = 0
= x2 + 2 − 2 + 2 ⇒ 2 − sin2 + sin x = 0 ⇒ f ′ ′ ( x ) − 2 f ′ ( x ) − 3f ′ ( x ) + 6f ( x ) ≥ 0
x
436 JEE Advanced~Practice Set 8

⇒ f ′ ′ ( x ) − 2 f ′ ( x ) ≥ 3f ′ ( x ) − 6f ( x ) 4x + 3y − 6 = 0 and GM = abcd 4

Put f ′ ( x ) − 2 f( x ) = g ( x ) 2 tan θ 8
52. (b) We know that, tan 2θ = . Now, AM = = 2, GM = 4 16 = 2
⇒ f ′ ′( x) − 2 f ′( x) = g ′( x) 1 + tan2 θ 4
∴ g ′ ( x ) ≥ 3g ( x ) 4 AM and GM are equal, so the number
∠DPT = θ Slope of PT = −
⇒ g ( x ) ≥ Ae 3x 3 are equal.
⇒ g ( 0) ≥ A It means a = b = c = d = 2,
⇒ −2≥ A P(–3, 6) L=0 so, a+ b=4
[Q g ( 0) = f ′ ( 0) − 2 f( 0) = 0 − 2 = − 2 ] 54. (c) Given that a −5, a −4 , 3 a −3, 1, a 8 and
90° θ
∴ g ( x ) ≥ − 2e 3x a10 are real numbers where a > 0.
C . θ
⇒ f ′ ( x ) − 2 f ( x ) ≥ − 2e , ∀ x ≥ 0
3x
θ 2θ 90°–θ D Now, we know AM of numbers > GM
⇒ f( x )e −2x ≥ − 2e x , ∀ x ≥ 0 T of numbers.
⇒ f( x )e −2x + 2e x ≥ 0 AM of 8 numbers
a −5, a −4 , a −3, a −3, a −3, 1, a 8, a10
⇒ f( x )e −2x + 2e x ≥ 3
a −5 + a −4 + a −3 + a −3 + a −3 + 1
⇒ f( x ) ≥ 3e 2x − 2e 3x , ∀ x ≥ 0 + a 8 + a10
=
Comparing ah ( bx ) − bh( ax ) with 8
165
Let PT = l, tan2 θ = …(i) and GM
3e 2x − 2e 3x , we get 28 l
h( x ) = e x , a = 3, b = 2 11 130
cosθ = …(ii) = 8 a −5 ⋅ a −4 ⋅ a −3 ⋅ a −3 ⋅ a −3 ⋅ 1 ⋅ a 8 ⋅ a10
∴ ( a + b ) h( x ) = 5e x 13l
Dividing Eq. (i) by Eq. (ii) a −5 + a −4 + a −3 + a −3 + a −3
⇒ ( a + b ) h ( 0) = 5e 0 = 5 + 1 + a 8 + a10
tan2θ 15 ⋅ 13 So,
= 8
51. (b) Use the concept of tangent to the cos θ 28. 13 ⋅ 10
a −5 − 4 − 3 − 3 − 3 + 8 + 10
8
circle such as equation of the tangent
−56 10 ± 74 10 >
to the circle ⇒ sinθ =
60 13 a −5 + a −4 + 3 a −3 + 1 + a 8 + a10
x 2 + y 2 + 2 gx + 2 fy + c = 0 at the ⇒
(only positive value is possible) 8
point ( x1, y1 ) on it is
a −18 + 18
8
3 >
xx1 + yy1 + g ( x + x1 ) ⇒ tanθ =
−5 −4 −3
+ f( y + y1 ) + c = 0.
11 ⇒ a + a + 3a + 1 + a + a10
8

69 53. (b) It is given that 8


> 8 a0
14 ⋅ x ⋅ ( −3) + 14 ⋅ y ⋅ 6 + 108( x − 3) − a + b + c + d = 8 and abcd = 16
2 ⇒ 8 × 1= 8
( y + 6) + 432 = 0 We know that AM of
a+ b+c+d
x(108 − 42 ) + y 84 −
69  a, b, c, d =

 2  4
+ ( 432 − 531) = 0

Paper 2
1. (a, c, d) q 303.4 A0 A
Also, V( 3 µC ) = = = 1011
. V = λt 0 ⇒ λ = 0
Equivalent capacity is C1 3 2 2t 0
6×3 Number of radioactive nuclei after one
Ceq = = 2 µF = 2 × 10 −6 F 2. (a,c)
3+ 6 dN half-life,
A = ln
Time constant of the given dt N = N 0 e − λt
N = N 0 e − λt
arrangement is ln ( 2)
−λ N0
−6
CR = (2 × 10 ) ( 0.5 × 10 ) = 1 s 6 N = N0 e λ =
dN 2
∴ = − N 0 e − λt × λ
Final steady charge on arrangement, dt From equation y = mx + C ,
q 0 = Ceq V dN
= N 0 e − λt × λ C = ln| λN0| = A 0
= 2 × 10 −6 × 240 dt λN0 = e A0
= 480 × 10 −6 dN
C A = ln = ln| N0λ | − λt e A0 2t e A0 N
dt ⇒ N0 = = 0 QN= 0
Instantaneous charge at t = 1 s, λ A0 2
A = − λt + C
q = q 0 (1 − e − t /CR ) 1 2t 0 e A0 t e A0
A 0 = − λ( 0) + C N= × = 0
= 480  1 −  = 303.4 [Q e ≈ 2.7 ]
1 2 A0 A0
A0
 e = − λ (t 0 ) + C
2 t 0 e A0
N=
A0
JEE Advanced~Practice Set 8 437

m1 m2 2m × m 2m x′
3. (a, b, c, d) = =
Potential on surface of any of shell is m1 + m2 2 m + m 3
due to combined effect of charges on 2m
T = 2π
each of shell. So, we have, 3k
σ = ( v A )CMmax + ( v) CM
VA = ( ra − rb + rc ) (b) ( v A )w.r. t..max x
ε0 max

See carefully that velocity of A will


8.6 × 10 −9  3 
= 1 − + 2  be maximum when its velocity is
8.6 × 10 −12  2  maximum in frame of CM is So, the centre of the cylinder goes
3 maximum and in the direction of down by (w.r.t. the liquid surface)
= 10 × = 150
3
. × 10 3 V
2 CM velocity itself. 4 l
2 mv 0 2 v 0 ( x + x ′) = x ≤
σ  ra2  v CM = = 3 10
VB =  − rb + rc  3m 3
ε0  rb  3I
2v0 v ⇒ x≤ = 6 cm
v A/ CM = ← v B/ CM = 0 → 40
8.6 × 10 −9  1
− + 2 
3 3 3
=  8. (3.33) Velocity of block at A is
8.6 × 10 −12  3 / 2 2 
( v A )max w.r. t. G =
2 v 0 2 v 0 4v 0
+ = according to energy conservation,
7 3 3 3
= × 10 3 V = 116
. × 10 3 V mv 2 kx 2
1 2 =
6 (c) KE w.r. t. CM = µvrel 2 2
2
σ  ra2 rb2  1 × v A 1350 × (0.1) 2
2
VC =  − + c vrel = relative velocity of one w.r.t. =
ε0  c c  2 2
other
8.6 × 10 −9 11 1  2 m mv 02 ⇒ vA =
2 1350
= × = 1375
. × 10 3 V KE w.r. t. CM = ×  (v 0 ) =
2
8.6 × 10 −12 8 100
2  3  3
1350
For VA = VC , we have ⇒ vA = m/s
6. (b, d) 100
ra + rb = rc
A C During motion of a block from A to B,
⇒ rc = 1 + 15
. = 2.5 m B
we get
Tension in wire, T = 80 N
4. (b, d) v B2 = v A2 + 2 as
A Mass per unit length of AB,
[here, a = − µg = − 0.3 × 10
1 = − 3 m / s 2]
m1 = kgm−1
80 1350
Mass per unit length of BC, = − 2 × 3 ×2
60° 100
30° 1
m2 = kgm−1 1350
12.8 = − 12
100
Speed of wave in AB, 1350 − 1200 150
T = = m /s 2
v1 = = 80 ms −1 100 100
B C m1 Now, with this KE it will compress right
For incidence over face AB, Speed of wave in BC, spring.
sin i 1 = µ sin r1 ⇒ sin 60° = 3 sin r1 T mv B2 kx 2
v2 = = 32 ms −1 = f
1 m2 2 2
⇒ sin r1 = or r1 = 30°
2 1 150 1350
So, time to cross AB, × = × xf2
As, r1 + r2 = A AB 4.8 2 100 2
t1 = = s 150 1 1
⇒ r2 = A − r1 = 30° − 30° = 0° v1 80 ⇒ xf2 = × =
So, ray of light falls perpendicularly 1350 100 900
and time to cross BC, 1 100
over face AC. ⇒ xf = = = 3.33 cm
BC 2.56
When light ray emerges from AC, t2 = = s 30 30
v2 32
multiple reflections occurs between dδ
AC and surface of film. Intensity is Time to reach end C,Qt 1 + t 2 = 014
. s 9. (3.00) As, φ =

maximum when constructive Also, amplitude of transmitted wave, This can be written as
interference takes place. 2v2  2 × 32 
At = × A1 =   × 3.5 dδ dµ
⇒ ∆L = path difference = 2µt = nλ φ= × , δ = (µ − 1) A 0
v1 + v 2  32 + 80  dµ dλ
λ
⇒ t = (minimum thickness, n = 1) = 2.0 cm dδ
2µ = A0
6600 7. (6.00) Cylinder can perform SHM only dµ
⇒ t = = 1500 Å till it is partially submersed. When B dµ 2B
2 × 2.2 µ = A+ 2 ⇒ =− 3
cylinder goes down by x inside the λ dλ λ
5. (a,b,c) liquid level comes up by x ′ (say) 2 BA 0 1
( 4a − a ) x ′ = xa So, φ = − ⇒ φ ∝
µ λ3 λ3
(a) T = 2 π for two blocks spring
k x
⇒ x′ = So, N = 3
system, where µ reduce mass, 3
438 JEE Advanced~Practice Set 8

10. (0.06) 2 m ( 0) + m ( 0) = − 2 mu + mV Now, electric field due to plate


v = 2u σ
Current across ammeter is Ig = 0.5% …(i) = =2
0.5 2 E0
of I = I = 0.005 I Conservation of angular momentum
100 with respect to centre of rod, s = 4e 0
So, current across shunt resistance is Li = Lf ∴ n=4
I s = I − Ig = I − 0.005 I = 0.995 I 2 mL2 14. (6.00) Take the circular tube as a long
× ω + 0 = 0 + mvx
So, shunt resistance is 12 solenoid. The wires are closely
I G 0.005I × G G 2ωL2 wound. Magnetic field inside the
S = g = = = vx …(ii)
Is 0.995 I 199 12 solenoid is
Hence, resistance of ammeter is Velocity of separation of B = µ 0ni
point of contact Here, n = number of turns per unit
G e =
×G Velocity of approach of
GS 199 G length
= = = point of contact
G+S G+ G 200 ∴ ni = current per unit length
u + v
199 e = =1 In the given problem,
12 ωx
= = 0.06 Ω I
ni =
200 ⇒ v + u = ωx …(iii) L
11. (6.00) From the given conditions, we From Eqs. (i) and (iii), we get µ I
∴ B= 0
get 3u = ωx L
E n − E 2 = (10.2 + 17 ) eV ⇒ ω=
3u Flux passing through the circular coil
= 27.2eV …(i) x is
µ I
E n − E 3 = ( 4.25 + 5.95) eV Putting values in Eq. (ii), we get φ = BS =  0  ( πr 2 )
2ωL2  L 
= 10.2 eV …(ii) = vx
Subtracting Eq. (ii) from Eq. (i), 12 Induced emf,
we get, 2 × 3u dφ  µ πr 2  dI
× L2 = 2 ux e =− =− 0 ⋅
E 3 − E 2 = 17.0 eV 12 x dt  L  dt
 1 1 2 × 3l 2
2
Z (13.6) − = 17.0 ⇒ x2 = Induced current,
 4 9  24
e  µ πr 2  dI
2 2 i = =− 0 ⋅
Z 2(13.6)   = 17.0
5 2l l
= = R  LR  dt
 36  8 4
90 × 90 Magnetic moment = iA = iπr 2
Z 2 = 9 or Z = 3 ⇒ x =
2
= ( 45)2
4  µ π 2r 4  dI
From Eq. (i), we get or M =− 0 ⋅ …(i)
⇒ x = 45 cm  LR  dt
Z 2(13.6)  − 2  = 27.2
1 1
x 45
 4 n  ∴ = = 9 cm Given, I = I0 cos 300 t
5 5
dI
3 (13.6)  − 2  = 27.2
2 1 1 ∴ = − 300I0 sin 300 t
13. (4.00) The electric field produced by
 4 n  σ
dt
thin charged sheet is given by . Substituting in Eq. (i), we get
1 1 2 ε0
− = 0.222
4 n2 σ σ  300 π 2r 4 )
E= E= M =  µ 0I0 sin 300 t
1 2ε0 2ε0  LR 
⇒ 2 = 0.0278 ⇒ n2 = 36 ⇒ n = 6 +
n + 300 π 2r 4
+ ∴ N=
12. (9.00) LR
Just before collision +
+ Substituting the values, we get
2m = 600g m = 300g
+ 300 (22 / 7)2 (0.1)4
v ¹ wx N=
w x w (10) (0.005)
[where, σ = surface charge density]
= 5.926
∴ An external electric field is also
present, so we will use principle of −6
or N ~
superposition to find net electric field. 15. (a) For torque, we will use τ = M × B
There is no external force, so linear
From the figure, it is clear that the and we will use φ = Li
plate is placed in an external electric where, i is the current.
momentum will be conserved.
field. Let the electric field due to plate µ I
Before collision After collision
is E and E 0 be the external electric φ (t ) = Bs cos ωt =  0  ( πa 2 )cos ωt
 2b 
field.
v
E 0 + E = 12 V / m µ 0 π a 2I
…(i) = cos ωt
u 2b
E0 − E = 8 V / m …(ii)
dφ µ ωπa 2I sinωt
Solving Eqs. (i) and (ii), we get e = − = 0
dt 2b
E = 2 V/m
JEE Advanced~Practice Set 8 439

e µ 0ωπa 2I [Co(NH 3 )5NO 2 ] Cl 2 CO2OH


∴i = = sinωt
R 2 bR → [Co(NH 3 )5NO 2 ] + 2Cl + − O
H H H
µ I
τ = MBsinωt = ( πa i ) 0  sinωt
2 An aqueous solution on treatment with OH H
 2b  an excess of AgNO 3 gives two moles OH
µ 20ωπ 2a 4 I 2 of AgCl because we have two moles H OH
= 2
sin2 ωt of Cl sion in aqueous solution.
4b R
Induced emf in larger loop, Linkage isomers : [Co(NH 3 )5NO 2 ] Cl 2 O
and [Co(NH 3 )5(ONO)]Cl 2 HOH2C O
di
|e| = − N
dt The nitrite ligand is bound through
H
oxygen (  ONO ) in red form. The H CO2OH
 µ πa µ ω πa I 
2 2 2
= 0  0  cos ωt nitrite ligand is bound through
 2b   2b  nitrogen ( NO 2 ) in yellow form. OH H
µ 20 π 2a 4ω 2I 20. (b) The reaction involve in roasting CH2OH
= cos ωt
4b 2 process are as follow O
H H
2 ZnS + 3O 2 → 2 ZnO + 2SO 2 H
16. (a) Flux, φ = BA cos ωt Hydrolysis
OH H
− dφ 2PbS + 3O 2 → 2PbO + 2SO 2 HO OH
emf, e = = BAω sinωt
dt 2Cu2S + 3O 2 → 2Cu2O + 2SO 2 H OH
V α-D(–)-glucopyranose
rms current i rms = rms The SO 2 produced is utilised for
Z manufacturing of H 2SO 4 .
Energy, E = Vrms i rms cos φ t O
21. (c) The structure of compound(B) is HOH2C OH
φ = BA cos ωt OH
+
dφ H H CH2OH
e =− = BAω sinωt MeO CHO
dt
ω OH H
BA
Vrms 2 OMe β-D(–)-fructofuranose
i rms = =
Z 2
 ωL − 1  + R 2 1
  23. (a, d) Number of ( P ) atoms = 8 × = 1
 ωC  Me 8
. × 10 −3 × 63
01 C C Number of (Q ) atoms
= 2
H 1 1
2  63 × 01  + (10)2
1 H 2C = 2 × + 12 × = 4
. − 
 63 × 0.01 2 4
1
= 4.0 × 10 −4 A Me Number of ( R ) atoms= 4 × + 1 × 4 = 6
2
. × 10 −5 J
E = Vrmsi rms cos φ t = 812 (X)
Me Thus, the formula of compound
1 1 = PQ 4 R 6.
ω= = O3/Red
O+
LC . × ( 0.01)
01 H2C Now, as per the conditions given,
= 31.6 rad/s there can be two possibilities.
17. (a) Here, A → p, B → q, C → r, D → s Me (i) If the four-fold axis passes through
We use, at position θ, (A ) face centres where atoms (Q ) lies,
then the remaining number of Q
mv 2
T = mg cos θ +  1 
O OMe atoms = 3 Q 4 −  2 × 
r HC   2  
where, v 2 = x 2 − 2 gr(1 − cos θ)
(B) Thus, the formula of remaining
18. (b) Here, A—s, B—r, C—q, D—p compound = PQ 3R 6.
He, Ne are monoatomic. Mild (ii) If the four-fold axis passes through
MeO COOH
∴ f =3 Oxid face centres where atoms ( R ) lies,
H 2, O 2 are diatomic. then the remaining number of R
∴ f =5 –CH3I atoms
 1 
H 2O, SO 2 are triatomic non-linear. =5 Q 6 −  2 × 
  2  
∴ f =7 HO COOH
CO 2, BeCl 2 are triatomic linear. Thus, the formula of remaining
compound = PQ 4 R 5.
∴ f =6
22. (b) X is sucrose and it is a 24. (d) Total work done ( −W ) = nRT ln 3
19. (c) The complex [Co(NH 3 )5NO 2 ] Cl 2
non-reducing sugar. It has Let volume is V2 when half-work is
produces three ions in aqueous
α-glucosidic linkage. done.
solution.
440 JEE Advanced~Practice Set 8

⇒ −
W
=
nRT V
ln 3 = nRT ln 2 27. (0.33) Partial pressure of N 2 on = ( −077
. + 0226
. ) = −07926
. V
2 2 1 2 × 80 ∴ E ° 2+ = +0793
. V
surface = = 1.6 atm Hg 2 /Hg
V2 = 1732
. L 100
p
Also, V = πr 2h; when V = 1L; Partial pressure of N 2 inside sea 31. (3.01) Given, =K
v
10 × 80
1000 cm3 = = 8 atm For one mole of ideal gas, pV = RT
h1 = = 19.90 cm 100
. × 16 cm2
314 ∴ p2 = KRT ……(i)
Now, according to Henry’s law
V2 1732 h2
⇒ = = Solubility 8.4 × 10 −4  p2 
2
V1 1000 h1 KH = = ⇒   = 2
T Q p1 = 2 bar 
Partial pressure 16
.  p1  T1  p2 = 4 bars 
1732
∴ h2 = 19.9 × = 34.47cm = 5.25 × 10 −4
1000 ⇒ T2 = 4T1
Thus, distance travelled ∴Moles of N 2 dissolved in 8 L blood ⇒ ∆U = C V ∆T = 3C VT1 …(ii)
= ( h2 − h1 ) = ( 34.47 − 19.90) inside sea at 23°C
Also, −dW = pdV = kVdV
= 5.25 × 10 −4 × 8 × 8
= 14.57 ≈ 14.6 cm K
= 336 × 10 −4 mol ⇒ −W = ( V22 − V12 )
25. (4) Complex EAN ( Z − O.N+2 × C.N ) 2
Moles of N 2 dissolved in 8 L blood at
= ( p − p12 ) 2 Q = K ,given
K 2 1 p
K 3[Fe(CN)6 ] 26 − 3 + 12 = 35 surface
2 k  V 
[Ru(CO)5 ] 44 − 0 + 10 = 54 . × 10 −4 × 16
= 525 . ×8
(noble gas) 1 2 6
⇒ ( 4 − 2 2 ) = bar 2 …(iii)
[Cr(NH 3 )6 ]3 + 24 − 3 + 12 = 35 = 67.2 × 10 −4 mol 2K K
Thus, moles of N 2 which will escape Now, from Eq.(i)
[Co(NH 3 )6 ]3 + 27 − 3 + 12 = 36
out = ( 336 − 67.2 ) × 10 −4 mol p2 1 RT RT1
(noble gas)
−4
= RT ⇒ = 21 =
[Ni(NH 3 )6 ]2 + 28 − 2 + 12 = 38 = 268.8 × 10 mol K K p1 4 bar 2
Q pV = nRT 1
[Fe(CO)5 ] 26 − 0 + 10 = 36 Substituting in eq(iii) gives
(noble gas) 268.8 × 10 −4 × 0.0821 × 296 K
∴ V =
74 − 0 + 12 = 86 2 RT1 3
[W(CO)6 ] −W = 6 bar 2 × = RT1 …(iv)
(noble gas) = 0.33 L 4bar 2 2
(Note Here O.N. is oxidation number, 28. (4) The given compound X is twistane. On dividing Eq.(ii) and Eq.(iv), we get
C.N. is coordination number) It has four equivalent chiral centres. In ∆U C T −2C V
the bridged ring compound, certain = V 1 ×2 =
26. (94.67) Rate of the forward reaction W −3RT1 R
diastereomers cannot form due to
 dx  = (145
. × 1013mol −3 L3s −1 ) −2 × 12.5
  steric reason. So, exists only in two = = −3.01
 dt  f enantiomeric form. 8.314
[Fe 2+ ][dipy ]3 29. (0.1434) Moles of percipitate 32. (6.48) Mass of CaCO 3 = 0.065 g
0209
.
Rate of the backward reaction when = . × 10 −3
= 177 Moles of CaCO 3 =
0.065
= 6.5 × 10 −4
complex disappears 118 100
 dx  = ( x )[Fe(dipy)2 + ] . × 10 −3 × 81
⇒ Mass of KCNO = 177 Volume of CO 2 in air
  3
 dt  b = 01434
. g
nRT 6.5 × 10 −4 × 0.082 × 298
= =
At equilibrium,   =  
dx dx 30. (0.79) p 1
 dt  f  dt  b 2Hg + 2Fe 3 + → Hg 2+
2 + + 2Fe
2+
= 0.01588 L
. × 1013mol −3 L3s −1 )[Fe 2+ ][dipy ]2
∴(145 At equili. 10−3 × 5 10−3 × 95 10−3 × 95
excess
For pH : Volume fraction of
100 100 100
0.01588
= ( x )[Fe(dipy)2+
3 ] CO 2 = = 1588
. × 10 −3
At equilibrium Ecell = 0 10
. × 1013mol −3 L3s −1
145
⇒ ° 0.0591 [Hg 22 + ][Fe 2+ ]2 Solubility of CO 2 in water =
x ∴0 = Ecell − log
2+
2 [Fe 3+ ]2 3.35 × 10 −4 × 1588
. × 10 −3 = 5.32 × 10 −7
[Fe(dipy)3 ]
= = Keq ⇒ [E ° + E° ] The solubility of CO 2 in water will be
[Fe 2 + ][dipy ]3 Hg/ /Hg 2+ Fe 3+ /Fe 2+
initial concentration of H 2CO 3
. × 10
145 13
 10 −3 × 95   10 −3 × 95 
2
∴ x= s −1   
(carbonic acid) in lake water.
. × 1017
119 0.0591  2 × 100   100  H 2CO 3 = H+ + HCO −3
x = 1.22 × 10 −4 s −1 − log 2 C(1− α ) Cα Cα
2  10 −3 × 5 
Since, half-life for first order reaction is   Cα 2
 100  k a = 4.2 × 10 −7 =
t 1/ 2 =
0.693 1− α
K 0.0591 5.32 × 10 −7α 2
⇒ E° = −077
. + log ⇒
Hg/Hg 2+
0.693 2 2 1− α
∴ t 1/ 2 = = 5680.33 s
1.22 × 10 −4 ( 95)3 × 10 −5
⇒ 5.32α 2 + 4.2α − 4.2 = 0
= 94.67 mins 25 × 2
JEE Advanced~Practice Set 8 441

4.2 ± 17.64 + 89.38 O O O O


α =− = 0.57
10.64 NO2
H2SO4 HN Fuming HN HN HN
⇒ [H ] = Cα
+
heat HNO3
−7
= 5.32 × 10 × 0.57 O N Me O N COOH O N NH O N NH2
= 3 × 10 −7 H H H H
Here, H+ from carbonic acid is 6-methyluracil 5-nitrouracil
6-carboxylic acid O
negligibly small and hence, H2O, heat NO
contribution of H 2O in ionisation HNO2 HN NH4HS
becomes significant. O O
N NH2
⇒ [H+ ] = [HCO −3 ] + [OH − ] = 3 × 10 −7 OH NO2 O
H
HN Sn, HCl HN O
+10 −14 [H + ]
N N NH2
[H+ ]2 = 3 × 10 −7[H+ ] + 10 −14 O O HN Cl COOEt
H H NaOH
−17 −14 5-hydroxyuracil
∴[H ] − 3 × 10
+ 2
[H ] − 10
+
=0 N NH2
+ O
⇒ O H
O
3 × 10 −7 ± 9 × 10 −14 + 4 × 10 −14 HN
NH2 H
[H+ ] = N
2 HN CO2Et 180ºC
= 3.3 × 10 −7 O N
H N NH2
∴pH = − log[H+ ] = − log[3.3 × 10 −7 ] 5-amino uracil O
H
O
⇒ pH = 6.48 O O H
OH HN N
33. (b) P→ 4; Q→ 1; R → 2; S→ 5; T→ 3 HN
OH
O
Br2 HN
P. Aldehydes are more reactive than H 2O N
O N
ketones. So, with 1 mole of LAH/ether, O N N OH H
O H
H H
the aldehyde group will reduce first.
So, the answer is 4. O 35. (a) A → 12
, , B → 13
, , C → 167
, , , D→ 16
, ;
H E→ 4,5,67 ,
Q. NaBH 4 / MeOH (the number of N
Sn, HCl HN
moles not mentioned, so complete O Li (lithium) shows diagonal
H2SO4, N
reduction occur) would reduce both N relationship and has maximum
heat O
the aldehydes and ketones to H standard reduction electrode potential
H
alcohols. So, the answer is 1. Uric acid ( E° ).
R. Aldehydes are more reactive than Be (beryllium) shows diagonal
NH2 HOOC relationship and its cation is maximum
ketones, so ( CHO ) is protected first POCl3
(Q) OC + CH2 hydrated.
OH
with 1 mol of glycol and their NH2 HOOC Mg (magnesium) shows diagonal
OH O relationship, coordination number is
reduction of ketones with O
NOH NH2 six in the complexes, ions are involved
NaBH 4 / MeOH takes place, and HN
HN NH4H5 in the transmission of electrical
finally on hydrolysis the ( CHO )
impulses in human body.
group is obtained back. N O N O
O O Al (aluminium) shows diagonal
S. In Clemmenson reduction, H H
relationship, ions are involved in the
O H
( Zn -Hg ) / HCl converts the C O transmission of electrical impulses in
NCO human body.
group to the ( CH 2  ) group and KCNO, H2O HN
simultaneously the dehydration of NH2 Ca (calcium) cation has maximum
N ionic mobility in aqueous solution. Its
alcohol to alkene also takes place in O O
H fluoride and carbonates are least
acidic condition. So, the answer is 5.
O soluble in water. Coordination number
T. Wolff- kishner reduction (PhNHNH 2 + H is six in the complex. Ions are involved
HN N in the transmission of electrical
glycol + KOH) converts the C O 20% HCl heat
O impulses in human body.
group to the CH 2 group. So, the N
O N 36. (a) i → p, ii → s, iii → q, iv → t
answer is 3. H
H
34. (b) P → 1, 2, 3;Q → 4, 5 ; R → 6, 7 (i) CH 3COONH 4 is a salt of weak acid
NH2 EtO2C and weak base.
POCl3
NH2 EtO2C (iii) OC + CH2 pK a − pK b
(P) OC + CH2 pH = 7 +
NH2 NC 2
NH2 O C O
pK a(CH 3COOH ) = 474
.
Urea
Me C pK a(NH+4 )= 9.26
HN CH2 NaOH
EAA
OC CN pK b (NH 3 ) = 474
.
NH2 ∴ pH = 7
442 JEE Advanced~Practice Set 8

(ii) C 6 H 5 COOK is a salt of weak acid and   ⇒ a=


1
strong base. Thus, solution is basic.  3+ 7/n 
S n = tan  −1
 e 1 − e2
C 6H 5COO − + H 2 O  1 + 9 + 10 
 n n 2 Let S be the area of the ellipse. Then,
= C 6H 5 COOH + OH − π
  S = πab =
pK a log C  3+ 7/n  e2 1 − e2
pH = 7 + + S∞ = lim tan  −1

2 2 n→ ∞
 1 + 9 + 10  π
=
420
. log 01
.  n n 2
=7+ + e − e6
4
2 2 S ∞ = tan−1( 3)
The area is minimum if f(e ) = e 4 − e 6
= 7 + 2.10 − 0.5 = 8.60 1
tan(S ∞ ) = 3 or cot(S ∞ ) = is maximum when
(iii) C 6H 5NH+3 Cl − is a salt of weak base and 3
f ′ (e ) = 4e 3 − 6e 5 = 0
strong acid. Thus, solution is acidic. 1 + tan (S ∞ ) = 1 + ( 3)
2 2
⇒ e = 2/3
C 6H 5 NH+3 + H 2O = 1 + 9 = 10
So, S is the least when = e = 2 / 3
= C 6H 5NH 3OH + H+ 38. (a,c) We have, x 2 > 0 and lim
f( x )
=2 Therefore, the ellipse is
x2 x→ 0
pK aC (C 6H 5NH+3 ) = 4.62 2 x 2 + 6 y2 = 9
∴ f ( x ) must be a positive quantity.
pK b (C 6H 5NH 2 ) = 9.38 The equation of auxiliary equation is
Also, x 2 → 0 as x → 0 and limit is
∴pH = 7 −
pK b log C
− finite.∴ f ( x ) should also approach to x 2 + y2 = 9 / 2
2 2 zero as x → 0 Length of latusrectum of ellipse
9.38 log 01 .
=7− − = 2.81 lim[f ( x )] = 0 2 b2 2 × 3 / 2
2 2 x→ 0 = = = 2
a 3/ 2
lim 
f( x )
= lim  x ⋅ 2  = 0
(iv) NaCN is a salt of weak acid and f( x )
Now, Foci of ellipse ( ±
x→ 0+  x  x→ 0+  x  3, 0).
strong base. This solution is basic. 
CN − + H 2O = HCN + OH − 40. (c, d) We have,
lim 
f( x )
= lim  x ⋅ 2  = −1
f( x )
x→ 0− 
 x  x→ 0−  x  X ′ = − X, Y ′ = − Y , Z ′ = Z
pK a(HCN )= 9.21
(a)
∴ lim 
pK a log C f( x )
∴ pH = 7 + + does not exist. ( Y 3Z 4 − Z 4 Y 3 )′ − ( Y 3Z 4 )′ − ( Z 4 Y 3 )′
2 2  x 
x→ 0 

921
. 01
. = ( Z ′ )4 ( Y ′ )3 − ( Y ′ )3( Z ′ )4
=7+ + log = 1110
. x2 y2
39. (b) We have, + =1 … (i)
2 2 a2 b2 = Z 4 ( − Y )3 − ( − Y )3( Z )4
37. (c, d) We have, and ( x − 1)2 + y 2 = 1 … (ii) = Y 3Z 4 − Z 4 Y 3
S n = tan−1   + tan−1  
1 2 (symmetric matrices)
Solving both the equation, we have
2  9
x 2
1 − ( x − 1) 2 (b) ( X 44 + Y 44 )′ = ( X 44 )′ + ( Y 44 )′
+ =1
+ tan−1   + tan−1   + ...
1 2 = ( X ′ )44 + ( Y ′ )44 = ( − X )44 + ( − 4)44
a2 b2
 8  25 
2
⇒ (b − a )x + 2 a x − a b = 0
2 2 2 2 2 2
= X 44 + Y 44 (symmetric matrices)
 2 
Tn = tan−1   For least area, the circle must touch (c) ( X 4 Z 3 − Z 3 X 4 )′ = ( X 4 Z 3 )′
 n + 1 ellipse. − ( Z 3 X 4 )′
 2  Therefore, D = 0 = ( Z ′ )3( X ′ )4 − ( X ′ )4 ( Z ′ )3
Tn = tan−1  
 ( n + 1) 
2
Y = ( Z 3 ) ( − X )4 − ( − X )4 ( Z )3
 2  = Z 3X 4 − X 4Z 3
Tn = tan−1  2 
 n + 2 n + 1 = − ( X 4Z 3 − Z 3X 4 )
−1  r+2 −r 
n (skew symmetric)
Sn = ∑ tan  
 1 + r( r + 2 )
X′
O (1, 0)
X
(d) ( X 23 + Y 23 )′ = ( X 23 )′ + ( Y 23 )′
r =1
n = ( X ′ )23 + ( Y ′ )23
∑ (tan
−1
Sn = ( r + 2 ) − tan−1 r )
r =1 = ( − X )23 + ( − Y )23
Y′
S n = (tan−1 3 − tan−1 1) + (tan−1 4 = − ( X 23 + Y 23 )
4a + 4a b ( b − a ) = 0
4 2 2 2 2
− tan−1 2 ) + (tan−1 5 − tan−1 3) + ..... [skew symmetric]
−1 −1 ⇒ a 2 + b 2( b 2 − a 2 ) = 0  a3 a2 − 3 
(tan ( n + 2 ) − tan n) 41. (a, b, d) The points  , ,
⇒ a + b (− a e ) = 0 a −1 a −1 
2 2 2 2
S n = tan−1 ( n + 2 ) + tan−1( n + 1)
1  b3 b2 − 3  c 3 c 2 − 3
− tan−1 2 − tan−1 1 ⇒ b=  , ,  , 
e  b − 1 b − 1  c − 1 c − 1 
 3n 2 + 7 n  b2 1
S n = tan−1  2  Also, a2 = − lies on the line
 n + 9n + 10  1 − e2 e 2(1 − e 2 )
l x + my + n = 0
JEE Advanced~Practice Set 8 443
n
 1 
g ( x ) + 2 x 2g  
So, a, b, c are the root of the equation
⇒ ∑ f( kπ ) = − 2 πe − π
 u3   u 2 − 3   x 
l  + m  + n=0
k=1 (1 + 2e − π + 3e −2π ... ] ⇒ f( x ) = −  
 u − 1   u −1 n  3 x2 
∑ f(kπ ) = − 2 πe
−π
lim (1 − e − π )−2  
⇒ lu 3 + mu 2 + nu − ( 3m + n) = 0 n→ ∞
k=1
 g ( − x ) + 2 x 2g ( − 1 / x ) 
m − 2 πe π Now, f( − x ) = −  
∴ a+ b+c=− … (i) = 3 x2
 
l (e π − 1)2
n  g ( x ) + 2 x 2g (1/ x )
⇒ ab + bc + ca = … (ii) [Q(1 − n)−2 = 1 + 2 x + 3 x 2 + ...] = 
l  3 x2 
3m + n ∴ λ =2
⇒ abc = … (iii) ∴ f( x ) = − f( − x )
l 44. (30) p = z z + ( z − 3)( z − 3)
f( x ) is an odd function.
From Eqs. (i), (ii) and (iii), we get + ( z − 6i )( z + 6i ) But f( x )is given to be an even function.
abc − ( bc + ca + ab ) = 3 z z − 3 ( z + z) + 9 + 6 ( z − z) i + 36 ∴ f ( x ) = 0 ∀ x ⇒ f ( 5) = 0
+ 3 (a + b + c ) = 0 = 3 ( x 2 + y 2 ) − 3 (2 x ) + 9
42. (a,c) Given that u and v are roots of 47. (2) Given equation is
(let z = x + iy  sin2 x + (cos x − 1) sin x
equation  then z = x − iy 
+ 6 (2 i y) i + 36   − cos x − k sin x + k = 0.
x + px + q = 0
2
z+ z = 2x
 
So, u + v =−p  z − z = 2 iy)  Given equation can be rewritten as
⇒ uv = q sin2 x + sin x ⋅ cos x − k sin x − sin x
= 3 ( x 2 + y 2 ) − 6 x + 9 − 12 y + 36
1 − cos x + k = 0
(a) Now, equation whose roots are = 3 [ x 2 + y 2 − 2 x − 4 y + 15]
v ⇒ (sin x − 1) (sin x + cos x − k ) = 0
and
1 = 3 [( x − 1)2 + ( y − 2 )2 + 10] ⇒ sin x = 1 or sin x + cos x = k
u sin x = 1 has exactly one real root
For minimum value of p, x = 1, y = 2
x 2 −  +  x +
1 1 1
⇒ =0 Minimum value of p = 3 (10) = 30 on the interval ( 0, 2 π )
u v uv
∴sin x + cos x = k should have
u + v 1 45. (4) Let
⇒ x2 − x+ =0 exactly two roots in the interval ( 0, 2 π )
uv uv OA = a ,OB = b,OC = c,OD = d
⇒ − 2 < k< 2
⇒ uvx − (u + v ) x + 1 = 0
2 AB = b − a , BC = c − b, CD = d − c
The integral value of k is − 1, 0 and 1.
⇒ qx 2 + px + 1 + 0 AD = d − a ,CA = a − c,BD = d − b
π
Then, AB × CD + BC + CA × BD But k ≠ 1  since, k = 1 ⇒ x = 
(b) Now, equation whose roots are u 2  2
= (b − a ) × (d − c ) + (c − b ) × (d − a )
and v 2. ⇒ k = 0, − 1
+ (a − c ) × (d − b ) Hence, there are two possible integral
So, equation will be
= b × d − b× c − a × d + a × c values of k.
x 2 − (u 2 + v 2 ) x + u 2v 2 = 0
+ c × d − c × a − b× d + b× a 48. (23) a1, a2, a3 ... an is an odd number
So, equation will be
+ a ×d−a ×b−c×d+ c×b not divisible by a prime greater than 5.
x − ( p − 2q ) x + q = 0
2 2 2
= 2 (b × a ) + 2 (c × b ) + 2 (a × c ) So ai,( i = 1, 2, 3, ..., n) can be written
Q u 2 + v 2 = (u + v )2 − 2uv p2 − 2q Now, as ai = 3 a5 b , where a, b are
and u 2v 2 = q 2 = 0 AB × CD + BC × AD + CA × BD non-negative integer.
= 2 (b × a ) + 2 (c × b ) + 2 (a × c ) Thus, for all n ∈ N
43. (2) We have,
= 2|a × b + b × c + c × a| 1 1 1 1
f ′ ( x ) + f( x ) = 4 xe − x sin2 x + + + ... +
1 a1 a2 a3 an
⇒ f ′ ( x )e + f( x ) ⋅ e = 4 x sin2 x
x x = 4 ⋅ |a × b + b × c + c × a|
<  1 + + 2 ....  1 + + 2 ....
2 1 1 1 1
⇒ d e x f( x ) = 4 x sin2 x = 4 (area of ∆ABC)  3 3  5 5 
∴k=4 1 1 1 1
On integrating both sides w.r.t. x, we ⇒ + + + .... +
get a1 a2 a3 an
46. (0) x 2 f( x) − 2 f   = g ( x)
1
…(i)
e x f( x ) = (sin2 x − 2 x cos 2 x ) + C  x    
 1   1 
⇒ f( 0) = 0 1
Replacing x by , we have <   
⇒ C =0 x 1 − 1 1 − 1
 3  5
1  1
f   − 2 f( x ) = g  
∴ f( x ) = e − x (sin2 x − 2 x cos 2 x ) 1
15
x2  x   x =
Now, 8
f( kπ ) = e − kπ (sin2 kπ − 2 kπ cos 2 kπ ) Multiplying by 2 x 2 1 1 1 1
Hence, + + .... + is less
2 f   − 4 x 2f( x ) = 2 x 2g  
1 1
⇒ f( kπ ) = e − kπ ( 0 − 2 kπ ) = − 2 kπe − kπ …(ii) a1 a2 a3 an
 x  x
n n 15
∑ f(kπ ) = − 2 π ∑ ke
− kπ than
Adding Eqs. (i) and (ii), we get 8
k=1 k=1
−3 x 2f (×) = g ( x ) + 2 x 2g  
1 ⇒ m + n = 23
= − 2 π [e − π + 2e −2π + 3e −3π ... ]  x
444 JEE Advanced~Practice Set 8

49. (6) Equation of plane containing the (Q) OQ is the radius of circle. Q. The two ‘E’ are together but not two ‘
lines π A’s are 6C 2 × 5 ! = 15 × 120 = 1800
∴ OQ = r = |OA| sin
x−1 y−2 z−3 3
= = R. Neither two A’s nor two ‘E’s are
2 3 4 3 together 7560 − 1800 = 5760
=2 × = 3 and r 2 = ( 3 )2 = 3
x−2 y−3 z−4 2
and = = is S. No two vowels are together
3 4 5 (R) Area of region form by the point P( z)
4!
inside the incircle of hexagon 4 ! × 5C 4 × = 24 × 5 × 6 = 720
x−1 y−2 z−3 π 5π 2 !21
satisfying ≤ arg( z) ≤
2 3 4 =0 3 6 54. (a) We have, ABCD is a square whose
3 4 5 π 3 each side is 1 unit and E, F, G, H is
= × ( 3) = π
2
4 4 distinct points in a square such that
⇒ ( x − 1)(15 − 16) − ( y − 2 )(10 − 12 )
∠EDC = ∠ECD = ∠HDA = ∠HAD =
+ ( z − 3) ( 8 − 9) = 0
∠GAB = ∠GBA = ∠FBC
⇒ − ( x − 1) + 2 ( y − 2 ) − ( z − 3) = 0 = ∠FCB = 15°
|d| √3 √3 D C
⇒ x − 2 y + z = 0 or = 6
6 15º 15º
90º
⇒ |d| = 6

15º
E

º
15
50. (2) Given y = [|sin x| + |cos x|] ∴ m+ n= 3+ 4=7
1 ≤|sin x| + |cos x| ≤ 2 (S) z42 − z12 − z22 − z32 − z52 − z62 H F
2π 4π
∴ [|sin x| + |cos x|] = 1 ⇒ y = 1 … (i) i i
z1 = − 2, z2 = − 2e 6 , z3 = − 2e 6 ,

15º
⇒ x 2 + y2 = 5

15º
… (ii) G
6π 8π
i i
Solving Eqs. (i) and (ii), we get z4 = −2e 6 , z5 = − 2e 6 , 15º 15º
(2, 1) and (− 2, 1) 10π
i A M B
Slope of tangent at (2, 1) z6 = −2e 6

P. FH = 1 − 2  tan15°  =
1
3 −1
 x ⇒ z42 − z12 − z22 − z32 − z52 − z62 2 
= −  = −2
 y  at ( 2, 1) 8π
EG = 1 − 2  tan15°  =
i 1
= 4 (e i2π − 1 − e i 2π / 3 − e i 4 π / 3 − e 3 ⇒ 3 −1
and slope of tangent at 2 
10π
 x −e
i
Area of quadrilateral EFGH
( − 2, 1) =  −  =2 3
 y  ( − 2, 1) 1
= 4(2 ) = 8 = FH × EG
∴ m=±2 2
Hence, | m| = 2 52. (b) Given f( x) = ax 3 + bx 2 + cx + d ⇒
1
( 3 − 1)2 = 2 − 3
2
51. (a) Given ABCDEF is a regular Let g ( x ) = ax 3 + bx 2 + cx + d
∴ a+ b=2 + 3= 5
hexagon f( x ) = g ( x )
Q. EM = EG + GM
A( z1 ), B( z2 ), C ( z3 ), D( z4 ), E( z5 ), F( z6 ) f( x ) has local maxima and local
1 3
is anticlockwise direction and minima at x = 2 and x = − 2 = 3 − 1+ tan15° =
respectively. 2 2
A( z1 ) = − 2 and C ( z3 ) = 1 − 3i
⇒ g ( x ) has some local maxima and In ∆ AEM
E (z5) D (z4)
local minima at x = 2 and x = − 2 tan ∠EAM =
EM
=
3 /2
= 3
respectively AM 1/ 2
O ⇒ a < 0, a = − 2 ∴ ∠EAM = 60°
F (z6) C (z3)
3ax 2 + 2 bx + c ∆AEB is equilateral triangle,
f ′( x) = =0
r
2 ax 3 + bx 2 + cx + d π
∴ ∠AEB = , k = 3
3
A (z1) B (z2) f ′ ( − 2 ) = 0 and f ′ (2 ) = 0
AD 1
⇒ b = 0, c = 24 R. 2 R = = = 2 ⇒R = 1
(P) Let O be the centre of regular sin150° 1 / 2
Also, g(2 ) > 0 and g( − 2 ) > 0
hexagon 1/ 2 3
⇒ − 16 + 48 + d > 0 S. h1 = a sin 60° = ×
∴ A( z1 ) = − 2 cos 15° 2
2π and 16 − 48 + d > 0
i 1 3  =
3
B( z2 ) = z1 e 6 = −2  + i d > − 32 and d > 32 ⇒ d > 32
2 2  2 ( 3 + 1)
53. (d) Given word BREAKAGE
= − (1 + 3i ) 1 1 1  2( 4 + 2 3 ) 
P. The two ‘A’ are not together is ⇒ + + =3 
∴ z2 = − 1 − h12 h22 h32  3 
3i 61 21 × 720
7
C2 × =
Here, a = − 1, b = − 3 21 2 = 8+ 4 3= 8+ 48
2 a 2 + b 2 = 2( − 1)2 + ( − 3 )2 = 7560 b 48
∴ = =6
=2 + 3= 5 a 8
PRACTICE SET - 9
Paper 1
1. (b,d) So, mass of daughter nucleus is magnitude of Q1 is more, the two
Values of options (b) and (d) do not = mPo − mα − mQ fields may cancel each other,
match with the mirror formula, = 209.9829 − 4.0026 − 0.0058 because distance of that point from Q1
1 1 1 will also be more.
+ = = 205.9745 u
v u f ∴The correct options are (a) and (d).
4. (a, b)
2. (c,d) 6. (a,b,c,d)
This question can be solved, if right
P1 hand side chamber is assumed open, At low temperature (60 K), the
so that its pressure remains constant hydrogen molecule has only
∆x= d sinθ translatory motion, therefore it has
even, if the piston shifts towards right.
only 3 degrees of freedom.
d sinθ θ p0 So, for 1 mole of gas,
3 3
S1 S2 P2 p 1= p 0 U = NA k B T = RT
2 2
d 3
⇒ C V = R at 60 K
x 2
At temperatures above 60 K, i.e.
p 0 = p1 between 250 K to 750 K; molecule of
hydrogen began to rotate and
λ = 600 nm p
behaves like a dumb-bell. It has now
At P1, ∆x = 0 kx
5 degrees of freedom.
At P2, ∆x = 1. 8 mm = nλ 5 5
So, U = NA k B T = RT
Number of maximas will be pV = nRT 2 2
∆x 1. 8 mm 5
n= = ⇒ p∝
T or C V = R, at 250 to 750 K
λ 600 nm V 2
= 3000 Temperature is made three times and At temperature around 4000 K, the
volume is doubled, molecular collisions are so vigorous
At P2, ∆x = 3000λ that the atoms begins to vibrate and
3
Hence, bright fringe will be formed. p2 = p1 the molecules now have 7 degrees of
2 freedom.
At P2, 3000 th maxima is formed.
Further, 7
For option (a), ∴ C V = R at 4000 K
∆V V − V1 2 V1 − V1 V1 2
∆x = d sinθ x= = 2 = =
A A A A
d∆x = d cosθd θ 7. (a) As, shown in the figure
3 p1 kx
Rλ = d cos θRdθ p2 = = p1 + For 0≤ x≤ a
2 A
Rλ φ = Bx 2
⇒ Rdθ = ⇒ kx =
p1 A
d cos θ 2 ∴Induced emf,
As, we move from P1 to P2. dφ dx
Energy of spring, ε=− = − 2 Bx ⋅
θ ↑ cos θ ↓ Rdθ ↑ 1 2 p1V1 dt dt
kx =
3. (a,c,d) 2 4 or ε = − 2 Bx ⋅ v
210
84 Po → 42 He + 206
82 X Also, ∆U = n C V ∆T At x = 0, ε = 0
206 pV 3 At x = a, ε = − 2 Bav
So, daughter nucleus is 82 Pb. = 1 1 × R × ( 3T1 − T1 )
RT1 2 At x = 2 a, ε = 0 as φ = constant
Disintegration energy is
= 3 p1V1 (i.e. when the loop reaches completely
A
Q= × KE α inside the field)
A−4 5. (b,c)
Now, when the loop starts coming out
 210  From the behaviour of electric lines, from the field after sometime, then
=  × 5.30 we can say that Q1 is positive and Q 2
 210 − 4  φ = B (2 a − x12 )
is negative. Further,|Q1| > |Q 2|.
= 5.40 MeV At some finite distance to the right of ∴ ε = − 2 B (2 a − x ) ( − v )
Mass equivalence of disintegration Q 2, electric field will be zero. Because or ε = 2 B (2 a − x ) ⋅ v
energy is electric field due to Q1 is towards right At x = 0, ε = 4Bav
5.40 (away from Q1 ) and due to Q 2 is x = 2 a, ε = 0
MQ = = 0.0058 u At
931 towards left (towards Q 2). But, since Hence, (a) is the correct option.
446 JEE Advanced~Practice Set 9

x dB 0
8. (2) Q E = 10. (8)  η   x2 
2 dt Neutron Deuteron K1 K2 = λ g   
 1 − η   2 (1 − η ) ⋅ l
3Kxt 2
E= m M m M η ⋅ mgl
2 = − (1 − η )
2
3Kxt 2
2 πxdx K
dτ = × q⋅x  1  1 0.80 × 9.8 × 15
.
2 πr 2 Momentum conservation gives = − 1 −  × ×
r
 3 3 2
3Kt 2q 2 mK = 2 mK1 + 2 MK 2 …(i)
⇒ τ = ∫ x dx
3
= − 13
. J
r2 0 Also, as collision is elastic, W
∴ =1
3Kq ⋅ t 2 2 K = K1 + K 2 …(ii) 1.3
τ = ⋅r … (i)
4 Eqs. (i) and (ii) gives, 12. (N = 8)
Torque due to friction force, 2 mK = 2 m( K − K 2 ) + 2 MK 2
dτ = µdmgx ⇒ ( 2 mK − 2 MK 2 )2 v2
r
qm = ( 2 m( K − K 2 ) 2
τ = 2µg ∫ x dx
2
r2 0 ⇒ K 2( m + M ) = 2 mMKK 2 A
R B
R 3R
2 4mM
= µmgr …(ii) ⇒ K2 = ⋅K
3 ( M + m)2
From Eqs. (i) and (ii), we get So, fraction of energy lost, v1
3Kq ⋅ t 2r 2 2 K 4mM
= µmgr η= 2 =
4 3 Using, angular momentum
K ( m + M )2
conservation at A and B, we get
8 µmg 4 × 1× 2
t = So, η = =
8
mv1(2 R ) = mv 2( 4R )
9Kqr (1 + 2 )2 9
⇒ v1 = 2 v 2 …(i)
=2s 9η = 8 Using conservation of energy, we get
9. (3) As the cart is drawn by a force F 11. (1) 1 GMm 1 GMm
the water in the vessel takes up a
R mv12 − = mv 22 − …(ii)
2 2R 2 4R
slant position rising upward at the x
back wall of the vessel. To prevent From Eqs. (i) and (ii), we have
water flowing out of the hole H, the fr GM
v1 =
acceleration of the vessel should have 6R
such a value that it occupies a face 2GM
λxg and v2 =
area DBH and a width of vessel given 3R
A
by . Now, if r = radius of curvature at B,
L
1 then
Area of ∆DBH = bc mv12 GMm
2 =
D λ(l–x). g r ( 4R )2
L From the initial condition of the 16v 22R 2
problem, the limiting friction between ⇒ r=
mg GM
b the chain and table is equal to the
8R
weight of the overhanging part of the =
mg 3
chain, i.e.
θ (by substituting for v 2)
H F ληlg = kλ(1 − η) lg
B
where, λ is the linear mass density of 13. (d) When u A = 6gl
the chain. 1
1 A Total energy = mu A2 = 3 gl
Volume of liquid retained = bc × η 2
∴ k=
2 L 1− η and tension at point,
bcAρ
Mass of cart and water = M + Let x is the length of chain on the mu A2
2L C = − 5mg
table at some moment. l
ma
tanθ = ∴ fr = k ⋅ R = kλxg = 6mg − 5mg
mg
b
Work done by the frictional force is = mg = weight of body
a = g tanθ = g × dW = fr ⋅ ds cos180°
c 14. (c) When u A > 2 gl and u A < 5gl .
bcAρ  gb = − fr ⋅ ds
Required force =  M +
Let’s say it is 3gl, then total energy is
 or dW = − kλxg ( − dx )
 2L  c E = K E + PE
Here, ds = − dx, because length of = KE at A
= [1 + 0.5] × 50
chain decreases with time. 1 3
= 15
. × 50 = 75 N 0 = mu A2 = gl

F
=3 ∴ W = ∫ λgkx dx 2 2
25 (1 − η ) l
JEE Advanced~Practice Set 9 447

Also, as (u A )min = 5gl for looping the Any of the ions, I− or Ag + which is Now, K p = KC ⋅ RT ∆n
loop, hence particle leaves circular present in excess decides the charge
path when its tension is zero on ion. Kp
(somewhere between B and C). KC =
(1) If I − is in excess then charge on RT ∆n
15. (b) When u A is less than 2 gl particle solution will be negative.
0.1687
=
does not reaches upto B. (2) If Ag + is in excess then charge on (0.0821 × 973) 2
So, motion of particle is simple solution will be positive.
harmonic and it oscillates about A. = 0.2645 × 10−4
(a) 100 mL of 0.1M AgNO 3
Also, tension is non-zero everywhere. 24. (a,c,d) The correct sequence is
= 10 millimol Ag +
16. (a) From FBD, we have − Step I
100 mL of 0.01 M KI = 10 millimol I
NaOH, air
T1 = T2 and T1 = m1a AgI is only precipitated and solution is FeCr2O 4 → Na 2 CrO 4
Also, m2a = m2g − T2 uncharged. ∆
C, ∆
17. (b) Equations are (b) 20 millimoles of Ag + > 10 Step II Na 2 CrO 4 → Cr2O 3
m1g sinθ − T1 = m1a millimoles of I− [AgI] ⋅ Ag + is
Al, ∆
T2 = m2a colloidal sol is formed. Step III Cr2O 3 → Cr

Also, T1 = T2 (c) 20 millimoles of I > 10 millimoles
25. (b, d) Concentration of NaOH
18. (c) Equations are of Ag + . [AgI] I− is formed. 4
= mol / L = 0.1 M
T1 = m1a (d) 25 millimoles I− > 15 millimoles 40
T2 = m2g − m2
a Ag + . [AgI] I− is formed. Q 0.1 mole of NaOH is added to 1 L of
2 the buffer solution containing 1 mol
22. (a,b,c,d)
Also, T1 = T2 each of CH3 COOH and CH3 COONa;
19. (d) Problem is based on the concept 0.1 mole of NaOH will neutralise
→ Gives free radical
of π-bonding tendency and 0.1 mole of CH3 COOH, reducing the
substitution
polymerising tendency. concentration of acetic acid from 1 M to
Polymerising tendency 0.9 M and increasing the concentration
→ Gives SE reaction
1 of sodium acetate from 1 M to 1.1 M.

π - bonding tendency i.e. [CH3 COOH] = 1 − 0.1 = 0.9 M
Cl [CH3 COONa] = 1 + 0.1 = 1.1 M
As π-bonding tendency increases,
[Salt]
polymerising tendency decreases. ∴ pH = pK a + log
The correct order of polymerising → Gives SE reaction [Acid]
1.1
tendency can be represented by = − log (0.18 × 10−4 ) + log = 4.83
0.9
option (d). Cl 2Zr

Si O > P O > S O > Cl O
26. (13.23) f( r ) = 4 πr 2ψ r 2 = Kr 2 e na0
←
Gives substitution −2Zr
Internuclear distance decreases
reaction For 1s-orbital, n = 1 f( r ) = K r 2e a0
π-bonding ability increases
df( r )
Polymerising tendency decreases NO2 For maximum, =0
dr
20. (b) As we know at high pressure. 2Zr 2Zr
− −
23. (a, b, c) Use K p = KC RT ∆n to solve  −2Z 
Z = 1+
pb
(at high pressure) ⇒e a0
×2r + r2 ×e a0
×  =0
RT this problem.  a0 
b π 2−
2 rZ
=0
Slope = = Se6 ( g ) r 3 Se2 ( g )
RT 492.6 a0
Initially 1 0
π a0 52.9
b= × 0.0821 × 300 …(i) At equilibrium [1 − x ] 3x r= = = 13.23 pm
492.6 Z 4
4 Total pressure at equilibrium
b = πr 3 × 4 N av …(ii) 27. (1) This problem is based
3 = 1 − x + 3x = 1 + 2 x
on Faraday law of electrolysis
Equating (i) and (ii) 3x
Pressure of Se2 ( pSe 2 ) = × p and pH of electrolysed solution
4 3 1+2x
πr × 4Nav using faraday law of electrolysis.
3
1− x 0.965 × t 0.05 × 100
π Pressure of Se6 ( pSe 6 ) = × p =
= × 0.0821 × 300 × 10 − 3 1+2x
Q
2 × 96500 1000
492.6
3
⇒ r = 2.5 Å ⇒ d = 5 Å  3x  ⇒ t = 1000 s
pSe 3  1 + 2 x × p 8
21. (c,d) Here, AgI is precipitated by Kp = 2
=  ( nH + )excess = ( nOH − )discharged = ne − =
F
reaction of equivalent amount of pSe 6 pSe 6
i ×t 0.965 × 1000
AgNO 3 and KI. = = = 10 − 2
= 0.1687 F 96500
448 JEE Advanced~Practice Set 9

MV M × 100 [Q given, [S ] = 2 ⇒ 2 ≤ S < 3]


nH + = ; 10 − 2 = 33. (c) Claisen condensation takes place
1000 1000 in the presence of a base. α +2 α +2
⇒ ≥ 2 or <3
⇒ M = 0.1 ⇒ [H+ ] = 0.1 ⇒ pH = 1 CH 3 C OC 2H 5 + CH 3 C OC 2H 5 2 2
  α +2 α +2
28. (6) Fe (OH)3 + HCl → FeCl 3 O O ⇒ ≥ 2 or ≤ −2
K 4 ⋅[ Fe(CN )6 ] NaOC 2H 5 2 2
→ Blue ppt. → CH 3 C CH 2 C OC 2H 5 + C 2H 5OH
⇒ α ≥ 2 or α ≤ − 6 ...(ii)
 
Excess K 4 [ Fe(CN )6 ]
O O α +2
→ Fe 4 [Fe (CN )6 ]3 (β -keto ester) < 3 ⇒ |α + 2|< 6
2
Prussian blue
This reaction involves the formation of ⇒− 6< α + 2 < 6
CN
NC CN carbanion.
⇒ − 8< α < 4 ...(iii)
34. (d) CsCl has bcc structure, so it’s From Eqs. (ii) and (iii)
Fe
coordination number is 8 and radius
− 8 < α ≤ − 6 or 2 ≤ α < 4
NC CN ratio is 0.732-1.000.
CN ⇒ α = − 7, − 6, 2, 3
35. (a) As octahedral geometry has a
coordination number of Fe in complex bond angle of 90° with radius ratio as Possible coordinates of A are
is 6. 0.414-0.732 and coordination number ( − 7, − 11), ( − 6, − 9) ,(2, 7 ), ( 3, 9)
29. (6) Number of tetrahedral voids is 6. 39. (b, c, d) We have, f( x) = 2 x − x 2
= 2 × Number of constituent units 36. (b) Tetrahedral geometry has and g ( x ) = x n
3
present in the packing sp hybridisation which has
Y
=2 × 3= 6 0.225−0.414 radius ratio and
30. (3) Substrate undergoing Hofmann coordination number is 4. g(x)=x1
bromamide reaction must contain 37. (a, b, d) We have, (1, 1)
O
f( x ) = max{1 + sin x, 1, 1 − cos x}
 f(x)=2x–x2
R  C NH2 group where f( 0) = max{1 + sin 0, 1, 1 − cos 0}
= max{1, 1, 0} = 1
R = alkyl/group X
O g ( x ) = max{1,| x − 1|}
 g( 0) = max{1, 1} = 1
Br2 + KOH Now, 2 x − x2 = xn
Ph  CH2  C  NH2 → g ( f( 0)) = max{1,| f( 0) − 1|}
PhCH2NH2 = max{1, 0} = 1 [Q f( 0) = 1] ⇒ xn − 2 x + x2 = 0
Br2 + KOH
f( g ( 0)) = max{1 + sin1, 11
, − cos 1|} ⇒ x ( x n − 1 − 2 + x) = 0
CH3 CONH2 → CH3NH2
Br2 + KOH = 1 + sin1 ⇒ x = 0 and x = 1
CH3 CH2 CONH2 →
f(1) = max{1 + sin 1, 1, 1 − cos 1} Let A be the area of between f( x ) and
CH3 CH2NH2 1
= 1 + sin 1
31. (a) Fries rearrangement takes place in
g ( x ), then A = ∫0(2 x − x 2 − x n ) dx
f( f(1)) = max{1 + sin(1 + sin 1), 1
presence of AlCl 3 catalyst.
1 − cos(1 + sin 1)} = 1 + sin(1 + sin 1)}
 x3 xn + 1 
=  x2 − −
O  3 n + 1 0
g ( f(1)) = max{1,| f(1) − 1|}
O  C CH3 = max{1,| 1 + sin 1 − 1|} = 1 1 1 2 1
OH O = 1− − = −
3 n+1 3 n+1
Hence, a, b, d are correct answer.
CH3 C Cl 1
acetylation 38. (a, b, c, d) We have,A (α, β ), B (1, 2 ) Since, A =
2
and C(2, 3) are the vertices of ∆ABC. 2 1 1
OH ∴ − =
Since, A (α, β ) lies on the line 3 n+1 2
CCH3 y = 2 x + 3, therefore
AlCl3 , ∆ 1 2 1 1
β = 2α + 3 … (i) ⇒ = − =
H 2O O n+1 3 2 6
Hydroxyaceto phenone Also, given that area of ∆ABC = S
⇒ n + 1= 6⇒n = 5
This rearrangement involves the α β 1
1 Thus, n is a divisor of 15, 20, 30.
migration of acyl group. ∴S = 1 2 1
2 40. (a, c, d) We have,
32. (b) Baeyer-Villiger reaction involves 2 3 1
z = f( x ) + ig ( x ), where
ketone as a reactant. α 2α + 3 1 f, g : ( 0, 1) → ( 0, 1)
1
O = 1 2 1 [from Eq. (i)] 1  1 
2 (a) z = + i 
C6H5COOH
2 3 1 1 − ix  1 + ix 
C CH3 COCH3
H+
⇒ 2S =|α(2 − 3) − (2α + 3)(1 − 2 ) On simplification,
O O
+ 1( 3 − 4)| 1+ x  1+ x 
(Acetophenone) (Methyl z= + i 
(ketone) benzoate ester) 2S = | − α + 2α + 3 − 1| = | α + 2 | 1 + x2 1 + x 
2

α +2 α +2 1+ x
In this reaction, a ketone is converted into S = ⇒2≤ <3 ∴ f( x ) = g ( x ) =
an ester with the acyl or alkyl migration. 2 2 1 + x2
JEE Advanced~Practice Set 9 449

1  1 3 x 2 + 13 x + 14 1
For x = , f   > 1, which is out of and lim f( x ) = lim ∆ =
ab sin C
2 2 x→ −2 x → − 2 ( x + 2 ) ( x − 1) 2
range. Hence (a) is not correct. (3 x + 7 ) ( x + 2 ) 1
= lim ⇒ 15 3 = × 6 × 10 sin C
1  1  x → − 2 ( x + 2 ) ( x − 1) 2
(b) z = + i  3
1 + ix  1 − ix  3x + 7 −1 ⇒ sin C =
= lim = 2
x → −2 x − 1 3
On simplification,
Now, ∠ACB is obtuse
1− x  1− x  42. (a, b, c, d) Let T5 be numerically the
z= + i  ∴ c = 120°
1 + x2 1 + x 
2 greatest term in the expansion of
10 c= a 2 + b 2 − 2 ab cos C
1− x  1 + x  then,
∴ f( x ) = g ( x ) = ,  
 3 c= 6 2 + 10 2 − 2 × 6 × 10 cos 120°
1 + x2
T5 T
≥ 1 and 6 ≤ 1
f( x ) and g ( x ) ∈ ( 0, 1) if x ∈ ( 0, 1) 36 + 100 − 2 × 6 × 10  − 
1
T4 T5 c=
Hence, (b) is correct.  2
Tr + 1 10 − r + 1 x
1  1  Now = ⋅ c= 36 + 100 + 60 = 196 = 14
(c) z = + i  Tr r 3
1 + ix  1 + ix  ∆ 15 3
T5 10 − 4 + 1 x r= ⇒ r=
⇒ = × ≥1 S 6 + 10 + 14
On simplification, T4 4 3 2
1+ x  1− x 
z= + i  T6 10 − 5 + 1 x 15 3
1 + x2 1 + x 
2 and = × ≤1 r= = 3 ⇒ r = 1.73
T5 5 3 15
1+ x
where, f( x ) = and ⇒
7
x ≥ 1 and
2x
≤1 46. (7) Given hyperbola is
1 + x2 12 5 3 x 2 − 2 y2 = 6
1− x 12 5
g ( x) = ⇒ | x| ≥ and| x | ≤ x2 y2
1 + x2 7 2 ⇒ − =1
12 5 2 3
For, x =
1  1
, f   > 1. Hence, (c) is ⇒ ≤ | x| ≤
7 2 Tangents from the point (α, β)
2 2
 − 5 − 12  ∪  12 , 5  y = mx ± a 2m 2 − b 2
not correct. ⇒ x∈ ,
 2 7   7 2 
1  1  ⇒ ( y − mx ) = a 2m2 − b 2
2
(d) z = + i  Hence, (a, b, c, d) are correct answer.
1 − ix  1 − ix  ⇒ (β − mα) 2 = 2 m2 − 3
43. (a, c, d) Since, ( − 2, 3) lies inside the
On simplification, [Q a 2 = 2 , b 2 = 3]
circle
1− x 1+ x  ⇒ β + m α − 2 mαβ = 2 m2 − 3
2 2 2
z= + i  x 2 + y 2 + 8 x −10 y − 40 = 0
1+ x 1 + x 
2 2
⇒ m2(α 2 − 2 ) − 2 mαβ + β 2 + 3 = 0
centre of circle ( − 4, 5)
1− x 1+ x
where, f( x ) = , g ( x) = β2 + 3
1 + x2 1 + x2 m1m2 = = 2 = tan θ ⋅ tan φ
α2 − 2
1  1
For x = , g   > 1. Hence, (d) is ∴ β 2 + 3 = 2α 2 − 4
2 2 O
(–4, 5) ⇒ 2α 2 − β 2 = 3 + 4 = 7
not correct.
41. (a, b, c, d) We have, A 47. (96) We have,
(–2, 3)
3 x + ax + a + 1
2  − 3 6n
f( x ) =
x + x−2
2 I = lim  n
n→ ∞ 
2
 ∑ r
Radius of circle = 9   r =1

3 x 2 + ax + a + 1 1+ 2 + 3+K+
= a = 9 + OA 6n
( x + 2 ) ( x − 1) = lim
a=9+ ( − 2 + 4) + ( 3 − 5)
2 2 n→ ∞ n n
As x → 1, D r → 0, therefore for limit to 1 6n r
a=9+2 2
be exist N r → 0 when x → 1 and b = 9 − OA = 9 − 2 2
= lim
n→ ∞

nr =1 n
⇒ 3 + 2a + 1 = 0 ⇒ a = − 2 ∴ a + b = 9 + 2 2 + 9 − 2 2 = 18 6
2 3  2
a−b=9+2 2 −9+2 2 =4 2 6
As x → − 2, D → 0, therefore for limit ∫0
r
= x dx =  x 2  = 6 6
to be exist ab = ( 9 + 2 2 ) ( 9 − 2 2 )  3  0 3
N r → 0 when x → − 2 = 81 − 8 = 73
= 96 ⇒ λ = 96
⇒ 12 − 2 a + a + 1 = 0 ⇒ a = 13
44. (8) In a skew symmetric matrix
48. (6) Let there be x black socks and y
diagonal elements are zero. Also
3 x2 − 2 x − 1 white socks. Let x > y
Now, lim f( x ) = lim aij + a ji = 0
x→1 x → 1 ( x + 2 ) ( x − 1) x
C 2 + yC 2 1
Hence, the number of matrices Then, x +y
=
( 3 x + 1)( x − 1) 4 C2 2
= lim = =2 ×2 ×2 = 8
x → 1 ( x + 2 ) ( x − 1) 3 x( x − 1) + y( y − 1) 1
45. (1.73) Given, a = 6, b = 10, ∆ = 15 3 ⇒ =
( x + y) ( x + y − 1) 2
450 JEE Advanced~Practice Set 9

⇒ 2 x 2 − 2 x + 2 y2 − 2 y (Take α1 = 1, α = 2, β1 = 3 and β = 4) ⇒ y = 2 x + 12
= x + 2 xy + y − x − y
2 2 Clearly, f( x ) is increasing in (α1, β1 ) and ∴Correct combination is (II) (I) (S)
(β1, β ). Also, f( x ) has no 53. (c) Equation of normal is
⇒ x + y − 2 xy = x + y
2 2
maxima/minima in [α1, β1 ] and no
x − 3 y − 33 = 0
⇒ ( x − y) = ( x + y)
2 maxima/minima in [α, β ].
1
When, α1 < β1 < α < β, then graph of ∴ m=
⇒ | x − y| = x+ y 3
f( x ) is
Since, x + y ≤ 15 Point of contact ( 6, − 9)
∴ x − y ≤ 15 2a a
∴ = 6 and − 2 = − 9
As x − y must be an integer m m
∴ x− y=3 ...(i) 1
m= ,a=1
x+ y=9 ...(ii) y=1 3
X
α1 β1 α β
Solving Eqs. (i) and (ii), we get x = 6. y=0 Equation of normal
1 1
Solution (49, 50 and 51) ⇒ y = ( x ) − 2(1) − 2
3  1
Clearly, f( x ) is not defined when x = α1  
and x = β1. So, x = α1 and x = β1 will  3
be the asymptote to the curve. 1
⇒ y= x − 11
Now, when α1 < α < β1 < β, then (Take, α1 = 1; β1 = 2; α = 3 and β = 4) 3
graph of f( x ) is Clearly, f( x ) is not monotonic in any of ⇒ 3 y = x − 33 ⇒ x − 3 y − 33 = 0
the intervals (α1, β1 ), (α, β ); (β1, β ) and
54. (c) Equation of normal is y = x + 6
(α1, α ). Also, f( x ) has a maxima in
[α1, β1 ] and a minima in [α, β ]. m=1
asymptote
Hence, 49. (a); 50. (b) and 51. (c) focus of parabola is ( 0, 2 )
asymptote

asymptote

y=1
β1 52. (b) Equation of normal is y = 2 x + 12 ∴ a=2
X
α1 α β Equation of normal
∴ m = 2, a = 1
a
point of contact (i) ( − 4, 4) y = mx + 2 a +
m2
Equation of normal (S ) = y = 2 x + 2(1)
y = x + 2(2 ) + 2 ⇒ y = x + 6
(2 ) + (1) (2 )3

Paper 2
1. (d) Mass of the ring, M = ρL. 1 ga 2 ∴From question,
b = a tanθ −
Let R be the radius of the ring, then 2 u cos 2 θ
2
λ1 = 4λ 3
L ga 2 and λ2 = 2λ3
L = 2 πR or R = = a tanθ − sec θ 2

2π 2u 2  dN 
Activity   = − λN
Moment of inertia about an axis ga 2  dt 
= a tanθ − (1 + tan2 θ)
passing through O and parallel to XX' 2u 2 As activities are same at, t = 0.
will be or ga 2 tan2 θ − 2 au 2 tanθ + ga 2 ∴ N1 : N2 : N3 = 1 : 2 : 4
1
I0 = MR 2 + 2 bu 2 = 0 Hence, probability of α-emission at
2 t = 1600 s.
Therefore, moment of inertia about XX ′ For real roots of this quadratic λ1 ⋅ N1
(from parallel axes theorem) will be equation,
= 16
given by D≥ 0 λ1 ⋅ N1 λ 2 ⋅ N2 λ 3 ⋅ N3
1 3 + +
IXX ′ = MR 2 + MR 2 = MR 2 i.e. 4a 2u − 4ga 2 ( ga 2 + 2 bu 2 ) ≥ 0 16 4 2
2 2 1
or u − 2 gbu 2 − g 2a 2 ≥ 0 =
Substituting values of M and R, 1+ 4 + 8
or u − 2 gbu 2+ b 2g 2 ≥ a 2g 2 + b 2g 2
3  L2  3 ρL3 1
IXX ′ = (ρL )   = =
2  4π 
2
8π2 or (u 2 − bg )2 ≥ ( a 2 + b 2 )g 2 13
2. (a) or u ≥ bg + g a2 + b 2 Hence, (d) is the correct option.
Equation of trajectory is given as Hence, (a) is the correct option. 4. (a)
1 gx 2 ∴ W = ∆U = U f − U i
y = x tanθ − 3. (d)
2 u cos 2 θ
2 = U∞ − UP
Let disintegration constant for
When particle passes through a point = − UP
α-emission is λ1, β-emission is λ 2
A( a, b ), then = − mVP = − VP (as m = 1)
and deuteron emission is λ 3.
JEE Advanced~Practice Set 9 451
P
In the second case, 11. (a,c)
It corresponds to third harmonic. In the given figure,
4R √16R2 + r2 8. (a,c,d) Process AB is an isobaric process.
1 During this process,
I(t ) = ( I1 + I2 ) = 25 3 (1 + 2 sinωt )
R V ∝T
r Heat produced in one cycle, But, pV = nRT ⇒ pV ∝ T
dr 2π / ω
3 Thus, during this process, pressure p
∫ R dt =
2
J
remains constant.
0
In process BC, temperature
Heat produced in 14 min,
Potential at point P will be obtained by decreases, while volume remains
3
integration as given below. Q = 14 × 60 × 50 × = 63000 J constant.
2
Let dM be the mass of small ring as Process CA is an isothermal process.
shown. ∴ Q = ms ∆θ, ∆θ = 5° C
Hence, on T-V diagram, process AB
M ∴ Tf = 5 + 20 = 25° C
dM = (2 πr ) d r will be a straight line parallel to the
π( 4R )2 − π ( 3R )2 and heat produced by direct current, T-axis, during which temperature
2 Mr dr I 2Rt = Q increase. Process BC will be a straight
= 2 line passing through origin. During
7R Q
∴ I= = 1.5 A which temperature and volume both
G dM RT
dVP = − decreases and process CA will be a
16R 2 + r 2 9. (a,b) straight line parallel to the V-axis.
2 GM 4R
r The change in momentum for each Hence, option (a) is correct.
=−
7 R2
∫ dr time interval t 0 is ∆p = Ft 0, which is On p-V diagram, process AB will be a
3R 16R + r
2 2
constant. straight line parallel to the V-axis,
2 GM process BC will be a straight line
=− ( 4 2 − 5) But the change in kinetic energy in
7R each time interval t 0 is different. parallel to the p-axis and CA will be a
2 GM rectangular hyperbola.
∴ W =+ ( 4 2 − 5) First t 0 interval,
7R Hence, (c) is the correct option.
∆p2 F 2t 02
∆K 1 = =
4T 4T 2m 2m 12. (b,c) Use relations, Q = Q 0(1 − e − t / RC )
5. (b) ∆p1 = and ∆p2 =
r1 r2 Second t 0 interval, and i = i 0e − t / RC
r1 < r2 (2 ∆p)2 − ( ∆p)2 3F 2t 02
∆K 2 = = 13. (a,c)
∴ ∆p1 > ∆p2 2m 2m v 3
Q = ⇒ v = 3u
∴ Air will flow from 1 to 2 and volume The change in kinetic energy is u 1
of bubble at end 1 will decrease. ∆K = Fx0 = constant. 1 1 1
∴ − =
6. (b) First x0 displacement, v u f
Force from right hand side liquid on ∆p1 = 2 m∆K1 ⇒
1 1 1
+ = ⇒ f = 30 cm
left hand side liquid. = 2 m( Fx0 ) 3u u f
(i) Due to surface tension force = 2 RT Second x0 displacement, 14. (a,b,c)
(towards right)
∆p2 = 2 m(2 Fx0 ) − 2 m( Fx0 ) Given, φ = 4t n + 6
(ii) Due to liquid pressure force
= 0.414 2 m( Fx0 ) dφ
x=h ∴ = 4n ⋅ t n − 1
= ∫ ( p0 + ρgh)(2 R ⋅ x ) dx 10. (b,c) dt
4n
x=0 Motion of m2 starts, when kx = µ. | e | = 4nt n − 1 ⇒ | e | =
m2. g , where x = extension in the t1 − n
= (2 p0Rh + Rρgh ) (towards left)
2
spring, x = µm2g / k. 15. (c)
∴Net force is |2 p0Rh + Rρgh2 − 2 RT| The minimum force will be such that
Angular momentum is conserved
7. (a) m1 has no kinetic energy. Applying,
about centre of mass of system,
work-energy principle on body of
Let f0 = frequency of tuning fork. mass m1,
because no external torque is there
5 9g x during collision.
Then, f0 =
2l µ ∫ (Fmin − µm1g − kx ) dx = 0 L
Li (about CM) = Mv 0 (CW )
0
y 4
(where, µ = mass per unit 1 2
length of wire ) ⇒ Fmin x − µ m1gx − kx = 0
2
3 Mg x
= ⇒ Fmin = µm1g + kx 
1
2l µ  2 
Solving this, we get CM
µm2g 
= µm1g +
M = 25 kg  2  (L/4, – L/4)
In the first case,
µm2g
Frequency corresponds to fifth ⇒ Fmin = µm1g +
2 (0, – L)
harmonic.
452 JEE Advanced~Practice Set 9

After collision, combined system For equilibrium of plate


23. (c) σ
rotate about centre of mass. A :(2 k ) x1 = 0.375
For equilibrium of plate NaH
So, Lf = ICM ω
B : kx2 = 0.375
 L2 L2 


1 – H2
ICM = (2 M ) L2 − 2 m  +  or x2 = 2 x1 ...(i)
3  16 16  (Stable)
Due to the extension of the springs,
5
= mL2 ⇒ Li = Lf the separation between the plates will
12 reduce by
L 5 3 v0 Br
⇒ Mv 0 = ML2 ω ⇒ ω = x1 + x2 = 4 mm …(ii)
4 12 5L
(because the plates separation
(As, position vector is opposite to be reduces 6 mm to 2 mm) SN2
direction x), so ω will be in the
On solving these two Eqs. (i) and (ii),
direction opposite to positive Z-axis. 24. (b) C V (gas A)
we get
16. (c) x2 = 2.67 mm 3 3
= R + R + 6R = 9R
Velocity of v A = v A /CM + vCM, so v A will 2 2
19. (a) This problem can be solved by
be maximum, when velocity of A w.r.t.
using the concept of half-life time of Let C V for gas B be x.
CM will be in the direction of velocity
radioactive decay. Heat lost by the gas
of CM.
Total time = nt 1/ 2 ⇒ 30 = n.6 ⇒ n = 5
A = 7 × 9R ×  T −
Mv 0 v 0 5T 
v CM = = 
2M 2 Initial activity = 0.01 × 2 5 = 0.32 µCi  6
[conservation of linear momentum] Heat gain by the gas
20. (b) Preparation of Copper Sulphate
v A / CM = ω R A /C
B = 6 x ⋅ 
Cu(SO 4 ) 5T T 
− 
[where, R A /C = distance between A When copper oxide is treated with  6 3
and CM] sulphuric acid it produces copper
⇒ 63R   = 6 x ⋅  
T T
sulphate [X].
2
 L − 0 +  − L + L 
2
10 L  6 2
=     = CuO + H 2SO 4 → CuSO 4 + H 2O
4   4  4 7
[X ] ⇒ x= R
10 L In aqueous solution it exist as 2
3v 0
v A/ CM = × pentahydrate copper sulphate which
5L 4 25. (a) Wilkinson catalyst Tristriphenyl
is blue in colour. phosphine rhodium (I) chloride is
v 0 3v 0 10 L
v A /max = + × [Cu(H2O) 4 ] SO 4 ⋅ H2O −→ Blue colour known as Wilkinson catalyst, in which
2 5L 4
Chemical Properties of Copper oxidation state of rhodium is + I.
v0  3 
= 1 +  Sulphate Structure of Wilkinson’s catalyst is
2  10  When copper sulphate is treated with square planar.
17. (c) It is given that, electric potential excess of aqueous solution of Ph3P +1 PPh3
energy stored in the capacitor is ammonia it produces [Cu (NH3 ) 4 ] 2+
Rh
7.5 × 10 −4 J. In the charged state, which has deep blue colour.
Ph3P Cl
since the capacitance is C f and the [Cu (H2O) 4 ] SO 4 ⋅ H2O + NH3 →
Hybridisation of Wilkinson catalyst is
battery has remained connected. [Cu (NH3 ) 4 ] SO 4
dsp2, an inner orbital complex is
1 Deep blue colour
Therefore, C f V 2 = 7.5 × 10 −4 (needle like crystal) formed due to the presence of strong
2
[ X ] = [Cu (H2O) 4 ] SO 4 ⋅ H2O ligand PPh3.
1
C f (25)2 = 7.5 × 10 −4
2 [Y ] = [Cu (NH3 ) 4 ] SO 4 26. (a) Use Raoult’s law to solve the
−6 question.
C f = 2.4 × 10 F 21. (b) Order of strength of
intermolecular forces Let the mole fraction of P in liquid
Therefore, C f = 2.4 µF phase be x and the mole fraction of P
Elastomer < Thermoplastic < Fibre
C in vapour phase will be (1 − x ).
or C i = f = 0.8 µF < Thermosetting
3 Buna-S < Polythene < Nylon-6, 6 Similarly, mole fraction of Q in liquid
18. (a) < Bakelite. According to given figures phase will be (1 − x ) and in vapour will
R < A < G < U. be x.
When the switch S is closed, the
capacitor becomes charged and the 22. (a) Out of the given elements, only xPP°
⇒ (1 − x ) = …(i)
plates A and B, then carry + Q and Cu and Pb undergo self reduction. pvap
− Q charges. Consequently, the plates The sulphide of Pb, Cu are heated in (1 − x ) PQ°
develop a force of attraction given by air to either convert the complete ore ⇒ xp = …(ii)
Q2 1 Q2 pvap
F = = or part of ore into oxide or sulphate
2 ε0 A 2 C id which react with remaining sulphide Eliminate x from both equations
(because ε0 A = Ci d ) ore to give metal and SO 2. ⇒ pvap = PP° × PQ° = 400 × 100
where, C i = 0.8 µF, d = 6 mm and So, option which contain any of these
pvap = 200 torr
F = 0.375 N two metal is not correct and does not
extracted by involving self reduction 27. (c) ∆G = − 2.303 RT log K
(each plate attracts the
other by this force) method. − nFE ° = − 2.303 RT log K
JEE Advanced~Practice Set 9 453

nFE ° nFE ° Z and W are same compounds.


log K = = 0.4342 LiAlH4 Conc. H2SO4
2.303 RT RT Hence, (c) is correct choice.
or ⊕
nFE ° H3PO4 35. (a) B  Cl B Br B F
ln K = HO 2.0 3.0 2.8 4.0
RT
(E) 
nFE °

K = e RT Most electronegative
Ring expansion –H⊕
28. (a,b,c) Isoelectric point, ⊕
Since, F is most electronegative,
pK a1 + pK a2 hence electron density is mostly bent
(PI) =
2 towards F hence % s-character of B
Reagents like Al 2 O 3 , ThO 2 shows
2.3 + 97. decreases.
PI = =6 dehydration via E 2 elimination.
s
2 32. (a,b,d) Preparation of O 2 36. (b) cos θ =
s−1
At PI,
Thermal decomposition of KClO 3
(i) Maximum concentration of Zwitter where, s = percentage, s-character,
produces O 2.
-ion will be present. ∆
θ = bond angle.
(ii) Net charge on amino acid is zero. 2KClO 3 → 2KCl + 3 O 2 F
670 - 720 K
(iii) Amino acid is undeflected in the According to molecular orbital theory, F
presence of electric field. C
outer molecular orbital electronic H 112°
H
T V
29. (a,b,d) ∆S = n1 ln 2 + n2 ln 2 configuration of O 2 = π *2 p1x π *2 p1y s
T1 V1 For CH2F2 , cos θ =
Number of unpaired electrons = 2 s−1
373 10
∆S = 5 ln + 2 ln Chemical reactions of O 2 s
298 1 cos 112 ° =
O 2 on reaction with Al produces Al 2O 3 s− 1
∆E = nC V ∆T = 5 (75)
which when dissolved in water s
= 375 cal − 0.3729 =
produces Al(OH) 3 which is an s−1
∆H = ∆E + R∆T amphoteric hydroxide.
% s = 27.1%.
∆H = 375 + 2(75) Al + 3O 2 → Al 2O 3
Al 2O 3 + H2O → Al(OH) 3
37. (d) We have,
∆H = 525 cal
1 + x, if x < 0
Q ∆G = ∆H − T∆S Amphoteric f( x ) = 
S + O 2 → SO 2 (1 − x ) ( px + q ), if x ≥ 0
∴ ∆G of the process can be calculated
using the above information. SO 2 + H2O → H2 SO 3 For f( x ) to satisfy Rolle’s theorem in
33. (c) Addition of H+ to alkene produces [− 1, 1]
30. (a,c,d) Charge transfer
spectrum The charge transfer more stable carbocation. f( x ) should be continuous in [− 1, 1]
spectrum is obtained due to transfer ⇒ lim f( x ) = lim f( x ) = f( 0)
+ x → 0− x → 0+
of charge from one position to another
: :

position as in case of KMnO 4 where O ⇒ lim (1 + x ) = lim (1 − x )( px + q )


charge on oxygen atoms transfer from H H x → 0− x → 0+
H+
O to metal. Metal accept electron
Electrophilic
⇒ 1= q
density in its vacant d-orbital. In AgI, Is addition and f( x ) should differentiable in ( − 1, 1)
transfers its electron to vacant
d-orbital of Ag + .
H ρ ⇒ f ′( 0− ) = f ′( 0 + )
OH OH
⇒ 1= p− q = p− 1
31. (b,c) Conversion of E to A using
appropriate reagents. ⇒ p=2
– H+ Therefore, ordered pair ( p, q ) = (2, 1)
O3 Mg + Hg 38. (b) We have,
O== ==O
Zn + H2O H 2O x − y + 2z = a ...(i)
(Z) x + 2y − z = b ...(ii)
(A ) (B) 34. (c) 2y − 2z = c ...(iii)
OH Br
OH From Eqs. (ii) and (i),
⊕ 3y − 3z = b − a
H
b − a
H2SO4, H2O (i) BH3, THF HBr
HO— —OH ⊕
y − z = 

HO—
 ...(iv)
(Z )
(ii) OH /H2O
(X ) (Y )
 3 
(C) (i) (CH3CO)2O, Hg, H2O
From Eqs. (iii) and (iv),
(ii) NaBH4
Rearrangement 2( b − a )
⊕ OH c≠ for inconsistency
3
HO O Clearly, ( a, b, c ) ≡ (2, 2, 1) satisfies
(D)
this condition.
(W )
454 JEE Advanced~Practice Set 9

39. (c) Let θ1 be the angle between a and Divide by cos 2 x, we get dr
⇒ = a2 − r 2
b, θ 2 be the angle between c and u 2 sec 2 x − 2u tan x + 1 = 1 + sin2 θ dθ
⇒ u 2 tan2 x − 2u tan x + 1 dr
d and φ be angle between a × b and ⇒ = dθ
c × d, then = 1 + sin2 θ − u 2 a − r2
2

(a × b ) ⋅ (c × d ) = 1 ⇒ (u tan x − 1)2 = 1 + sin2 θ − u 2 On integrating both sides, we get


⇒ |a × b||c × d| cos φ = 1 ⇒ 1 + sin2 θ − u 2 ≥ 0 dr
⇒|a||b| sinθ1 .|c||d| sinθ 2 cos φ = 1 ∫ 2 2 = ∫ dθ
⇒ u 2 ≤ 1 + sin2 θ a −r
⇒ sinθ1 sinθ 2 cos φ = 1
⇒ sin−1   + C = θ = tan−1  
π π ⇒ Maximum value of u is 1 + sin2 θ r y
⇒ θ1 = , θ 2 = and φ = 0  a  x
2 2
1  π3 π5 π7
Now, as φ = 0 , therefore a × b and 43. (a) We have,  − + ...  x 2 + y2 
2 π  1!⋅ 3 3 !⋅ 5 5 !⋅ 7 ⇒ y = x tan  sin−1 + C
c × d are parallel.  
 a 
⇒ a , b, c and d are coplanar ( − 1)n − 1( π) 2n + 1 
...
1 (2 n − 1)! (2 n + 1)   
x 2 + y 2 = a sin  tan−1   + C 
Also, we have a ⋅ c = y
or
2   x 
1 πx 2
x x4  6

2 π ∫0  1!
1
⇒ cosθ 3 = (where, θ 3 is the angle =  − + K dx
2 3! 5!  46. (b, c, d) We have,
between a and c) 1 π x x x 3 5 a, b, c are in AP
2 π ∫0  1! 3 !
π = x − + K dx ∴ 2b = a + c
⇒ θ3 = 5! 
3 a + 8b c + 8b
1 π Now, x = +
π
2 π ∫0
⇒ Angle between b and d is . = x sin x dx 2b − a 2b − c
3 a + 4( a + c ) c + 4( a + c )
∴ b and d are non-parallel.  x x3 x5  ⇒x= +
Q sin x = 1! − 3 ! + 5 ! + K a+c−a a+c−a
40. (c) We have, f( x)= ax 2 − c   5a + 4c 4a + 5c
1 π
⇒x= +
∴ − 4≤ a − c ≤ − 1 = [− x cos x + sin x ]0 c a

and − 1 ≤ 4a − c ≤ 5  a c
1 1 ⇒ x = 5 +  + 8
⇒ 0 ≤ a ≤ 3, 1 ≤ c ≤ 7 = [ π] = c a
2π 2
f ′ ( x ) = 2 ax Q a + c ≥ 2 
44. (a,d) We have, px 2 + y 2 + qz2 + 2 yz ⇒ x ≥ 5(2 ) + 8 = 18
For greatest value of f( 3) a should be  c a 
maximum + zx + 3 xy = 0 ∴ x 2 ≥ 324
2 2
⇒ p
x  y
f(2 ) = 5 ⇒ f( x ) = 3 x 2 − 7 2  
x y
 +  + + Hence, option b, c, d are correct
 z  z z  z
⇒ f( 3) = 3( 3)2 − 7 = 20 answer.
+ 3    + q = 0
x y
For least value of f( 3) should be 47. (b, d) On substituting x = 1 in
 z  z
minimum y = 2 x − 3, we get y = − 1. Clearly,
f(2 ) = − 1 This represents a equation of plane the (1, − 1) will lie on y = x 2 + px + q .
1 3 1
∴ f( x ) = − 1, f( 3) = − 1 ∴∆ = pq + 2 × × 1 × − p − ∴ We get − 1 = 1 + p + q
2 2 4
∴ − 1 ≤ f( 3) ≤ 20 2 ⇒ p + q = −2
 3
−q   = 0
41. (d) The probability that one test is 2 Now, distance of the vertex of the
parabola from the X-axis is
held, is 2  ×  = ⇒ 4 pq − 4 p − 9q + 5 = 0
1 4 8
p2 p2 p2
 5 5  25 D = f  −  =
p
and also for perpendicularity − + q =q −
 2 4 2 4
The probability that test is held on p+ q + 1= 0
1 1
both days is × =
1
∴ 4 p( − p − 1) − 4 p + 9( p + 1) + 5 = 0 p2
= −2 − p− [Q p + q = − 2 ]
5 5 25 4
⇒ 4 p2 − p − 14 = 0
∴The probability that the student p2
⇒ (4 p + 7 ) ( p − 2 ) = 0 ⇒ D= −2 − p−
misses at least one test is 4
7 3
8
+
1
=
9 ⇒ p = 2, − , q = − 3, ⇒
dD
= − 1−
p
25 25 25 4 4 dp 2
5
42. (b) We have, ∴ p − q = 5, − dD
2 Now, =0
u = cos x(sin x + sin2 x + sin2 θ ) dp
45. (a, d) Let x = r cosθ and y = r sinθ, so p
⇒ (u − cos x sin x )2 ⇒ − 1− = 0
that x 2 + y 2 = r 2; x dx + y dy = r dr 2
= cos x(sin x + sin θ)
2 2 2
and x dy − y dx = x 2 sec 2 θ dθ = r 2dθ ⇒ p= −2
⇒ u − 2u cos x sin x = cos x sin θ
2 2 2
Now, the given equation can be ⇒ q =0
⇒ u 2 − 2u cos x sin x + cos 2 x transformed into Clearly, the least distance of the vertex
rdr a2 − r 2 from X-axis is 1.
= cos 2 x(1 + sin2 θ) =
r 2 dθ r2
JEE Advanced~Practice Set 9 455

48. (b, d) Since a and b are equally +


1
cos( a3 + x ) ∴ Maximum value of ∆ = 3 7 sq unit
→ →
inclined to c , c must be of the form, 22 52. (d) Since, mid-point of A and B is
1
 a b + ... + cos( an + x ) ( x1, y1 ).
λ + . 2 n−1
| a | |b|  ∴ A ≡ (2 x1, 0)
 cos a2 cos a3
⇒ f( x ) =  cos a1 + + and B ≡ ( 0, 2 y1 )
Now,  2 22
|b| |a| 1 Clearly, slope of AB is m
a + b= cos an  2 y1 − 0
|a| + |b| |a| + |b| |a| + |b| + K+  cos x =m
2n − 1  0 − 2 x1
(|b| a + |a| b )  sin a2 sin a3
|a||b| (|b|a + |a|b ) −  sin a1 + + + ... ⇒
y1
=m
=  2 22
|a| + |b| |a||b| − x1
sin an 
 sin x ⇒ y1 = − mx1
[multiply and divide by|a||b|] 2n − 1 
⇒ mx1 + y1 = 0
|a||b|  a b ⇒ f( x ) = A cos x − B sin x
=  + , 53. (a) Using three basic colours Red,
|a| + |b| |a| |b|  Now, f( x1 ) = f( x2 ) = 0
Blue, Green. There are four different
 a ⇒ A cos x1 − B sin x1 = 0,
b colours made by using two or more
which is of the form λ  +  A cos x2 − B sin x2 = 0
|a| |b|  coloums. i.e. RB, RG, BG, RBG.
|a||b| ⇒ tan x1 = tan x2 ⇒ x1 = nπ + x2 Hence, there are total seven colours
where, λ = .
|a| + |b| ⇒ x1− x2 = nπ are available i.e, R, B, G, RB, RG, BG,
RBG.
|b| |a| (Q. No. 51 and 52)
Also, a + b Therefore 7 colours are available for n
2|a| + |b| 2|a| + |b| Here, we have ae = 7 and
a 16
= verticals stripes are painted.
e 7
|a||b|  a b ∴ For first stripe all seven colours are
=  +  ⇒ a 2 = 16 ⇒ a = 4
2|b| + |b| |a| |b|  available after words each stripes has
7 6 possibilities only.
Other two vectors cannot be written in ⇒ e =
∴ tn +1 = 6tn
 a b 4
the form λ  + . 54. (d) t n → Number of ways of painting ‘
b 2 = a 2(1 − e 2 ) = 16  1 −
|a| |b|  7 
Now, 
 16  n’ stripes
49. (a, d) We have, f ′ ( x) = g ( x)( x − a )2,
= 16 − 7 = 9 an → Numbers of ways of painting ‘n
g ( a ) ≠ 0 and g ( x ) is continuous at stripes when, it ends with a particular
x2 y2
x = a. ∴ Equation of ellipse is + =1 basic colour (R, B or G)
16 9
Since, g ( a ) ≠ 0, therefore either b n → Numbers of ways of painting n
g ( a ) > 0 or g ( a ) < 0. 51. (c) Now, let the coordinates of P be stripes when, it ends with a particular
( 4 cos φ, 3 sin φ) . Then, area of
Let g ( a ) > 0. Since, g ( x ) is continuous non basic colours (RB, RG, BG)
1
at x = a ∆ PSS ′ = × SS ′ × PM ∴ t n = 3( an + b n ) ...(i)
2
∴ There exist a neighbourhood of a in an = 2 an − 1 + b n − 1 ...(ii)
1
which g ( x ) > 0 = × 2 7 × 3 sin φ b n = an − 1 ...(iii)
2
⇒ f ′ ( x ) > 0 in that neighbourhood of a. Eqs. (ii) and (iii) gives
= 3 7 sin φ sq unit
⇒ f( x ) is increasing in the bn + 1 = 2 bn + bn − 1
Y
neighbourbood of a.
On solving characteristic equation
Let g ( a ) < 0. Since g ( x ) is continuous
B x 2 − 2 x − 1 = 0, we get
at x = a
b n = α(1 + 2 )n − 1 + β(1 − 2 )n − 1
∴ There exist a neighbourhood of a in
which g ( x ) < 0 b 2 = 1 ⇒ 1 = α + β + 2 (α − β)
⇒ f ′ ( x ) < 0 in that neighbourhood of a. ⇒α =β =
1
X′ O M S
⇒ f( x ) is decreasing in the S′ A X 2
(– √ 7, 0) ( √ 7, 0) 1
neighbourhood of a. ⇒ b n = [(1 + 2 )n − 1 + (1 − 2 )n − 1 ]
x 2+y 2=r 2 2
50. (a, b, c) We have,
3
f( x ) = cos( a1 + x ) ⇒ tn = [(1 + 2 )n − (1 − 2 )n ]
2
1
+ cos( a2 + x )
2 Y′
PRACTICE SET - 10
Paper 1
1. (b, c) At maximum angle, tension 4. (a) For a reflecting surface, radiation
Here, the motion of two balls at pressure
curves should be observed. The I
T p=
motion of ball B will be initially ma0 C
supported by the gravity but motion of
πr 2I
ball B will be initially against the ∴ Force on sphere =
gravity. C
Over horizontal part both will have 5. (a,d)
mg
The   >  
same speed. The motion over bump dp dp
will going to create the difference.  dV  adiabatic  dV  isothermal
Velocity of B first increases, then T = m g 2 + a02
So, CA is adiabatic and BC is
decreases but during entire motion in
T = m g 2 + g 2/3 isothermal.
bump it speed will be greater than v.
Whereas for A, its speed first 2 mg Process CA will be adiabatic and BC
T = will be isothermal.
decreases, then increases but during 3
complete motion it is less than v. So, pC VC = pBVB
B will go, faster, so it takes less time. 3. (d) V0
∴ 3 p0 ( VC ) = p0V0 ⇒ VC =
B cover same distance in less time, so The problem is basically of 3
discharging of CR circuit, because the
average velocity of B is greater than A. pC VCγ = pA VAγ
change on the plates of the capacitor γ
2. (a,d)
3 p0 ×  0  = 0 V0γ
decreases with time. There is V p
Here, we can apply pseudo force, if capacitor as well as resistance as  3 2
we observe from the accelerated shown in figure 3 1 ln ( 6)
= ⇒ γ=
frame of ring. We can apply C 3γ 2 ln ( 3)
work-energy theorem to find the value
at θ 0. 6. (a,b,c,d)

Let us discuss the motion of particle From conservation of energy, we can
with respect to ring by applying say that velocity of cylinder just before
R
pseudo force. For motion of particle getting onto plank is
from A to B apply work-energy Resistance, R =
d R = l  1
Mgh = Mv 02
theorem from A to B. σA  σA  2
Ring Kε 0 A where, v 0 = 2 gh
and capacitance, C =
d FBD for motion on rough plank.
K ε0
θ0 ∴Time constant, τc = CR = …(i)
σ
va
Substituting the values of ε0, σ and K
B
in Eq. (i), we get µMg
ma0 µMg
5 × 8.86 × 10 −12
τc = = 5.98 s
ma0 A 7.4 × 10 −12
Charge at any time decreases So, cylinder is having retardation of
(W ALL )AB = ∆K + ∆U exponentially as linear motion,
ma0 l sin θ 0 = 0 + mg l (1 − cos θ 0 ) q = q 0 e −t / τc v = v 0 − µ gt
a sin θ 0 = g (1 − cos θ 0 )
Here, q 0 = 8.85 × 10 −6C and it is having angular acceleration,
g
sinθ 0 = g (1 − cos θ 0 ) µ MgR 2µg
3 (charge at t = 0) α = =
Therefore, discharging (leakage) MR 2 /2 R
 g 
∴a = 2µgt
 0 3 
current at time t will be given by
So, ω=0+
−dq  q 0 −t / τc
sin θ 0 = 3− 3 cos θ 0 i = = e R
 dt  τc Plank will be having accelerated
1 3 3 or current at, i = 12 s is motion,
sin θ 0 + cos θ 0 =
2 2 2 v ′ = 0 + µgt
8.85 × 10 −6
i = × e −12/ 5.98
3 At pure rolling, v ′ = v − ωR
sin [θ 0 + 30° ] = 5.98
2 = 0198
. × 10 −6 A = 0198
. µA µgt = v 0 − µgt − 2µgt
θ 0 + 30° = 60° t = 0
v
i = 0198
. µA
θ 0 = 30° 4µg
JEE Advanced~Practice Set 10 457
Velocities of cylinder and wedge,  2 × 10 5  Substituting, P = µW from Eq.(i)
3v 0 v = 2 × R × 600 ln 
v = , v′ = 0  1 × 10 
5 µW
sin θ =
4 4 4W
= 1200 R ln (2 ) µW +
Mv + Mv ′ v + v ′ v 0 3π
v COM = = = nR ⋅ ∆T − 2 R × 300
2M 2 2 WCA = = 3 µπ
1− γ 1 sin θ =
Q For minimum length of plank let us 1− 3 µπ + 4
discuss cylinder ω rt plank 2
3 µπ
µMg  pT = K  ⇒ θ = sin−1
p V = K
2
 3 µπ + 4
(Pseudo
force) v0  p × V 1/ 2 = K  3 µπ
θ = sin−1
= − 1200 R 3 µπ + 4
µMg W = 600R + 1200R ln(2 ) − 1200R 7 sin θ = 3
For linear motion distance moved by W = 600R [2 ln (2 ) − 1] 10. (3.00)
COM in time of pure rolling. Heat supplied, Consider a small element of radius r
1 5R
S = v 0 t − (2µg ) t 2 Q AB = nC p ∆T = 2 × × 300 and width dr.
2 2 F
2
 v  1  v  = 1500R
= v 0  0  − × 2 µg ×  0 
 4 µg  2  4 µg  Q BC = W BC = 1200R ln(2 )
v 02 v 02 3v 02 600R [2 ln (2 ) − 1]
= − = η=
4 µg 16 µg 16 µg 1500R + 1200R ln (2 ) r
12 ln(2 ) − 6
7. (0.40) =
15 + 12 ln(2 ) dF
1 2
H= gt
2 21
η =1 −
1 15 + 12 ln (2 )
⇒ 500 = × 10 × t 2 r
2 3×7 dr
=1−
⇒ t = 10 s 15 + 12 ln (2 )
Also, R = ut ∴ x=7
R F
⇒ u = = 40 ms −2 9. (3.00) Normal reaction, dF = × 2 π rdr
t π 2
In this problem, concept of friction as D
u 4
well as the concept of torque will be
Frictional force, dFf = µdF
used. For the equilibrium, resultant
force will be zero. We will also balance Frictional moment, dM = rdFf = r µ dF
H the torque for equilibrium. Total moment,
ΣF =0 D/2 4F
M = ∫ dM = ∫ rµ × × 2 πrdr
Στ =0
0 πD 2
Under equilibrium, 4F ( D /2 )3
= × µ × 2π ×
R ΣFH = 0 = P − µN = 0 ⇒ P = µN πD 2
3
µFD
If recoil velocity of gun is v, then ΣFV = 0 = N − W = 0 ⇒ N = W M = ⇒ N=3
m1 v = m2 u 3
P = µW ...(i)
1 11. (2.00)
or v = × 40 = 0.4 ms −1
100
E1 [= 2 → 1 in H-atom]
O
8. (7.00)
R
Find the net work done for the N
process, then find heat supplied. By D θ
G P W0 e–
this you can find the value x with the λ
K, E1
help of efficiency formula. A
Total work done We have E1 = W 0 + KE 1 ...(i)
Efficiency = W
Total heat supplied Moment about contact point A,
E2 [= 2 → 1 in unknown gas]
W AB = p ( ∆V ) = nR ∆T P ( AD ) = W (GD )
(isochoric process) P (OA − OD ) = W (GD )
25 4r
=2 × × 300 = 5000 J P ( r − r sin θ) = W sin θ W0
3 3π e–
K, E2
T  p P
W BC = nRT ln  f  = nRT ln  i  sin θ = ...(ii) E 2 = W 0 + KE 2 ...(ii)
 Ti   pf  4W
P+ 1
3π Also, λe − ∝
(isothermal process) KE
458 JEE Advanced~Practice Set 10

KE 2  λ1 
2  γ +1  17. (c)
⇒ =   = 6.1 360 = 15 p0 V0  
KE 1  λ 2   2 ( γ − 1) Heat supplied to gas is equal to heat
  360 15 ( γ + 1) loss across resistance. Discharging
1 = ⋅
Q λ 2 = 6.1 λ1  56 4 γ current through resistance,
  7 2
12 γ = 7γ + 7, γ = = 1 + q = Q 0 e −t / RC
2  1 1
Also, E1 = 13.61 × 1  2 − 2  5 f
1 2  ⇒f = 5 dq
i =−
= 10.2eV ...(iv) dt
14. (2.00) For the given area total force
Now, ionisation energy of H-atoms, Q 0 −t / RC
will be u dA. = e
= − E1 = 13.6eV energy of first line in RC
When dA is the area, then
Lyman series = 13.6 Z 2  2 − 2  eV
1 1
electrical force per unit area Charge at capacitor at, t = 2.5 ln ( 4)
1 2 
2 2.5 ln ( 4 ) × 60

= 10.2 Z 2 eV 1 1  σ  σ 2
2000 × 75 × 10−34
= ε 0E 2 = ε 0   = q = Q0 e
2 2  2 ε0  2 ge 0
and energy of series limit of Balmer 2.5 ln ( 4 ) × 60
− ln
series = 13.6 × Z 2  2 − 2  eV Projection area = π r 2
1 1 = Q 0e 150
2 ∞   σ2  −
25 × 60 ln ( 4 )
⇒ Net electrical force =   ( πR )
2
= 3.4 Z 2eV  2 ε0  = Q 0e 150 × 10

Given, 13.6 × 2 = 10.2 Z − 3.4 Z


2 2
Q0
In equilibrium, this force should be = Q 0 e − ln ( 4 ) =
+ 4
⇒ Z =2 ⇒ He ion equal to the applied force,
Loss in potential energy of capacitor is
12. (1.00) σ 2R 2 4 ε 0
F = = =2 equal to heat supplied.
For constructive interference, intensity 2 ε0 2 ε0
Q 02 Q 02 /16
(I) is maximum, ∆U = −
15. (d) 2C 2C
When cos φ = +1 Let compression is x, then FBD of disc Q 02 15 15 Q 02
The path difference at point P, = × =
3 C 16 32 C
∆x = (SS 2 − SS1 ) + (S 2P − S1P ) kx 640
dy d (d /2 ) a α Q 0 = CV0 = 75 × 10 −3 ×
= + fs 3
D1 D2 1
kx − f s = Ma, = 16000 × = 16 C
For constructive interference, 1000
MR 2 15 Q 02
dy d2 f sR = α ∆U =
∆x = + = nλ 2 32 C
D1 2 D2
a = Rα 15 × 16 × 16
(10−3 ) ( 0.5 sin π t ) × 10 −3 (10 −3 )2 =
+ kx − f s = Ma, 32 × 75 × 10 −3
1 2 ×2 M Ma
fs = ( Rα ) = = 1600 J = 1.6 kJ
= nλ 2 2
π 18. (a) Gas expands at constant pressure,
0.5 sin   t × 10 −6 + 0.25 × 10 −6 kx − f s = 2 f s so work done will be
 6
kx
⇒ fs = W = nR∆T
= ( 5000 × 10 −10 ) n = 0.5 × 10 −6 n 3
= 1 × 8.3 × 72
π 0.5 n − 0.25
sin   t = Now, we can see f s depend on x, so f s
= 0.596 J = 0.6 kJ
 6 0.5 will be maximum when x is maximum
and that is A. ∆U = ∆Q − W
For the minimum value of t , n = 1
kA = 1.6 kJ − 0.6 kJ = 1 kJ
π π π
sin   t = ⇒   t = f smax =
1
Charcoal
 6 2  6 6 3 19. (d) Cl 2 + SO 2 → SO 2Cl 2( R )
or t = 1s For no slipping, 10SO 2Cl 2 + P4 → 10SO 2
kA
13. (5.00) 0<< µMg + 4PCl 5(S )
3
γ γ PCl 5 + 4H 2O → 5HCl + H 3PO 4 (T )
∆Q AB = nC p ∆T = nR∆T = 3 µMg
γ −1 γ −1 0< A< 20. (a,b,c,d)
k
γ (a) When the two H - atoms are away from
[3 p0V0 − p0V0 ] = 2 pV0 × 16. (b)
γ −1 points D, potential energy is zero.
kx
∆Q AC = ∆U + ∆W Fnet = −( kx – f ) = − ( kx – ) (b) As two H - atoms approach in the
3
nR 1 formation of H 2 molecule, potential
= ∆T + × 3V0 [ p0 + 4 p0 ] –2
γ −1 2 or Fnet = kx energy falls.
3
[16 p0V0 − p0V0 ] 15 p0V0 (c) Stability is said to be maximum at
= + –2 kx
So, a =
γ−1 2 minimum energy.
3M
γ (d) When atoms approach more closely
56 = 2 p0 V0 × 2k
γ −1 Q ω= = 10 / 3 (C to B) potential energy increases.
3M
JEE Advanced~Practice Set 10 459

21. (a,b,d) Corrected Statement ∆H reaction Q 1 mole of K 2 Cr2 O 7 oxidised 3 moles


It is 369 K or 96°C. The stable form at = − ( ∆Hsolution + ∆Hcal ) = −9.69 of Sn2 + .
room temperature is rhombic sulphur 9.69 = [0.400 + 4148. × 0.435][T − 293.48] Thus, amount of Sn in the ore
which transforms to monoclinic T = 29812
. K or 25.12°C = 3 × 2.4 × 10 −4
sulphur when heated about 369 K. Thus, option (b) is correct. = 7.2 × 10 −4 mol
22. (d) 25. (6) = 7.2 × 10 −4 × 118.7 g
(i) Bz NHOH
NaOH, DMF, German silver - Cu + Zn + Ni = 0.0855 g
BnO 2 OTs 80% Nichrome - 60% Ni + 20% Cr + Fe
NHCbz
Therefore, % of Sn
Monel metal - Cu + 66% Ni
0.085 × 100
Constanton - Cu +Ni = = 34.16%
NHBz 0.25
Invar - Fe + Ni
BnO 2 O 28. (230) Moles of metal
Alnico - Al + Ni +Co + Cu
NHCbz pV 12.5 0.5
r(Cl 2 and Clmix ) ( n) = = ×
HCl/EtOH
26. (0.14) RT 760 0.082 × 1075
.
r(Kr)
(80%) = 9.33 × 10 −5
reflux M (Kr )
= 116
. ⇒ Total number of atoms
NH2 Mav (Cl 2 + Cl )
= 9.33 × 10 −5 × 6.023 × 10 23
BnO 2 O 8.38
= = 5.62 × 1019 atoms
NH2 Mav
( OTS being a good leaving reagent 83.8 ∴In bcc, there are 2 atoms per unit cell.
16 Mav = 2
= 62 .28 g mol −1 ⇒ Number of unit cells in given sample
is readily replaced by  O NHB z . (116
. )
1
= × 5.62 × 1019
Both NHBz groups are hydrolysed Cl 2 - 2Cl 2
to give 1° amine). Initial mol 1 0 = 2.81 × 1019
23. (a,b,c) After dissociation 1− α 2α
⇒ Volume of one unit cell
Statement a,b and c are correct. (α = degree of dissociation)
. × 01
(162 . )3
NO 2 being a deactivating group, Total moles after dissociation =
2.81 × 1019
deactivates the ring to such an extent = 1 − α + 2α
that it does not undergo further = 1513
. × 10 −22cm3
= 1+ α
nitration (electrophilic substitution). Edge length of unit cell
(1 − α ) M (Cl 2 ) + 2α M (Cl )
200 × 0.862 ∴ = 62.28 1/ 3
24. (a,b) Moles of HCl = (1 + α ) = (1513
. × 10 −22)
1000
(1 − α ) × 71 + 2α × 35.5
= 62.28 = 5.32 × 10 −8 cm
= 01724
. mol = 01724
. mol H+
1+ α = 532 pm
200 × 0.431
Moles of Ba(OH)2 = ∴ α = 014
. Also, in bcc, 4r = 3a
1000
= 0.0862 mol 27. (34.16) The reactions are as follows: 3 3
∴ r= a= × 532 = 230 pm
2 × 200 × 0.431 Sn2+ → Sn4+ +2e − ] × 3 4 4
Moles of (OH − ) =
100 Cr2O 27− + 14H+ + 6e − → 2Cr 3+ 29. (1.43) Specific conductivity of KCl
= 01724
. mol OH − + 7H 2O solution is
∧C 150 × 0.01
+
Thus, moles of H = moles of OH − Overall Reaction : K(KCl) = =
1000 1000
and three will be complete reaction of 3Sn2+ + Cr2O 27− + 14H+ → 3Sn4+
. × 10 −3S cm−1
= 15
HCl and Ba(OH)2. + 2Cr 3+ + 7H 2O
R1 K 2
Hence, option (c) and (d) are Also, = when cell constant are
Molar mass of R 2 K1
incorrect.
K 2Cr2O 7 = (2 × 391
. ) + 52 × 2 + (16 × 7 ) same.
∴Enthalpy change of reaction of HCl
and Ba(OH)2 =− 56.2 × 0.1724 = 294 .20 g mol −1 R(KCl )
⇒ K(NH 4OH ) = K(KCl )
= − 9.69 kJ Molarity of R(NH 4OH )
Thus, option (a) is correct. Mass 525
K 2 Cr2 O 7 = . × 10 −3 ×
= 15
Now ∆H calorimeter = C p ∆T Molecular mass × V 2030
∆H calorimeter =
18
. = 3.88 × 10 −4 S cm−1 (NH+4 + OH − )
−1
= 435JK (T − 293.48) K . × 0.38
2942 = 73.4 + 198.6 = 272 S cm2 mol −1
= 0.016 M
= 0.435(T − 293.48) kJ Let c be the molarity of NH 4+ and OH −
∴Moles of K 2 Cr2 O 7 required to reach
Total volume of solution = 400 mL in the solution.
equivalence point
Assume density = 1 g/L 3.88 × 10 −4 × 1000
0.016 × 15 ⇒ 272 =
∴∆H solution =
1000 c
= 0.400 × 4184
. (T − 293.48) kJ . × 10 −3 M
c = 143
= 2.4 × 10 −4 mol
460 JEE Advanced~Practice Set 10

Now, A equilibrium. ⇒ W AB = nC V (TB − TA ) Structure of (A)



NH 4OH - NH+4 + OH = − 2 × 15
. R × 153.43 (pent-1-yne)
Me
. (1 − α )
010 0.01α . α
010 = − 3826.85 J
⇒ 010 . × 10 −3
. α = 143 ⇒ W BC = − p∆V = − nR∆T
Structure of (B)
. × 10 −2
α = 143 = − 2 × 8.314 × 153.43
= − 2551.23 J Me
30. (0.01) Given, initial conc. of
pC –Me (pent-1-yne)
CH 3 COOH = 0.06 M ⇒ WCA = − nRT ln
pA
∴Residual CH 3 COOH is unadsorbed [O]
2 Me
CH 3 COOH. = − 2 × 8.314 × 500 ln
10 mL of M ′ (unadsorbed) 5
COOH + HOOC –Me
CH 3 COOH = 14 mL of 0.03 M NaOH = 7618.04 J Propanoic acid Acetic acid
14 × 0.03 ∴ W total = 1239.96 J
∴ M′ = = 0.042 M H + Pt
10 32. (200) 34. (c) C →
2
C 5 H10 (Deficiency
Initial millimoles of CH 3 COOH = COCl 2( g ) - CO( g ) + Cl 2( g ) unit (DU) shows a 5 C - atom cyclic
50 × 0.06 M = 3 millimoles CH 3 COOH Initial conc. 1 0 0
compound)
Unadsorbed millimoles of At equilibrium 1 − x x x H2 + Pt
CH 3 COOH = 50 × 0.042 = 2.1millimole
2 x2 Cyclopentene
Thus, equilibrium concentration of KC = = 15
. (C)
1− x
CH 3 COOH = 2.1 millimole
∴CH 3 COOH adsorbed ⇒ x 2 + 075
. x − 075
. =0 35. (a) On proceeding reverse, following
= 3.0 − 2.1 = 0.9 millimole observations are made:
−075
. + ( 075
. ) + 3 2

0.9 x= l
Acetone obtained from (G)
= × 60 g 2 suggests that (G) is calcium
1000
= 0.568 mol acetate, (CH 3COO)2Ca .
x = 0.054 g l
Therefore, compound (D) is
Now, let the volume of flask containing
m = adsorbent = 3 g acetone (Me 2 C = O)
Cl 2( g ) = V mL and it contains y mole of
x 0.054
l
Thus, compound (F) would be
∴ = = 0.018 gas.
isopropyl alcohol (Me 2CHOH).
m 3 V 500
x = = 318.87 …(i) l
Since, (D) is obtained from (C), (C)
Now, = KC1/ 2 y 1568. would be
m Me Me
Now, let (z) mole of Cl 2 reacts back.
∴ 0.018 = K(121
/2
. )
COCl 2 - Co + Cl 2
0.018 Me Me
K = = 0.012 0.432 + z 0.568 − z 0.568 + y − z
2.1 0.432 + z
⇒ [COCl 2 ] = × 1000 = 0.694 Me Me
31. (1239.96) The change can be 500 + V (B) would be
depicted on a p –V diagram as: ⇒ 432 + 1000Z = 347 + 0.694V Me Me
OH
0.694V − 85
z= …(ii)
A 1000 Structure of (D) and (F) suggest that
p Rev. ester (A)
Also,
Rev. isothermal ( 0.568 + y − z) ( 0.568 − z)
adiabatic compression KC = 15
. =
( 0.432 + z) Me Me
expansion
should be O
1000
= …(iii) Me Me
B C ( 500 + V )
36. (c)
Solving Eqs. (i), (ii) and (iii), we have
V V = 200 mL Me Me 2 MeMgBr Me Me
Given, C V = 15
. R O
5 33. (b) (C) → White ppt. Me Me H3O+ Me
H OH
Me
C p = 2.5R and γ = [ Ag(NH 3 )2 ]+ ] H3O+
3 (B) H+
Applying adiabatic reversible (Terminal alkyne)] –H2O

conditions between A and B H2+Pt Me Me Me


(A) and (B) Me
1− γ 1−γ OH + HO
pA TA = pB TB (Indicates Me Me Me Me
γ γ straight chain (E) (F) (C)
1 −γ in A and B)
Iodoform O3/Red
p  γ
(B) ⇒ (Internal alkyne ) − Does not
⇒ TB =  A  Iodoform
 pB  react with ammonium AgNO 3 solution CHI3↓ + (MeCOO)2Ca 2Me
−2 Me (D)

=  
5 5
× 500
2
= 346.57 K
JEE Advanced~Practice Set 10 461

37. (a, b, d) We have,| f( x)| ≤ 1 3 x2 1 x2 42. (a, b, c, d)


= + − = +1
Apply LMVT in x ∈ ( 0, 1) 2 2 2 2 (a) P( E1 ) = 1 − P( RRR )
| f ′ ( x )| = | f(1) − f( 0)|  x 2
∴f( x ) =  2 + 1, x > 2
= 1 −  × ×  = 1 − 01
1 2 3
⇒ | f(1) − f( 0)| ≤ 2  5 x + 1, x ≤ 2 . = 0.9
 3 4 5
⇒| f ′ ( x )| ≤ 2 for at least one x in (0, 1) 4
Now, f(2 + ) = + 1 = 3 2 1 2
Similarly,| f ′ ( x )| ≤ 2 for at least one x 2 (b) P( E 2 ) = 3P( BRR ) = 3 × × ×
in ( − 1, 0) − 3 4 5
f(2 ) = 5(2 ) + 1 = 11
Now, g ( x ) = ( f( x ))2 + ( f ′ ( x ))2 = 02
.
∴ f( x ) is not continuous at x = 2.
For atleast one x in (0, 1) and ( − 1, 0) (c) P( E 3 ) = P[( RRR ) / ( RRR ∪ BBB)]
Now, f(2 ) = 11 , f(2 + ) = 3 P( RRR )
| f ′ ( x )| ≤ 2 and| f( x )| ≤ 1 i.e. RHL ≠ f(2 ) =
P( RRR ) + P( BBB)
⇒ ( f ′ ( x ))2 ≤ 4 and ( f( x ))2 ≤ 1 ∴RHD at x = 2 does not exist. 01
. 01
.
⇒ ( f ′ ( x ))2 + ( f( x ))2 ≤ 5 = =
40. (b, c) Let A ′( x2, y2, z2 ) be the image 2 3 4 01. + 0.4
. +
01 × ×
⇒ g ( x ) ≤ 5, for atleast one x in (− 1, 0) of A (2, 1, 6) about mirror 3 4 5
and (0, 1) x + y − 2 z = 3. Then, 01
.
= = 02
.
38. (a, b, c) Let z = x + iy, where x, y x2 − 2 y − 1 z2 − 6 0.5
= 2 =
satisfy the given equation. 1 1 −2 (d) P( E 4 ) = 1 − P( BBB) = 1 − 0.4 = 0.6
Hence, ( x 2 + y 2 ) ( x 2 − y 2 ) = 175 − 2 (2 + 1 − 12 − 3)
= =4 43. (9) We have,
12 + 12 + 2 2
⇒ x 2 + y 2 = 25 and f( x ) = x 4 + ax 3 + bx 2 + cx + d
x 2 − y2 = 7 ⇒ ( x2, y2, z2 ) = ( 6, 5, − 2 )
Given, f(2 i ) = f(2 + i ) = 0
∴ The coordinate of A′ is (6, 5, − 2).
Hence, possible value of z will be ∴ 2 i , − 2 i , 2 + i , 2 − i are the roots of
x−2 y−1 z− 6
4 + 3i , 4 − 3i , − 4 + 3i , − 4 − 3i Let = = =λ f( x ).
3 4 5
Clearly, it will form a rectangle having ∴ f( x ) = ( x − 2 i ) ( x + 2 i ) ( x − 2 − i )
length of the diagonal 10. x = 2 + 3λ, y = 1 + 4λ, z = 6 + 5λ
(x − 2 + i)
lies on plane x + y − 2 z = 3
Y ⇒ f( x ) = ( x 2 + 4) [( x − 2 )2 + 1]
⇒ 2 + 3λ + 1 + 4λ − 2 ( 6 + 5λ ) = 3
⇒ λ=−4 ⇒ f(1) = (1 + 4) ((1 − 2 )2 + 1)
z3 (–4+3i) z2 (4+3i)
∴ The coordinate of B is ⇒ f(1) = ( 5) (2 ) = 10
( − 10, − 15, − 14) Now, f(1) = 1 + a + b + c + d
X
41. (a, b, c, d) In ∆AFE, AD is angle ⇒ 10 = 1 + a + b + c + d
z4 (–4–3i) bisector of ∠A and AD is ∴a+ b+c+d =9
z1 (4–3i)
perpendicular EF.
44. (0.5) Let α = cos − 1 p,
∴D is mid-point of EF and ∆AEF is
−1
isosceles triangle β = cos 1− p
From the diagram, A −1
and γ = cos 1− q
(a) Area of rectangle = 8 × 6 = 48 sq. unit
(b) z1, z2, z3, z4 are vertices of rectangle Then, cos α = p,
and z1 + z2 + z3 + z4 = 0 E
cos β = 1 − p,
(c) Rectangle is symmetric about real axis B C
D and cos γ = 1 − q
(d) arg ( z1 − z3 ) = tan−1   or
3
 4 ⇒ sin α = 1 − p, sin β = p and
F
sin γ = q
tan−1  −  Area of ∆ABC = Area of ∆ABD
3

 4 + Area of ∆ADC We can write α + β + γ =
1 1 4
39. (a,d) For x > 2, ⇒ bc sin A = ADc sin A / 2 3π
1
2 2 ⇒ α +β= −γ
f( x ) = ∫0 (1 + |1 − t|) dt 1
+ b AD sin A / 2
4

x 2 ⇒ cos (α + β ) = cos  − γ 
+ ∫1 (1 + |1 − t|)dt A  4 
⇒ bc sin A = AD sin ( b + c )
2 cos α cos β − sin α sin β
1 x
= ∫0 {1 + (1 − t )}dt + ∫1 (1 + t − 1)dt ⇒ AD =
2 bc cos A / 2
= cos

cos γ + sin

sin γ
1 x b+c
4 4
= ∫0 (2 − t )dt + ∫1 t dt
Also, AD = AE cos A / 2 [from ∆ADE]
p 1− p − 1− p p
1 x 2 bc
 t 2 t 2  ⇒ AE = = HM of b and c 1 1
= 2t −  +   b+c =− cos γ + sin γ
 2  0  2 1 2 2
Again, EF = 2 DE = 2 AD tan A /2
   x2 1  0 = − cos γ + sin γ
=  2 −  − ( 0 − 0) + 
1 4 bc sin A / 2
−  =
  2 
2 2 b+c = − 1− q + q
462 JEE Advanced~Practice Set 10

⇒ q = 1− q 47. (25) Clearly, Q | sin 4 x| ∈ [0, 1 ]


⇒ q = 1 − q ⇒q =
1 1 + 2 + 3 + 4 + ... + n − 2 ≤ 1224 ∴ 1 + | sin 4 x| ∈ [1, 2 ]
2 ≤ 3 + 4 + 5 + ... + n ⇒ y 2 ∈ [1, 2 ]
x− x 3 ( n − 2 ) ( n − 1)
45. (1) Let h ( x) = − sin x, then ⇒ ≤ 1224 ⇒ y ∈ [1, 2 ]
n − 2
≤ 
6 2
 ( 3 + n) Thus,| sin 2 x| + | cos 2 x | ∈ [1, 2 ]
1 − 3 x2  2 
h′ ( x) = − cos x and we know| sin x | ≤ 1
6 ⇒ n2 − 3n − 2446 ≤ 0 So, solution is possible if both the
1 x 2  n2 + n − 2454 ≥ 0 sides are equal to 1.
= − + cos x  and
6 2 
⇒ 49 < n < 51 ⇒ n = 50
⇒ h′ ( x ) < 0 ∀ x > 0, n ( n + 1)
∴ − (2 k + 1) = 1224
⇒ h ( x ) is decreasing function. 2 1
Thus, x > 0 ⇒ h ( x ) < h ( 0) ⇒
50( 51)
− (2 k + 1) = 1224
x − x3 2
− sin x < 0 π/2 π 3π/2 2π
6 ⇒ 2 k + 1 = 1275 − 1224
x − x3 y = |sin x|
⇒ < sin x ⇒ k = 25
6 1 4 x 3(1 + ( x 4 )2016 ) Y
Also sin x < x, ∀ x > 0 48. (2018) Let I = ∫0 (1 + x ) 4 2018
dx
x − x3
∴ < sin x < x, ∀ x > 0 √2
6 put, x 4 = t ⇒ 4 x 3 dx = dt
 x − x3  1
1 1 + t 2016
⇒ f 
 6 
 > f (sin x ) > f ( x ) ∴ I= ∫0 (1 + t )2018 dt 0 π/4 π/2 3π/4 π X
[Q f is a positive decreasing 1 dt 1 t 2016 f(x) = |sin 2x| + |cos 2x|
function]
⇒ I= ∫0 (1 + t )2018 + ∫0 (1 + t )2018 dt From the graphs, solutions are
 x − x3  dt 1 π 3π
1 1
x=± ,±
f  
 6  f (sin x )
⇒ I= ∫0 (1 + t )2018 + ∫0 2018
dt
2 2
t  1 + 
2
⇒ > >1 1
f ( x) f ( x)  t ∴ Number of solutions are 4.

According to Sandwich theorem, −2017 


1 50. (1) We can write the given expression
 1
1 +  as follows
f (sin x )
=1  (1 + t ) 
−2017 

1 
lim t 10 2
x→ ∞ f ( x) ⇒I =   +  1

10 2n
C1 + 2n
C2
 −2017  0  2017  81 n
81 n
81n
46. (10) We have,  
 0 10 3 10 2n
R Q − C 3 + ... +
2n
−1  1
− 1
n
T ⇒I = 81 81n
2017  2 2017  1
= [ 2nC 0 − C1 10 +
2n 2n
C 2 10 2 −
1  1 81n
+ − 0
2017  2 2017  2n
C 3 10 3 + ... + 2n
C 2n 10 2n ]
S P
1  1 1  1
→ ⇒I = − + 1 + 2017 = [1 − 10]2n
PR = 3$i + $j − 2k$ 2017  2 2017 2  81n
→
SQ = i − 3 j − 4k =
1 ( −9)2n 81n
= = =1
2017 81 n
81n
1 → →
Area of base ( PQRS ) = | PR × SQ| Here, m = 1, n = 2017
2 51. (b) For common tangent
i j k ∴m + n = 1 + 2017 = 2018 a 2m 2 + b 2 = r 2 (1 + m2 )
1 1 49. (4) Let f ( x) = | sin 2 x| + | cos 2 x |
= 3 1 −2 = | − 10$i + 10$j − 10 k$| r2 − b2
2 2 ⇒ m2 =
1 −3 4 π π
Then, f  + x  = sin 2  + x  a2 − r 2
= 5 | $i − $j + k$| = 5 3 4  4 
Since, two values of m is possible and
π
+ cos 2  + x 
for each value of m two tangents are
Height = Projection of PT on i − j + k
4  possible. Hence, numbers of common
( i + 2 j + 3k ) ⋅ ( i − j + k ) tangents will be 4.
= = | cos 2 x| + | sin 2 x| = f ( x )
| i − j + k|
Thus, the function is periodic with
52. (c) In 4th quadrant slope of common
1− 2 + 3 2 π tangent is positive and intercept on
= = period .
3 3 4 Y-axis will be negative so equation
∴ Volume of parallelopiped by the will be,
Let y = | sin 2 x| + | cos 2 x |
→ → →  r2 − b2 
 x−r a −b
2 2
vectors PQ , PT and PS y 2 = sin2 2 x + cos 2 2 x + 2 | sin 2 x| y=
 a2 − r 2  a −r
2 2
 2  | cos 2 x |  
=5 3  = 10 cu units
 3 = 1 + | sin 4 x |
JEE Advanced~Practice Set 10 463

53. (d) We have, g ( x + y) = g ( x) ∴ g ( x) = x 3 − 4 x + C …(ii) 54. (a) Given function is g ( x) = x3 − 4 x


+ g ( y) + 3 xy ( x + y) …(i) Putting x = 0 = y in Eq. (i), we get For the domain of the function,
g ( x + y) = g ( x ) g ( 0) = g ( 0) + g ( 0) + 0 x3 − 4 x ≥ 0
+ g ( y) + 3 x 2 y + 3 xy 2 ⇒ g ( 0) = 0
x( x 2 − 4) ≥ 0
g ′ ( x + y) = g ′ ( x ) + 6 xy + 3 y 2 Putting x = 0 in Eq. (ii), we get
g ( 0) = C x ( x − 2 )( x + 2 ) ≥ 0
[keeping y, as constant] 0=C – + – +
Putting x = 0, we get ∴ g ( x) = x 3 − 4 x –2 0 2
g ′ ( y) = g ′ ( 0) + 3 y 2
Now, g ( x ) = 0 ⇒ x 3 − 4 x = 0 ∴ x ∈ [−2, 0] ∪ [2, ∞ )
⇒ g ′ ( y) = − 4 + 3 y 2
⇒ x ( x − 4) = 0 ⇒ x = 0, 2, − 2
2

⇒ g ′( x) = 3 x 2 − 4
Hence, the number of roots of
g ( x ) = 0 is 3.

Paper 2
1. (a,b,c) For small value of θ,
τ R = 
Work done by a constant force 3mgl 
θ q
= Force × Displacement in the  2 
direction of force I 17 ml 2 × 2
T = 2π = 2π
So, W F1 = F1 × 2 R = 20 × 2 × 6 K 12 × 3mgl e–
= 120 2 J
17 l
W F2 = F1 × R = 30 × 6 = 180 J = 2π
18 g
F3 is a variable in direction only, so work The potential of the grounded sphere
done in small tangential displacement 17 l
T = 2π has to be zero, due to particle charge
W = ∫ F3 ds = ∫ F3R dθ = F3R ( π /2 ) 18 g the potential of the sphere is positive,
= 15 × 6 × π / 3 = 45 πJ so negative charge must be
3. (b, c)
possessed by sphere, so that total
2. (a,c) The idea is that by comparing the potential of sphere becomes zero.
Moment of inertia for AB rod about A u sin θ and u cos θ of two projectile This negative charge is acquired by
is ml 2 / 3 and of rod CD about A is motions, we can solve this question
sphere from the ground, hence
A easily.
current flows into the ground. As, the
Slope at t = 0 gives initial vertical charged particle comes nearer to
speed. So, sphere the potential of sphere due to
 dy  <  
dy positive charge increases and hence
 
 dt  1 at t = 0  dt  2 at t = 0 more electron per unit time will flow
B form ground to sphere and thus
C D u sin θ1 < u sin θ 2
magnitude of current increases.
Ml 2 13 ml 2 θ1 < θ 2 (angle of projection)
= + ml 2 = 5. (b,d)
12 12 θ1 < θ 2
For reflection at general point A,
17 ml 2
So, cos θ1 > cos θ 2 y
Total moment of inertia is
12 u cos θ1 > u cos θ 2
∴ From graph, we can see that
i
θ T1 > T2
i
So, u cos θ1T1 > u cos θ 2T2
(x, y)
R1 > R 2
i
mg
4. (a,c) x

mg R v q 2 i = 90°
So, i = 45°
Restoring force about A,
l So, dy /dx at A should be 45°
τ R = mg sin θ + mglsin θ e–
πx
sin  
2L
2 y=
3 mgl π  L 
= sin θ
2
464 JEE Advanced~Practice Set 10

πx π πx
cos   × = 2 cos   mv 2 = 2  mv 02  + kx 2
dy 2 L 8. (2.00) 1 1 1
=
dx π  L  L  L  Apply Bernoulli’s equations at pipe 2 2  2
xπ (1), (2) and (3),
1 = 2 cos   ⇒ mv 2 = 2 mv 02 + kx 2
 L  1 1
p3 + ρv 32 = p2 + ρv 22 = p1 + ρ/2 v12
πx π ⇒ mv 2 − kx 2 = 2 mv 02
= 2 2
L 3 1 ρ mv 2 − kx 2
p0 + ρgh3 + ρv 32 = p0 + ρgh2 + v 22 ⇒ v0 =
3L 2 2 2m
⇒ x = L / 3, y = ρ
π = p0 + ρgh1 + v12 1× 4−2 ×1
2 =
and another point will be the just 2 ×1
v 32 v 22 v12
opposite point, so its coordinates will gh3 + = gh2 + = gh1 + …(i) v 0 = 1 m/s
3L 2 2 2
be 2 L / 3, . 10. (2.00)
π Equations of continuity at (3) and (2) are
v 3 A3 = v 2 A2 This problem is based on Moseley’s
6. (c)
observation can be mathematically
For toppling, v3 × A = v2 × 2 …(ii)
expressed as
F v 2 × 2 = v1 × 1 …(iii)
ν = a (Z − b )
From Eq. (i) , we get
where ν is frequency, Z is atomic
N v 22 v12
gh2 + = gh1 + number and a, b are constants. This
2 2 relation is known as Moseley’s law.
fs
35 v2 20 v2 According to Moseley’s law, K α X-rays
10 × + 2 = 10 × + 1
O 100 2 100 2 emission
mg 15 v12 − v 22
= ν = a ( Z − 1)
τ0 = 0 10 2 ⇒ ( Z − 1)2 ∝ ν
Fh = mgr 4v 22 − v 22
= or ( Z − 1)2 ∝
1
mgr 2 λ
F =
h 3v 22 (ZMo − 1) 2 λ Cu
= ⇒ =
For slipping, 2 (Z Cu − 1) 2 λMo
F = f smax = µ mg v 2 = 1 m/s, v1 = 2 m/s
( ZMo − 1)2
For slipping before toppling, From Eq. (i), we get ⇒ λ Cu = λMo
( Z Cu − 1)2
mgr v 32 v2
≥ µmg gh3 + = gh2 + 2 2
= 0.71 ×  
h 2 2 41
r v2  28 
⇒ µ≤ = 10 ×
37.5
+ 3
h 100 2 = 1.52 Å
r
µ max = 35 1
h = 10 × + λ = 2Å
100 2
7. (1.00) 11. (4.00)
v 32 1 35 37.5
The problem is based on thermal = + − Let the element be X.
2 2 10 10
conductivity of the material λα
1 2.5 1 25 X → Y + α
∆Q A (T1 − T2 ) T1 − T2 = − = −
=k = 2 10 2 100 λβ
∆t x x / kA ⇒ X → Z + β
v 32 1 1
where, k is constant for the material of = ⇒ v 32 =
2 4 2 When a substance decays by α and
the slab and x / kA is a thermal β-emissions simultaneously, the
1
resistance. The idea is that by ⇒ v3 = ,X =2
average rate disintegration λ overall is
comparing the power produced by 2
given by
heater and rate of heat flow through 9. (1.00)
window, we can find area λ overall = λ α + λ β
As, the collision is elastic the first
power produced by heater = rate of where, λ α = disintegration constant
block will transfer its speed completly
heat flow through window. to second block. So, initial scenerio for α-emission only
V 2 Temperature difference will be like this and λ β = disintegration constant for
∴ =
R Thermal resistance v′=0 v β-emission only.
(20 − Q ) V 2l 1
= ⇒ Q = 20 −
m m m Mean life is given by τ =
l / kA RkA λ
⇒ At maximum compression 1
Substituting the values, we get v0 v0 (The common ⇒ λ overall = λ α + λ β =
( τ )overall
. × 10 −2 )
(200)2 × ( 02 speed at maximum
Q = 20 − m m compression)
=
1
+
1
. × 42
02 . × A × 20 τα τβ
. × 10 −2 )
(200)2 × ( 02 Let the maximum compression is x.
⇒ 1524
. = 20 − =
1
+
1
. × 42
( 02 . ) × A × 20 So, by mechanical energy 1600 400
conservation,
⇒ A = 10004
. m2 ≈ 1m2 = 3.125 × 10 −3 yr −1
JEE Advanced~Practice Set 10 465

N0 v2 = √2gl So, slope of tangent will be 90° − i.


λ overall t = 2.303 log10
Nt mg dy 2 2
= tan ( 90° − i ) = cot i =
100 T2 v1 = √4gl dx 1
⇒ (3.125 × 10 −3 )t = 2.303 log10
25 dy
At y = h , = 2x
1 dx
⇒ t = 2.303 × log10 4 T1
3.125 × 10 −3 2x = 2 2 , x = 2 m
At horizontal position,
= 443.5 yr y = h = ( 2 )2 = 2 m
mv12
Number of significant digit = 4 T1 = = 4 mg …(i) 15. (b)
l
12. (1.00) Whenever magnetic flux is cut by a
At highest point,
Here, velocity of efflux at point 2 is rotating rod, the emf will be induced in
given by v = 2 gy and as condition is mv 22
T2 + mg = = 2 mg the rod, i.e. given by
given in l 1
E induced = Bω l 2
dy T2 = mg …(ii) 2
1 A vg = dt
From above equation, we get where, B is magnetic field, ω is
T1 angular velocity and l is the length.
y =4
dx T2 For torque, we will use τ ext = Iα
2 vx =
dt 14. (2.00) where, α = angular acceleration,
h T=
2h I is moment of inertia and for the total
g
1 i1 charge, we will use
x
Q = ∫ i dt
i2
question that block should move in 2 where, i is instantaneous current.
such a way that stream always hits the Let ω be the angular velocity at time t.
dx H2 H1 i3
block. So, velocity of block, v b = . 3
dt
H3 Q ω
Also, the velocity at point 1 will be rate
dy P O
of decrease in level of water, i.e. .
dt Consider the rays as shown in figure,
Besides these concept, we have to for ray 1 with angle of incidence i 1
use equation of continuity. ( i 1 < θc ), where (θc is critical angle) the The induced emf across OP (or OQ)
2
ray will get refracted, but H1 is not the Bω l 2
Velocity of efflux, v = 2 gy Bω   =
1 l
minimum height. For ray 3 with angle =
2h 2 2 8
Range, x = 2 gy × of incidence i 3 ( i 3 > θc ) the ray will get
g Bω l 2
reflected (due to phenomenon of TIR), Current, i = ...(i)
The velocity of the block must be  
dx 8R
now for light ray 2 with angle of
 dt 
incidence i 2 ( i 2 = θc ) the ray will just Torque on the rod
dx 2h 1 dy
∴ vb = = × 2g × refracted (limiting ray) and l/ 2 l/ 2
dt g 2 y dt correspondingly H2 will be minimum = 2∫ Bixdx = 2 Bi ( x 2 / 2 )0
0
h dy height, so that light will just refracted. l 2 Bil 2
vb = ⋅ …(i) = 2 Bi × = ...(ii)
y dt For TIR at that point of incidence on 8 4
curved surface. We can easily say that
Using equation of continuity, Substituting the value of i from Eq. (i)
as h increases, i decreases and for
Ady
= a 2 gy TIR at the point of incidence minimum Bl 2  Bω l 2  B2ω l 4
…(ii) ⇒ Torque =   =
dt value of i should be θc and 4  8R  32 R
By Eqs. (i) and (ii), we get corresponding to that h will be
Now, we will have
h a minimum for refraction.
vb = × 2 gy Ml 2  dω  − B2ω l 4
y A   =
12  dt  32 R
a 1
v b = 2 gh × = 20 × = 1 ms − 1 y = x2 ω dω 3 B2l 2 t
A 20 i ⇒ ∫ω 0 ω 8 RM ∫0
= ⋅ dt
13. (4.00)
h 3 B 2l 2
For vertical circular motion, student ⇒ ln(ω /ω 0 ) = − t
8 RM
should always remember that
⇒ ω = ω 0e − α t
v ′ = √(N – 4) gl At just TIR by Snell’s law,
v = √(N – 2) gl 3 B 2l 2
1 where, α =
3 sin i = 1 sin 90° sin i = 8 RM
3

Q= ∫0 i dt
v = √(N – 2) gl i Bω 0l 2
u = √Ngl =
90°– i 8Rα
For given, u = 6gl 90°– i
466 JEE Advanced~Practice Set 10

 Bω 0l 2   8 R M  ω 0M 18. (b) A - r, B - q, C - s and D - p Since, it reacts with only one mole of


=  2 2  =
 8 R   3B l  3B Velocity of particle, MeI, so it contains only one —OH
dr group.
1 2 Ml 2ω 20 v = = 3 t 2$i + 2t$j + k$
Heat generated = Iω 0 = dt Vanillin is not steam distilled which
2 24 suggests that — OH and — CHO
v (t = 1s ) = 3$i + 2 $j + k$
16. (b) Here, we have use first law of groups are not in o-position.
thermodynamics, i.e. ∆Q = ∆U + ∆W | v | (at t = 1s) = 14 ms − 1 C 8H 8O 3 [CHO OH C 6H 4 ]
where, ∆V = nC V ∆T Acceleration of particle at t = 1s; It also has OCH 3 group.
∆W = pdV a (at t = 1s) = 6$i + 2 $j The reactions are as follows:
Again, gas equation will be used, i.e.
⇒ | a | at t = 1s = 6 ⋅ 32 ms − 2 CHO CHO
pV = nRT
dx Now, radius of curvature, –
Velocity, v = OH KMnO4
dt v3 v3 14 14 MeI
R = = = ≈ 6m OMe OMe
dv d 2 x |a × v| 76 76
and acceleration, a = = 2
dt dt Centripetal acceleration is OH OMe
Vanillin (A )
Heat produced by coil inside the v 2 14
| ac | = = = 2.33 ms − 2 COOH COOH
cylinder in time dt is dQ = I 2rdt . R 6
(A) From first law of thermodynamics Tangential acceleration is Conc.
p0 A a t = a 2 − a c2 HI
OMe OH
49
= 40 − OMe OH
9

mg = 5.88 ms − 2 COOH
pA
∆Q = ∆V + ∆W 19. (a) [Ag(NH3)2]+
⇒ I 2rdt = C VdT + pdV 2KMnO 4 → K 2 MnO 4 + MnO 2 + O 2
(A) (B) (C) (D) OMe
dT dT
⇒ I 2r = C V +R
dt dt Hence, A = KMnO 4 and C = MnO 2 OH
dT 20. (c) ‘A’ is insoluble in dil. HNO 3. ‘A’ (B)
= Cp
dt belongs to group II and is HgS Br
As, T = T0 + at + bt 2 because HgS is only insoluble in
dT dil. HNO 3. 2Br2
= a + 2 bt –CO2
dt ( A ) + Conc . 3HCl + Conc .HNO 3
14444 4244444 3 (ipso Br OMe
1/ 2 Aqua -regia attack)
I= (2 bt + a )
5R
⇒ → HgCl 2 OH
 2 r 
( B) (C)
HgCl 2 +2NaOH → HgO ↓
(B) pV = RT
Yellow ppt.(C) 22. (b) Formation of triacetate shows the
⇒ pdV = RdT presence of only three —OH groups.
+ 2NaCl + H 2O
⇒ pAdx = RdT Oxidation with Br2 − water shows the
HgCl 2 + 2KI → HgI 2 ↓ + 2KCl
dx R dT presence of a —CHO group.
Velocity, v = = ⋅ (D) - orange
dt pA dt Reduction with HI to n - pentane,
HgI 2 + 2KI → K 2HgI 4
R shows the presence of a straight
= (2 bt + a ) (E) - soluble
pA chain of 5 carbons.

17. (a) A - q, B - r, C - s, D - p NH+4 + NaOH → NH 3 Formation of only 1 molecule of CH 2O
+ and 1 molecule of HCOOH suggests
Radius of orbit, + H 2O + Na
that its structure is as given a side.
 n2  3 NaOH + NH3 + 2K2HgI4 CHO
rn = 0.53  Å
Z Hg Brown ppt.
H OH
O NH2I ↓ (iodide of Million's base)
Speed of electron, Hg +4Ki + 3 NaI + 2H2O H H
v n = 2.2 × 10 6   ms − 1
Z H OH
 n HgCl 2 + H 2S → HgS ↓ + 2HCl
( B) (Black)
CH2OH
µ i
Magnetic field at centre, Bn = 0 n 21. (c) Vanillin gives colour with FeCl 3 ,
2 rn 23. (a,b,c,d)
which shows presence of a phenolic
πm2e 7µ 0  Z 3  group. 2 RT 8RT
= ⋅  (a) Ump = = uav =
8ε30h5  n5  Reaction with Tollen’s reagent M pM

me 5 Z2 [Ag(NH 3 )2 ] + suggests the presence of 3RT


Current = ef = ⋅ 3 Urms =
4 πε0h  n 
2 3 a — CHO group in it. M
JEE Advanced~Practice Set 10 467

Thus, ump < uav < urms is correct. ∴ [H + ] = 317


. × 10 −12 Now, similarly mole of C 2H 6 actually
3 RT − required
(b) (K.E)av = . It is independent of ⇒ [OH ] in left half - cell
2 No 2 × 55 100 100
Kw 10 −14 = × × = 2.479 mol
amounts, thus correct. = +
= 58 85 90
3 [H ] . × 10 −12
317 Therefore, volume of C 2H 6 at NTP
(c) K .E = RT per mol
2 . × 10 −3
= 315 actually required
2 RT [C 5 H10 NH+2 ] [OH − ] 22.4 × 2.479 L = 55.53 L
⇒ Ump = ∴K b =
M [C 5H10 NH ] 30. (67.36) Let the sample contain x
2
(Ump )M mmol of K 3[Fe(C 2O 4 )3 ] ⋅ 3H 2O and y
∴ = RT . × 10 −3 )
( 0.05) ( 315
2 = . × 10 −3
= 158 mmol of FeCl 3 ⋅ 6H 2O.
( 01
.)
3(Ump )2 M In first titration meq. of MnO −4 used up
∴K.E = is correct. 27. (1.09) In bcc structure body diagonal for 20 mL
4
= 4r(Fe) = 3a
3RT = 375
. × 0.005 × 5 = 0.09375
(d) Urms = 4
a= = r(Fe) ⇒ Total meq. of oxalate present in
M 3
sample
(Urms )2 M z = 2 atoms per unit cell
⇒ = RT = 0.09375 × 5 = 0.46875
3 Mz
∴d (α -form) = ∴Total mmol of oxalate present
3 3
K.E = RT N 0a
0.46875
2
56 × 2 = = 0234375
.
2
3 Urms = 3
2
= M  4r  ⇒ mmol of K 3[Fe(C 2O 4 )3 ] .3H 2O,
2 3 ( 6.02 × 10 23 )  
 3 i.e. x
(Urms )2
= M is correct. In fcc structure face diagonal 1
= × 0.234375
2
= 4r(Fe) = 2 a ′ 3
24. (a) From graphs I and II, a ′ = 2 2 r(Fe) = 78125
. × 10 −3
∆Tb = (Tb − Tb ° ) = 105 − 100 = 5 K
z = 4 atoms per unit cell ⇒ Mass of K 3[Fe(C 2O 4 )3 ] ⋅ 3H 2O
Q ∆Tb = K b × m
∴d (r - form ) =
Mz = 78125
. × 10 −3 × 491 × 10 −3
∴ K b = 5 K × 1 mol Kg −1 N0( a ′ )3
= 38.36 × 10 −3 g
K b = 5 mol K kg −1 56 × 4
= ∴Mass percentage of
From graphs I and III ( 6.02 × 10 23 ) (2 2 r )3
K 3[Fe(C 2O 4 )3 ] ⋅ 3H 2O
∆Tb = (Tb − Tb ° ) = 110 − 100 = 10 K Thus, density ratio of r-form to α - form 38.36 × 10 −3
3 = × 100
 4  0.32
K b × wY × 1000 2× r
Q ∆Tb =  3  = 12 %
WX × M Y =
(2 2 r )3 Also, mmol of Fe 2+ (total)
1000 × 5 × 10
∴M Y = = 10887
. ≈ 109
. = x + y = 17.5 × 0.005 × 5 × 2 = 0.875
10 × 100
28. (5) There are five chiral centers in the y = 0796875
.
= 50 g mol −1
given molecule (estrogen). ⇒ Mass of FeCl 3 ⋅ 6H 2O
25. (4) H 3O+ , OSF2, SO 32 and NF3 have OH
270.5
pyramidal shape. = 0.796875 ×
H 1000
26. (1.58) Half-cell reactions can be
writlen as: = 02155
. g
H2 - −
2H + 2e , E° = 0
+
H H Thus, mass percentage of FeCl 3 ⋅ 6H 2O
HO present in original sample
Hg 2Cl 2 +2e − - 2Hg + 2Cl − , 02155
.
29. (55.53) Given reactions are: = × 100 = 67.36%
E° = 0242
. V 0.32
Overall cell reaction (i) 2C 2H 6 + 2Br2 → 2C 2H 5Br + 2HBr
31. (4.04) E f − E b = − 35 kJ
Hg 2Cl 2 + H 2 - 2H+ + 2Hg + 2Cl − (ii)
2C 2H 5Br + 2Na → C 4H10 + 2NaBr ⇒ E f = 155 − 35 = 120 kJ / mol
,
At 300 K, t 1/ 2 for the forward reaction
E° = 0242
. V Mole ratio of reactions (i) and (ii)
is 2 hr.
According to Nernst equation, C 2H 6 : C 2H 5Br : C 4H10 :: 2 : 2 : 1 ln2 0.693
0.0591 [H + ]2 55 Kf = = . hr −1
= 01
Ecell = E °cell − log Now, mole of butane formed = t 1/ 2 6.93
n pH 2 58
E af
⇒ log A f = log K f +
0.0591 [H + ]2 Mole of C 2H 5 Br actually required
2.303RT
∴ 0.902 = 0242
. − log
2 pH 2 2 × 55 100 120 × 1000
= × ∴log A f = − 1 +
58 85 2.303 × 8.314 × 300
[H + ]2 0.66 × 2
⇒ log =− = − 22.30 (∴reaction efficiency is 85% )
02. 0.0591 = 19.9
468 JEE Advanced~Practice Set 10

OH OCH3
⇒ A f = 8 × 1019 hr −1 34. (a)
Kf NH2 EtO2C
Also, =K NaOH
P. O C + CMe (R)
Kb CH3Cl
NH2 H2C
−1
A
= f e RT ( E f − E b ) OCH3 OCH3
Ab
Me Br +
Cl
∆H H2SO4 Cl2
A − Heat HN Br2 HN
= f e RT AcOH Me HNO3 AlCl3
Ab N N
 A  ∆H H H NO2 NO2
or ln K = ln  f  −
 Ab  RT C5H5N (i)
C  HBr OH
 A  35000
. = ln  f  −
⇒ ln 016 NaOH, H2O2
 A b  8.314 × 300 (S)
Br (Dakin’s reaction)
Af HN CHO
∴ = 199
. × 10 5 Me
Ab OH
N
8 × 1019 −1 H
⇒ Ab = = 4.02 × 10 hr
14
. × 10 5
199 NH2 EtO2C
Q. O C + CMe (ii) OH
32. (0.96) + –
Oxidation NH2 NaOHC
OH
3CH3CH2OH+2Cr2O72–+16H+ 3CH3COOH OH
+4Cr3++11H2O Me Me CHCl3
HN (T)
CH2ClCO2H HN NaOH
Reduction CHO
S N N
H H
∴ E °cell = E °red − E °oxi
OH
∴ E°cell = 133
. − 0.06 = 127
. V NH2 HO2C
H2O2/OH–
Reaction quotient R. O C + CHMe
NH2 CN OH
(ii)
3 3+ 4 11
[CH 3COOH] [Cr ] [H 2O] CO
(q ) = Me NO2 NO2
[CH 3CH 2OH]3 [Cr2O 72− ]2 [H+ ]16 Ac2O HN CHMe H HN
2 + NH3 –+
1 1 Pt Cl2 CH3ONa
(q ) = = = 10 64 C CN N (U)
[H + ]16 (10 −4 )16 NH2 H AlCl3
Cl
(∴pH = 4 given)
35. (c) NO2 NO2
0.0591
∴ Ecell = E °cell − log 10 64 NO2 NO2
12
NaOH Cl2/AlCl3
0.0591
= 127
. − × 64 log 10 (P) 150ºC
12 OCH3 OCH3
Cl Cl Cl O–
= 0.955 ≈ 0.96 V Cl
NO2 (iii)
33. (a) P → ii, v ; Q → i,iii ; R → ii, iii, v ;
S → ii, iv CH3Cl 36. (b) i →(q, r, u) ; ii → (s) ; iii → (p) ;
K 2CrO 4 - Transition element in iv → (s, t)
+6 oxidation state and manufactured Cl OCH3
from chromite ore. (iv)
KMnO 4 - Baeyer’s reagent, oxidising
NO2 NO2 [A]t–1
agent in acidic medium. p
Cl2,
K 2Cr2O 7 - Transition element in AlCl3
+6 oxidation state. (Q)
Cl O Time (t)
l
Oxidising agent in acidic medium.
l
Manufactured from chromite ore. NO2 NO2
This corresponds to second order
K 2MnO 4 - Transition element in reaction
+6 oxidation state. Br2 130ºC
–+ 1 1 1 
FeBr3 CH3ONa k= −
t  [ A ]t [ A ]0 
l
Manufactured from pyrolusite.
Cl Cl
Br OCH3 1 1
= kt +
(i) [ A ]t [ A ]0
JEE Advanced~Practice Set 10 469

1 It refers to first order reaction. d ⋅ (a + b ) d ⋅ (b + c )


Thus, OP = = 0.5 =− −
[ A ]0 Thus, (iv) → (s,t) [a b c ] [a b c ]
∴ [ A ]0 = 2.0 x n r d ⋅ (a + c )

∴ rate = k[ A ]20 = 4 K 37. (a, b, c) We have, m y r = 0 [a b c ]
m n z − 2 ⋅ [a ⋅ d + b ⋅ d + c ⋅ d ]
For second order reaction, rate is =−
changed to four times on doubling the [a b c ]
Applying, R1 → R1 − R 2 and
concentration.
R 2 → R 2 − R 3, we get [Qd ⋅ (a + b + c) = 0]
log   = log k + nlog[A ] t
dx ∴ sin x + cos y + 2 = 0
x−m n− y 0
 dt  ⇒ sin x + cos y = − 2
0 y−n r− z =0
Q y = mx + c
m n z ⇒ sin x = − 1, cos y = − 1
∴ OP = 0.3010 = log k π
⇒ ( x − m) ( y − n ) z + ( n − y ) ⇒ x=− ,y= π
⇒ k=2 2
Half-life of second order reaction ( r − z)m − n( r − z) ( x − m) = 0
[Q we want the minimum
=
1
=
1
=1 Dividing by ( x − m) ( y − n) ( z − r ), value of x 2 + y 2]
k[ A ]0 2 × 0.5 we get
z m n π2
Thus, (i) → (q,r,u) + + =0 ∴ x 2 + y2 = + π2
z−r x−m y−n 4
(ii)
z m n 5π2
⇒ + + 1+ + 1= 2 =
z−r x−m y−n 4
log [dx/dt]

P z x y 39. (a, c, d) Circle


⇒ + + =2
z−r x−m y−n x 2 + y 2 − 4 x − 16 y + 64 = 0
x y z centre (2, 8), radius r = 2
Time (t) ∴ + + =2
x−m y−n z−r
Parabola = y 2 = 4 x
Graph is for logarithm form of x y z
or −1+ − 1+ −1 Normal to the parabola
 dx  = k[ A ] n x−m y−n z−r
  t y = mx − 2 m − m3
 dt  =2 − 3
For shortest distance from the centre
log  ) = log k + nlog[ A ] t
dx m n r
⇒ + + = −1
 dt x−m y−n z−r normal must passes through the
centre of circle
y = mx + c Now, AM ≥ GM
x y z 8 = 2 m − 2 m − m3
∴ OP = C , log k = 0.3010 + +
⇒ k=2 x−m y−n z−n ⇒ m= −2
Rate ⇒ 4 = k[A ] = 2[2 ] = 4 n n 3 ⇒ Normal at P, y = − 2 x + 12
1/ 3
 xyz 
⇒ n=1 ≥ 
( x − m )( y − n ) ( z − r ) S (2, 8)
Thus, (ii) → (s)  
3
(iii) ⇒ 2 ≥ xyz
  Q y2=4x
 3 ( x − m) ( y − n)( z − r )
∴ Greatest value of P (4, 4)
[(Ao – (A)t]

P xyz 8
is
( x − m) ( y − n) ( z − r ) 27

Time (t) 38. (b, c) Here,


d ⋅ a = {(a × b )sin x + (b × c ) cos y
If initial concentration = a = [ A ]0
+ 2 (c × a )} ⋅ a
After time t , concentration
[Q(a × b ) ⋅ a = 0, (c × a ) ⋅ a = 0]
= ( a − x) = [ A ] t
= [a b c ] cos y …(i) ∴Point P ≡ ( am2, − 2 am) ≡ ( 4, 4)
∴ x = [A ]0 − [ A ] t
Also, d ⋅ (a + b + c ) = 0 ∴ SP = ( 4 − 2 )2 + ( 4 − 8)2 = 2 5
For zero order reaction, x increases
d ⋅ a = − d ⋅ (b + c ) …(ii)
with time t ∴ SQ : QP = 2 : (2 5 − 2 )
dx From Eqs. (i) and (ii), we get 1
∴ = k[ A ]0 = k − d ⋅ (b + c ) Slope of tangent at Q =
dt cos y = 2
Thus, (iii) → (p) [abc ]
The intercept of the normal
(iv) d ⋅ (a + b )
Similarly, sin x = − y = − 2 x + 12 is 6.
[a b c ]
40. (a, b, c, d) We have,
d ⋅ (a + c )
−2 =
Log [A] t

P
and f( x ) = x 2e −|x|
[a b c ]
Now, sin x + cos y + 2  x 2e − x , x≥ 0
⇒ f( x ) =  2 x
xe , x< 0
Time (t)
470 JEE Advanced~Practice Set 10

e − x (2 x − x 2 ), x≥ 0 42. (a, b, d) We have, x 2 − y 2 = 1


⇒ f ′( x) =  x 2
 e ( x + 2 x ), x< 0
Y r
l
f( x ) increases in ( − ∞, − 2 ) ∪ ( 0, 2 ) P (x1, y1)
and f( x ) decreases in ( − 2, 0) ∪ (2, ∞ ). r
∴ y = f( x ) has two point of maxima. r p x
Now, C1 C2
X′ X 4 10
e − x ( x 2 − 4 x + 2 ), x ≥ 0 O M N (x2, 0) C3
f ′ ′( x) =  x 2
 e ( x + 4 x + 2 ), x < 0
Y
–2, 4/e 2 2, 4/e 2
Y′

X′ X As shown in figure,
–2 –1 0 1 2 Now, C1C 3 = r − 4
Hyperbola and circle touch at P( x1, y1 ).
C 2C 3 = r − 10
Equation of tangent to hyperbola C1C 2 = 2 r − 14 = 14
Y′ x 2 − y 2 = 1 at ( P( x1, y1 ) is r = 14
f ′ ′ ( x ) = 0 has four roots. xx1 − yy1 = 1 Now, PC 3 = x = 14 − 8 = 6
Hence, there are four points of  1  ∴ l 2 = r 2 − x2
If cuts the X-axis at M  , 0 .
inflections.  x1  l 2 = (14)2 − ( 6)2 − 160
f ′ ( x ) = 0 has three roots i.e. ( − 2, 0, 2 ) Now, centroid of ∆PMN is ( l, m). ⇒ l = 4 10
Clearly, from graph y = f( x ) has only 1
x1 + x2 + 8 10
one asymptote. x1 ∴ 2l =
So, l = 1
41. (a, c, d) We have, 3
∴ a = 8, b = 10, c = 1
s− x s− y s− z x12 − 1
= = and m=
y1
= ∴ a + b + c = 8 + 10 + 1 = 19
4 3 2 3 3
3 s − ( x + y + z) s 45. (0.5) Let L = lim n2 ∫
1/ n 2018x + 1
x dx
= =  dy   dy 
4+3+ 2 9 Now, `  =  n→ ∞
 dx  H at P  dx  S at P 1/ n 2018x + 1
⇒ x=
5s
,y=
2s
,z=
7s
x1 x − x1 ⇒ L = lim
∫0 x dx
9 3 9 ⇒ = 2
y1 y1
n→ ∞ 1 / n2

(a) Area of incircle = πr 2 = 1
+1
3 ⇒ x2 = 2 x1  1
2018
n −1
  .
⇒ ∆ 2 = s 2r 2 3 x1 +
1  n n2
⇒ L = lim
x1 1 n→ ∞ 2
8s 2 ∴ l= = x1 + −
⇒ ∆2 = 3 3 x1 n3
3
dl 1 [Q apply L’ Hospital rule]
8s 2 ⇒ = 1−
⇒ s( s − x ) ( s − y) ( s − z) = dx1 3 x12  1
2018n
⋅    − 2 
1 1
3  
dm x1  n  n  n 
4s s 2 s 8s 2 ⇒ = ⇒ L = lim
⇒s. . . = dx1 3 x 2 − 1 n→ ∞ −2
9 3 9 3 1
dm 1 n3
⇒ s=9 ⇒ = 2018
dy1 3 1  1  n
8 ⇒ L= lim    
∴∆ = × 9 = 6 6 sq units 2 n→ ∞
3 43. (121) Consider two sets 1, 3, 5, 7, 9  n 
1 
5s 2 s 7 s … (i) 1
× × ⇒ L=
1
× 1 = Q lim ( x )x = 1
xyz and 0, 2, 4, 6, 8 … (ii)
(b) R = = 9 3 9 2 2  x → 0+ 
4∆ 4×6 6 Required number of ways = (any two 
35 from set (i) + any two from set (ii)) 46. (7) For idempotent matrix A 2 = A
=6
24 (excluding zero)) 3! +
( I + A )n = nC 0I + nC1 A + nn2 A 2
X Y Z r 0 along with any one from set (ii) ⋅ 4 +
(c) sin sin sin =
2 2 2 4R all four alike
+ nC 3 A 3 + ... + nC n A n
8/ 3 4 = ( 5C 2 + 4C 2 )3 ! + 4C1 ⋅ 4 + 9
= =
35 35 = (10 + 6) × 6 + 4 × 4 + 9
4 6 = nC 0I + nC1 A + nC 2 A + nC 3 A + ...
24 = 96 + 16 + 9 = 121
X + Y + nC n A
(d) sin2   44. (19) Radius of circle C1 = 4
 Z  = I + A ( C1 + C 2 + C 3 + ... + nC n )
n n n

Z 1 + cos Z 3 Radius of circle C 2 = 10


∴ cos 2 = = ⇒ I + 127 A = I + A (2 n − 1)
2 2 5 Radius of circle C 3 = r
JEE Advanced~Practice Set 10 471

⇒ 127 = 2 n − 1 49. (10) It is given that α, β are the roots 51. (b) Total number of songs = 10
⇒ 2 = 128 = 2
n 7 of equation Number of classic songs = 6
1 − 8 (log10 x )2 and number of western songs = 4
⇒ n=7 =1
log10 x − 2 [log10 x ]2 P. Required number of ways
47. (17) Let the given point is A (1, 1,1) and
Let log10 x = y = 6C 3 × 4C 2 × 5 ! × 5 !
the plane is P : x + y + z = 1 and the
1 − 8 y2
given line is, So, =1 6×5×4 4×3
x−2 y−3 z−4 y − 2( y)2 = × × 5! × 5!
L: = = 3×2 ×1 2 ×1
1 2 3 1 − 8 y2 = y − 2 y2
= 5 × 4 × 3 × 2 × 5! × 5!
Suppose a line parallel to L passing ⇒ − 8y + 2 y − y + 1 = 0
2 2
= ( 5 !)3
through A intersects the plane P at B, ⇒ − 6 y2 − y + 1 = 0
then we have to find distance AB. Q. Required number of ways
⇒ 6 y2 + y − 1 = 0
Let’s find equation of AB. Now, AB = 6C1 × 9 !
passes through A (1, 1, 1) and is 6 y2 + 3 y − 2 y − 1 = 0
= 6 × 9!
parallel to L. So, direction ratios of AB 3 y (2 y + 1) − 1 (2 y + 1) = 0
R. Required number of ways
will be same as L, i.e. < 1, 2, 3 >. 3 y − 1 = 0, 2 y + 1 = 0
= 7C1 × 6 ! × 4 !
Hence, equation of AB is 1 −1
y = and y =
x−1 y−1 z−1 3 2
= = .
1 2 3 1 1
log10 x = − and log10 x =
Now, coordinates of any point on line 2 3 C C C C C C
x−1 y−1 z−1 −
1 1
AB : = = = k are of x = 10 or x = 2 10 3 = 7 × 6! × 4! = 7 ! × 4!
1 2 3
1 S. Required number of ways
type ( k + 1, 2 k + 1, 3k + 1). x = 1/ 2 or x = 101/ 3 10 ! 10 !
Also, point B( k + 1, 2 k + 1, 3k + 1) lies 10 = = = 10P4
1 6 ! (10 − 4)!
on the plane P : x + y + z = 1. So, α = 1/ 2 β = 101/ 3
∴ k + 1 + 2 k + 1 + 3k + 1 = 1 10 52. (c) We have,
2
2 m3 − 3m2 − 3m + 2 = 0
−2 −1 Now, α 2β 3 + 1 =  1/ 2  ⋅ (101/ 3 )3 + 1
1
⇒ B  , , 0
2 1
k= =
6 3 3 3   10  ⇒ 2 m3 − 3m( m + 1) + 2 = 0
1
Hence, the length = × 10 + 1 ⇒ 1 + 1 = 2 ⇒ 2( m3 + 1) − 3m( m + 1) = 0
2 2
10
AB =  2  +  1  + 12 = 14 ∴ (α 2β 3 + 1)2 = 2 2 = 4 ⇒ ( m + 1) [2 ( m2 + 1 − m) − 3m] = 0
   
 3  3 3
Again, α 4 = 10 − 2 ⇒ ( m + 1) [2 m2 + 2 − 2 m − 3m] = 0
∴ Here a = 14, b = 3 ;∴a + b = 17 1 1 ⇒ ( m + 1) [2 m2 − 5m + 2 ] = 0
⇒ α4 = or = 100
48. (4) Domain of f( x) is [− 1, 1] 100 α4 ⇒ ( m + 1) (2 m − 1) ( m − 2 ) = 0
 (α 2β 3 + 1)2 
f( x ) = sin− 1 x + 2 tan− 1 x + x 2 + 4 x + 1 Now,  − 30000 α 4  ⇒
1
m = − 1, m = , m = 2
 α 4
 2
1 2 1  2 1  Also, we have equation of line which
f ′( x ) = + + 2x + 4 = (2 ) 100 − 30000 ×
1 + x2 10  100  can be written as, mi2 x − mi y = − 1
1− x 2

1 100 x y
1 2 = [400 − 300] = = 10 ⇒ + =1
= + + 2(x + 2) 10 10 1 1
1+ x 2 − 2
1 − x2 mi mi
50. (6) The graph of y = sin { x} and
∀ x ∈ [− 1, 1], f ′ ( x ) > 0 y = cos { x} are shown below; P. Clearly, algebraic sum of the
∴f( x ) is increasing function. Y intercepts made by the lines on
∴ p = minimum value of f( x ) X-axis
= − ∑ 2 = −  1 + 4 + 
1 1
= sin− 1( − 1) + 2 tan− 1( − 1) + ( − 1)2 mi  4
+ 4 ( − 1) + 1 1 21
y = sin{x} =−
π − π − 2 1/√2 4
=− +2  
2  4 0.5 Q. Clearly, algebraic sum of the
y = cos{x}
π π intercepts made by the lines on
=− − −2 = − π − 2 X
2 2 1 1
O 1 2 3 4 5 6 2π Y-axis = Σ = − 1+ 2 +
and q = maximum value of f( x ) mi 2
Points of intersection are,
= sin− 1(1) + 2 tan− 1(1) + 1 + 4 + 1 = 1+
1 3
=
π π π
π π x = π / 4, 1 + , 2 + , 3 + , 2 2
= + 2   + 6 = π + 6 4 4 4
2  4 π π R. Let d i ( i = 1, 2, 3) denotes the
4+ ,5+ perpendicular distance of the line
∴So, range of f( x ) is [− π − 2, π + 6] 4 4
from the origin, respectively.
∴p + q = π + 6 − π − 2 = 4 i.e. numbers of solutions are 6.
472 JEE Advanced~Practice Set 10

53. (c) We have, S. If a = 2, b = 2 then,


−1 π π
Then, d i = =
1 −1
sin ( ax ) + cos ( y) + cos−1 −1
( bxy) = sin−1 2 x + cos −1 y + cos −1 2 xy =
2 2
1
+
1 mi2 +1
mi4 mi2 P. If a = 1, b = 0 then, ⇒ cos −1 2 x − cos −1 y = cos −1 2 xy
mi4 π
−1 −1 −1
sin x + cos y + cos ( 0) = ⇒ 2 xy + (1 − 4 x 2 ) 1 − y 2 = 2 xy
mi2 2
= π π
1 + mi2 sin−1 x + cos −1 y + = = ( 4 x 2 − 1) ( y 2 − 1) = 0
2 2
Q cos −1 0 = π   x 2, x< 0
Required sum 54. (d) f2( f1 ) = 
1 1/ 4 4  2 
2x
e , x≥ 0
= + +
1+ 1 1 + 1/ 4 1+ 4 ⇒ sin−1 x + cos −1 y = 0 f4 : R → [0, ∞ )
x< 0
sin−1 x = − cos −1( y) f4 ( x ) =  2 1
1 1/ 2 4 ⇒ f ( f ( x )),
= + +
2 5 5 f (
2 1 f ( x )) − 1 x≥ 0
⇒ x + y2 = 1
2

1 1 4 Q. If a = 1 and b = 1, then  x 2, x< 0


= + + =  2x
2 2 5 5 −1 −1 π −1 e − 1, x≥ 0
sin x + cos y + cos xy =
1 2
= (5 2 + 5 + 8 5 )
10 ⇒ cos −1 x − cos −1 y = cos −1 xy
y=f1(x)
=
1
(5 2 + 9 5 ) ⇒ cos −1 ( xy + 1 − x2 1 − y2 )
10 = cos −1 xy
S. Let l i denotes the length of line ⇒ xy + 1− x 2
1 − y = xy
2
intercepted between the coordinate
axes. Then, ⇒ (1 − x 2 )(1 − y 2 ) = 0
2 2 ⇒ ( x 2 − 1)( y 2 − 1) = 0 y=f3(x)
 1   1
li =  2 +   R. If a = 1, b = 2 then,
 i 
m  mi  π
sin−1 x + cos −1 y + cos −1 2 xy =
1 1 2
= 1+ 1 + 16 + 4 + +
16 4 ⇒ cos −1 x − cos −1 y = cos −1 2 xy
5 ⇒ xy + 1 + x2 1 − y 2 = 2 xy y=f4(x)
= 2 +2 5+
4
⇒ 1 − x2 1 − y 2 = xy
(4 2 + 9 5 )
= ⇒ (1 − x )(1 − y 2 ) = x 2 y 2
2
4
⇒ x 2 + y2 = 1
JEE Advanced~Solved Paper 2019 1

JEE ADVANCED
SOLVED PAPER 2019
Duration : 3 Hours Max. Marks . 360

Paper 1
PHYSICS
Section 1 (Maximum Marks : 12)
l This section contains FOUR (04) questions.
l Each question has FOUR options. ONLY ONE of these four options is the correct answer.
l For each question, choose the option corresponding to the correct answer.
l Answer to each question will be evaluated according to the following marking scheme:
Full Marks : + 3 If ONLY the correct options is chosen.
Zero Marks : 0 If none of the options is chosen. (i.e., the question is unanswered)
Negative Marks : - 1 In all other cases.

40
1. In a radioactive sample, 19 K nuclei either decay 4P(T(t ) - T0 )4 4P(T(t ) - T0 )3
(a) (b)
40 b 4T05 b 4T04
into stable 20 Ca nuclei with decay constant
4 .5 ´ 10 - 10 per year or into stable 18
40
Ar nuclei with 4P(T(t ) - T0 ) 4P(T(t ) - T0 )2
(c) (d)
b 4T02 b 4T03
decay constant 0 .5 ´ 10 - 10 per year. Given that in
40 40
this sample all the stable 20 Ca and 18 Ar nuclei are 3. A thin spherical insulating shell of radius R carries
produced by the 40
19 K nuclei only. In time t ´ 10 9 a uniformly distributed charge such that the
40 potential at its surface is V0 . A hole with a small
years, if the ratio of the sum of stable 20 Ca and
area a 4 pR 2 (a << 1) is made on the shell without
40 40
18 Ar nucleito the radioactive 19 K nuclei is 99, the affecting the rest of the shell. Which one of the
value of t will be following statements is correct?
[Given: In 10 = 2.3] (a) The ratio of the potential at the center of the shell to that
(a) 9.2 (b) 1.15 1
of the point at R from center towards the hole will be
(c) 4.6 (d) 2.3 2
1- a
.
2. A current carrying wire heats a metal rod. The wire 1 - 2a
provides a constant power (P) to the rod. The metal (b) The potential at the center of the shell is reduced by
rod is enclosed in an insulated container. It is 2 aV0 .
observed that the temperature (T) in the metal rod (c) The magnitude of electric field at the center of the shell is
æ 1ö aV
reduced by 0 .
changes with time (t) as T(t) = T0 ç1 + bt 4 ÷,
ç ÷ 2R
è ø (d) The magnitude of electric field at a point, located on a
where, b is a constant with appropriate dimension line passing through the hole and shell’s centre, on a
distance 2R from the center of the spherical shell will be
while T0 is a constant with dimension of aV
temperature. The heat capacity of the metal is reduced by 0 .
2R
2 JEE Advanced~Solved Paper 2019

4. Consider a spherical gaseous cloud of mass density particle number density n(r) = r(r) / m is [G is
r(r) in free space where r is the radial distance from universal gravitational constant]
its center. The gaseous cloud is made of particles of K K
(a) (b)
equal mass m moving in circular orbits about the 6pr 2 m2G pr 2 m2G
common center with the same kinetic energy K. 3K K
(c) (d)
The force acting on the particles is their mutual pr 2 m2G 2 pr 2 m2G
gravitational force. If r(r) is constant in time, the

Section 2 (Maximum Marks : 32)

l This section contains EIGHT (08) questions.


l Each question has FOUR options. ONE OR MORE THAN ONE of these four option(s) is (are) correct options(s).
l For each question, choose the options(s) corresponding to (all) the correct answer(s).
l Answer to each question will be evaluated according to the following marking scheme:
Full Marks : + 4 If only (all) the correct option(s) is (are) chosen.
Partial Marks : + 3 If all the four options are correct but ONLY three options are chosen.
Partial Marks : + 2 If three or more options are correct but ONLY two options are chosen, both of which are
correct.
Partial Marks : + 1 If two or more options are correct but ONLY one option is chosen and it is a correct option;
Zero Marks : 0 If none of the option is chosen (i.e., the question is unanswered);
Negative Marks : - 1 In all other cases.
l
For example, in a question, if (A), (B) and (D) are the ONLY three options corresponding to correct answers, then
choosing ONLY (A), (B) and (D) will get + 4 marks
choosing ONLY (A) and (B) will get + 2 marks
choosing ONLY (A) and (D) will get + 2 marks
choosing ONLY (B) and (D) will get + 2 marks
choosing ONLY (A) will get + 1marks
choosing ONLY (B) will get + 1marks
choosing ONLY (D) will get + 1marks
choosing no option (i.e., the question is unanswered) will get 0 marks; and
choosing any other combination of options will get - 1mark.

5. A cylindrical capillary tube of 0.2 mm radius is (b) For case I, if the capillary joint is 5 cm above the water
made by joining two capillaries T1 and T2 of surface, the height of water column raised in the tube
different materials having water contact angles of will be more than 8.75 cm. (Neglect the weight of the
0º and 60º, respectively. The capillary tube is water in the meniscus).
dipped vertically in water in two different (c) The correction in the height of water column raised in the
configurations, case I and II as shown in figure. tube, due to weight of water contained in the meniscus,
Which of the following option(s) is(are) correct? will be different for both cases.
[Surface tension of water = 0 .075 N / m, density of (d) For case II, if the capillary joint is 5 cm above the water
water = 1000 kg / m 3 , take g = 10 m / s 2 ] surface, the height of water column raised in the tube
will be 3.75 cm. (Neglect the weight of the water in the
meniscus).
T2 T1
6. A conducting wire of parabolic shape, initially
Case I Case II
y = x 2 , is moving with velocity v = v 0 $i in a
T1 T2
æ b
æyö ö
non-uniform magnetic field B = B 0 ç1 + ç ÷ ÷ k$ , as
ç è L ø ÷ø
è
(a) For case I, if the joint is kept at 8 cm above the water
shown in figure. If V0 , B 0 , L and b are positive
surface, the height of water column in the tube will be constants and Df is the potential difference
7.5 cm. (Neglect the weight of the water in the developed between the ends of the wire, then the
meniscus). correct statement(s) is/are
JEE Advanced~Solved Paper 2019 3

y (a) Work done in this thermodynamic cycle


B 1
(1 ® 2 ® 3 ® 4 ® 1) is| W | = RT0 .
2
L (b) The ratio of heat transfer during processes 1 ® 2 and
Q 5
2 ® 3 is 1 ® 2 = .
Q2 ® 3 3
(c) The above thermodynamic cycle exhibits only isochoric
V=V0 ^i and adiabatic processes.
(d) The ratio of heat transfer during processes 1 ® 2 and
Q 1
3 ® 4 is 1 ® 2 = .
Q3 ® 4 2
x
0 √L
4
9. In the circuit shown, initially there is no charge on
(a)| Df| = B0 V0 L for b = 2 the capacitors and keys S1 and S2 are open. The
3
values of the capacitors are C1 = 10 mF, C2 = 30 mF
(b)| Df| remains the same if the parabolic wire is replaced and C3 = C4 = 80 mF.
by a straight wire, y = x initially, of length 2L
1 S1 C4 5 V 30 Ω
(c)| Df| = B0 V0 L for b = 0 P
2
(d)| Df| is proportional to the length of the wire projected on
the y-axis. S2

7. A thin convex lens is made of two C1 C2 C3


materials with refractive indices n1 and 30 Ω
n 2 , as shown in the figure. The radius of n1 n2 70 Ω
curvature of the left and right spherical 10 V
surfaces are equal. f is the focal length of Q 100 Ω
the lens when n1 = n 2 = n. The focal
length is f + Df when n1 = n and Which of the statement(s) is/are correct?
n 2 = n + Dn. Assuming Dn << (n - 1) and 1 < n < 2, (a) The key S1 is kept closed for long time such that
the correct statement(s) is/are capacitors are fully charged. Now, key S 2 is closed, at
Dn Df this time, the instantaneous current across 30 W resistor
(a) If < 0 then >0 (between points P and Q) will be 0.2 A (round off to 1st
n f
decimal place).
. , Dn = 10- 3 and f = 20 cm, the value of| Df | will
(b) For n = 15 (b) If key S1 is kept closed for long time such that capacitors
be 0.02 cm (round off to 2nd decimal place). are fully charged, the voltage across the capacitor C1
Df Dn will be 4V.
(c) <
f n (c) At time t = 0, the key S1 is closed, the instantaneous
Df Dn current in the closed circuit will be 25 mA.
(d) The relation between and remains unchanged if (d) If key S1 is kept closed for long time such that the
f n
both the convex surfaces are replaced by concave capacitors are fully charged, the voltage difference
surfaces of the same radius of curvature. between points P and Q will be 10 V.

8. One mole of a monatomic ideal gas goes through a 10. A charged shell of radius R carries a total charge Q.
thermodynamic cycle, as shown in the volume Given f as the flux of electric field through a closed
versus temperature (V - T) diagram. The correct cylindrical surface of height h, radius r and with its
center same as that of the shell. Here, center of the
statement(s) is/are [R is the gas constant]
cylinder is a point on the axis of the cylinder which
V is equidistant from its top and bottom surfaces.
3 2 Which of the following option(s) is/are correct?
2V0
[Î0 is the permittivity of free space]
V0 (a) If h > 2 R and r = 4R / 5 then f = Q / 5 Î0
4 1
(b) If h > 2 R and r = 3R / 5 then f = Q / 5 Î0
T (c) If h < 8R / 5 and r = 3R / 5 then f = 0
T0 /2 T0 3T0 /2 2T0 (d) If h > 2 R and r > R then f = Q/ Î0
4 JEE Advanced~Solved Paper 2019

11. Two identical moving coil galvanometers have 10 W (c) If the ideal cell is replaced by a cell having internal
resistance of 5W then the measured value of R will be
resistance and full scale deflection at 2mA current. more than 1000 W.
One of them is converted into a voltmeter of 100 (d) The measured value of R will be
mV full scale reading and the other into an 978 W < R < 982 W.
ammeter of 1 mA full scale current using
appropriate resistors. These are then used to 12. Let us consider a system of units in which mass
measure the voltage and current in the Ohm’s law and angular momentum are dimensionless. If
experiment with R = 1000 W resistor by using an length has dimension of L, which of the following
ideal cell. Which of the following statement(s) statement(s) is/are correct?
is/are correct? (a) The dimension of force is [L ]- 3 .
(a) The resistance of the voltmeter will be 100 kW. (b) The dimension of power is [L ]- 5 .
(b) The resistance of the ammeter will be 0.02 W. (c) The dimension of energy is [L ]- 2 .
(round off to 2nd decimal place). (d) The dimension of linear momentum is [L ]- 1.

Section 3 (Maximum Marks : 18)


l This section contains SIX (06) questions. The answer to each question is a NUMERICAL VALUE.
l Four each question, enter the correct numerical value of the answer using the mouse and the on-screen virtual
numeric keypad in the place designated to enter the answer. If the numerical value has more than two decimal places,
truncate/round-off the value to TWO decimal places.
l Answer to each question will be evaluated according to the following marking scheme:
Full Marks : + 3 If ONLY the correct numerical value is entered.
Zero Marks : 0 In all other cases.

13. A parallel plate capacitor of capacitance C has 15. A block of weight 100 N is suspended by copper
spacing d between two plates having area A. The and steel wires of same cross-sectional area 0 .5 cm 2
region between the plates is filled with N dielectric
layers, parallel to its plates, each with thickness, and length 3 m and 1 m, respectively. Their other
d ends are fixed on a ceiling as shown in figure. The
d = . The dielectric constant of the m th layer is
N angles subtended by copper and steel wires with
æ mö ceiling are 30º and 60º, respectively. If elongation
K m = K ç1 + ÷. For a very large N(> 10 3), the in copper wire is (Dl C) and elongation in steel wire
è Nø
Dl
æ Ke A ö is (DlS), then the ratio C is ……… .
capacitance C is a ç 0 ÷. DlS
è d ln 2 ø
The value of a will be .............. . [Young’s modulus for copper and steel are
[Î0 is the permittivity of free space.] 1 ´ 10 11 N / m 2 and 2 ´ 10 11 N/m 2 respectively.]

14. A planar structure of length L and width W is 60° 30°


made of two different optical media of refractive Steel wire Copper wire
indices n1 = 15. and n 2 = 1.44 as shown in figure. If 1m √3 m
L >> W, a ray entering from end AB will emerge
from end CD only if the total internal reflection Block
condition is met inside the structure. For L = 9 .6 m,
if the incident angle q is varied, the maximum time 16. A liquid at 30ºC is poured very slowly into a
taken by a ray to exit the plane CD is t ´ 10 - 9 s, Calorimeter that is at temperature of 110ºC. The
where, t is .............. boiling temperature of the liquid is 80ºC. It is
[Speed of light, c = 3 ´ 10 8 m / s] found that the first 5 gm of the liquid completely
evaporates. After pouring another 80 gm of the
n2
A C liquid the equilibrium temperature is found to be
Air 50ºC. The ratio of the latent heat of the liquid to its
n1 W
θ specific heat will be ............. ºC.
B n2 D
[Neglect the heat exchange with surrounding]
L
JEE Advanced~Solved Paper 2019 5

17. A particle is moved along a path 18. A train S1, moving with a uniform velocity of
AB-BC-CD-DE-EF-FA, as shown in figure, in 108 km/h, approaches another train S2 standing
presence of a force F = (ay$i + 2ax$j) N, where x and on a platform. An observer O moves with a
y are in meter and a = - 1 Nm - 1 . The work done uniform velocity of 36 km/h towards S2, as shown
in figure.
on the particle by this force F will be ..............
Joule. 108 km/h
S2
y
S1
800 m
A B
1.0

600 m

36 km/h
D
0.5 C
O
F E
x Both the trains are blowing whistles of same
0 0.5 1.0
frequency 120 Hz. When O is 600 m away from S2
and distance between S1 and S2 is 800 m, the
number of beats heard by O is ……… .

CHEMISTRY
Section 1 (Maximum Marks : 12)
Λm CMC
Instructions: Same as given in Physics. Λm
(a) (b)
19. The correct order of acid strength of the following CMC
carboxylic acids is
O
O √C √C
OH
I H II H
OH CMC
H Λm Λm CMC
O (c) (d)
III MeO IV H3C OH
OH O
(a) III > II > I > IV (b) I > II > III > IV
√C √C
(c) II > I > IV > III (d) I > III > II > IV
20. The green colour produced in the borax bead test Section 2 (Maximum Marks : 32)
of a chromium (III) salt is due to
Instructions: Same as given in Physics.
(a) Cr 2 O 3 (b) CrB (c) Cr(BO 2 )3 (d) Cr2 (B 4O 7 )3
21. Calamine, malachite, magnetite and cryolite, 23. Choose the reaction(s) from the following options,
respectively, are for which the standard enthalpy of reaction is
(a) ZnCO 3 , CuCO 3 , Fe 2O 3 , Na 3 AlF6 equal to the standard enthalpy of formation.
(b) ZnSO 4 , CuCO 3 , Fe 2O 3 , AlF3 (a) 2C(g ) + 3 H2 (g ) ¾® C 2H6 (g )
(c) ZnSO 4 , Cu(OH)2 , Fe 3O 4 , Na 3 AlF6 (b) 2H2 (g ) + O 2 (g )¾® 2H2O(l )
(d) ZnCO 3 , CuCO 3 × Cu(OH)2 , Fe 3O 4 , Na 3 AlF6 3
(c) O 2 (g ) ¾® O 3 (g )
2
22. Molar conductivity (L m) of aqueous solution of 1
sodium stearate, which behaves as a strong (d) S8 (s ) + O 2 (g ) ¾® SO 2 (g )
8
electrolyte, is recorded at varying concentrations (C)
of sodium stearate. Which one of the following 24. A tin chloride Q undergoes the following reactions
plots provides the correct representation of micelle (not balanced)
formation in the solution? Q + Cl - ¾® X
(critical micelle concentration (CMC) is marked Q + Me3N ¾® Y
with an arrow in the figures) Q + CuCl 2 ¾® Z + CuCl
6 JEE Advanced~Solved Paper 2019

X is a monoanion having pyramidal geometry. 29. Fusion of MnO 2 with KOH in presence of O 2
Both Y and Z are neutral compounds. produces a salt W. Alkaline solution of W upon
Choose the correct option(s). electrolytic oxidation yields another salt X. The
(a) There is a coordinate bond in Y manganese containing ions present in W and X,
(b) The central atom in Z has one lone pair of electrons respectively, are Y and Z. Correct statement(s) is
(c) The oxidation state of the central atom in Z is + 2 (are)
(d) The central atom in X is sp3 hybridised
(a) Both Y and Z are coloured and have tetrahedral shape
25. In the decay sequence. (b) Y is diamagnetic in nature while Z is paramagnetic
- x1 - x2 - x3
238
92 U ¾¾
¾® 234
90 Th ¾¾
¾® 91 Pa ¾ ¾
234
¾® (c) In both Y and Z, p-bonding occurs between p-orbitals of
- x4 oxygen and d-orbitals of manganese
Z ¾¾ ¾® 234 230
90 Th
(d) In aqueous acidic solution, Y undergoes
x1 , x 2 , x 3 and x 4 are particles/radiation emitted by disproportionation reaction to give Z and MnO 2
the respective isotopes. The correct option(s)
is(are) 30. Each of the following options contains a set of four
(a) Z is an isotope of uranium molecules. Identify the option(s) where all four
(b) x2 is b - molecules posses permanent dipole moment at
(c) x1 will deflect towards negatively charged plate room temperature.
(d) x3 is g-ray (a) SO 2 , C 6H5Cl, H2Se, BrF5 (b) BeCl 2 , CO 2 , BCl 3 , CHCl 3
(c) NO 2 , NH3 , POCl 3 , CH3Cl (d) BF3 , O 3 , SF6 , XeF6
26. Which of the following statement(s) is(are) correct
regarding the root mean square speed (U rms) and Section 3 (Maximum Marks : 32)
average translational kinetic energy (Eav) of a
Instructions: Same as given in Physics.
molecule in a gas at equilibrium?
(a) Urms is inversely proportional to the square root of its 31. Among B 2H 6, B 3N 3H 6, N 2O, N 2O 4, H 2S2O 3 and
molecular mass
H 2S2O 8 , the total number of molecules containing
(b) Urms is doubled when its temperature is increased four
times covalent bond between two atoms of the same
(c) Eav is doubled when its temperature is increased four
kind is .............
times 32. On dissolving 0.5 g of a non-volatile non-ionic solute to
(d) Eav at a given temperature does not depend on its
molecular mass 39 g of benzene, its vapour pressure decreases from
650 mmHg to 640 mmHg. The depression of
27. Choose the correct option(s) for the following set freezing point of benzene (in K) upon addition of
of reactions.
the solute is ............
(i) MeMgBr Conc. HCl
C6H10O Q S (Given data : Molar mass and the molal freezing
(ii) H2O
(major)
point depression constant of benzene are 78 g mol - 1
20 % H3PO4, 360 K
. K kg mol - 1 , respectively).
and 512
(i) H2, Ni HBr, benzoyl peroxide
T R U
(major) (ii) Br2, hν (major) ∆ 33. Consider the kinetic data given in the following
(major)
CH3 H C Br H3 C Br CH3 table for the reaction A + B + C ¾® Product
3
Cl Cl
(b) Rate of
(a)
Experiment [A] [B] [C] reaction
S T U S No. (mol dm- 3 ) (mol dm- 3 ) (mol dm- 3 ) (mol dm- 3s - 1 )
CH3
CH3 H3 C Br
H3C Cl
Br 1 0.2 0.1 0.1 6.0 ´ 10 - 5
Br
(c) (d)
2 0.2 0.2 0.1 6.0 ´ 10 - 5
U T
S U
3 0.2 0.1 0.2 . ´ 10 - 4
12
28. Which of the following statement(s) is(are) true? 4 0.3 0.1 0.1 9.0 ´ 10 - 5
(a) The two six-membered cyclic hemiacetal forms of
D-(+ ) -glucose are called anomers
(b) Oxidation of glucose with bromine water gives glutamic The rate of the reaction for [ A] = 0 .15 mol dm - 3 ,
acid [B] = 0 .25 mol dm - 3 and [C] = 0 .15 mol dm - 3 is
(c) Monosaccharides cannot be hydrolysed to given
polyhydroxy aldehydes and ketones
found to be Y ´ 10 - 5 mol dm - 3s - 1 . The value of Y
(d) Hydrolysis of sucrose gives dextrorotatory glucose and is ............
laevorotatory fructose
JEE Advanced~Solved Paper 2019 7

34. For the following reaction, the equilibrium Scheme 1


NH2
. ´ 10 .
constant K c at 298 K is 16 17
(i) Br2(excess), H2O
2+ 2- (ii) NaNO2, HCl, 273 K
Fe (aq) + S (aq) - FeS (s) (iii) CuCN/KCN
2+ (iv) H3O+, ∆
Q
When equal volumes of 0 .06 M Fe (aq) and (v) SOCl2, pyridine
(major)

0 .2 M S2 - (aq) solutions are mixed, the equilibrium P


concentration of Fe2 + (aq) is found by Y ´ 10 - 17 M. Scheme 2
The value of Y is ........... (i) Oleum
(ii) NaOH, ∆
35. At 143 K, the reaction of XeF4 with O 2F2 produces S
(iii) H+
(iv) Br2, CS2, 273 K (major)
a xenon compound Y. The total number of lone R
pair(s) of electrons present on the whole molecule
of Y is ............ Scheme 3
(i) NaOH, ∆
36. Schemes 1 and 2 describe the conversion of P to Q S T
(ii) Q
and R to S, respectively. Scheme 3 describes the (major)
synthesis of T from Q and S. The total number of Br
atoms in a molecule of T is ..............

MATHEMATICS
Section 1 (Maximum Marks : 12) (a) 6 £ m < 8 (b) - 3 £ m < - 1
(c) 4 £ m < 6 (d) 2 £ m < 4
Instructions: Same as given in Physics.
40. The area of the region {(x , y) : xy £ 8 ,1 £ y £ x 2} is
37. Let S be the set of all complex numbers z satisfying 14 7
| z - 2 + i | ³ 5. If the complex number z 0 is such (a) 8 loge 2 - (b) 8 loge 2 -
3 3
1 14
that is the maximum of the set (c) 16 loge 2 - (d) 16 loge 2 - 6
| z 0 - 1| 3

ì 1 ü Section 2 (Maximum Marks : 32)


í : z Î Sý, then the principal argument of
î| z - 1| þ Instructions: Same as given in Physics.
4 - z 0 - z0
is 41. Let G denote a curve y = y(x) which is in the first
z 0 - z0 + 2 i
p 3p p p
quadrant and let the point (1, 0) lie on it. Let the
(a) (b) (c) - (d) tangent to G at a point P intersect the y-axis at YP .
4 4 2 2
If PYP has length 1 for each point P on G, then
é sin 4 q - 1 - sin 2 q ù -1 which of the following options is/are correct?
38. Let M = ê ú = a I + bM ,
ë1 + cos q
2
cos 4 q û (a) xy¢ + 1 - x2 = 0

where a = a (q) and b = b(q) are real numbers, and I (b) xy¢ - 1 - x2 = 0
is the 2 ´ 2 identity matrix. If a * is the minimum æ1 + 1 - x2 ö÷
(c) y = loge ç - 1 - x2
of the set {a (q): q Î[0 , 2 p)} and b * is the minimum ç x ÷
è ø
of the set
æ1 + 1 - x2 ö÷
{b(q) : q Î[0 , 2 p)}, then the value of a * + b * is (d) y = - loge ç + 1 - x2
ç x ÷
(a) -
17
(b) -
31
(c) -
37
(d) -
29 è ø
16 16 16 16
42. Define the collections {E1 , E2 , E3 , K} of ellipses and
39. A line y = mx + 1 intersects the circle {R1 , R 2 , R 3 , K} of rectangles as follows :
(x - 3) + (y + 2) = 25 at the points P and Q. If the
2 2
x 2 y2
E1 : + =1
midpoint of the line segment PQ has x-coordinate 9 4
3
- , then which one of the following options is R1 : rectangle of largest area, with sides parallel to
5 the axes, inscribed in E1 ;
correct?
8 JEE Advanced~Solved Paper 2019

x2 y2 2 $ $
(a) r = (2 i - j + 2 k$ ) + t (2 $i + 2 $j - k$ ), t Î R
En : ellipse + = 1 of the largest area inscribed 9
a 2n bn2
1
in R n - 1 , n > 1; (b) r = (2 $i + k$ ) + t (2 $i + 2 $j - k$ ), t Î R
3
R n : rectangle of largest area, with sides parallel to 2 $ $ $
the axes, inscribed in En , n > 1. (c) r = (4 i + j + k ) + t(2 $i + 2 $j - k$ ), t Î R
9
Then which of the following options is/are correct?
(d) r = t(2 $i + 2 $j - k$ ), t Î R
(a) The eccentricities of E18 and E19 are NOT equal.
5 46. There are three bags B1, B 2 and B 3. The bag B1
(b) The distance of a focus from the centre in E9 is .
32 contains 5 red and 5 green balls, B 2 contains 3 red
N
and 5 green balls, and B 3 contains 5 red and 3
(c) å(area of Rn ) < 24, for each positive integer N. green balls. Bags B1 , B 2 and B 3 have probabilities
n =1
3 3 4
1 , and respectively of being chosen. A bag is
(d) The length of latusrectum of E9 is 10 10 10
6
selected at random and a ball is chosen at random
43. In a non-right-angled triangle DPQR, let p, q , r from the bag. Then which of the following options
is/are correct?
denote the lengths of the sides opposite to the
(a) Probability that the chosen ball is green, given
angles at P , Q , R respectively. The median from R 3
meets the side PQ at S, the perpendicular from P that the selected bag is B3 , equals .
8
meets the side QR at E, and RS and PE intersect at (b) Probability that the selected bag is B3 , given that
O. If p = 3, q = 1, and the radius of the 5
the chosen ball is green, equals .
circumcircle of the DPQR equals 1, then which of 13
the following options is/are correct? (c) Probability that the chosen ball is green
1 39
(a) Length of OE = equals .
6 80
7 (d) Probability that the selected bag is B3 and the
(b) Length of RS = chosen ball is green equals .
3
2 10
3
(c) Area of DSOE = é0 1 a ù é-1 1 - 1ù
12
ê
47. Let M = 1 2 3 and adj M = ê 8 - 6
ú 2ú
(d) Radius of incircle of DPQR =
3
(2 - 3) ê ú ê ú
2 êë 3 b 1 úû êë - 5 3 - 1úû
44. Let a and b be the roots of x 2 - x - 1 = 0, with where a and b are real numbers. Which of the
a > b. For all positive integers n, define following options is/are correct?
a n - bn (a) det(adj M 2 ) = 81
an = , n ³ 1,
a -b é aù é 1ù
b1 = 1 and bn = a n - 1 + a n + 1 , n ³ 2 (b) If M êb ú = ê2 ú, then a - b + g = 3
ê ú ê ú
Then which of the following options is/are correct? êë g úû êë 3úû
¥
b 8 (c) (adj M )- 1 + adj M - 1 = - M
(a) å 10nn =
89 (d) a + b = 3
n =1

(b) bn = a + b for all n ³ 1


n n
48. Let f : R ® R be given by
(c) a1 + a2 + a3 + K + an = an + - 1for all n ³ 1
¥
2
ì x 5 + 5 x 4 + 10 x 3 + 10 x 2 + 3 x + 1, x < 0;
a 10
(d) å nn = ï
n =1 10 89 ïï x - x + 1,
2
0 £ x < 1;
f (x) = í 2 3 8
x - 4x + 7x - ,
2
1 £ x < 3;
45. Let L1 and L 2 denote the lines ï 3 3
r = $i + l(- $i + 2 $j + 2 k$), l ÎR ï (x - 2) log (x - 2) - x + 10 , x ³ 3;
ïî e
3
and r = m(2 $i - $j + 2 k$), m ÎR
Then which of the following options is/are correct?
respectively. If L 3 is a line which is perpendicular (a) f is increasing on (- ¥, 0)
to both L1 and L 2 and cuts both of them, then (b) f¢ is NOT differentiable at x = 1
which of the following options describe(s) L 3 ? (c) f is onto
(d) f¢ has a local maximum at x = 1
JEE Advanced~Solved Paper 2019 9

Section 3 (Maximum Marks : 18) 51. Let w ¹ 1 be a cube root of unity. Then the
Instructions: Same as given in Physics. minimum of the set {| a + bw + cw2 |2 : a , b, c
distinct non-zero integers} equals ............
49. Let S be the sample space of all 3 ´ 3 matrices with p/ 4
2 dx
entries from the set {0, 1}. Let the events E1 and E2 52. If I =
p ò
- p/ 4 (1 + ) (2 - cos 2 x)
sin x
be given by e

E1 = { A Î S : det A = 0 } and then 27 I 2 equals .........


E2 = { A Î S : sum of entries of A is 7}. 53. Three lines are given by r=l$i , l Î R,
If a matrix is chosen at random from S, then the r=m($i + $j), m Î R and r=n($i + $j + k$), n Î R
conditional probability P(E1 | E2) equals ...........
Let the lines cut the plane x + y + z = 1 at the points
50. Let the point B be the reflection of the point A(2, 3) A , B and C respectively. If the area of the triangle
with respect to the line 8 x - 6 y - 23 = 0 . Let GA ABC is D then the value of (6 D)2 equals ..........
and GB be circles of radii 2 and 1 with centres A
and B respectively. Let T be a common tangent to 54. Let AP(a ; d) denote the set of all the terms of an
the circles GA and GB such that both the circles are infinite arithmetic progression with first term a
on the same side of T. If C is the point of and common difference d > 0. If
intersection of T and the line passing through A AP(1 ; 3) Ç AP(2 ; 5) Ç AP(3 ; 7) = AP(a ; d)
and B, then the length of the line segment AC is
then a + d equals ................
...........

Paper 2
PHYSICS
Section 1 (Maximum Marks : 32)
l
This section contains EIGHT (08) questions.
l
Each question has FOUR options for correct answer(s). ONE OR MORE THAN ONE of these four option(s) is (are)
correct options(s).
l
For each question, choose the correct options(s) to answer the question.
l
Answer to each question will be evaluated according to the following marking scheme:
Full Marks : + 4 If only (all) the correct option(s) is (are) chosen.
Partial Marks : + 3 If all the four options are correct but ONLY three options are chosen.
Partial Marks : + 2 If three or more options are correct but ONLY two options are chosen, both of which are correct
options.
Partial Marks : + 1 If two or more options are correct but ONLY one option is chosen and it is a correct option.
Zero Marks : 0 If none of the options is chosen (i.e. the question is unanswered).
Negative Marks : - 2 In all other cases.
l
For example: in a question, if (A), (B) and (D) are the ONLY three options corresponding to correct answer, then
choosing ONLY (A), (B) and (D) will get +4 marks; choosing ONLY (A) and (B) will get +2 marks;
choosing ONLY (A) and (D) will get +2 marks; choosing ONLY (B) and (D) will get +2 marks;
choosing ONLY (A) will get +1mark; choosing ONLY (B) will get +1mark;
choosing ONLY (D) will get +1mark;
choosing no option (i.e. the question is unanswered) will get 0 marks; and
10 JEE Advanced~Solved Paper 2019

1. An electric dipole with y 4. A block of mass 2M is 2M


p attached to a massless a1
dipole moment 0 ($i + $j) is
2 spring with
B A
held fixed at the origin O spring-constant k.
45° 45°
This block is
in the presence of a O x
R connected to two a2
uniform electric field of a3
other blocks of masses
magnitude E0 . If the M and 2M using two M
potential is constant on a massless pulleys and
circle of radius R centered 2M
strings. The
at the origin as shown in accelerations of the blocks are a 1 , a 2 and a 3 as
figure, then the correct statement(s) is/are, shown in the figure. The system is released from
(Î0 is the permittivity of the free space, rest with the spring in its unstretched state. The
R >> dipole size) maximum extension of the spring is x 0 . Which of
(a) The magnitude of total electric field on any two points of the following option(s) is/are correct?
the circle will be same. [g is the acceleration due to gravity. Neglect
®
(b) Total electric field at point B is E B = 0 friction]
1/ 3 (a) a2 - a1 = a1 - a3
æ p0 ö
(c) R = ç ÷ x0
è 4p Î0 E0 ø (b) At an extension of of the spring, the magnitude of
4
® 3g
(d) Total electric field at point A is E A = 2 E0 ($i + $j ) acceleration of the block connected to the spring is .
10
2. A small particle of v 4Mg
(c) x0 =
mass m moving L k
inside a heavy, x
(d) When spring achieves an extension of 0 for the first
hollow and straight tube along the tube axis 2
undergoes elastic collision at two ends. The tube time, the speed of the block connected to the spring is
has no friction and it is closed at one end by a flat 3g
M
.
surface while the other end is fitted with a heavy 5k
movable flat piston as shown in figure. When the
distance of the piston from closed end is L = L 0 , the 5. A thin and uniform rod of mass M and length L is
particle speed is v = v 0 . The piston is moved inward held vertical on a floor with large friction. The rod
dL is released from rest so that it falls by rotating
at a very low speed V such that V << v 0 , where about its contact-point with the floor without
L slipping. Which of the following statement(s)
dL is the infinitesimal displacement of the piston. is/are correct, when the rod
Which of the following statement(s) is/are correct?
makes an angle 60° with vertical?
(a) After each collision with the piston, the particle speed
[g is the acceleration due to gravity]
increases by 2V.
(b) If the piston moves inward by dL, the particle speed 2g
(a) The angular acceleration of the rod will be .
dL L
increases by 2v .
L (b) The normal reaction force from the floor on the rod will
(c) The particle’s kinetic energy increases by a factor of 4 Mg
be .
when the piston is moved inward from L0 to 1/2 L0 . 16
(d) The rate at which the particle strikes the piston is v / L. (c) The radial acceleration of the rod’s center of mass will be
3g
.
3. Three glass cylinders I II III 4
of equal height H = 30 3g
(d) The angular speed of the rod will be .
cm and same 2L
refractive index n = 15 . H H H
are placed on a 6. In a Young’s double Screen
y
horizontal surface as slit experiment, the
P
shown in figure. X X X slit separation d is 0.3
Cylinder I has a flat top, cylinder II has s convex mm and the screen a d
x
top and cylinder III has a concave top. The radii of distance D is O
1 m. A parallel beam
curvature of the two curved tops are same (R = 3
of light of wavelength D
m). If H1 , H 2 , and H 3 are the apparent depths of a
600 nm is incident on
point X on the bottom of the three cylinders, the slits at angle a as shown in figure. On the
respectively, the correct statement(s) is/are screen, the point O is equidistant from the slits and
(a) H2 > H1 (b) H3 > H1 distance PO is 11.0 mm. Which of the following
(c) 0.85 cm < (H2 - H1 ) < 0.9 cm (d) H2 > H3 statement(s) is/are correct?
JEE Advanced~Solved Paper 2019 11

(a) For a = 0, there will be constructive interference at point (a) The ratio of kinetic energy of the electron in the state
P. 1
0.36 n = m to the state, n = 1is
(b) For a = degree, there will be destructive 4
p (b) m = 2
interference at point P. Dp 1
0.36 (c) a =
(c) For a = degree, there will be destructive Dpe 2
p
interference at point O. (d) le = 418 nm
(d) Fringe spacing depends on a . 8. A mixture of ideal gas containing 5 moles of
monatomic gas and 1 mole of rigid diatomic gas is
7. A free hydrogen atom after absorbing a photon of initially at pressure P0 , volume V0 , and temperature
wavelength l a gets excited from the state n = 1 to T0 . If the gas mixture is adiabatically compressed
the state n = 4. Immediately after that the electron V
to a volume 0 , then the correct statement(s)
jumps to n = m state by emitting a photon of 4
wavelength l e . Let the change in momentum of is/are
atom due to the absorption and the emission be
(Given, 21. 2 = 2.3; 2 3. 2 = 9 .2 ; R is a gas constant)
l 1
Dpa and Dpe , respectively. If a = , which of the (a) The final pressure of the gas mixture after compression
le 5
is in between 9P0 and 10P0 .
option(s) is/are correct? (b) The average kinetic energy of the gas mixture after
[Use hc = 1242 eVnm; 1 nm = 10 -9 m, h and c are compression is in between 18 RT0 and 19 RT0 .
Planck’s constant and speed of light in vacuum, (c) Adiabatic constant of the gas mixture is 1.6.
respectively] (d) The work|W| done during the process is 13RT0 .

Section 2 (Maximum Marks : 18)


l
This section contains SIX (06) questions. The answer to each question is a NUMERICAL VALUE.
l
For each question, enter the correct numerical value of the answer using the mouse and the on-screen virtual
numeric keypad in the place designated to enter the answer. If the numerical value has more than two decimal
places, truncate/round-off the value to TWO decimal places.
l
Answer to each question will be evaluated according to the following marking scheme:
Full Marks : + 3 If ONLY the correct numerical value is entered as answer.
Zero Marks : 0 In all other cases.

226
9. Suppose a 88 Ra nucleus at rest and in ground Air
state undergoes a-decay to a 222
86 Rn
nucleus in its θ 75°
n
excited state. The kinetic energy of the emitted a
particle is found to be 4.44 MeV. 222
86 Rn nucleus
n0=√3
then goes to its ground state by g-decay. The
energy of the emitted g photon is ……… keV. 11. A perfectly reflecting mirror of mass M mounted
on a spring constitutes a spring-mass system of
[Given : atomic mass of 226
88 Ra = 226 .005 u, atomic 4 pMW
angular frequency W such that = 10 24 m -2
mass of 222
86 Rn = 222.000 u, atomic mass of a h
particle = 4 .000 u, 1 u = 931 MeV/e 2 , c is speed of with h as Planck’s constant. Nphotons of
wavelength l = 8 p ´ 10 -6 m strike the mirror
the light]
simultaneously at normal incidence such that the
10. A monochromatic light is incident from air on a mirror gets displaced by 1 mm. If the value of N is
refracting surface of a prism of angle 75° and x ´ 10 12 , then the value of x is ……… .
refractive index n 0 = 3 . The other refracting [Consider the spring as massless]
surface of the prism is coated by a thin film of
material of refractive index n as shown in figure.
The light suffers total internal reflection at the Light
coated prism surface for an incidence angle of
q £ 60 °. The value of n 2 is ……… . Mirror
12 JEE Advanced~Solved Paper 2019

12. An optical bench has 1.5 m long scale having four 14. A 10 cm long perfectly conducting wire PQ is
equal divisions in each cm. While measuring the moving with a velocity 1 cm/s on a pair of
focal length of a convex lens, the lens is kept at 75 horizontal rails of zero resistance. One side of the
cm mark of the scale and the object pin is kept at rails is connected to an inductor L = 1 mH and a
45 cm mark. The image of the object pin on the resistance R = 1 W as shown in figure. The
other side of the lens overlaps with image pin that horizontal rails, L and R lie in the same plane with
is kept at 135 cm mark. In this experiment, the
a uniform magnetic field B = 1 T perpendicular to
percentage error in the measurement of the focal
the plane. If the key S is closed at certain instant,
length of the lens is ………
the current in the circuit after 1 millisecond is
13. A ball is thrown from ground at an angle q with x ´ 10 -3 A, where the value of x is …… .
horizontal and with an initial speed u 0 . For the [Assume the velocity of wire PQ remains constant
resulting projectile motion, the magnitude of
average velocity of the ball up to the point when it (1 cm/s) after key S is closed. Given, e-1 = 0 .37 ,
hits the ground for the first time is V1 . After hitting where e is base of the natural logarithm]
the ground, the ball rebounds at the same angle q B
u P
but with a reduced speed of 0 . Its motion
a
continues for a long time as shown in figure. If the L
magnitude of average velocity of the ball for entire 1 cm/s
duration of motion is R
0.8 V1 , the value of a is ………… .
u0 u0/α S Q
u0/α2
u0/αm
θ θ θ
θ

Section 3 (Maximum Marks : 12)


l
This section contains TWO (02) List-Match sets.
l
Each List-Match set has TWO (02) Multiple Choice Questions.
l
Each List-Match set has two lists : List-I and List-II
l
List-I has Four entries (I), (II), (III) and (IV) and List-II has Six entries (P), (Q), (R), (S), (T) and (U).
l
FOUR options are given in each Multiple Choice Question based on List-I and List-II and ONLY ONE of these four
options satisfies the condition asked in the Multiple Choice Questions.
l
Answer to each question will be evaluated according to the following marking scheme :
Full Marks : + 3 If ONLY the option corresponding to the correct combination is chosen:
Zero Marks : 0 If none of the options is chosen (i.e. the question is unanswered).
Negative Marks : - 1 In all other cases.

Answer the following by appropriately matching the lists based on the information given in the paragraph
A musical instrument is made using four different metal
strings, 1, 2, 3 and 4 with mass per unit length m, 2m, 3m List-I List-II
and 4m respectively. The instrument is played by (I) String-1 (m ) (P) 1
vibrating the strings by varying the free length in (II) String-2 (2m ) (Q) 1/2
between the range L 0 and 2L 0 . It is found that in
(III) String-3 ( 3m ) (R) 1/ 2
string-1 (m) at free length L 0 and tension T0 the
(IV) String-4 ( 4m ) (S) 1/ 3
fundamental mode frequency is f0 .
(T) 3/16
List-I gives the above four strings while list-II lists the
magnitude of some quantity. (U) 1/16
JEE Advanced~Solved Paper 2019 13

15. If the tension in each string is T0, the correct (III) Heat absorbed by the system in (R) RT0
process 1 ® 2 ® 3
match for the highest fundamental frequency in f0
(IV) Heat absorbed by the system in (S) 4
units will be RT0
process 1 ® 2 3
(a) I ® P, II ® Q, III ® T, IV ® S
(T) 1
(b) I ® P, II ® R, III ® S, IV ® Q RT0( 3 + ln2 )
3
(c) I ® Q, II ® S, III ® R, IV ® P
(U) 5
(d) I ® Q, II ® P, III ® R, IV ® T RT0
6
16. The length of the strings 1, 2, 3 and 4 are kept
3L 0 5L 0 7L 17. If the process on one mole of monatomic ideal gas
fixed at L 0 , × and 0 , respectively. Strings 1
2 4 4 is as shown in the TV-diagram with P0 V0 = RT0 ,
3
1, 2, 3 and 4 are vibrated at their 1st, 3rd, 5th and the correct match is,
14th harmonics, respectively such that all the
T
strings have same frequency.
T0 3
The correct match for the tension in the four strings
in the units of T0 will be T 1
(a) I ® P, II ® R, III ® T, IV ® U —0 2
3
(b) I ® P, II ® Q, III ® R, IV ® T V
V0 2V0
(c) I ® P, II ® Q, III ® T, IV ® U
(d) I ® T, II ® Q, III ® R, IV ® U (a) I ® P, II ® R, III ® T, IV ® S
Answer the following by appropriately matching (b) I ® P, II ® T, III ® Q, IV ® T
the lists based on the information given in the paragraph (c) I ® S, II ® T, III ® Q, IV ® U
In a thermodynamic process on an ideal monatomic gas, (d) I ® P, II ® R, III ® T, IV ® P
the infinitesimal heat absorbed by the gas is given by
TDX. where, T is temperature of the system and DX is the 18. If the process carried out on one mole of
infinitesimal change in a thermodynamic quantity X of monoatomic ideal gas is as shown in the
1
the system. For a mole of monatomic ideal gas, PV-diagram with p0 V0 = RT0 , the correct match is,
3
3 æ T ö æ V ö
X = R ln ç ÷ + R ln ç ÷
2 è TA ø è VA ø p
3 p0 3
Here, R is gas constant, V is volume of gas, TA and VA —
are constants. 2
The List-I below gives some quantities involved in a 1
p0 2
process and List-II gives some possible values of these
V
quantities. V0 2V0
List-I List-II
(a) I ® S, II ® R, III ® Q, IV ® T
(I) Work done by the system in (P) 1
RT0 ln2 (b) I ® Q, II ® R, III ® P, IV ® U
process 1 ® 2 ® 3 3
(c) I ® Q, II ® S, III ® R, IV ® U
(II) Change in internal energy in (Q) 1
RT0 (d) I ® Q, II ® R, III ® S, IV ® U
process 1 ® 2 ® 3 3
14 JEE Advanced~Solved Paper 2019

CHEMISTRY
Section 1 (Maximum Marks : 32) (a) It is a 4d state
Instructions: Same as given in Physics. (b) The nuclear charge experienced by the electron in this
19. Choose the correct option(s) for the following state is less than 2e, where e is the magnitude of the
reaction sequence electronic charge
(c) It has 2 angular nodes
CHO (i) Hg2+, dil. H2SO4 (i) SOCl2
(ii) AgNO3, NH4OH pyridine
(d) It has 3 radial nodes
Q
MeO
(iii) Zn-Hg, conc. HCl (ii) AlCl 3 22. Consider the following reactions (unbalanced).
Zn-Hg Zn + Hot conc. H 2SO 4 ¾® G + R + X
conc. HCl
R S Zn + conc. NaOH ¾® T + Q
Consider Q, R and S as major products. G + H 2S + NH 4OH ¾® Z (a precipitate) + X + Y
Choose the correct option(s).
(a) The oxidation state of Zn in T is +1
(a) MeO MeO (b) R is a V-shaped molecule
R O S (c) Bond order of Q is 1 in its ground state
(d) Z is dirty white in colour
CO2H
(b)
MeO
23. With reference to aqua-regia, choose the correct
MeO
option(s).
Q R O (a) Aqua-regia is prepared by mixing conc. HCl and conc.
HNO 3 in 3 : 1 (v / v ) ratio
(b) The yellow colour of aqua-regia is due to the presence of
(c) MeO NOCl and Cl 2
MeO
(c) Reaction of gold with aqua-regia produces an anion
O
R S having Au in +3 oxidation state
OH (d) Reaction of gold with aqua regia produces NO 2 in the
absence of air
CO2H
(d) 24. Choose the correct option(s) from the following.
MeO
MeO (a) Teflon is prepared by heating tetrafluoroethene in
Q S
presence of a persulphate catalyst at high pressure
20. Choose the correct option(s) that give(s) an (b) Natural rubber is polyisoprene containing trans alkene
aromatic compound as the major product. units
(c) Cellulose has only a-D-glucose units that are joined by
Br glycosidic linkages
NaOEt
(a) (d) Nylon-6 has amide linkages
25. The cyanide process of gold extraction involves
UV, 500 K leaching out gold from its ore with CN - in the
(b) + Cl2 (excess)
presence of Q in water to form R. Subsequently, R
is treated with T to obtain Au and Z. Choose the
(i) Alc. KOH correct option(s).
(c) H 3C Br (ii) NaNH2
(iii) Red hot iron tube, 873 K (a) Q is O 2 (b) Z is [Zn(CN)4 ]2-
Br
(c) T is Zn (d) R is [Au(CN)4 ]-
NaOMe 26. Which of the following reactions produce(s)
(d)
propane as a major product?
(a) H3C Cl Zn, dil. HCl
21. The ground state energy of hydrogen atom is
Br
-13.6 eV. Consider an electronic state Y of He+
whose energy, azimuthal quantum number and (b) H3C Br Zn

magnetic quantum number are -3.4 eV, 2 and 0,


respectively. (c) H3C COONa NaOH, CaO, ∆

Which of the following statement(s) is(are) true for COONa + H2O


(d) H3C Electrolysis
the state Y?
JEE Advanced~Solved Paper 2019 15

List-I List-II
Section 2 (Maximum Marks : 18) (I) Radius of the n th orbit (P) µ n -2
Instructions: Same as given in Physics. (II) Angular momentum of the electron in (Q) µ n -1
the nth orbit
27. The decomposition reaction (III) Kinetic energy of the electron in the nth (R) µ n0
D orbit
2N 2O 5(g) ¾® 2N 2O 4 (g) + O 2 (g) is started in a (IV) Potential energy of the electron in the (S) µ n1
closed cylinder under isothermal isochoric nth orbit
condition at an initial pressure of 1 atm. After (T) µ n2
Y ´ 10 3 s, the pressure inside the cylinder is found (U) µ n1/ 2
to be 1.45 atm. If the rate constant of the reaction
Which of the following options has the correct
is 5 ´ 10 -4 s -1 , assuming ideal gas behaviour, the
combination considering List-I and List-II?
value of Y is ……… (a) (III), (P) (b) (III), (S) (c) (IV), (U) (d) (IV), (Q)
28. The mole fraction of urea in an aqueous urea 34. Consider the Bohr’s model of a one-electron atom
solution containing 900 g of water is 0.05. If the where the electron moves around the nucleus. In
density of the solution is 1.2 g cm -3 , then molarity the following List-I contains some quantities for
of urea solution is …… (Given data : Molar masses the nth orbit of the atom and List-II contains
of urea and water are 60 g mol -1 and 18 g mol -1 , options showing how they depend on n.
respectively)
List-I List-II
29. Total number of hydroxyl groups present in a (I) Radius of the nth orbit (P) µ n -2
molecule of the major product P is ………
(II) Angular momentum of the electron in (Q) µ n -1
the nth orbit
(i) H2, Pd-BaSO4, quinoline (III) Kinetic energy of the electron in the nth (R) µ n0
(ii) Dil. KMnO4, (excess), 273 K
P orbit
(IV) Potential energy of the electron in the (S) µ n1
nth orbit
(T) µ n2
30. Total number of cis N ¾ Mn ¾ Cl bond angles (that (U) µ n1/ 2
is Mn ¾ N and Mn ¾ Cl bonds in cis positions)
present in a molecule of cis [Mn(en)2 Cl 2 ] complex Which of the following options has the correct
is ……… combination considering List-I and List-II?
(en = NH 2CH 2CH 2NH 2 ) (a) (II), (R) (b) (I), (P)
(c) (I), (T) (d) (II), (Q)
31. The amount of water produced (in g) in the oxidation
of 1 mole of rhombic sulphur by conc. HNO 3 to a 35. List-I includes starting materials and reagents of
compound with the highest oxidation state of selected chemical reactions. List-II gives structures
sulphur is …… (Given data : Molar mass of water of compounds that may be formed as intermediate
= 18 g mol -1 ) products and/or final products from the reactions
of List-I.
32. Total number of isomers considering
List-I List-II
both structural and stereoisomers of cyclic ethers
with the molecular formula C4H 8O is ……… (I) CN (i) DIBAL-H (P) CHO
(ii) Dil. HCl
O
Section 3 (Maximum Marks : 12) (iii) NaBH4
(iv) Conc. H2SO4
CO2H
O
Instructions: Same as given in Physics.
(II) (i) O3 (Q) OH
Answer the following by appropriately matching the lists (ii) Zn, H2O
based on the information given in the paragraph. CO2H (iii) NaBH4 OH
(iv) Conc. H2SO4
33. Consider the Bohr’s model of a one-electron atom
where the electron moves around the nucleus. In (III) Cl (i) KCN (R)
(ii) H3O+,∆
the following List-I contains some quantities for
(iii) LiAIH4 O
the nth orbit of the atom and List-II contains CO2CH3
(iv) Conc. H2SO4
options showing how they depend on n.
16 JEE Advanced~Solved Paper 2019

(IV) CO2Me (S) OH List-I List-II


(iii) LiAIH4
(II) (i) O3 (Q) OH
CO2Me (iv) Conc. H2SO4 (ii) Zn, H2O
CO2H
CO2H (iii) NaBH4 OH
(T) (iv) Conc. H2SO4
CO2H

CO2H (III) (i) KCN (R)


Cl
(ii) H3O+,∆
(U) (iii) LiAIH4 O
CO2CH3
(iv) Conc. H2SO4
O
(IV) CO2Me (S) OH
(iii) LiAIH4
O
CO2Me (iv) Conc. H2SO4
CO2H
Which of the following options has correct
combination considering List-I and List-II? (T) CO2H
(a) (III), (S), (R) (b) (IV), (Q), (R)
CO2H
(c) (III), (T), (U) (d) (IV), (Q), (U)
(U)
36. List-I includes starting materials and reagents of
O
selected chemical reactions. List-II gives structures
of compounds that may be formed as intermediate O
products and/or final products from the reactions of
List-I. Which of the following options has correct
combination considering List-I and List-II?
List-I List-II (a) (II), (P), (S), (U)
(I) CN (i) DIBAL-H (P)
(ii) Dil. HCl
CHO (b) (I), (Q), (T), (U)
O (iii) NaBH4
(c) (II), (P), (S), (T)
CO2H
(iv) Conc. H2SO4 (d) (I), (S), (Q), (R)
O

MATHEMATICS
Section 1 (Maximum Marks : 32) (a) F( x) ¹ 0 for all x Î(0, 5)
(b) F has a local maximum at x = 2
Instructions: Same as given in Physics.
(c) F has two local maxima and one local minimum in (0, ¥)
37. For non-negative integers n, let (d) F has a local minimum at x = 1
n
æ k +1 ö æk + 2 ö sin px
åsinçè n + 2 p ÷ø sinçè n + 2 p ÷ø 39. Let, f (x) =
x2
,x >0
k=0
f (n) = Let x1 < x 2 < x 3 < … < x n < … be all the points of
n
æ k +1 ö
åsin2 çè n + 2 p ÷ø local maximum of f and y1 < y 2 < y 3 < … < y n < …
k=0 be all the points of local minimum of f.
Assuming cos -1 x takes values in [0 , p], which of Then which of the following options is/are correct?
the following options is/are correct? (a)| xn - yn| > 1for every n
-1 (b) xn + 1 - xn > 2 for every n
(a) If a = tan(cos f(6)), then a + 2 a - 1 = 0 2
(c) x1 < y1
3
(d) xn Î æç2 n, 2 n + ö÷ for every n
(b) f(4) = 1
2 è 2ø
(c) sin(7 cos -1 f(5)) = 0
40. Three lines L1 : r = l$i, l ÎR, L 2 : r = k$ + m$j, m ÎR
1
(d) lim f(n) = and L 3 : r = $i + $j + vk$ , v Î R
n® ¥ 2
are given. For which point(s) Q on L 2 can we find a
38. Let f : R ® R be given by f (x) = (x - 1)(x - 2)(x - 5). point P on L1 and a point R on L 3 so that P , Q and R
x
Define F(x) = ò f (t)dt, x > 0 are collinear?
0 (a) k$ (b) k$ + $j
1 1
Then which of the following options is/are correct? (c) k$ + $j (d) k$ - $j
2 2
JEE Advanced~Solved Paper 2019 17

41. For a Î R,|a| > 1, let 6


é2 1 3 ù
æ ö and X = å k êê1 0 2úúPkT
P
ç ÷ k =1
ç 1+ 2 +¼ + n
3 3
÷ êë3 2 1 úû
lim = 54
n ®¥ ç æ ö÷
ç n7/ 3 ç
1
+
1
+ ¼ +
1
÷ where, PkT denotes the transpose of the matrix Pk .
÷
è è (an + 1)2 (an + 2)2 (an + n)2 ø ø Then which of the following option is/are correct?
Then the possible value(s) of a is/are (a) X is a symmetric matrix
(a) -6 (b) 7 (c) 8 (d) -9 (b) The sum of diagonal entries of X is 18
42. Let f : R be a function. We say that f has (c) X - 30 I is an invertible matrix

f (h) - f (0) é1ù é1ù


PROPERTY 1 if lim exists and is finite, (d) If X ê1ú = a ê1ú, then a = 30
|h|h ®0 êú êú
êë1úû êë1úû
f (h) - f (0)
and PROPERTY 2 if lim exists and is
h ®0 h2
finite. Section 2 (Maximum Marks : 18)
Then which of the following options is/are correct? Instructions: Same as given in Physics.

(a) f( x) = sin x has PROPERTY 2 45. Let a = 2$i + $j - k$ and b = $i + 2$j + k$ be two vectors.
(b) f( x) = x 2/ 3
has PROPERTY 1 Consider a vector c = aa + bb, a , b ÎR. If the
(c) f( x) = | x| has PROPERTY 1 projection of c on the vector (a + b) is 3 2, then the
(d) f( x) = x| x| has PROPERTY 2 minimum value of (c - (a ´ b)) × c equals ………

43. Let x Î R and let 46. Let|X| denote the number of elements in a set X.
Let S = {1, 2 , 3 , 4 , 5 , 6 } be a sample space, where each
é 1 1 1ù é2 x xù element is equally likely to occur. If A and B are
P = 0 2 2 , Q = ê0
ê ú 4 0 ú and R = PQP -1 , independent events associated with S, then the
ê ú ê ú
êë0 0 3 úû êë x x 6 úû number of ordered pairs (A , B) such that
1 £ |B| < |A|, equals ………
the which of the following options is/are correct? é n n ù
(a) There exists a real, number x such that PQ = QP ê åk å n Ck k 2 ú
é 1ù é 1ù
47. Suppose det ê nk = 0 k=0 ú = 0 holds for
ê n

(b) For x = 0, if R ê a ú = 6 ê a ú, then a + b = 5 ê å Ck k å Ck 3 ú
n n
ê ú ê ú
ëk = 0 k=0 û
ëê b ûú êë b úû
n n
Ck
(c) For x = 1, there exists a unit vector a $i + b$j + gk$ for which some positive integer n. Then å equals
k=0 k +1
é aù é 0ù ………
R êb ú = ê 0ú
ê ú ê ú 48. Five persons A , B , C, D and E are seated in a
êë g úû êë 0úû circular arrangement. If each of them is given a hat
é2 x xù of one of the three colours red, blue and green,
(d) det R = det ê 0 4 0ú + 8, for all x Î R
then the number of ways of distributing the hats
ê ú such that the persons seated in adjacent seats get
êë x x 5úû
different coloured hats is ………

é1 0 0 ù é1 0 0ù 49. The value of the integral


44. Let P1 = I = ê0 1 0 ú, P2 = ê0 0 1 ú, p/ 2
3 cos q
ê ú ê ú
êë0 0 1 úû êë0 1 0 úû ò ( cos q + sin q )5
d q equals …………
0
é0 1 0ù é0 1 0ù
P3 = ê 1 0 0 ú, P4 = ê0 0 1 ú,
50. The value of
ê ú ê ú æ 1 10
êë0 0 1 úû êë 1 0 0 úû æ 7 p kp ö æ 7 p (k + 1)p ö ö÷
sec -1 çç å sec ç + ÷ sec ç + ÷
è 4 k = 0 è 12 2 ø è 12 2 ø ÷ø
é0 0 1ù é0 0 1ù
P5 = ê 1 0 0 ú, P6 = ê0 1 0ú é p 3p ù
ê ú ê ú in the interval ê - , ú equals……
êë0 1 0 úû êë 1 0 0 úû ë 4 4 û
18 JEE Advanced~Solved Paper 2019

Which of the following is the only CORRECT


Section 3 (Maximum Marks : 12) combination?
Instructions: Same as given in Physics. (a) (II), (Q), (T)
(b) (II), (R), (S)
Answer the following by appropriately matching the lists based (c) (I), (P), (R)
on the information given in the paragraph. (d) (I), (Q), (U)
51. Let f (x) = sin(p cos x) and g (x) = cos(2p sin x) be two 53. Let the circles C1 : x 2 + y 2 = 9 and
functions defined for x > 0. Define the following
C2 : (x - 3)2 + (y - 4)2 = 16 , intersect at the points X
sets whose elements are written in the increasing
order : and Y . Suppose that another circle
X = { x : f (x) = 0 }, Y = { x : f ¢ (x) = 0 } C3 : (x - h)2 + (y - k)2 = r2 satisfies the following
Z = { x : g (x) = 0 }, W = { x : g ¢ (x) = 0 } conditions :
List-I contains the sets X , Y , Z and W. List-II (i) Centre of C3 is collinear with the centres of C1
contains some information regarding these sets. and C2 .
(ii) C1 and C2 both lie inside C3 and
List-I List-II
(iii) C3 touches C1 at M and C2 at N.
p 3p
(I) X (P) Ê ìí , , 4p, 7 p üý
î2 2 þ Let the line through X and Y intersect C3 at Z and
W, and let a common tangent of C1 and C3 be a
(II) Y (Q) an arithmetic progression
tangent to the parabola x 2 = 8 ay.
(III) Z (R) NOT an arithmetic progression
p 7 p 13p ü There are some expression given in the List-I
(IV) W (S) Ê æç , , ý
è6 6 6 þ whose values are given in List-II below.
p 2p ü
(T) Ê æç , , pý List-I List-II
è3 3 þ (I) 2h + k (P) 6
p 3p ü
Ê ìí , (II) Length of ZW (Q) 6
(U) ý
î6 4 þ Length of XY
(III) Area of triangle MZN (R) 5
Which of the following is the only CORRECT
Area of triangle ZMW 4
combination?
(a) (IV), (P), (R), (S) (b) (III), (P), (Q), (U) (IV) a (S) 21
5
(c) (III), (R), (U) (d) (IV), (Q), (T)
(T) 2 6
52. Let f (x) = sin(p cos x) and g (x) = cos(2p sin x) be two (U) 10
functions defined for x > 0. Define the following 3
sets whose elements are written in the increasing
order : Which of the following is the only INCORRECT
combination?
X = { x : f (x) = 0 }, Y = { x : f ¢ (x) = 0 } (a) (III), (R)
Z = { x : g (x) = 0 }, W = { x : g ¢ (x) = 0 } (b) (IV), (S)
(c) (I), (P)
List-I contains the sets X , Y , Z and W. List-II
(d) (IV), (U)
contains some information regarding these sets.
54. Let the circle C1 : x 2 + y 2 = 9 and
List-I List-II
p 3p C2 : (x - 3)2 + (y - 4)2 = 16 , intersect at the points X
(I) X (P) Ê ìí , , 4p, 7 p üý
î2 2 þ and Y . Suppose that another circle
(II) Y (Q) an arithmetic progression
C3 : (x - h)2 + (y - k)2 = r2 satisfies the following
(III) Z (R) NOT an arithmetic progression
conditions :
p 7 p 13p ü (i) centre of C3 is collinear with the centers of C1
(IV) W (S) Ê æç , , ý and C2 .
è6 6 6 þ
p 2p ü
(ii) C1 and C2 both lie inside C3 , and
(T) Ê æç , , pý (iii) C3 touches C1 at M and C2 at N.
è3 3 þ
p 3p ü Let the line through X and Y intersect C3 at Z and
(U) Ê ìí , ý W, and let a common tangent of C1 and C3 be a
î6 4 þ
tangent to the parabola x 2 = 8 ay.
JEE Advanced~Solved Paper 2019 19

There are some expression given in the List-I whose values are given in List-II below.
List-I List-II
(I) 2h + k (P) 6
(II) Length of ZW (Q) 6
Length of XY
(III) Area of triangle MZN (R) 5
Area of triangle ZMW 4
(IV) a (S) 21
5
(T) 2 6
(U) 10
3

Which of the following is the only CORRECT combination?


(a) (II), (T) (b) (I), (S)
(c) (II), (Q) (d) (I), (U)

Answers
Paper 1
1. a 2. b 3. a 4. d 5. a,c,d 6. a,b,d 7. a,b,d 8. a,b 9. b,c 10. b,c,d
11. b,d 12. a,c,d 13. 1.00 14. 50.00 15. 2.00 16. 270.00 17. 0.75 18. 8.13 19. b 20. c
21. d 22. b 23. c,d 24. a,d 25. a,b,c 26. a,b,d 27. c,d 28. a,c,d 29. a,c,d 30. a,c
31. 4.00 32. 1.02 33. 6.75 34. 8.9 35. 19 36. 4 37. c 38. d 39. d 40. c
41. a,c 42. c,d 43. a,b,d 44. b,c,d 45. a,b,c 46. a,c 47. b,c,d 48. b,c,d 49. 0.50 50. 10
51. 3.00 52. 4.0 53. 0.75 54. 157.00

Paper 2
1. c,d 2. a,c 3. a,d 4. a 5. b,c,d 6. b 7. a,b 8. a,c,d 9. 135.00 10. 1.50
11. 1.00 12. 1.38 & 13. 4.00 14. 0.63 15. b 16. c 17. d 18. d 19. a,b 20. c,d
1.39
21. a,c 22. b,c,d 23. a,b,c 24. a,d 25. a,b,c 26. a,c 27. 2.3 28. 2.98 29. 6 30. 6
mole
31. 288 32. 10.0 33. a 34. c 35. b 36. a 37. a,b,c 38. a,b,d 39. a,b,d 40. c,d
41. c,d 42. b,c 43. b,d 44. a,b,d 45. 18 46. 1523 47. 6.20 48. 30 49. 0.5 50. 0
51. a 52. a 53. b 54. c
20 JEE Advanced~Solved Paper 2019

Answer with Explanations


Paper 1
1. (a) lCa
Ca 0 V at point B,
KQ K(αQ)
K VB = − = V0 (1 − 2α)
lAr R R/2
Ar 0 VC 1−α
∴ =
at t = 0 dissipated energy, VB 1 − 2α
dN
= −(λ1 + λ 2) × N
dt 4. (d)
dN
⇒ = −(λ1 + λ 2) dt
N
Integration on both sides, we get M v
N
log e   = −(λ1 + λ 2)t
 N0  m

 N0  −10
2.3 × log10   = 5 × 10 t Gravitational force = Centripetal force of the earth
 N 0 / 100 
GMm mv 2
2.303 × 2 = (Q M = total mass from 0 to r)
=t r2 r
5 × 10−10
=  mv 2 
2 1
2.303 × 0.4 × 1010 = t ⇒ t = 9.2 × 109 Yr r2 
2. (b) Heat capacity, dQ = H dT ⇒
GMm 2K
= ⇒M=
2Kr Q 1 mv 2 = K 
 
dt dt r2 r Gm  2 
1
Power of the rod, P = H. T0 .β. . t − 3/ 4 Differentiate on both sides, we get
4 2K 2K
⇒ dM = dr ⇒ 4 πr 2drρ = dr
4P −3/ 4  4P  3/ 4 Gm Gm
=t ⋅H ⇒ H =   t …(i)
T0 .β  T0β  (Q volume = mass × density)
K
Now, T − T0 = T0βt1 / 4 ∴ ρ=
2πGmr 2
3
 T − T0  ρ K Q ρ = volume
So, t 3/ 4 =   ∴ =  
 T0 β  m 2πr 2m2G  m 

Substituting this value of t 3/ 4 in equation (i) we get, 5. (a, c, d) Heights if only single material tubes are used
4P(T − T0)3 of sufficient length,
H=
T04β 4 2R cosθ 2 × 0.075 × cos 0°
h1 = = = 7.5cm
ρrg 1000 × 2 × 10−4 × 10
3. (a) 2T cosθ′ 2 × 0.075 × cos 60°
R/2 h2 = = = 3.75cm
ρrg 1000 × 2 × 10−4 × 10
B Case- I
C
T2

Given, V at surface of the sphere


T1
KQ
V0 = 8 cm
R
1 7.5 cm
Here, K= = constant
4 π ε0
V at point C,
KQ KαQ
VC = − = V0 (1 − α)
R R
JEE Advanced~Solved Paper 2019 21

Case-II 7. (a, b, d) 1 = (n − 1)  1 − 1  ⇒ 1 = 2(n − 1) …(1)


f R ∞ f0 R

= (n + ∆n − 1) − 
1 1 1
T2 ∆f R ∞
(n − 1)
+ (n + ∆n − 1) 
1 1
=
T1 f0 + ∆f0 R R 
1 2n + ∆n − 2
5 cm = …(2)
f0 + ∆f0 R
From Eqs. (i) and (ii), we get
2(n − 1)
f0 + ∆f0 R
=
f0 2n + ∆n − 2
2. Liquid will rise only upto height of 5 cm and
meniscus will adjust by changing is radius of R
∆f 2(n − 1)
curvature. If the liquid goes up in tube 2 then it will 1+ 0 =
not be able to support the weight of the liquid. f0 (2n + ∆n − 2)
3. Weight of water in meniscus will be different in two ∆f0 − ∆n
=
cases because angle of contact is different. f0 (2n + ∆n − 2)
∆f0 10−3
= ⇒ ∆f0 = −2 × 10−2
20 3 + 10−3 − 2
|∆f0|= 0.02 cm
T1
8. (a, b) P - V graph of the given V- T graph is shown
below.
T2 p
5 cm 2p0 1 T0 2 2T0

3.75 cm
p0 T0/2
3 T0
4

V0 2V0 V
6. (a, b, d) y
∆Q1→ 2 NCp ∆T1→ 2 T
(d) = = 0 =2
B ∆Q 3→ 4 NCp ∆T3→ 4 T0 / 2
L
y=x 2
∆Q1→ 2 NCp ∆T1→ 2 C 5
dy (b) = = p =
V0 ∆Q 2→ 3 NCp ∆T3→ 4 CV 3
V0
(a) Wcycle = p0 V0 = nR  0 
y T
x  2 
O
Motional emf across the length dy is, Note For ideal gas equation,
 y 
β pV = nRT
dε = BV0 dy = B 0 1 +    V0 dy T0 T
  L   ∴ ( pV ) 4 = ( nRT ) 4 or p0V0 = nR
2
=R 0
2
L  y 
β as n= 1
∴ ε = ∫ B 0 1 +    V0 dy
  L   (c) Wrong as no adiabatic process is involved.
0 
 1  9. (b, c) Just after closing of switch charge on any
= B 0 V0 1 +  capacitor is zero.
 β + 1
∴Replace all capacitors by conducting wires.
emf in loop is proportional to L for given value of β,
30W
β = 0, ε = 2B 0 V0L
5V
β = 2, ε = B 0 V0L 1 +  = B 0 V0L
1 4
 3 3 70W
The length of projection of the wire Y = X of length 2L on
the y-axis is L thus, the answer remain unchanged. i
Therefore, correct options are are (a), (b) and (d). 100W
22 JEE Advanced~Solved Paper 2019

Current flow in the circuit, ∴ φ=0


5 5 8R
i= = = 25 mA so, for h < then φ = 0.
70 + 100 + 30 200 5
4R
Now S1 is kept closed for long time circuit is in steady (c) For h = 2R and r =
state. 5

P –q q 5V 4R/5
80mF –q
53°
10mF 80mF
q
R
Q

q q q 10q
+ + − 5= 0 ⇒ =5
10 80 80 80
Q Q
∴ q = 40 µC Shaded charge = 2π (1 − cos 53º) × =
40 4π 5
∴ Vacross C1 = = 4V 2Q
10 ∴ qenclosed =
Now just after closing S2 charge on each capacitor remains 5
2Q
same. ∴ φ=
5ε0
–40 40 5 30W
0 80mF 4R
40 x–y ∴ for h > 2R and r =
10mF 0 –40 5
y –40
30mF

30Ω 2
2Q
70
1
40 ∴ φ=
x
10V 100W 5ε0
3R
Applying KVL, (d) like option c for h = 2R and r =
5
40
−10 + x × 30 + + y × 70 = 0 Q Q
10 qenclosed = 2 × 2π (1 − cos 37º) =
4π 5
30x + 70y = 6 …(1) Q
40 40 ∴ Electric flux, φ =
− + 5 + (x − y) 30 − + (x + y) × 100 − 10 + x × 30 = 0 5ε0
80 80
160x − 130y − 6 = 0 …(2) 11. (b, d) Rg
Ig
96
y= G
1510
V = 100 × 10−3 = 10−1 V
x = 0.05A
V = l g(R g + R V)
10. (b, c, d)
10−1
(a) h > 2R and r > R = Rg + RV
2 × 10−6
r
5 × 104 Ω ≈ R V (Q R V < 105 Ω)
RV
V
RA
R A
Q i 1000W
i
Q
φ= , clearly from Gauss’ Law
ε0 ε
8R 3R l gR g = (l − l g)S
(b) suppose h = and r =
5 5
S
3R/5
I – Ig

4R/5 G
Ig

2 × 10−6 × 10
S=
10−3 − 2 × 10−6
S = 2 × 10−5 × 103 = 2 × 10−2 = 20mΩ
JEE Advanced~Solved Paper 2019 23
SR g 20 × 10−3 × 10 According to total internal reflection (TIR),
RA = = ≈ 20 × 10−3 Ω
S+ Rg 10 + 20 × 10−3 1.5sinθc = 1.44sin 90º
1.44 24
ε 51ε sinθc = =
i= = (Q R A → 0) 1.50 25
 1000 × 50 × 103  5 × 104
  x 24 25x
 51 × 103  ∴ sinθc = = ⇒ d=
d 25 24
ε
i ′ = i 
RV  ∴ Total length travelled by light,
 =
 51 × 103  1000 S 10 1
∴ t= = = × 10−7 = 5 × 10−8
Measured resistance,  c 3 × 108 2
i ′ × 1000 ε  
∴ Rm = = × 5 × 10 = 980.4 Ω  n1  1.5
i 51ε
t = 50 ns ⇒ t = 50 × 10−9
If the voltmeter shows full scale deflection then
ε
× 
1000  −1 15. (2.00) Ts Tc
 × 5 × 10 = 10
4
980  51 × 103 
∴ ε = 999.6 mV
Since, i A = 1 mA so maximum reading of R can be 60° 30°
999.6 mV
= 999.6 Ω
1 mA

12. (a,c,d) [M] =[Mass] =[M 0L0 T0 ] 100 N


[J] =[Angular momentum] = [ML2T–1 ] TS 3
= TC
2 2
[L] = [Length]
TS = 3 TC
Now, [ML2T−1 ] = [M 0L0 T0 ]
Tl
∴ [L2] = [T] ∆l =
Ay
Power [P] = [MLT−2.LT −1 ] = [ML2T−3] = [L2 L–6]
∆l C  TC   l C   YS 
[P] =[L−4 ] ∴ =     
∆lS  TS   lS   YC 
Energy/ Work [W] =[MLT −2.L]
1  3   2 × 1011 
= [L2 L−4 ] = [L−2] =       = 2.00
 3   1   1 × 1011 
Force [F] = [MLT − 2] = [L.L−4 ] = [L−3]
16. (270.00) Case-I 5C × 50 + 5 L = C2 × 30 …(i)
Linear momentum [p] = [MLT−1 ] = [L.L−2]
Case-II 80C[50 − 30] = C2 [80 − 50] …(ii)
[p] =[L−1 ]
By Eq. (i) and (ii)
13. (1.00) Parallel plate capacitor, 1600C = 250 + 5L
x d L 1350
= ∴ = = 270° C
m N C 5
d  =
1 dx dx dx
= = 17. (0.75) d = F ⋅ dr
 C  Kmε0 A 
Kε0 A 1 +  Kε0 A 1 + 
m x
d = αydx + 2αxdy
 N  d 1
A → B , y = 1, dy = 0 then WA → B = ∫ αydx = α1∫ dx = α
Integration on both sides, we get 0

= d  =
0. 5
1 1 D d dx B → C, x = 1, dx = 0 then WB→ C = 2α1∫ dy = −2α(0.5) = −α
Ceq ∫  C  ∫0 Kε0 A(d + x) 1
C → D , y = 0.5, dy = 0 then
1 d K ε0 A
Ceq ∫ Kε0 A
= ln 2 ⇒ Ceq = 0. 5
1 0. 5 α
d ln 2 WC→ D = ∫ αydx = α. 2∫1 dx = −
4
1
Therefore, α = 1.
D → E , x = 0.5, dx = 0 then
0
14. (50.00) B 1 α
WD→ E = 2 α ∫ xdy = 2 α.
2 0∫. 5
dy = −
2
d
qc C E → F , y = 0, dy = 0 then WEF = 0
A F → A , x = 0, dx = 0 then WF→ A = 0
q x
α α 3α
∴ W = α −α − − = −
4 2 4
P 3
Given, α = −1 ⇒ W = J = 0.75 J
9.6 m 4
24 JEE Advanced~Solved Paper 2019

18. (8.13) Vsound = 330 m/s 21. (d) ZnCO 3-Calamine (zinc ore)
30 m/s CuCO 3 ⋅ Cu(OH)2-Malachite (copper ore)
S1 S2 Fe3O 4 -Magnetite (iron ore)
Na 3AlF6-Cryolite (aluminium ore)
Thus, option (d) is correct.
10 m/s
22. (b) Key Idea The aqueous solution of ionic surfactant, i.e.
 330 + 10 cos 53°  – +
f1 = 120   Hz sodium stearate (C17H 35CO O N a) acts as a strong
 330 − 30 cos 37° 
univalent type of electrolyte in the concentration range
330 + 10 
f2 = 120  Hz below the CMC and the linear function of dependence
 330 
of Λ m on C has a small negative slope.
∆f = (f2 − f1) = 120 × 
336 34 
− = 8.13Hz
 306 33
CMC of Lm on ÖC has a
Lm small negative slope.
19. (b) Acidic nature depends upon nature of electron
withdrawing group and electronegativity.
Electronegativity further depends on % s character.
Higher the s-character, greater will be the ¾
√C
electronegativity and hence tendency to loose H
increases thus acidic character also increases. At normal or low concentration, sodium stearate
O [CH 3(CH 2)16COO −Na + ] behaves as strong electrolyte and
O H for strong electrolyte, molar conductance (Λm) decreases
(I) H (II) OH with increase in concentration.
OH H Above particular concentration, sodium stearate forms
sp- hybridisation sp 2 -hybridisation aggregates known as micelles. The concentration is called
(50% s character) (30-33% s character) as CMC. Since, number of ions decreases and hence Λm
(pKa = 1.86) (pKa = 4.3) also decreases.
O Hence, option (b) is correct.
OH
(III) MeO (IV) H3C 23. (c, d) The standard enthalpy of formation is defined as
OH O standard enthalpy change for formation of 1 mole of a
sp2 -hybridisation sp3 -hybridisation substance from its elements, present in their most stable
(Resonance effect) (25% s-character) state of aggregation.
(pKa = 4.5) (pKa = 4.8) 3
O 2(g) → O 3(g) ;
Hence, acidic order I > II > III > IV. 2
1
II is more acidic than III since electron donating group s + O 2(g) → SO 2(g)
S8 ()
(OCH 3) is attached to benzene ring in III which 8
decreases the acidic character. In the above two reactions standard enthalpy of reaction
On the other hand, pK a value also determined acidic is equal to standard enthalpy of formation.
nature, lower pK a value gives maximum acidic character.
24. (a, d)
Hence, option (b) is correct.
Sn
20. (c) Borax bead test is performed only for coloured salt. – –
Borax (sodium pyroborate), Na 2B 4O 7 ⋅10H 2O on heating SnCl2 + Cl SnCl3 Cl Cl
Tin chloride (X ) Cl
gets fused and lose water of crystallisation. It swells up sp3 (pyramidal)
(Q)
into fluffy white porous mass which melts into a
colourless liquid which later form a clear transparent SnCl −3 has (3σ + 1lp) and exist in pyramidal structure.
glassy bead consisting of boric anhydride and sodium
Me Cl
metaborate.
SnCl2+Me3N SnCl2 × NMe3 Me N Sn

Na 2B 4O 7 ⋅10H 2O → Na 2B 4O 7 +10H 2O ↑ (3°amine) (Y) Me Cl
(Q)

Na 2B 4O 7 → B 2O 3 + 2NaBO 2 Y complex has coordinate bond in between nitrogen and Sn
Boric Sodium metal.
anhydride metaborate
144424443
glassy bead Reduction

Boric anhydride is non-volalite. When it react with Cr(III)


+2 +2 +4 +1
salt then deep green complex is formed. SnCl2 + 2CuCl2 SnCl4 + 2CuCl
2Cr 3+ + 3B 2O 3 → 2Cr(BO 2)3 (Q) (Z )
Deep green Oxidation
Hence, option (c) is correct.
JEE Advanced~Solved Paper 2019 25
Z is oxidised product and oxidation state of Sn is +4 in Z CH3 Br CH3
compound. Structure of SnCl 4 (Z) is (i) H2,Ni 20% H3PO4,
(Reduction) 360K
Cl + H2 O
(ii) Br2,hn Dehydration
Sn major (free radical major of alcohol
Cl (T) substitution) (R)
Cl Cl
CH3
Thus, options (a, d) are correct.
Br
25. (a,b,c) Key Idea The lose of one α-particle will decrease
HBr
the mass number by 4 and atomic number by 2. On Benzoyl peroxide
the other hand, loss of β-particle will increase the (anti-Markovnikov's rule) major
atomic number by 1. (U)

In decay sequence, Hence, options (c, d) are correct.

92U
238
90Th
234
+ 2He (or a)
4
28. (a, c, d) The explanation of given statements are as
X1 particle
follows:
234 (a) Two six membered cyclic hemiacetal form of
92U
234
+ (b– or –1e0) 91Pa + (b– or –1e0)
234 X2 particle
D-(+)- glucose are called anomers.
Z is isotope X3 particle
of uranium CH2OH CH2OH

H O OH H O H
90Th
230
+ 2He4 (or a) H H
X4 particle OH H OH H
X1 particle will deflect towards negatively charged plate OH H OH OH
due to presence of positive charge on α- particles. H OH H OH
Hence, options (a, b, c) are correct. a-D(+)-glucopyranose b-D(+)-glucopyranose
26. (a, b, d) The explanation of given statements are as Both are anomers.
follows: (b) Oxidation of glucose in presence of Br2 water gives
(a) Urms is inversely proportional to the square root of its gluconic acid.
molecular mass.
CHO COOH
3RT
Urms = H OH H OH
M
Hence, option (a) is correct. HO H Br2 HO H
(b) When temperature is increased four times then Urms H OH Water H OH
become doubled. H OH H OH
3R
Urms = × 4T CH2OH CH2OH
M
D-(+)-glucose Gluconic acid
3RT
Urms = 2 × (c) Monosaccharides can not be hydrolysed into
M
polyhydroxy aldehydes and ketones.
Hence, option (b) is correct.
(d) Hydrolysis of sucrose gives D-glucose and L-fructose.
(c) and (d) E av is directly proportional to temperature but
Invertase
does not depends on its molecular mass at a given C12H 22O11 + H 2O → C6 H12O 6
3
temperature as E av = KT. If temperature raised four D-glucose
2 or dextrorotatory
times than E av becomes four time multiple. + C6H12O 6
Thus, option (c) is incorrect and option (d) is correct. L-fructose
or laevorotatory
27. (c, d) The given road map problem is Hence, options (a, c, d) are correct.
H—OH
O CH3 O MgBr 29. (a, c, d) MnO 2 + 2KOH + 1 O 2 →

K 2MnO 4 + H 2O
2 (W) potassium
CH3MgBr manganate
H2 O Hydrolysis
+ 2–
K2MnO4 (aq) 92K (aq) + MnO 4 (aq)
(C6H10O) (W) (Y)
CH3 Cl CH3 OH O
é
3

é
H sp- hybridisation,
Br Mn tetrahedral (manganate ion) é
Conc. HCl + Mg –
–H2O O O Green coloured
OH complex
Major (Q) O
( S)
26 JEE Advanced~Solved Paper 2019

MnO 2−4 ion has one unpaired electrons, therefore it gives


F F
H F F
d-d transition to form green colour. Y complex has O
H C Cl , B , ,
paramagnetic nature due to presence of one unpaired Xe
electron. F F O O
H m¹0 m=0
In aqueous solution, (polar) (non-polar) m¹0 F F
Electrolytic oxidation (polar) F
K 2MnO 4 + H 2O → H 2 + KOH + KMnO 4 m»0
( W) ( X) (non-polar)
Thus, options (a, c) are correct.
O
31. (4.00) N 2O, N 2O 4 , H 2S2O 3 and H 2S2O 8 molecules are
KMnO4(aq)
D
K+ + MnO4- é Mn
sp3, tetrahedral
(purple coloured
é
é containing covalent bond between two atoms.
(Z) complex ion)
O O S
O
MnO −4 ions gives charge transfer spectrum in which a N ºº N ¾® O ,
O O
S
fraction of electronic charge is transferred between the (N2O) N¾N ,
molecular entities. HO OH O O
O (N2O4)
Electrolytic
Q MnO 24− → MnO 4− + e− (H2S2O3)
( Y) oxidation ( Z)
O O
In acidic medium, Y undergoes disproportionation H H
H
reaction. S S B B
,
3MnO 24− (aq) + 4H+ → 2MnO −4 + MnO 2 + 2H 2O O¾O
(Y) (Z) HO OH H H
O O H
(B2H6)
MnO 2−4 and MnO −4 both ions form π-bonding between (H2S2O8)
( Y) ( Z) H
p-orbitals of oxygen and d-orbitals of manganese. |
H B H
Thus, options (a, c, d) are correct.
N N
30. (a,c) Key Idea Dipole moment of a bond depends on the
difference in the electronegativities of bonded atoms.
B B
More is the difference in the electronegativities,
greater will be the dipole moment. Also, H N H
For symmetrical molecule, µ = 0 (B3N3H6)
For unsymmetrical molecule, µ ≠ 0 B 2H 6 and B 3N 6H 6 have polar bond, but do not have same
kind of atom.
The molecules which gives permanent dipole moment
are polar in nature. 32. (1.02) Key Idea First calculate, molar mass of solute using
Cl p ° − ps nsolute
the formula, = and then
p° nsolute + nsolvent
S Se
, , calculate ∆Tf by applying the formula; ∆Tf = K f × m.
O O H H
m¹0 m¹0 m¹0 When 0.5 g of non-volatile solute dissolve into 39 gm of
(polar) (polar) (polar) benzene then relative lowering of vapour pressure occurs.
F Hence, vapour pressure decreases from 650 mmHg to
F F 640 mmHg.
m1 m2 Given, vapour pressure of solvent (p°) = 650 mmHg
Br
, Cl Be Cl Vapour pressure of solution (ps) = 640 mmHg
F F mnet» 0
m¹0 Weight of non-volatile solute = 0.5 g
(non-polar)
(polar) Weight of solvent (benzene) = 39 g
Cl From relative lowering of vapour pressure,
m1 m2
, p° − ps nsolute
O C O B Cl = xSolute =
p° nsolute + nsolvent
mnet 0 m 0
Cl 0.5
(non-polar) (non-polar)
650 − 640 molar mass
=
650 0.5 39
Cl
N N Cl +
molar mass 78
H C Cl , , , O P Cl , 0.5
O O H H H
Cl 10 molar mass
m¹0
Cl
m¹0 m¹0 m¹0 =
650 0.5
(polar) (polar) (polar) (polar) + 0.5
molar mass
JEE Advanced~Solved Paper 2019 27
0.5 + 0.5 × molar mass = 65 × 0.5 35. (19) XeF4 reacts with O 2F2 to form XeF6 ⋅ O 2F2 is
∴ Molar mass of solute = 64 g fluoronating reagent.
From molal depression of freezing point, 143 K
∆Tf = Kf × molality XeF4 + O 2F2 → XeF6 + O 2
( Y)
Kf × wsolute
=
(MW)solute × wsolvent The structore of XeF6 is
0.5 × 1000
∆Tf = 512
. × ⇒ ∆Tf = 1.02K F
64 × 39 F F
Xe
33. (6.75) Rate = k[A] [B] [C] x y z
F
F
(Rate)1 [0.2]x [01 . ]z 6 × 10−5
. ]y [01 F
= =
x y
(Rate)2 [0.2] [0.2] [01 . ]z 6 × 10−5
Y compound (XeF6) has 3 lone pair in each fluorine and one
⇒ y=0 lone pair in xenon.
(Rate)1 [0.2]x [01
. ]y [01
. ]z 6 × 10−5
= = Hence, total number of lone pairs electrons is 19.
x
(Rate)3 [0.2] [01 y
. ] [0.2] z
1.2 × 10−4
36. (4) Scheme -1
⇒ z =1 NH2 NH2
(Rate)1 [0.2]x [01 . ]z 6 × 10−5
. ]y [01
= = Br Br
x
(Rate)4 [0.3] [01 y
. ] [01. ]2 9 × 10−5 (i) Br2(excess) (ii) NaNO2 +HCl
¾¾¾¾¾¾®
⇒ x =1 H2O 273K
So, rate = k[A]1 [C]1 Aniline (diazotisation
(P) Br of aniline )
From exp-Ist, 2,4,6-tribromoaniline + –
Rate = 6.0 × 10−5 mol dm −3 s −1 CN
N 2 Cl
−5 Br Br
6.0 × 10 = k[0.2] [011
.] 1
Br Br
−3
(iii) CuCN/KCN
k = 3 × 10
®
+ N2 –
Cl is displace
Given, [A] = 015mol
. dm −3 by CN–ion
Br
[B] = 0.25 mol dm −3 Br 2, 4, 6 tribromo
¾¾¾®

diazonium chloride salt


. mol dm −3
(complete hydrolysis of
[C] = 015 +
¾ CN group which
(iv) H3O
−3
∴ Rate = (3 × 10 ) × [015
. ] [0.25] [015
. ] 1 0 1 convert into ¾COOH) O
−3
= 3 × 10 × 015
. × 015
. COOH C ¾Cl
−5 −3 −1
Rate = 6.75 × 10 mol dm s Br Br (v) SOCl2, Br Br
Pyridine
Thus, Y = 6.75 ¾¾¾¾®

34. (8.9) Given, equilibrium constant (K c) at 298 K


Br Br
= 1.6 × 1017 Major
(Q)
Fe2+ (aq) + S2− (aq) 1 FeS ()
s
At initial concentration 0.06 M 0.2 M – Scheme-2
(Before mixing) SO3H SO3Na
At initial concentration 0.03 M 0.1 M –
(After mixing) (i) Oleum (ii) NaOH, D
At equilibrium 0.03-X – 0.1 – 0.03 = 0.07 ¾¾¾¾® ¾¾¾¾®
H2S2O7
[Here, K c >> 103 , thus limiting reagent will be consumed (H2SO4+ SO3)
(R )
almost completely, 0.03 − X = 0 ∴ X = 0.03] (Sulphonation)
From equilibrium constant, OH
[FeS]
KC = +
(iii) H Hydrolysis
[Fe2+][S2− ] ¾¾¾¾¾¾¾® + NaHSO3
1
KC = [For FeS(s) = 1 mol L −1 ]
X × 0.07 (Pure solid) OH OH
1 Br
1.6 × 10 =
17
Br2
X × 0.07 ¾¾¾¾® +
1 CS2, 273K
X= = 8.9 × 10−17 (Bromination)
1.6 × 1017 × 0.07 minor
Br
2- bromophenol
−17 −17 major (S)
Given, X = Y × 10 = 8.9 × 10
4- bromophenol
∴ Y = 8.9
28 JEE Advanced~Solved Paper 2019

Scheme-3
O
Br
C¾Cl O
Br Br O
OH O–Na+ C
Br Br
(i) NaOH, D (Q) Br
¾¾¾® + NaCl

Br Br Br
(S) (T)
(Major)
T compound has total number of Br atom =4

37. (c) The complex number z satisfying |z − 2 + i|≥ 5,  sin4 θ −1 − sin2 θ α 0  β


and   =  0 α + (adj M)
which represents the region outside the circle 1 + cos 2
θ cos 4
θ    |M |
(including the circumference) having centre (2, − 1) and  adj M 
−1
radius 5 units. Q M = |M| 
Y  
 sin4 θ −1 − sin2 θ α 0 
⇒  =
1 + cos θ
2
cos4 θ   0 α 
z0 (x,y)
β  cos4 θ 1 + sin2 θ
(1,0) +  
X¢ O Ö5 ¾
X |M|  −1 − cos θ sin4 θ 
2

(2,-1)   a b  d − b 
Q adj  = 
  c d  − c a  
Y¢ ⇒ β = −|M|and α = sin4 θ + cos4 θ
1 1
Now, for z0 ∈ S is maximum. ⇒ α = α (θ) = 1 − sin2(2θ), and
|z0 − 1| 2
When |z0 − 1| is minimum. And for this it is required that  2
7
β = β(θ) = −  sin2 θ cos2 θ +  + 
1
z0 ∈ S, such that z0 is collinear with the points (2, − 1) and  
 2 4
(1, 0) and lies on the circumference of the circle
|z − 2 + i|= 5.  sin2(2θ) 1  7 
2

So let z0 = x + iy, and from the figure 0 < x < 1 and y > 0. = −  +  + 
4 − z0 − z0 4 − x − iy − x + iy  4 2 4

So, =
z0 − z0 + 2i x + iy − x + iy + 2i 1
Now, α = αmin = and β = βmin = −
* * 37
2(2 − x)  2− x  2 16
= = −i  Q α is minimum at sin2(2θ) = 1 and β is minimum at
2i(y + 1)  y + 1
2− x 4 − z0 − z0 sin2(2θ) = 1
Q is a positive real number, so is purely 1 37 29
y +1 z0 − z0 + 2i So, α * + β * = − =−
negative imaginary number. 2 16 16
 4 − z0 − z0  π 39. (d) Key Idea Firstly find the centre of the given circle and
⇒ arg   =− write the coordinates of mid point ( A), of line segment PQ,
 z0 − z0 + 2i  2
since AC ⊥ PQ therefore use (slope of AC) × (slope of PQ)
38. (d) It is given that matrix = −1

 sin4 θ −1 − sin2 θ It is given that points P and Q are intersecting points of


−1
M=  = α I + βM , where circle
1 + cos θ cos4 θ 
2
(x − 3)2 + (y + 2)2 = 25 …(i)
α = α(θ) and β = β(θ) are real numbers and I is the 2 × 2
Line y = mx + 1 …(ii)
identity matrix. −3
And, the mid-point of PQ is A having x-coordinate
Now, det(M) =|M|= sin4 θ cos4 θ + 1 + sin2 θ + cos2 θ 5
3
+ sin2 θ cos2 θ so y-coordinate is1 − m.
5
= sin4 θ cos4 θ + sin2 θ cos2 θ + 2
So, A  − , 1 − m
3 3
 5 5 
JEE Advanced~Solved Paper 2019 29
From the figure, 2
h2 + h2   = 1
dy
Q AC ⊥ PQ ⇒
 dx 
Y
dy 1 − x2
⇒ =± [on replacing h by x]
Q dx x
A 1 − x2
⇒ dy = ± dx
x
O X On putting x = sinθ, dx = cosθ dθ, we get
P C
(3,-2)
1 − sin2 θ cos2 θ
dy = ± cosθdθ = ± dθ
sinθ sinθ
= ± (cos ecθ − sinθ)dθ
⇒ (slope of AC) × (slope of PQ) = −1
⇒ y = ± [ln (cos ecθ − cot θ) + cosθ] + C
 −2 − 1 + 3 m 
   1 − cosθ  
⇒ 5  × m = −1 ⇒ y = ± ln + cosθ + C

3    sinθ  
 3+
 5    1 − 1 − sin2 θ  
(3/ 5) m − 3 3m − 15 ⇒ y = ±  ln   + 1 − sin2 θ  + C
⇒ m = −1 ⇒   m = −1   sinθ 
 
18 / 5  18 
⇒ 3m2 − 15m + 18 = 0  1 − 1 − x 2  
⇒ y = ±  ln   + 1 − x2  + C [Q x = sinθ]
⇒ m2 − 5m + 6 = 0    x  

⇒ m = 2 or 3
 1 + 1 − x2 
40. (c) The given region {(x , y) : xy ≤ 8, 1 ≤ y ≤ x 2}. = ±  − ln + 1 − x2  + C
 x 
From the figure, region A and B satisfy the given region,
but only A is bounded region, so area of bounded region [on rationalization]
Y
QThe curve is in the first quadrant so y must be positive, so
xy=8 1 + 1 − x 2 
y = ln  − 1 − x2 + C
y=x2  x 
 

Q (2,4)
As curve passes through (1, 0), so
0 = 0 − 0 + c ⇒ c = 0, so required curve is
B (1,1) A R(8,1)
y=1 1 + 1 − x 2 
P y = ln   − 1 − x2
 x 
X¢ X  
O
and required differential equation is
dy 1 − x2
=− ⇒ xy′+ 1 − x 2 = 0
dx x
xy=8 Y¢ Hence, options (a) and (c) are correct.
2 8 8 
A= ∫1 (x − 1) dx + ∫2  − 1 dx
2
x  42. (c,d) Given equation of ellipse
[Q Points P(1, 1), Q (2, 4) and R(8, 1)] x 2 y2
2 E1 : + =1 …(i)
x 3 9 4
=  − x  + [8log|x|− x]2
8

 3 1 Now, let a vertex of rectangle of largest area with sides


parallel to the axes, inscribed in E1 be (3cosθ, 2sinθ).
=  − 2 − + 1 + 8 log 8 − 8 − 8 log 2 + 2
8 1
3 3  So, area of rectangle R 1 = 2 (3cosθ) × 2 (2sinθ)
14 14 = 12sin(2θ)
= − + 16 log 2 = 16 log 2 −
3 3 Y

41. (a,c) Let a point P(h, k) on the curve y = y(x), so


equation of tangent to the curve at point P is R1 E1
E2
y − k =   (x − h)
dy R2
…(i)
 dx  h, k O X
Now, the tangent (i) intersect the Y-axis at Yp , so
coordinates Yp is  0, k − h
dy  dy  dy 
 , where = 
 dx  dx  dx  (h, k )
So, PYp = 1 (given)
30 JEE Advanced~Solved Paper 2019

π
The area of R 1 will be maximum, if θ = and maximum 43. (a,b,d) Let a non-right angled ∆PQR.
4
Now, by sine rule
area is 12 square units and length of sides of rectangle R 1 P
are 2 a cosθ = 2 a = 3 2 = length of major axis of ellipse
E 2 and 2 bsinθ = 2 b = 2 2 = length of minor axis of
rS q=1
ellipse E 2.
x2 y2 O
So, E 2 : 2
+ 2
= 1 and maximum area of
 a   b Q E ¾ R
    p=Ö 3
 2  2
P q r

rectangle R 2 = 2     and so on. = = = 2 × circumradius
a b
 2  2 sinP sinQ sinR
3 1 r
x2 y2 ⇒ = = = 2 ×1 [circumradius = 1 unit]
So, En = 2
+ 2
= 1, and maximum area sinP sinQ sinR
 a   b 
    3 1
n −1
 ( 2)   ( 2) 
n −1 ⇒ sinP = and sinQ =
2 2
 a   b  ⇒ P = 120º and Q = 30º
of rectangle R n = 2  n −1
  n −1

 ( 2)   ( 2)  (Q ∆PQR is non-right angled triangle)
Now option (a), So, R = 30º
Since, eccentricity of ellipse ⇒r = 1, so ∆PQR is an isosceles triangle. And, since RSand
PE are the median of ∆PQR, so ‘O’ is centroid of the ∆PQR.
(bn)2
En = en′ = 1 − Now,
(an)2
Option (a),
2
 b  From Apollonius theorem,
 
2(PE 2 + QE 2) = PQ 2 + PR 2 ⇒ 2  PE 2 +  = 1 + 1
n −1 3
 ( 2)  b2 4 5
= 1− 2
= 1− = 1− =  4
 a  a2 9 3
  3 1 1
n −1
 ( 2)  ⇒ PE 2 = 1 − ⇒ PE 2 = ⇒ PE = units
4 4 2
is independent of ‘n ’, so eccentricity of E18 and E19 are 1 1
and OE = PE = units [QO divides PE is 2 : 1]
equal. 3 6
Option (b), Option (b),
Distance between focus and centre of E 9 = e ⋅ a 9 Again from Apollonius theorem,
a 3 5 5
2(PS2 + RS2) = PR 2 + QR 2 ⇒ 2 + RS2  = 1 + 3
= (e) = 4 × = unit 1
( 2)8 2 3 16 4 
Option (c), 1 7 7
N ⇒RS2 = 2 − ⇒ RS2 = ⇒ RS = units
Q ∑ (area of R n) < (area of R 1) + (area of R 2)+ ..... ∞ 4 4 2
n =1 Option (c),
ab ab 1
< 2ab + 2 + 2 + ....... Area of ∆SOE = (OE) (ST)
2 22 2
< 2ab1 + + + .....
1 1 1 1
= × [(PS)sin 60º]
 2 22  2 6
 1  1 1
= × ×
3
< 12  
 1 − 1 / 2 12 2 2
3
N
= square units
⇒ ∑ (area of R n) < 24, for each positive integer N. 48
n =1
Option (d),
Option (d), 1 1 1
pqsinR ( 3)()
1
2b29 2b2 ∆
Length of latusrectum E 9 = = Q Inradius of ∆PQR = = 2 = 2 2
a9 a ( 2)8 s 1 (p + q + r) 1 ( 3 + 1 + 1)
2 2
2× 4 1
= = units 3
3 × 16 6 = (2 − 3) units
2
Hence, options (c) and (d) are correct. Hence, options (a), (b) and (d) are correct.
JEE Advanced~Solved Paper 2019 31
∞ ∞
44. (b,c,d) Given quadratic equation x 2 − x − 1 = 0 having an αn − βn
And, now ∑ =∑
n =1 (α − β) 10
n n
roots α and β, (α > β) n =1 10
1+ 5 1− 5 1 ∞ α β 
n ∞ n
So, α = and β =
2 2 =  ∑   − ∑   
α − β n=1 10  
n =1 10 
and α + β = 1, αβ = −1
 α β 
αn − βn
Q an = , n≥1 1  10   α β 
α −β = − 10 , as < 1 and <1
α − β 1 − α 1 − β   10 10 
So,  10 10 
αn+ 1 − βn+ 1
an+ 1 = = αn + αn−1 β + αn− 2 β 2 + ....+ αβn−1 + βn 1  α β  1 10α − αβ − 10β + αβ 
α −β =  −  =
α − β  10 − α 10 − β  α − β 100 − 10 (α + β) + αβ 
= αn − αn− 2 − αn− 3 β −....− βn−2 + βn [as αβ = −1]
10(α − β)
= αn + βn − (αn− 2 + αn− 3β + ...+ βn− 2) =
(α − β) [100 − 10 (α + β) + αβ]
= αn + βn − an−1 10 10
= =
 αn−1 − βn−1  100 − 10 − 1 89
 as an−1 = = αn− 2 + αn− 3 β + ...+ βn− 2 
 α −β  Hence, options (b), (c) and (d) are correct.

⇒ an+ 1 + an−1 = αn + βn = bn , ∀n ≥ 1 45. (a,b,c) Given lines


So, option (b) is correct. L1: r = $i + λ(− $i + 2$j + 2k$), λ ∈ R and
∞ ∞
b αn + βn L : r = µ (2$i − $j + 2k$), µ ∈ R
Now, ∑ nn = ∑ n
[as, bn = αn + βn] 2
n =1 10 n =1 10 and since line L 3 is perpendicular to both lines L1 and L 2.
∞ n ∞ n
α +  β  Q α < 1 and β < 1 Then a vector along L 3 will be,
= ∑ 10  ∑ 10 
 10 10  $i $j k$
n =1 n =1
α β –1 2 2 = $i(4 + 2) − $j(−2 − 4) + k$ (1 − 4)
10 10 α β 2 –1 2
= + = +
α β 10 − α 10 − β
1− 1− = 6$i + 6$j − 3k$ = 3(2$i + 2$j − k$) …(i)
10 10
10α − αβ + 10β − αβ Now, let a general point on line L1 .
=
(10 − α) (10 − β) P(1 − λ , 2λ , 2λ) and on line L 2.
10(α + β) − 2αβ as Q(2µ , − µ , 2µ) and let P and Q
=
100 − 10(α + β) + αβ are point of intersection of lines L1 , L 3 and L 2 , L 3, so
101 () − 2(−1) direction ratio’s of L 3
= [as α + β = 1 and αβ = −1] (2µ + λ − 1, − µ − 2λ , 2µ − 2λ) …(ii)
100 − 101 () − 1
2µ + λ − 1 −µ − 2λ 2µ − 2λ
12 Now, = =
= 2 2 −1
89
[from Eqs.(i) and (ii)]
So, option (a) is not correct. 1 2
Q α 2 = α + 1 and β 2 = β + 1 ⇒ λ = and µ =
9 3
⇒ αn+ 2 = αn+ 1 + αn and βn+ 2 = βn+ 1 + βn 
So, P  , ,  and Q  , − , 
8 2 2  4 2 4
⇒ (αn+ 2 + βn+ 2) = (αn+ 1 + βn+ 1) + (αn + βn)  9 9 9 9 9 9
⇒ an+ 2 = an+ 1 + an Now, we can take equation of line L 3 as
Similarly, an+ 1 = an + an−1 r = a + t(2$i + 2$j − k$), where a is position vector of any
an = an−1 + an−2 point on line L 3 and possible vector of a are
………  8 $i + 2 $j + 2 k$  or  4 $i − 2 $j + 4 k$  or  2 $i + 1 k$ 
………      
a 3 = a 2 + a1 9 9 9  9 9 9  3 3 
On adding, we get Hence, options (a), (b) and (c) are correct.
an+ 2 = (an + an−1 + an− 2 + ....+ a 2 + a1) + a 2
 α2 − β2  46. (a,c) Key Idea: Use conditional probability,
Q a 2 = = α + β = 1 total probability and Baye’s theorem.
 α −β 
So, an+ 2 − 1 = a1 + a 2 + a 3 + .....+ an It is given that there are three bags B1 , B 2 and B 3 and
So, option (c) is also correct. probabilities of being chosen B1 , B 2 and B 3 are
respectively.
32 JEE Advanced~Solved Paper 2019

∴ P(B1) =
3
, P(B 2) =
3 4
and P(B 3) = . 0 1 a 0 −2 −4
So 1 2  |M| 
10 10 10 3 = −2 −4 −6
  4  
5 R 3 R 5 R  3 b 1   −6 −2 −2
5 G 5 G 3 G
⇒|M|= −2, a = 2and b = 1
B1 B2 B3  0 1 2
Now, probability that the chosen ball is green, given that ⇒ M = 1 2 3
G 3  
selected bag is B 3 = P   =  3 1 1 
 B3  8
α  1 
Now, probability that the selected bag is B 3, given that the Now, If M β  =  2
   
chosen ball is green = P  3 
B
G  γ   3
G  0 1 2 α  1 
P   P(B 3)
 B3  ⇒ 1 2 3 β  =  2
=     
G G G  3 1 1   γ   3
P   P(B1) +P   P(B 2) + P   P(B 3)
 B1   B2   B3  ⇒B + 2γ = 1, α + 2β + 3γ = 2and 3α + β + γ = 3
[by Baye’s theorem] ⇒α = 1, β = −1 and γ = 1
3× 4 1 ∴α − β + γ = 3
  And (adj M)−1 + adj (M −1) = 2(adj M)−1
 8 10  2 4
= = =
 5 × 3  +  5 × 3  +  3 × 4  1 + 5 + 1 13 [Q adj (M −1) = (adj M)−1 ]
     
 10 10   8 10   8 10  2 8 2
= 2 −  = − M
M M
[Q(adj M)−1 = from Eq. (i)]
Now, probability that the chosen ball is green  2 |M|
G G G
= P(G) = P(B1)P   + P(B 2)P   + P(B 3)P   andQ a = 2and b = 1, so a + b = 3
 B1   B2   B3  Hence, options (b), (c) and (d) are correct.
[By using theorem of total probability]
48. (b,c,d) Given function f : R → R is
=  ×  +  ×  +  × 
3 5 3 5 4 3
 10 10   10 8   10 8   x 5 + 5x 4 + 10x 3 + 10x 2 + 3x + 1 , x<0

3 3 3 12 + 15 + 12 39  x2 − x + 1 , 0≤ x <1
= + + = = f(x) =  2 8
20 16 20 80 80 x − 4x 2 + 7x −
3
, 1≤ x < 3
 3 3
Now, probability that the selected bag is B 3 and the chosen  10 , x≥3
G 4  (x − 2) log e (x − 2) − x +
3 3
ball is green = P(B 3) × P   = × = 3
 B 3  10 8 20  5x 4 + 20x 3 + 30x 2 + 20x + 3 , x<0
 0< x <1
Hence, options (a) and (c) are correct. 2x − 1 ,
So, f ′(x) = 
0 1 a  2x 2 − 8x + 7 , 1< x < 3
47. (b,c,d) Given square matrix M = 1 2 3 
 log e(x − 2) , x>3
 
 3 b 1  − +
At x = 1, f′′ (1 ) = 2 > 0 and f′′ (1 ) = 4 − 8 = −4 < 0
 −1 1 −1 ∴f ′(x) is not differentiable at x = 1 and
and adj (M) =  8 −6 2 f ′(x) has a local maximum at x = 1.
 
 −5 3 −1 For x ∈ (−∞ , 0)
−1 1 −1 f ′(x) = 5x 4 + 20x 3 + 30x 2 + 20x + 3
Q adj (M) =|M|2 = 8 −6 2 and since f′(−1) = 5 − 20 + 30 − 20 + 3 = −2 < 0
−5 3 −1 So, f(x) is not increasing on x ∈ (−∞ , 0).
⇒ |M|2 = −1(6 − 6) − 1(−8 + 10) − 1(24 − 30) Now, as the range of function f(x) is R, so f is onto
function.
= −2 + 6 = 4
Hence, options (b), (c) and (d) are correct.
⇒ |M|= ±2
∴det (adj M 2) =|M 2|2 =|M|4 = 16 49. (0.50) Given sample space (S) of all 3 × 3 matrices with
entries from the set {0, 1} and events
As we know A(adj A) =|A|I
E1 = {A ∈ S : det(A) = 0} and
⇒ M = |M|(adj M)−1 …(i)
T E 2 = {A ∈ S : sum of entries of A is 7}.
0 −2 −6  0 −2 −4
 −2 For event E 2, means sum of entries of matrix A is 7, then
−2 =  −2 −4 −6
1 1
Q(adj M)−1 = −4
|adj M|   4  we need seven 1s and two 0s.
 −4 −6 −2  −6 −2 −2
JEE Advanced~Solved Paper 2019 33
9! 1
∴Number of different possible matrices = = {(a − b)2 + (b − c)2 + (c − a)2}
7! 2! 2
⇒ n(E 2) = 36 Q a, b and c are distinct non-zero integers.
For event E1 ,|A|= 0, both the zeroes must be in same For minimum value a = 1, b = 2and c = 3
row/column. 1 6
∴|a + bω + cω2|2min = {12 + 12 + 2 2} = = 300 .
∴Number of matrices such that their determinant is zero 2 2
3! π/4
= 6 × = 18 = n(E1 ∩ E 2) 52. (4.0) Given, I = 2 ∫ dx
…(i)
2! π − π/4 (1 + esin x) (2 − cos 2x)
 E1  n(E1 ∩ E 2) b b
∴Required probability, P   = On applying property ∫ f(x)dx = ∫ f(a + b − x) dx, we get
 E2  n(E 2) a a

18 1 2 π/4 esin x dx
= = = 0.50
36 2
I=
π ∫− π/4 (1 + esin x) (2 − cos 2x) ....(ii)

50. (10) According to given informations the figure is as On adding integrals (i) and (ii), we get
2 π/4 dx
following 2I = ∫
π − π/4 2 − cos 2x
Q 1 π/4 dx  1 − tan2 x 
⇒ I= ∫  as cos 2x = 
P 90° π − π/4 1 − tan x 
2
1 + tan2 x 
2 2−
q 1 1 + tan x
2

C B M A (2,3) 2 π / 4 sec2 x
= ∫ dx
π 0 1 + 3tan2 x
8x-6y-23=0  sec2 x 
From the figure, Q is even function
 1 + 3tan x
2
2 
AC = …(i)
sinθ Put 3tan x = t ⇒ 3sec2 dx = dt , and at x = 0, t = 0 and
1 at x = 3, t = 3
Q sinθ = (from ∆CPB)…(ii)
2 3 1 dt 2
CB So, I= ∫ = [tan−1 t]0 3
2 2 π 0 3 1 + t2 3π
and sinθ = = (from ∆CQA)...(iii)
AC CB + AB 2  π 2
=   = ⇒ 27I 2 = 4.00
Q AB = AM + MB = 2AM [QAM = MB] 3π  3  3 3
|(8 × 2) − (6 × 3) − 23| 2 × 25
=2 = = 500
. 53. (0.75) Given three lines
64 + 36 10
r = λ$i , λ ∈ R,
From Eqs. (ii) and (iii), we get
1 2 r = µ ($i + $j), µ ∈ R
sinθ = =
CB CB + AB and r = v ($i + $j + k$), v ∈ R
1 2
⇒ = [QAB = 5] cuts the plane x + y + z = 1 at the points A , B and C,
CB CB + 5 respectively. So, for point A, put (λ , 0, 0) in the plane, we
1 get λ + 0 + 0 = 1 ⇒ λ = 1 ⇒ A ≡ (1,0, 0). Similarly, for point
⇒ CB + 5 = 2CB ⇒ CB = 5 =
sinθ 1
B, put (µ , µ , 0) in the plane, we get µ + µ + 0 = 1 ⇒µ = ⇒
From the Eq. (i), we get 2
B ≡  , , 0 .
2 1 1
AC = = 2 × 5 = 10.00
sinθ 2 2 
and for point C, put (v , v, v ) in the plane we get
51. (3.00) Given, ω ≠ 1 be a cube root of unity, then
⇒ C ≡  , , 
1 1 1 1
|a + bω + cω |22
v + v + v =1 ⇒ v =
3  3 3 3
= (a + bω + cω2) (a + bω + cω2), [Q zz = |z|2 ] 1
Now, area of ∆ABC = |AB × AC|= ∆
= (a + bω + cω2) (a + bω + 2 cω 2 ) 2
1$ 1$ 2 1 1
= (a + bω + cω2) (a + bω2 + cω) [Q ω = ω2 and ω 2 = ω] Q AB = − i + j, and AC = − $i + $j + k$
2 2 3 3 3
= a + abω + acω + abω + b ω + bcω2 + acω2 + bcω4 + c2ω3
2 2 2 3
$i $j k$
= a 2 + b2 + c2 + ab(ω2 + ω) + bc(ω2 + ω4) + ac(ω + ω2)
∴AB × AC = −1
2
1
2 0
[as ω3 = 1] −2 1 1
3 3 3
= a 2 + b2 + c2 + ab(−1) + bc(−1) + ac(−1)
= $i   − $j  –  + k$  – +  = ($i + $j + k$)
1 1 1 2 1
[as ω + ω2 = −1, ω4 = ω]
 6  6  6 6 6
= a + b + c − ab − bc − ca
2 2 2
34 JEE Advanced~Solved Paper 2019

1 1 Then
⇒|AB × AC| = 3=
62 3 3m − 2 = 5n − 3 = 7r − 4
⇒∆ =
1
⇒(6∆)2 = 36
1 3
= = 0.75 Now, for AP(1 ; 3) ∩ AP(2 ; 5) ∩ AP(3; 7), the common
4 3 16 × 3 4 5n − 1
terms of first and second progressions, m =
3
54. (157.00) Given that, AP(a; d) denote the set of all the ⇒ n = 2, 5, 11, … and the common terms of second and the
terms of an infinite arithmetic progression with first third progressions,
term ‘a’ and common difference d > 0. 5n + 1
r= ⇒ n = 4, 11,…
Now, let mth term of first progression 7
AP(1 ; 3) = 1 + (m − 1)3 = 3m − 2 …(i) Now, the first common term of first, second and third
and nth term of progression progressions (when n = 11), so a = 2 + (11 − 1)5 = 52
AP(2; 5) = 2 + (n − 1)5 = 5n − 3 …(ii) and d = LCM(3, 5, 7) =105
and rth term of third progression AP (3; 7) So, AP(1 ; 3) ∩ AP(2 ; 5) ∩ AP(3 ; 7) = AP(52 ; 105)
= 3 + (r − 1)7 = 7r − 4 …(iii) So, a = 52and d = 105
are equal. ⇒ a + d = 157.00

Paper 2
1. (c, d) R >> Dipole size.
v+dv
Circle is equipotential. v0
So, Enet should be perpendicular to surface hence,
1/ 3
kp0  kp  x
= E0 ⇒ R =  0 
r3  E0  v
f=
At point B, net electric field will be zero. 2x
EB = 0 dv
= f × 2v
2kp0 dt
(E A )net = + E 0 = 3E 0 v
R3 dv = 2vdt
3 2x
Electric field at point A, E A = E 0 [$i + $j] v
2 dv = 2(− dx)
2x
(E B)net = 0
Integration on both sides limits v 0 to v, we get
y v x
dv − dx
E0 ∫v = ∫ x
v 0 l

kp0 B A 2kp0 v x
+E0 ⇒ ln = − ln
45° 45° R3
R3 O v0 l
x
R v0 l
⇒ v=
x
l
where, x = , v = 2v 0
2
2 v 0 2v 0
2. (a, c) so, f= =
l l
2
2
v0 v
kf
∴ =4
Before collision ki

3. (a, d) Case-I
v0+2v v H = 30 cm
3
After Collision n=
2
Change in speed = (2v + v 0 − v 0) = 2v H n
H 30 × 2
H1 = ⇒ = 20 cm
In every collision it acquires 2v , n 3
X
JEE Advanced~Solved Paper 2019 35
Case-II 14M 8Mg
or a1 = − kx …(iii)
R = 300 cm 3 3
R
n2 n1 n2 − n1 This is equation of SHM
− =
v u R Maximum extension = 2 × amplitude
3 H n 8Mg
1− i.e., x0 = 2 ×
1 3 2 3k
− =
−H 2 −2 × 30 −300 X
x 0 3Mg 3k
600 Amplitude = = and ω =
H2 = = 20.684 cm 2 3k 14M
29
x0
Case-III R At , acceleration is easily found from Eq. (iii) ,
4
3
1− 14M 8Mg 4Mg
n2 n1 n2 − n1 1 3 2 a1 = −
− = ; − = 3 3 3
v u R −H 3 −2 × 30 300 H n
2g
600 a1 =
H3 = = 19.354 cm 7
31 X
x0
At , speed of the block (2M) = ω × amplitude
4. (a) 2M 2
2T
3k 8Mg
= ×
14M 3k
A 2T
a1 ∴ option (a) is correct
B
5. (b, c, d)
T T
a2 M C l/4
C 60°
2M a3 l l/2
cos 60°
acm av 2
In the frame of pulley B, O
the hanging masses have accelerations : Can be treated like a thin rod
M → (a 2 − a1), 2M → (a 3 − a1) : downward. ∆K + ∆U = 0
∴ (a 2 − a1) = − (a 3 − a1) [constant] 1
I 0ω2 = − ∆U
Assuming that the extension of the spring is x 2
1 ml 2 2
ω = −  − mg 
We consider the FBD of A : l
d2x 2 3  4
2M ⋅ 2 = 2T − kx
dt 3g
ω=
2M 2l
kx 2T 2l 3g l 3g
⇒ aradial = ω = = ⇒ τ = I 0α
2 2l 2 4
a1
l
d2x mg sin 60°
where, a1 ≡ …(i) α= 2 =
3 3g
dt 2 l2 4l
m
and the FBD of the rest of the system in the frame of 3
pulley B :
a v =  α  sin 60° + ω2 cos 60°
l l
T T ⇒
 2 2
3 3g 3 3g 9g 6g
M 2M av = + ⇒ av = +
8 2 8 16 16
mg − N = ma v
M (g–a1) 2M (g–a1) mg
N=
Upward acceleration of block M w.r.t. the pulley B 16
= Downward acceleration of block 2M w.r.t the pulley, P
6. (b) S1S2=d
T − M(g − a1) 2M(g − a1) − T S1
= y
M 2M
a q
4M Q
⇒ T= (g − a1) …(ii) d
3
Substituting in Eq. (i), we get S2 D
8M
2M ⋅ a1 = (g − a1) − kx ∆x = dsinα + dsinθ
3
36 JEE Advanced~Solved Paper 2019

θ and α are small angles Also


KE α m
= Rn
dy KE Rn mα
∴ ∆ x = dα +
D mα 4
(a) α = 0 ⇒KE Rn = ⋅ KE α = × 4.44 = 0.08 MeV
dy 0.3 × 11 mRn 222
∴ ∆x = = = 33 × 10−4 mm
D 1000 ∴Energy of γ − Photon = 4.655 − (4.44 + 0.08)
33 × 10−4 11λ = 0135
. MeV = 135keV
∆x in terms of λ = λ=
600 × 10−6 2 10. (1.50) For TIR at coating,
λ
as ∆x = (2n − 1)
2
There will be destructive interference. 75° Coating
0.36 π 0.3 mm × 11 mm
(b) ∆x = 0.3 mm × × +
π 180 1000
q
= 39 × 10−4 mm (75°–qc)
qc
−9
−4 600 × 10 × 10 3
39 × 10 = (2n − 1) ×
2 RI = 3
n=7
n
So, there will be destruction interference. sinθc =
0.36 π 3
(c) ∆x = 3mm × × + 0 = 600 nm
π 180 Applying snell’s law at first surface
600 nm = nλ sinθ = 3sin(75° − θc)
n =1 For limiting condition, at θ = 60°
So, there will be construction interference. sin 60° = 3sin(75° − θc)
(d) Fringe width does not depend on α. 3
= 3sin(75° − θc)
1
1− 2
λ a E 4 − E1 16 1 1
7. (a, b) = = = = sin(75° − θc) ⇒sin 30° = sin(75° − θc)
λ e E 4 − Em 1

1 5 2
m2 16 30° = 75° − θc ⇒θc = 45°
On solving we get, n 1 3
= ⇒ n2 = = 1.50
m=2 3 2 2
12400 × 4
λe = = 3647 11. (1.00) Momentum transferred on mirror = 2Nh
136. λ
K 2 12 1 1 2Nh
= = as kinetic energy is proportional to 2 . = MV(mean position)
K1 22 4 n λ
V(mean position) = Ω A (where, A = 1 µm)
8. (a, c, d) γmix = n1CP1 + n2CP2 = 8
n1CV1 + n2CV 2 2Nh
5 = MΩ A (where, λ = 8 π × 10−6)
P V − P2V2 λ
W= 1 1
γ −1 MΩ(10−6)λ MΩ 8 π × 10−6 × 10−6
N= =
8/ 5 2h 2h
P0 V08 / 5 = P2  0 
V
4 π MΩ
 4 N= −12
× 10 = 10 × 10−12
24
h
P2 = 9.2P0 N = 1 × 1012
V0
P0 V0 − 9.2P0 x =1
W= 4 = − 13RT
0
3/ 5 12. (1.38 & 1.39) u = (x 2 − x1) = 75 − 45 = 30 cm
∴ |W| = 13RT0 1 1 1
∆u = ∆x 2 + ∆x1 = + = cm
(b) T1 V1γ − 1 = T2V2γ − 1 4 4 2
v = (x 3 − x 2) = 135 − 75 = 60 cm
T2 = T1 (2)6/ 5 = 23
. T0 1 1 1
∆v = ∆x 3 + ∆x 2 = + = cm
Average kinetic energy of gas mixture = nCVmix T2 4 4 2
= 23RT0 135 cm
9. (135.00) Mass defect ∆m = 226.005 − 222.000 − 4.000
45 cm
= 0.005amu O
∴ Q value = 0.005 × 931.5 = 4.655 MeV O
75 cm 60 cm I
Momentum P = 2km = constant
JEE Advanced~Solved Paper 2019 37
1 1 1 1 1 1
∴ − = ⇒ + = 16. (c)
v u f 60 30 f
Case 1. L = L 0 , T = T0 , f = f0
− dv − du − df
∴ f = 20 cm Also, 2 + 2 = 2
v u f 1 T0
f1 =
df  dv du   1 1 1 2L 0 µ
⇒ = f 2 + 2 = 20 +
f  v u   602 302  2 3L 0
Case 2. L =
× 100 = 10  + =
df 1 1 50 2

f  36 9  36 3 T2
f2 = = f0
= 1.38 and 1.39 (both) 3L
2× 0 2µ
2
13. (4.00) For first projectile,
1 T2 T
< V> =
R
= Ux = v1 ⇒ f0 = ⇒ T2 = 0
T 2L 0 µ 2
For journey, 5L 0
Case 3. L =
R + R 2 + … + Rn 4
< v > 1 to n = 1
T1 + T2 + … + Tn 5 T3
f3 = = f0
5L 0 3µ
2u x1 u y1 2u x 2 u y 2 2u xn u yn 2×
+ + …+ 4
= g g g 2 T3 3T
2u y1 2u y 2 2u yn ⇒ f0 = ⇒ T3 = 0
+ +… 3L 0 µ 16
g g g 7L 0 14 T4
Case 4. L = ⇒ f4 = = f0
1 + 1
+
1
+ … 2n 
1 4 7L 0 4µ
 2×
ux  α2 α4 α  = 0.8v 4
1 1 1  1
2 T4 T
1 + + + …+ n ⇒ f0 = ⇒ T4 = 0
 α α2 α  L0 µ 16
 
 1  17. (d) 1-2 process is isothermal and 2-3 process is
v0 
1  isochoric.
1 − 2 
 α  = 0.8v ⇒ α = 0.8 (I) W1→ 2 = nRI ln
Vf T V
= 1 × R 0 ln 2
1
  1+α Vi 3 V1
 1  RT0 2V RT
 1 = ln 0 = 0 ln 2
1 −  3 V0 3
 α
W2 → 3 = 0 (Isochoric process)
⇒ α=4
RT0
W1→ 2 → 3 = W1→ 2 + W2 → 3 = ln 2 (I → P)
14. (0.63) Motional emf, e = (v × B)dl = 10−2 × 1 × 10−1 3
e = 10−3 V (II) ∆U =
f
nR (Tf − Ti)
2
τ L = LR = (10−3)()
1 = 10−3 s = 1 ms
3 T 
10−3 ∆U1→ 2 → 3 = 1 × R  T0 − 0 
i = i 0 (1 − e−t / τ L ) = (1 − e−1) 2   3  
1
= RT0 (II → R)
i = 10−3(1 − 0.37)
(III) Q1 → 2 → 3 = ∆U1 → 2 → 3 + W1 → 2 → 3
l = 0.63 mA
(First law of thermodynamics)
15. (b) Fundamental frequency is maximum when length is
RT0
minimum i.e. L 0 , = RT0 + ln 2
3
1 T0 RT
Case 1. L = L 0 , T = T0 , f = f0 ; f1 = = 0 [3 + ln 2] (III → T)
2L 0 µ 3
1 T2 f (IV) Q1→ 2 = ∆U1→ 2 + W1→ 2
Case 2. f2 = = 0
L0 2µ 2 RT0 RT
=0+ ln 2 = 0 ln 2 (IV → P)
1 T2 f 3 3
Case 3. f3 = = 0
L0 3µ 3 18. (d) (I) W1→ 2 → 3 = W1→ 2 + W2 → 3
= P0 [2V0 − V0 ] + 0 = P0 V0
1 T2 f
Case 4. f4 = = 0 RT
L0 4µ 2 W1→ 2→ 3 = P0 V0 = 0 (I → Q)
3
38 JEE Advanced~Solved Paper 2019

3  3P0
× 2V0 − P0 V0  = × 2P0 V0 = 3P0 V0 = RT0
3
(II) U1→ 2→ 3 = (II → R)
2  2  2
RT 4RT0
(III) Q1→ 2→ 3 = U1→ 2→ 3 + W1→ 2→ 3 = RT0 + 0 = (III → S)
3 3
5 5 5 5 RT0 5
(IV) Q1→ 2 = nCP ∆T = n R (T2 − T1) = [P0 2V0 − P0 V0 ] = P0 V0 = = R 0 T0 (IV → U)
2 2 2 2 3 6

19. (a, b)
C C CH2 CHO C CH CH2 CHO
(i) Hg2+, dil. H2SO4
Hydration of Alkyne OH
CH3O CH3O
Keto–enol
Tautomerism
(ii) AgNO3+NH4OH
C CH2 CH2 C O– C CH2 CH2 CHO
(Tollen's reagent)

O O (¾CHO converts into COO–) O


CH3O CH3O
CH2 CH2
(iii) Zn – Hg + Conc. HCl CH2 (i) SOCl2+Pyridine CH2
(Clemmensen reduction) (ii) AlCl3
CH3O CH2 CH3O H
C ¾ reduce to ¾ CH2 Cl CH2
(Q) C
COOH
O
O
CH2 CH2
CH2 CH2
Zn-Hg/Conc. HCl Electrophilic

CH2
Clemmensen reduction CH2 substitution reaction
CH3O CH2 CH3O C
(S)
O
(R)

20. (c, d) Key Idea An aromatic compound must be cyclic and planar. It must follow ( 4 n + 2 ) e − rule and have the conjugated system in it.

(a) Br OEt
NaOEt Dimerise
+
Substitution product Ellimination product (Non-aromatic)

(b) Benzene react with Cl 2 (excess) in presence of UV light and 500 K of temperature to form benzene hexachloride
(non-aromatic).
Cl
Cl Cl
UV
+ Cl2 (excess) 500 K

Cl Cl
Cl
(Non-aromatic)

CH3

(c) CH3 (i) Alc. KOH (iii) Red hot iron tube
CH3C CH
Br (ii) NaNH2 873 K (Trimerisation)
Br CH3 CH3
(Aromatic)
H
σ Na+
(d) NaOMe
–MeOH

(Aromatic ion)
Thus, (c) and (d) options are correct.
JEE Advanced~Solved Paper 2019 39

21. (a, c) Given, ground state energy of hydrogen atom 24. (a, d) The explanation of given statements are as
= − 136
. eV follows: (a) Teflon is prepared by heating
Energy of He + = − 34
. eV, Z = 2 tetrafluoroethene in presence of persulphate catalyst at
higher pressure.
. × Z2
136
Energy of He + , E = − eV nCF2 ==CF2
Persulphate
CF2
n2 catalyst
CF2
− 136
. × (2)2 . ×4
136 Thermoplastic Polymer
. eV =
− 34 ⇒ n= ⇒ n=4 (PTFE)
n2 34
.
Thus, option (a) is correct.
Given, azimuthal quantum number () l = 2(d – subshell (b) Natural rubber is polyisoprene containing cis alkene
Magnetic quantum number (m) = 0 units.
∴ Angular nodes () l =2
Radial node = n − l − 1 = 4 − 2 − 1 = 1 CH2 CH2 n
nl = 4d state nCH2 C CH CH2 C C
Hence, options (a), (c) are correct. CH3 H
CH3 cis-ployisoprene unit
22. (b, c, d) When Zn react with hot conc. H 2 SO4 then SO2 Isoprene (natural rubber)
is released and ZnSO 4 is obtained. Thus, option (b) is incorrect.
Zn + 2H 2 SO4 → ZnSO4 + SO2 ↑ + 2H 2 O (c) Cellulose has only β-D-glucose units that are joined
(Hot + Conc.) (G) (R) (X) together by glycosidic linkages as shown in the following
structure:
R(SO2) molecule is V-Shaped
OH OH OH OH
S O O O
O OH O
O OH 1 4 OH
, OH O
O O O
Thus, option (b) is correct.
When Zn is react with conc. NaOH then H 2 gas is evolved OH OH OH O
n OH
and Na2ZnO2 is obtained. (cellulose)
(1, 4, b -linkage)
Zn + 2NaOH (conc.) → Na2ZnO2 + H 2 ↑
(T) ( Q)
Thus, option (c) is incorrect.
In ground state, H—H (Q) (bond order = 1) (d) Nylon-6 has amide linkages.
Thus, option (c) is correct. O
The oxidation state of Zn in T(Na 2ZnO 2) is +2 O
C
Thus, option (a) is incorrect. NH ∆, H2O –nH2O
[H2N(CH2)5COOH] C (CH2)5 NH
ZnSO4 + H 2 S + NH 4 OH → ZnS↓ + 2H 2 O ∆
n
( Z) Nylon-6
( G) ( X)
Caprolactam
+ (NH 4 )2 SO4
( Y)
Thus, option (d) is correct.
ZnS (Z) compound is dirty white coloured.
25. (a, b, c) Cyanide process of gold extraction involves
Thus, option (d) is correct.
leaching out gold from its ore with CN − in the presence of
23. (a, b, c) The explanation of given statements are as O 2 (Q) in water to form [Au(CN)2 ]− (R).
follows: When [Au(CN)2 ]− reacts with Zn (T), it froms [Zn(CN)4 ]2−
(a) Aqua-regia is prepared by mixing conc. HCl and conc. (Z) and Au.
HNO 3 in 3:1 (v/v) ratio and is used in oxidation of
The corresponding reactions are as follows :
gold and platinum. Hence, option (a) is correct.
H O + O2 ( Q)
(b) Yellow colour of aqua-regia is due to its decomposition s + 8 CN − (aq) →
4 Au () 2
4[Au(CN)2 ]− + 4OH − (aq)
into NOCl (orange yellow) and Cl 2 (greenish yellow). (R)
Hence, option (b) is correct. 2[Au(CN)2]− (aq) + Zn()
s → [Zn(CN)4 ]2− (aq) + 2 Au()
s
(c) When gold reacts with aqua-regia then it produces ( R) (T)
AuCl −4 anion complex in which Au has +3 oxidation Hence, options (a, b, c) are correct.
state.
0 +3 26. (a, c) The given reactions takes place as follows:
Au+ HNO 3 +4HCl → AuCl –4 + H 3O + + NO + H 2O
Cl Zn, dil. HCl
Oxidation
CH3 CH2 CH3
(a) CH3 (Propane)
Hence, option (c) is correct. Br
(d) Reaction of gold with aqua-regia produces NO gas in Br Zn
absence of air. CH3 CH CH2 + ZnBr2
(b) CH ∆
3
(Elimination (Propene)
Hence, option (d) is incorrect.
reaction)
40 JEE Advanced~Solved Paper 2019

CH3 NaOH, CaO, D


CH3 CH2 CH3+ CO2 29. (6)
Electrolysis
(c) COONa (Propane)

(d) CH3 Electrolysis (i) H2, Pd–BaSO4


CH3 (CH2)4 CH3
n-hexane Quinoline (reduction)
COONa
(Triple bond converts into
Thus, options (a, c) are correct. double bond)

27. (2.3) At constant V, T



2N 2 O5 (g) → 2N 2 O4 (g) + O2(g)
CH2 CH2
At initial t = 0 1 0 0 (ii) Dil. KMnO 4
C C
t = Y × 103 sec 1 − 2p 2p p (excess) 273K
H H
pTotal = 1 − 2p + 2p + p OH
1. 4 = 1 + p H H
p = 0.45 atm
According to first order reaction, C C CH2 OH
pi OH
2303
.
k= log OH OH
t pi − 2p OH
(P) (Major product)
pi = 1 atm (given)
Compound (P) has total number of hydroxyl groups = 6
2p = 2 × 0.45 = 0.9 atm
On substituting the values in above equation, 30. (6) The structure of cis-[Mn(en)2Cl 2] complex is
1 Cl (a) Cl (a)
2k ⋅ t = 2.303log Cl N1 Cl (b)
1 − 0.9 (b)
Mn Mn
1
2 × 5 × 10−4 × y × 103 = 2.303log en en N2 N4
.
01 N
y = 2.303 = 2.3 3
Bond angles (Mn—N and Mn—Cl bond in cis positions)
Note Unit of rate constant (k), i.e. s−1 represents that it is a
Cl (a) —— Mn —— N(1)
first order reaction.
Cl (a) —— Mn —— N(2)
28. (2.98 mole) Cl (a) —— Mn —— N(4)
Number of moles of solute × 1000 Cl (b) —— Mn —— N1
Key Idea Molarity ( M) =
Volume of solution (in mL) Cl (b) —— Mn —— N3
Cl (b) —— Mn —— N4
Mass
Also, volume = Number of cis Cl—Mn—N = 6
Density
Given, mole fraction of urea (χ urea) = 0.05 31.(288) Key Idea Rhombic sulphur (S 8 ) gets oxidised into
sulphuric acid and water, NO 2 gas is released on
Mass of water = 900g
reaction with conc. HNO 3.
Density = 1.2 g/cm 3
nurea 900 When rhombic sulphur (S8 ) is oxidised by conc. HNO3
χ urea = [Q Moles of water = = 50] then H 2 SO4 is obtained and NO 2 gas is released.
nurea + 50 18
S8 + 48HNO3 → 8H 2 SO4 + 48NO2 + 16 H 2O
nurea
0.05 = ⇒ 19nurea = 50 1 mole of rhombic sulphur produces = 16 moles of H 2 O
nurea + 50
∴ Mass of water = 16 × 18 (molar mass of H 2 O)
nurea = 2.6315 moles = 288 g
w urea = nurea × (M ⋅ wt) urea = (2. 6315 × 60)g
2.6315 × 60 + 900
32. (10.0) The structure of cyclic ether with molecular
V= formula, C4H 8O are as follows:
1.2
Q Density = Mass of solution  (chiral carbon)
 Volume of solution  O (2)
(1) *
= 881.57 mL R isomer
= 2 isomer
, O S isomer
Now, molarity
1000
= Number of moles of solute × O
Volume of solution (mL) R isomer = 2 isomer
(3) (4)
2.6315 × 1000 O , * S isomer
= = 2.98 M
881.57 (1-chiral carbon)
JEE Advanced~Solved Paper 2019 41

O O R, R isomer O
(5) (6) R, S isomer = 3 isomer
* * , S, S isomer C OMe
(i) LiAlH4
(2-chiral carbon) (IV) CH2OH
C OMe
Total number of isomers of cyclic ether with molecular CH2 (Q)
formula, C4 H 8 O are 10. CH2OH
O
33. (a) (III) Kinetic energy of the electron in n th orbit,
(ii) conc. H2SO4
Z2 + H2O
K.E. = + 136 . × 2 O
Dehydration
n
(R)
1
or K.E. ∝ 2 or K.E. ∝ n−2
n Hence, correct match of IV is Q , R .
From list-II, correct match is (III P). Hence, correct matching from list-I and list II on the
(IV) Potential energy of the electron in the n th orbit, basis of given option is (IV), Q, R.
Z2 36. (a)
P.E. = − 2 × 136. × 2
n O
1
P.E. ∝ 2 CH2 C H
n
CN
P.E. ∝ n−2
(i) DIBAL–H
(I)
O (ii) Dil. HCl OH
From List II, correct match is (IV P). CH
Hence, correct matching from list-I and list-II on the basis O OH
–H2O
of given option is (III, P). O
34. (c) (I) Radius of the n th orbit, CH2OH (iii) NaBH4 CH
n2 (reduction) H
r = 0.529 × CH2 OH
Z (Q) C
Here, r ∝ n2 O
From list-II, correct match is (I, T) (iv) Conc. H2SO4
(II) Angular momentum of the electron, (dehydration)
O
nh
mvr = or mvr ∝ n (R)

Hence, correct match of (I) are (Q, R)
From list-II, correct match (II, S) O
Hence, correct matching from list-I and list-II on the basis
of given option is (I, T). CH2 C H
(i) O3, (ii) Zn + H2O
(II) (Ozonolysis)
35. (b)
COOH COOH
(S) (P)
(III) Cl (i) KCN CN
Nucleophilic (iii) NaBH4
substitution reaction
C OCH3 (Cl– replaces by CN– ion) C OCH3
(iv) Conc. H2SO4 CH2 OH
O O
–H2O
O
COOH C OH
CH3OH + (ii) H3O+
O
∆ (U) O
LiAlH4

C OH (Complete hydrolysis
(T )
of —CN give —COOH Hence, correct match of II is (P ,S, U).
O and ester also get hydrolysed
into —COOH and alcohol ) Hence, correct matching from list-I and list-II on the
CH2 CH2 OH basis of given option is (II), P , S, U.
(iv) conc. H2SO4
H 2O + 37. (a, b, c) It is given, that for non-negative integers ‘n’,
D O
(Q) CH2 OH n k +1  k + 2 
(R) Σ sin  π  sin  π
n + 2 
k =0 n + 2 
f(n) =
Hence, correct match of (III) are T, Q , R .
n k +1 
Σ sin2  π
k =0 n + 2 
42 JEE Advanced~Solved Paper 2019

n   2k + 3   π
Σ  cos
π
− cos  π  Now, f(6) = cos 
k = 0 n+ 2  n + 2    8
=
n   2k + 2   Q cos−1 cos x = x 
π  
Σ 1 − cos π  Q α = tan(cos−1 f((6))) = tan   if x ∈  0, π  
k = 0  n + 2    8   
 2  
[Q2sin A sinB = cos(A − B) − cos(A + B) and
= 2 −1
2sin2 A = 1 − cos 2A]
cos 3π + cos 5π + cos 7π  ⇒(α + 1) = 2 ⇒(α + 1)2 = 2 ⇒α 2 + 2α + 1 = 2
  π  n  n+ 2 n+ 2 n + 2  ⇒α 2 + 2α − 1 = 0
 cos  
  Σ 1 −  
  n + 2  k = 0   2n + 3    π   π 3
+ ..... + cos  π Now, f(4) = cos   = cos  = ,
  n + 2    4 + 2  6 2
= 
cos 2π + cos 4π + cos 6π +     π 
 n+ 2 n+ 2 n + 2  Now, sin(7 cos−1 f(5)) = sin  7 cos−1  cos 
   5 + 2  
n
Σ 1−   
k =0
  2n + 2  
..... + cos  π  π 
  n + 2   = sin  7   = sin π = 0
   7
π
 nπ  and Now, lim f(x) = lim cos = cos 0 = 1
sin  n→ ∞ n→ ∞ n+ 2
 π   n + 2 n + 3 
(n + 1) cos   − cos  π Hence, options (a), (b) and (c) are correct.
 n + 2  π  n + 2 
sin 
 n + 2 38. (a, b, d) Given, f : R → R and f(x) = (x − 1) (x − 2) (x − 5)
=
 nπ  x
sin  
n + 2 
Since, F(x) = ∫0 f(t) dt , x > 0
 n + 2
(n + 1) − cos  π So, F ′(x) = f(x) = (x − 1)(x − 2)(x − 5)
 π  n + 2 
sin   According to wavy curve method
 n + 2
- + - +
[Q cos(α) + cos(α + β) + cos(α + 2β) + ...
1 2 5
+ cos(α + (n − 1)β)
F ′(x) changes, it’s sign from negative to positive at x = 1
 nβ   and 5, so, F(x) has minima at x = 1 and 5 and as F ′(x)
sin 
 2 2α + (n − 1)β  
= cos  
changes, it’s sign from positive to negative at x = 2, so F(x)
β   has maxima at x = 2.
sin   2
 2  2 2
QF(2) = ∫ f(t) dt = ∫ (t 3 − 8t 2 + 17t − 10)dt
 π 
sin  π −  0 0
 π   n + 2  π 
(n + 1) cos   − cos  π +  t 4
t 3
t2 
2
 n + 2  π   n + 2 =  − 8 + 17 − 10t 
sin   4 3 2
 n + 2  0
= 64 124 10
 π  =4− + 34 − 20 = 38 − =−
sin π −  3 3 3
 n + 2
(n + 1) − cos(π) Q At the point of maxima x = 2, the functional value
 π  10
sin  F(2) = − , is negative for the interval x ∈(0, 5), so F(x) ≠ 0
 n + 2 3
 π  for any value of x ∈(0, 5),
sin  
 π   n + 2  π  Hence, options (a), (b) and (d) are correct.
(n + 1) cos   + cos  
 n + 2
sin 
 π 

 n + 2 39. (a, b, d) Given, f(x) = sin(2πx) , x > 0
 n + 2 x
=
 π  x 2 π cos(πx) − 2x sin(πx)
sin  ⇒ f ′(x) =
 n + 2 x4
(n + 1) + xπ
 π  2x cos(πx) − tan(πx)
sin    2 
 n + 2 =
x4
 π 
(n + 2) cos  xπ
 n + 2  π  2cos(πx)  − tan(πx)
= = cos    2
 
=
(n + 2)  n + 2 x3
 π  Since, for maxima and minima of f(x), f ′(x) = 0
⇒ f(n) = cos  
n + 2
JEE Advanced~Solved Paper 2019 43
πx
⇒ cos(πx) = 0 or tan(πx) = , (as x > 0) 41. (c, d) Since,
2  
πx  
Q cos(πx) ≠ 0 ⇒ tan(πx) = 1 + 2 + 3 + ... + n
3 3 3
2 lim  ,
n→ ∞  
Y y=tan (px) 7/ 3  1 1 1
n  + + ... + 
 (an + 1) (an + 2) 2 (an + n)2  
2

y=px a ∈ R ,|a|> 1
2 1/ 3
Σ  
n n r 1
Σ (r1 / 3)
X r =1 r = 1  n n
1 1 P1
3
2 P2
5
3 P3
7 = lim = lim n
O 4 9 n→ ∞ n 1 n→ ∞ 1 1
2 2 2 2 n 7/ 3
Σ Σ
2
r = 1 (an + r) 2 r =1 2
a + r  n
 
 n
1

∫x
1/ 3
dx
0 ⇒ x = P1 ∈ 1,  is point of local
3
Qf ′(P1−) < 0 and f ′(P1+ ) > = 0 = 54 , (given)
 2 1
dx
minimum. ∫ (a + x)2
Qf ′(P2−) > 0 and f ′(P2+ ) < 0 ⇒ x = P2 ∈  2,  is point of local
5 0
3 4/ 3 1
 2 [x ]0
maximum. ⇒ 4 = 54
From the graph, for points of maxima x1 , x 2 , x 3 …… it is 1
 1 
clear that − x + a 
 0
5 9 13 17
− x1 > − x 2 > − x3 > − x 4 ...... 3/ 4
2 2 2 2 ⇒ = 54
1 1
⇒ xn + 1 − xn > 2, ∀ n. − +
a+1 a
From the graph for points of minima y1 , y 2 , y 3 ....., it is 3 1
clear that ⇒ = ⇒ a 2 + a = 72
4 × 54 a(a + 1)
3 5 7 9
− y1 > − x1 > − y 2 > − x 2 ......
2 2 2 2 ⇒ a 2 + 9a − 8a − 72 = 0
|xn − yn|> 1, ∀ n and x1 > (y1 + 1) ⇒ a(a + 9) − 8(a + 9) = 0
And x1 ∈  2,  , x 2 ∈  4,  , x 3 ∈  6,  ........
5 9 13 ⇒ (a − 8) (a + 9) = 0
 2  2  2 ⇒ a = 8 or − 9
⇒ xn ∈  2n, 2n +  , ∀ n.
1 Hence, options (c) and (d) are correct.
 2
Hence, options (a), (b) and (d) are correct.
42. (b, c) It is given, that f : R → R and
f(h) − f(0)
Property 1 : lim exists and finite, and
40. (c, d) Key Idea Points, A, B, C are collinear ⇒ AB , BC are h→ 0 |h|
collinear vectors ⇒ AB = λBC for some non-zero
scalar λ. f(h) − f(0)
Property 2 : lim exists and finite.
Given lines,
h→ 0 h2
L1 : r = λ$i , λ ∈ R … (i) Option a,
sin h − sin 0 1  sin h 
L 2 : r = µ $j + k$ , µ ∈ R … (ii) P2 : lim = lim   = doesn’t exist.
h→ 0 h2 h→ 0 h  h 
and L : r = $i + $j + vk$ , v ∈ R
3 … (iii) Option b,
Now, let the point P on L1 = (λ , 0, 0) h2/ 3 − 0
P1 : lim = lim h2/ 3 − 1 / 2 = lim h1 / 6 = 0
the point Q on L 2 = (0, µ , 1), and h→ 0 |h| h→ 0 h→ 0
the point R on L 3 = (1, 1, v)
exists and finite.
For collinearity of points P ,Q and R,there should be a
Option c,
non-zero scalar ‘m’, such that PQ = m PR
|h| − 0
⇒ (− λ$i + µ$j + k$) = m [(1 − λ)$i + $j + νk$ ] P1 : lim = lim |h| = 0, exists and finite.
h→ 0 |h| h→ 0
λ µ 1
⇒ = = Option d,
λ −1 1 ν
h|h| − 0 |h|  1, if h → 0+
1 µ P2 : lim = lim = 
⇒ v = and λ = where, µ ≠ 0 and µ ≠ 1 −
µ µ −1
h→ 0 h2 h→ 0 h
−1, if h → 0
f(h) − f(0)
⇒Q ≠ k$ and Q ≠ k$ + $j So lim does not exist.
h→ 0 h2
Hence, Q can not have coordinater (0, 0, 1) and (0, 1, 1) Hence, options (b) and (c) are correct.
Hence, options (c) and (d) are correct.
44 JEE Advanced~Solved Paper 2019

43. (b, d) It is given, that matrices 1 1 1   2 0 0  6 −3 0  1  1 


1 
1 1 1   2 x x ⇒ 0 2 2  0 4 0 0 3 −2 a = 6  a 
    
6      
P =  0 2 2 , Q =  0 4 0   0 0 3  0 0 6  0 0 2   b  b
   
 0 0 3  x x 6  2 4 6   6 −3 0  1  1 
1
adj (P) ⇒ 0 8 12 0 3 −2 a = 6  a
    
∴ P −1 = 6     
|P|  0 0 18  0 0 2   b  b
 6 0 0
T
12 6 4  1  1 
 0 24 8   a = 36  a
as|P| = 6 and adj P =  −3 3 0 ⇒
    
 
 0 −2 2  0 0 36  b  b
 6 −3 0  12 + 6a + 4b   36 
 0 + 24a + 8b =  36a
P −1 =  0 3 −2
1 ⇒
⇒    
6 
 0 + 0 + 36b   36b
 0 0 2 
∴ |R| = |PQP −1| [QR = PQP −1 (given)] ⇒ 6a + 4b = 24 and 12a = 8b
−1 −1 ⇒ 3a + 2b = 12 and 3a = 2b
⇒ |R| = |P||Q||P | = |Q| [Q|P||P | = |I| = 1]
⇒ a = 2and b = 3
2 x x 2 x x 2 x 0
So a + b = 5.
= 0 4 0 = 0 4 0 + 0 4 0
α   0
x x 6 x x 5 x x 1 Now, R β  =  0 and α$i + β$j + γk$ is a unit vector,
   
2 x x  γ   0
= 0 4 0 + 2 (4 − 0) − x (0 − 0) + 0(0 − 4x) so det (R) = 0
x x 5 ⇒ det(Q) = 0 [Q R = PQP −1 So,|R|=|Q|]
2 x x 2 x x
= 0 4 0 + 8, for all x ∈ R ⇒ 0 4 0 =0
x x 5 x x 6
1 1 1   2 x x  ⇒ 2 (24 − 0) − x (0 − 0) + x(0 − 4x) = 0
Q PQ =  0 2 2  0 4 0  ⇒ 48 − 4x 2 = 0
  
 0 0 3  x x 6  ⇒ x 2 = 12 ⇒ x = ± 2 3
 2 + x 4 + 2x x + 6 So, for x = 1, there does not exist a unit vector α$i + β$j + γk$ ,
=  2x 2x + 8 12  α   0
 
 3x 3x 18  for which R β  =  0
   
 2 x x  1 1 1   γ   0
and QP =  0 4 0   0 2 2
   Hence, options (b) and (d) are correct.
 x x 6   0 0 3
44. (a, b, d) Given matrices,
 2 2 + 2x 2 + 5x  1 0 0 1 0 0  0 1 0
= 0 8 8  0 0 1 , P =
P1 = I =  0 1 0 , P2 = 1 0 0
      3  
 x 3x 3x + 18
 0 0 1   0 1 0  0 0 1 
There is no common value of ‘x’, for which each  0 1 0 00 1 0 0 1
corresponding element of matrices PQ and QP is equal.
P4 =  0 0 1  , P5 = 10 0 , P6 =  0 1 0
 2 0 0      
1 0 0 1 0
 0 1 0 0
For x = 0,Q =  0 4 0
 
 0 0 6  2 1 3
6
and X = Σ Pk 1 0 2 PKT
1  1  K =1  
 3 2 1 
then, if R  a = 6  a
   
 b  b Q P1T = P1 , P2T = P2, P3T = P3, P4T = P5 , P5T = P4 and

1  1   2 1 3
⇒ PQP −1  a = 6  a [Q R = PQP ]−1 P6T = P6 and Let Q = 1 0 2 andQQ T = Q
     
 b  b  3 2 1 
Now, X = (P1QP1T ) + (P2QP2T ) + (P3QP3T ) + (P4QP4T )
+ (P5QP5T ) + (P6QP6T )
JEE Advanced~Solved Paper 2019 45

So, X T = (P1QP1T ) T + (P2QP2T ) T + (P3QP3T ) T + (P4QP4T ) T The minimum value of 6 (4 − 2α + α 2) = 6(3) = 18


+ (P5QP5T )T + (P6QP6T ) T D
[As minimum value of ax 2 + bx + c = − , if a > 0]
4a
= P1QP1T + P2QP2T + P3QP3T + P4QP4T + P5QP5T + P6QP6T 46. (1523) Given sample space S = {1, 2, 3, 4, 5, 6} and let
[Q(ABC)T = CT B T A T and (A T )T = A and Q T = Q] there are i elements in set A and j elements in set B.
⇒X = XT Now, according to information 1 ≤ j < i ≤ 6. So, total
number of ways of choosing sets A and
⇒ X is a symmetric matrix. 6
B=Σ Σ Ci 6Cj
The sum of diagonal entries of X = Tr(X) 1 ≤ j <i ≤ 6
6 2
= Σ Tr (Pi QPiT )  6 6  6
 Σ Cr  − Σ ( Cr)
6 2
i =1
r = 1  r =1 (26 − 1)2 − (12C6 − 1)
6 = =
= Σ Tr (QPiT Pi) [Q Tr (ABC) = Tr (BCA)] 2 2
i =1
12!
6 (63)2 − +1
= Σ Tr (QI) [QPi ’s are orthogonal matrices] 6! 6! 3969 − 924 + 1 3046
i =1
= = = = 1523
2 2 2
6
= Σ Tr (Q) = 6 Tr (Q) = 6 × 3 = 18 47. (6.20) It is given that
i =1
n n
1 Σ k Σ n
Ck k 2
Now, Let R = 1, then k =0 k =0
=0
 n n
1 Σ n
Ck . k Σ n
Ck 3k
k =0 k =0
6 6
XR = Σ (PK QPKT )R = Σ (PK QPKT R) n(n + 1)
K =1 K =1 n(n + 1)2 n − 2
⇒ 2 =0
n.2 n − 1
6
= Σ (PK QR) [Q PKT R = R ] 4n
K =1

 6  6  2 2 2  6  n n(n + 1) n 
6 6
 Qk Σ k= Σ nCk k = n.2 n − 1 ,
= Σ PK  3 = Σ PK  3 =  2 2 2  3 ,
=0 2 =0
K =1   K =1        n
k
n 
 6  6  2 2 2  6  Σ nCk k 2 = n(n + 1)2n − 2 and Σ nCk 3k = 4n 
k = 0 k = 0 
 30 1 1
n(n + 1) n
XR =  30 ⇒ XR = 30R ⇒ X 1 = 30 1
2n − 3
⇒ ⇒ 4 − n (n + 1) 2
2
=0
    2
 30 1 1
4n 4n−1
⇒ −n = 0⇒n = 4
⇒(X − 30I) R = 0 ⇒|X − 30I| = 0 2 2
So, (X − 30I) is not invertible and value of α = 30. n n
Ck 4 4
Ck 1 4 5 1
Hence, options (a), (b) and (d) are correct. ∴ Σ = Σ = Σ Ck + 1 = (25 − 1)
k =0 k +1 k =0 k +1 5k = 0 5
45. (18) Given vectors a = 2$i + $j − k$ 1
= (32 − 1) =
31
= 6.20
and b = $i + 2$j + k$ 5 5

So, a + b = 3$i + 3$j ⇒ |a + b| = 3 2 48. (30) Given that, no two persons sitting adjacent in
circular arrangement, have hats of same colour. So, only
Since, it is given that projection of c = α a + β b on the
possible combination due to circular arrangement is
vector (a + b) is 3 2, then
2 + 2 + 1.
(a + b). c
=3 2 So, there are following three cases of selecting hats are
|a + b| 2R + 2B + 1G or 2B + 2G + 1R or 2G + 2R + 1B.
⇒ (a + b) . (αa + βb) = 18 To distribute these 5 hats first we will select a person
⇒ α (a. a) + β (a. b) + α (b. a) + β (a . b) = 18 which we can done in 5C1 ways and distribute that hat
⇒ 6α + 3β + 3α + 6β = 18 which is one of it’s colour. And, now the remaining four
⇒ 9α + 9β = 18 ⇒ (α + β) = 2 … (i) hats can be distributed in two ways.
Now, for minimum value of (c − (a × b)) . c So, total ways will be 3 × 5C1 × 2
= (α a + β b − (a × b)) . (αa + βb) = 3 × 5 × 2 = 30
= α 2 (a. a) + αβ(a. b) + αβ(a . b) + β 2(b . b) a a
49. (0.5) Key Idea Use property ∫ f ( x) dx = ∫ f ( a − x) dx
[Q (a × b) . a = 0 = (a × b) . b] 0 0

= 6 α 2 + 6 αβ + 6β 2 = 6 (α 2 + β 2 + αβ) The given integral


π/2
= 6 [(α + β) − αβ] = 6 [4 − αβ] = 6 [4 − α (2 − α)]
2
3 cosθ
I= ∫ ( cosθ + sinθ)5
dθ … (i)
= 6 [4 − 2α + α 2] 0
46 JEE Advanced~Solved Paper 2019

π/2 1 2
3 sinθ = = =4
⇒ I= ∫ ( sinθ + cosθ)5
dθ …(ii) π
sin cos
π
sin
π
0
12 12 6
a a
[Using the property ∫ f(x) dx = ∫ f(a − x) dx] −1  1
10
So, sec  Σ sec   7 π kπ 
+  7 π + (k + 1) π  
 sec  
0 0  4k = 0  12 2  12 2 
Now, on adding integrals (i) and (ii), we get = sec−1 ()
1 =0
π/2
3
2I = ∫ ( sinθ + cosθ)4
dθ 51. (a) For Z = {x : g(x) = 0}, x > 0
0
π/2 Q g(x) = cos(2π sin x) = 0
3sec2 θ π
= ∫ (1 + tanθ)4
dθ ⇒ 2π sin x = (2n + 1) , n ∈Integer
0 2
Now, let tanθ = t 2 ⇒sec2 θ dθ = 2 t dt 2n + 1
⇒ sin x =
π 4
and at θ = , t → ∞ 3 1 1 3
2 ⇒ sin x = − , − , , [Q sin x ∈ [− 1, 1]]
4 4 4 4
and at θ = 0, t → 0
∞ t + 1 −1 3 1 1 3
∞ 6 t dt here values of sin x , − , − , , are in an A.P. but
So, 2I = ∫ =6∫ dt 4 4 4 4
0 (1 + t) 4 0 (t + 1) 4
corresponding values of x are not in an AP so, (iii) → R.

∞ For W = {x : g ′(x) = 0}, x > 0
 ∞ dt ∞ dt   1 1 
I = 3 ∫ −∫  = 3 − +  so, g ′(x) = − 2π cos x sin(2π sin x) = 0
 (t + 1)
0 (t + 1)
 2(t + 1) 3(t + 1)  0
0 3 4 2 3
 ⇒ either cos x = 0 or sin(2π sin x) = 0
π
⇒ either x = (2n + 1) or 2π sin x = nπ , n ∈Integers.
⇒ I = 3  −  = 3  = ⇒I = 0.5
1 1 1 1 2
 2 3  6 2
Q 2π sin x = nπ
50. (0) Q Σ sec  7 π + kπ  sec  7 π + (k + 1) π 
10 n 1 1
⇒ sin x = = − 1, − , 0, , 1 {Q sin x ∈ [− 1, 1)}
k =0  
12  2 12  2 2 2 2
π π
x = nπ, (2n + 1) or nπ + (− 1)n  ± 
10 1
= Σ ∴
7 π kπ   7 π + (k + 1) π   6
cos 
k =0 2
+  cos  
 12 2  12 2  ⇒ (iv) → P, R, S
 7 π (k + 1) π   7 π k π  
sin 
Hence, option (a) is correct.
+  − + 
10   12 2   12 2  
= Σ 52. (a) For, X = {x : f(x) = 0}, x > 0
k =0
cos 
7 π kπ 
+  7 π + (k + 1) π 
 cos   Now, f(x) = 0
 12 2  12 2 
⇒sin(π cos x) = 0, x > 0
 7 π (k + 1) π  7 π kπ  π π 
Q + − +  = and sin = 1 ⇒ π cos x = nπ, n ∈ Integer.
 12 2  12 2 2 2 
⇒ cos x = n
 7 π (k + 1) π   7 π kπ 
sin  +  cos  +  ⇒ cos x = − 1, 0, 1 {Q cos x ∈ [− 1, 1]}
 12 2   12 2 π
⇒ x = nπ or (2n + 1) , n is an integer. so, (i) → (P), (Q)
− sin 
7 π kπ 
+  7 π + (k + 1) π  2
 cos  
10  12 2  12 2 
= Σ For, Y = {x : f ′(x) = 0}, x > 0
k =0  7 π kπ   7 π (k + 1) π 
cos  +  cos  +  Now, f ′(x) = 0
 12 2  12 2 
⇒ − π sin x cos(π cos x) = 0
 7 π (k + 1) π   7 π + kπ  
tan 
10
= Σ +  − tan   ⇒ either sin x = 0 ⇒ x = nπ, n is an integer,
k =0   12   12 2  
2 or cos(π cos x) = 0
7π π 7π  π
= tan  +  − tan   ⇒ π cos x = (2n + 1) , n is an integer
 12 2  12  2
7 π 2π   7π + π  2n + 1
+ tan  +  − tan   ⇒ cos x =
 12 2  12 2 2
1
. ⇒ cos x = ± , {Q cos x ∈ [− 1, 1]}
. 2
. π 2π
⇒ x = 2nπ ± or 2nπ ± , n is an integer.

+ tan 
7 π 11 π 
+  7 π + 10 π  3 3
 − tan  
 12 2   12 2  So, (ii) → (Q), (T)
7 π 11 π  7π π π
= tan 
Hence, option (a) is correct.
+  − tan = tan + cot
 12 2  12 12 12
JEE Advanced~Solved Paper 2019 47

3  + 4   − 9
53. (b) It is given that, the centres of circles C1 , C2 and C3 are 9 12
 5  5 6
co-linear, Now, C3P = =
0 0 1 5 5
36 864
∴ 3 4 1=0 So, PW 2 = C3W 2 − C3P 2 = 36 − = {Q C3W = r = 6}
25 25
h k 1
12 6
⇒ PW =
⇒ 4h = 3k … (i) 5
and MN is the length of diameter of circle C3, so 24 6
Q ZW = 2PW =
MN = 3 + (3 − 0)2 + (4 − 0) 2 + 4 = 3 + 5 + 4 = 12 5
So, radius of circle C3 , r = 6 length of ZW
… (ii) ∴ = 6
Since, the circle C3 touches C1 at M and C2 at N, so length of XY
|C1 C3| = |r − 3| Now, area of
∆MZN = (MN) (PZ) = × (12)  WZ
1 1 1
⇒ h + k =3
2 2
{QMN = 12}
2 2 2 
⇒ h2 + k 2 = 9 … (iii)
24 6 72 6
From Eqs. (i) and (iii), we get = 3WZ = 3 =
5 5
16h2 1
h2 + = 9 ⇒ 25h2 = 81 and area of ∆ZMW = (ZW)(MP)
9 2
9 12 1  24 6 
⇒ h=+ and k = + =   (MG + GP)
5 5 2 5 
18 12
So, 2h + k = + =6 12 6  9
5 5 = 3 + 
5  5
Now, equation common chord XY of circles C1 and C2 is
C1 − C2 = 0 Q MG = 3 and GP = 9 
 
⇒ 6x + 8y = 18  5
⇒ 3x + 4y = 9 … (iv) 12 6  24  288 6
=   =
Now, 5  5 25
N 72 6
Area of ∆MZN 5 5
∴ = =
(3,4) Area of ∆ZMW 288 6 4
W C2 25
Y Q Common tangent of circles C1 and C3 is C1 − C3 = 0
C3
C2  2 2 
⇒(x 2 + y 2 − 9) −   x −  +  y −  − 36 = 0
P 9 12
3
(0,0) X  5   5 
C1 C1 Z  
18 24
3x+4y=9 ⇒ x+ y + 18 = 0 ⇒ 3x + 4y + 15 = 0 … (v)
M 5 5
9
C1P = Q Tangent (v) is also touches the parabola x 2 = 8αy,
5 2
∴− 2 α  −  = − ⇒α =
3 15 10
 4 4 3
Now, PY 2 = GY 2 − GP 2
81 144 So combination (iv), (S) is only incorrect.
=9− = Hence, option (b) is correct.
25 25
⇒ PY =
12 54. (c) Q length of ZW = 6
5 length of XY
12 24 So, combination (ii), Q is only correct.
Q XY = 2PY = 2 × =
5 5 Hence, option (c) is correct.

You might also like